CMA Part2 MT 1-10 Q&A

You might also like

Download as docx, pdf, or txt
Download as docx, pdf, or txt
You are on page 1of 893

CMA PASS MOCK TEST 1

Question #1: Which one of the options below best describes a public offering where there is less
price uncertainty due to the existence of a benchmark price?
A. Shelf registration.
B. A subsequent or secondary offering.
C. A red herring registration.
D. An initial public offering.
Explanation:
Answer (A) is incorrect.
A shelf registration does not best describe a public offering where there is less price uncertainty
due to the existence of a benchmark. Under a shelf registration, a master registration statement is
filed for securities that the company reasonably expects to sell within 2 years. However, they are
not put on the shelf" until the most opportune time for offering is determined.
Answer (B) is correct.
Later issues of stock by the same company are subsequent offerings. Secondary markets provide
for the trading of previously issued securities. The sale of the stock in the primary market can be
used as a benchmark because the same type of securities were already issued in this market.
Answer (C) is incorrect.
A red herring does not best describe a public offering where there is less price uncertainty due to
the existence of a benchmark. A red herring is a preliminary registration statement that must be
filed with the SEC describing a new issue of stock and the prospects of the issuing company.
Answer (D) is incorrect.
An initial public offering does not best describe a public offering where there is less price
uncertainty due to the existence of a benchmark. An initial public offering is a firm's first
issuance of securities to the public. Thus, there is no benchmark.

Question #2: Fact Pattern: Depoole Company is a manufacturer of industrial products that uses a
calendar year for financial reporting purposes. Assume that total quick assets exceeded total
current liabilities both before and after the transaction described. Further assume that Depoole
has positive profits during the year and a credit balance throughout the year in its retained
earnings account.
Question: Depoole's issuance of serial bonds in exchange for an office building, with the first
installment of the bonds due late this year.
A. Decreases net working capital.
B. Decreases the current ratio.
C. Decreases the quick ratio.
D. Affects all of the answers as indicated.
Explanation:
Answer (A) is incorrect.
The bond issuance would also decrease the current ratio and the quick ratio.
Answer (B) is incorrect.
The bond issuance would also decrease net working capital and the quick ratio.
Answer (C) is incorrect.
The bond issuance would also decrease net working capital and the current ratio.
Answer (D) is correct.
The first installment is a current liability: thus the amount of current liabilities increases with no
corresponding increase in current assets. The effect is to decrease working capital. the current
ratio, and the quick ratio.

Question #3: Fact Pattern: The data presented below show actual figures for selected accounts of
McKeon Company for the fiscal year ended May 31, Year 1, and selected budget figures for the
Year 2 fiscal year. McKeon's controller is in the process of reviewing the Year 2 budget and
calculating some key ratios based on the budget. McKeon Company monitors yield or return
ratios using the average financial position of the company. (Round all calculations to three
decimal places if necessary.)

5/31/Year 2 5/31/Year 1

Current assets $210,000 $180,000

Noncurrent assets 275,000 255,000

Current liabilities 78,000 85,000

Long-term debt 75,000 30,000

Common stock ($30 par value) 300,000 300,000

Retained earnings 32,000 20,000


Year 2
Operations
Sales* $350,000
Cost of goods sold 160,000
Interest expense 3,000
Income taxes (40%rate) 48,000
Dividends declared and paid in Year 2 60,000
Administrative expense 67,000

*All sales are credit sales.

Current Assets

5/31/Year 2 5/31/Year 1
Cash $20,000 $10,000
Accounts receivable 100,000 70,000
Inventory 70,000 80,000
Prepaid expenses 20,000 20,000

A. 0.352.
B. 0.315.
C. 0.264.
D. 0.237.
Explanation:
Answer (A) is incorrect.
The debt ratio equals total debt at year-end divided by total assets at year-end.
Answer (B) is correct.
The debt ratio is equal to the total debt at year-end divided by total assets at year-end. Total debt
at year-end is $153,000 ($78,000 current liabilities + $75,000 long-term debt). Total assets equal
$485,000 ($210,000 current assets + $275,000 noncurrent assets). Thus, the debt ratio is .315
($153,000+ $485,000).
Answer (C) is incorrect.
The debt ratio equals total debt at year-end divided by total assets at year-end.
Answer (D) is incorrect.
The debt ratio equals total debt at year-end divided by total assets at year-end.

Question #4: An entity has developed and patented a new laser disc reading device that will be
marketed internationally. Which of the following factors should the entity consider in pricing the
device?
I. Quality of the new device.
II. Life of the new device.
III. Customers' relative preference for quality compared with price.

A. I and II only.
B. I and III only.
C. II and III only.
D. I, II and III.
Explanation:
Answer (A) is incorrect.
The customers' preference is also important when determining the price of a product.
Answer (B) is incorrect.
The life of the product should also be considered when pricing a product.
Answer (C) is incorrect.
The quality of a product is important when determining how much to charge for it.
Answer (D) is correct.
Product pricing is a function of consumer demand, competitive factors, and the seller's cost
structure and profit objectives. Thus, the seller must consider the trade-off between the price and
quality effects on demand. A better-quality product, for example, one with a relatively long
useful life, is more costly to produce and therefore sells for a higher price, which in turn reduces
the amount demanded.

Question #5: A transit company is considering two alternative buses to transport people between
cities that are in the same region. A gas-powered bus has a cost of $55,000, and will produce
end-of-year net cash flows of $22,000 per year for 4 years. A new electric bus will cost $90,000,
and will produce cash flows of $28,000 per year for 8 years. The company must provide bus
service for 8 years, after which it plans to give up its franchise and to cease operating the route.
Inflation is not expected to affect either costs or revenues during the next 8 years. If the cost of
capital is 16%, by what amount will the better project increase the company's value?
A. $6,556
B. $(14,432)
C. $13,112
D. $31,632
Explanation:
Answer (A) is incorrect.
It is based on only the first four years for the gas-powered bus.
Answer (B) is incorrect.
It did not discount the purchase price of the second gas-powered bus.
Answer (C) is incorrect.
It is simply a doubling of the NPV for the first four years of the gas-powered bus.
Answer (D) is correct.
The NPV of the electric bus is $31,632, which is greater than that of two gas-powered buses
bought 4 years apart. The NPV for the $90,000 electric bus involves multiplying the $28,000
annual cash flows times the present value factor of 4.344, which equals $121,632. Deducting the
$90,000 initial cost results in an NPV of $31,632. The NPV for the two gas-powered buses is
$10,208, calculated as follows:
$22,000 x 4,344 $95,568
Less: first bus (55,000)
Less: second bus (55,000 x .552) (30,360)
NPV $10,208

Question #6: After a petition for bankruptcy liquidation has been filed and the court has issued
an order for relief.
A. The court usually appoints a permanent trustee to take control of the debtor's estate.
B. Creditors must immediately cease their collection activities.
C. The bankruptcy judge notifies creditors, collects the debtor's nonexempt property, and
distributes that property to the creditors.
D. A meeting is held by the creditors to vote on a plan of reorganization.
Explanation:
Answer (A) is incorrect.
The court usually appoints a temporary trustee, and the creditors usually elect a permanent
trustee.
Answer (B) is correct.
A debtor files a bankruptcy petition to obtain relief from creditors' collection efforts. Creditors
may file to ensure an equitable division of the debtor's estate, an outcome that may not be
reached if creditors are allowed to continue their individual collection activities. Thus, the order
for relief stays those activities.
Answer (C) is incorrect.
The trustee, not the judge, administers the estate.
Answer (D) is incorrect.
The creditors vote on a plan of reorganization under Chapter 11.

Question #7: Under the Foreign Corrupt Practices Act (FCPA), an action may be brought that
seeks
A. Treble damages by a private party.
B. Injunctive relief by a private party.
C. Criminal sanctions against both the corporation and its officers by the Department of
Justice.
D. Damages and injunctive relief by the Securities and Exchange Commission.
Explanation:
Answer (A) is incorrect.
Private parties may not bring an action under the FCPA.
Answer (B) is incorrect.
Private parties may not bring an action under the FCPA.
Answer (C) is correct.
The SEC may investigate violations of the FCPA, bring civil actions for its enforcement, and
recommend that the Justice Department prosecute criminal violations.
Answer (D) is incorrect.
Although the SEC is empowered to seek injunctions, the Justice Department must seek penalties.
Damages are sought by private parties who cannot sue under this statute.
Question #8: A company is reviewing its trade credit policy with respect to the small retailers to
which it sells. Four plans have been studied and the results are as follows:
Annual Bad Collection Accounts Inventor
Plan revenue debt costs receivable y
A $200,000 $1,000 $1,000 $20,000 $40,000
B 250,000 3,000 2,000 40,000 50,000
C 300,000 6,000 5,000 60,000 60,000
D 350,000 12,000 8,000 80,000 70,000

A. Plan A.
B. Plan B.
C. Plan C.
D. Plan D.
Explanation:
Answer (A) is incorrect.
Plan A has a lower incremental profit than Plan B.
Answer (B) is correct.
The following schedule presents the after-tax profit for each plan:

Plan A Plan B Plan C Plan D


Annual revenue $200,000 $250,000 $300,000 $350,000
Times: GP % × 20% × 20% × 20% × 20%
Gross profit $40,000 $50,000 $60,000 $70,000
Less: bad debt -1,000 -3,000 -6,000 -12,000
Less: collection cost -1,000 -2,000 -5,000 -8,000
Gross incremental profit $38,000 $45,000 $49,000 $50,000
Less: income taxes (30%) -11,400 -13,500 -14,700 -15,000
After tax profit $26,000 $31,000 $34,300 $35,000

The following schedule presents the investment in current assets for each plan:
Plan A Plan B Plan C Plan D
Accounts receivable $20,000 $40,000 $60,000 $80,000
Inventory 40,000 50,000 60,000 70,000
capital invested $60,000 $90,000 $90,000 $120,000
Times: cost of capital × 15% × 15% × 15% × 15%
Dollar cost of capital 9,000 13,500 18,000 22,500
The difference between after-tax profit and cost of capital is the net incremental profit Plan B’s is
the highest:

Plan A Plan B Plan C Plan D


After tax profit $26,600 $31,500 $34,300 $35,000
Less: dollar cost of capital -9,000 -13,500 -18,000 -22,000
net incremental profit $17,600 $18,000 $16,300 $12,500

Answer (C) is incorrect.


Plan C has a lower incremental profit than Plan B.
Answer (D) is incorrect.
Plan D has a lower incremental profit than Plan B.

Question #9: What law prohibits U.S. companies from paying bribes to foreign officials for the
purpose of obtaining or retaining business?
A. Federal Ethical Standards Act.
B. Robinson-Patman Act.
C. Foreign Corrupt Practices Act.
D. North American Free Trade Agreement.
Explanation:
Answer (A) is incorrect.
The Federal Ethical Standards Act does not deal with international payments.
Answer (B) is incorrect.
The Robinson-Patman Act of 1936 prohibits price discrimination.
Answer (C) is correct.
The Foreign Corrupt Practices Act of 1977 prohibits bribes to foreign officials for purposes of
obtaining or retaining business. The act also requires companies to maintain effective systems of
internal control.
Answer (D) is incorrect.
The North American Free Trade Agreement (NAFTA), passed in 1993, provides for free among
the nations of Canada, Mexico and the U.S.
Question #10: A firm is using cost-based pricing to determine the selling price for its new
product based on the following information.
Annual volume 25,000 units
Fixed costs $700,000 per year
Variable costs $200 per unit
Plant investment $3,000,000
Working capital $1,000,000
Effective tax rate 40%
The target price that the firm needs to set for the new product to achieve a 15%after-tax return on
investment (ROI) would be
A. $228
B. $238
C. $258
D. $268
Explanation:
Answer (A) is incorrect.
The amount of $228 results from simply adding the per-unit variable and fixed costs.
Answer (B) is incorrect.
The amount of $238 results from failing to include plant investment in invested capital.
Answer (C) is incorrect.
The amount of $258 results from failing to include working capital in invested capital.
Answer (D) is correct.
A 15% after-tax return on investment requires net income of $600,000 ($4,000,000 invested
capital x .15). Per-unit net income must therefore be S24 ($600,000 + 25,000 units). Per-unit
operating income must be $40 [S24 net income ÷ (1.0 -.40 tax rate)]. Per-unit fixed cost in the
firm's relevant range is $28 ($700,000 ÷ 25,000 units), so per-unit contribution margin must be
S68 ($40+ $28). Per-unit variable costs are given as $200, so the selling price necessary to
generate the desired return is $268 ($200+$68).

Question #11: The method that divides a project's annual after-tax net income by the average
investment cost to measure the estimated performance of a capital investment is the
A. Internal rate of return method.
B. Accounting rate of return method.
C. Payback method.
D. Net present value (NPV) method.
Explanation: Answer (A) is incorrect.
The internal rate of return is the rate at which NPV is zero. The minimum desired rate of return is
not used in the discounting.
Answer (B) is correct.
The accounting rate of return uses undiscounted net income (not cash flows) to determine a rate
of profitability. Annual after-tax net income is divided by the average carrying amount (or the
initial value) of the investment in assets.
Answer (C) is incorrect.
The payback period is the time required to complete the return of the original investment. This
method gives no consideration to the time value of money or to returns after the payback period.
Answer (D) is incorrect.
The NPV method computes the discounted present value of future cash inflows to determine
whether it is greater than the initial cash outflow.

Question #12: The statement below that best represents the definition of capital rationing is a
A. Situation where a constraint is placed on the total size of capital expenditures during a
particular period.
B. Budget constraint limiting total depreciation expense related to new capital expenditures
for a given period.
C. Limitation where a company selects projects that will meet the minimum number of cash
inflow requirements in a given year.
D. Policy that helps optimize investment policy by ensuring that the best mix of products is
selected.
Explanation:
Answer (A) is correct.
Capital rationing exists when a firm sets a limit on the amount of funds to be invested during a
given period. Placing a constraint on the total size of capital expenditures is a way of setting the
limit.
Answer (B) is incorrect.
Capital rationing puts a constraint on the amount of funds used for new capital expenditures, not
its depreciation expense.
Answer (C) is incorrect.
Capital rationing puts a limit on the amount of funds used for new capital projects, not how many
cash inflows the project will have.
Answer (D) is incorrect.
Capital rationing only sets a limit on the amount of funds invested: it does not choose the best
projects that meet that limit.

Question #13: If inventories are expected to change, the type of costing that provides the best
information for breakeven analysis is
A. Job order costing.
B. Variable (direct) costing.
C. Joint costing.
D. Absorption (full) costing.
Explanation:
Answer (A) is incorrect.
Job order costing does not separate fixed costs from variable costs.
Answer (B) is correct.
A variable (direct) costing system is needed to perform CVP analysis because variable costing
separates fixed costs from variable costs.
Answer (C) is incorrect.
Joint costing does not separate fixed costs from variable costs.
Answer (D) is incorrect.
Absorption (full) costing does not separate fixed costs from variable costs.

Question #14: Fact Pattern: Donnelly Corporation manufactures and sells T-shirts imprinted with
college names and slogans. Last year, the shirts sold for $7.50 each, and the variable cost to
manufacture them was $2.25 per unit. The company needed to sell 20,000 shirts to break even.
The net income last year was $5,040. Donnelly's expectations for the coming year include the
following:
The sales price of the T-shirts will be $9
Variable cost to manufacture will increase by one-third
Fixed costs will increase by 10%
The income tax rate of 40% will be unchanged
Question: The number of T-shirts Donnelly Corporation must sell to break even in the coming
year is
A. 17.500
B. 19.250
C. 20.000
D. 22.000
Explanation: Answer (A) is incorrect.
This amount is based on the preceding year's fixed costs.
Answer (B) is correct.
The breakeven point in units equals total fixed costs divided by the unit contribution margin.
Fixed cost for the previous year was
$105,000 (20,000 units at breakeven x $5.25 UCM). Fixed cost for the current year is $115,500
($105,000 x 1.1). The new UCM is $6, ($9 selling price - $3 unit variable cost). Accordingly, the
BEP is 19,250 units ($115,500÷ $6).
Answer (C) is incorrect.
The preceding year's BEP is 20,000.
Answer (D) is incorrect.
This amount is the breakeven point if the current year's fixed costs of $115,500 ($105,000 x 1.1)
is divided by last year's contribution margin of $5.25.

Question #15: A company has received an offer from a supplier to produce units that the
company currently produces and sells. The unit price quoted by the supplier is higher than the
company's variable production cost per unit but lower than the price at which the company can
market the units. Under which circumstance would the company's profits increase by purchasing
units from the supplier?
A. Market demand for the product exceeds the company's capacity.
B. The company's fixed overhead would remain the same if the company purchased units
from the supplier.
C. The company has significant sunk costs.
D. The company's administrative costs are zero.
Explanation:
Answer (A) is correct.
If not enough capacity is available to produce all products demanded, products should be
outsourced or capacity should be expanded to increase profits.
Answer (B) is incorrect.
Costs that do not differ between two alternatives should be ignored because they are irrelevant to
the decision being made.
Answer (C) is incorrect.
Sunk costs are irrelevant in decision making.
Answer (D) is incorrect.
This answer is a distractor and does not have a bearing on the decision. Also, costs that do not
differ between two alternatives should be ignored because they are irrelevant to the decision
being made.

Question #16: The fraud risk factor that may be mitigated by internal controls is
A. Rationalization.
B. Motive.
C. Pressure.
D. Opportunity.
Explanation:
Answer (A) is incorrect.
Rationalization cannot be affected by controls because it is a mental state of a potential
perpetrator of a fraud.
Answer (B) is incorrect.
Like rationalization, motive is a mental state that cannot be mitigated by controls.
Answer (C) is incorrect.
Pressure relates to motive and cannot be mitigated by controls.
Answer (D) is correct.
The opportunity for individuals to perpetrate a fraud can be mitigated by proper controls.
Examples are appropriate oversight, segregation of duties, and the audit process itself.

Question #17: Assume that an investment project's assumed cash flows are not changed. but the
assumed weighted-average cost of capital is reduced. What impact would this have on the net
present value (NPV) and the internal rate of return (IRR) of this project?
A. NPV would increase, and IRR would increase.
B. NPV would decrease, and IRR would increase.
C. NPV would not change, and IRR would not change.
D. NPV would increase, and IRR would not change.
Explanation:
Answer (A) is incorrect.
The IRR will not increase because the project's assumed cash flows are not changing. The IRR is
the rate that makes the present value of the expected cash inflows equal the present value of the
expected cash outflows. Thus, the IRR will remain the same.
Answer (B) is incorrect.
If the cost of capital is reduced, the PV factors are increased. This would result in a higher
discounted cash inflow each year, resulting in a higher NPV, not a decreased NPV. In addition,
the IRR will not increase because the project's assumed cash flows are not changing. The IRR is
the rate that makes the present value of the expected cash inflows equal the present value of the
expected cash outflows. Thus, the IRR will remain the same.
Answer (C) is incorrect.
If the cost of capital is reduced, the PV factors are increased. This would result in a higher
discounted cash inflow each year, resulting in a higher NPV, not an unchanged one.
Answer (D) is correct.
If the cost of capital is reduced, the PV factors are increased. This would result in a higher
discounted cash inflow each year, resulting in a higher NPV. However, because the project’s
assumed cash flows are not changed, the IRR will not change. The IRR is the rate that makes the
present value of the expected cash inflows equal the present value of the expected cash outflows.

Question #18: A firm earned $10.000 before interest and taxes, has a 36% tax rate, and has the
following debt outstanding:

First mortgage bond, 9.0% $5,000


Debenture, 10.2% 10,000
Subordinated bond, 12.0% 6,000
Total long-term debt $21,000

The annual coverage of the firm’s debt is


A. 4.57 times.
B. 2.92 times.
C. 11.85 times.
D. 3.57 times.
Explanation:
Answer (A) is correct.
The times interest earned (interest coverage) ratio is computed by dividing the income available
for paying interest (pretax. pre-interest income) by the annual interest expense. The first step is
to determine the annual interest expense:

First mortgage bond 9.0% × $5,000


Debenture 10.2%× 10,000
Subordinated bond 12.0%× 6,000
Total interest expense $ 2,190

Answer (B) is incorrect.


After-tax, pre-interest income of 56.400 divided by the interest expense equals 2.92 times.
Answer (C) is incorrect.
Pre-tax, pre-interest income should be divided by interest expense to find the times interest
earned ratio.
Answer (D) is incorrect.
Pre-tax, after-interest income (57.810) divided by $2.190 equals 3.57 times.

Question #19: Question: With all other things constant, if Geary Manufacturing is able to reduce
the direct materials for an electric mixer to $6 per unit, the company should
[Fact Pattern I
Geary Manufacturing has assembled the data appearing in the next column pertaining to two
products. Past experience has shown that the unavoidable fixed manufacturing factory overhead
included in the cost per machine hour averages $10. Geary has a policy of filling all sales orders,
even if it means purchasing units from outside suppliers. Total machine capacity is 50,000 hours.
Electric
Blender mixer
Direct materials $6 $ 11
Direct labor 4 9
Manufacturing overhead at $16 per hour 16 32
Cost if purchased from an outside supplier 20 38
Annual demand (units) 20,000 28,000

A. Produce 25.000 electric mixers and purchase all other units as needed.
B. Produce 20.000 blenders and 15.000 electric mixers, and purchase all other units as
needed.
C. Produce 20.000 blenders and purchase all other units as needed.
D. Purchase all units as needed.
Explanation:
Answer (A) is correct.
Reducing unit direct materials cost for mixers from $11 to $6 decreases unit variable cost to $27
($6 DM + $9 DL + $12 VOH) and increases the cost savings of making a mixer from $6 to $11,
or $5.50 per hour ($11 + 2 hours per unit). Given a cost savings per hour for blenders of $4, the
company can minimize total variable cost by making 25,000 mixers (50,000 hours capacity ÷ 2).
Total variable cost will be $1,189,000 [(25,000 mixers x $27) + (3,000 mixers x $38) + (20,000
blenders x $20)].
Answer (B) is incorrect.
Producing 20,000 blenders and 15,000 mixers results in a total variable cost of $1,219,000.
Answer (C) is incorrect.
Producing 20,000 blenders results in a total variable cost of $1,384,000.
Answer (D) is incorrect.
The variable cost of making these items is less than the cost of purchase.

Question #20: A company invests in a project with expected cash inflows of $9.000 per year for
4 years. All cash flows occur at year end. The required return on investment is 9%. If the project
generates a net present value (NPV) of $3,000, what is the amount of the initial investment in the
project?
A. $11,253
B. $13,236
C. $26,160
D. $29,160
Explanation:
Answer (A) is incorrect.
The amount of $11,253 ignores the fact that 59,000 will be the inflow in each of the 4 years.
Answer (B) is incorrect.
The amount of $13,236 ignores the fact that 59,000 will be the inflow in each of the 4 years.
Answer (C) is correct.
The initial investment in this project can be calculated as follows:
Annual cash inflows $ 9,000
Times: PV factor x 3,240
PV of cash inflows $29,160
Less: NPV of project (3000)
Amount invested $26,160
Answer (D) is incorrect.
The amount of $29,160 is the present value of the cash inflows.

Question #21: For a given level of sales and holding all other financial statement items constant,
a company's return on equity (ROE) will
A. Increase as their debt ratio decreases.
B. Decrease as their cost of goods sold as a percent of sales decrease.
C. Decrease as their total assets increase.
D. Increase as their equity increases.
Explanation:
Answer (A) is incorrect.
The debt ratio is total liabilities to total assets. If this ratio decreases, then either liabilities went
down or assets went up or both: since all other financial statement line items are held constant, it
will have no effect on the return on equity.
Answer (B) is incorrect.
If cost of goods sold as a percent of sales decreases, return on equity will not be affected.
Answer (C) is correct.
A firm's return on equity is a measure of how much equity capital is employed to generate its
level of earnings. In this case, an increase in total assets means an increase in equity (since all
other financial statement items are being held constant). Equity is the denominator, and an
increase in the denominator means a decrease in the overall ratio.
Answer (D) is incorrect.
If equity increases, return on equity will decrease.

Question #22: Which one of the following actions may increase a company's return on assets?
A. Purchase of a new corporate headquarters.
B. An increase in inventory levels for a future store expansion.
C. Replacement of capital equipment via an operating lease.
D. Reduction of long-term debt through the issuance of common stock.
Explanation:
Answer (A) is incorrect.
ROA is calculated by dividing a firm's net income by its average total assets. Since this action
causes total assets to increase. ROA would decrease overall.
Answer (B) is incorrect.
ROA is calculated by dividing a firm's net income by its average total assets. Since this action
causes total assets to increase. ROA would decrease overall.
Answer (C) is correct. Return on assets, or ROA (also called return on total assets. or ROTA), is
a straightforward measure of how well management is deploying the firm's assets in the pursuit
of a profit. ROA is calculated by dividing a firm's net income by its average total assets.
Answer (D) is incorrect.
This action would not affect ROA.

Question #23: IMAs Statement of Ethical Professional Practice includes an integrity standard,
which requires an IMA member to
A. Decline to become a minority partner in a partnership that is a supplier of the member's
employer.
B. Report all information that could influence users of financial statements.
C. Disclose confidential information when authorized by his or her firm or required under
the law.
D. Refuse gifts from anyone.
Explanation:
Answer (A) is correct.
One of the responsibilities of an IMA member under the integrity standard is to "refrain from
engaging in any conduct that would prejudice carrying out duties ethically."
Answer (B) is incorrect.
The credibility standard requires an IMA member to 'disclose all relevant information that could
reasonably be expected to influence an intended user's understanding of the reports, analyses,
and recommendations:'
Answer (C) is incorrect.
The confidentiality standard requires an IMA member to "keep information confidential except
when disclosure is authorized or legally required."
Answer (D) is incorrect.
The integrity standard requires an IMA member to "mitigate actual conflicts of interest.
Regularly communicate with business associates to avoid apparent conflicts of interest. Advise
all parties of any potential conflicts.”

Question #24: Which one of the following is not considered a quantitative risk assessment
technique?
A. Self-assessment questionnaires.
B. Benchmarking.
C. Cash-flow at risk.
D. Scenario analyses.
Explanation:
Answer (A) is correct.
Self-assessment questionnaires are not considered a quantitative assessment technique. They are
not statistical techniques.
Answer (B) is incorrect.
Benchmarking is considered a quantitative risk assessment technique.
Answer (C) is incorrect.
Cash-flow at risk is considered a quantitative risk assessment technique.
Answer (D) is incorrect.
Scenario analyses are considered a quantitative risk assessment technique.

Question #25: When managing cash and short-term investments, a corporate treasurer is
primarily concerned with
A. Maximizing rate of return.
B. Minimizing taxes.
C. Investing in Treasury bonds since they have no default risk.
D. Liquidity and safety.
Explanation:
Answer (A) is incorrect.
Most companies are not in business to earn high returns on liquid assets (i.e.. they are held to
facilitate operations).
Answer (B) is incorrect.
The holding of cash and cash-like assets is not a major factor in controlling taxes.
Answer (C) is incorrect.
Investments in Treasury bonds do not have sufficient liquidity to serve as short-term assets.
Answer (D) is correct.
Cash and short-term investments are crucial to a firm's continuing success. Sufficient liquidity
must be available to meet payments as they come due. At the same time, liquid assets are subject
to significant control risk. Therefore, liquidity and safety are the primary concerns of the
treasurer when dealing with highly liquid assets. Cash and short-term investments are held
because of their ability to facilitate routine operations of the company. These assets are not held
for purposes of achieving investment returns.

Question #26: All of the following are assumptions of cost-volume-profit analysis except
A. Total fixed costs do not change with a change in volume.
B. Revenues change proportionately with volume.
C. Variable costs per unit change proportionately with volume.
D. Sales mix for multi-product situations do not vary with volume changes
Explanation:
Answer (A) is incorrect.
One of the assumptions of cost-volume-profit analysis is that total fixed costs do not change with
a change in volume.
Answer (B) is incorrect.
One of the assumptions of cost-volume-profit analysis is that revenues change proportionately
with volume.
Answer (C) is correct.
CVP analysis assumes the variable costs per unit are constant over the relevant range.
Answer (D) is incorrect.
One of the assumptions of cost-volume-profit analysis is that the sales mix for multi-product
situations do not vary with volume changes.
Question #27: The days' sales in receivables ratio will be understated if the company
A. Uses a natural business year for its accounting period.
B. Uses a calendar year for its accounting period.
C. Uses average receivables in the ratio calculation.
D. Does not use average receivables in the ratio calculation.
Explanation:
Answer (A) is correct.
The days' sales in receivables ratio equals the days in the year divided by the receivables
turnover ratio (sales ÷ average receivables). Days' sales may also be computed based only on
ending receivables. In either case, use of the natural business year tends to understate the ratio
because receivables will usually be at a low point at the beginning and end of the natural year.
For example, a ski resort may close its books on May 31, a low point in its operating cycle.
Answer (B) is incorrect.
Using a calendar year will not necessarily affect the usefulness of the days' sales ratio.
Answer (C) is incorrect.
Using average receivables would not always understate the ratio. The ratio could be higher or
lower depending on changes in sales volume or the percentage of credit to cash sales, or other
factors.
Answer (D) is incorrect.
The ratio could be higher or lower depending on changes in sales volume or the percentage of
credit to cash sales, or other factors.

Question #28: All else being equal, a company with a higher dividend-payout ratio will have a
debt-to-assets ratio and a current ratio.
A. Higher Higher
B. Higher Lower
C. Lower Higher
D. Lower Lower
Explanation:
Answer (A) is incorrect.
The current ratio will be lower.
Answer (B) is correct.
A company with a higher dividend payout ratio is distributing more of its earnings as dividends
to common shareholders. It will have less cash and less total assets than a comparable firm with a
lower payout ratio. The debt-to-assets ratio will be higher because total assets are lower, and the
current ratio will be lower because cash is lower.
Answer (C) is incorrect.
The debt-to-assets ratio will be higher and the current ratio will be lower.
Answer (D) is incorrect.
The debt-to-assets ratio will be higher.

Question #29: Which one of the following pricing methods takes into consideration a product's
entire life cycle?
A. Target pricing.
B. Transfer pricing.
C. Market-based pricing.
D. Cost-based pricing.
Explanation:
Answer (A) is correct.
A target price is the expected market price for a product or service, given the company's
knowledge of its consumers' perceptions of value and competitors' responses. Subtracting the
unit target operating income determines the long-term unit target cost. Relevant costs are all
future value-chain costs, whether variable or fixed.
Answer (B) is incorrect.
Transfer pricing does not take into consideration a product's entire life cycle.
Answer (C) is incorrect.
Market-based pricing does not take into consideration a product's entire life cycle.
Answer (D) is incorrect.
Cost-based pricing does not take into consideration a product's entire life cycle.

Question #30: An entity has total assets of $7.500.000 and a current ratio of 2.3 times before
purchasing $750.000 of merchandise on credit for resale. After this purchase, the current ratio
will
A. Remain at 2.3 times.
B. Be higher than 2.3 times.
C. Be lower than 2.3 times.
D. Be exactly 2.53 times.
Explanation:
Answer (A) is incorrect.
The current ratio is the ratio of current assets to current liabilities. When the ratio is greater than
one, any change of equal dollar amount on both the numerator and denominator will result in a
lowering of the overall ratio (since the denominator will increase by a proportionally greater
amount). The purchase of merchandise on credit is an example of such a change: inventory
increases in the numerator and accounts payable increase in the denominator by an equal dollar
amount.
Answer (B) is incorrect.
The current ratio is the ratio of current assets to current liabilities. When the ratio is greater than
one, any change of equal dollar amount on both the numerator and denominator will result in a
lowering of the overall ratio (since the denominator will increase by a proportionally greater
amount). The purchase of merchandise on credit is an example of such a change: inventory
increases in the numerator and accounts payable increase in the denominator by an equal dollar
amount.
Answer (C) is correct.
The current ratio is the ratio of current assets to current liabilities. When the ratio is greater than
one, any change of equal dollar amount on both the numerator and denominator will result in a
lowering of the overall ratio (since the denominator will increase by a proportionally greater
amount). The purchase of merchandise on credit is an example of such a change: Inventory
increases in the numerator and accounts payable increases in the denominator by an equal dollar
amount.
Answer (D) is incorrect.
The current ratio is the ratio of current assets to current liabilities. When the ratio is greater than
one, any change of equal dollar amount on both the numerator and denominator will result in a
lowering of the overall ratio (since the denominator will increase by a proportionally greater
amount). The purchase of merchandise on credit is an example of such a change: inventory
increases in the numerator and accounts payable increase in the denominator by an equal dollar
amount.

Question #31: The loss of a key customer has temporarily caused Bedford Machining to have
some excess manufacturing capacity. Bedford is considering the acceptance of a special order,
one that involves Bedford's most popular product. Consider the following types of costs.
I. Variable costs of the product
II. Fixed costs of the product
III. Direct fixed costs associated with the order
IV. Opportunity cost of the temporarily idle capacity
Which one of the following combinations of cost types should be considered in the special
order acceptance decision?
A. I and II.
B. I and IV.
C. II and III.
D. I, III, and IV.
Explanation:
Answer (A) is incorrect.
Fixed costs of the product are not relevant.
Answer (B) is incorrect.
Direct fixed costs of the order are relevant also.
Answer (C) is incorrect.
Fixed costs of the product are not relevant.
Answer (D) is correct.
Fixed costs of the product do not vary whether the special order is accepted or not, making them
irrelevant. All the other costs will change if the order is accepted and are therefore relevant. The
opportunity cost of the temporarily idle facility, if there are any, should also be considered.

Question #32: A retailer buys virtually all of its merchandise from manufacturers in a country
experiencing significant inflation. The retailer is considering changing its method of inventory
costing from first-in, first-out (FIFO) to last-in, first-out (LIFO). What effect would the change
from FIFO to LIFO have on the retailer's current ratio and inventory turnover ratio?
A. Both the current ratio and the inventory turnover ratio would increase.
B. The current ratio would increase but the inventory turnover ratio would decrease.
C. The current ratio would decrease but the inventory turnover ratio would increase.
D. Both the current ratio and the inventory turnover ratio would decrease.
Explanation:
Answer (A) is incorrect.
The current ratio would decrease due to the lower inventory value under LIFO.
Answer (B) is incorrect.
The current ratio would decrease due to the lower inventory value under LIFO.
Answer (C) is correct.
During periods of high inflation, manufacturers and retailers often switch to LIFO inventory
valuation as a tax postponement tool. The higher costs attaching to more recent inventory pass
into cost of goods sold, reducing net income and tax liability. Since cost of goods sold is the
numerator of the inventory turnover ratio, turnover will increase. Also, inventory will be lower
under LIFO, which reduces the current ratio and increases the turnover ratio.
Answer (D) is incorrect.
The inventory turnover would increase due to higher cost of goods sold and lower inventory.

Question #33: The accounting rate of return


A. Is synonymous with the internal rate of return.
B. Focuses on income as opposed to cash flows.
C. Is inconsistent with the divisional performance measure known as return on investment.
D. Recognizes the time value of money.
Explanation:
Answer (A) is incorrect.
The I RR is the rate at which the net present value is zero. Thus, it incorporates time value of
money concepts, whereas the accounting rate of return does not.
Answer (B) is correct.
The accounting rate of return (also called the unadjusted rate of return or book value rate of
return) is calculated by dividing the increase in accounting net income by the required
investment. Sometimes the denominator is the average investment rather than the initial
investment. This method ignores the time value of money and focuses on income as opposed to
cash flows.
Answer (C) is incorrect.
The accounting rate of return is similar to the divisional performance measure of return on
investment.
Answer (D) is incorrect.
The accounting rate of return ignores the time value of money.

Question #34: A forward contract involves a commitment today to purchase a product


A. On a specific future date at a price to be determined sometime in the future.
B. At some time during the current day at its present price.
C. On a specific future date at a price determined today.
D. Only when its price increases above its current exercise price.
Explanation:
Answer (A) is incorrect.
The price of a future contract is determined on the day of commitment, not sometime in the
future.
Answer (B) is incorrect.
Performance is deferred in a future contract, and the price of the product is not necessarily its
present price. The price can be any price determined on the day of commitment.
Answer (C) is correct.
A forward contract is an executory contract in which the parties involved agree to the terms of a
purchase and a sale, but performance is deferred. Accordingly, a forward contract involves a
commitment today to purchase a product on a specific future date at a price determined today.
Answer (D) is incorrect.
A forward contract is a firm commitment to purchase a product. It is not based on a contingency.
Also, a forward contract does not involve an exercise price (exercise price is in an option
contract).

Question #35: Fact Pattern: Calamity Cauliflower Corporation is considering undertaking a


capital project. The company would have to commit $24,000 of working capital in addition to an
immediate outlay of $160,000 for new equipment. The project is expected to generate $100,000
of annual income for 10 years. At the end of that time, the new equipment, which will be
depredated on a straight-line basis, is expected to have a salvage value of $10.000. The existing
equipment that would be sold to make room for the project has a historical cost of $220.000 and
accumulated depreciation of $208.000.1t has an estimated remaining useful life of 2 years and
the remaining carrying amount is being depreciated on a straight-line basis. A scrap dealer has
agreed to buy it for $8.000.
The company's effective tax rate is 40%.
Question: Calamity Cauliflower's relevant after-tax annual cash inflow from the ongoing
operations of the project i.
A. $100,000
B. $60,000
C. $40.000
D. $0
Explanation:
Answer (A) is incorrect.
The amount of $100,000 fails to take income tax expense into account.
Answer (B) is correct.
The relevant after-tax annual cash inflow for the project consists of the gross cash inflow
($100,000) minus income tax expense ($100,000 x .40), or $60,000.
Answer (C) is incorrect.
The amount of $40,000 is the expected annual income tax expense.
Answer (D) is incorrect.
The company does have a relevant after-tax cash inflow.

Question #36: An adequate system of internal controls is most likely to detect a fraud perpetrated
by a
A. Group of employees in collusion.
B. Single employee.
C. Group of managers in collusion.
D. Single manager.
Explanation:
Answer (A) is incorrect.
A group has a better chance of successfully perpetrating a fraud than does an individual
employee.
Answer (B) is correct.
Segregation of duties and other control procedures serve to prevent or detect a fraud committed
by an employee acting alone. One employee may not have the ability to engage in wrongdoing or
may be subject to detection by other employees in the course of performing their assigned duties.
However, collusion may circumvent controls. For example, comparison of recorded
accountability with assets may fail to detect fraud if persons having custody of assets collude
with record keepers.
Answer (C) is incorrect.
Management can override controls.
Answer (D) is incorrect.
Even a single manager may be able to override controls.
Question #37: Question: Assume net credit sales and cost of goods sold for Year 2 were
$300.000 and $220,000, respectively. Lisa, Ines accounts receivable turnover for Year 2 was
Lisa, Inc.
Statement of Financial Position
December 31, Year 2
(000s)

Assets Year 2 Year 1


Current assets:
Cash $ 30 $ 25
Trading securities 20 15
Accounts receivable (net) 45 30
Inventories (at lower of cost or market) 60 50
Prepaid items 15 20
Total current assets 170 140
Long-term Investments
Securities (at cost) 25 20
Property, plant & equipment
Land (at cost) 75 75
Building (net) 80 90
Equipment (net) 95 100
Intangible assets
Patents (net) 35 17
Goodwill (net) 20 13
Total long-term assets 330 315
Total assets $ 500 $ 455
Liabilities & Shareholders' equity
Current Liabilities:
Notes payable $ 23 $ 12
Accounts payable 47 28
Accrued interest 15 15
Total current Liabilities 85 55
Long-term debt:
Notes payable 10% due 12/31/Year 9 10 10
Bonds payable 12% due 12/31/Year 8 15 15
Total long-term debt 25 25
Total liabilities $110 $80
Shareholders' equity:
Preferred - 5% cumulative, $100 par, non-participating, 1,000 shares
authorized, 15,000 issued and outstanding $100 $100
Common- $10 par 20,000 shares authorized, 15,000 issued and
outstanding shares 150 150
Additional paid-in capital- common 75 75
Retained earnings 65 50
Total shareholders' equity $390 $375
Total liabilities & equity $500 $455

A. 4.9 times.
B. 5.9 times.
C. 6.7 times.
D. 8.0 times.
Explanation:
Answer (A) is incorrect.
The number 49 can be obtained only by using year-end receivables in the denominator and cost
of goods sold in the numerator.
Answer (B) is incorrect.
Cost of goods sold divided by average receivables equals 5.9.
Answer (C) is incorrect
The number 6.7 is based on the ending receivables.
Answer (D) is correct.
The accounts receivable turnover is computed by dividing the net credit sales by average
accounts receivable. The average is $37,500 [($45.000 + $30,000) ÷ 2]. Hence, the turnover is
8.0 ($300,000 ÷$37,500).

Question #38: Fact Pattern: Excerpts from the statement of financial position for Markham
Corporation as of April 30 of the current year are presented as follows:
Cash $ 725,000
Accounts receivable(net) 1,640,000
Inventories 2,945,000
Total current assets $5,310,000
Accounts payable $1,236,000
Accrued liabilities 831,000
Total current liabilities $2,067,000

The board of directors of Markhammet on May 5 of the current year and declared a quarterly
cash dividend in the amount of $800,000 ($.50 per share). The dividend was paid on May 28 of
the current year to shareholders of record as of May 15 of the current year. Assume that the only
transactions that affected Markham during May of the current year were the dividend
transactions and that the closing entries have been made.
Question: Markham's working capital would be
A. Decreased by the dividend declaration and increased by the dividend payment.
B. Unchanged by either the dividend declaration or the dividend payment.
C. Decreased by the dividend declaration and unchanged by the dividend payment.
D. Increased by the dividend declaration and unchanged by the dividend payment.
Explanation:
Answer (A) is incorrect.
The dividend payment will not change working capital, but the declaration of the dividend
decreases working capital.
Answer (B) is incorrect.
The dividend payment will not change working capital, but the declaration of the dividend
decreases working capital.
Answer (C) is correct.
Working capital is the excess of current assets over current liabilities. The declaration of a
dividend reduces retained earnings and creates a new current liability. Thus, the declaration of a
dividend reduces working capital because current liabilities are increased without a
corresponding increase in current assets. The subsequent payment of the dividend has no effect
on working capital because current assets (cash) will be reduced by the same amount that current
liabilities (dividends payable) are reduced.
Answer (D) is incorrect
The dividend payment will not change working capital, but the declaration of the dividend
decreases working capital.
Question #39: Question: The net benefit (loss) per machine hour that would result if Stewart
accepts the supplier's offer of $13.50 per unit for Component 618 is
(Fan Pattern I Stewart Industries has been producing two bearings, components 812 and 818. for
use in production.

B12 B18
Machine hours required per unit 2.5 3.0
Standard cost per unit:
Direct material $2.25 $3.75
Direct labor 4.00 4.50
Manufacturing overhead:
variable (see note 1) 2.00 2.25
fixed (see note 2) 3.75 4.50
$12.00 $15.00

Stewart's annual requirement for these components is 8,000 units of 812 and 11,000 units of
B18. Recently, Stewart's management decided to devote additional machine time to other
product lines resulting in only 41,000 machine hours per year that can be dedicated to the
production of the bearings. An outside company has offered to sell Stewart the annual supply of
the bearings at prices of $11.25 for B12 and $13.50 for B18. Stewart wants to schedule the
otherwise idle 41,000 machine hours to produce bearings so that the company can minimize its
mots (maximize its net benefits).
Note I: Variable manufacturing overhead is applied on the basis of direct labor hours.
Note 2: Fixed manufacturing overhead is applied on the basis of machine hours.
A. $.50
B. $(1.00)
C. $(1.75)
D. Some amount other than those given.
Explanation:
Answer (A) is incorrect.
Subtracting $13.50 from $15.00 and dividing by 3 machine hours results in $.50.
Answer (B) is correct.
The variable costs of producing B18 total $10.50 ($3.75 + $4.50 + $2.25). Thus, purchasing at
$13.50 would result in a loss of $3 per bearing. Given that each bearing requires 3 hours of
machine time, the loss is $1 per machine hour.
Answer (C) is incorrect
Not including variable manufacturing overhead when calculating the cost of producing B18
results in $(1.75) [($8.25 - $13.50) ÷3 machine hours].
Answer (D) is incorrect.
The loss per machine hour that results from buying a component is $(1.00) [($10.50 - $13.50) ÷
3].

Question #40: According to IMAs Statement of Ethical Professional Practice, a member has a
responsibility to recognize professional limitations. Under which standard of ethical conduct
would this responsibility be included?
A. Competence.
B. Confidentiality.
C. Integrity.
D. Credibility.
Explanation:
Answer (A) is correct.
The competence standard pertains to an IMA member's responsibility to 'recognize and
communicate professional limitations or other constraints that would preclude responsible
judgment or successful performance of an activity."
Answer (B) is incorrect.
The confidentiality standard concerns an IMA member's responsibility not to disclose or use the
firm's confidential information.
Answer (C) is incorrect
The integrity standard deals with conflicts of interest avoidance of acts discreditable to the
profession, and refraining from activities that prejudice the ability to carry out duties ethically.
Answer (D) is incorrect.
Credibility is the fourth standard of IMA's Statement of Ethical Professional Practice. It requires
that information be communicated "fairly and objectively' and that all information that could
reasonably influence users be disclosed.

Question #41: Inappropriate earnings management is typically considered one form of

A. Embezzlement.
B. Fraudulent financial reporting.
C. Theft of assets.
D. Misappropriation of assets.
Explanation: Answer (A) is incorrect.
Embezzlement is a form of misappropriation of assets.
Answer (B) is correct
In-appropriate earnings management has been defined as the purposeful intercession in the
financial reporting process.
Answer (C) is incorrect
Theft is a form of misappropriation of assets.
Answer (D) is incorrect.
Misappropriation of assets is one of the two main classifications of employee fraud.

Question #42: Question: The discounted, net-of-tax amount that relates to disposal of the
existing asset is (please refer the fact pattern below)
Fact Pattern 1561
The following data pertain to a 4-year project being considered by Metro Industries:
 A depreciable asset that costs $1,200,000 will be acquired on January 1. The asset;
which is expected to have a $200,000 salvage value at the end of 4 years, qualifies as
3-year property under the Modified Accelerated Cost Recovery System (MACRS).
 The new asset will replace an existing asset that has a tax basis of $150,000 and can
be sold on the same January 1 for $180,000.
 The project is expected to provide added annual sales of 30,000units at $20.
Additional cash operating costs are: variable, $12 per unit; fixed, $90,000 per year.
 A $50,000 working capital investment that is fully recoverable at the end of the fourth
year is required.
 Metro is subject to a 40%income tax rate and rounds all computations to the nearest
dollar .Assume that any gain or loss affects the taxes paid at the end of the year in
which it occurred. The company uses the net present value method to analyze
investments and will employ the following factors and rates.

Period Present value of $1 at 12% Present value of $1 Annuity at 12% MACRS


1 0.89 0.89 33%
2 0.80 1.69 45
3 0.71 2.40 15
4 0.64 3.04 7
A. $168,000
B. $169,320
C. $180,000
D. $190,680
Explanation:
Answer (A) is incorrect.
The amount of $168,000 fails to discount the outflow for taxes.
Answer (B) is correct.
The cash inflow from the existing asset is $180,000, but that amount is subject to tax on the
$30,000 gain ($180,000 - $150.000 tax basis). The tax on the gain is $12,000
($30,000×40%). Because the tax will not be paid until year end, the discounted value is
$10,680 ($12,000 ×.89 PV of $1 at 12% for one period). Thus, the net-of-tax inflow is
$169,320 ($180,000 - $10,680).
NOTE: This asset was probably a Section 1231 asset, and any gain on sale qualifies for the
special capital gain tax rates. Had the problem not stipulated a 40% tax rate, the capital gains
rate would be used. An answer based on that rate is not among the options.
Answer (C) is incorrect
The amount of $180,000 ignores the impact of income taxes.
Answer (D) is incorrect.
The discounted present value of the income taxes is an outflow and is deducted from the
inflow from the sale of the asset.

Question #43: When none of the three fraud risk factors are present, an accountant
A. Can rule out the presence of fraud.
B. Should consider the likelihood of fraud to be high.
C. Should not rule out the presence of fraud completely.
D. Will likely search mare diligently for fraud.
Explanation:
Answer (A) is incorrect
The accountant would need more solid evidence to rule out the presence of fraud.
Answer (B) is incorrect
When all three factors are present fraud risk is high.
Answer (C) is correct
Even when no factors are observed, an accountant cannot completely exclude the risk of
fraud. Factors may be present but hidden from the accountant
Answer (D) is incorrect.
When fraud risk factors are present the accountant will likely search more diligently for
fraud.

Question #44: A manufacturer is considering dropping a product line. It currently produces a


multipurpose woodworking clamp in a simple manufacturing process that uses special
equipment Variable costs amount to $6.00 per unit Fixed overhead costs, exclusive of
depreciation, have been allocated to this product at a rate of $3.50 a unit and will continue
whether or not production ceases. Depreciation on the special equipment amounts to $20,000
a year. Fixed costs are $18,000. The damp has a selling price of $10 a unit. Ignoring tax
effects, the minimum number of units that would have to be sold in the current year to break
even on a cash flow basis is
A. 4,500units.
B. 5,003units.
C. 20,000 units.
D. 36,000 units.
Explanation:
Answer (A) is correct
The BEP in units is equal to fixed costs divided by the unit contribution margin ($10 unit
selling price-$6 unit variable cost). Hence, the number of units must be sold to break even on
continuation of the product line is 4,500 [$18,000fixed costs÷ ($10- 56)]. Fixed overhead
allocated is not considered in this calculation because it is not a cash flow and will continue
regardless of the decision.
Answer (B) is incorrect.
The BEP is equal to the salvage value (not depreciation) divided by the UCM of $4 ($10-
$6).Depreciation is a non-cash flow and therefore should not be considered in the cash flow
breakeven point calculation.
Answer (C) is incorrect
The BEP is equal to the salvage value (not depreciation) divided by the UCM of $4 ($10-$6).
Depreciation is a non-cash flow and therefore should not be considered cash flow breakeven
point calculation.
Answer (D) is incorrect.
Unit fixed costs should not be subtracted in determining the unit contribution margin. The
fixed costs will continue regardless so they are not included in the calculation. Therefore the
$18,000 salvage value will be divided by the $4 unit contribution margin in determining the
cash flow breakeven point in units.

Question #45: A company uses cost-volume-prone (CVP) analysis to evaluate a new product.
The total fixed cost of production is $600,000. If the product would break even with 10,000
units sold per year, and the unit variable cost is $25, the unit selling price is
A. $35
B. $60
C. $85
D. $1,500
Explanation:
Answer (A) is incorrect
The amount of $35 result from subtracting variable costs twice.
Answer (B) is incorrect
The amount of $60 is the unit contribution margin
Answer (C) is correct
The selling price can be found using the breakeven point in units formula:
Breakeven point in units = Fixed toss÷ Unit contribution margin
Breakeven point in units = Fixed costs÷ (Selling price per unit -Variable cost per unit)
10,000 = $600,000 ÷ (Price - $25)
Price-$25 = $600,000 ÷ $10,000
Price- $25 = $60
Price= $85
Answer (D) is incorrect.
The sales price is not found by multiplying the variable and fixed costs and dividing by the
number of units.

Question #46: If an investment project has a profitability index of 1.15, the


A. Project's internal rate of return is 15%.
B. Project's cost of capital is greater than its internal rate of return.
C. Project's internal rate of return exceeds its net present value.
D. Net present value of the project is positive.
Explanation:
Answer (A) is incorrect
The I RR is the discount rate at which the NPV is $0, which is also the rate at which the
profitability index is 1.0. The IRR cannot be determined solely from the index.
Answer (B) is incorrect
If the index is 1.15 and the discount rate is the cost of capital, the NPV is positive, and the
IRR must be higher than the cost of capital.
Answer (C) is incorrect
The IRR is a discount rate, whereas the NPV is an amount
Answer (D) is correct.
The profitability index (excess present value index) of an investment is the ratio of the
present value of the future net cash nova (or only cash inflows) to the net initial investment.
It is a variation of the NPV method that facilitates comparison of different-sized investments.
A profitability index greater than 10 indicates a profitable investment i.e., one that has a
positive net present value.

Question #47: In the context of the capital asset pricing model (CAPM), the beta coefficient
of a stock that has the same systematic risk as the market as a whole is equal to
A. 0
B. -1
C. 1
D. 0.5
Explanation:
Answer (A) is incorrect.
A beta of 0 indicates that the stock return is not affected by a change in market return.
Answer (B) is incorrect.
A beta of -1 indicates that the stock return has the exact opposite reaction to a change in
market return.
Answer (C) is correct.
A beta of 1 indicates that for every change in the market return, the stock return has an
identical change: thus, they have the same systematic risk.
Answer (D) is incorrect.
A beta of less than 1.0 means that the security is less volatile than the market; e.g., if the
market return increases by 20% and the security's return increases only 10%, the security has
a beta of 0.5.

Question #48: Question: The annual sales volume required for Madengrad Company to break
even is
Madengrad Company manufactures a single electronic product called Precision mix. This
unit is a batch-density monitoring device attached to large industrial mixing machines used in
flour, rubber, petroleum, and chemical manufacturing Precision mix sells for $900 per unit.
The following variable costs are incurred to produce each precision mix device.

Direct labor $180


Direct materials 240
Factory overhead 105
Variable production
costs $525
Marketing costs 75
Total variable costs $600

Madengrad income tax rate is 40%, and animal fixed costs are $6,600,000. Except for an
operating loss incurred in the year of incorporation, the firm has been profitable over the Last
5 years.
A. 22,000 units
B. 11,003 units.
C. 8,400 units.
D. 13,888 units.
Explanation:
Answer (A) is correct.
The formula for the breakeven point in units divides the fixed costs by the unit contribution
margin ($900 selling price - $600 unit variable cost = $300). Hence, the breakeven point is
22,003 units ($6,600,000 ÷ 300).
Answer (B) is incorrect.
The breakeven point is determined by dividing the fixed costs ($6,600,000) by the unit
contribution margin ($900 selling price - $600 unit variable cost = $300).
Answer (C) is incorrect.
The breakeven point is determined by dividing the fixed costs ($6,600,000) by the unit
contribution margin ($900 selling price - $600 unit variable cost = $300).
Answer (D) is incorrect.
The breakeven point is determined by dividing the fixed costs ($6,600,000) by the unit
contribution margin ($900 selling price - $600 unit variable cost = $300).

Question #49: Fact Pattern: A company is evaluating the possible introduction of a new
version of an existing product that will have a 2-year life cyd e. At the end of 2 years, this
version will be obsolete, with no additional cash flows or salvage value. The initial and sole
outlay for the modified product is $6 million, and the company's desired rate of return is
10%. Following are the potential cash flows (assumed to occur at the end of each year) and
their probabilities if the product is marketed:
The following interest factors for the present value of $1 at 10% are relevant: Period 1.909
2826
Question: Assume the company has the real option to abandon the project at the end of Year
1. If the salvage value at that time is $3 million and the desired rate of return remains at 10%,
what is the project's net present value?
A. $878,050
B. $1,203,550
C. $2,746,450
D. $4,454,100
Explanation:
Answer (A) is incorrect.
The amount of $878,050 equals the NPV with no real option of abandonment.
Answer (B) is correct.
If the cash flows at the end of Year 1 equal $2 million, the expected value of the Year 2 cash
flows is only $2 million [(.5 x 50) + (.5 x $4 million)]. If the cash flows at the end of year 1
equal $4 million or $6 million, the expected value of the Year 2 cash flows equals $4 million
[(.25 x $6.4 million) + (.75 x 53.2 million)] or $5.75 million [(.4 x 56.875 million) + (.6 x 55
million)], respectively. After discounting these expected values to the end of Year 1, the
present values are $1,818,000 (.909 x $2 million) given a 52 million Year 1 cash flow,
$3,636,000 (.909 x $4 million) given a $4 million Year 1 cash flow, and $5,226,750 (.909 x
$5.75 million)given a $6 million Year 1 cash flow. Accordingly, the real option of
abandonment is preferable if the Year 1 cash flow is $2 million. The $3 million salvage value
exceeds the expected value of the Year 2 cash flows discounted to the end of Year 1 in this
case only. If the real option of abandonment is exercised only when Year 1 cash flows equal
$2 million, the expected value of the cash flows at the end of Year 1 is $4.9 million ([.3 x ($2
million + $3 million salvage)] + (.4 x $4 million) + (.3 x $6 million)), and the present value
of this amount is $4,454,100 (.909 x $4.9 million). The expected value of the cash flows at
the end of Year 2 if the real option is exercised only when Year 1 cash flows equal $2 million
is $3,325,000 (.3 x 1.0 x $0) + (.4 x.25 x $6.4 million) + (.4 x.75 x $3.2 million) + (.3 x.4 x
$6.875 million) + (.3 x .6 x $5 million), and the present value of this amount is $2,745,450
(.826 x $3,325,000). Consequently, the NPV with an abandonment option is $1,200,550
($4,454,100 + $2,746,450 - $6 million initial outlay). This amount is substantially greater
than the NPV with no abandonment option.
Answer (C) is incorrect.
The amount of $2,746,450 equals the expected present value of the Year 2 cash flows.
Answer (D) is incorrect.
The amount of $4,454,100 equals the expected present value of the Year 1 cash flows.

Question #50: The CFO at a manufacturer of computer equipment learned last week that the
accounting department has not completed any bank reconciliations for the last 6 months due
to the implementation of a new accounting software package. What type of risk has been
identified?
A. Financial risk
B. Hazard risk
C. Operational risk
D. Strategic risk.
Explanation:
Answer (A) is incorrect
Financial risks encompass interest-rate risk exchange-rate risk commodity risk credit risk
liquidity risk and market risk.
Answer (B) is incorrect
Hazard risks are risks that are insurable. Example include natural disasters, the incapacity or
death of senor officers, sabotage, and terrorism.
Answer (C) is correct
Operational risks are the risks related to the enterprise's ongoing everyday operations.
Operational risk is the risk of loss from inadequate or failed internal processes, people, and
systems. These failures can relate to human resource (e.g., inadequate hiring or training
practice), business processes (poor internal controls), product failure (customer ill will,
lawsuits), occupational safety and health incidents, environmental damage, and business
continuity (power outage, natural disasters).
Answer (D) is incorrect.
Strategic risks include global economic risk political risk and regulatory risk

Question #51: A debt to equity ratio is


A. About the same as the debt to ante ratio.
B. Higher than the debt to assets ratio.
C. Lower than the debt to assets ratio.
D. Not correlated with the debt to assets ratio.
Explanation:
Answer (A) is incorrect
The ratios would always be different unless either debt or equity equaled zero.
Answer (B) is correct.
Because debt plus equity equals assets, a debt to equity ratio would have a lower
denominator than a debt to assets ratio. Thus, the debt to equity ratio would be higher than
the debt to assets ratio.
Answer (C) is incorrect
The lower denominator in the debt to equity ratio means that it would always be higher than
the debt to assets ratio.
Answer (D) is incorrect.
The two ratios are related in that they always move in the same direction.

Question #52: Which of the following statements is true with respect to international transfer
pricing by a US. firm?
A. Transfer prices charged to foreign subsidiaries must be the same as those charged to
domestic subsidiaries
B. The existence of tariffs in the foreign country may necessitate a higher transfer price be
charged a foreign subsidiary.
C. Limitations on taking profits out of a foreign country can be avoided by charging the
foreign subsidiary a higher transfer Price.
D. The transfer price must consider the Internal Revenue Code limitation that the amount of
taxable income that can be claimed by a foreign subsidiary can be no more than 2556 of
the total ;parent plus subsidiary) taxable income.
Explanation:
Answer (A) is incorrect Transfer prices charged to foreign subsidiaries are often quite
different from those charged domestic subsidiaries.
Answer (B) is incorrect
The existence of tariffs in the foreign country would necessitate that a lower transfer price be
charged to avoid a high tariff.
Answer (C) is correct
Transfer pricing is an important aspect of the tax calculation for multinational corporations
that transfer inventories between branches indifferent countries. Transfer prices charged to
foreign subsidiaries may differ substantially from those charged to domestic subsidiaries for
a variety of reasons. Limitations on taking profits out of a foreign country can be avoided by
charging the foreign subsidiary a higher transfer price so that little or no profit exists to be
repatriated.
Answer (D) is incorrect.
The tax lens limit the profit of foreign subsidiaries to 50% of the total.

Question #53: If an I MA member has a problem in identifying unethical behavior or


resolving an ethical conflict, the first action (s)he should normally take is to
A. Consult the board of directors.
B. Discuss the problem with his or her immediate superior.
C. Notify the appropriate law enforcement agency.
D. Resign from the company.
Explanation:
Answer (A) is incorrect
The board would be consulted initially only if the immediate superior is the chief executive
officer and that person is involved in the ethical conflict
Answer (B) incorrect
IMAs Statement of Ethical Professions Practice states that the member should first discuss an
ethical problem with his or her immediate superior. If the superior is involved, the problem
should be taken initially to the next higher managerial level.
Answer (C) is incorrect
An IMA member should keep information confidential except when disclosure is authorized
or legally required.
Answer (D) is incorrect.
Resignation is a last resort.

Question #54: Fact Pattern:


Basic Computer Company (BCC) sells its micro-computers using bid pricing. It develops
bids on a full cost basis. Full cost includes estimated material, labor, variable overheads,
fixed manufacturing overheads, and reasonable incremental computer assembly
administrative costs, plus a 10% return on full cost. BCC's current cost structure, based on its
normal production levels, is $500 for materials per computer and 520 per labor hour. BCCs
variable manufacturing overhead is $2 per labor hour, fixed manufacturing overhead is 53
per labor hour, and incremental administrative costs are 58 per computer assembled.
Question: BCC has received a request from a research lab for 200 computers. Assembly and
testing of each computer will require 17 labor hours. BCC believes bids in scars of $1,050
per computer are not likely to be considered. Using the full-cost criteria and desired level of
return, which one of the following prices should be recommended to BCC's management for
bidding purposes?
A. $874.00
B. $382.00
C. $961.40
D. $1026.30
Explanation:
Answer (A) is incorrect
The amount of $874.00 results from failing to include fixed overhead, the incremental
administrative cost, and the 10% required return.
Answer (B) is incorrect
The amount of $882.00 result from failing to include fixed overhead and the 10% required
return
Answer (C) is incorrect
The amount of $961.40 result from failing to include fixed overhead and the incremental
administrative cost
Answer (D) incorrect.
The price that BCC should bid can be calculated as follows:
$
Materials (given) 500,000
Direct labor (17 hours ×$20) 340.00
Variable overhead(17 hours ×$2) 34.00
Fixed overhead (17 hours ×$3) 51.00
Incremental administrative(given) 8.00
Base cost $933.00
Required return (10%) 93.00
$1,026.3
Total cost 0

Fixed overhead and the 10% required return are included because BCC is using the full-cost
criteria on this bid.

Question #55: A company has decided to discontinue a product produced on a machine


purchased 4 years ago at a cost of $70,000.7he machine has a current book value of $30,000.
Due to technologically improved machinery now available in the marketplace the existing
machine has no current salvage value. The company is reviewing the various aspects
involved in the production of a new product. The engineering staff advised that the existing
machine can to used to produce the new product. Other costs involved in the production of
the new product will be materials of $20,000 and labor priced at $5,000. Ignoring income
taxes, the costs relevant to the decision to produce or not to produce the new product would
be
A. $25,000
B. $30,030
C. $55,030
D. $95,090
Explanation:
Answer (A) is correct
The only relevant costs are those not yet incurred. Thus, the historical cost and book value of
the machine are irrelevant to any future action. Only the incremental materials and labor
needed to bring the machine into readiness to produce the new product have relevance since
these amounts have not yet been spent ($20,000+$5,000- $25,030).
Answer (B) is incorrect
The $30,000 book value is irrelevant
Answer (C) is incorrect.
The sum of the book value and the incremental costs to refurbish the machine is $55,000.
Answer (D) is incorrect.
The sum of the historical cost and the incremental costs to refurbish the machine is $95,000.

Question #56: If Company A has a higher rate of return on assets than Company B. the
reason may be that Company A has a pram margin on sales, a asset turnover ratio, or both.
A. Higher (List A), (List B) Higher
B. Higher (List A), (List B) Lower
C. Lower (List A), (List B) Higher
D. Lower (List A), (List B) Lower
Explanation:
Answer (A) is correct. The DuPont model treats the return on assets as the product of the
profit margin and the asset turnover:
Return on assets= Profit margin + Asset turnover
Net Income Net income Sales
= =
Assets Sales Assets
If one company has a higher return on assets than another, it may have a higher profit
margin, a higher asset turnover. or both.
Answer (B) is incorrect. A higher profit margin on sales or a higher asset turnover ratio may
explain a higher return on assets.
Answer (C) is incorrect.
A higher profit margin on sales or a higher asset turnover ratio may explain a higher return
on assets.
Answer (D) is incorrect.
A higher profit margin on sales or a higher asset turnover ratio may explain a higher return
on assets.

Question #57: Public record searches may be effective in certain instances. Which of the
following is a limitation on public record searches?
A. It is often very costly to search public records.
B. Very few types of information are available
C. The information from public sources is most often incorrect.
D. Availability of records may be limited
Explanation:
Answer (A) is incorrect
Use of public sources of information is usually cost effective. In fact, Internet searches may
be virtually costless.
Answer (B) is incorrect
A vast amount of information is available from public sources
Answer (C) is incorrect
In most cases, the information obtained is correct. However, care should be taken because
information may have changed since it was first collected and reported
Answer (D) is correct.
Records for early periods may be limited or nonexistent.

Question #58: Question: The Company has a fixed assets balance of


[Fact Pattern #14]
A company has a current ratio of 1.4, a quick, or acid test, ratio or 1.2, and the following
partial summary balance sheet:

Cash $10
Accounts Receivable
Inventory
Fixed assets
Total Assets $100
Current liabilities $
Long term liabilities 40
Stockholder's Equity 30
Total liabilities and equity $

A. $16
B. $58
C. $64
D. $84
Explanation: Answer (A) is incorrect
Neglecting to subtract the equity balance when calculating the current liability balance results
in $16.
Answer (B) is correct.
Total assets (given as $100) equals the sum of cash (given as $10), account receivable ($26
as calculated using the quick ratio), inventory, and fixed assets. Inventory can be determined
because it is included in current, but not quick asset, and the current and quick ratios are
known. Current assets equal $42 (1.4 current ratio× $30 current liabilities), and the quick
assets equal $36 (1.2 quick ratio× $30 current liabilities). Thus, inventory, which is the only
difference in this question between current and quick assets, equals $6 ($42 - $36). Fixed
assets must then equal $58 ($100 total assets - $10cash - $26 accounts receivable - $6
inventory).
Answer (C) is incorrect
Assuming that inventory is $0 results in $64.
Answer (D) is incorrect
Ignoring account receivable result in $84.

Question #59: Year-end bonuses at each branch office are based on branch profitability. Due
to a slow economy, profitability through the third quarter at the Northeast branch is under
budget. To address this issue, the accounting staff at the Northeast branch develops a list of
end-of-year actions designed to boost earnings for the year. Which one of the following is
most likely to violate IMAs Statement of Ethical Professional Practice?
A. Requesting the branch's advertising agency to delay billing third-quarter advertisements
until January.
B. Deferring advertising expense by reducing the number of newspaper advertisements run
in the third quarter.
C. Deferring planned painting and refurbishment of the warehouse until the following year.
D. Offering additional discounts to customers to entice them to increase purchases in the
third quarter.
Explanation:
Answer (A) is correct
IMAs Statement of Ethical Professions Practice states that its members have responsibilities
in the areas of competence, confidentiality, integrity, and credibility. Requesting the branch's
advertising agency to delay billing third quarter advertisements until January will most likely
violate. IMAs Statement of Ethical Professional Practice in the areas of integrity and
credibility. Integrity is impaired because an IMA member should abstain from engaging in or
supporting any activity that might discredit the profession, and credibility is impaired
because an IMA member should disclose all relevant information that could reasonably be
expected to influence an intended use's understanding of the reports, analyses, or
recommendations.
Answer (B) is incorrect
The IMAs Statement of Ethical Professional Practice states that its members have
responsibilities in the areas of competence, confidentiality, integrity, and credibility.
Deferring advertising expense by reducing the number of newspaper advertisements run in
the third quarter does not violate the IMAs Statement of Ethical Professional Practice-the
expense twenty matches the number of newspaper advertisements run in the respective
period.
Answer (C) is incorrect.
The IMAs Statement of Ethical Professional Practice states that its members have
responsibilities in the areas of competence, confidentiality, integrity, and credibility.
Deferring planned painting and refurbishment of the warehouse until the following year does
not violate the IMAs Statement of Ethical Professional Practice-changing plans is ethical
since there is no mention of a contractual deadline
Answer (D) is incorrect.
The IMAs Statement of Ethical Professional Practice states that its members have
responsibilities in the areas of competence confidentiality, integrity, and credibility. Offering
additional discounts to customers to entice them to increase purchases in the third quarter
does not violate the IMAs Statement of Ethical Professional Practice-offering additional
discounts is used widely in practice and is not unethical unless it involves predatory pricing

Question #60: Following are the operating results of the two segments of Parklin
Corporation.

Segment A Segment B Segment C


Sales $10,000 $15,000 $25,000
Variable costs of goods sold -4,000 -8,500 -12,500
Fixed costs of goods sold -1,500 -2,500 -4,000
Gross margin $4,500 $4,000 $8,500
Variable selling and administrative -2,000 -3,000 -5,000
Fixed selling and administrative -1,500 -1,500 -3,000
Operating income (less) $1,000 ($500) $500

Fixed costs of goods sold are allocated to each segment based the number of employees.
Fixed selling and administrative expense are allocated equally. If Segment B is eliminated,
$1,500 of fixed costs of goods sold would be eliminated. Assuming segment B is closed, the
effect on operating income would be a(n)
A. Increase of $500.
B. Increase of $2,000.
C. Decrease of $2,090.
D. Decrease of $2,500.
Explanation:
Answer (A) is incorrect.
A $500 increase results from simply adding back Segment B's operating loss.
Answer (B) is incorrect
A $2,000 increase results from reversing the effects of deleting the elements of Segment B.
Answer (C) is correct
The effect of closing Segment B on operating income can be calculated as follows:

($15,000
Sales eliminated )
Variable costs of goods sold eliminated 8,500
Fixed costs of goods sold eliminated 1,500
Variable S&A expenses eliminated 3,000
Net effect ($2,000)

Answer (D) is incorrect.


A $2,503 decrease results from removing all the relevant elements of Segment B and
removing the operating loss, in effect double-counting.

Question #61: Selected financial data for the year is shown below:

Beginning of year End of year


Assets $9,600,000 $10,000,000
Liabilities $6,200,000 $6,800,000
Shares outstanding 1,400,000 1,500,000
Market price per share $2.40 $2.50

Sales $22,000,000
Earnings before interest and taxes $1,700,000
Interest expense $500,000
Tax rate 40%
A. 15%
B. 19%
C. 22%
D. 36%
Explanation:
Answer (A) is incorrect
This percentage incorrectly uses net income instead of sales for the equity multiplier.
Answer (B) is incorrect
This percentage uses ending equity instead of average equity for the equity multiplier.
Answer (C) is correct
The DuPont model depicts return on equity as the profit margin (net income divided by sales)
times total asset turnover (sales divided by average total assets) times the equity multiplier
(average total assets divided by average total equity). Thus, the return on equity using the
DuPont model is 3.27% ($720,000 ÷ $22,000,000) x 2.24 ($22,000,000 ÷ $9,800,000) x 2.97
($9,800.000 ÷ $3,300,000) 22%
Answer (D) is incorrect.
This percentage uses the incorrect net income amount when calculating profit margin. The
net income amount only deducts interest expense from earrings before interest and tax and
fails to deduct taxes.

Question #62: A corporation has 6,000 shares of 5% cumulative. $100 par value preferred
stock outstanding and 200,000 shares of common stock outstanding. The corporation's board
of directors last declared dividends for the year ended May 31.Year 1, and there were no
dividends in arrears. For the year ended May31. Year 3. The corporation had net income of
$1.750000. The board of directors is declaring a dividend for common shareholders
equivalent to 20% of net income. The total amount of dividends to be paid at May 31, Year
3.
A. $350,000
B. $380,000
C. $206,000
D. $410,000
Explanation:
Answer (A) is incorrect
The amount of $350,000 is the common stock dividend.
Answer (B) is incorrect
The amount of $380,000 omits the $30,000 of cumulative dividends for the year ended
May31Year 2
Answer (C) is incorrect
The amount of $206,000 is based on a flat rate of $1 per share of stock.
Answer (D) is correct.
If a company has cumulative preferred stock all preferred dividends for the current and any
unpaid prior years must be paid before any dividends can be paid on common stock. The
total preferred dividends that must be paid equal $60,000 (6,000 share x $100par x 5%x 2
years), and the common dividend is $350,000 ($1,750,000x 20%), for total of $410,000.

Question #63: Fact Pattern: A firm's dividend policy may treat dividends either as the
residual part of a financing decision or as an active policy strategy.
Question: Treating dividends as the residual part of a financing decision assumes that
A. Earnings should be retained and reinvested as long as profitable projects are available.
B. Dividends are important to shareholders, and any earnings left over after paying
dividends should be invented in high-return assets.
C. Dividend payments should be consistent
D. Dividends are relevant to a financing decision.
Explanation:
Answer (A) is correct
According to the residual theory of dividends, the amount (residual) of earnings paid as
dividends depends on the available investment opportunities and the debt-equity ratio at
which cost of capital is minimized. The rational investor should prefer reinvestment of
retained earnings when the return exceeds what the investor could earn on investments of
equal risk However, the firm may prefer to pay dividends when interment opportunities are
poor and the use of internal equity financing would move the firm away from its ideal capital
structure.
Answer (B) is incorrect
A residual theory assume that investors want the company to reinvest earnings in worthwhile
projects, not pay dividends.
Answer (C) is incorrect
Dividend payments will not be consistent under a residual theory. The corporation will pay
dividends only when internal investment options are unacceptable
Answer (D) is incorrect.
Dividends would not be important to a financing decision under the residual theory.

Question #64: Mat is the role of gold in the present international monetary system?
A. Gold is quoted in United States dollars only.
B. All of the major currencies of the world, except the United States dollar, have a fixed
value in terms of gold.
C. Gold is like any other asset whose value depends upon supply and demand.
D. Gold is the reserve asset of the International Monetary Fund.
Explanation:
Answer (A) is incorrect
Although most exchange quote the price of gold in U.S. dollars, the dollar's value is not
linked to that of gold.
Answer (B) is incorrect
Floating exchange rates have existed since about 1973. Tying currency values to a gold
standard, in effect, fixes exchange rates.
Answer (C) is correct
Gold has no special rose in the modern international monetary system. The present system is
based upon managed floating currency exchange rates. Consequently, gold is treated as a
commodity, the price of which depends upon supply and demand.
Answer (D) is incorrect.
The only reserves of the IMF are international currencies.

Question #65: (Please refer the fact pattern below)


Question: What is the breakeven point of the large venue?
Harper and her band want to put on a concert. They have looked at two venues, A small one
and a large one, and have compiled the following information:

Small Large
Capacity of venue 400 1,200
Fixed costs $2,000 $5,000

The variable cost per customer for both venues is $2. The band will charge $10 per customer
for the small venue or $14 for the large venue.
A. 358
B. 375
C. 381
D. 417
Explanation:
Answer (A) is incorrect
Not subtracting the variable cost per person from the revenues per person results in 358 ticket
sales.
Answer (B) is incorrect
The amount needed to break even is 417 ticket sales.
Answer (C) is incorrect
The amount needed to break even is 417 ticket sales.
Answer (D) is correct.
The breakeven point in units equals total fixed costs divided by the unit contribution margin.
For the large venue, UCM is $12 ($14 selling price - $2 variable cost) and the breakeven
point is 416.6 tickets ($5,000 ÷ $12).

Question #66: A company is in the process of evaluating a major product line expansion.
Using a 14% discount rate, the firm has calculated the present value of both the project's cash
inflows and cash outflows to be $15.8 million. The company will likely evaluate this project
further by
A. Taking a closer look at the expansion's contribution margin
B. Comparing the internal rate of return versus the accounting rate of return.
C. Comparing the internal rate of return versus the company's cost of capital.
D. Comparing the internal rate of return versus the company's cost of capital and hurdle rate.
Explanation:
Answer (A) is incorrect
Contribution margin is not a comprehensive measure of a projects profitability.
Answer (B) is incorrect
The accounting rate of return is derived from accrual-basis numbers, making it an
inappropriate measure for assessing capital projects.
Answer (C) is incorrect
The cost of capital by itself is not a sufficient measure of the suitability of a capital projects.
Answer (D) is correct.
The discount rate at which a projects discounted net cash inflows equal its discounted net
cash outflows is referred to as the internal rate of return (IRR). At this discount rate, the
projects net present value is $0. To determine whether a project with a certain IRR is
acceptable, this rate of return must be compared with the firm's current cost of capital and its
hurdle rate, i.e., the rate of return that management has chosen as the benchmark for
acceptable projects.

Question #67: Fact Pattern: Jensen Corporation's board of directors met on June 3 and
declared a regular quarterly cash dividend of $.40 per share for a total value of $200,000. The
dividend is payable on June 24 to all stockholders of record as of June 17. Excerpts from the
statement of financial position for Jensen Corporation as of May 31 are presented as follows.
Cash $400,000
Accounts receivables (net) 800,000
Inventories 1,200,000
Total current assets $2,400,000
Total current liabilities $1,000,000

A. Unchanged by the dividend declaration and decreased by the dividend payment


B. Decreased by the dividend declaration and increased by the dividend payment
C. Unchanged by either the dividend declaration or the dividend payment.
D. Decreased by the dividend declaration and unchanged by the dividend payment
Explanation:
Answer (A) is incorrect
The declaration of a dividend increases current liabilities and reduce working capital. The
subsequent payment has no effect on working capital.
Answer (B) is incorrect
The declaration of a dividend increases current liabilities and reduce working capital. The
subsequent payment has no effect on working capital.
Answer (C) is incorrect
The declaration of a dividend increases current liabilities and reduce working capital. The
subsequent payment has no effect on working capital.
Answer (D) is correct.
Working capital is defined as current assets minus current liabilities. The declaration of a
dividend increases current liabilities and thus reduces working capital. The subsequent
payment has no effect on working capital since current assets and current liabilities decrease
by the same amount.

Question #68: Fact Pattern: Regis Company manufactures plugs used in its manufacturing
cycle at a cost of $36 per unit that includes $8 of fixed overhead. Regis needs 30,000 of these
plugs annually, and Orlan Company has offered to sell these units to Regis at $33 per unit. If
Regis decides to purchase the plugs. $60,000 of the annual fixed overhead applied will be
eliminated, and the company may be able to rent the facility previously used for
manufacturing the plugs.
Question: If the plugs are purchased and the facility rented. Regis Company wishes to realize
$100,000 in savings annually. To achieve this goal, the minimum annual rent on the facility
must be
A. $10,000
B. $40,000
C. $70,000
D. $190,000
Explanation:
Answer (A) is incorrect
The loss from purchasing must be added to the targeted savings to calculate the minimum
annual rate.
Answer (B) is incorrect
The loss from purchasing must be added to the targeted savings to calculate the minimum
annual rate.
Answer (C) is incorrect
The loss from purchasing must be added to the targeted savings to calculate the minimum
annual rate.
Answer (D) is correct.
If Regis purchases the plugs. Regis will still incur fixed costs per unit of $6 [$8 - ($60,000 ÷
$30,000)], however, since these costs are committed (sunk costs), they are not relevant to this
decision. Thus, the relevant cost per unit will be $30 ($36 cost - $6 fixed cost per unit).
Without regard to rental of idle production capacity, the company will lose $3 per unit ($33
purchase price - $30 relevant cost) by purchasing the plugs. The total annual loss will be
$90,000 (30,000 units x $3). Consequently, to achieve the targeted savings, the minimum
annual rent must be $190,000 ($90,000 loss from purchasing + $100,000 targeted savings).

Question #69: A corporation's inventory expressed as a percentage of current assets increased


from 25% last July to 35% this July. The factor that is least likely to cause this increase is
that the corporation

A. Is a seasonal company with traditionally higher activity in the summer months


B. Is beginning to experience high growth.
C. Has inventory that is becoming obsolete.
D. Used a material amount of cash from selling its short-term investments to purchase land.
Explanation:
Answer (A) is correct
This statement is least likely to explain an increase in current assets from last July, to this
July. If the corporation was a seasonal company with traditionally higher activity in the
summer months, it would budget similar amounts for each summer in expectation of the high
activity. The sudden increase in current assets for the following summer would not be
explained by the fact that they are a seasonal company.
Answer (B) is incorrect
If the corporation was beginning to experience high growth, it would have to purchase more
inventory in order to meet the higher demand from the growth. This would cause the current
assets account to increase.
Answer (C) is incorrect.
Obsolete inventory refers to inventory held by a company that is at the end of its product life
cycle and has not seen any sates or usage for a set period of time. If the corporation has
inventory that is becoming obsolete the inventory will be held by the company instead of
being sold. This would cause an increase in the current assets on the books.
Answer (D) is incorrect.
It can be assumed that the sale of short-term investments generated a gain, which caused a
bigger cash inflow than the outflow from the short-term investments. This means that the
current assets account increased. The fact that the company bought land with scene of the
cash is meant as a distractor, as the question does not state how much of the cash was used to
purchase the land.
Question #70: An abbreviated common-size income statements for Year 1’s actual results
and Year 2's anticipated results are shown below.

Year 1 Year 2
Sales 100% 100%
Costs of goods sold 50% 50%
Selling and administrative expenses 40% ?
Operating income 10% ?

The corporation estimates that units sold will increase by 5% in Year 2 with no price increase
to its customers and no anticipated cost increases from its vendors. Assume selling and
administrative expenses are 5% variable and 95% fixed. If all predictions materialize, the
corporation should expect selling and administrative expenses in Year 2 to be
A. Less than 40% of sales.
B. 40% of sales.
C. Greater than 40% but no more than 42% of sales.
D. Greater than 42%of sales.
Explanation:
Answer (A) is correct
This question is bet answered using actual numbers. Assume that sales in Year 1 are $500.
Because total selling and administrative expenses are 40% of Year 1 sale, selling and
administrative expenses equal $200 ($500 x 40%). Given that 5% of this is variable and 95%
a fixed, variable expense equals $10 ($200x 5%) and fixed expense equals $190 ($200x
95%). In relation to sale, variable selling and administrative expenses are equal to 2% ($10
variable ÷ $500 sales) of sales. This percent will help calculate the variable selling and
administrative expenses in Year 2.
In Year 2, sales increase by 5%, making Year 2 sales equal to $525 ($500Year 1 sales x 1.05
increase). The fixed portion of the selling and administrative expense is equal to $190. The
variable portion can be solved by multiplying 2% by sales of $525, which results in $10.50.
Therefore total selling and administrative expenses are equal to $200.50, about 38.20%
($200.50 ÷ $525) of Year 2 sales, which is less than 40%.
Answer (B) is incorrect
Selling and administrative expense are not the same in Year 2 as they were in Year 1.
Answer (C) is incorrect
Year 2 selling and administrative expenses are not greater than 40% of sales.
Answer (D) is incorrect.
Year 2 selling and administrative expense are not greater than 42% of sales.
Question #71: In the current year, a firm had $15 million in sales, while total fixed costs were
held to $6 million. The firm's total assets averaged $20 million and the debt-to-equity ratio
was calculated at 0.60. If the firms Ear is $3 million, the interest on all debt is 9%, and the
tax rate is 40%, what is the firm's return on equity?
A. 11.16%
B. 14.4%
C. 18.6%
D. 24.0%
Explanation:
Answer (A) is correct
The first step is to determine the amount of equity. Since the debt-to-equity ratio is .6, 60D=
100E. Additionally, since A =L + E, we can substitute D for L, plug in the value of assets and
solve for E as follows:
$20 million = D+E
$20 million = .6E + E
$20 million = 1.6E
Thus, E (equity) equals $12.5 million. Debt is therefore $7.5 million (.6 × $12.5 million).
At 9%, interest on $7.5 million of debt is $675,000. Earnings before taxes are $2,325,000
($3,000,000 EBIT - $675,000 interest). At a 40% tax rate, taxes are $930,000, which leaves a
net income of $1,395,000. Return on equity is calculated by dividing the $1,395,000 net
income by the $12,500,000 of equity capital, giving an ROE of 11.16%
Answer (B) is incorrect
A failure to deduct interest expense results in 14.4%
Answer (C) is incorrect
A failure to deduct income taxes results in 18.6%
Answer (D) is incorrect
Using the wrong amount of equity results in 24.0%

Question #72: Corporations have the responsibility to issue financial statements that are
timely, accurate• and transparent, reflecting all the transactions of the company. Which of the
following documents refer to this responsibility?
I. IMAs Statement of Ethical Professional Practice
II. SOX Section 406: Code of Ethics for Senior Financial Officers
III. IMAs Statement on Management Accounting “Values and Ethic: From Inception to
Practice”
IV. U.S. Foreign Corrupt Practices Act
A. I and II only.
B. I and III only.
C. II and III only.
D. II and IV only.
Explanation:
Answer (A) is incorrect
The IMAs Statement of Ethical Professional Practice discusses ethical principles and standards
that should be followed by members of the IMA. This does not refer to the responsibility to issue
financial statement that are timely, accurate, and transparent reflecting all the transactions of the
company.
Answer (B) is incorrect
The IMAs Statement on Management Accounting “Values and Ethics: From Inception to
Practice” is a useful document for understanding ethical concepts in an organizational context.
This does not refer to the responsibility to issue financial statements that are timely, accurate,
and transparent, reflecting all the transactions of the company. The IMAs Statement of Ethical
Professional Practice discusses ethical principles and standards that should be followed by
members of the IMP. This does not refer to the responsibility to issue financial statements that
are timely accurate and transparent reflecting all the transactions of the company.
Answer (C) is incorrect
The IMAs Statement on Management Accounting “Values and Ethics: From Inception to
Practice” is a useful document for understanding ethical concepts in an organizational context.
This does not refer to the responsibility to issue financial statements that are timely, accurate,
and transparent, reflecting all the transactions of the company.
Answer (D) is correct.
SOX Section 406: Code of Ethic for Senior Financial Officers and the U.S. Foreign Corrupt
Practices Act both refer to the corporate responsibility to issue financial statement that are
timely, accurate, and transparent reflecting all the transactions of the company.

Question #73: A measure of long-term debt-paying ability is a company's

A. Length of the operating cycle.


B. Return on assets.
C. Inventory turnover ratio.
D. Times interest earned ratio.
Explanation:
Answer (A) is incorrect
The length of the operation cycle does not affect long-term debt-paying ability. By
definition, long-term means longer than the normal operating cycle.
Answer (B) is incorrect
Return on assets measures only how well management uses the assets that are available It
does not compare the return with debt service costs.
Answer (C) is incorrect
The inventory turnover ratio is a measure of how well a company is managing one of its
current assets.
Answer (D) is correct.
The times interest earned ratio is one measure of a firm's ability to pay the interest on its
debt obligations out of current earnings. This ratio equals earnings before interest and
taxes divided by interest expense.

Question #74: Costs relevant to an insourcing vs. outsourcing decision include variable
manufacturing costs as well as

A. Avoidable fixed con.


B. Factory depreciation.
C. Property taxes.
D. Factory management costs.
Explanation:
Answer (A) is correct
Relevant costs are anticipated costs that will vary among the choices available. If two courses
of action share some costs, those costs are not relevant because they will be incurred
regardless of the decision made. Relevant costs include fixed costs that could be avoided if
the items were purchased from an outsider.
Answer (B) is incorrect
Depreciation should not be considered unless it can be avoided.
Answer (C) is incorrect
Property taxes are not affected by the decision and are therefore not relevant unless the
decision to buy leads to sale of the property.
Answer (D) is incorrect.
Factory management costs are not affected by the decision and are therefore not relevant
unless the decision to buy reduces the number of factory managers.

Question #75: A condensed comparative balance sheet for a company appears below:

12/31/Year 1 12/31/Year 2
Cash $40,000 $30,000
Accounts receivable 120,000 100,000
Inventory 200,000 300,000
Property, plant, & equipment 500,000 550,000
Accumulated depreciation -280,000 -340,000
Total Assets $580,000 $640,000
Current liabilities $60,000 $100,000
Long-term liabilities 390,000 420,000
Stockholders' equity 130,000 120,000
Total liabilities and equity $580,000 $640,000

A. Increased (Current Ratio), (Acid Test Ratio) Increased


B. Increased (Current Ratio), (Add Test Ratio) Decreased
C. Decreased (Current Ratio), (Acid Test Ratio) Increased
D. Decreased (Current Ratio), (Acid Test Ratio) Decreased
Explanation:
Answer (A) is incorrect
Both ratios decreased.
Answer (B) is incorrect
Both ratios decreased.
Answer (C) is incorrect
Both ratios decreased.
Answer (D) incorrect.
The current ratio is determined by dividing current assets by current liabilities. The acid test
ratio is determined by dividing quick assets by current liabilities. At December 31, Year 1
the, current ratio is 6 to 1[($40,000 + $120,000 + $200,000) ÷ $60,000]. At December 31,
Year 2, the current ratio is 4.3 to 1 [($30,000 + $100,000 + $300,000) ÷ $100,000]. Hence,
there was a decrease in the current ratio At December 31, Year 1, the acid test ratio is 2.667
to 1 [($40,000 + $120,000) ÷ $60,000). At December 31, Year 2, the acid test ratio is 1.3 to
1[($30,000 + $100,000) ÷ $100,000]. Thus, the add test ratio also declined.

Question #76: A credit manager considering whether to grant trade credit to a new customer
is most likely to place primary emphasis on
A. Profitability ratios.
B. Valuation ratios.
C. Growth ratios.
D. Liquidity ratios.
Explanation:
Answer (A) is incorrect
Equity investors are concerned with profitability ratios.
Answer (B) is incorrect
Equity investors are concerned with valuation ratios.
Answer (C) is incorrect
Equity investors are concerned with growth ratios.
Answer (D) is correct.
Liquidity is a firm's ability to pay its short-term obligations as they come due. Trade payables
are the most common form of shore-term obligation Thus, a credit manager is most interested
in assessing a potential customer's liquidity.

Question #77: A company is considering four independent investment proposals. The


company has $3 million available for investment during the present period. The investment
outlay for each project and its projected net present value (NPV) is presented below.
Investment
Project cost NPV
I $500,000 $40,000
II 900,000 120,000
III 1,200,000 180,000
IV 1,600,000 150,000

Which of the following project options should be recommended to the company'


management?
A. Projects I,II, and III only
B. Projects I,II, and IV only
C. Projects II,III and IV only
D. Projects III and IV only
Explanation:
Answer (A) is correct. Capital rationing exists when a firm sets a limit on the amount of funds to
be invested during a given period. In such situations, a firm cannot afford to undertake all
profitable projects. The profitability index (or excess present value index) is a method for
ranking projects to ensure that limited resources are placed with the investments that will return
the highest net present value (NPV).
NPV of future cash flows
Profitability index=
Net investment
The indexes for Woods’ potential projects can thus be calculated as follows

Project Investment cost NPV Profitability Index

I $500,000 $40,000 0.08

II 900,000 120,000 0.133

III 1,200,000 180,000 0.15

IV 1,600,000 150,000 0.094

Answer (B) is incorrect


Project III is more desirable than Project IV.
Answer (C) is incorrect.
While Project IV is more desirable than Project I. insufficient funding is available to engage
Project IV.
Answer (D) is incorrect.
Projects I and II are also desirable and sufficient funding is available

Question #78: Powell Industries deals with customers throughout the country and is attempting
to more efficiently collect its accounts receivable. A major bank has offered to develop and
operate a lockbox system for Powell at a cost of $90,000 per year. Powell averages 300 receipts
per day at an average of $2,500 each. Its short-term interest cost is 8% per year. Using a 360-day
year, what reduction in average collection time would be needed in order to justify the lockbox
system?
A. 0.67days.
B. 1.20days.
C. 1.25days.
D. 1.50days.
Explanation:
Answer (A) is incorrect
The amount of 0.67days results from reversing the order of division of the cost of the
lockbox ant the amount of float gained.
Answer (B) is incorrect
The amount of 120days results from multiplying by the number of days in the year rather
than the number of payments per day, and dividing this amount by $60,000.
Answer (C) is incorrect
The amount of 125 days results from multiplying by the number of days in the year rather
than the number of payments per day.
Answer (D) is correct.
The amount Powell could potentially earn by investing its cash collections is calculated as
follows:

Average amount per Transaction $2,500


Times: payments per day -300
Daily collections $750,000
×
times: money market rate 8%
Potential return on daily collections $60,000

The reduction in average collection time that justifies the lockbox system is the ratio of its
cost to the potential return ($90,000 ÷ $60,000 = 1.5 days).

Question #79: A construction company has signed 51.000.000 in new contracts. During the
current year. 10% of the required work for these contracts was performed. Historically, the
controller has recognized revenue when the contract work was completed using the
completed contract method. This year, the company’s auditors are requiring the new
contracts to be recognized under the percentage of completion method. The change in
revenue recognition methods will result in a revenue change of
A. $0
B. $(900,000)
C. $100,000
D. $100,000
Explanation:
Answer (A) is incorrect
$100,000.10% of the $1,000,000 contract should be recognized as revenue compared with $0
of revenue recognized under the completed contract method, since the contract is not fully
completed.
Answer (B) is incorrect
$100,000.10% of the $100,000 contract should be recognized as revenue. The completed
contract method recognized $0, not $100,000.
Answer (C) is correct
When the outcome of a transaction involving the rendering of services (e.g. a construction
project cannot be estimated reliably, revenue must be recognized only to the extent of the
expenses recognized that are recoverable. If it is probable that the entity will recover the
transaction costs incurred, revenue is recognized only to the extent of those costs that are
expected to be recoverable. Thus, $100,000, 10% of the $100,000 contract, should be
recognized as revenue compared with $0 of revenue recognized under the completed contract
method, since the contract is not fully completed.
Answer (D) is incorrect.
Only $100,000, 10%d the $1,000,000 contract should be recognized as revenue compared
with $0 of revenue recognized under the completed contract method, since the contract is not
fully completed. The full $1,000,000 will be recognized as the contract is completed.

Question #80: If consumers in Japan decide they would like to increase their purchases of
consumer products made in the United States, in foreign currency markets there will be a
tendency for
A. The supply of dollars to increase.
B. The supply of dollars to decrease
C. The Japanese yen to appreciate relative to the US dollar.
D. The demand for dollars to increase
Explanation:
Answer (A) is incorrect
The demand for dollars, not the supply, will be affected by the decision to purchase
additional US, products.
Answer (B) is incorrect
The demand for dollars, not the supply, will be affected by the decision to purchase
additional US, products.
Answer (C) is incorrect
The dollar should appreciate relative to the yen awing to the increased demand for dollars.
Answer (D) is correct.
The increase in demand for US, products will increase the demand for the dollars necessary
to pay for those products.

Question #81: Question: The times interest earned ratio for Ostrander Corporation for Year 2
is (please refer the fact pattern below)
[fact pattern #29]
Selected data from Ostrander Corporation’s financial statement for the year indicated are
presented in thousands.

Year 2 Operations
Net credit sales $4,175
Cost of goods sold 2,880
Interest expense 50
Income tax 120
Gain on disposal of a segment (net of
tax) 210
Administrative expense 950
Net income 385

December 31

Year 2 Year 1
Cash $32 $28
Trading securities 169 172
Accounts receivable (net) 210 204
Merchandise inventory 440 420
Tangible fixed assets 480 440
Total assets 1,397 1,320
Current liabilities 370 368
Total liabilities 790 750
Common stock outstanding 226 210
Retained earnings 381 360

A. .57 times.
B. 7.70 times.
C. 3.50 times.
D. 6.90 times.
Explanation:
Answer (A) is incorrect
This figure is the debt ratio.
Answer (B) is incorrect
This figure is based on net income from operations after taxes and interest
Answer (C) is incorrect
This figure results from not adding interest and taxes to net income after the gain on &pose'
is subtracted.
Answer (D) is correct.
The interest coverage ratio is computed by dividing earnings before interest and taxes by
interest expense. Net income of $385, minus the disposal gain of $210, is added to income
taxes of $120 and interest expense of $50 to produce a ratio numerator of $345. Dividing
$345 by $50 results in an interest coverage of 6.90 times.

Question #82: As a company becomes more conservative with respect to working capital
policy, it would tend to have a(n)
A. Increase in the ratio of current liabilities to noncurrent liabilities.
B. Decrease in the operating cycle.
C. Decrease in the quick ratio.
D. Increase in the ratio of current assent to noncurrent assets.
Explanation:
Answer (A) is incorrect
An increase in current liabilities relative to noncurrent liabilities would increase liquidity
risk.
Answer (B) is incorrect
A decrease in the normal operating cycle permits a lower level of working capital. If assets
can be converted to cash more quickly, current assent can be reduced.
Answer (C) is incorrect
A decrease in the quick ratio signifies that quick assets (cash receivable, and marketable
securities) are decreasing relative to current liabilities.
Answer (D) is correct.
A conservative working capital policy results in an increase in working capital (current asses
- current liabilities). It is typified by a reduction in liquidity risk Increasing the current ratio,
whether by decreasing current liabilities or increasing current assets, minimizes the risk that
the company will not be able to meet its obligations as they fall due Thus, an increasing ratio
of current to noncurrent assets means that a company is forgoing the potentially higher
returns on long-term assets in order to guard against short-term cash flow problems.

Question #83: If a multinational firm were to raise equity capital on the London Stock
Exchange, this would be referred to as a
A. Morey market transaction
B. Primary market transaction
C. Secondary market transaction
D. Mortgage market transaction.
Explanation:
Answer (A) is incorrect
Money market transactions involve debt securities with maturities of less than 1 year.
Answer (B) is correct.
The primary market is one in which a firm raises additional long-term debt or equity capital.
It is a market in which newly created securities are bought and sold for the first time.
Answer (C) is incorrect
Secondary market transactions involve the trading of already outstanding securities by
investors.
Answer (D) is incorrect.
Mortgage market transactions relate to loans on residential, commercial, industrial, and farm
real estate.

Question #84: A staff accountant becomes aware of an off-balance-sheet bank account where
funds have been diverted with offsetting credits approved by his immediate supervisor. His
immediate supervisor refuses to discuss it and suggests the staff accountant forget about it.
Which one of the following should be the staff accountant's next course of action in this
circumstance?
A. Put concerns in writing to the immediate supervisor and copy the company’s independent
auditor.
B. Discuss concerns with the level of management above the immediate supervisor.
C. Communicate concerns confidentially to the company's independent auditor.
D. Communicate concerns confidentially to the company’s external legal counsel.
Explanation:
Answer (A) is incorrect
When the immediate supervisor appears to be involved, it is not appropriate to discuss the
matter with him or her.
Answer (B) is correct.
In applying the Standards of Ethical Professional Practice, it may be necessary to identify
unethical behavior or resolve an ethical conflict. When faced with ethical issues, it is
important to follow an organization’s established policies on the resolution of such conflict.
If these policies do not resolve the ethical conflict, the first option involves discussing the
issues with the immediate superior. However, when it appears that the supervisor is involved,
the issue should be presented to the next level
Answer (C) is incorrect
It is not necessary to communicate the concerns to the company’s independent auditor unless
a satisfactory resolution was not achieved by different levels of management above the
immediate superior, the audit committee, executive committee, board of directors, board of
trustees, or owners.
Answer (D) is incorrect.
The company’s external legal counsel should not be the next course of action but it may be a
much later course of action.

Question #85: question: Assume Leland Manufacturing is able to rent all idle capacity for
$25,000 per month. If Leland decides to purchase the 10 units from Scott Supply. Leland's
monthly cost for KJ37 would
[Fact pattern]
Leland manufacturing use 10 units of Part number KJ37 each month in the production of
radar equipment. The unit cost to manufacture 1 unit of KJ37 s presented below:

Direct materials $1,000


Materials handling(20% of direct material cost) 200
Direct labor 8,000
Manufacturing overhead (150% of direct labor) 12,000
Total manufacturing cost $21,000

Material handling represents the direct variable costs of the receiving department that are
applied to the direct materials and purchased components on the basis of their cost. This is a
separate charge in addition to manufacturing overhead. Leland’s annual manufacturing
overhead budget is one-third variable and two-third fixed. Scott supply, one of Leland’s
reliable vendors, has offered to supply part number KJ37 at a unit price of $15,000.
A. Increase $48,000.
B. Increase $23,000
C. Decrease $7,000
D. Change by some amount other than those given.
Explanation:
Answer (A) is incorrect
An increase of $48,000 does not include the $25,000 of rental income
Answer (B) is correct.
In addition to the $15,000 purchase price the company would still incur $8,000 per unit of
unavoidable (fixed) manufacturing overhead (2/3 of $12,000). The materials handling charge of
20% of the purchase price of components would add another $3,000 per unit ($15,000×.2).
Therefore, the unit cost of purchase would be $26,000 ($15,000 + $8,000 + $3,000). Purchasing
would increase unit cost by $4,800 ($26,000 cost to purchase - $21,200cost to manufacture), an
increase of $48,000 per month (10 units x $4,800). However, the $25,000 of rental income would
reduce the increase in net cost to $23,000 per month
The cost to purchase equals the $15,000 unit purchase price, plus unavoidable (fixed)
manufacturing overhead (66 2/3 x $12,000= $8,000), plus the materials handling charge
($15,000 x 20% $3,000). The full cost to purchase is $26,000 ($15,000+ $8,000+ $3,000).
Answer (C) is incorrect
A decrease of $7,000 assumes an increase in unit cost of $1,800.
Answer (D) is incorrect.
The monthly cost would increase by $23,000.

Question #86: Basic time value of money concepts concern


A. Yes (Interest Factors), (Risk) Yes, (Cost of capital) No
B. Yes (Interest Factors), (Risk) No, (Cost of capital) Yes
C. No (Interest Factors), (Risk) Yes, (Cost of capital) No
D. No (Interest Factors), (Risk) No, (Cost of capital) Yes
Explanation:
Answer (A) is correct
The time value of money is concerned with two issues: (1) the investment value of money,
and (2) the risk (uncertainty) inherent in any executory agreement Thus, a dollar today is
worth more than a dollar in the future, and the longer we waits for a dollar, the more
uncertain the receipt is. The cost of capital involves a specific application of the time value of
money principle. It is not a basic concept thereof.
Answer (B) is incorrect
Risk is a basic time value of money concept Cost of capital is not
Answer (C) is incorrect
The interest effect is a basic time value of money concept.
Answer (D) is incorrect.
The interest effect and risk we basic time value of money concept. Cost of capital is not

Question #87: Communicating information related to risks is very important in enterprise risk
management. Which individual is most likely in the best position to recognize problems as they
arise related to customer product design needs?
A. Risk manager.
B. Sales representative.
C. Internal auditor.
D. Production manager.
Explanation:
Answer (A) is incorrect
The risk manager will more likely recognize these problems after a sale representative.
Answer (B) is correct.
Unlike the other individuals, sales representative interact with customers.
Answer (C) is incorrect
The internal auditor will most likely recognize these problems after a sales representative.
Answer (D) is incorrect.
The production manager will most likely recognize these problems after a sales
representative.

Question #88: With respect to the use of commercial paper by an industrial firm, which one
of the following statements is most likely to be true?
A. The commercial paper is issued through a bark
B. The commercial paper has a maturity of 60-270days.
C. The commercial paper is secured by the issuer’s asses
D. The commercial paper issuer is a small company.
Explanation:
Answer (A) is incorrect
Large corporations with high credit ratings, not banks, issue commercial paper.
Answer (B) is correct.
Most commercial paper has a maturity of between 60 and 270 days.
Answer (C) is incorrect
Commercial paper consists of unsecured notes payable issued in large denominations
($100,000or more).
Answer (D) is incorrect.
Commercial paper is issued by large companies with high credit ratings to other companies
and institutional investors.

Question #89: Question: Grimaldi's acid test ratio or quick ratio at the end of the year is
[Fact Pattern #17] Shown below ace beginning and ending balances for arum of Grimaldi.
Inc.’s accounts.
December
January 1 31
Cash $48,000 $62,000
Marketable securities 42,000 35,000
Accounts receivable (net) 68,000 47,000
Inventory 125,000 138,000
Property, plant, and equipment (net) 325,000 424,000
Accounts payable 32,000 84,000
Accrued liabilities 14,000 11,000
deferred txes 15,000 9,000
7% bonds payable 95,000 77,000

Grimaldi’s net income for the year was $96,000.


A. 0.83
B. 1.02
C. 1.15
D. 1.52
Explanation:
Answer (A) is incorrect
A ratio of 0.83 results from improperly including bonds payable in the denominator.
Answer (B) is incorrect
A ratio of 1.02 results from failing to include accounts receivable in the numerator.
Answer (C) is incorrect
A ratio of 1.15 results from failing to include marketable securities in the numerator.
Answer (D) is correct.
The acid test or quick ratio consists of the quick assets (cash, marketable securities, and
accounts receivable) divided by current liabilities. Grimaldi's quick ratio at year end is thus
1.516 [($62,000 + $35,000 + $47,000) ÷ ($84,000+ $11,000)].

Question #90: A company uses straight-line depreciation for both tax and financial reporting
purposes. The following data relate to Machine No. 108, which cost $400,000 and is being
written off over a 5-year life.
Operating Savings in Cash
Operating
Year Income costs
$150,00
1 0 $230,000
2 200,000 280,000
3 225,000 305,000
4 225,000 305,000
5 175,000 255,000

All of these amounts are on a before-tax basis. The company is subject to, 40% income tax
rate. The company strives for a 12% rate of return. The traditional payback period for
Machine No.108 would be .
A. 2.14years.
B. 2.44years.
C. 2.58 years.
D. 3.41years.
Explanation:
Answer (A) is correct
The after-tax payback for the firm 3 years would be calculated as follows:
Taxable
Income Tax Net cash flow
150,000 x 40% = 60,000 170,000
200,000 x 40% = 80,000 200,000
225,000 x 40% = 90,000 215,000
By the end of 2 years, the total recovery would be $370,000 ($170,000+ 200,000).
Subtracting $370,000 from the $400,000 initial cost leaves $30,000 to be recovered in Year
3. The $30,000 represents 14% of the Year 3 inflows of $215,000. Thus, the total payback
period would be 2.14 years.
Answer (B) is incorrect
The period of 2.44years used a 60% tax rate rather than 40%
Answer (C) is incorrect
The period of 2.58years calculated taxes on cash flows rather than on income.
Answer (D) is incorrect.
The period of 3.41years deducted tax payments from income rather than cash flows.

Question #91: Financial statements show the following information:


Accounts receivable, end of Year 1
$320,000
Credit sales for Year 2
3,600,000
Accounts receivable, end of Year 2
400,000
The accounts receivable turnover ratio is
A. 0.10
B. 9.00
C. 10.00
D. 11.25
Explanation:
Answer (A) is incorrect
A ratio of 0.1 results from reversing the numerator and denominator of the ratio.
Answer (B) is incorrect
A ratio of 9 times results from using ending rather than averages receivables in the
denominator.
Answer (C) is correct
The accounts receivable turnover ratio equals net o-edit sales dividend by average accounts
receivable. The accounts receivable turnover ratio is thus 10 times [$3,600,000 ÷
[($320,000+ $400,000) ÷2])
Answer (D) is incorrect.
A ratio of 11.25 times results from using beginning rather than average, receivables in the
denominator.

Question #92: Based on the fraud risk model, which of the following is the most likely
motive for employee theft?
A. Gambling losses.
B. Ineffective internal controls.
C. Belief that rank has its privileges
D. Ineffective supervision.
Explanation:
Answer (A) is correct
An employee's actual or perceived need for assets, for example, from gambling losses, can
create the motivation to commit fraud.
Answer (B) is incorrect
Ineffective internal controls create the opportunity to commit fraud.
Answer (C) is incorrect
The belief that rank has its privileges is a justification that some employees may use to justify
fraud.
Answer (D) is incorrect.
Ineffective supervision creates the opportunity to commit fraud.

Question #93: A farmer’s cooperative has a large amount of grain that it has gathered from
its members and has stored in silos. Prices for grain are high, but none of the cooperative's
customers is prepared to purchase any for the next 3 months. In order to hedge against an
unfavorable change in grain prices over the next 3 months, the cooperative will employ a
financial risk management technique known as a
A. Short hedge.
B. Long hedge
C. Naked option
D. Interest rate snap.
Explanation:
Answer (A) is correct
An extremely common form of financial risk management is called hedging. Hedging is the
process of using offsetting commitments to minimize or avoid the impact of adverse price
movements. A person who would like to sell an asset in the future has a long position in the
asset because (s)he benefits from a rise in value of the asset. To protect against a decline in
value the owner can enter into a short hedge, i.e., obtain an instrument whose value will rise
if the asset's value falls.
Answer (B) is incorrect
Along hedge is obtained by a party who fears a rise, not a fall, in the value of the underlying
asset
Answer (C) is incorrect
A naked option is one in which the seller of the option does not already possess the
underlying asset
Answer (D) is incorrect.
An interest rate swap is a hedging tool for parties who are attempting to smooth a flow of
interest payments, not lock in the price of a commodity.

Question #94: Fact Pattern: The information below pertains to Devlin Company.
Statement of Financial Position as of May 31 (in Thousands)

Assets Year 2 Year 1


Current Assets
Cash $ 45 $ 38
Trading securities 30 20
Accounts receivable (net) 68 48
Inventory 90 80
Prepaid expenses 22 30
Total current assets $255 $216
Investments, at equity 38 30
Property, plant, and equipment(net) 375 400
Intangible assets (net) 80 45
Total assets $748 $691
Liabilities
Current liabilities
Notes payable $ 35 $18
Accounts payable 70 42
Accounts expenses 5 4
Income taxes payable 15 16
total current liabilities $125 $80
long-term debt 35 35
deferred taxes 3 2
total liabilities $163 $117
Equity
Preferred stock, 6%, $100 par value,
cumulative $ 150 $ 150
common stock, $10 par value 225 195
Additional paid-in capital-common stock 114 100
Retained earnings 96 129
Total equity $585 $574

Income statement for the year ended May 31 (in thousands)

Year 2 Year 1
Net sales $ 480 $460
Costs and expenses
Costs of goods sold 330 315
Selling, general, and administrative 52 51
Interest expense 8 9
Income before taxes $90 $85
Income taxes 36 34
Net income $54 $51

Question: Devlin Company’s rate return on assets for the year ended May 31, Year 2, was
A. 7.2%
B. 7.5%
C. 7.8%
D. 11.3%
Explanation: Answer (A) is incorrect
The figure of 7.2% uses ending total assets instead of average total assets.
Answer (B) is correct.
The rate of return on assets equals net income divided by average total assets. Accordingly,
the rate of return is 7.5% {$54 ÷ [($748 + $691) ÷ 2).
Answer (C) is incorrect
Net income divided by beginning total assets equals 7.8%.
Answer (D) is incorrect. The return on sales is 11.3%.

Question #95: Which we of the following is a true statement regarding organizational ethics?

A. As long as officer and employee behavior meet the requirements of the law, the
organization can be considered to have a functioning system of ethical behavior.
B. A strong sense of ethics on the part of employees who are in the best position to
appropriate cash and other asset is the most vital part of a functioning system of ethical
behavior.
C. If an organization has a strong code of ethical conduct in oboe, the role of employee
training can be downplayed.
D. Paying attention to "whistleblowers” plays a significant role in maintaining an effective
ethical atmosphere.
Explanation:
Answer (A) is incorrect
A sense of ethics requires an ability to distinguish between ethical and merely legal behavior.
“Values and Ethics: From Inception to Practice” states, in part, “Many individuals at the center
of corporate scandals [of the late 20th and early 21th century] have professed the belief that they
were innocent of any wrongdoing, including Kenneth Lay of Enron or Conrad Black of Holinger.
The problem is that individuals did not define their behavior by what most of society would see
as reasonable, ‘but rather they followed their own particular code-in some cases limiting the
definition of ethical behavior to require compliance with the law and nothing more’ (II
Introduction.)
Answer (B) is incorrect
“Values and Ethics: From Inception to Practice” states. in part. ' Ethical behavior is not
something that applies to someone else-every single individual is responsible for behaving
ethically. Nowhere is this more important than the demonstration of ethical behavior that
managers and supervisors exhibit in the way they execute their day-to-day work” this
phenomenon is referred to as the “tone at the top” (VI. Leadership by example.)
Answer (C) is incorrect
Employee training is important to maintain an ethical organizational culture. “Values and Ethics:
From inception to practice” states, in part, “every existing member of staff should receive
ongoing training, starting at the board level and cascading down throughout the organization.
Ethics training for employees should focus on covering ethical concepts, the organization’s code,
and compliance. To achieve this, training should include: ethical concepts and thinking: What is
‘behind’ the issue of ethical action? And the organization’s code of ethics and any supporting
‘rules’(VIII. Practical Application: converting Intent into Operational reality.)
Answer (D) is correct.
“Values and Ethics: From Inception to Practice” states, in part, “A whistleblowing framework
(e.g., an ethics helpline) is an important component in maintaining an ethical organizational
culture. An effective feedback system includes having a confidential framework for employees
to report possible violations of the organization's code of ethics and to receive advice on the
ethical aspects of challenging decisions. Statistics show that a large number of occupational
fraud cases are detected through an employee “hotline” or other reporting method. “(IX.
Measuring and Improving Ethical Compliance.)

Question #96: Which of the following Isamu is addressed by Section 406 of the Sarbanes-Oxley
Act?
I. Full, fair, timely and accurate financial statement disclosure.
II. Whistleblower protection.
III. Form 8-K disclosure of changes to the Ethics Code for Senior Financial Officers.
IV. Compliance with the US. Foreign Corrupt Practices Act.
V. Reporting the existence of an Ethics Code for Senior Financial Officers.
A. I, II, III and IV only.
B. II, IV and V only.
C. I, III and V only.
D. III and IV only.
Explanation:
Answer (A) is incorrect
Whistleblower protection and compliance with the U.S. Foreign Corrupt Practices Act are
not addressed by Section 406 of the Sarbanes-Oxley Act Whistleblower hotlines are required,
but there are no protections offered to the users of those hotlines.
Answer (B) is incorrect
Whistleblower protection and compliance with the U.S. Foreign Corrupt Practices Act are
not addressed by Section 406 of the Sarbanes-Oxley Act Whistleblower hotlines are required,
but there are no protections offered to the users of those hotlines.
Answer (C) is correct
Section 406 ci the Sarbanes-Oxley Act addresses full, fair, timely, and accurate financial
statement disclosure: Form 8-K disclosure of changes to the Ethic Code for Senior Financial
Officers; and reporting the existence of an Ethics Code for Senior Financial Officers.
Answer (D) is incorrect.
Compliance with the US. Foreign Corrupt Practices Act is not addressed by Section 406 of
the Sarbanes Oxley Act.
Question #97: For a firm with a degree of operating leverage of 3.5, an increase in sales of 6%
will
A. Increase pre-tax profits by 3.5%.
B. Decrease pre-tax profits by 3.3%.
C. Increase pre-tax profits by 21%.
D. Increase pre-tax profits by 1.71%.
Explanation:
Answer (A) is incorrect
The 3.5 is multiplied times the increase in sales.
Answer (B) is incorrect
A firm with operating leverage will experience a rise in pre-tax profits when sales increase.
Answer (C) is correct
A degree of operating leverage (DOL) of 3.5 means that operating income (EBIT) will
increase 3.5 times greater than any sales increase. Multiplying 3.5 times the 6% sales
increase results in a pre-tax profit increase of 21%.
Answer (D) is incorrect.
The DOL is multiplied by the sales change, not divided into it.

Question #98: A corporation's return on equity can be calculated if you know its
A. Sustainable equity growth rate and dividend payout ratio.
B. Debt-equity ratio and market-to-book ratio.
C. Market-to-book ratio and equity multiplier.
D. Dividend yield and earnings yield.
Explanation:
Answer (A) is correct
The sustainable equity growth rate can be found by multiplying return on equity by 1 minus
the dividend payout ratio. Thus, the return on equity can be derived given the sustainable
growth rate and the dividend payout ratio.
Answer (B) is incorrect
The market-to-book ratio cannot be used to calculate return on equity.
Answer (C) is incorrect
The equity multiplier and return on assets could calculate return on equity.
Answer (D) is incorrect.
These numbers would not provide the information needed to find the net income or average
total equity.

Question #99: The technique used to evaluate all possible capital projects of different dollar
amounts and then rank them according to their desirability is the
A. Profitability index method.
B. Net present value method.
C. Payback method.
D. Discounted cash flow method.
Explanation:
Answer (A) is correct
The profitability index (excess present value index) of an investment is the ratio of the
present value of the future net cash flow (or only cash inflows) to the net initial investment:
that is, the figures are those used to calculate the net present value (NPV), but the numbers
are divided rather than subtracted. This variation of the NPV method facilitates comparison
of different-sized investments. It provides an optimal ranking in the absence of capital
rationing.
Answer (B) is incorrect
The net present value method does not provide a return per dollar invested and is therefore
not as effective as the profitability index in the absence of capital rationing.
Answer (C) is incorrect
The payback method give no consideration to the time value of money or to returns after the
payback period.
Answer (D) is incorrect.
The profitability index method and the NPV method are discounted cash flow methods.
However the profitability index method is the variant that purports to calculate a return per
dollar of investment.

Question #100: When choosing a merger over an acquisition of stock to accomplish a


business combination, which of the following is irrelevant to the decision?
A. Dealing directly with shareholders in an acquisition of stock
B. Absence of tender by some minority shareholders in a tender offer.
C. Resistance to an acquisition by the target's management usually causing an increase in the
stack price
D. Whether the companies are in the same industry.
Explanation:
Answer (A) is incorrect
It supports choosing a merger over an acquisition.
Answer (B) is incorrect
It supports choosing a merger over an acquisition.
Answer (C) is incorrect
It supports choosing a merger over an acquisition.
Answer (D) is correct.
Many factors influence whether a transaction should be a merger or an acquisition of stock.
Whether the companies are in the same industry or not is usually not a factor. In an
acquisition of stock an acquiring firm usually makes a tender offer directly to the
shareholders of another firm to obtain a controlling interest. Therefore, the acquiring firm
must directly deal with shareholders of the other firm. There is the possibility that some
minority shareholders will not tender their shares. Management may be hostile to the
combination, which usually cause an increase in the stock price. This increase will require
the acquiring firm to pay more money in its tender offer. On the other hand, a merger is much
more straightforward legally. It is usually a negotiated arrangement between a single bidder
and the acquired firm. However, a merger does require a formal vote of the shareholders of
each of the merging firms, whereas an acquisition does not.
MOCK TEST 2

Question #1: According to the IMA Statement of Ethical Professional Practice, one of the
overarching ethical principles is "Objectivity." "Objectivity" means
A. Doing what you say you will do when you say you will do it. •
B. Basing a judgment on an established set of criteria. •
C. Acting in an impartial manner and being free from dishonesty or injustice. •
D. Fairness and straightforwardness of conduct.
Correct answer: B. basing a judgment on an established set of criteria.
Explanation:
A. This is the definition of "Responsibility.”
B. "Objectivity" means basing a judgment on an established set of criteria. It is the state of being
free from personal feelings or prejudice and basing analyses and decisions on the facts alone.
C. This is the definition of "Fairness”
D. This is the definition of "Honesty."

Question #2: Accounts receivable turnover will normally decrease as a result of


A. A significant sales volume decrease near the end of the accounting period. •
B. A change in credit policy to lengthen the period for cash discounts. •
C. An increase in cash sales in proportion to credit sales.
D. The write-off of an uncollectible account (assume the use of the allowance for doubtful
accounts method).
Correct answer: B. A change in credit policy to lengthen the period for cash discounts.
Explanation:
Accounts receivable turnover is calculated as annual credit sales divided by the average accounts
receivable. The A/R turnover will decrease if there is a decrease in credit sales or an increase in
the average receivables or if both decrease but the decrease in credit sales is proportionally
greater than the decrease in average receivables.
A decrease in credit sales at the end of the period will decrease both the credit sales and the
receivables balance at the end of the period. Because annual credit sales is a larger amount than
average accounts receivable, the amount of decrease in sales will not be proportionally greater
than the amount of the decrease in average accounts receivable. Therefore, the accounts
receivable turnover will not normally decrease as a result of a significant sales volume decrease
near the end of the accounting period.
B. Accounts receivable turnover is calculated as annual credit sales divided by the average
accounts receivable. It measures the number of times the accounts receivable "turn over" during
a year's time. If the company lengthens the period for cash discounts, more customers will take
longer to pay their bills, which will increase the average receivables. This will, in turn, decrease
the accounts receivable turnover ratio. A decrease in the accounts receivable turnover ratio
means the accounts turn over less frequently; and in this case, that is because the level of
accounts receivable is higher.
C. Accounts receivable turnover is calculated as annual credit sales divided by the average
accounts receivable. Therefore, this number will decrease if there is a decrease in credit sales or
an increase in the average receivables. An increase in cash sales in proportion to credit sales may
or may not cause a decrease in the accounts receivable turnover ratio. It would depend upon the
magnitude of the change in both annual credit sales and average accounts receivable.
D. Accounts receivable turnover is calculated as annual credit sales divided by the average
accounts receivable. Therefore, this number will decrease if there is a decrease in credit sales or
an increase in the average receivables. Under the allowance for doubtful accounts method, the
writing off of a receivable will not effect the accounts receivable turnover, because the net
average accounts receivable is used in the accounts receivable turnover ratio (i.e., net of the
allowance for doubtful accounts). The net accounts receivable balance does not change when an
account is written off. Accounts receivable decreases, but the negative (credit) balance in the
allowance for doubtful accounts also decreases. The effect on the combination of the two
account balances is zero.

Question #3: If income tax considerations are ignored, how is depreciation handled by the
following capital budgeting techniques?

Internal Accounting
Rate of return Rate of return Payback
I Excluded Included Excluded
II Excluded Excluded Included
II Included Excluded Included
IV Included Included Included

A. IV.
B. I.
C. II.
D. III.
Correct answer: B. I.

Explanation: A. If income tax considerations are ignored, depreciation would be excluded from
the internal rate of return and payback calculations, because the IRR and the payback period are
based upon cash flows.
B. If income tax considerations are to be ignored, then the depreciation tax shield is ignored.
Therefore, the income tax savings from the depreciation are not included in the capital budgeting
analyses. If the income tax savings from the depreciation are excluded, then depreciation is
ignored in the calculations of internal rate of return and payback. However, depreciation is
included in the calculation of the accounting rate of return, because the accounting rate of return
is based upon book income, which includes depreciation.
C. If income tax considerations are ignored, depreciation would be included in the accounting
rate of return calculation, because the accounting rate of return is based upon book income,
which includes depreciation. Furthermore, depreciation would be excluded from the payback
period calculation, because the payback period calculation is based upon cash flows, not book
income.
D. If income tax considerations are ignored, depreciation would be excluded from the internal
rate of return and payback calculations, because the IRR and the payback period are based upon
cash flows. Depreciation would be included in the calculation of the accounting rate of return,
because the accounting rate of return is based upon book income, which includes depreciation.

Question #4: A company had total sales of $500,000 in the first quarter of the year, which was
the same amount as it recorded in the first quarter of the prior year. However, its accounts
receivable balance increased from $230,000 last year to $300,000 this year. Which one of the
following is the most likely explanation for the increase in the accounts receivable balance?
A. The company discontinued the use of factoring in the current year.
B. The company hired more people in its credit and collections department.
C. The company initiated the use of factoring in the current year.
D. The company shortened its payment terms in the current year from 60 days to 30 days.
Correct answer: A. The Company discontinued the use of factoring in the current year.
Explanation:
A. Discontinuing the use of factoring would cause the accounts receivable balance to increase
because the company would be carrying more of its own accounts receivable on its balance sheet
instead of selling the receivables.
B. Hiring more people in its credit and collections department would not cause the company's
accounts receivable to increase.
C. If the company had initiated the use of factoring in the current year, the accounts receivable
balance would be expected to decline, not increase.
D. Shortening its payment terms would not cause the company's accounts receivable to increase.

Question #5: A company uses cost-volume-profit analysis to evaluate a new product. The total
fixed costs of production per year are $160,000. The unit variable cost is $50. Which one of the
following combinations of unit selling price and breakeven number of units sold per year is
correct?
A. $100 selling price and 1,600 breakeven number of units.
B. $50 selling price and 3,200 breakeven number of units.
C. $25 selling price and 6,400 breakeven number of units.
D. $70 selling price and 8,000 breakeven number of units.
Correct answer: D. $70 selling price and 8,000 breakeven number of units.
Explanation:
A. This answer results from mistaking the contribution margin for the selling price. If the
contribution margin were $100, the breakeven number of units given $160.000 in fixed
production costs would be $160,000 ÷ $100 = 1,600 units. However, the question is asking for
the selling price and the breakeven number of units, not the contribution margin and the
breakeven number of units. The selling price is the contribution margin plus the unit variable
cost of $50; and the contribution margin is the selling price minus the unit variable cost of $50.
The breakeven number of units at a $100 selling price would be $160,000 divided by the
contribution margin of ($100 - $50), which equals 3,200 units. So this is not a true statement.
B. This answer results from mistaking the contribution margin for the selling price. If the
contribution margin were $50, the breakeven number of units given $160,000 in fixed production
costs would be $160.000 ÷ $50 = 3,200 units. However, the question is asking for the selling
price and the breakeven number of units, not the contribution margin and the breakeven number
of units. The selling price is the contribution margin plus the unit variable cost of $50; and the
contribution margin is the selling price minus the unit variable cost of $50. The breakeven
number of units at a $50 selling price does not exist. The unit contribution margin would be $50
- $50, or zero. There would be no contribution margin available to cover the fixed production
costs and no volume at which the company could break even.
C. This answer results from mistaking the contribution margin for the selling price. If the
contribution margin were $25, the breakeven number of units given $160,000 in fixed production
costs would be $160,000 ÷ $25 = 6,400 units. However, the question is asking for the selling
price and the breakeven number of units, not the contribution margin and the breakeven number
of units. The selling price is the contribution margin plus the unit variable cost of $50; and the
contribution margin is the selling price minus the unit variable cost of $50. The breakeven
number of units at a $25 selling price does not exist. The unit contribution margin would be $25
- $50, or a $25 loss on each unit sold. There would be no contribution margin available to cover
the fixed production costs and no volume at which the company could break even.
D. This is a true statement. With a selling price of $70, the contribution margin would be $70 -
$50 = $20. With fixed production costs of $160,000, the breakeven point would be $160,000 ÷
$20 = 8,000 units.

Question #6: A printing company is considering replacing an old printing press. The old printing
press has a book value of $24,000 and a trade-in value of $14,000. A new printing press would
cost $85,000 after trade-in of the old press. It is estimated that the new printing press would
reduce operating costs by $20,000 per year. If the company decides not to purchase the new
press, the $85,000 could instead be used to retire debt that is currently costing $9,000 per year in
interest. Which of the given amounts is an example of a sunk cost?
A. The trade-in value of the old printing press.
B. The estimated reduction in operating costs.
C. The interest on the existing debt
D. The book value of the old printing press.
Correct answer: D. The book value of the old printing press.
Explanation:
A. A sunk cost is a cost that has already been incurred and is not relevant to the decision process.
The trade-in value of the old printing press will affect the decision process, so therefore, is
relevant and is not a sunk cost.
B. A sunk cost is a cost that has already been incurred and is not relevant to the decision process.
The estimated reduction in operating expenses is the difference between keeping the old printing
press and purchasing the new one. It is relevant to the decision process and is not even a cost.
C. The interest on debt is an avoidable cost that is relevant to the decision process.
D. A sunk cost is a cost that has already been incurred and is not relevant to the decision process.
The amount paid for the old printing press (less accumulated depreciation) is an example of a
sunk cost.

Question #7: The degree of operating leverage (DOL) is


A. A measure of the change in earnings before interest and taxes (EBIT) resulting from a
given change in sales.
B. A measure of the change in earnings available to common stockholders associated with a
given change in operating earnings.
C. Higher if the degree of total leverage is lower, other things held constant.
D. Lower if the degree of total leverage is higher, other things held constant.
Correct answer: A. A measure of the change in earnings before interest and taxes (EBIT)
resulting from a given change in sales.

Explanation:
A. The degree of operating leverage (DOL) is a measure of the change in earnings before interest
and taxes associated with a given change in sales volume. For a particular level of output. Degree
of Operating Leverage is calculated as follows: % Change in EBIT % Change in Revenue
B. The degree of financial leverage is a measure of the change in earnings available to common
stockholders associated with a given change in operating earnings.
C. The degree of total leverage is the multiple of the degree of operating leverage and the degree
of financial leverage. Other things being equal. DOL is higher if the degree of total leverage is
higher.
D. The degree of total leverage is the multiple of the degree of operating leverage and the degree
of financial leverage. Other things being equal. DO L is higher if the degree of total leverage is
higher.

Question #8: Based on the assumptions of the Capital Asset Pricing Model, the risk premium on
an investment with a beta of 0.5 is equal to
A. Twice the risk premium on the market.
B. Half the risk premium on the market.
C. The risk-free rate.
D. The risk premium on the market.
Correct answer: B. half the risk premium on the market.
Explanation:
A. The risk premium for a particular security is its β (RM - RF), or the beta coefficient for that
particular security multiplied by the market risk premium. The beta coefficient for the market as
a whole is 1.0. If the beta coefficient for a given security is 0.5, the risk premium for that security
cannot be twice the risk premium for the market as a whole.
B. The risk premium for a particular security is its β (RM - RF), or the beta coefficient for that
particular security multiplied by the market risk premium. The beta coefficient for the market as
a whole is 1.0. If the beta coefficient for a given security is 0.5, it means the risk premium for
that security is half the risk premium for the market as a whole.
C. The risk premium on an investment with a beta of 0.5 cannot be equal to the risk-free rate, as
the risk-free rate is only one component in the calculation of the risk premium for an investment.
The risk premium for a particular security is its β (RM - RF), or the beta coefficient for that
particular security multiplied by the market risk premium.
D. The risk premium on an investment with a beta of 0.5 cannot be equal to the risk premium on
the market, as the risk premium on the market is a component in the calculation of the risk
premium for an investment. The risk premium for a particular security is its β (RM - RF), or the
beta coefficient for that particular security multiplied by the market risk premium.

Question #9: Douglas Company purchased 10.000 shares of its common stock at the beginning
of the year for cash. This transaction will affect all of the following except the
A. Net profit margin.
B. Debt-to-equity ratio.
C. Current ratio.
D. Earnings per share.
Correct answer: A. net profit margin
Explanation:
A. When a company repurchases shares of its own outstanding stock, the stock that is
repurchased is called treasury stock. Purchase of treasury stock reduces cash and equity on the
balance sheet. It does not affect any items on the income statement and thus, net profit margin is
not affected.
B. When a company repurchases shares of its own outstanding stock, the stock that is
repurchased is called treasury stock and its purchase is recorded as a reduction in equity. Since
equity is reduced, the debt-to-equity ratio will be increased.
C. When a company repurchases shares of its own outstanding stock, the stock that is
repurchased is called treasury stock. The purchase reduces cash, which in turn reduces current
assets and the current ratio.
D. When a company repurchases shares of its own outstanding stock, the stock that is
repurchased is called treasury stock. The repurchase reduces the number of outstanding shares of
the company's common stock. Since the number of outstanding shares of common stock is
reduced, earnings per share will be increased.

Question #10: Please see the Question Below:


Gordon has had the following financial results for the last four years.
Year 1 Year 2 Year 3 Year 4
Sales $1,250,000 $1,300,000 $1,359,000 $1,400,000
Cost of goods sold 750,000 785,000 825,000 850,000
Gross profit $500,000 $515,000 $534,000 $550,000

Inflation factor 1.00 1.03 1.07 1.10

Gordon has analyzed these results using vertical common-size analysis to determine trends. The
performance of Gordon can best be characterized by which one of the following statements?
A. The common-size gross profit percentage has decreased as a result of an increasing
common-size trend in cost of goods sold.
B. The common-size trend in cost of goods sold is decreasing which is resulting in an
increasing trend in the common-size gross profit margin.
C. The increased trend in the common-size gross profit percentage is the result of both the
increasing trend in sales and the decreasing trend in cost of goods sold.
D. The common-size trend in sales is increasing and is resulting in an increasing trend in the
common-size gross profit margin.
Correct answer: A. The common-size gross profit percentage has decreased as a result of an
increasing common-size trend in cost of goods sold.

Explanation:
To solve this, we need to calculate the percentage of sales represented by COGS and Gross Profit
for each of the years and then analyze the trends. Year 1 Year 2 Year 3 Year 4 Sales$1,250,000
100.0% $1,300,000 100.0% $1,359,000 100.0% $1,400,000 100.0% COGS 750,000 60.0%
785,000 60.4% 825,000 60.7% 850,000 60.7% Gr. profit$ 500,000 40.0% $ 515,000 39.6% $
534,000 39.3% $ 550,000 39.3%
When we look at the trend, it is easy to see that the common-size gross profit percentage has
decreased as a result of an increasing common-size trend in cost of goods sold. None of the other
answers is a true statement.
B. The trend in cost of goods sold is not decreasing.
C. The gross profit percentage is not increasing.
D. Although the gross profit is increasing, the gross profit margin is not increasing.

Question #11: When using the net present value method for capital budgeting analysis, the
required rate of return is called all of the following except the
A. Risk-free rate.
B. Cost of capital.
C. Hurdle rate.
D. Discount rate.
Correct answer: A. Risk free rate.
Explanation:
A. The required rate of return, which is the rate used to discount future cash flows in a capital
budgeting analysis, is not the risk-free rate. There is risk inherent in all capital budgeting
projects, and the required rate of return incorporates a risk premium.
B. The required rate of return may be equal to the firm's cost of capital, if the firm has not seen
fit to adjust its cost of capital to reflect higher or lower risk.
C. The required rate of return may be called the "hurdle rate" because it is the minimum rate of
return that is acceptable for an investment. A firm should invest money in a project only if the
project provides a higher rate of return than this rate. Investments with a return higher than the
hurdle rate will increase the value of the firm and thus stockholders' wealth.
D. The required rate of return is the discount rate used in a capital budgeting analysis.

Question #12: Which of the following represents a firm's average gross receivables balance?
I. Days' sales in receivables x accounts receivable turnover.
II. Average daily credit sales x average collection period.
III. Net sales + average gross receivables.
A. II only.
B. II and III only.
C. I only.
D. I and II only.
Correct answer: A. II only.
Explanation:
A. This is one of the ways in which the average receivables balance may be calculated.
Multiplying the average credit sales each day by the average number of days receivables
are outstanding provides the average receivables balance.
B. This answer cannot be correct because item III includes the value that we are trying to
solve for.
C. Neither of these items are dollar amounts so they cannot together calculate the average
receivables balance.
D. Neither of the items in item I are dollar amounts so they cannot together calculate the
average receivables balance.
Question #13: Essential elements in the development of an organization's ethics policy include
all of the following except
A. Input from the board of directors in addition to management and employees.
B. Allowances for exceptional circumstances.
C. Relevance to day-to-day implementation.
D. Articulation of organizational values.
Correct answer: B. Allowances for exceptional circumstances.
Explanation:
A. Development of the ethics policy should have input from the board of directors because the
ultimate responsibility for ethical conduct lies with the board of directors. The board of directors
should oversee the development of the policy. Input should also be received from a cross-
functional group. In an international organization and where size permits, the group should
include international personnel. Groups of employees and others can help identify risks and help
in defining and reviewing the content of the policy.
B. Allowances for exceptional circumstances are not an essential element in the development of
an organization's ethics policy.
C. The ethics policy should have relevance to day-to-day implementation. While an ethics policy
cannot specifically cover every potential ethical dilemma, it should utilize the organization's
values and guidelines as the basis for ethical decision-making on a day-to-day basis.
D. An organization's values are at the heart of its ethics policy. The ethics policy should be
values-driven, so it should include articulation of organizational values.

Question #14: A 10% stock dividend most likely


A. Increases the size of the firm.
B. Increases shareholders' wealth.
C. Decreases future earnings per share.
D. Decreases net income.
Correct answer: C. Decreases future earnings per share
Explanation:
A. A stock dividend does not affect the size of the company.
B. A stock dividend does not affect the shareholders' wealth since there is no distribution of
assets of the company and each individual shareholders market value of their investment remains
unchanged after the stock dividend.
C.A stock dividend will increase the number of shares outstanding and this in turn will most
likely decrease earnings per share in the future since the profits of the company will need to be
divided among more shares.
D. A stock dividend does not impact the income of the company. It is accounted for entirely
within the owners' equity section of the balance sheet.

Question #15: Hobart Corporation evaluates capital projects using a variety of performance
screens; including a hurdle rate of 16%, payback period of 3 years or less, and an accounting rate
of return of 20% or more. Management is completing review of a project on the basis of the
following projections.
$200,00
Capital investment 0
Annual cash flows $74,000
Straight-line
depreciation 5 years
Terminal value $20,000

The projected internal rate of return is 20%. Which one of the following alternatives reflects the
appropriate conclusions for the indicated evaluative measures?
Internal rate of Paybac
return k
I Accept Reject
II Reject Reject
III Accept Accept
IV Reject Accept

A. IV
B. II
C. I
D. III
Correct answer: D. III
Explanation:
A. The IRR of 20% is higher than the hurdle rate of 16%, so on the basis of internal rate of
return, the project should be accepted. Please see the correct answer for a complete explanation.
B. This project is acceptable based on both its internal rate of return and its payback period.
Please see the correct answer for a complete explanation.
C. Since all of the annual cash flows are the same, the payback period can be calculated by
simply dividing the capital investment amount of $200,000 by the annual cash flow amount of
$74,000. The result is 2.7 years, which meets the required payback period of 3 years or less.
Therefore, the project is acceptable based on its payback period. Please see the correct answer
for a complete explanation.
D. This project is acceptable based on both its internal rate of return and its payback period. The
I RR of 20% is higher than the hurdle rate of 16%, so on the basis of internal rate of return, the
project should be accepted.
Since all of the annual cash flows are the same, the payback period can be calculated by simply
dividing this project is acceptable based on both its internal rate of return and its payback period.
The IRR of 20% is higher than the hurdle rate of 16%, so on the basis of internal rate of return,
the project should be accepted. Since all of the annual cash flows are the same, the payback
period can be calculated by simply dividing the

Question #16: Which of the following actions is not an appropriate course of action when
encountering problems identifying unethical behavior or resolving an ethical conflict, if your
organization's established policies do not resolve the ethical conflict?
A. Communicate your concerns to the authorities or to persons outside the organization who
may be interested parties.
B. Discuss the issue with your immediate supervisor except when it appears that the
supervisor is involved. In that case, present the issue to the next level. If you cannot
achieve a satisfactory resolution, submit the issue to the next management level.
C. Consult your own attorney regarding your legal obligations and rights concerning the
ethical conflict.
D. Clarify relevant ethical issues by initiating a confidential discussion with an IMA Ethics
Counselor or other impartial advisor to obtain a better understanding of possible courses
of action.
Correct answer: A. Communicate your concerns to the authorities or to persons outside the
organization who may be interested parties.
Explanation:
A. Communication of problems to authorities or individuals not employed or engaged by the
organization is not considered an appropriate means of resolving an ethical conflict, unless you
believe there is a clear violation of the law.
B. Discussing the issue with your immediate supervisor except when it appears that the
supervisor is involved is a course of action to consider.
C. Consulting your own attorney regarding your legal obligations and rights concerning the
ethical conflict is a course of action to consider.
D. Initiating a confidential discussion with an IMA Ethics Counselor or other impartial advisor is
a course of action to consider.

Question #17: A change from one generally accepted accounting principle to another generally
accepted accounting principle should be accounted for in comparative reports by
A. A line item below extraordinary items on the current income statement
B. Pro forma amounts for key figures shown supplementary on the income statement for all
periods presented.
C. A cumulative adjustment to carrying amounts of assets and liabilities as of the beginning
of the first period presented, an offsetting adjustment to the opening balance of retained
earnings of the same period, and by adjusting prior periods' statements presented for the
effects of the change in each period.
D. Only a footnote disclosure in the current period.
Correct answer: A cumulative adjustment to carrying amounts of assets and liabilities as of the
beginning of the first period presented, an offsetting adjustment to the opening balance of
retained earnings of the same period, and by adjusting prior periods' statements presented for the
effects of the change in each period.

Explanation:
A. No line item for any cumulative effect adjustment is to be used on the current or any prior
income statement.
B. Pro forma amounts are not to be used for reporting accounting changes.
C. A change of accounting principle is to be accounted for by adjusting carrying amounts of
assets and liabilities as of the beginning of the first period presented for the cumulative effect of
the change on periods prior to those presented in the financial statements. The effect of this prior
period adjustment is offset by adjusting the opening balance of retained earnings of the first
period presented. Financial statements for all periods presented are adjusted for the effects of the
change in each specific period, unless it is impracticable to do so.
D. The financial statements in the period the change is made must include disclosure of the
change, the reason for the change, and an explanation of why the company's management
believes the new accounting principle to be preferable from the perspective of financial reporting
(i.e.. not merely because it will result in favorable income tax consequences). However, footnote
disclosure is not the only requirement.

Question #18: Jones & Company is considering the acquisition of scanning equipment to
mechanize its procurement process. The equipment will require extensive testing and debugging,
as well as user training prior to its operational use. Projected after-tax cash flows are as follows.
Time period After-tax Cash
Year Inflow/(Outflow)
0 $(600,000)
1 $(500,000)
2 $450,000
3 $450,000
4 $350,000
5 $250,000

Management anticipates the equipment will be sold at the beginning of year 6 for $50.000 and its
book value is zero. Jones' internal hurdle and effective income tax rates are 14% and 40%,
respectively. Based on this information, a negative net present value was computed for the
project. Accordingly, it can be concluded that
A. The project has an internal rate of return (IRR) less than 14% since IRR is the interest
rate at which net present value is equal to zero.
B. Jones should examine the determinants of its hurdle rate further before analyzing any
other potential projects.
C. Jones should calculate the project payback to determine if it is consistent with the net
present value calculation.
D. The project has an I RR greater than 14% since IRR is the interest rate at which net
present value is equal to zero.
Correct answer: The project has an internal rate of return (IRR) less than 14% since IRR is
the interest rate at which net present value is equal to zero.

Explanation:
A. The IRR is the discount rate at which the net present value of the project is zero. If the
project's net present value computed using the company's hurdle rate of 14% is negative,
its I RR must be lower than 14%. Any project with a net present value of less than zero or
an IRR of less than the hurdle rate is not an acceptable project.
B. The purpose of the hurdle rate is to establish the minimum return the company expects
from a project. The fact that a project is unacceptable when that hurdle rate is used to
discount a project's future cash flows is not an indication that there is anything wrong
with the hurdle rate used. It is more likely an indication that the project does not meet the
company's requirements. Changing the company's requirements to make the project
acceptable is generally not the proper course of action.
C. The payback period method of capital budgeting should not be used to confirm a net
present value calculation for a project. The payback method can be helpful when the
company needs to recoup its investment quickly, perhaps because the project is in a
politically unstable area of the world or because it utilizes high technology that quickly
becomes obsolete. However, its usefulness is limited because it does not incorporate the
time value of money and it ignores the cost of capital and all cash flows beyond the
payback point.
D. The IRR is the discount rate at which the net present value of the project is zero. If the
project's net present value computed using the company's hurdle rate of 14% is negative,
its IRR must be lower than 14%. Any project with a net present value of less than zero or
an IRR of less than the hurdle rate is not an acceptable project.

Question #19: Which of the following methods can be used as a tool to identify process controls
related to ethical or behavioral issues?
A. Ratio analysis
B. Decision trees
C. Business process re-engineering
D. Sensitivity analysis
Correct answer: C. business process re-engineering.
Explanation:
A. This is not correct because ratio analysis is a financial analysis technique.
B. This is not correct because a decision tree is a method to determine the best course of
action in a decision situation based upon a range of possible courses of action, their
expected outcomes and their probabilities of occurring.
C. This is the correct answer because business process engineering involves understanding a
process and identifying activities in the process, including internal controls, which impact
ethical behavior.
D. This is not correct because sensitivity analysis is a technique to analyze how the outcome
of a decision would be impacted by changes in the variables that lead to the outcome.

Question #20: Allred Company sells its single product for $30 per unit. The contribution margin
ratio is 45%, and fixed costs are $10,000 per month. Allred has an effective income tax rate of
40%. If all read sells 1.000 units in the current month. Allred's variable expenses would be
A. $13,500.
B. $16,500.
C. $12,000.
D. $9,900.
Correct answer: $16,500.
Explanation:
A. This is the contribution margin, not the variable costs. Revenue minus variable costs
equals the contribution margin. Therefore, revenue minus the contribution margin would
equal variable costs.
B. Revenue for sales of 1,000 units would be $30 x 1,000, or $30,000. The contribution
margin is total revenue minus total variable costs. If the contribution margin ratio is 45%
of revenue, the variable costs must be 100% - 45%, or 55% of revenue. That is $30,000 x
0.55, or $16,500.
C. This is total revenue multiplied by the tax rate. That is not variable expenses.
D. This is the variable costs multiplied by (1- the tax rate). That is not variable expenses.

Question #21: Northville Products is changing its credit terms from net 30 to 2/10, net 30. The
least likely effect of this change would be a (n)
A. Increase in sales.
B. Lower number of days sales outstanding.
C. Increase in short-term borrowings.
D. Shortening of the cash conversion cycle.
Correct answer: C. increase in short-term borrowings
Explanation:
A. An increase in sales is not the least likely effect of this change in credit terms. It is
possible that some additional sales would be made to customers who would be planning
to take advantage of the 2% discount by paying within 10 days. Therefore, an increase in
sales could be a likely effect.
B. A lower number of days sales outstanding is not the least likely effect of this change in
credit terms. Receivables will be collected more quickly because customers will have an
incentive to pay within the 10 days to receive a 2% discount. The number of days sales
outstanding will likely decrease as a result.
C. An increase in short-term borrowing is the least likely effect of this change in credit
terms. Northville's short-term borrowings will probably decrease, because receivables
will be collected more quickly. Customers will have an incentive to pay within the 10
days to receive a 2% discount.
D. Shortening of the cash conversion cycle is not the least likely effect of this change in
credit terms. A shorter cash conversion cycle is a likely effect, because receivables will
be collected more quickly. Customers will have an incentive to pay within the 10 days to
receive a 2% discount.

Question #22: Gleason Co. has two products, a frozen dessert and ready-to-bake breakfast rolls,
ready for introduction. However, plant capacity is limited, and only one product can be
introduced at present Therefore, Gleason has conducted a market study, at a cost of $26,000, to
determine which product will be more profitable. The results of the study follow.

Sales of desserts at $1.80/unit Sales of rolls at $1.20/unit


Volume Probability Volume Probability
250,000 0.30 200,000 0.20
300,000 0.40 250,000 0.50
350,000 0.20 300,000 0.20
400,000 1.10 350,000 0.10

The costs associated with the two products have been estimated by Gleason's cost accounting
department and are shown as follows.

Dessert Rolls
Ingredients per unit $0.40 $0.25
Direct labor per unit 0.35 0.30
Variable overhead per unit 0.40 0.20
Production tooling* 48,000 25,000
Advertising 30,000 20,000

*Gleason treats production tooling as a current operating expense rather than capitalizing it as a
fixed asset.
In order to recover the costs of production tooling and advertising for the breakfast rolls,
Gleason's sales of the breakfast rolls would have to be
A. 60,000 units.
B. 100,000 units.
C. 37,500 units.
D. Some amount other than those given.
Correct answer:
Explanation:
A. This answer results from dividing fixed costs (advertising and production tooling cost) by
the variable cost per unit $45,000 / ($0.25 + $0.30 + $0.20)]. The formula for the
breakeven volume is Total Fixed Costs / Unit Contribution Margin.
B. The formula for the breakeven volume is Total Fixed Costs/ Unit Contribution Margin.
The total fixed cost is $45,000 ($25,000 + $20,000). The unit contribution margin is
$0.45 ($1.20 - $0.75). $45,000 / $0.45 = 100,000 units that will need to be sold to break
even.
C. This answer results from dividing the total fixed cost of $45,000 by the sale price of
$1.20. The formula for the breakeven volume is Total Fixed Costs / Unit Contribution
Margin.
D. The correct answer is one of the answer choices given.

Question #23: The basis for the components of an Enterprise Risk Management system as
defined by COSO in Enterprise Risk Management - Integrated Framework is
A. The organization's control activities.
B. The internal environment of the organization.
C. Assessment of risk.
D. Identification of the organization's strategic objectives.
Correct answer: B. the internal environment of the organization.
Explanation:
A. Control activities are the policies and procedures implemented to ensure that risk
responses are effectively implemented. They are not the basis for the other components.
B. The internal environment of the organization - the attitude in the organization toward risk
and risk management - is the basis for all the other components of an Enterprise Risk
Management (ERM) system.
C. Risk assessment - the process of analyzing risks from the perspective of the likelihood of
the risk's occurring and the potential impact of the event if it does occur - is the core of an
Enterprise Risk System (ERM). However, it is not the basis for the other components.
D. Identification of the organization's strategic objectives is an important component of an
Enterprise Risk Management (ERM) system. However. it is not the basis for the other
components.

Question #24: All of the following financial indicators are measures of either liquidity or activity
except the
A. Merchandise inventory turnover.
B. Accounts receivable turnover.
C. Times-interest-earned ratio.
D. Average collection period in days.
Correct answer: C. Times-interest-earned ratio.
Explanation:
A. Liquidity ratios have to do with the ability of the company to pay its short-term liabilities
as the become due. Activity ratios measure the ability of the company to manage its
resources efficiently, specifically the current assets of accounts receivable and inventory,
and to manage its accounts payable effectively. The merchandise inventory turnover is a
measure of activity.
B. Liquidity ratios have to do with the ability of the company to pay its short-term liabilities
as the become due. Activity ratios measure the ability of the company to manage its
resources efficiently, specifically the current assets of accounts receivable and inventory,
and to manage its accounts payable effectively. The accounts receivable turnover ratio is
a measure of activity.
C. Liquidity ratios have to do with the ability of the company to pay its short-term liabilities
as the become due. Activity ratios measure the ability of the company to manage its
resources efficiently, specifically the current assets of accounts receivable and inventory,
and to manage its accounts payable effectively. Times interest earned measures neither
the ability to pay current liabilities nor the company's management of its resources and its
accounts payable.
D. Liquidity ratios have to do with the ability of the company to pay its short-term liabilities
as the become due. Activity ratios measure the ability of the company to manage its
resources efficiently, specifically the current assets of accounts receivable and inventory,
and to manage its accounts payable effectively. The average collection period is a
measure of activity current assets of accounts receivable and inventory, and to manage its
accounts payable effectively. Times interest earned measures neither the ability to pay
current liabilities nor the company's management of its resources and its accounts
payable.

Question #25: Leland Manufacturing uses 10 units of Part Number KJ37 each month in the
production of radar equipment. The unit cost to manufacture 1 unit of KJ37 is presented below.

Direct materials $1,000


Materials handling (20% of direct material
cost) 200
Direct labor 8,000
Manufacturing overhead (150% of Direct
labor) 12,000
$21,00
Total manufacturing cost 0

Material handling represents the direct variable costs of the Receiving Department that are
applied to direct materials and purchased components on the basis of their cost. This is a separate
charge in addition to manufacturing overhead. Leland's annual manufacturing overhead budget is
one-third variable and two-thirds fixed. Scott Supply, one of Leland's reliable vendors, has
offered to supply Part Number KJ37 at a unit price of $15,000.
If Leland purchases the KJ37 units from Scott, the capacity Leland used to manufacture these
parts would be idle. Should Leland decide to purchase the parts from Scott, the unit cost of KJ37
would
A. Decrease by $3,200.
B. Decrease by $6,200.
C. Increase by $4,800.
D. Change by some amount other than those given.
Correct answer: Increase by $4,800
Explanation:
A. This is the difference between the total manufacturing cost and the total cost to purchase
the units. It does not take into consideration the fact that the fixed manufacturing
overhead is unavoidable and would continue even if the KJ37 is not being manufactured.
Since the fixed manufacturing overhead is the same whether Leland manufactures the
KJ37 or purchases it, it is an irrelevant cost and should not be included in the total
manufacturing cost used to compare with the cost to purchase the item.
B. This answer assumes that there are no other costs involved in the decision besides the
initial purchase price of $15,000 ($21,200 - $15,000 = $6,200). Other relevant costs
would include unavoidable variable cost and unavoidable fixed cost
C. Even though the manufacturing overhead is being applied on the basis of direct labor
hours and no direct labor hours would be required for production if the KJ37 is purchased
outside, the fixed portion of manufacturing overhead would continue to be incurred even
if Leland purchases the KJ37 from Scott Supply. Therefore, the fixed component of the
manufacturing overhead (2/3 of $12,000. or $8,000) is not relevant to the decision and
can be ignored. Total variable manufacturing cost per unit for Leland is $13,200 ($1,000
DM + $200 materials handling + $8.000 DL + $4,000 variable overhead [1/3 of $12,000]
If Leland purchases the units from Scott then the per unit cost will be $18,000 ($15,000
purchase price + $3,000 materials handling charge [20% of $15,000)). By purchasing
from Scott the per unit cost increase of component KJ37 will be $4,800 ($18,000 -
$13,200).
D. The correct answer is one of the answers that is given.

Question #26: A company obtained a short-term bank loan of $250,000 at an annual interest rate
of 6%. As a condition of the loan, the company is required to maintain a compensating balance
of $50,000 in its checking account. The company's checking account earns interest at an annual
rate of 2%. Ordinarily, the company maintains a balance of $25,000 in its checking account for
transaction purposes. What is the effective interest rate of the loan?
A. 6.66%
B. 6.44%
C. 5.80%
D. 7.00%
Correct answer: B. 6.44%
Explanation:
A. This is $15,000 interest per year on the loan divided by the net usable loan proceeds of
$225,000. This answer fails to take into account the interest that will be earned on the
money deposited to meet the compensating balance requirement. See the correct answer
for a complete explanation.
B. In a loan with a compensating balance, the borrower pays interest on the full amount of
the loan but does not receive the use of the full amount of the loan in cash, since they are
required to leave some of it on deposit as a compensating balance. In this case, since they
already maintain a $25,000 balance at the bank, they will need to add only $25,000 from
the loan proceeds to meet the compensating balance requirement. Therefore, the company
will have the use of $225,000 of the loan but they will pay interest of 6% on the full
$250,000 loan amount. $250,000 x 0.06 equals $15,000 of annual interest expense.
However, this interest expense is reduced by the interest that will be earned on the money
that was deposited to meet the compensating balance requirement. The incremental
amount of the deposit increase is not the full $50,000 of the required compensating
balance, but only the $25,000 that the company needed to add to what was already in the
bank. Interest earned on additional $25,000 at 2% per annum equals $500 interest
received. This interest received offsets the larger interest cost, making a net interest
expense of $14,500. The effective interest rate on the loan is thus $14,500 + the $225,000
received, or 6.44%.
C. This answer is incorrect because in a loan with a compensating balance the effective
interest rate is higher than the nominal interest rate. See the correct answer for a complete
explanation.
D. This answer fails to take into consideration the fact that the company ordinarily maintains
a balance of $25,000 in its checking account for transaction purposes. Thus, the amount
of the loan proceeds that will be added to the checking account needs to be only $25,000,
not the full $50,000 required.

Question #27: Which of the following financial ratios is used to assess the liquidity of a
company?
A. Profit Margin on Sales.
B. Total Debt to Total Assets Ratio.
C. Current Ratio.
D. Days' Sales Outstanding.
Correct answer: C. Current ratio.
Explanation:
A. Profit margin on sales is a profitability measure.
B. Total debt to total assets ratio is a measure of the use of debt in the company.
C. The current ratio is one of the ratios used to measure the liquidity of a company.
D. Days' Sales Outstanding is not a ratio at all because it does not include information on
whether the days' sales outstanding is for receivables or for inventory.

Question #28: The following data pertains to XYZ Company for the current year of operations.

Total Per unit


$1,000,00
Sales (40,000 units) 0 $25
Raw materials 160,000 4
Direct labor 280,000 7
Factory overhead
Variable 80,000 2
Fixed 360,000
Selling and general
expenses:
Variable 120,000 3
Fixed 225,000

How many units does XYZ Company need to produce and sell to make a before-tax profit of
10% of sales?
Correct Answer: B. 90,000 units.
A. 36,562 units.
B. 90,000 units.
C. 25,000 units.
D. 65,000 units.
Correct answer: B. 90,000 units.
Explanation:
A. This is total fixed costs divided by variable costs per unit. The correct answer is total
fixed costs divided by an adjusted contribution margin where the required profit per unit
is included as a variable cost.
B. Variable costs per unit total $16 ($4 + $7 + $2 + $3). The required before-tax profit of
10% of the sales price needs to be considered an additional variable cost for this purpose
and the contribution margin needs to be adjusted accordingly. 10% of the $25 sales price
is $2.50. Thus the contribution margin for this purpose is $25 - $16 - $2.50 = $6.50.
Fixed costs include both factory overhead ($360,000) and selling & administrative costs
($225,000), or $585,000. Therefore, the number of units XYZ Company needs to produce
and sell to make a before-tax profit of 10% of sales is $585,000 + $6.50 = 90,000.
C. This is the fixed selling and administrative costs divided by the contribution margin per
unit. Total fixed costs include both factory overhead and selling & administrative costs:
and the contribution margin per unit needs to be adjusted to include the amount of
required profit per unit as a variable cost.
D. 65,000 units is the breakeven volume, where operating profit is zero.

Question #29: The current market price of Action Pharmaceutical's common stock is $34. A 6-
month call option has been written on the stock. The option has an exercise price of $40 and a
market value of $4.A financial analyst estimates that, at the end of 6 months, the expected value
of the stock is $42.
What is the theoretical value of exercising the option on the date it is written?
Correct Answer: • C. $0 All Possible Answers:
A. $6.00
B. $8.00
C. $0
D. $4.00
Explanation:
A. This is the difference between the exercise price of the option and the current market
price of the stock. However, that is not the theoretical value of the option on the date it is
written.
B. This is the difference between the expected market value of the stock and the current
market value of the stock. However, this is not the theoretical value of the option on the
date it is written.
C. At the time the option was written, it had no theoretical value. This is because the option
exercise price ($40) was higher than the market price ($34). Because of this, no one
would use the option to buy a share since it would be cheaper to buy a share on the open
market.
D. This is the market value of the option. However, the market value is not the same as the
theoretical value of the option on the date it is written.
Question #30: With respect to a given risk, an expected loss is
Correct Answer: A an amount that management expects to be lost per year on average over a
period of several years.
A. An amount that management expects to be lost per year on average over a period of
several years.
B. The amount expected to be lost every year to that risk.
C. The amount that management thinks could be lost to the risk in a very bad year.
D. A loss that management knows will occur.
Explanation:
A. An amount that management expects to be lost per year on average over a period of
several years is the definition of expected loss. The expected loss to a given risk can be
calculated as a weighted average of the possible losses to that risk, weighted according to
their probabilities of occurring.
B. An expected loss is not the amount expected to be lost every year to a given risk.
C. The amount that management thinks could be lost to the risk in a very bad year is the
maximum probable loss.
D. If management knows without a doubt that a loss will occur, there is no risk involved.

Question #31: An advantage of the net present value method over the internal rate of return
model in discounted cash flow analysis is that the net present value method
Correct Answer: • B. Can be used when there is no constant rate of return required for each year
of the project.
A. Uses a discount rate that equates the discounted cash inflows with the outflows.
B. Can be used when there is no constant rate of return required for each year of the project.
C. Computes a desired rate of return for capital projects.
D. Uses discounted cash flows whereas the internal rate of return model does not.
Explanation:
A. The internal rate of return is the discount rate that equates the discounted cash inflows
with the outflows for a project. However, the net present value method does not use the
projects internal rate of return as the discount rate when determining the present value of
the cash inflows and outflows.
B. Since each year's net cash flow is calculated individually and may be discounted
individually using net present value analysis, the net present value method can
incorporate varying rates of returns during the various years of the projects life. In
contrast, the internal rate of return method is used to determine the single discount rate at
which the net present value of a project is zero. Thus, the internal rate of return model
cannot be used when there is no constant rate of return required for each year of a project.
C. The net present value method does not compute a desired rate of return for capital
projects. The desired rate of return must be determined by management and then is used
to determine the present value of the cash flows, both positive and negative.
D. Both the net present value method and the internal rate of return method use discounted
cash flows.

Question #32: Please see the question below:


Garland Corporation's Income Statement for the year just ended is shown below.
$900,00
Net sales 0
Cost of goods sold:
$125,00
Invetory-beginning 0
Purchases 540,000
Goods availbale for sale 665,000
Inventory-ending 138,000
Cost of goods sold: 527,000
Gross profit 373,000
Operating expenses 175,000
$198,00
income from operations 0

Garland's average inventory turnover ratio is


Correct Answer: A. 4.01.
A. 4.01.
B. 6.84.
C. 3.82.
D. 6.52.
Explanation:
A. The inventory turnover ratio is Cost of Goods Sold + Average Inventory. Average
inventory is the average of the beginning and ending inventories, which is ($125,000 +
$138,000) ÷ 2, or $131,500. So the inventory turnover ratio is $527,000 + $131,500,
which equals 4.01.
B. The inventory turnover ratio is Cost of Goods Sold + Average Inventory. This is Sales +
Average Inventory.
C. The inventory turnover ratio is Cost of Goods Sold + Average Inventory. This is Cost of
Goods Sold + Ending Inventory.
D. The inventory turnover ratio is Cost of Goods Sold + Average Inventory. This is Sales +
Ending Inventory.

Question #33: A company is in the process of evaluating a major product line expansion. Using a
14%discount rate, the firm has calculated the present value of both the project's cash inflows and
cash outflows to be $15.8 million. The company will likely evaluate this project further by
Correct Answer: D. comparing the internal rate of return versus the company's cost of capital and
hurdle rate.
A. Comparing the internal rate of return versus the discount rate used.
B. Taking a closer look at the expansion's contribution margin.
C. Comparing the internal rate of return versus the accounting rate of return.
D. Comparing the internal rate of return versus the company's cost of capital and hurdle rate.
Explanation:
A. The IRR is the discount rate at which the NPV of the project is zero. Since the present
value of both the project's cash inflows and cash outflows are the same ($15.8 million)
when a discount rate of 14% is used, the project's NPV is zero at that discount rate. Thus,
the internal rate of return is the same as the discount rate, and there is no new knowledge
to be gained by comparing the two.
B. The contribution margin of a project is not a part of a capital budgeting analysis.
C. When the accounting rate of return method is used for capital budgeting, management
sets a required accounting rate of return, and projects whose returns exceed that rate are
considered acceptable. Comparison with the internal rate of return of the project would
not be meaningful, because the internal rate of return is based on cash flow, whereas the
accounting rate of return is based on accounting income.
D. Since the present values of both the project's cash inflows and cash outflows are the same
($15.8 million), the NPV of this project is zero, when a discount rate of 14% is used. If
the discount rate the company has used as a hurdle rate is truly representative of its cost
of capital and the anticipated risk in the project, this is not a project the company should
embark upon, because it will not earn any profit for the shareholders. So the company
should evaluate whether the 14% that it used as its hurdle rate is reasonable. To do that, it
should compare the company's cost of capital with the hurdle rate it used. This is the only
answer choice that includes comparing the company's cost of capital with the hurdle rate.

Question #34: Which of the following will result in raising the breakeven point?
Correct Answer: • C. An increase in the semi-variable cost per unit.
A. A decrease in income tax rates.
B. An increase in the contribution margin per unit.
C. An increase in the semi-variable cost per unit.
D. A decrease in the variable cost per unit.
Explanation:
A. Income taxes are considered to be a variable expense, and as such a decrease in the
income tax rate would increase the contribution margin, thereby lowering the breakeven
point.
B. An increase in the contribution margin per unit would decrease the breakeven point.
C. Semi-variable costs have a fixed component and a variable component. A basic fixed
amount must be paid even if there is no activity: and added to that fixed amount is a
variable amount that varies with activity. An increase in either a fixed or variable expense
will increase the breakeven point. The formula to calculate the breakeven point in units is
Fixed Costs / Unit Contribution Margin. If variable costs increase, the Unit Contribution
Margin in the denominator will decrease, which will cause the breakeven point to
increase. If fixed costs increase, the numerator will increase, which will also cause the
breakeven point to increase. Therefore, an increase in a semi-variable cost would increase
the breakeven point as well.
D. A decrease in variable cost per unit would increase the contribution margin, thereby
decreasing the breakeven point.
Question #35: A company with $280,000 of fixed costs has the following data:

Product A Product B
Sales price per unit $5 $6
Variable costs per unit $3 $5

Assume three units of A are sold for each unit of B sold. How much will sales be in dollars of
product B at the breakeven point?
Correct Answer: • C. $240,000
A. $280,000
B. $200,000
C. $240,000
D. $840,000
Explanation:
A. This is not the correct answer. Please see the correct answer for an explanation.
B. This is not the correct answer. Please see the correct answer for an explanation.
C. The first step is to calculate the composite units contribution margin for products A and
B. The formula is: [3 units of A x ($5 - $3)] + [1 unit of B x ($6 - $5)] = (3 x $2) + (1 x
$1) = $6 + $1 = $7 composite unit contribution margin. This is the contribution margin
for a basket of goods that includes 3 units of A and 1 unit of B.
The second step is to calculate the composite breakeven point in number of baskets.
Fixed cost $280,000 ÷ Composite CM $7 = 40,000 composite baskets of goods
containing 3 units of A and 1 unit of B. The third step is to calculate the breakeven point
in sales for Product B. If the breakeven number of baskets is 40.000 and each basket
contains 1 unit of Product B. then the breakeven number of units of Product B is 40.000 x
1, or 40,000. 40,000 units of Product B x $6 sales price of Product B = $240,000 sales
revenue for Product B at the breakeven point.
D. This is not the correct answer. Please see the correct answer for an explanation.

Question #36: An agreement to exchange a fixed interest rate on a loan with a floating interest
rate on a loan is called a (n)
Correct Answer: A. interest rate swap.
A. Interest rate swap.
B. Basis swap.
C. Swaption.
D. Interest rate guarantee.
Explanation:
A. An interest rate swap takes place when two parties exchange interest payments, one at a
fixed rate and one at a floating (or variable) rate that is pegged to some sort of market rate
of interest and changes whenever the market rate changes. The primary purpose of an
interest rate swap is to match the characteristics of the firm's revenue stream with the
characteristics of its payment stream. For example, if a firm has a revenue stream that
increases or decreases with the market rate of interest, it would want its payment stream
to also increase or decrease with interest rates. If the firm has a fixed rate loan, swapping
the fixed rate loan for a floating rate loan would achieve this goal, and reduce the firm's
overall risk.
B. A basis swap has is an interest rate swap with floating payments on both sides, each tied
to two different indexes. So a basis swap does not involve a fixed interest rate on one
side.
C. A swaption is an option to enter into a swap transaction at a specified future date, with
the terms of the swap being fixed at the time the swaption is transacted. A swaption may
be an option to enter into an agreement to exchange a fixed interest rate on a loan with a
floating interest rate on a loan. However. a swaption is not the agreement to do so.
D. An agreement to exchange a fixed interest rate on a loan with a floating interest rate on a
loan is not an interest rate guarantee.

Question #37: Book value per common share represents the amount of shareholders' equity
assigned to each outstanding share of common stock. Which one of the following statements
about book value per common share is correct?
Correct Answer: • Book value per common share can be misleading because it is based on
historical cost
A. Book value per common share can be misleading because it is based on historical cost
B. A market price per common share that is greater than book value per common share is an
indication of an overvalued stock
C. Book value per common share is the amount that would be paid to shareholders if the
company were sold to another company
D. Market price per common share usually approximates book value per common share.
Explanation:
A. Because the calculation of book value per share is based on balance sheet amounts, it is
very possible that the book value per share will not reflect the current situation of the
company. This is demonstrated by assuming that a company owns an asset that has
appreciated greatly in value while the company has held it. This asset will be recorded on
the books at its lower cost of acquisition, and this will lead to an understated book value
per share.
B. The market and book value of the shares may be different and a greater market price does
not indicate an overvalued share. This may happen if the company owns an asset that has
appreciated greatly in value while the company has held it. This asset will be recorded on
the books at its lower cost of acquisition and this will lead to an understated book value
per share.
C. The amount that would be paid to a shareholder in the event that the company is sold to
another would be determined by the specifics of the transaction itself, not by the book
value per share.
D. The market value and the book may approximate each other, but they do not have to. If
the company has an asset that has appreciated greatly in value, this increased value of the
asset will not be reflected in book value per share, but the market will have taken it into
account when setting the market price of the shares.

Question #38: A company has developed a new technologically advanced paper-thin solar panel
for residential home use. The company has decided to start selling this solar panel worldwide
next month. As this is a technologically innovative product, one risk that the company faces to
sell the solar panel is possible product failure. This risk of product failure is best representative
of what type of risk to the company?
Correct Answer: • B. Operational risk.
A. Inherent risk.
B. Operational risk
C. Residual risk.
D. Business risk.
Explanation:
A. Inherent risk is the level of risk in each event before any mitigation action is taken. The
risk of product failure is not an inherent risk.
B. Operational risk is risk that results from inadequate or failed internal processes, people or
systems. The risk of product failure is an operational risk.
C. Residual risk is the level of risk that remains after management has taken action to
mitigate the risk. The risk of product failure is not a residual risk.
D. Business risk for a firm is the risk of changes in its earnings before interest or taxes.
Business risk depends on a variety of factors, including the variability of demand, sales
price, and the price of inputs as well as the amount of the company's operating leverage.
The risk of product failure is not a business risk.

Question #39: A company has a 50%gross margin, general and administrative expenses of $50,
interest expense of $20, and net income of $10 for the year just ended. If the corporate tax rate is
50%, the level of sales revenue for the year just ended was
Correct Answer: • $180
A. $135
B. $150
C. $180
D. $90
Explanation:
A. This not the correct answer. Please see the correct answer for a complete explanation.
B. We have been unable to determine how to calculate this incorrect answer choice. If you
have calculated it, please let us know how you did it so we can create a full explanation
of why this answer choice is incorrect.
C. If net income is $10 and the tax rate is 50%, we know that income before taxes is $20
($10 ÷ [1- tax rate]). We also know that the cost of goods sold is 50% of the sales price
since the gross margin is 50% (Sales - COGS = Gross Margin; therefore, Sales - Gross
Margin = COGS). Total expenses are $70 (general and administrative $50 plus interest
$20).
Plugging the numbers we know into the net income (before taxes) function, we get the
following formula, where R represents revenue: R - 0.5R - 70 = 20.
1) Combine like terms, the like terms being the Rs. R - 0.5R = 0.5R. Therefore. 0.5R - 70
= 20
2) Add 70 to both sides of the equation to isolate the term containing the unknown on one
side of the equals
sign: 0.5R = 90
3) Divide both sides of the equation by 0.5 to find R: R = 180 $180 is the revenue of this
company.
D. This is the amount of the cost of goods sold. See the correct answer for a complete
explanation.
Question #40: A large multinational company currently has its information technology
department located in Germany. In order to reduce the risk of system failure, the company has
decided to split up the information technology department into two geographically separate
locations and set up a new location in Singapore. The company can still face a catastrophic
system failure, but the risk will be greatly reduced. The risk that remains after the company sets
up the second information technology department in Singapore is best described as
Correct Answer: • D. residual risk.
A. Hazard risk.
B. Inherent risk.
C. Business risk.
D. Residual risk.
Explanation:
A. Hazard risk is the type of risk that is can be insured against. Common examples are
natural disasters (property insurance), death of a key employee (key person life
insurance), personal injury that takes place on the premises of the business (liability
insurance), and any other unexpected event that can be insured against. The risk that
remains after the company sets up the second information technology department in
Singapore is not hazard risk.
B. Inherent risk is the level of risk in each event before any mitigation action is taken. Since
action has been taken to mitigate the risk by splitting up the information technology
department into two geographically separate locations, the risk remaining is not inherent
risk.
C. Business risk for a firm is the risk of changes in its earnings before interest or taxes.
Business risk depends on a variety of factors, including the variability of demand, sales
price, and the price of inputs as well as the amount of the company's operating leverage.
The more stable all of these variables are, the less business risk a company will
experience. The risk that remains after the company sets up the second information
technology department in Singapore is not business risk.
D. Residual risk is the level of risk that remains after management has taken action to
mitigate the risk. Thus the risk that remains after the company sets up the second
information technology department in Singapore is best described as residual risk.

Question #41: The term relevant cost applies to all the following decision situations except the
Correct Answer: • D. Determination of a product price.
A. Replacement of equipment.
B. Acceptance of a special order.
C. Manufacture or purchase of component parts.
D. Determination of a product price.
Explanation:
A. Relevant costs are incremental or differential costs that vary among the possible choices.
Relevant costs will be important in the decision of whether or not to replace equipment
because the relevant costs will be different for each of the options.
B. Relevant costs are important to the decision whether to accept a special order or not.
C. Relevant costs are incremental or differential costs that vary among the possible choices.
Relevant costs will be important to the decision whether to manufacture or purchase
component parts because the relevant costs will be different for each of the options.
D. Relevant costs are incremental or differential costs that vary among the possible choices.
The costs will be the same regardless of what product price is chosen.
Question #42: What are the two primary areas where management accountants have a
responsibility to act as change agents within their organizations where ethics are concerned?
Correct Answer: • D. Internal control and risk management
A. Internal control and financial reporting
B. Management accounting and risk management
C. Financial reporting and risk management
D. Internal control and risk management
Explanation:
A. Although ethics are very important in preventing fraudulent financial reporting, financial
reporting is not a primary area where management accountants have a responsibility to
act as change agents within their organizations where ethics are concerned.
B. Management accounting is not a primary area where management accountants have a
responsibility to act as change agents within their organizations where ethics are
concerned.
C. Although ethics are very important in preventing fraudulent financial reporting, financial
reporting is not a primary area where management accountants have a responsibility to
act as change agents within their organizations where ethics are concerned.
D. Internal control and risk management are the two areas that are primarily impacted by
human behavior. Therefore, those are the primary areas where management accountants
have a responsibility to act as change agents within their organizations where ethics are
concerned.

Question #43: Which of the following is not a benefit that will result when a company invests in
efforts to create a values-based organizational culture?
Correct Answer: B. Lower government taxes because of aggressive and spirited defense of the
company towards tax authorities by motivated employees.
A. Better teamwork
B. Lower government taxes because of aggressive and spirited defense of the company
towards tax authorities by motivated employees.
C. Less fraud through a sense of ownership and identification with the company
D. Higher levels of productivity through motivated. engaged employees
Explanation:
A. This is a benefit of creating a values based organizational culture.
B. Payment of lower taxes is not a benefit that results from an investment to develop an
ethics based business environment.
C. This is a benefit of creating a values based organizational culture.
D. This is a benefit of creating a values based organizational culture.
Question #44: DRP Insurance Company wants to be "best in class" in terms of Enterprise Risk
Management (ERM) implementation. To achieve this goal, the company plans to identify events
that affect the implementation of strategy and achievement of objectives. Which of the following
best reflects an analysis that would help its identification process?
Correct Answer: • A. Review of incidents and new market conditions.
A. Review of incidents and new market conditions.
B. Analysis of default histories and dispersion.
C. Review of accidents and operational measures.
D. Summary of driving records and age.
Explanation:
A. Enterprise risk management (ERM) is defined as "a process, effected by an entity's board
of directors, management and other personnel, applied in strategy setting and across the
enterprise, designed to identify potential events that may affect the entity, and manage
risk to be within its risk appetite, to provide reasonable assurance regarding achievement
of entity objectives:' A review of incidents and new market conditions is a strategy
designed to identify potential events that may affect the entity and to manage risk.
B. Enterprise risk management (ERM) is defined as "a process, effected by an entity's board
of directors, management and other personnel, applied in strategy setting and across the
enterprise, designed to identify potential events that may effect the entity, and manage
risk to be within its risk appetite, to provide reasonable assurance regarding achievement
of entity objectives: Analysis of default histories and dispersion is not a strategy designed
to identify potential events that may affect the entity and to manage risk.
C. Enterprise risk management (ERM) is defined as "a process, effected by an entity's board
of directors, management and other personnel, applied in strategy setting and across the
enterprise, designed to identify potential events that may effect the entity, and manage
risk to be within its risk appetite, to provide reasonable assurance regarding achievement
of entity objectives: A review of accidents and operational measures is not a strategy
designed to identify potential events that may affect the entity and to manage risk.
D. Enterprise risk management (ERM) is defined as "a process, effected by an entity's board
of directors, management and other personnel, applied in strategy setting and across the
enterprise, designed to identify potential events that may effect the entity, and manage
risk to be within its risk appetite, to provide reasonable assurance regarding achievement
of entity objectives." A summary of driving records and age is not a strategy designed to
identify potential events that may affect the entity and to manage risk.

Question #45: Which of the following statements is a responsibility for an IMA member
regarding the "Integrity" standard of ethical conduct?
Correct Answer: • C. Refrain from engaging in any conduct that would prejudice carrying out
duties ethically.
A. Disclose all relevant information that could reasonably be expected to influence an
intended user's understanding of the reports, analyses, or recommendations.
B. Perform professional duties in accordance with relevant laws, regulations, and technical
standards.
C. Refrain from engaging in any conduct that would prejudice carrying out duties ethically.
D. To refrain from using confidential information for unethical or illegal advantage.
Explanation:
A. This is one of the responsibilities for an IMA member regarding the "Credibility"
standard of ethical conduct.
B. This is one of the responsibilities for an IMA member regarding the "Competence
standard of ethical conduct.
C. Each IMA member has the following responsibilities with respect to the "Integrity"
standard of ethical conduct:
1) Mitigate actual conflicts of interest: regularly communicate with business associates
to avoid apparent conflicts of interest. Advise all parties of any potential conflicts.
2) Refrain from engaging in any conduct that would prejudice carrying out duties
ethically.
3) Absain from engaging in or supporting any activity that might discredit the
profession.
D. This is one of the responsibilities for an IMA member regarding the "Confidentiality"
standard of ethical conduct.

Question #46: The Doll House, a very profitable company, plans to introduce a new type of doll
to its product line. The sales price and costs for the new dolls are as follows. Selling price per
doll $100 Variable cost per doll $60 Incremental annual fixed costs $456,000 Income tax rate
30%
If 10,000 of the new dolls are produced and sold, the effect on Doll House's profit (loss) would
be
Correct Answer: D. $(39,200).
A. $(176,000).
B. $(56,000).
C. $280,000.
D. $(39,200).
Explanation:
A. This is the net incremental before-tax operating loss of $(56,000) minus $120,000, which
is 30% of the incremental contribution margin of $400,000. The incremental tax effect of
the $(56,000) incremental operating loss is 30% of the ($56,000) incremental operating
loss, not 30% of the incremental contribution margin. Furthermore, the tax effect of a loss
reduces the loss instead of increasing it. Assuming the company's other operations are
profitable, the loss will shield other net income from tax, reducing the overall tax due.
B. This is the incremental operating loss before the tax effect of the loss is taken into
consideration.
C. This answer does not include any deductions for the incremental fixed costs and the tax
effect of the fixed costs.
D. The incremental revenues and costs from the manufacture and sale of the new doll are as
follows:
Incremental sales: $100 x 10,000 $1,000,000
Incremental variable cost: $60 x 10,000 600,000
Incremental contribution margin: $40 x 10,000 $ 400,000
Incremental fixed costs 456,000
Net incremental operating loss $ (56,000)
Plus income tax effect of incremental loss: 30% of $56,000 16,800
Net incremental after-tax operating loss $(39,200)

Question #47: Which of the following scenarios would encourage a company to use short-term
loans to retire its 10-year bonds that have 5 years until maturity?
Correct Answer: • C. Interest rates have declined over the last 5 years.
A. Interest rates have increased over the last 5 years.
B. The company is experiencing cash flow problems.
C. Interest rates have declined over the last 5 years.
D. The company expects interest rates to increase over the next 5 years.
Explanation:
A. If interest rates have increases in the past five years the company is better off keeping the
bonds that they have issued because the interest rate on the bonds is lower than what the
interest rate on the new loans would be.
B. If the company switches to short term loans, they will essentially have to retire the bonds
early as the loans become due. If they are having cash flow problems, this will make
those problems worse by increasing the short term cash outflows that will be required.
C. If interest rates have declined since the bonds were issued, the company can take out new
financing at a lower rate of interest and use the money from the new financing to retire
the original, more expensive debt. This will lower their cost of interest for the next five
years.
D. If short-term rates will rise in the future, the company will not benefit by switching to
short-term loans, for which the rate will go up in future periods.

Question #48: A manufacturer with seasonal sales would be most likely to obtain which one of
the following types of loans from a commercial bank to finance the need for a fixed amount of
additional working capital during the busy season?
Correct Answer: • A Unsecured short-term term loan.
A. Unsecured short-term term loan.
B. Transaction loan.
C. Installment loan.
D. Insurance company term loan.
Explanation:
A. A short-term loan is a loan with a maturity date of less than one year in the future. A
short-term loan, either secured or unsecured, would be an appropriate type of loan to be
used to finance the need for a fixed amount of additional working capital during the busy
season. The collection of receivables from the selling season is the source of the
repayment of the loan, so the loan should be paid off about a month following the end of
the company's busy season. Therefore, the loan should have a maturity date of about one
month beyond the end of the company's busy season, so the company will have a chance
to collect the receivables from the selling season to use to pay off the loan. If the loan
cannot be paid off on its maturity date, then something is wrong, because it means the
company has used the collection of the receivables for something other than to pay off
the seasonal loan.
B. A transaction loan is a loan made for a specific purchase, such as a mortgage loan made
for the purchase of real estate or a term loan made for the purchase of equipment.
Usually, the disbursement check is made out to the seller of the item, so the lender can be
certain that the loan is being used for its designated purpose. A working capital loan such
as the one mentioned in this problem would not be called a transaction loan, because it
would not be used to make just a single purchase from a single supplier. It would
probably be used for multiple inventory purchases from multiple suppliers, possibly over
a period of several months. So it would not be practical for the loan to be disbursed by
means of checks made payable to each of the suppliers.
C. An installment loan is a type of long-term financing, and it would have a maturity date of
at least one year in the future. Installment loans are generally used for the purchase of
vehicles and other smaller fixed assets. Financing that provides additional working
capital to support the busy season is short-term financing, because it can be repaid when
the busy season is over and the receivables from the sales are collected. So an installment
loan would not be appropriate for this purpose.
D. An insurance company term loan, or any term loan no matter who the lender is, would be
used for long-term financing. A term loan is a loan made to a business for long-term
needs, such as purchase of fixed assets that has a maturity date of one year or more in the
future. Financing that provides additional working capital to support the busy season is
short-term financing, because it can be repaid when the busy season is over and the
receivables from the sales are collected. So a term loan would not be appropriate for this
purpose.

Question #49: McLean Inc. is considering the purchase of a new machine that will cost
$160,000. The machine has an estimated useful life of 3 years. Assume that 30% of the
depreciable base will be depreciated in the first year, 40% in the second year, and 30% in the
third year. The new machine will have a $10,000 resale value at the end of its estimated useful
life. The machine is expected to save the company $85,000 per year in operating expenses.
McLean uses a 40% estimated income tax rate and a 16% hurdle rate to evaluate capital projects.
Discount rates fora 16% rate are as follows: Present Value Present Value of an Ordinary
of $1 Annuity of $1 Year 1 0.862 0.862 Year 2 0.743 1.605 Year 3 0.641 2.246
What is the net present value of this project?
Correct Answer: D. $6.270
A. $5,842
B. $8,834
C. $30,910
D. $6,270
Explanation:
A. An answer of $5,842 results from subtracting the resale value of the equipment from the
purchase cost to calculate the depreciable base for tax purposes. For tax purposes, 100%
of an asset's cost is always depreciated.
B. An answer of $8,834 results from not calculating income tax due on the disposition of the
equipment at the end of 3 years. However, there will be a gain on the sale, since 100% of
the asset's cost will have been depreciated for tax purposes by the end of the third year.
C. An answer of $30,910 results from including only the initial investment and the gross
operating cash flows in the calculation of the net present value.
D. The cash flows are as follows:
The cash flows are as follows:

Year 0 Year 1 Year 2 Year 3


(160,000
Initial investment )
Depreciation 48,000 64,000 48,000
Depreciation Tax Sheild (Depr. ×
0.40) 19,200 25,600 19,200
Cash from disposition (after tax) 6,000
Operating cash flows 85,000 85,000 85,000
(34,000 (34,000 (34,000
Tax on operating cash flow at 40% ) ) )
Net cash flow 70,200 76,000 76,200
Discount factor:16% 0.862 0.743 0.641
(160,000
Discounted cash flow ) 60,512 56,914 48,844

The net present value is $(160,000) + $60,512 + $56,914 + $48,844 = $6,270

Question #50: A stock dividend


Correct Answer: • B. decreases future earnings per share.
A. Decreases the size of the firm.
B. Decreases future earnings per share.
C. Increases the debt-to-equity ratio of a firm.
D. Increases shareholders' wealth.
Explanation:
A. A stock dividend does not impact the size of the firm.
B. In a stock dividend more shares are issued to existing shareholders. Since there is no
increase in income from this event but there are more shares outstanding, future earnings
per share will decrease as a result of the stock dividend.
C. A stock dividend has no impact on the book value of the company's total equity.
Therefore, there is no effect on the debt-to-equity ratio.
D. A stock dividend does not in itself increase shareholder wealth. The stock dividend
provides more shares to each shareholder, but the total value of the shares remains
unchanged.

Question #51: The interest expense for a company is equal to its earnings before interest and
taxes (EBIT). The company's tax rate is 40%. The company's times-interest earned ratio is equal
to
Correct Answer: A. 1.0.
A. 2.0.
B. 0.6.
C. 1.0.
D. 1.2
Explanation:
A. This is not the correct answer. Please see the correct answer for an explanation. We have
been unable to determine how to calculate this incorrect answer choice. If you have
calculated it, please let us know how you did it so we can create a full explanation of why
this answer choice is incorrect. Please send us an email at admin@cmapass.com. Include
theTopic Name. Question Details or Screenshot and the actual incorrect answer choice --
not its letter, because that can change with every study session created.. Thank you in
advance for helping us to make your CMA PASS study materials better.
B. This answer results from reducing Earnings Before Interest and Taxes (EBIT) by income
tax due on it based on the tax rate, in calculating the times interest earned (interest
coverage) ratio. EBIT should not be reduced by income tax due on it.
C. The times interest earned ratio (interest coverage ratio) is EBIT ÷ Interest Expense. Since
EBIT and Interest Expense are the same, the company's times interest earned ratio must
be 1.0.
D. This is not the correct answer. Please see the correct answer for an explanation. We have
been unable to determine how to calculate this incorrect answer choice. If you have
calculated it, please let us know how you did it so we can create a full explanation of why
this answer choice is incorrect. Please send us an email at admin@cmapass.com. Include
the Topic Name. Question Details or Screenshot and the actual incorrect answer choice --
not its letter, because that can change with every study session created.. Thank you in
advance for helping us to make your CMA PASS study materials better.

Question #52: Lisa, Inc.


Statement of Financial Position

20 × 4 20 × 3
Assets
Current assets:
Cash $ 30 $ 25
Trading securities 20 15
Accounts receivable (net) 45 30
Inventories (at lower of cost of market) 60 50
Prepaid items 15 20
Total current assets $170 $140

Long-term assets:
Long-term investments:
Available-for-sale investments $ 25 $ 20
Property, plant & equipment
Land (at cost) 75 75
Building (net) 80 90
Equipment (net) 95 100
Intangible assets:
Patents (net) 35 17
Goodwill (net) 20 13
Total long-term assets $330 $315
Total Assets $500 $455

Liabilities and equity


Current liabilities:
Notes payable $23 $12
Accounts payable 47 28
Accrued interest $ 15 $ 15
Total current liabilities $ 85 $ 55

Long-term liabilities:
Long-term notes payable 10% due 12/31/20×6 $ 10 $ 10
Bonds payable 12% due 12/31/20×9 15 15
Total long-term debt $25 $25
Total liabilities $110 $80

Shareholders' equity
Preferred stock-5% cumulative, $100 par, nonparticipating
authorized, issued and outstanding $ 100 $ 100
Common stock-$10 par 20,000 shares authorized, 15,000
shares issued and outstanding 150 150
Additional paid-in capital-common 75 75
Retained earnings 65 50
Total equity $ 390 $ 375
Total liabilities & equity $500 455
Correct Answer: D.1.1
A. 1.8
B. 2.0
C. 0.6
D. 1.1
Explanation:
A. This answer incorrectly includes inventory in the calculation of the numerator.
B. This answer includes all current assets in the calculation, so it is the current ratio, not the
acid test (quick) ratio.
C. This answer does not include the receivables in the numerator of the calculation.
D. The acid test (or quick) ratio is calculated as follows: (Cash + Receivables + Trading
Securities) / Current Liabilities. Given the information in this question. we get ($30,000 +
$20,000 + $45,000) / $85.000. This is 1.1.

Question #53: In discounted cash flow techniques, which one of the following alternatives best
reflects the items to be incorporated in the initial net cash investment?

Capitalized Change in net Impact of spontaneous


expenditures (e.g., working Net proceeds from sale of old changes in current
shipping costs) capital asset in a replacement decision liabilities

I No Yes Yes Yes

II Yes No No No

III No Yes No No
IV Yes Yes Yes Yes

Correct Answer: • B. IV. All Possible Answers:


A. III.
B. IV.
C. II.
D. I.
Explanation:
A. Capitalized expenditures such as shipping costs are part of the cost of the fixed asset, and
so they are a part of the Year 0 cash outflow. If an old asset is being replaced, then the net
proceeds from the sale of the old asset are also a part of the Year 0 cash outflow. The
impact of a spontaneous change in current liabilities is a part of the Year 0 cash flow
because it is a part of the change in net working capital. An increase in accounts payable
is a spontaneous change in current liabilities. It is called "spontaneous" because it just
happens automatically. The company does not approach its vendors and ask them if it can
carry a larger balance due to them. But when the company increases its purchases that
automatically creates increased accounts payable. An increase in accounts payable would
function like a cash inflow. If accounts payable increase when inventory, a current asset,
increases that means that the full amount of the inventory increase has not been paid for
yet. Thus the apparent cash outflow caused by the increase in inventory is reduced by the
increase in accounts payable.
B. The cash outflow for the initial investment should include all of these things. Capitalized
expenditures such as shipping costs are part of the cost of the fixed asset, and so they are
a part of the Year 0 cash outflow. Any change in net working capital (usually an increase.
but it could also be a decrease) is also a part of the Year 0 cash outflow. If an old asset is
being replaced, then the net proceeds from the sale of the old asset are also a part of the
Year 0 cash outflow. The impact of a spontaneous change in current liabilities is a part of
the Year 0 cash flow because it is a part of the change in net working capital. An increase
in accounts payable is a spontaneous change in current liabilities. It is called
"spontaneous" because it just happens automatically. The company does not approach its
vendors and ask them if it can carry a larger balance due to them. But when the company
increases its purchases that automatically creates increased accounts payable. An increase
in accounts payable would function like a cash inflow. If accounts payable increase when
inventory, a current asset, increases, that means that the full amount of the inventory
increase has not been paid for yet. Thus the apparent cash outflow caused by the increase
in inventory is reduced by the increase in accounts payable.
C. Any change in net working capital (usually an increase. but it could also be a decrease) is
also a part of the Year 0 cash outflow. If an old asset is being replaced, then the net
proceeds from the sale of the old asset are also a part of the Year 0 cash outflow. The
impact of a spontaneous change in current liabilities is a part of the Year 0 cash flow
because it is a part of the change in net working capital. An increase in accounts payable
is a spontaneous change in current liabilities. It is called "spontaneous" because it just
happens automatically. The company does not approach its vendors and ask them if it can
carry a larger balance due to them. But when the company increases its purchases that
automatically creates increased accounts payable. An increase in accounts payable would
function like a cash inflow. If accounts payable increase when inventory, a current asset,
increases, that means that the full amount of the inventory increase has not been paid for
yet. Thus the apparent cash outflow caused by the increase in inventory is reduced by the
increase in accounts payable.
D. Capitalized expenditures such as shipping costs are part of the cost of the fixed asset, and
so they are a part of the Year 0 cash outflow.

Question #54: Leslie Corporation manufactures classroom desk chairs and tables. In the present
market, the company can sell as many units of product as it can manufacture, but it is constrained
by its availability of machine-hour capacity. Sales price and cost information for each unit of
product are shown below.
Desk chair Tables
Sales price $75 $180
Variable costs 60 155
Contribution margin $15 $25

Producing a desk chair requires 11/2 machine hours: producing a table requires 2% machine
hours. Which product, if any, is most profitable given the machine-hour constraints?
Correct Answer: • B. Both products are equally profitable.
A. There is not enough data to identify the most profitable product.
B. Both products are equally profitable.
C. Tables.
D. Desk chairs.
Explanation:
A. There is enough data to identify the product that is most profitable.
B. The product with the higher contribution margin per unit of the constrained resource is
the most profitable. The contribution margin per unit for desk chairs is $15 and desk
chairs require 1.5 machine hours each, so the contribution margin per machine hour for
desk chairs is $15 + 1.5, or 510. The contribution margin per unit for tables is $25 and
tables require 2.5 machine hours each, so the contribution margin per machine hour for
tables is $25 + 2.5, or $10. Therefore, desk chairs and tables are equally profitable, given
the machine-hour constraint.
C. The product with the higher contribution margin per unit of the constrained resource is
the most profitable. The contribution margin per machine hour for tables is not higher
than it is for desks.
D. The product with the higher contribution margin per unit of the constrained resource is
the most profitable. The contribution margin per machine hour for desks is not higher
than it is for tables.

Question #55: Which of the following is not a benefit of risk management?


Correct Answer: • B. Lower taxes.
A. Better contingency planning.
B. Lower taxes.
C. Improved cost control.
D. Increased shareholder value.
Explanation:
A. Contingency planning is the development of a "backup plan." or a plan that enables the
organization to respond appropriately to an unplanned event. It is used for emergency
response and post-disaster recovery. Contingency planning is particularly important in
information systems in order to insure that critical systems resources will be available
and operations can continue during an emergency situation. Better contingency planning
is a benefit of risk management because risk assessment and risk management enable
management to be aware of and prepared for more possible events that could prevent
them from achieving their objectives.
B. Lower taxes do not result from risk management, so they cannot be a benefit of risk
management.
C. Cost control is improved as a result of risk management because there are fewer
unpleasant surprises and fewer disruptions in the operations of the business.
D. Increased shareholder value is a benefit of risk management because risk management
can minimize losses and maximize opportunities.

Question #56: Bellcon is investigating the two projects shown below.

Required Return Holding Period


Investment A 5.75% 4 years
Investment B 6.25% 4 years
What is the most reasonable conclusion based on this limited information?
Correct Answer: • B. Investment B has greater risk than Investment A.
A. Investment B is the better choice for Bellcon.
B. Investment B has greater risk than Investment A.
C. Investment A has greater risk than Investment B.
D. Investment A is the better choice for Bellcon.
Explanation:
A. The problem gives only the required rates of return for both projects and the lengths of
both projects. Nothing in that information indicates that Investment B is a better choice
than Investment A.
B. From the limited information given, this is the only conclusion that can be drawn. A
higher required rate of return is used in capital budgeting when greater risk is perceived.
The required rate of return is higher for Investment B than it is for Investment A.
Therefore, we can conclude that Investment B has greater risk than Investment A.
C. A higher required rate of return is used in capital budgeting when greater risk is
perceived. Therefore, it is not correct to say that Investment A has greater risk than
Investment B. because Investment As required rate of return is lower than Investment B's,
not higher.
D. The problem gives only the required rates of return for both projects and the lengths of
both projects. Nothing in that information indicates that Investment A is a better choice
than Investment B.

Question #57: Datacomp Industries, which has no current debt, has a beta of 0.95 for its common
stock. Management is considering a change in the capital structure to 30% debt and 70% equity
This change would increase the beta on the stock to 1.05, and the after-tax cost of debt will be
7.5%. The expected return on equities is 16%, and the risk-free rate is 6%. Should Datacomp's
management proceed with the capital structure change?
Correct Answer: • D. Yes, because the weighted average cost of capital will decrease.
A. Yes, because there will be no effect on the weighted average cost of capital.
B. No, because the cost of equity capital will increase.
C. No, because the weighted average cost of capital will increase.
D. Yes, because the weighted average cost of capital will decrease.
Explanation:
A. Though the change should be made, this reason for making the change is incorrect. The
change should be made because the weighted average cost of capital will decrease. If the
weighted average cost of capital will remain the same, there is no need to make the
change.
B. The weighted average cost of capital will decrease if this change is made, so the change
should be made. See the correct answer for a complete explanation.
C. The weighted average cost of capital will decrease if this change is made, so the change
should be made. See the correct answer for a complete explanation.
D. Using the CAPM formula, which is r = r F β (r M – r F), we can determine that the
current cost of capital consisting only of equity for the firm is 15.5%:
r = 0.06 + 0.95(0.16 - 0.06) = 0.155.
The proposed capital structure will have an after tax cost equal to 13.8%:
(0.3 x 0.075) + (0.7 x [0.06 + 1.05(0.16 - 0.06)]) = 0.138.
So, the company should proceed with the capital change as it will reduce the WACC
from 15.5% to 13.8%.
Question #58: Please see the question below:
A company is considering the early retirement of its 10%, 10-year bonds payable. Before retiring
the bonds, the company's capital structure was
Correct Answer: • D. Financial leverage will decrease.
A. Asset turnover ratio will decrease.
B. Return on owner's equity will decrease.
C. Debt-equity ratio will increase.
D. Financial leverage will decrease.
Explanation:
A. Because the amount of debt will be decreased and their will be no change in equity, the
debt-equity ratio will decrease.
B. Because the bonds will be retired there will be less interest expense which will increase
net income. This increase in net income will cause the return on owners equity to
increase.
C. Because of the retirement of the bonds is done by spending cash, the total assets of the
company will decrease. This decrease in total assets will increase the asset turnover ratio.
D. Financial leverage relates to the amount of debt that a company uses for its financing. As
the amount of debt decreases, the leverage of the company will decrease. The fact that the
bonds will be retired at 103.5% of the face value is not relevant to this question.

Question #59: Given an acid test ratio of 2.0, current assets of $5,000, and inventory of $2,000,
the value of current liabilities is
Correct Answer: • C.51.500
A. $3,500
B. $2,500
C. $1,500
D. $6,000
Explanation:
A. This answer adds the value of inventory to the current assets figure instead of subtracting
it.
B. This answer incorrectly includes the value of the inventory in the numerator of the
calculation. Inventory is included in the current ratio, but not the acid test ratio.
C. The acid test ratio is calculated as follows: (Cash + Marketable Securities +
Receivables) / Current Liabilities. Inventory is a current asset, but it is not included in the
acid test ratio calculation. This means that based on the information given here, the
numerator is only $3,000 ($5,000 current assets - $2,000 inventory). Since the ratio is 2,
the current liabilities must be $1,500.
Note that the full information needed to calculate the numerator of the acid test ratio is
not given in this problem. We must assume that there are no pre-paids or other current
assets such as short-term deferred tax assets in current assets in order to calculate the
numerator this way. Pre-paids and other current assets are not included in the numerator
of the acid test, or quick, ratio. However, since total current assets and inventory are the
only information given for calculating the numerator, we must use what we have. But if
the individual components of current assets are given, the proper way to calculate the
numerator is Cash + Marketable Securities + Receivables, excluding any and all other
current assets.
D. This answer multiplies the numerator by 2 instead of dividing it by 2.

Question #60: Which one of the following items would likely increase earnings per share (EPS)
of a corporation?
Correct Answer: • Purchase of treasury stock.
A. An increase in the common stock shares authorized to be issued.
B. Declaration of a stock dividend.
C. Purchase of treasury stock.
D. Declaration of a stock split.
Explanation:
A. An increase in the number of authorized shares will have no impact on EPS because EPS
is based on the number of shares outstanding, not the number of shares authorized.
B. A stock dividend increases the number of shares outstanding and will therefore decrease
EPS.
C. The purchase of treasury stock decreases the number of shares that are outstanding. Since
treasury shares are not included in the EPS calculation, the purchase of treasury shares
will increase EPS by decreasing the number of shares to which the income must be
distributed.
D. A stock split increases the number of shares outstanding and will therefore decrease EPS.

Question #61: If the U.S. dollar declines in value relative to the currencies of many of the U.S.
trading partners, the likely result is that
Correct Answer: D. U.S. exports will tend to increase.
A. The U.S. balance of payments deficit will become worse.
B. Foreign currencies will depreciate against the dollar.
C. U.S. imports will tend to increase.
D. U.S. exports will tend to increase.
Explanation:
A. The decrease in the value of the dollar makes U.S. goods relatively less expensive
relative to foreign produced goods and this will increase U.S. exports. This will improve
the U.S. balance of payments deficit.
B. If the U.S. dollar declines in value (depreciates) relative to the currencies, the foreign
currencies must appreciate against the dollar.
C. The decrease in the value of the dollar makes U.S. goods relatively less expensive
relative to foreign produced goods and this will increase U.S. exports. Similarly, the
decline in the value of the dollar will make foreign goods relatively more expensive in
the U.S. and this will decrease imports.
D. The decrease in the value of the dollar makes U.S. goods relatively less expensive
relative to foreign produced goods and this will increase U.S. exports. This will improve
the U.S. balance of payments deficit.

Question #62: Oak Fine Furnishings manufactures a wide range of home furnishings. One of
their products is an oak headboard. The company currently sells 4,000 headboards at an average
price of $100 per unit. To manufacture the headboards, the variable costs are $55 per unit and the
total fixed costs assigned to the oak headboard are $150,000.1f the sale of headboards increases
by 50% and all else remains constant, this would result in
Correct Answer: • B. earnings before interest and taxes of $120,000.
A. A 50% increase in earnings before interest and taxes.
B. Earnings before interest and taxes of $120,000.
C. Fixed costs of $225,000.
D. A gross margin of $380,000.
Explanation:
A. Earnings will not increase by the same percentage as revenues increase because of the
presence of fixed costs within the company's cost structure. Fixed costs do not change
with changes in volume as long as the volume remains within the relevant range. Because
fixed costs assigned to headboard will not increase when sales increase, the increase in
earnings attributable to headboards should be more than 50%.
B. The contribution margin per unit is $45 ($100 - $55). With a 50% increase in sales, sales
will become 6,000 units and earnings before interest and taxes will become (6,000 x $45)
- $150,000 = $120,000.
C. By definition, even if production increases, the fixed costs will remain the same, or
$150,000.
D. This answer results from increasing revenue by 50% without also increasing variable
costs by 50%. Variable costs must increase as well.
Question #63: Which of the following statements is not a responsibility for an IMA member
regarding the "Credibility" standard of ethical conduct?
Correct Answer: • A. To recognize and communicate professional limitations or other constraints
that would preclude responsible judgment or successful performance of an activity.
A. To recognize and communicate professional limitations or other constraints that would
preclude responsible judgment or successful performance of an activity.
B. To disclose all relevant information that could reasonably be expected to influence an
intended user’s understanding of the reports, analyses, or recommendations.
C. To disclose delays or deficiencies in information, timeliness, processing, or internal
controls in conformance with organization policy and/or applicable law.
D. To communicate information fairly and objectively.
Explanation:
A. To recognize and communicate professional limitations or other constraints that would
preclude responsible judgment or successful performance of an activity is a responsibility
of an IMA member with respect to the "Competence standard of ethical conduct.
B. With respect to the "Credibility" standard of ethical professional conduct, each IMA
member has a responsibility to:
1) Communicate information fairly and objectively.
2) Disclose all relevant information that could reasonably be expected to influence an
intended user's understanding of the reports, analyses, or recommendations.
3) Disclose delays or deficiencies in information, timeliness, processing, or internal
controls in conformance with organization policy and/or applicable law.
C. With respect to the "Credibility" standard of ethical professional conduct, each IMA
member has a responsibility to:
1) Communicate information fairly and objectively.
2) Disclose all relevant information that could reasonably be expected to influence an
intended user's understanding of the reports, analyses, or recommendations.
3) Disclose delays or deficiencies in information, timeliness, processing, or internal
controls in conformance with organization policy and/or applicable law.
D. With respect to the "Credibility" standard of ethical professional conduct, each IMA
member has a responsibility to:
1) Communicate information fairly and objectively.
2) Disclose all relevant information that could reasonably be expected to influence an
intended user's understanding of the reports, analyses, or recommendations.
3) Disclose delays or deficiencies in information, timeliness, processing, or internal
controls in conformance with organization policy and/or applicable law.

Question #64: Please see the Question Below:


Select information from a company’s year-end balance sheet is shown below.
Balance Sheet
As of December 31, Year 1

Cash $50,000
Accounts receivable 120,000
Inventory 75,000
Property, plant and equipment, net 250,000
Total assets $495,000

Accounts payable $35,000


Long-term debt 100,000
Total liabilities $135,000

Common stock $300,000


Retained earnings 100,000
Total equity $360,000
Total liabilities and equity $495,000

Based on the above information, a common-size balance sheet for the company will show
Correct answer: A. accounts receivable at 24%.
A. Accounts receivable at 24%
B. Property, plant and equipment, net at 69%
C. Long-term debt at 74%
D. Retained earnings at 17%
Explanation:
A. A vertical common-size financial statement expresses each component as a percentage of
a total. For example, fixed assets will not be stated as a dollar amount but rather will be
stated as a percentage of total assets. Accounts receivable of $120,000 divided by total
assets of $495,000 are 24.2% of total assets.
B. This is property, plant and equipment, net, of $250,000 divided by total equity of
$360,000. A vertical common-size financial statement expresses each component as a
percentage of a total. That total is usually total assets (for a balance sheet). It does not
need to be total assets, but unless a question says differently, we should assume the use of
total assets. For example, fixed assets will not be stated as a dollar amount but rather will
be stated as a percentage of total assets.
C. This is long-term debt of $100,000 divided by total liabilities of $135,000. A vertical
common-size financial statement expresses each component as a percentage of a total.
That total is usually total assets (for a balance sheet). It does not need to be total assets,
but unless a question says differently, we should assume the use of total assets. For
example, fixed assets will not be stated as a dollar amount but rather will be stated as a
percentage of total assets.
D. This is retained earnings of $60,000 divided by total equity of $360,000. A vertical
common-size financial statement expresses each component as a percentage of a total.
That total is usually total assets (for a balance sheet). It does not need to be total assets,
but unless a question says differently, we should assume the use of total assets. For
example, fixed assets will not be stated as a dollar amount but rather will be stated as a
percentage of total assets.

Question #65: Which one of the following describes a disadvantage to a firm that issues
preferred stock?
Correct Answer: • B. Preferred stock is usually sold on a higher yield basis than bonds.
A. Preferred stock typically has no maturity date.
B. Preferred stock is usually sold on a higher yield basis than bonds.
C. Most preferred stock is owned by corporate investors.
D. Preferred stock dividends are legal obligations of the corporation.
Explanation:
A. Preferred stock usually does not have a maturity date, but that is not a disadvantage to the
firm that issues it.
B. Because equity carries no guarantee of a return, investors typically demand a higher yield
to compensate for the higher risk.
C. Preferred stock can be owned by corporate investors, institutional investors, or private
investors.
D. The payment of a preferred stock dividends is discretionary. Missing a dividend payment
will not be a default on the part of the issuer. The board of directors can omit paying a
preferred stock dividend if it chooses. Therefore, preferred stock dividends are not legal
obligations of the corporation.

Question #66: After completing a marketplace analysis of Product Z, a company's accountant has
determined that a price change from $25 to $20 will result in a demand increase for Product Z
from 1,000 units to 1.500 units. Based on the information provided, what is the price elasticity of
demand for Product Z using the midpoint formula?
Correct Answer: • C. 1.80.
A. 1.33
B. 0.56.
C. 1.80.
D. 2.50.
Explanation:
A. This answer results from an attempt to calculate the price elasticity of demand using the
percentage method. It is incorrect for two reasons:
(1) The calculation is done incorrectly for the percentage method. For the percentage
method, the percentages of change in the quantity and the price should be calculated
by dividing the amount of change by the initial quantity and price, whereas the
percentages used in this answer were calculated by dividing the amount of change by
the changed quantity and price.
(2) The question says to use the midpoint method, not the percentage method, to
calculate the price elasticity of demand. Using the midpoint method. the formula for
the price elasticity of demand is:
(Q2 -Q1)/(Q2 +Q1)/2]
Ed= ……………………………
(P2 -P 1 )/[(P 2 +P 1)/2]
Where: Q 1 and 2 = First and second
quantity points
P 1 and 2 = First and second price
points
B. Using the midpoint method. the formula for the price elasticity of demand is:
(Q2 -Q 1)/[(Q2 +Q1)/2]
Ed=………………………….
(P2 -P1)/[(P 2 +P 1)/2]
Where: Q 1 and 2 = First and second
quantity points
P 1 and 2 = First and second price
points
This answer results from reversing the numerator and denominator in the formula, as follows:
(P2 -P 1)/[(P 2+P 1)/2]
Ed= …………………………….
(Q2 -Q1)/[(Q2 +Q1)/2]
C. Using the midpoint method, the formula for the price elasticity of demand is:
(Q2 -Q 1)/[(Q2 +Q1)/2]
Ed=…………………………………..
(P2 -P1)/[(P 2+P 1)/2]
Where: Q 1 and 2 = First and second
quantity points
P 1 and 2 = First and second price points
500 / 1.250 0.400
Ed=……………………………… 1.80
$5/ $22.50 0.222
D. This is the price elasticity of demand using the percentage method. The question says to
use the midpoint method, not the percentage method. Using the midpoint method. the
formula for the price elasticity of demand is:
(Q2 -Q1)/[(Q2 -Q1)/2] E d (P2 -P1)/[(P 2 -P 1)/2]
Where: Q 1 and 2 = First and second
quantity points
P 1 and 2 = First and second price points

Question #67: The management of a company is considering making a capital investment to


acquire a machine for its manufacturing facility at a total cost of €600.000 for equipment and
installation. The machine has a useful life of 5 years and a zero salvage value at the end of its
useful life. The management of the company uses the straight-line depreciation method for all
machinery acquired. How much would the company's annual tax savings be upon acquiring the
machine if the company's income tax rate is 30%?
Correct Answer: • B. €36.000.
A. €120000.
B. €36.000.
C. €180.000.
D. €84.000.
Explanation:
A. €120,000 is the amount of the annual depreciation on the equipment. The annual tax
savings for the company is the amount of the annual depreciation multiplied by the
company's tax rate.
B. The annual depreciation expense will be €600,000 + 5, or €120,000. At an income tax
rate of 30%, the depreciation tax shield will be 30% of €120,000, or €36,000.
C. €180,000 is 30% of the €600,000 investment. The annual tax savings for the company is
the amount of the annual depreciation on the investment multiplied by the company's tax
rate.
D. €84,000 is the amount of the annual depreciation less the amount of the depreciation tax
shield. The annual tax savings for the company is only the amount of the depreciation tax
shield.

Question #68: When is it appropriate to communicate potential ethical issues to authorities or


individuals not employed or engaged by your organization?
Correct Answer: • D. You believe there is a clear violation of the law.
A. Your immediate supervisor is involved and you are concerned about potential negative
consequences for yourself if you follow established company policy.
B. Your immediate supervisor, who is aware of the issue but is not involved, does not want
to take any action because he is concerned about potential negative consequences for
himself if the matter is addressed.
C. You have escalated the issue, as directed by company policy, but are not satisfied with
the company's response.
D. You believe there is a clear violation of the law.
Explanation:
A. If your immediate supervisor is involved, then you should present the issue to the next
level up, regardless of the potential for negative consequences.
B. If your immediate supervisor is not willing to take any action to resolve the ethical
conflict, then you should submit the issue to the next management level. Contact with
levels above the immediate superior should be initiated only with your superior's
knowledge, assuming he or she is not involved.
C. The highest acceptable reviewing authority may be a group such as the audit committee,
executive committee, board of directors, board of trustees, or owners. If you have taken
your concerns all the way up to this level without receiving any resolution of the ethical
conflict, it is still not appropriate to communicate the problem to authorities or
individuals outside the organization, unless you believe a clear violation of the law has
taken or is taking place.
D. If the law is clearly being violated, and if you have not been able to obtain satisfactory
resolution of the ethical conflict through other means, then it may be necessary to inform
authorities.

Question #69: Watson Corporation computed the following items from its financial records for
the year just ended: Price-earnings ratiol2 Payout ratio 0.6 Asset turnover 0.9 The dividend yield
on Watson's common stock is
Correct Answer: D. 5.0%
A. 7.5%
B. 7.2%
C. 10.8%
D. 5.0%
Explanation:
A. This is the asset turnover ratio (0.9) divided by the P-E ratio (12). The dividend yield is
calculated as the cash dividend per share divided by the market price per share. Since we
are not given either of these items directly, we need to calculate them from the
information that is given. See the correct answer for a complete explanation.
B. This is the P-E ratio (12) multiplied by the payout ratio (0.6). The dividend yield is
calculated as the cash dividend per share divided by the market price per share. Since we
are not given either of these items directly, we need to calculate them from the
information that is given. See the correct answer for a complete explanation.
C. This is the P-E ratio (12) multiplied by the asset turnover ratio (0.9). The dividend yield
is calculated as the cash dividend per share divided by the market price per share. Since
we are not given either of these items directly, we need to calculate them from the
information that is given. See the correct answer for a complete explanation.
D. The dividend yield is calculated as the cash dividend per share divided by the market
price per share. Since we are not given these items directly, we need to calculate them
from the information that is given. Let us assume that the P-E ratio is made up of a $12
market price and $1 of earnings. This gives us the P-E ratio of 12 that we are told the
company has. The payout ratio tells us what percentage of earnings were distributed as
cash, and since it is 0.6, in this example $0.60 would have been distributed to the
shareholders. We now know the necessary information to calculate the dividend yield.
The market price is $12 and the cash dividend was $0.60. $0.60 divided by $12 equals a
dividend yield of 5%.

Question #70: How can a management accountant use the Fraud Triangle to identify and manage
the risk of fraud?
Correct Answer: • C. The Fraud Triangle provides a model for explaining the pressures,
rationalizations, and opportunities that influence people to commit fraud.
A. The Fraud Triangle provides a model for explaining how persuasion, coercion, and
conviction influence people to commit fraud.
B. The Fraud Triangle provides a model for explaining the motives, means, and
opportunities that influence people to commit fraud.
C. The Fraud Triangle provides a model for explaining the pressures, rationalizations, and
opportunities that influence people to commit fraud.
D. The Fraud Triangle provides a SOX-compliant model for examining the company's
internal control environment in terms of its risk of fraud.
Explanation:
A. The Fraud Triangle does not provide a model for explaining how persuasion, coercion,
and conviction influence people to commit fraud.
B. The Fraud Triangle does not provide a model for explaining the motives, means, and
opportunities that influence people to commit fraud.
C. According to the Fraud Triangle model, three conditions need to be present for an
employee to commit a fraudulent act against an employer. The three conditions are
pressure, opportunity, and rationalization.
D. The Fraud Triangle does not provide a SOX-compliant model for examining the
company's internal control environment in terms of its risk of fraud.

Question #71: What variable is measured on the horizontal axis of the yield curve?
Correct Answer: • D. Years to maturity of the bonds.
A. Par value of the bonds.
B. Duration of the bonds.
C. Yield of the bonds.
D. Years to maturity of the bonds.
Explanation:
A. The par value of an individual bond is its stated amount or its face value. The par value of
bonds is not represented graphically on a yield curve graph.
B. The duration of an individual fixed income security is a weighted average of the times
until the receipt of both interest and principal, weighted according to the proportion of the
total present value of the bond represented by the present value of each cash flow to be
received. Duration is not represented graphically on a yield curve graph.
C. The yield curve shows the market rates for various maturities of bonds on a given date.
The market rates, or the yields, are on the vertical axis, not the horizontal axis.
D. The yield curve shows the market rates for various maturities of bonds on a given date.
The years to maturity of the bonds are on the horizontal axis and the market rates are on
the vertical axis.

Question #72: Donnelly Corporation manufactures and sells T-shirts imprinted with college
names and slogans. Last year, the shirts sold for $7.50 each, and the variable cost to manufacture
them was $2.25 per unit. The company needed to sell 20,000 shirts to break even. The net
income last year was $5.040. Donnelly's expectations for the coming year include the following:
-The sales price of the T-shirts will be $9 -Variable cost to manufacture will increase by one-
third -Fixed costs will increase by 10% -The income tax rate of 40% will be unchanged If
Donnelly Corporation wishes to earn $22,500 in net income for the coming year. the company's
sales volume in dollars must be
Correct Answer: • A. $229.500
A. $229,500
B. Some amount other than those given.
C. $257,625
D. $213,750
Explanation:
A. This question is asking for the sales revenue given a requirement for an after-tax net
income of $22,500. To solve it, we need to use the version of the breakeven formula for
determining the sales revenue required to result in a specific dollar amount of profit.
Target Sales Revenue = (FC + Target Pretax Income) /Contribution Margin Ratio In
order to use this formula, we need to calculate three things:
(1) the amount of fixed cost in the coming year,
(2) the amount of desired net income before tax for the coming year. and
(3) the contribution margin ratio for the coming year.
(1) We are told that fixed cost for the coming year will be 10% higher than the
previous year's fixed cost. So we need to find what the fixed cost was for last
year. We know the break-even point in units for last year (20,000) and we know
the unit contribution margin for last year ($7.50 - $2.25 = $5.25). So we can find
the fixed cost for last year by using the Break-Even Point in Units formula and
solving for FC: FC / Unit Contribution Margin = BEP in Units.
FC / 5.25 = 20,000 Solving for FC, we get FC = $105,000. Since fixed cost for
the coming year will be 10% higher than last year, fixed cost for the coming year
will be $105,000 x 1.10, which is $115,500.
(2) The formula to find before-tax net income when we know the after-tax net income
is After-Tax NI / (1 - tax rate). Therefore, the desired before tax net income is
$22,500 / (1- 0.40), which is $37,500.
(3) We are told that variable cost to manufacture will increase by one-third. Variable
cost last year was $2.25 per unit. Therefore, variable cost in the coming year will
increase by 1/3 of $2.25, which is $0.75, so variable cost will be $3 per unit. We
are told that the sales price will be $9. Therefore, the Contribution Margin Ratio
will be $6 / $9, which is 2/3 or 0.666667. Now, we can calculate the Target Sales
Revenue, because we have the fixed cost, the target pretax income, and the
contribution margin ratio for the coming year. Target Sales Revenue = ($115,500
+ $37.500) /0.666667 = $229,500.
B. The correct answer is given.
C. This answer results from calculating the desired before-tax net income by dividing the
after-tax net income by the tax rate. To calculate the desired before-tax net income, the
after-tax net income should be divided by (1- the tax rate).
D. This answer results from using fixed costs of $105.000 to calculate the target sales in
dollars. Fixed costs are expected to increase by 10% in the coming year.

Question #73: Ray Corporation has long-term debt of $1,200,000 and equity of $1.000.000. The
board of directors has set a goal of 1:1 for the company's debt-equity ratio. Which of the
following could the company employ to achieve this goal?
Correct Answer: • Issuing rights to purchase new common stock.
A. Paying a stock dividend to the existing shareholders.
B. Paying a dividend on its common stock.
C. Issuing new bonds.
D. Issuing rights to purchase new common stock.
Explanation:
A. Payment of a stock dividend would not change the debt-equity ratio. When a stock
dividend is declared, the corporation does not pay out assets or incur a liability. It issues
additional shares of stock to each stockholder and nothing more. Total equity remains the
same.
B. All dividends, except for stock dividends, reduce the total stockholders' equity in the
corporation, because the equity is reduced either through an immediate or promised
future distribution of assets. Therefore, payment of a dividend on its common stock
would increase the debt-equity ratio, since it would decrease equity. This is the opposite
of management's goal, because the debt-equity ratio now is 1.2:1, and management wants
to decrease it to 1:1.
C. Issuing new bonds would increase long-term debt, which would increase the debt-equity
ratio. This is the opposite of management's goal, because the debt-equity ratio now is
1.2:1, and management wants to decrease it to 1:1.
D. The debt-equity ratio measures how much long-term debt a company has compared to its
total equity. A ratio in excess of 1:1 indicates more reliance on long-term debt financing
than on equity financing. Issuing rights to purchase new common stock would result in
increased equity because some of the rights would be exercised to purchase the newly-
issued stock. That would increase equity, which could decrease the debt-equity ratio.

Question #74: A example of a hazard risk is


Correct Answer: • C. the risk of the death of the company's CEO.
A. the risk of a major customer going bankrupt while owing a large amount.
B. the risk of a lawsuit.
C. the risk of the death of the company's CEO.
D. the risk of a recession occurring.
Explanation:
A. The risk of a major customer going bankrupt while owing a large amount is a credit risk,
and credit risk is a type of financial risk Financial risks are risks that are connected with
the company's financial health.
B. The risk of a lawsuit is an operational risk. Operational risks are risks that result from
inadequate or failed internal processes, people or systems.
C. A hazard risk is the type of risk that can be insured against. For example, the risk of a
natural disaster such as a fire or flood can be managed with property insurance: the risk
of the death of a key employee can be managed with key person life insurance: and the
risk of a person getting injured on the premises can be managed with liability insurance.
D. The risk of a recession occurring is a strategic risk. Strategic risks are risks that occur on
a more macro level such as political risks, economic risks (i.e., the risk of a recession),
the risk of customers' needs changing, or even some risks that are specific to the company
itself, such as risk to the company's reputation or to its patents or trademarks.

Question #75: Jenson Copying Company is planning to buy a copying machine costing $25,310.
The net present values (NPV) of this investment, at various discount rates, are as follows.

Discount Rate NPV


4% $2,440
6% 1,420
8% 460
10% (440)

Jenson's approximate internal rate of return on this investment is


Correct Answer: • D. 9%.
A. 6%.
B. 10%.
C. 8%.
D. 9%.
Explanation:
A. The internal rate of return is the discount rate at which the NPV of a project is zero. At a
discount rate of 6%, the NPV of the project is $1.420; so 6% cannot be the internal rate of
return for this project.
B. The internal rate of return is the discount rate at which the NPV of a project is zero.
Therefore. the IRR of this project will be the discount rate in between the discount rate of
the lowest positive NPV (the positive amount closest to zero) which is $460 and the
discount rate of the lowest negative NPV (the negative amount closest to zero) which is
($440). 10% is the discount rate of the lowest negative NPV.
C. The internal rate of return is the discount rate at which the NPV of a project is zero.
Therefore. the IRR of this project will be the discount rate in between the discount rate of
the lowest positive NPV (the positive amount closest to zero) which is $460 and the
discount rate of the lowest negative NPV (the negative amount closest to zero) which is
($440). 8% is the discount rate of the lowest positive NPV.
D. The internal rate of return is the discount rate at which the NPV of a project is zero.
Therefore. the IRR of this project will be the discount rate in between the discount rate of
the lowest positive NPV (the positive amount closest to zero) which is $460 and the
discount rate of the lowest negative NPV (the negative amount closest to zero) which is
($440). The discount rate of the lowest positive NPV is 8% and the discount rate of the
lowest negative NPV is 10%. Therefore, the IRR is in between 8% and 10%. Since the
size of the positive NPV and the size of the negative NPV that go with the 8% and 10%
discount rates are approximately equal, we also know that the IRR must be almost
exactly in between 8% and 10%. The only answer choice between 8% and 10% is 9%.

Question #76: Locar Corporation had net sales last year of $18.600.000 (of which 20% were
installment sales). It also had an average accounts receivable balance (including the installment
receivables) of $1.380.000. Credit terms are 2/10, net 30. Based on a 360-day year. Locar's
average collection period last year was
Correct Answer: • A 26.7 days.
A. 26.7 days.
B. 27.3 days.
C. 26.2 days.
D. 33.4 days.
Explanation:
A. This is not the correct answer. Please see the correct answer for a complete explanation.
B. This answer results from reducing the accounts receivable balance by 2% (to reflect the
discount for early payment) before calculating the average collection period. The
accounts receivable balance should be used as given. Although the term net annual credit
sales" is used to calculate the average collection period and the accounts receivable
turnover period, "net sales" refers to gross sales less returns and allowances, not
receivables less the discount for prompt payment (and not sales net of uncollectible
accounts. either).
C. This can be calculated either by including the installment sales in both the sales and the
receivables amounts, or by excluding the installment sales from both the sales and the
receivables amounts. The number of days in receivables (average collection period) is
360 / accounts receivable turnover ratio. Using the sales and receivables including the
installment sales, the accounts receivable turnover ratio is $18,600,000 ÷ $1,380,000,
which is 13.48 times. The average collection period is 360 ÷ 13.48= 26.7 days. Using the
sales and receivables excluding the installment sales, sales would be $18.600.000 x 0.80
= $14,880,000 and receivables would be $1,380,000 x 0.80 = $1,104,000. The accounts
receivable turnover ratio is $14,880,000 ÷ $1,104,000 = 13.48 times. And the average
collection period is 360 ÷13.48 = 26.7 days.
D. This answer results from reducing the sales by the 20% that were installment sales but
not reducing the average accounts receivable balance by 20%. The average collection
period can be calculated either by including the installment sales in both the sales and the
receivables amounts, or by excluding the installment sales from both the sales and the
receivables amounts. But including the installment sales in one amount while excluding
them from the other amount does not work. Question 180sales and the receivables
amounts. But including the installment sales in one amount while excluding them from
the other amount does not work.

Question #77: Elements of project risk identification include which one of the following?
Correct Answer: • A. Interviews and observations.
A. Interviews and observations.
B. Cost estimates.
C. Actual risk events.
D. Activity duration estimates.
Explanation:
A. Interviews and observations are risk identification techniques. Risk identification
techniques include the following: -Event inventories, detailed lists of potential events or
an archive of events that have occurred: -Internal analysis, for example the company's
own experience with similar projects: -Escalation or threshold triggers to alert
management to areas of concern: -Facilitated workshops used to identify events by
drawing on accumulated knowledge and experience of management, staff and other
stakeholders: -Interviews, questionnaires, and surveys: -Process flow analysis using flow
diagrams to consider the inputs, tasks, responsibilities, and outputs of the project: -
Leading event indicators to identify conditions that could give rise to an event: -Loss
event data on past loss events: and -Brainstorming sessions.
B. Cost estimates are not risk identification techniques. Risk identification techniques
include the following: -Event inventories, detailed lists of potential events or an archive
of events that have occurred: -Internal analysis, for example the company's own
experience with similar projects: -Escalation or threshold triggers to alert management to
areas of concern: -Facilitated workshops used to identify events by drawing on
accumulated knowledge and experience of management, staff and other stakeholders: -
Interviews, questionnaires, and surveys: -Process flow analysis using flow diagrams to
consider the inputs, tasks, responsibilities, and outputs of the project: -Leading event
indicators to identify conditions that could give rise to an event: -Loss event data on past
loss events: and -Brainstorming sessions.
C. Actual risk events are not risk identification techniques, though experience with historical
events is used in risk identification techniques. Risk identification techniques include the
following: -Event inventories, detailed lists of potential events or an archive of events
that have occurred: -Internal analysis, for example the company's own experience with
similar projects: -Escalation or threshold triggers to alert management to areas of
concern: -Facilitated workshops used to identify events by drawing on accumulated
knowledge and experience of management, staff and other stakeholders: -Interviews,
questionnaires, and surveys: -Process flow analysis using flow diagrams to consider the
inputs, tasks, responsibilities, and outputs of the project: -Leading event indicators to
identify conditions that could give rise to an event: -Loss event data on past loss events:
and -Brainstorming sessions.
D. Activity duration estimates are not risk identification techniques. Risk identification
techniques include the following: -Event inventories, detailed lists of potential events or
an archive of events that have occurred: -Internal analysis, for example the company's
own experience with similar projects: -Escalation or threshold triggers to alert
management to areas of concern: -Facilitated workshops used to identify events by
drawing on accumulated knowledge and experience of management, staff and other
stakeholders: -Interviews, questionnaires, and surveys: -Process flow analysis using flow
diagrams to consider the inputs, tasks, responsibilities, and outputs of the project: -
Leading event indicators to identify conditions that could give rise to an event: -Loss
event data on past loss events: and -Brainstorming sessions.

Question #78: Financial risk management is


Correct Answer:
• B. the use of financial instruments to manage exposure to credit, market and other financial
risks to create economic value to the company.
A. deciding which risks must be addressed and prioritizing them.
B. the use of financial instruments to manage exposure to credit, market and other financial
risks to create economic value to the company.
C. managing risks connected with the day-to-day operations of the company.
D. the level of risk that resides with an event or process prior to management taking a
mitigating action.
Explanation:
A. Deciding which risks must be addressed and prioritizing those that must be addressed is
one of the steps in the management of any type of risk, not only financial risk
management.
B. The process of using financial instruments to manage exposure to credit, market and
other financial risks in order to create value for the company is financial risk
management.
C. This is the definition of operational risk management, not financial risk management.
D. This is the definition of inherent risk.

Question #79: Calvin Inc. is considering the purchase of a new state-of-the-art machine to
replace its hand-operated machine. Calvin's effective tax rate is 40%. and its cost of capital is
12%. Data regarding the existing and new machines are presented below.
Existing New
machine machine
Original cost $50,000 $90,000
Installation cost 0 4,000
Freight and insurance 0 6,000
Expected end salvage
value 0 0
Depreciation - -
Expected useful life 10 years 5 years

The existing machine has been in service for seven years and could be sold currently for
$25,000. If the new machine is purchased, Calvin expects to realize a $30,000 before-tax annual
reduction in labor costs.
If the new machine is purchased, what is the net amount of the initial cash outflow at Time 0 for
net present value calculation purposes?
Correct Answer: A. $79,000.
All Possible Answers:
A. $79.000.
B. $65,000.
C. $75.000.
D. $100.000.
Explanation:
A. The existing machine originally cost $50,000 and it has been in service for 7 years. Its
expected useful life was 10 years when it was purchased and it is being depreciated on
the straight line basis. Therefore, $5,000 is being depreciated each year ($50,000 + 10).
So the book value of the existing machine at the time of replacement would be $50.000 -
(7 x $5,000), which is $15,000. If it is sold for $25,000, there will be a taxable gain of
$10,000 on the sale ($25,000 - $15,000). The company's tax rate is 40%, so the tax on the
gain will be 40% of $10,000, which is $4.000. The cost of the new machine, the
installation and the freight and insurance on its shipment will all be capitalized, so the tax
rate will not affect those costs in Year O. Therefore, the Year 0 net cash outflow will be:
Outflows for capitalized equipment: ($90,000) + ($4,000) + ($6,000) = ($100.000) Inflow
from sale of existing equipment: $25,000 before tax Outflow for tax on gain on sale of
existing equipment: ($4,000) The net cash outflow is ($100,000) + $25,000 + ($4,000) =
($79,000)
B. This is the cost of the new machine minus the sale price of the existing machine.
However, it does not include the installation cost_ the freight and insurance, or the tax
due on the gain on the sale of the existing machine.
C. This is the cost of the new machine plus the installation cost and freight and insurance
cost minus the sale price of the existing machine. However, it does not include the tax
due on the gain on the sale of the existing machine.
D. This is the cost of the new machine plus the installation cost and the freight and insurance
cost However, it does not include the net after-tax cash to be received from the sale of the
existing machine.

Question #80: Austin Manufacturing, which is subject to a 40% income tax rate, had the
following operating data for the period just ended.
Selling price per unit $60
Variable cost per unit $22
Fixed costs $504,000
Management plans to improve the quality of its sole product by (1) replacing a component that
costs $3.50 with a higher-grade unit that costs $5.50, and (2) acquiring a $180,000 packing
machine. Austin will depreciate the machine over a 10-year life with no estimated salvage value
by the straight-line method of depreciation. If the company wants to earn after-tax income of
$172,800 in the upcoming period, it must sell
Correct Answer: C.22.500 units.
A. 23,800 units.
B. 21,316 units.
C. 22,500 units.
D. 19,300 units.
Explanation:
A. This answer results from adding the full cost of the new machine and the desired after-tax
net income to the present fixed costs to calculate the numerator of the calculation of the
number of units to be sold. The full cost of the new machine should not be used, only one
year of depreciation expense on it should be used. And the after-tax net income needs to
be converted to before-tax net income before using it in the calculation.
B. This answer results from dividing the new fixed cost plus the required pre-tax net income
by the current unit contribution margin. The unit contribution margin will change because
of replacing the current component that costs $3.50 with a higher-grade unit that costs
$5.50.
C. Replacing the component with a higher priced component will add $2 to variable costs.
The new variable cost will be $24 per unit and the new contribution margin will be $60 -
$24, or $36. Acquiring the packing machine will add $18,000 in new depreciation
expense to fixed costs ($180,000 + 10 years life), so the new fixed cost will be $522,000.
In order to earn after-tax income of $172,800, pre-tax income needs to be $288,000
($172,800 + (1- 40%). Fixed costs of $522,000 + desired pre-tax income of $288,000
divided by the unit contribution margin of $36 = 22,500 units the company needs to sell
to earn the desired after-tax net income of $172,800.
D. This answer results from using the after-tax desired profit as part of the numerator in the
calculation of the number of units of sales required. The after-tax desired profit needs to
be converted to before-tax profit before using it in the calculation.

Question #81: Moorehead Manufacturing Company produces two products for which the
following data have been tabulated. Fixed manufacturing cost is applied at a rate of $1.00 per
machine hour.
Per Unit XY-7 BD-4
Selling price $4.00 $3.00
Variable manufacturing cost $2.00 $1.50
Fixed manufacturing cost $0.75 $0.20
Variable selling cost $1.00 $1.00
The sales manager has had a $160,000 increase in the budget allotment for advertising and wants
to apply the money to the most profitable product. The products are not substitutes for one
another in the eyes of the company's customers.
Suppose Moorehead has only 100,000 machine hours that can be made available to produce
additional units of XY-7 and BD-4. If the potential increase in sales units for either product
resulting from advertising is far in excess of this production capacity, which product should be
advertised and what is the estimated increase in contribution margin earned?
Correct Answer: D. Product BD-4 should be produced, yielding a contribution margin of
$250,000.
A. Product BD-4 should be produced, yielding a contribution margin of $187,500.
B. Product XY-7 should be produced, yielding a contribution margin of $75,000.
C. Product XY-7 should be produced, yielding a contribution margin of $133,333.
D. Product BD-4 should be produced, yielding a contribution margin of $250,000.
Explanation:
A. This is not the correct answer. Please see the correct answer for an explanation. We have
been unable to determine how to calculate this incorrect answer choice. If you have
calculated it, please let us know how you did it so we can create a full explanation of why
this answer choice is incorrect. Please send us an email at admin@cmapass.com. Include
the Topic Name, Question Details or Screenshot and the actual incorrect answer choice --
not its letter, because that can change with every study session created.. Thank you in
advance for helping us to make your CMA PASS study materials better.
B. This is not the correct answer. Please see the correct answer for an explanation. We have
been unable to determine how to calculate this incorrect answer choice. If you have
calculated it, please let us know how you did it so we can create a full explanation of why
this answer choice is incorrect. Please send us an email at admin@cmapass.com. Include
the Topic Name, Question Details or Screenshot and the actual incorrect answer choice --
not its letter, because that can change with every study session created.. Thank you in
advance for helping us to make your CMA PASS study materials better.
C. This is not the correct answer. Please see the correct answer for an explanation. We have
been unable to determine how to calculate this incorrect answer choice. If you have
calculated it, please let us know how you did it so we can create a full explanation of why
this answer choice is incorrect. Please send us an email at admin@cmapass.com. Include
the Topic Name, Question Details or Screenshot and the actual incorrect answer choice --
not its letter, because that can change with every study session created.. Thank you in
advance for helping us to make your CMA PASS study materials better.
D. In this problem it is necessary to optimize the contribution margin of a scarce resource.
The scarce resource in this problem is machine hours. XY-7 BD-4 Sales price $4.00
$3.00 Variable costs $3.00 $2.50 Contribution margin $1.00 $0.50
Fixed manufacturing costs are applied based on machine hours. Therefore, the machine hours
required to produce one unit of product XY-7 are 0.75 ($0.75 + $1.00) and 0.20 for product BD-
4 ($0.20 + $1.00). This means that the CM per machine hour for XY-7 is $1.33 ($1.00 + 0.75),
and the CM per machine hour for BD-4 is $2.50 ($0.50 +0.20). Since BD-4 has the highest
contribution margin per machine hours used and since the potential increase in sales units
resulting from the advertising is far in excess of this production capacity, we know that all that
Moorhead can produce can be sold. Therefore, only BD-4 should be produced. Since 100,000
machine hours are available and each unit of BD-4 requires 0.20 hour to produce, the company
can produce 500,000 units (100,000 x 0.20). The total contribution margin will be $250,000
($0.50 UCM x 500,000 units sold). It is always best to optimize the contribution margin of the
scarce resource.

Question #82: Sands, Inc. uses a calendar year for financial reporting. The company is
authorized to issue 5,000.000 shares of $10 par common stock. At no time has Sands issued any
potentially dilutive securities. Listed below is a summary of Sands' common stock activities.
Number of common shares issued and outstanding at Dec. 31, 20X6:1,000,000 Shares issued as a
result of a 10% stock dividend on Sept. 30, 20X7:100,000 Shares issued for cash on March 31,
20X8:1.000,000 Number of common shares issued and outstanding at Dec. 31, 20X8:2,100,000
A two-for-one stock split of Sands' common stock took place on March 31, 20X9. The weighted-
average number of common shares used in computing earnings per common share for 20X8 on
the 20X9 comparative income statement was:
Correct Answer: D. 3.700.000.
A. 2,100,000.
B. 1,600,000.
C. 1,850,000.
D. 3,700,000.
Explanation:
A. This answer assumes that the shares that were issued in 20X8 were outstanding for the
entire period. In fact, they were outstanding for only 9 months and therefore should be
considered to be only 750,000 (1,000,000 x 9/12) shares for the purposes of the
weighted-average number of common shares outstanding calculation.
B. This answer incorrectly assumes that the shares issued on March 31. 20X8 were
outstanding for only 6 months of the year, instead of 9 months.
C. At the end of 20X7 there were 1,100,000 shares outstanding. To this, the company needs
to add the effect of any 20X8 share transactions. The only transaction was the issuance of
1,000,000 shares on March 31. Since these shares were outstanding for only 9 months of
the year, they will count as only 75% for the weighted-average share calculation. These
shares will become 750,000 shares for this calculation, giving the company 1,850,000
shares outstanding for the 20X8 EPS calculation.
D. This answer includes the effect of the 2-for-1 stock split that occurred on March 31.
20X9. However, the question asks for the weighted-average number of common shares
outstanding for 20X8 on the 20X8 comparative income statement. The stock split did not
take place until after the 20X8 comparative income statement was issued.

Question #83: Lark Industries accepted a contract to provide 30.0x00 units of Product A and
20.000 units of Product B. Lark's staff developed the following information with regard to
meeting this contract.
Product Product
A B Total
Selling Price $75 $125
Variable Costs $30 $48
$1,600,00
Fixed overhead 0
Machine hours
required 3 5
Machine hours available 160,000
Cost if Outsourced $45 $60
Lark's operations manager has identified the following alternatives. Which alternative should be
recommended to Lark's management?
Correct Answer: • C. Make 30.000 units of Product A. utilize the remaining capacity to make
Product B. and outsource the remainder.
A. Make 25,000 units of Product A. utilize the remaining capacity to make Product B. and
outsource the remainder.
B. Make 20,000 units of Product A. utilize the remaining capacity to make Product B. and
outsource the remainder.
C. Make 30,000 units of Product A. utilize the remaining capacity to make Product B. and
outsource the remainder.
D. Rent additional capacity of 30,000 machine hours which will increase fixed costs by
$150,000.
Explanation:
A. This is a comparison between the cost to manufacture versus the cost to purchase outside.
Since the revenue to be received and the $1,600,000 fixed manufacturing costs will be the
same regardless of the decision that is made, those factors are irrelevant to this decision.
In this case, the contribution margin earned per machine hour is also not relevant.
Looking only at the contribution margin per machine hour for each product provides an
incomplete analysis since it does not include comparative information on the cost to
outsource. We need to find the least cost alternative for acquiring or producing this order.
If Lark chooses this alternative, its total variable cost for the order will be $1,971,000, as
follows: 25,000 units of A x 3 hours per unit = 75,000 hours, leaving 85,000 hours
(160,000 hours – 75,000 hours) available, which could be used to manufacture 17,000
units of B (85,000 hours ÷ 5 hours per unit). The total variable cost of the products
manufactured will be (25,000 x $30) + (17,000 x $48) = $1,566,000.
Outsourced units will be 5,000 units of A at a cost of $45 per unit and 3,000 units of B at
a cost of $60 per unit, or (5,000 x $45) + (3,000 x $60) = $405,000.
Total variable cost will be $1,566,000 + $405,000 = $1.971.000.
This is not the lowest-cost alternative. Please see the correct answer choice for a full
explanation of the process of making this decision.
B. This is a comparison between the cost to manufacture versus the cost to purchase outside.
Since the revenue to be received and the $1,600,000 fixed manufacturing costs will be the
same regardless of the decision that is made, those factors are irrelevant to this decision.
In this case, the contribution margin earned per machine hour is also not relevant.
Looking only at the contribution margin per machine hour for each product provides an
incomplete analysis since it does not include comparative information on the cost to
outsource. We need to find the least cost alternative for acquiring or producing this order.
If Lark chooses this alternative, its total variable cost for the order will be $2,010,000. as
follows: 20,000 units of A x 3 hours per unit = 60,000 hours, leaving 100,000 hours
(160,000 hours - 60,000 hours) available, which could be used to manufacture all 20,000
units of B required (100,000 hours ÷ 5 hours per unit). The total variable cost of the
products manufactured will be (20,000 x $30) + (20,000 x $48) = $1,560,000.
Outsourced units will be 10,000 units of A at a cost of $45 per unit, or $450,000. Total
variable cost will be $1,560,000 + $450,000 = $2,010,000. This is not the lowest-cost
alternative. Please see the correct answer choice for a full explanation of the process of
making this decision.
C. This is a comparison between the cost to manufacture versus the cost to purchase outside.
The revenue to be received and the $1,600,000 of fixed overhead will be the same
whether the product is manufactured or outsourced, so those factors are irrelevant to this
decision. In this case, the contribution margin earned per machine hour is also not
relevant. Looking only at the contribution margin per machine hour for each product
provides an incomplete analysis since it does not include comparative information on the
cost to outsource. We need to find the least cost alternative for acquiring or producing
this order. We can narrow the choices by comparing the variable cost per unit to
manufacture each product with the cost to outsource the product to find the savings
available by manufacturing each product. The product with the greatest variable cost
savings available by manufacturing internally is the product that should receive priority
for manufacturing internally.
Product A:
The variable cost per unit to manufacture is: $30
The variable cost per unit to outsource is: $45
The variable cost savings by manufacturing internally is: $15 per unit.
Product B:
The variable cost per unit to manufacture is: $48
The variable cost per unit to outsource is: $60
The variable cost savings by manufacturing internally is: $12 per unit.
Since the variable cost savings from manufacturing Product A internally is $15 while the
variable cost savings from manufacturing Product B internally is only $12, the company
should utilize its limited facilities to make as much as possible of Product A, first. The
amount needed of Product A is 30,000 units, and those will require 3 hours each, or
90,000 hours. That is well within the maximum number of machine hours available, so
the company should make all the units needed of Product A. That rules out the answer
choices that call for making only 25,000 and 20,000 units of A. However, we still need to
decide between renting the necessary capacity to manufacture the whole amount, or
making 30,000 units of A and as many units as possible of B and outsourcing the
remainder. To do that, we need to make a cost comparison of the incremental costs of
each answer choice. Rent additional capacity of 30,000 machine hours which will
increase fixed costs by $150,000: The total variable and incremental fixed cost would be
($30 x 30,000) + ($48 x 20,000) + $150,000 = $2,010,000.
Make 30,000 units of Product A, utilize the remaining capacity to make Product B. and
outsource the remainder: With 160,000 machine hours available, the company can use
90,000 of those hours to manufacture all of the 30.000 units needed of Product A (30.000
x 3 hours per unit). With the 70,000 remaining machine hours, they can manufacture
14,000 units of Product B (70,000 ÷ 5 hours per unit). The remaining 6,000 units needed
of Product B would be outsourced. The total variable cost would be ($30 x 30,000) +
($48 x 14,000) + ($60 x 6,000) = $1,932,000. Therefore, the least cost alternative is to
make 30,000 units of Product A, utilize the remaining capacity to make Product B, and
outsource the remainder.
D. This is a comparison between the cost to manufacture versus the cost to purchase outside.
Since the revenue to be received and the $1,600,000 fixed manufacturing costs will be the
same regardless of the decision that is made, those factors are irrelevant to this decision.
In this case, the contribution margin earned per machine hour is also not relevant.
Looking only at the contribution margin per machine hour for each product provides an
incomplete analysis since it does not include comparative information on the cost to
outsource. We need to find the least cost alternative for acquiring or producing this order.
If Lark chooses this alternative, the company will be able to manufacture the whole order
in-house. Its total variable and incremental fixed cost for the order will be $2,010,000, as
follows: The total variable cost of the products manufactured will be (30,000 x $30) +
(20,000 x $48) = $1,860,000. Incremental fixed cost for the rented capacity will be
$150,000. Total variable and incremental fixed cost will be $1,860,000 + $150,000 =
$2,010,000. This is not the lowest-cost alternative. Please see the correct answer choice
for a full explanation of the process of making this decision.

Question #84: A company has a current ratio of 2.0. Cash is 20%, accounts receivable is 40%,
and inventory is 40% of total current assets. What is the acid-test ratio for the company?
Correct Answer: B. 1.2.
A. 1.6.
B. 1.2.
C. 2.0.
D. 0.8.
Explanation:
A. This is Accounts Receivable + Inventory divided by Current Liabilities. The acid test (or
quick) ratio is Cash + Marketable Securities + Net Accounts Receivable divided by
Current Liabilities.
B. With a question like this, it is best to convert the percentages to currency a mounts. So
let's say cash is 20, accounts receivable is 40, inventory is 40, and total current assets
equal 100. Since the current ratio is 2.0, total current liabilities must be 50. The acid test
(or quick) ratio is Cash + Marketable Securities + Net Accounts Receivable divided by
Current Liabilities. Thus the acid test ratio is (20 + 0 + 40) + 50, or 1.2.
C. This is the current ratio. The acid test (or quick) ratio is Cash + Marketable Securities +
Net Accounts Receivable divided by Current Liabilities.
D. This is Accounts Receivable divided by Current Liabilities. The acid test (or quick) ratio
is Cash + Marketable Securities + Net Accounts Receivable divided by Current
Liabilities.

Question #85: Of the following, the working capital financing policy that would subject a firm to
the greatest level of risk is the one where the firm finances
Correct Answer: A. permanent current assets with short-term debt.
A. permanent current assets with short-term debt.
B. fluctuating current assets with short-term debt.
C. fluctuating current assets with long-term debt.
D. permanent current assets with long-term debt.
Explanation:
A. This question is about the maturity matching approach to financing. The maturity
matching approach to financing current assets (also called the hedging or the self-
liquidating approach) matches assets to be financed with financing having the same
maturity. Even though the assets being financed with short-term debt are current assets, if
they are permanent, they will not be liquidated the way other current assets would be.
Examples of permanent current assets are accounts receivable and inventory in a non-
seasonal business or in a growing business. Inventory is sold, but it is immediately
replaced with new inventory, so the level of investment in inventory remains the same.
Accounts receivable get collected, but they are immediately replaced by accounts
receivable for other sales, so the level of accounts receivable also remains the same. And
in a firm that is growing, accounts receivable and inventory don't just remain the same —
they increase. And they continue increasing until the firm's growth stops. In either a non-
seasonal business or in a growing business. the level of the accounts receivable and
inventory required to support sales will be permanent and they should be financed with
long-term debt (If the business is seasonal. at least a portion of their accounts receivable
and inventory will be liquidated after the selling season. so only a portion of their
accounts receivable and inventory will be permanent.) If a company finances permanent,
long-term assets with short-term debt, it may be able to lower its interest costs, if short-
term interest rates are lower than long-term fixed interest rates (which is usually the
case). However. it runs two risks: (1) of not being able to renew the short-term financing
when it matures and being required to pay it off when funds are not available because the
current assets have not been liquidated, potentially putting the firm into bankruptcy: and
(2) of being forced to renew the short-term debt at a higher interest rate in a period of
rising interest rates. If short-term interest rates increase enough, the firm may find itself
paying a higher rate of interest than it would have paid if it had originally financed the
permanent assets using long-term, fixed rate financing. Because of the risks involved,
using short-term financing for long-term, permanent assets is considered an aggressive,
risky approach to financing. The greater the proportion of permanent assets financed with
short-term debt, the more aggressive the financing is and the greater the level of risk the
firm faces.
B. This question is about the maturity matching approach to financing. The maturity
matching approach to financing current assets (also called the hedging or the self-
liquidating approach) matches assets to be financed with financing having the same
maturity. It is appropriate to finance fluctuating current assets with short-term debt. A
financing policy like this enables a firm to borrow only what it needs for only as long as
it needs it. This minimizes its interest costs because it does not have borrowed funds on
hand that it is not using.
C. This question is about the maturity matching approach to financing. The maturity
matching approach to financing current assets (also called the hedging or the self-
liquidating approach) matches assets to be financed with financing having the same
maturity. It is not appropriate to finance fluctuating current assets with long-term debt
because the firm could find at times that not all of its long-term borrowings are required
to finance the level of current assets. The result will be that the firm will be paying
interest on loan balances that it does not need. If the firm chooses to pay down the long-
term loan principal during a period when it does not need all of the borrowings, the next
time the current assets increase and the firm needs the financing for them, the financing
may not be there. However, this question asks for the working capital financing policy
that would subject a firm to the greatest level of risk. This policy is not the riskiest of the
answer choices and thus it is not the best answer choice.
D. It is appropriate to finance permanent current assets with long-term debt. This question is
about the maturity matching approach to financing. The maturity matching approach to
financing current assets (also called the hedging or the self-liquidating approach) matches
assets to be financed with financing having the same maturity. Examples of permanent
current assets are accounts receivable and inventory in a non-seasonal business or in a
growing business. Inventory is sold, but it is immediately replaced with new inventory, so
the level of investment in inventory remains the same. Accounts receivable get collected,
but they are immediately replaced by accounts receivable for other sales, so the level of
accounts receivable also remains the same. And in a firm that is growing, accounts
receivable and inventory don't just remain the same — they increase. And they continue
increasing until the firm's growth stops. In either a non-seasonal business or in a growing
business, the level of the accounts receivable and inventory required to support sales will
be permanent. (If the business is seasonal, at least a portion of their accounts receivable
and inventory will be liquidated after the selling season, so only a portion of their
accounts receivable and inventory will be permanent.)

Question #86: In foreign currency markets. the phrase "managed float" refers to the
Correct Answer: • A Discretionary buying and selling of currencies by central banks.
A. Discretionary buying and selling of currencies by central banks.
B. Tendency for most currencies to depreciate in value.
C. Necessity of maintaining a highly liquid asset, such as gold, to conduct international
trade.
D. Fact that actual exchange rates are set by private business people in trading nations.
Explanation:
A. Under the managed float system, the exchange rates are determined by the market, but
the government also sells and buys currency through its central bank in order to influence
the exchange rate.
B. Under the managed float system, the exchange rates are determined by the market, but
the government also sells and buys currency in order to influence the exchange rate. This
does not mean that most currencies will depreciate.
C. Under the managed float system, the exchange rates are determined by the market, but
the government also sells and buys currency in order to influence the exchange rate. This
does not require a nation to maintain highly liquid assets.
D. Under the managed float system, the exchange rates are determined by the market, but
the government also sells and buys currency in order to influence the exchange rate.

Question #87: In a make versus buy decision. the relevant costs include variable manufacturing
costs as well as
Correct Answer: • A avoidable fixed costs.
A. avoidable fixed costs.
B. general office costs.
C. factory management costs.
D. depreciation costs.
Explanation:
A. In the decision to make-versus-buy decision you want to look only at relevant costs that
differ among the options. Relevant costs include both avoidable fixed and variable costs.
B. General office costs will generally not differ among the options, therefore they are not
relevant.
C. Factory management costs will generally not differ among the options, therefore they are
not relevant.
D. Depreciation costs will generally not differ among the options, therefore they are not
relevant.

Question #88: In a decision analysis situation, which one of the following costs is generally not
relevant to the decision?
Correct Answer: • B. Historical cost.
A. Differential cost.
B. Historical cost
C. Incremental cost.
D. Avoidable cost.
Explanation:
A. Differential costs are costs that differ between two alternatives. These costs are relevant
to the decision making process.
B. Historical costs are sunk costs that are not relevant to the decision making process. They
are costs that have already been incurred, and nothing can be done to change them no
matter what decision is made. In the decision making process you want to look only at
costs that differ among the related options.
C. Incremental costs are additional costs that result when production, or some other factor is
increased. These cost are relevant to the decision making process.
D. Avoidable costs are relevant to decision making process because they are costs that can
be avoided depending upon the decision that is made. In the decision making process we
need to look at all avoidable (variable and fixed) costs that will vary among the related
options.

Question #89: A firm with an 18% cost of capital is considering the following projects (on
January 1, year1):
January 1, Year 1 December 31, Year 5

Cash Outflow Cash inflow Project Internal


(00's Omitted) (00's Omitted) Rate of return
Project A $3,500 $7,400 16%
Project B 4,000 9,950 ?

Correct answer: D. 20%


A. 15%
B. 18%
C. 16%
D. 20%
Explanation:
A. The internal rate of return is the discount rate at which the present value of the expected
cash inflows from a project equals the present value of the expected cash outflows, or the
discount rate at which the net present value is zero. A positive NPV would result from
using a discount rate of 15%, so that cannot be the project's IRR.
B. 18% is the company's cost of capital, which is given in the question.
C. 16% is approximately the internal rate of return for project A. but the question asks for
the internal rate of return for project B.
D. The internal rate of return is the discount rate at which the present value of the expected
cash inflows from a project equals the present value of the expected cash outflows, or the
discount rate at which the net present value is zero. To determine the internal rate of
return from the information given, we need to first know what discount factor for five
years would result in a present value of $9,950 that is equal to $4,000. To arrive at that
factor, we divide $4,000 by $9,950, and we get 0.402. We then look along the line of
factors for five years on the factor table given to locate a factor close to 0.402. That is
0.4019, which is in the 20% column. Thus, the internal rate of return is closest to 20%.

Question #90: Valley Inc. uses 400 lbs. of a rare isotope per year. The isotope costs $500 per lb.,
but the supplier is offering a quantity discount of 2% for order sizes between 30 and 79 lbs., and
a 6% discount for order sizes of 80 lbs. or more. The ordering costs are $200. Carrying costs are
$100 per year per lb. of material and are not affected by the discounts. If the purchasing manager
places eight orders of 50 lbs. each, the total cost of ordering and carrying inventory, including
discounts lost, will be
Correct Answer: • C. $12,100.
A. $1,600.
B. $4,100.
C. $12,100.
D. $6,600.
Explanation:
A. This is the ordering cost only. The total cost of ordering and carrying the inventory,
including discounts lost, will include the lost discount (from ordering at the 2% discount
level instead of at the 6% discount level), ordering costs and carrying costs.
B. This is the inventory ordering and carrying costs, but it does not include discounts lost.
The total cost of ordering and carrying the inventory, including discounts lost, will
include the lost discount (from ordering at the 2% discount level instead of at the 6%
discount level), ordering costs and carrying costs.
C. The total cost will include the lost discount (from ordering at the 2% discount level
instead of at the 6% discount level), ordering costs and carrying costs. Lost discount cost:
The company purchases 400 pounds at $500 per pound per year, at a total cost of
$200,000 (400 x $500). The cost of the lost discounts is the difference between 6% and
2% of $200,000, i.e., 4% multiplied by $200,000. $200,000 x 0.04 = $8,000. Ordering
cost: $200 per order x 8 orders per year = $1,600. Carrying cost: Each order consists of
50 pounds. It is assumed Valley Inc. will use those 50 pounds down to zero and then
receive another order of 50. We must also assume they will use the amount in each order
at a steady rate. Therefore, the average number of pounds in inventory throughout the
year will be the average of 50 and 0, which is (50 + 0) / 2, or 25 pounds. The carrying
cost of $100 per pound is multiplied by these 25 pounds, and the cost is $2,500. The total
cost will be $8,000+ $1,600 + $2,500 = $12,100.
D. This is the inventory ordering costs plus carrying costs, but the carrying costs are
calculated incorrectly. They are calculated using the number of units ordered each time
an order is placed. Furthermore, this answer does not include discounts lost. The
inventory carrying costs should be calculated using the average number of units in
inventory throughout the year, not the amount ordered each time an order is placed.

Question #91: If an entity's books of account are not maintained in its functional currency, the
FASB Accounting Standards Codification® requires re-measurement into the functional
currency prior to the translation process. An item that should be re-measured by use of the
current exchange rate is
Correct Answer: • an investment in bonds to be held until maturity
A. a plant asset and the associated accumulated depreciation.
B. inventories
C. the revenue from a long-term construction contract.
D. an investment in bonds to be held until maturity
Explanation:
A. Plant assets are nonmonetary assets and are therefore remeasured at the historical rates.
B. Inventories are nonmonetary items. Nonmonetary items are remeasured at the historical
exchange rate in effect when each transaction occurred.
C. Revenue items are usually remeasured at the historical rate that was in effect on the date
of the transaction.
D. An investment in bonds is a monetary asset and it is remeasured at the current exchange
rate. Most monetary assets are remeasured using the current exchange rate.

Question #92: Gleason Co. has two products, a frozen dessert and ready-to-bake breakfast rolls,
ready for introduction. However, plant capacity is limited, and only one product can be
introduced at present. Therefore, Gleason has conducted a market study, at a cost of $26,000, to
determine which product will be more profitable. The results of the study follow.
Sales of Desserts at $1.80/unit Sales of rolls at $1.20/unit

Volume Probability Volume Probability


250,000 0.30 200,000 0.20
300,000 0.40 250,000 0.50
350,000 0.20 300,000 0.20
400,000 0.10 350,000 0.10
The costs associated with the two products have been estimated by Gleason% cost accounting
department and are shown as follows.

Dessert Rolls
Ingredients per unit $0.40 $0.25
Direct labor per unit 0.35 0.30
Variable overhead per
unit 0.40 0.20
Production tooling* 48,000 25,000
Advertising 30,000 20,000

'Gleason treats production tooling as a current operating expense rather than capitalizing it as a
fixed asset.
According to Gleason's market study, the expected value of the sales volume of the breakfast
rolls is
Correct Answer: B. 260,000 units.
A. Some amount other than those given.
B. 260,000 units.
C. 275,000 units.
D. 125,000 units.
Explanation:
A. The correct answer is one of the answer choices given.
B. Expected sales volume is a weighted average of the possible sales volume, weighted
according to their probabilities. To find the expected sales volume, multiply each
estimated sales volume by its probability of occurring and sum the results.
(200,000 x 0.2) + (250,000 x 0.5) + (300,000 x 0.2) + (350,000 x 0.1) = 260,000.
Expected sales volume is 260,000 units.
C. This calculation represents the average of the possible sales volumes of the rolls. The
expected value is a weighted average with the various probabilities of each sales volume
as the weights.
D. The answer of 125,000 was calculated by using the estimated sales volume of 250,000
times its probability of 0.5. This is not expected value of the sales volume of rolls.

Question #93: Frasier Products has been growing at a rate of 10% per year and expects this
growth to continue and produce earnings per share of $4.00 next year. The firm has a dividend
payout ratio of 35% and a beta value of 1.25. If the risk-free rate is 7% and the return on the
market is 15%, what is the expected current market value of Frasier's common stock?
Correct Answer: A. $20.00.
A. $20.00.
B. $28.00.
C. $16.00.
D. $14.00.
Explanation:
A. This requires the use of both the Capital Asset Pricing Model and the Dividend Growth
Model. First, we use the CAPM to find the investors' required rate of return:
r = r F + I3(r M - r F )
Where:
r F = the risk-free rate
r M = the market rate of return
β= the stocks beta coefficient
Plugging the numbers into the formula, we have:
r = 0.07 + 1.25(0.15 - 0.07) = 0.17
Next, we take the investors' required rate of return and use it as the r in the Dividend
Growth Model. stated to solve for P 0 . to find the expected current market value of a
share of stock:
d1 P 0 = r- g
Where:
P 0 = the fair value today of a share of stock
d 1= the next annual dividend to be paid
r = the investors' required rate of return
g = the expected growth rate of the dividend
Earnings per share next year are expected to be $4, and the dividend payout ratio is 0.35.
Therefore, the next dividend will be $4 x 0.35, or $1.40.
Plugging these numbers into the Dividend Growth Model, we get
P 0 = $1.40 / (0.17 - 0.10) = $20
B. This answer results from using the return on the market minus the growth rate in the
denominator of the Dividend Growth Model. However, the denominator of the Dividend
Growth Model is r - g, where r = the investors' required rate of return and g = the growth
rate.
C. This answer could result from using 10% minus the beta coefficient in the denominator of
the Dividend Growth Model. However, the denominator of the Dividend Growth Model
is r - g, where r = the investors' required rate of return and g = the growth rate.
D. This answer results from using just the growth rate in the denominator of the Dividend
Growth Model. However, the denominator of the Dividend Growth Model is r - g, where
r = the investors' required rate of return and g = the growth rate.

Question #94: When compared to a debt-to-assets ratio, a debt to equity ratio would
Correct Answer: D. Be higher than the debt to assets ratio.
A. Be about the same as the debt to assets ratio.
B. Have no relationship at all to the debt to assets ratio.
C. Be lower than the debt to assets ratio.
D. Be higher than the debt to assets ratio.
Explanation:
A. These two ratios could not be the same because the assets and equity of a company will
be different from each other (except in the unusual situation in which the company has no
debt).
B. There is a relationship between the magnitude of a company's debt-to-equity ratio and the
magnitude of the same company' debt-to-assets ratio. See the correct answer for a
complete explanation.
C. Debt to equity is total liabilities divided by total equity. Debt to assets is total liabilities
divided by total assets. Because the asset balance of a company is higher than its equity. a
debt-to-equity ratio will be higher than a debt-to-asset ratio because when equity is used
in the denominator, the denominator will be smaller than when assets are used in the
denominator.
D. Debt to equity is total liabilities divided by total equity. Debt to assets is total liabilities
divided by total assets. Because the asset balance of a company is higher than its equity. a
debt-to-equity ratio will be higher than a debt-to-asset ratio because when equity is used
in the denominator, the denominator will be smaller than when assets are used in the
denominator. We know that the assets of the company must be higher than the equity of
the company because of the accounting equation of assets = liabilities + equity.

Question #95: Jorelle Company's financial staff has been requested to review a proposed
investment in new capital equipment. Applicable financial data is presented below. There will be
no salvage value at the end of the investment's life and, due to realistic depreciation practices, it
is estimated that the salvage value and net book value are equal at the end of each year. All cash
flows are assumed to take place at the end of each year. For investment proposals. Jorelle uses a
12% after-tax target rate of return.
Investment proposal
Annual
Cost and book Annual net after-tax Cash net
Year Value flow income
0 $250,000 $0 $0
1 168,000 120,000 35,000
2 100,000 108,000 39,000
3 50,000 96,000 43,000
4 18,000 84,000 47,000
5 0 72,000 51,000
Discount factors for a 12% Rate of return:
Present value of $1 present value of annuity of $1 received at end of each
Year Received at end of period period
1 0.89 .89
2 0.80 1.69
3 0.71 2.40
4 0.64 3.04
5 0.57 3.61
6 0.51 4.12

The traditional payback period for the investment proposal is

Correct Answer: • A. 2.23 years.


A. 2.23 years.
B. 1.65 years.
C. 2.83 years.
D. Over 5 years.
Explanation:
A. When cash flows are not constant over the life of the project, we must find the
cumulative cash inflows for each year to determine when the inflows will equal the
outflows.
The cash flows are as follows: Year 0 Year 1 Year 2 Year 3 Year 4 Year 5
Net initial investment (250,000)
After-tax cash flows from operations 120,000 108,000 96,000 84,000 72,000
Cumulative cash flows (250,000) (130,000) (22,000) 74,000 158,000 230,000
The cumulative cash flow from the project becomes positive sometime during year 3. If the cash
flows are assumed to occur evenly throughout the year, the exact payback period is 2.23 years.
calculated as follows:
Number of the project year in the final year when cash flow is negative: 2
Plus: a fraction consisting of
Numerator = the positive value of the negative cumulative inflow amount from the final negative
year: 22,000
Denominator = cash flow for the following year: 96,000
OR: 2 + (22.000/96,000) = 2.23
The initial investment will be recouped after 2.23 years.
B. An answer of 1.65 years results from adding together the annual after-tax cash flow and
the annual net income for each year and using the total to determine the payback period.
However, the payback period should be determined using annual after-tax cash flow only.
C. An answer of 2.83 years results from using the discounted annual after-tax cash flows to
determine the payback period. However, the traditional payback period uses
undiscounted cash flows to calculate the payback period.
D. An answer of "over 5 years" could result from using the annual net income amounts
instead of the annual after-tax cash flows to calculate the payback period.

Question #96: The president of Reading Manufacturing. Inc. is establishing performance goals
for each of the company's manufacturing plants. The data below represent prior year results for
one of the plants. manufacturing plants. The data below represent prior year results for one of the
plants.
Revenue $ 400,000 Variable costs 100.000 Fixed costs 200,000 Average assets 1.000.000
Average liabilities 200,000
The plant's return on assets is
Correct Answer: C.10.0%
A. 12.5%
B. 37.5%
C. 10.0%
D. 30.0%
Explanation:
A. Return on assets is net income divided by average total assets. This answer uses average
net assets (assets minus liabilities. which is stockholders' equity).
B. Return on assets is net income divided by average total assets. This answer does not
include the fixed costs in the calculation of net income. Also, this answer uses average
net assets (assets minus liabilities. which is stockholders' equity) instead of total assets.
C. Return on assets is net income divided by average total assets. Net income is 5400.000
revenues - $100.000 variable costs -5200.000 fixed costs = S100.000. $100.000 divided
by average assets of $1.000.000 = 0.10 or 10%.
D. Return on assets is net income divided by average total assets. This answer does not
include the fixed costs in the calculation of net income.

Question #97: Which statement below best indicates the order of the components that should be
implemented in a new Enterprise Risk Management program?
Correct Answer: • C. Set risk management strategy and objectives, identify risks, and assess
risks.
A. Communicate and monitor risks, identify risks, and treat risks.
B. Assess risks, control risks, and treat risks.
C. Set risk management strategy and objectives, identify risks, and assess risks.
D. Control risks, set risk management strategy and objectives, and monitor risks.
Explanation:
A. Information and communication involves communicating all the relevant information that
needs to be communicated to the appropriate people, within a time frame that will allow
them to carry out their duties. Information and communication cannot be implemented
for an ERM until the company has identified the risks, assessed the risks, and decided
how to respond to each identified risk and the system has been put into place.
B. Control activities are developed after the company has determined how it will respond to
each risk that has been identified and assessed.
C. The components of an Enterprise Risk Management program, in order of their
implementation are: -Objective setting. The organization's strategic objectives and goals
for its operations, reporting and compliance activities must be determined and
established. -Event identification. Events are the internal and external events that affect
the organization's implementation of its ERM strategy or the achievement of its
objectives. -Risk assessment. Risk assessment is the process of analyzing and considering
risks from three perspectives: (1) the likelihood of the risk's occurring. (2) the potential
impact of the event if it does occur, and (3) the interrelationship of the risks on a unit-by-
unit or total organization basis. -Risk response. Risk response is what the company
decides to do with respect to each of the risks identified. Management must develop a
response for each of its identified risks. -Control activities. Control activities are the
policies and procedures implemented to ensure the risk responses are carried out -
Information and communication. This includes all the relevant information that needs to
be communicated to the appropriate people, within a time frame that will allow them to
carry out their duties. -Monitoring. The system put in place needs to be monitored to
ensure it continues to be appropriate and continues to be properly operated.
D. The organization's strategic objectives and goals for its operations, reporting and
compliance activities must be determined and established before control activities or
monitoring can take place.

Question #98: Colonie Inc expects to report net income of at least $10 million annually for the
foreseeable future. Colonie could increase its return on equity by taking which of the following
actions with respect to its inventory turnover and the use of equity financing?
Correct Answer: • A Increase inventory turnover: decrease use of equity financing.
A. Increase inventory turnover: decrease use of equity financing.
B. Increase inventory turnover: increase use of equity financing.
C. Decrease inventory turnover: decrease use of equity financing.
D. Decrease inventory turnover: increase use of equity financing.
Explanation:
A. Return on equity is net income divided by average total equity. Increasing inventory
turnover will decrease inventory, thereby decreasing inventory holding costs and
increasing net income. Decreasing use of equity financing, for example by repurchasing
treasury stock, will decrease total equity. The combination of increased net income along
with decreased equity will lead to increased return on equity.
B. Return on equity is net income divided by average total equity. Increasing inventory
turnover will decrease inventory, thereby decreasing inventory holding costs and
increasing net income. This would increase return on equity if equity were unchanged.
However, increasing use of equity financing, for example by issuing new common stock,
will increase equity. Whether return on equity would be increased or decreased by this
combination would depend on the magnitude of the two changes, so it is not possible to
state that return on equity would be either increased or decreased based on the
information given here.
C. Return on equity is net income divided by average total equity. Decreasing inventory
turnover will increase inventory, increasing the inventory holding costs and decreasing
net income. This would decrease return on equity if equity were unchanged. However,
decreasing use of equity financing, for example by repurchasing treasury stock, will
decrease equity. Whether return on equity would be increased or decreased by this
combination would depend on the magnitude of the two changes, so it is not possible to
state that return on equity would be either increased or decreased based on the
information given here.
D. Return on equity is net income divided by average total equity. Decreasing inventory
turnover will increase inventory, thereby increasing inventory holding costs and
decreasing net income. Increasing use of equity financing, for example by issuing
additional common stock, will increase total equity. The combination of decreased net
income along with increased equity will lead to decreased return on equity.

Question #99: Regis Company, which is subject to an effective income tax rate of 30%, is
evaluating a proposed capital project. Relevant information for the proposed project is
summarized below.
Initial investment $500,000
Annual operating cash inflows
for the first three years:
Year 1 $185,000
Year 2 175,000
Year 3 152,000
Depreciation will be calculated under the straight-line method using an 8-year estimated service
life and a terminal value of 850,000. In determining the estimated total after-tax cash flow in
Year 2 of the project. Regis should consider the after-tax operating cash
Correct Answer: B. inflow plus annual depreciation tax shield.
A. inflow only.
B. inflow plus annual depreciation tax shield.
C. inflow plus annual depreciation expense.
D. inflow plus the net impact of the annual depreciation expense and depreciation tax shield.
Explanation:
A. The total after-tax cash flow for Year 2 of the project will include the net after-tax
operating cash flow, but that is not the only thing it will include.
B. The total after-tax cash flow for Year 2 of the project will consist of the net after-tax
operating cash flow plus the depreciation tax shield for the year. Depreciation expense is
a non-cash transaction that is not included in the after-tax operating cash flow, and it
should not be added back in. Depreciation expense is used only to calculate the amount of
the depreciation tax shield, which is a cash flow item.
C. Depreciation expense is a non-cash transaction that is not included in the after-tax
operating cash flow, and it should not be added back in. Depreciation expense is used
only to calculate the amount of the depreciation tax shield, which is a cash flow item.
D. Depreciation expense is a non-cash transaction that is not included in the after-tax
operating cash flow, and it should not be added back in. Depreciation expense is used
only to calculate the amount of the depreciation tax shield. which is a cash flow item.

Question #100: A bill of lading is a document that


Correct Answer: • D. Is used to transfer responsibility for goods between the seller of goods and
a common carrier.
A. Is sent with the goods giving a listing of the quantities of items included in the shipment
B. Summarizes data relating to a disbursement and represents final authorization for
payment.
C. Reduces a customer's account for goods returned to the seller.
D. Is used to transfer responsibility for goods between the seller of goods and a common
carrier.
Explanation:
A. The description given is that of a packing slip, not a bill of lading.
B. The description given is related to a payment order, not a bill of lading.
C. The description is given is that of a credit memo, not a bill of lading.
D. A bill of lading is a document that transfers possession of goods from the seller to a
common carrier.
Question #1: According to the IMA Statement of Ethical Professional Practice, one of the
overarching ethical principles is "Objectivity." "Objectivity" means
E. Doing what you say you will do when you say you will do it. •
F. Basing a judgment on an established set of criteria. •
G. Acting in an impartial manner and being free from dishonesty or injustice. •
H. Fairness and straightforwardness of conduct.
Correct answer: B. basing a judgment on an established set of criteria.
Explanation:
A. This is the definition of "Responsibility.”
B. "Objectivity" means basing a judgment on an established set of criteria. It is the state of being
free from personal feelings or prejudice and basing analyses and decisions on the facts alone.
C. This is the definition of "Fairness”
D. This is the definition of "Honesty."
Question #2: Accounts receivable turnover will normally decrease as a result of
E. A significant sales volume decrease near the end of the accounting period. •
F. A change in credit policy to lengthen the period for cash discounts. •
G. An increase in cash sales in proportion to credit sales.
H. The write-off of an uncollectible account (assume the use of the allowance for doubtful
accounts method).
Correct answer: B. A change in credit policy to lengthen the period for cash discounts.
Explanation:
A.Accounts receivable turnover is calculated as annual credit sales divided by the average
accounts receivable. The A/R turnover will decrease if there is a decrease in credit sales or an
increase in the average receivables or if both decrease but the decrease in credit sales is
proportionally greater than the decrease in average receivables.
A decrease in credit sales at the end of the period will decrease both the credit sales and the
receivables balance at the end of the period. Because annual credit sales is a larger amount than
average accounts receivable, the amount of decrease in sales will not be proportionally greater
than the amount of the decrease in average accounts receivable. Therefore, the accounts
receivable turnover will not normally decrease as a result of a significant sales volume decrease
near the end of the accounting period.
B. Accounts receivable turnover is calculated as annual credit sales divided by the average
accounts receivable. It measures the number of times the accounts receivable "turn over" during
a year's time. If the company lengthens the period for cash discounts, more customers will take
longer to pay their bills, which will increase the average receivables. This will, in turn, decrease
the accounts receivable turnover ratio. A decrease in the accounts receivable turnover ratio
means the accounts turn over less frequently; and in this case, that is because the level of
accounts receivable is higher.
C. Accounts receivable turnover is calculated as annual credit sales divided by the average
accounts receivable. Therefore, this number will decrease if there is a decrease in credit sales or
an increase in the average receivables. An increase in cash sales in proportion to credit sales may
or may not cause a decrease in the accounts receivable turnover ratio. It would depend upon the
magnitude of the change in both annual credit sales and average accounts receivable.
D. Accounts receivable turnover is calculated as annual credit sales divided by the average
accounts receivable. Therefore, this number will decrease if there is a decrease in credit sales or
an increase in the average receivables. Under the allowance for doubtful accounts method, the
writing off of a receivable will not effect the accounts receivable turnover, because the net
average accounts receivable is used in the accounts receivable turnover ratio (i.e., net of the
allowance for doubtful accounts). The net accounts receivable balance does not change when an
account is written off. Accounts receivable decreases, but the negative (credit) balance in the
allowance for doubtful accounts also decreases. The effect on the combination of the two
account balances is zero.

Question #3: If income tax considerations are ignored, how is depreciation handled by the
following capital budgeting techniques?

Internal Accounting
Rate of return Rate of return Payback
I Excluded Included Excluded
II Excluded Excluded Included
II Included Excluded Included
IV Included Included Included

E. IV.
F. I.
G. II.
H. III.
Correct answer: B. I.

Explanation: A. If income tax considerations are ignored, depreciation would be excluded from
the internal rate of return and payback calculations, because the IRR and the payback period are
based upon cash flows.
B. If income tax considerations are to be ignored, then the depreciation tax shield is ignored.
Therefore, the income tax savings from the depreciation are not included in the capital budgeting
analyses. If the income tax savings from the depreciation are excluded, then depreciation is
ignored in the calculations of internal rate of return and payback. However, depreciation is
included in the calculation of the accounting rate of return, because the accounting rate of return
is based upon book income, which includes depreciation.
C. If income tax considerations are ignored, depreciation would be included in the accounting
rate of return calculation, because the accounting rate of return is based upon book income,
which includes depreciation. Furthermore, depreciation would be excluded from the payback
period calculation, because the payback period calculation is based upon cash flows, not book
income.
D. If income tax considerations are ignored, depreciation would be excluded from the internal
rate of return and payback calculations, because the IRR and the payback period are based upon
cash flows. Depreciation would be included in the calculation of the accounting rate of return,
because the accounting rate of return is based upon book income, which includes depreciation.
Question #4: A company had total sales of $500,000 in the first quarter of the year, which was
the same amount as it recorded in the first quarter of the prior year. However, its accounts
receivable balance increased from $230,000 last year to $300,000 this year. Which one of the
following is the most likely explanation for the increase in the accounts receivable balance?
E. The company discontinued the use of factoring in the current year.
F. The company hired more people in its credit and collections department.
G. The company initiated the use of factoring in the current year.
H. The company shortened its payment terms in the current year from 60 days to 30 days.
Correct answer: A. The Company discontinued the use of factoring in the current year.

Explanation:
A. Discontinuing the use of factoring would cause the accounts receivable balance to increase
because the company would be carrying more of its own accounts receivable on its balance sheet
instead of selling the receivables.
B. Hiring more people in its credit and collections department would not cause the company's
accounts receivable to increase.
C. If the company had initiated the use of factoring in the current year, the accounts receivable
balance would be expected to decline, not increase.
D. Shortening its payment terms would not cause the company's accounts receivable to increase.

Question #5: A company uses cost-volume-profit analysis to evaluate a new product. The total
fixed costs of production per year are $160,000. The unit variable cost is $50. Which one of the
following combinations of unit selling price and breakeven number of units sold per year is
correct?
E. $100 selling price and 1,600 breakeven number of units.
F. $50 selling price and 3,200 breakeven number of units.
G. $25 selling price and 6,400 breakeven number of units.
H. $70 selling price and 8,000 breakeven number of units.
Correct answer: D. $70 selling price and 8,000 breakeven number of units.
Explanation:
A. This answer results from mistaking the contribution margin for the selling price. If the
contribution margin were $100, the breakeven number of units given $160.000 in fixed
production costs would be $160,000 ÷ $100 = 1,600 units. However, the question is asking for
the selling price and the breakeven number of units, not the contribution margin and the
breakeven number of units. The selling price is the contribution margin plus the unit variable
cost of $50; and the contribution margin is the selling price minus the unit variable cost of $50.
The breakeven number of units at a $100 selling price would be $160,000 divided by the
contribution margin of ($100 - $50), which equals 3,200 units. So this is not a true statement.
B. This answer results from mistaking the contribution margin for the selling price. If the
contribution margin were $50, the breakeven number of units given $160,000 in fixed production
costs would be $160.000 ÷ $50 = 3,200 units. However, the question is asking for the selling
price and the breakeven number of units, not the contribution margin and the breakeven number
of units. The selling price is the contribution margin plus the unit variable cost of $50; and the
contribution margin is the selling price minus the unit variable cost of $50. The breakeven
number of units at a $50 selling price does not exist. The unit contribution margin would be $50
- $50, or zero. There would be no contribution margin available to cover the fixed production
costs and no volume at which the company could break even.
C. This answer results from mistaking the contribution margin for the selling price. If the
contribution margin were $25, the breakeven number of units given $160,000 in fixed production
costs would be $160,000 ÷ $25 = 6,400 units. However, the question is asking for the selling
price and the breakeven number of units, not the contribution margin and the breakeven number
of units. The selling price is the contribution margin plus the unit variable cost of $50; and the
contribution margin is the selling price minus the unit variable cost of $50. The breakeven
number of units at a $25 selling price does not exist. The unit contribution margin would be $25
- $50, or a $25 loss on each unit sold. There would be no contribution margin available to cover
the fixed production costs and no volume at which the company could break even.
D. This is a true statement. With a selling price of $70, the contribution margin would be $70 -
$50 = $20. With fixed production costs of $160,000, the breakeven point would be $160,000 ÷
$20 = 8,000 units.

Question #6: A printing company is considering replacing an old printing press. The old printing
press has a book value of $24,000 and a trade-in value of $14,000. A new printing press would
cost $85,000 after trade-in of the old press. It is estimated that the new printing press would
reduce operating costs by $20,000 per year. If the company decides not to purchase the new
press, the $85,000 could instead be used to retire debt that is currently costing $9,000 per year in
interest. Which of the given amounts is an example of a sunk cost?
E. The trade-in value of the old printing press.
F. The estimated reduction in operating costs.
G. The interest on the existing debt
H. The book value of the old printing press.
Correct answer: D. The book value of the old printing press.
Explanation:
A. A sunk cost is a cost that has already been incurred and is not relevant to the decision process.
The trade-in value of the old printing press will affect the decision process, so therefore, is
relevant and is not a sunk cost.
B. A sunk cost is a cost that has already been incurred and is not relevant to the decision process.
The estimated reduction in operating expenses is the difference between keeping the old printing
press and purchasing the new one. It is relevant to the decision process and is not even a cost.
C. The interest on debt is an avoidable cost that is relevant to the decision process.
D. A sunk cost is a cost that has already been incurred and is not relevant to the decision process.
The amount paid for the old printing press (less accumulated depreciation) is an example of a
sunk cost.

Question #7: The degree of operating leverage (DOL) is


E. A measure of the change in earnings before interest and taxes (EBIT) resulting from a
given change in sales.
F. A measure of the change in earnings available to common stockholders associated with a
given change in operating earnings.
G. Higher if the degree of total leverage is lower, other things held constant.
H. Lower if the degree of total leverage is higher, other things held constant.
Correct answer: A. A measure of the change in earnings before interest and taxes (EBIT)
resulting from a given change in sales.

Explanation:
A. The degree of operating leverage (DOL) is a measure of the change in earnings before interest
and taxes associated with a given change in sales volume. For a particular level of output. Degree
of Operating Leverage is calculated as follows: % Change in EBIT % Change in Revenue
B. The degree of financial leverage is a measure of the change in earnings available to common
stockholders associated with a given change in operating earnings.
C. The degree of total leverage is the multiple of the degree of operating leverage and the degree
of financial leverage. Other things being equal. DOL is higher if the degree of total leverage is
higher.
D. The degree of total leverage is the multiple of the degree of operating leverage and the degree
of financial leverage. Other things being equal. DO L is higher if the degree of total leverage is
higher.
Question #8: Based on the assumptions of the Capital Asset Pricing Model, the risk premium on
an investment with a beta of 0.5 is equal to
E. Twice the risk premium on the market.
F. Half the risk premium on the market.
G. The risk-free rate.
H. The risk premium on the market.
Correct answer: B. half the risk premium on the market.
Explanation:
A. The risk premium for a particular security is its β (RM - RF), or the beta coefficient for that
particular security multiplied by the market risk premium. The beta coefficient for the market as
a whole is 1.0. If the beta coefficient for a given security is 0.5, the risk premium for that security
cannot be twice the risk premium for the market as a whole.
B. The risk premium for a particular security is its β (RM - RF), or the beta coefficient for that
particular security multiplied by the market risk premium. The beta coefficient for the market as
a whole is 1.0. If the beta coefficient for a given security is 0.5, it means the risk premium for
that security is half the risk premium for the market as a whole.
C. The risk premium on an investment with a beta of 0.5 cannot be equal to the risk-free rate, as
the risk-free rate is only one component in the calculation of the risk premium for an investment.
The risk premium for a particular security is its β (RM - RF), or the beta coefficient for that
particular security multiplied by the market risk premium.
D. The risk premium on an investment with a beta of 0.5 cannot be equal to the risk premium on
the market, as the risk premium on the market is a component in the calculation of the risk
premium for an investment. The risk premium for a particular security is its β (RM - RF), or the
beta coefficient for that particular security multiplied by the market risk premium.

Question #9: Douglas Company purchased 10.000 shares of its common stock at the beginning
of the year for cash. This transaction will affect all of the following except the
E. Net profit margin.
F. Debt-to-equity ratio.
G. Current ratio.
H. Earnings per share.
Correct answer: A. net profit margin
Explanation:
A. When a company repurchases shares of its own outstanding stock, the stock that is
repurchased is called treasury stock. Purchase of treasury stock reduces cash and equity on the
balance sheet. It does not affect any items on the income statement and thus, net profit margin is
not affected.
B. When a company repurchases shares of its own outstanding stock, the stock that is
repurchased is called treasury stock and its purchase is recorded as a reduction in equity. Since
equity is reduced, the debt-to-equity ratio will be increased.
C. When a company repurchases shares of its own outstanding stock, the stock that is
repurchased is called treasury stock. The purchase reduces cash, which in turn reduces current
assets and the current ratio.
D. When a company repurchases shares of its own outstanding stock, the stock that is
repurchased is called treasury stock. The repurchase reduces the number of outstanding shares of
the company's common stock. Since the number of outstanding shares of common stock is
reduced, earnings per share will be increased.

Question #10: Please see the Question Below:


Gordon has had the following financial results for the last four years.

Year 1 Year 2 Year 3 Year 4


Sales $1,250,000 $1,300,000 $1,359,000 $1,400,000
Cost of goods sold 750,000 785,000 825,000 850,000
Gross profit $500,000 $515,000 $534,000 $550,000

Inflation factor 1.00 1.03 1.07 1.10

Gordon has analyzed these results using vertical common-size analysis to determine trends. The
performance of Gordon can best be characterized by which one of the following statements?
E. The common-size gross profit percentage has decreased as a result of an increasing
common-size trend in cost of goods sold.
F. The common-size trend in cost of goods sold is decreasing which is resulting in an
increasing trend in the common-size gross profit margin.
G. The increased trend in the common-size gross profit percentage is the result of both the
increasing trend in sales and the decreasing trend in cost of goods sold.
H. The common-size trend in sales is increasing and is resulting in an increasing trend in the
common-size gross profit margin.
Correct answer: A. The common-size gross profit percentage has decreased as a result of an
increasing common-size trend in cost of goods sold.

Explanation:
To solve this, we need to calculate the percentage of sales represented by COGS and Gross Profit
for each of the years and then analyze the trends. Year 1 Year 2 Year 3 Year 4 Sales$1,250,000
100.0% $1,300,000 100.0% $1,359,000 100.0% $1,400,000 100.0% COGS 750,000 60.0%
785,000 60.4% 825,000 60.7% 850,000 60.7% Gr. profit$ 500,000 40.0% $ 515,000 39.6% $
534,000 39.3% $ 550,000 39.3%
When we look at the trend, it is easy to see that the common-size gross profit percentage has
decreased as a result of an increasing common-size trend in cost of goods sold. None of the other
answers is a true statement.
B. The trend in cost of goods sold is not decreasing.
C. The gross profit percentage is not increasing.
D. Although the gross profit is increasing, the gross profit margin is not increasing.

Question #11: When using the net present value method for capital budgeting analysis, the
required rate of return is called all of the following except the
E. Risk-free rate.
F. Cost of capital.
G. Hurdle rate.
H. Discount rate.
Correct answer: A. Risk free rate.
Explanation:
A. The required rate of return, which is the rate used to discount future cash flows in a capital
budgeting analysis, is not the risk-free rate. There is risk inherent in all capital budgeting
projects, and the required rate of return incorporates a risk premium.
B. The required rate of return may be equal to the firm's cost of capital, if the firm has not seen
fit to adjust its cost of capital to reflect higher or lower risk.
C. The required rate of return may be called the "hurdle rate" because it is the minimum rate of
return that is acceptable for an investment. A firm should invest money in a project only if the
project provides a higher rate of return than this rate. Investments with a return higher than the
hurdle rate will increase the value of the firm and thus stockholders' wealth.
D. The required rate of return is the discount rate used in a capital budgeting analysis.

Question #12: Which of the following represents a firm's average gross receivables balance?
IV. Days' sales in receivables x accounts receivable turnover.
V. Average daily credit sales x average collection period.
VI. Net sales + average gross receivables.
E. II only.
F. II and III only.
G. I only.
H. I and II only.
Correct answer: A. II only.
Explanation:
E. This is one of the ways in which the average receivables balance may be calculated.
Multiplying the average credit sales each day by the average number of days receivables
are outstanding provides the average receivables balance.
F. This answer cannot be correct because item III includes the value that we are trying to
solve for.
G. Neither of these items are dollar amounts so they cannot together calculate the average
receivables balance.
H. Neither of the items in item I are dollar amounts so they cannot together calculate the
average receivables balance.

Question #13: Essential elements in the development of an organization's ethics policy include
all of the following except
E. Input from the board of directors in addition to management and employees.
F. Allowances for exceptional circumstances.
G. Relevance to day-to-day implementation.
H. Articulation of organizational values.
Correct answer: B. Allowances for exceptional circumstances.
Explanation:
A. Development of the ethics policy should have input from the board of directors because the
ultimate responsibility for ethical conduct lies with the board of directors. The board of directors
should oversee the development of the policy. Input should also be received from a cross-
functional group. In an international organization and where size permits, the group should
include international personnel. Groups of employees and others can help identify risks and help
in defining and reviewing the content of the policy.
B. Allowances for exceptional circumstances are not an essential element in the development of
an organization's ethics policy.
C. The ethics policy should have relevance to day-to-day implementation. While an ethics policy
cannot specifically cover every potential ethical dilemma, it should utilize the organization's
values and guidelines as the basis for ethical decision-making on a day-to-day basis.
D. An organization's values are at the heart of its ethics policy. The ethics policy should be
values-driven, so it should include articulation of organizational values.

Question #14: A 10% stock dividend most likely


E. Increases the size of the firm.
F. Increases shareholders' wealth.
G. Decreases future earnings per share.
H. Decreases net income.
Correct answer: C. Decreases future earnings per share
Explanation:
A. A stock dividend does not affect the size of the company.
B. A stock dividend does not affect the shareholders' wealth since there is no distribution of
assets of the company and each individual shareholders market value of their investment remains
unchanged after the stock dividend.
C.A stock dividend will increase the number of shares outstanding and this in turn will most
likely decrease earnings per share in the future since the profits of the company will need to be
divided among more shares.
D. A stock dividend does not impact the income of the company. It is accounted for entirely
within the owners' equity section of the balance sheet.

Question #15: Hobart Corporation evaluates capital projects using a variety of performance
screens; including a hurdle rate of 16%, payback period of 3 years or less, and an accounting rate
of return of 20% or more. Management is completing review of a project on the basis of the
following projections.
$200,00
Capital investment 0
Annual cash flows $74,000
Straight-line
depreciation 5 years
Terminal value $20,000

The projected internal rate of return is 20%. Which one of the following alternatives reflects the
appropriate conclusions for the indicated evaluative measures?
Internal rate of Paybac
return k
I Accept Reject
II Reject Reject
III Accept Accept
IV Reject Accept

E. IV
F. II
G. I
H. III
Correct answer: D. III
Explanation:
A. The IRR of 20% is higher than the hurdle rate of 16%, so on the basis of internal rate of
return, the project should be accepted. Please see the correct answer for a complete explanation.
B. This project is acceptable based on both its internal rate of return and its payback period.
Please see the correct answer for a complete explanation.
C. Since all of the annual cash flows are the same, the payback period can be calculated by
simply dividing the capital investment amount of $200,000 by the annual cash flow amount of
$74,000. The result is 2.7 years, which meets the required payback period of 3 years or less.
Therefore, the project is acceptable based on its payback period. Please see the correct answer
for a complete explanation.
D. This project is acceptable based on both its internal rate of return and its payback period. The
I RR of 20% is higher than the hurdle rate of 16%, so on the basis of internal rate of return, the
project should be accepted.
Since all of the annual cash flows are the same, the payback period can be calculated by simply
dividing this project is acceptable based on both its internal rate of return and its payback period.
The IRR of 20% is higher than the hurdle rate of 16%, so on the basis of internal rate of return,
the project should be accepted.

Question #16: Which of the following actions is not an appropriate course of action when
encountering problems identifying unethical behavior or resolving an ethical conflict, if your
organization's established policies do not resolve the ethical conflict?
E. Communicate your concerns to the authorities or to persons outside the organization who
may be interested parties.
F. Discuss the issue with your immediate supervisor except when it appears that the
supervisor is involved. In that case, present the issue to the next level. If you cannot
achieve a satisfactory resolution, submit the issue to the next management level.
G. Consult your own attorney regarding your legal obligations and rights concerning the
ethical conflict.
H. Clarify relevant ethical issues by initiating a confidential discussion with an IMA Ethics
Counselor or other impartial advisor to obtain a better understanding of possible courses
of action.
Correct answer: A. Communicate your concerns to the authorities or to persons outside the
organization who may be interested parties.
Explanation:
A. Communication of problems to authorities or individuals not employed or engaged by the
organization is not considered an appropriate means of resolving an ethical conflict, unless you
believe there is a clear violation of the law.
B. Discussing the issue with your immediate supervisor except when it appears that the
supervisor is involved is a course of action to consider.
C. Consulting your own attorney regarding your legal obligations and rights concerning the
ethical conflict is a course of action to consider.
D. Initiating a confidential discussion with an IMA Ethics Counselor or other impartial advisor is
a course of action to consider.

Question #17: A change from one generally accepted accounting principle to another generally
accepted accounting principle should be accounted for in comparative reports by
E. A line item below extraordinary items on the current income statement
F. Pro forma amounts for key figures shown supplementary on the income statement for all
periods presented.
G. A cumulative adjustment to carrying amounts of assets and liabilities as of the beginning
of the first period presented, an offsetting adjustment to the opening balance of retained
earnings of the same period, and by adjusting prior periods' statements presented for the
effects of the change in each period.
H. Only a footnote disclosure in the current period.
Correct answer: A cumulative adjustment to carrying amounts of assets and liabilities as of the
beginning of the first period presented, an offsetting adjustment to the opening balance of
retained earnings of the same period, and by adjusting prior periods' statements presented for the
effects of the change in each period.

Explanation:
A. No line item for any cumulative effect adjustment is to be used on the current or any prior
income statement.
B. Pro forma amounts are not to be used for reporting accounting changes.
C. A change of accounting principle is to be accounted for by adjusting carrying amounts of
assets and liabilities as of the beginning of the first period presented for the cumulative effect of
the change on periods prior to those presented in the financial statements. The effect of this prior
period adjustment is offset by adjusting the opening balance of retained earnings of the first
period presented. Financial statements for all periods presented are adjusted for the effects of the
change in each specific period, unless it is impracticable to do so.
D. The financial statements in the period the change is made must include disclosure of the
change, the reason for the change, and an explanation of why the company's management
believes the new accounting principle to be preferable from the perspective of financial reporting
(i.e.. not merely because it will result in favorable income tax consequences). However, footnote
disclosure is not the only requirement.

Question #18: Jones & Company is considering the acquisition of scanning equipment to
mechanize its procurement process. The equipment will require extensive testing and debugging,
as well as user training prior to its operational use. Projected after-tax cash flows are as follows.

Time period After-tax Cash


Year Inflow/(Outflow)
0 $(600,000)
1 $(500,000)
2 $450,000
3 $450,000
4 $350,000
5 $250,000

Management anticipates the equipment will be sold at the beginning of year 6 for $50.000 and its
book value is zero. Jones' internal hurdle and effective income tax rates are 14% and 40%,
respectively. Based on this information, a negative net present value was computed for the
project. Accordingly, it can be concluded that
E. The project has an internal rate of return (IRR) less than 14% since IRR is the interest
rate at which net present value is equal to zero.
F. Jones should examine the determinants of its hurdle rate further before analyzing any
other potential projects.
G. Jones should calculate the project payback to determine if it is consistent with the net
present value calculation.
H. The project has an I RR greater than 14% since IRR is the interest rate at which net
present value is equal to zero.
Correct answer: The project has an internal rate of return (IRR) less than 14% since IRR is
the interest rate at which net present value is equal to zero.

Explanation:
E. The IRR is the discount rate at which the net present value of the project is zero. If the
project's net present value computed using the company's hurdle rate of 14% is negative,
its I RR must be lower than 14%. Any project with a net present value of less than zero or
an IRR of less than the hurdle rate is not an acceptable project.
F. The purpose of the hurdle rate is to establish the minimum return the company expects
from a project. The fact that a project is unacceptable when that hurdle rate is used to
discount a project's future cash flows is not an indication that there is anything wrong
with the hurdle rate used. It is more likely an indication that the project does not meet the
company's requirements. Changing the company's requirements to make the project
acceptable is generally not the proper course of action.
G. The payback period method of capital budgeting should not be used to confirm a net
present value calculation for a project. The payback method can be helpful when the
company needs to recoup its investment quickly, perhaps because the project is in a
politically unstable area of the world or because it utilizes high technology that quickly
becomes obsolete. However, its usefulness is limited because it does not incorporate the
time value of money and it ignores the cost of capital and all cash flows beyond the
payback point.
H. The IRR is the discount rate at which the net present value of the project is zero. If the
project's net present value computed using the company's hurdle rate of 14% is negative,
its IRR must be lower than 14%. Any project with a net present value of less than zero or
an IRR of less than the hurdle rate is not an acceptable project.

Question #19: Which of the following methods can be used as a tool to identify process controls
related to ethical or behavioral issues?
E. Ratio analysis
F. Decision trees
G. Business process re-engineering
H. Sensitivity analysis
Correct answer: C. business process re-engineering.
Explanation:
E. This is not correct because ratio analysis is a financial analysis technique.
F. This is not correct because a decision tree is a method to determine the best course of
action in a decision situation based upon a range of possible courses of action, their
expected outcomes and their probabilities of occurring.
G. This is the correct answer because business process engineering involves understanding a
process and identifying activities in the process, including internal controls, which impact
ethical behavior.
H. This is not correct because sensitivity analysis is a technique to analyze how the outcome
of a decision would be impacted by changes in the variables that lead to the outcome.

Question #20: Allred Company sells its single product for $30 per unit. The contribution margin
ratio is 45%, and fixed costs are $10,000 per month. Allred has an effective income tax rate of
40%. If All read sells 1.000 units in the current month. Allred's variable expenses would be
E. $13,500.
F. $16,500.
G. $12,000.
H. $9,900.
Correct answer: $16,500.
Explanation:
E. This is the contribution margin, not the variable costs. Revenue minus variable costs
equals the contribution margin. Therefore, revenue minus the contribution margin would
equal variable costs.
F. Revenue for sales of 1,000 units would be $30 x 1,000, or $30,000. The contribution
margin is total revenue minus total variable costs. If the contribution margin ratio is 45%
of revenue, the variable costs must be 100% - 45%, or 55% of revenue. That is $30,000 x
0.55, or $16,500.
G. This is total revenue multiplied by the tax rate. That is not variable expenses.
H. This is the variable costs multiplied by (1- the tax rate). That is not variable expenses.

Question #21: Northville Products is changing its credit terms from net 30 to 2/10, net 30. The
least likely effect of this change would be a (n)
E. Increase in sales.
F. Lower number of days sales outstanding.
G. Increase in short-term borrowings.
H. Shortening of the cash conversion cycle.
Correct answer: C. increase in short-term borrowings
Explanation:
E. An increase in sales is not the least likely effect of this change in credit terms. It is
possible that some additional sales would be made to customers who would be planning
to take advantage of the 2% discount by paying within 10 days. Therefore, an increase in
sales could be a likely effect.
F. A lower number of days sales outstanding is not the least likely effect of this change in
credit terms. Receivables will be collected more quickly because customers will have an
incentive to pay within the 10 days to receive a 2% discount. The number of days sales
outstanding will likely decrease as a result.
G. An increase in short-term borrowing is the least likely effect of this change in credit
terms. Northville's short-term borrowings will probably decrease, because receivables
will be collected more quickly. Customers will have an incentive to pay within the 10
days to receive a 2% discount.
H. Shortening of the cash conversion cycle is not the least likely effect of this change in
credit terms. A shorter cash conversion cycle is a likely effect, because receivables will
be collected more quickly. Customers will have an incentive to pay within the 10 days to
receive a 2% discount.

Question #22: Gleason Co. has two products, a frozen dessert and ready-to-bake breakfast rolls,
ready for introduction. However, plant capacity is limited, and only one product can be
introduced at present Therefore, Gleason has conducted a market study, at a cost of $26,000, to
determine which product will be more profitable. The results of the study follow.

Sales of desserts at $1.80/unit Sales of rolls at $1.20/unit


Volume Probability Volume Probability
250,000 0.30 200,000 0.20
300,000 0.40 250,000 0.50
350,000 0.20 300,000 0.20
400,000 1.10 350,000 0.10

The costs associated with the two products have been estimated by Gleason's cost accounting
department and are shown as follows.

Dessert Rolls
Ingredients per unit $0.40 $0.25
Direct labor per unit 0.35 0.30
Variable overhead per unit 0.40 0.20
Production tooling* 48,000 25,000
Advertising 30,000 20,000

*Gleason treats production tooling as a current operating expense rather than capitalizing it as a
fixed asset.
In order to recover the costs of production tooling and advertising for the breakfast rolls,
Gleason's sales of the breakfast rolls would have to be
E. 60,000 units.
F. 100,000 units.
G. 37,500 units.
H. Some amount other than those given.
Correct answer:
Explanation:
E. This answer results from dividing fixed costs (advertising and production tooling cost) by
the variable cost per unit $45,000 / ($0.25 + $0.30 + $0.20)]. The formula for the
breakeven volume is Total Fixed Costs / Unit Contribution Margin.
F. The formula for the breakeven volume is Total Fixed Costs/ Unit Contribution Margin.
The total fixed cost is $45,000 ($25,000 + $20,000). The unit contribution margin is
$0.45 ($1.20 - $0.75). $45,000 / $0.45 = 100,000 units that will need to be sold to break
even.
G. This answer results from dividing the total fixed cost of $45,000 by the sale price of
$1.20. The formula for the breakeven volume is Total Fixed Costs / Unit Contribution
Margin.
H. The correct answer is one of the answer choices given.

Question #23: The basis for the components of an Enterprise Risk Management system as
defined by COSO in Enterprise Risk Management - Integrated Framework is
E. The organization's control activities.
F. The internal environment of the organization.
G. Assessment of risk.
H. Identification of the organization's strategic objectives.
Correct answer: B. the internal environment of the organization.
Explanation:
E. Control activities are the policies and procedures implemented to ensure that risk
responses are effectively implemented. They are not the basis for the other components.
F. The internal environment of the organization - the attitude in the organization toward risk
and risk management - is the basis for all the other components of an Enterprise Risk
Management (ERM) system.
G. Risk assessment - the process of analyzing risks from the perspective of the likelihood of
the risk's occurring and the potential impact of the event if it does occur - is the core of an
Enterprise Risk System (ERM). However, it is not the basis for the other components.
H. Identification of the organization's strategic objectives is an important component of an
Enterprise Risk Management (ERM) system. However. it is not the basis for the other
components.

Question #24: All of the following financial indicators are measures of either liquidity or activity
except the
E. Merchandise inventory turnover.
F. Accounts receivable turnover.
G. Times-interest-earned ratio.
H. Average collection period in days.
Correct answer: C. Times-interest-earned ratio.
Explanation:
E. Liquidity ratios have to do with the ability of the company to pay its short-term liabilities
as the become due. Activity ratios measure the ability of the company to manage its
resources efficiently, specifically the current assets of accounts receivable and inventory,
and to manage its accounts payable effectively. The merchandise inventory turnover is a
measure of activity.
F. Liquidity ratios have to do with the ability of the company to pay its short-term liabilities
as the become due. Activity ratios measure the ability of the company to manage its
resources efficiently, specifically the current assets of accounts receivable and inventory,
and to manage its accounts payable effectively. The accounts receivable turnover ratio is
a measure of activity.
G. Liquidity ratios have to do with the ability of the company to pay its short-term liabilities
as the become due. Activity ratios measure the ability of the company to manage its
resources efficiently, specifically the current assets of accounts receivable and inventory,
and to manage its accounts payable effectively. Times interest earned measures neither
the ability to pay current liabilities nor the company's management of its resources and its
accounts payable.
H. Liquidity ratios have to do with the ability of the company to pay its short-term liabilities
as the become due. Activity ratios measure the ability of the company to manage its
resources efficiently, specifically the current assets of accounts receivable and inventory,
and to manage its accounts payable effectively. The average collection period is a
measure of activity current assets of accounts receivable and inventory, and to manage its
accounts payable effectively. Times interest earned measures neither the ability to pay
current liabilities nor the company's management of its resources and its accounts
payable.
Question #25: Leland Manufacturing uses 10 units of Part Number KJ37 each month in the
production of radar equipment. The unit cost to manufacture 1 unit of KJ37 is presented below.

Direct materials $1,000


Materials handling (20% of direct material
cost) 200
Direct labor 8,000
Manufacturing overhead (150% of Direct
labor) 12,000
$21,00
Total manufacturing cost 0

Material handling represents the direct variable costs of the Receiving Department that are
applied to direct materials and purchased components on the basis of their cost. This is a separate
charge in addition to manufacturing overhead. Leland's annual manufacturing overhead budget is
one-third variable and two-thirds fixed. Scott Supply, one of Leland's reliable vendors, has
offered to supply Part Number KJ37 at a unit price of $15,000.
If Leland purchases the KJ37 units from Scott, the capacity Leland used to manufacture these
parts would be idle. Should Leland decide to purchase the parts from Scott, the unit cost of KJ37
would
E. Decrease by $3,200.
F. Decrease by $6,200.
G. Increase by $4,800.
H. Change by some amount other than those given.
Correct answer: Increase by $4,800
Explanation:
E. This is the difference between the total manufacturing cost and the total cost to purchase
the units. It does not take into consideration the fact that the fixed manufacturing
overhead is unavoidable and would continue even if the KJ37 is not being manufactured.
Since the fixed manufacturing overhead is the same whether Leland manufactures the
KJ37 or purchases it, it is an irrelevant cost and should not be included in the total
manufacturing cost used to compare with the cost to purchase the item.
F. This answer assumes that there are no other costs involved in the decision besides the
initial purchase price of $15,000 ($21,200 - $15,000 = $6,200). Other relevant costs
would include unavoidable variable cost and unavoidable fixed cost
G. Even though the manufacturing overhead is being applied on the basis of direct labor
hours and no direct labor hours would be required for production if the KJ37 is purchased
outside, the fixed portion of manufacturing overhead would continue to be incurred even
if Leland purchases the KJ37 from Scott Supply. Therefore, the fixed component of the
manufacturing overhead (2/3 of $12,000. or $8,000) is not relevant to the decision and
can be ignored. Total variable manufacturing cost per unit for Leland is $13,200 ($1,000
DM + $200 materials handling + $8.000 DL + $4,000 variable overhead [1/3 of $12,000]
If Leland purchases the units from Scott then the per unit cost will be $18,000 ($15,000
purchase price + $3,000 materials handling charge [20% of $15,000)). By purchasing
from Scott the per unit cost increase of component KJ37 will be $4,800 ($18,000 -
$13,200).
H. The correct answer is one of the answers that is given.

Question #26: A company obtained a short-term bank loan of $250,000 at an annual interest rate
of 6%. As a condition of the loan, the company is required to maintain a compensating balance
of $50,000 in its checking account. The company's checking account earns interest at an annual
rate of 2%. Ordinarily, the company maintains a balance of $25,000 in its checking account for
transaction purposes. What is the effective interest rate of the loan?
E. 6.66%
F. 6.44%
G. 5.80%
H. 7.00%
Correct answer: B. 6.44%
Explanation:
E. This is $15,000 interest per year on the loan divided by the net usable loan proceeds of
$225,000. This answer fails to take into account the interest that will be earned on the
money deposited to meet the compensating balance requirement. See the correct answer
for a complete explanation.
F. In a loan with a compensating balance, the borrower pays interest on the full amount of
the loan but does not receive the use of the full amount of the loan in cash, since they are
required to leave some of it on deposit as a compensating balance. In this case, since they
already maintain a $25,000 balance at the bank, they will need to add only $25,000 from
the loan proceeds to meet the compensating balance requirement. Therefore, the company
will have the use of $225,000 of the loan but they will pay interest of 6% on the full
$250,000 loan amount. $250,000 x 0.06 equals $15,000 of annual interest expense.
However, this interest expense is reduced by the interest that will be earned on the money
that was deposited to meet the compensating balance requirement. The incremental
amount of the deposit increase is not the full $50,000 of the required compensating
balance, but only the $25,000 that the company needed to add to what was already in the
bank. Interest earned on additional $25,000 at 2% per annum equals $500 interest
received. This interest received offsets the larger interest cost, making a net interest
expense of $14,500. The effective interest rate on the loan is thus $14,500 + the $225,000
received, or 6.44%.
G. This answer is incorrect because in a loan with a compensating balance the effective
interest rate is higher than the nominal interest rate. See the correct answer for a complete
explanation.
H. This answer fails to take into consideration the fact that the company ordinarily maintains
a balance of $25,000 in its checking account for transaction purposes. Thus, the amount
of the loan proceeds that will be added to the checking account needs to be only $25,000,
not the full $50,000 required.

Question #27: Which of the following financial ratios is used to assess the liquidity of a
company?
E. Profit Margin on Sales.
F. Total Debt to Total Assets Ratio.
G. Current Ratio.
H. Days' Sales Outstanding.
Correct answer: C. Current ratio.
Explanation:
E. Profit margin on sales is a profitability measure.
F. Total debt to total assets ratio is a measure of the use of debt in the company.
G. The current ratio is one of the ratios used to measure the liquidity of a company.
H. Days' Sales Outstanding is not a ratio at all because it does not include information on
whether the days' sales outstanding is for receivables or for inventory.

Question #28: The following data pertains to XYZ Company for the current year of operations.

Total Per unit


$1,000,00
Sales (40,000 units) 0 $25
Raw materials 160,000 4
Direct labor 280,000 7
Factory overhead
Variable 80,000 2
Fixed 360,000
Selling and general
expenses:
Variable 120,000 3
Fixed 225,000

How many units does XYZ Company need to produce and sell to make a before-tax profit of
10% of sales?
Correct Answer: B. 90,000 units.
E. 36,562 units.
F. 90,000 units.
G. 25,000 units.
H. 65,000 units.
Correct answer: B. 90,000 units.
Explanation:
E. This is total fixed costs divided by variable costs per unit. The correct answer is total
fixed costs divided by an adjusted contribution margin where the required profit per unit
is included as a variable cost.
F. Variable costs per unit total $16 ($4 + $7 + $2 + $3). The required before-tax profit of
10% of the sales price needs to be considered an additional variable cost for this purpose
and the contribution margin needs to be adjusted accordingly. 10% of the $25 sales price
is $2.50. Thus the contribution margin for this purpose is $25 - $16 - $2.50 = $6.50.
Fixed costs include both factory overhead ($360,000) and selling & administrative costs
($225,000), or $585,000. Therefore, the number of units XYZ Company needs to produce
and sell to make a before-tax profit of 10% of sales is $585,000 + $6.50 = 90,000.
G. This is the fixed selling and administrative costs divided by the contribution margin per
unit. Total fixed costs include both factory overhead and selling & administrative costs:
and the contribution margin per unit needs to be adjusted to include the amount of
required profit per unit as a variable cost.
H. 65,000 units is the breakeven volume, where operating profit is zero.

Question #29: The current market price of Action Pharmaceutical's common stock is $34. A 6-
month call option has been written on the stock. The option has an exercise price of $40 and a
market value of $4.A financial analyst estimates that, at the end of 6 months, the expected value
of the stock is $42.
What is the theoretical value of exercising the option on the date it is written?
Correct Answer: • C. $0
E. $6.00
F. $8.00
G. $0
H. $4.00
Explanation:
E. This is the difference between the exercise price of the option and the current market
price of the stock. However, that is not the theoretical value of the option on the date it is
written.
F. This is the difference between the expected market value of the stock and the current
market value of the stock. However, this is not the theoretical value of the option on the
date it is written.
G. At the time the option was written, it had no theoretical value. This is because the option
exercise price ($40) was higher than the market price ($34). Because of this, no one
would use the option to buy a share since it would be cheaper to buy a share on the open
market.
H. This is the market value of the option. However, the market value is not the same as the
theoretical value of the option on the date it is written.

Question #30: With respect to a given risk, an expected loss is


Correct Answer: • A an amount that management expects to be lost per year on average over a
period of several years.
E. An amount that management expects to be lost per year on average over a period of
several years.
F. The amount expected to be lost every year to that risk.
G. The amount that management thinks could be lost to the risk in a very bad year.
H. A loss that management knows will occur.
Explanation:
E. An amount that management expects to be lost per year on average over a period of
several years is the definition of expected loss. The expected loss to a given risk can be
calculated as a weighted average of the possible losses to that risk, weighted according to
their probabilities of occurring.
F. An expected loss is not the amount expected to be lost every year to a given risk.
G. The amount that management thinks could be lost to the risk in a very bad year is the
maximum probable loss.
H. If management knows without a doubt that a loss will occur, there is no risk involved.

Question #31: An advantage of the net present value method over the internal rate of return
model in discounted cash flow analysis is that the net present value method
Correct Answer: • B. Can be used when there is no constant rate of return required for each year
of the project.
E. Uses a discount rate that equates the discounted cash inflows with the outflows.
F. Can be used when there is no constant rate of return required for each year of the project.
G. Computes a desired rate of return for capital projects.
H. Uses discounted cash flows whereas the internal rate of return model does not.
Explanation:
E. The internal rate of return is the discount rate that equates the discounted cash inflows
with the outflows for a project. However, the net present value method does not use the
projects internal rate of return as the discount rate when determining the present value of
the cash inflows and outflows.
F. Since each year's net cash flow is calculated individually and may be discounted
individually using net present value analysis, the net present value method can
incorporate varying rates of returns during the various years of the projects life. In
contrast, the internal rate of return method is used to determine the single discount rate at
which the net present value of a project is zero. Thus, the internal rate of return model
cannot be used when there is no constant rate of return required for each year of a project.
G. The net present value method does not compute a desired rate of return for capital
projects. The desired rate of return must be determined by management and then is used
to determine the present value of the cash flows, both positive and negative.
H. Both the net present value method and the internal rate of return method use discounted
cash flows.

Question #32: Please see the question below:


Garland Corporation's Income Statement for the year just ended is shown below.
$900,00
Net sales 0
Cost of goods sold:
$125,00
Invetory-beginning 0
Purchases 540,000
Goods availbale for sale 665,000
Inventory-ending 138,000
Cost of goods sold: 527,000
Gross profit 373,000
Operating expenses 175,000
$198,00
income from operations 0

Garland's average inventory turnover ratio is


Correct Answer: A. 4.01.
E. 4.01.
F. 6.84.
G. 3.82.
H. 6.52.
Explanation:
E. The inventory turnover ratio is Cost of Goods Sold + Average Inventory. Average
inventory is the average of the beginning and ending inventories, which is ($125,000 +
$138,000) ÷ 2, or $131,500. So the inventory turnover ratio is $527,000 + $131,500,
which equals 4.01.
F. The inventory turnover ratio is Cost of Goods Sold + Average Inventory. This is Sales +
Average Inventory.
G. The inventory turnover ratio is Cost of Goods Sold + Average Inventory. This is Cost of
Goods Sold + Ending Inventory.
H. The inventory turnover ratio is Cost of Goods Sold + Average Inventory. This is Sales +
Ending Inventory.

Question #33: A company is in the process of evaluating a major product line expansion. Using a
14%discount rate, the firm has calculated the present value of both the project's cash inflows and
cash outflows to be $15.8 million. The company will likely evaluate this project further by
Correct Answer: D. comparing the internal rate of return versus the company's cost of capital and
hurdle rate.
E. Comparing the internal rate of return versus the discount rate used.
F. Taking a closer look at the expansion's contribution margin.
G. Comparing the internal rate of return versus the accounting rate of return.
H. Comparing the internal rate of return versus the company's cost of capital and hurdle rate.
Explanation:
E. The IRR is the discount rate at which the NPV of the project is zero. Since the present
value of both the project's cash inflows and cash outflows are the same ($15.8 million)
when a discount rate of 14% is used, the project's NPV is zero at that discount rate. Thus,
the internal rate of return is the same as the discount rate, and there is no new knowledge
to be gained by comparing the two.
F. The contribution margin of a project is not a part of a capital budgeting analysis.
G. When the accounting rate of return method is used for capital budgeting, management
sets a required accounting rate of return, and projects whose returns exceed that rate are
considered acceptable. Comparison with the internal rate of return of the project would
not be meaningful, because the internal rate of return is based on cash flow, whereas the
accounting rate of return is based on accounting income.
H. Since the present values of both the project's cash inflows and cash outflows are the same
($15.8 million), the NPV of this project is zero, when a discount rate of 14% is used. If
the discount rate the company has used as a hurdle rate is truly representative of its cost
of capital and the anticipated risk in the project, this is not a project the company should
embark upon, because it will not earn any profit for the shareholders. So the company
should evaluate whether the 14% that it used as its hurdle rate is reasonable. To do that, it
should compare the company's cost of capital with the hurdle rate it used. This is the only
answer choice that includes comparing the company's cost of capital with the hurdle rate.

Question #34: Which of the following will result in raising the breakeven point?
Correct Answer: • C. An increase in the semi-variable cost per unit.
E. A decrease in income tax rates.
F. An increase in the contribution margin per unit.
G. An increase in the semi-variable cost per unit.
H. A decrease in the variable cost per unit.
Explanation:
E. Income taxes are considered to be a variable expense, and as such a decrease in the
income tax rate would increase the contribution margin, thereby lowering the breakeven
point.
F. An increase in the contribution margin per unit would decrease the breakeven point.
G. Semi-variable costs have a fixed component and a variable component. A basic fixed
amount must be paid even if there is no activity: and added to that fixed amount is a
variable amount that varies with activity. An increase in either a fixed or variable expense
will increase the breakeven point. The formula to calculate the breakeven point in units is
Fixed Costs / Unit Contribution Margin. If variable costs increase, the Unit Contribution
Margin in the denominator will decrease, which will cause the breakeven point to
increase. If fixed costs increase, the numerator will increase, which will also cause the
breakeven point to increase. Therefore, an increase in a semi-variable cost would increase
the breakeven point as well.
H. A decrease in variable cost per unit would increase the contribution margin, thereby
decreasing the breakeven point.
Question #35: A company with $280,000 of fixed costs has the following data:

Product A Product B
Sales price per unit $5 $6
Variable costs per unit $3 $5

Assume three units of A are sold for each unit of B sold. How much will sales be in dollars of
product B at the breakeven point?
Correct Answer: • C. $240,000
E. $280,000
F. $200,000
G. $240,000
H. $840,000
Explanation:
E. This is not the correct answer. Please see the correct answer for an explanation.
F. This is not the correct answer. Please see the correct answer for an explanation.
G. The first step is to calculate the composite units contribution margin for products A and
B. The formula is: [3 units of A x ($5 - $3)] + [1 unit of B x ($6 - $5)] = (3 x $2) + (1 x
$1) = $6 + $1 = $7 composite unit contribution margin. This is the contribution margin
for a basket of goods that includes 3 units of A and 1 unit of B.
The second step is to calculate the composite breakeven point in number of baskets.
Fixed cost $280,000 ÷ Composite CM $7 = 40,000 composite baskets of goods
containing 3 units of A and 1 unit of B. The third step is to calculate the breakeven point
in sales for Product B. If the breakeven number of baskets is 40.000 and each basket
contains 1 unit of Product B. then the breakeven number of units of Product B is 40.000 x
1, or 40,000. 40,000 units of Product B x $6 sales price of Product B = $240,000 sales
revenue for Product B at the breakeven point.
H. This is not the correct answer. Please see the correct answer for an explanation.

Question #36: An agreement to exchange a fixed interest rate on a loan with a floating interest
rate on a loan is called a (n)
Correct Answer: A. interest rate swap.
E. Interest rate swap.
F. Basis swap.
G. Swaption.
H. Interest rate guarantee.
Explanation:
E. An interest rate swap takes place when two parties exchange interest payments, one at a
fixed rate and one at a floating (or variable) rate that is pegged to some sort of market rate
of interest and changes whenever the market rate changes. The primary purpose of an
interest rate swap is to match the characteristics of the firm's revenue stream with the
characteristics of its payment stream. For example, if a firm has a revenue stream that
increases or decreases with the market rate of interest, it would want its payment stream
to also increase or decrease with interest rates. If the firm has a fixed rate loan, swapping
the fixed rate loan for a floating rate loan would achieve this goal, and reduce the firm's
overall risk.
F. A basis swap has is an interest rate swap with floating payments on both sides, each tied
to two different indexes. So a basis swap does not involve a fixed interest rate on one
side.
G. A swaption is an option to enter into a swap transaction at a specified future date, with
the terms of the swap being fixed at the time the swaption is transacted. A swaption may
be an option to enter into an agreement to exchange a fixed interest rate on a loan with a
floating interest rate on a loan. However, a swaption is not the agreement to do so.
H. An agreement to exchange a fixed interest rate on a loan with a floating interest rate on a
loan is not an interest rate guarantee.

Question #37: Book value per common share represents the amount of shareholders' equity
assigned to each outstanding share of common stock. Which one of the following statements
about book value per common share is correct?
Correct Answer: • Book value per common share can be misleading because it is based on
historical cost
E. Book value per common share can be misleading because it is based on historical cost
F. A market price per common share that is greater than book value per common share is an
indication of an overvalued stock
G. Book value per common share is the amount that would be paid to shareholders if the
company were sold to another company
H. Market price per common share usually approximates book value per common share.
Explanation:
E. Because the calculation of book value per share is based on balance sheet amounts, it is
very possible that the book value per share will not reflect the current situation of the
company. This is demonstrated by assuming that a company owns an asset that has
appreciated greatly in value while the company has held it. This asset will be recorded on
the books at its lower cost of acquisition, and this will lead to an understated book value
per share.
F. The market and book value of the shares may be different and a greater market price does
not indicate an overvalued share. This may happen if the company owns an asset that has
appreciated greatly in value while the company has held it. This asset will be recorded on
the books at its lower cost of acquisition and this will lead to an understated book value
per share.
G. The amount that would be paid to a shareholder in the event that the company is sold to
another would be determined by the specifics of the transaction itself, not by the book
value per share.
H. The market value and the book may approximate each other, but they do not have to. If
the company has an asset that has appreciated greatly in value, this increased value of the
asset will not be reflected in book value per share, but the market will have taken it into
account when setting the market price of the shares.
Question #38: A company has developed a new technologically advanced paper-thin solar panel
for residential home use. The company has decided to start selling this solar panel worldwide
next month. As this is a technologically innovative product, one risk that the company faces to
sell the solar panel is possible product failure. This risk of product failure is best representative
of what type of risk to the company?
Correct Answer: • B. Operational risk.
E. Inherent risk.
F. Operational risk
G. Residual risk.
H. Business risk.
Explanation:
E. Inherent risk is the level of risk in each event before any mitigation action is taken. The
risk of product failure is not an inherent risk.
F. Operational risk is risk that results from inadequate or failed internal processes, people or
systems. The risk of product failure is an operational risk.
G. Residual risk is the level of risk that remains after management has taken action to
mitigate the risk. The risk of product failure is not a residual risk.
H. Business risk for a firm is the risk of changes in its earnings before interest or taxes.
Business risk depends on a variety of factors, including the variability of demand, sales
price, and the price of inputs as well as the amount of the company's operating leverage.
The risk of product failure is not a business risk.

Question #39: A company has a 50%gross margin, general and administrative expenses of $50,
interest expense of $20, and net income of $10 for the year just ended. If the corporate tax rate is
50%, the level of sales revenue for the year just ended was
Correct Answer: • $180
E. $135
F. $150
G. $180
H. $90
Explanation:
E. This not the correct answer. Please see the correct answer for a complete explanation.
F. We have been unable to determine how to calculate this incorrect answer choice. If you
have calculated it, please let us know how you did it so we can create a full explanation
of why this answer choice is incorrect. Please send us an email at admin@cmapass.com.
Include the Topic Name. Question Details or Screenshot and the actual incorrect answer
choice -- not its letter, because that can change with every study session created.. Thank
you in advance for helping us to make your CMA PASS study materials better. This not
the correct answer. Please see the correct answer for a complete explanation. We have
been unable to determine how to calculate this incorrect answer choice. If you have
calculated it, please let us know how you did it so we can create a full explanation of why
this answer choice is incorrect. Please send us an email at admin@cmapass.com. Include
the Topic Name. Question Details or Screenshot and the actual incorrect answer choice --
not its letter, because that can change with every study session created.. Thank you in
advance for helping us to make your CMA PASS study materials better.
G. If net income is $10 and the tax rate is 50%, we know that income before taxes is $20
($10 + [1- tax rate]). We also know that the cost of goods sold is 50% of the sales price
since the gross margin is 50% (Sales - COGS = Gross Margin: therefore. Sales - Gross
Margin = COGS). Total expenses are $70 (general and administrative $50 plus interest
$20). Plugging the numbers we know into the net income (before taxes) function, we get
the following formula, where R represents revenue: R - 0.5R - 70 = 20. 1) Combine like
terms, the like terms being the Rs. R - 0.5R = 0.5R. Therefore. 0.5R - 70 = 20 2) Add 70
to both sides of the equation to isolate the term containing the unknown on one side of
the equals sign: 0.5R = 90 3) Divide both sides of the equation by 0.5 to find R: R = 180
$180 is the revenue of this company.
H. This is the amount of the cost of goods sold. See the correct answer for a complete
explanation.

Question #40: A large multinational company currently has its information technology
department located in Germany. In order to reduce the risk of system failure, the company has
decided to split up the information technology department into two geographically separate
locations and set up a new location in Singapore. The company can still face a catastrophic
system failure, but the risk will be greatly reduced. The risk that remains after the company sets
up the second information technology department in Singapore is best described as
Correct Answer: • D. residual risk.
E. Hazard risk.
F. Inherent risk.
G. Business risk.
H. Residual risk.
Explanation:
E. Hazard risk is the type of risk that is can be insured against. Common examples are
natural disasters (property insurance), death of a key employee (key person life
insurance), personal injury that takes place on the premises of the business (liability
insurance), and any other unexpected event that can be insured against. The risk that
remains after the company sets up the second information technology department in
Singapore is not hazard risk.
F. Inherent risk is the level of risk in each event before any mitigation action is taken. Since
action has been taken to mitigate the risk by splitting up the information technology
department into two geographically separate locations, the risk remaining is not inherent
risk.
G. Business risk for a firm is the risk of changes in its earnings before interest or taxes.
Business risk depends on a variety of factors, including the variability of demand, sales
price, and the price of inputs as well as the amount of the company's operating leverage.
The more stable all of these variables are, the less business risk a company will
experience. The risk that remains after the company sets up the second information
technology department in Singapore is not business risk.
H. Residual risk is the level of risk that remains after management has taken action to
mitigate the risk. Thus the risk that remains after the company sets up the second
information technology department in Singapore is best described as residual risk.

Question #41: The term relevant cost applies to all the following decision situations except the
Correct Answer: • D. Determination of a product price.
E. Replacement of equipment.
F. Acceptance of a special order.
G. Manufacture or purchase of component parts.
H. Determination of a product price.
Explanation:
E. Relevant costs are incremental or differential costs that vary among the possible choices.
Relevant costs will be important in the decision of whether or not to replace equipment
because the relevant costs will be different for each of the options.
F. Relevant costs are important to the decision whether to accept a special order or not.
G. Relevant costs are incremental or differential costs that vary among the possible choices.
Relevant costs will be important to the decision whether to manufacture or purchase
component parts because the relevant costs will be different for each of the options.
H. Relevant costs are incremental or differential costs that vary among the possible choices.
The costs will be the same regardless of what product price is chosen.
Question #42: What are the two primary areas where management accountants have a
responsibility to act as change agents within their organizations where ethics are concerned?
Correct Answer: • D. Internal control and risk management
E. Internal control and financial reporting
F. Management accounting and risk management
G. Financial reporting and risk management
H. Internal control and risk management
Explanation:
E. Although ethics are very important in preventing fraudulent financial reporting, financial
reporting is not a primary area where management accountants have a responsibility to
act as change agents within their organizations where ethics are concerned.
F. Management accounting is not a primary area where management accountants have a
responsibility to act as change agents within their organizations where ethics are
concerned.
G. Although ethics are very important in preventing fraudulent financial reporting, financial
reporting is not a primary area where management accountants have a responsibility to
act as change agents within their organizations where ethics are concerned.
H. Internal control and risk management are the two areas that are primarily impacted by
human behavior. Therefore, those are the primary areas where management accountants
have a responsibility to act as change agents within their organizations where ethics are
concerned.

Question #43: Which of the following is not a benefit that will result when a company invests in
efforts to create a values-based organizational culture?
Correct Answer: B. Lower government taxes because of aggressive and spirited defense of the
company towards tax authorities by motivated employees.
E. Better teamwork
F. Lower government taxes because of aggressive and spirited defense of the company
towards tax authorities by motivated employees.
G. Less fraud through a sense of ownership and identification with the company
H. Higher levels of productivity through motivated. engaged employees
Explanation:
E. This is a benefit of creating a values based organizational culture.
F. Payment of lower taxes is not a benefit that results from an investment to develop an
ethics based business environment.
G. This is a benefit of creating a values based organizational culture.
H. This is a benefit of creating a values based organizational culture.
Question #44: DRP Insurance Company wants to be "best in class" in terms of Enterprise Risk
Management (ERM) implementation. To achieve this goal, the company plans to identify events
that affect the implementation of strategy and achievement of objectives. Which of the following
best reflects an analysis that would help its identification process?
Correct Answer: • A. Review of incidents and new market conditions.
E. Review of incidents and new market conditions.
F. Analysis of default histories and dispersion.
G. Review of accidents and operational measures.
H. Summary of driving records and age.
Explanation:
E. Enterprise risk management (ERM) is defined as "a process, effected by an entity's board
of directors, management and other personnel, applied in strategy setting and across the
enterprise, designed to identify potential events that may affect the entity, and manage
risk to be within its risk appetite, to provide reasonable assurance regarding achievement
of entity objectives:' A review of incidents and new market conditions is a strategy
designed to identify potential events that may affect the entity and to manage risk.
F. Enterprise risk management (ERM) is defined as "a process, effected by an entity's board
of directors, management and other personnel, applied in strategy setting and across the
enterprise, designed to identify potential events that may effect the entity, and manage
risk to be within its risk appetite, to provide reasonable assurance regarding achievement
of entity objectives: Analysis of default histories and dispersion is not a strategy designed
to identify potential events that may affect the entity and to manage risk.
G. Enterprise risk management (ERM) is defined as "a process, effected by an entity's board
of directors, management and other personnel, applied in strategy setting and across the
enterprise, designed to identify potential events that may effect the entity, and manage
risk to be within its risk appetite, to provide reasonable assurance regarding achievement
of entity objectives: A review of accidents and operational measures is not a strategy
designed to identify potential events that may affect the entity and to manage risk.
H. Enterprise risk management (ERM) is defined as "a process, effected by an entity's board
of directors, management and other personnel, applied in strategy setting and across the
enterprise, designed to identify potential events that may effect the entity, and manage
risk to be within its risk appetite, to provide reasonable assurance regarding achievement
of entity objectives." A summary of driving records and age is not a strategy designed to
identify potential events that may affect the entity and to manage risk.

Question #45: Which of the following statements is a responsibility for an IMA member
regarding the "Integrity" standard of ethical conduct?
Correct Answer: • C. Refrain from engaging in any conduct that would prejudice carrying out
duties ethically.
E. Disclose all relevant information that could reasonably be expected to influence an
intended user's understanding of the reports, analyses, or recommendations.
F. Perform professional duties in accordance with relevant laws, regulations, and technical
standards.
G. Refrain from engaging in any conduct that would prejudice carrying out duties ethically.
H. To refrain from using confidential information for unethical or illegal advantage.
Explanation:
E. This is one of the responsibilities for an IMA member regarding the "Credibility"
standard of ethical conduct.
F. This is one of the responsibilities for an IMA member regarding the "Competence
standard of ethical conduct.
G. Each IMA member has the following responsibilities with respect to the "Integrity"
standard of ethical conduct:
4) Mitigate actual conflicts of interest: regularly communicate with business associates
to avoid apparent conflicts of interest. Advise all parties of any potential conflicts.
5) Refrain from engaging in any conduct that would prejudice carrying out duties
ethically.
6) Absain from engaging in or supporting any activity that might discredit the
profession.
H. This is one of the responsibilities for an IMA member regarding the "Confidentiality"
standard of ethical conduct.

Question #46: The Doll House, a very profitable company, plans to introduce a new type of doll
to its product line. The sales price and costs for the new dolls are as follows. Selling price per
doll $100 Variable cost per doll $60 Incremental annual fixed costs $456,000 Income tax rate
30%
If 10,000 of the new dolls are produced and sold, the effect on Doll House's profit (loss) would
be
Correct Answer: D. $(39,200).
E. $(176,000).
F. $(56,000).
G. $280,000.
H. $(39,200).
Explanation:
E. This is the net incremental before-tax operating loss of $(56,000) minus $120,000, which
is 30% of the incremental contribution margin of $400,000. The incremental tax effect of
the $(56,000) incremental operating loss is 30% of the ($56,000) incremental operating
loss, not 30% of the incremental contribution margin. Furthermore, the tax effect of a loss
reduces the loss instead of increasing it. Assuming the company's other operations are
profitable, the loss will shield other net income from tax, reducing the overall tax due.
F. This is the incremental operating loss before the tax effect of the loss is taken into
consideration.
G. This answer does not include any deductions for the incremental fixed costs and the tax
effect of the fixed costs.
H. The incremental revenues and costs from the manufacture and sale of the new doll are as
follows:
Incremental sales: $100 x 10,000 $1,000,000
Incremental variable cost: $60 x 10,000 600,000
Incremental contribution margin: $40 x 10,000 $ 400,000
Incremental fixed costs 456,000
Net incremental operating loss $ (56,000)
Plus income tax effect of incremental loss: 30% of $56,000 16,800
Net incremental after-tax operating loss $(39,200)

Question #47: Which of the following scenarios would encourage a company to use short-term
loans to retire its 10-year bonds that have 5 years until maturity?
Correct Answer: • C. Interest rates have declined over the last 5 years.
E. Interest rates have increased over the last 5 years.
F. The company is experiencing cash flow problems.
G. Interest rates have declined over the last 5 years.
H. The company expects interest rates to increase over the next 5 years.
Explanation:
E. If interest rates have increases in the past five years the company is better off keeping the
bonds that they have issued because the interest rate on the bonds is lower than what the
interest rate on the new loans would be.
F. If the company switches to short term loans, they will essentially have to retire the bonds
early as the loans become due. If they are having cash flow problems, this will make
those problems worse by increasing the short term cash outflows that will be required.
G. If interest rates have declined since the bonds were issued, the company can take out new
financing at a lower rate of interest and use the money from the new financing to retire
the original, more expensive debt. This will lower their cost of interest for the next five
years.
H. If short-term rates will rise in the future, the company will not benefit by switching to
short-term loans, for which the rate will go up in future periods.

Question #48: A manufacturer with seasonal sales would be most likely to obtain which one of
the following types of loans from a commercial bank to finance the need for a fixed amount of
additional working capital during the busy season?
Correct Answer: • A Unsecured short-term term loan.
E. Unsecured short-term term loan.
F. Transaction loan.
G. Installment loan.
H. Insurance company term loan.
Explanation:
E. A short-term loan is a loan with a maturity date of less than one year in the future. A
short-term loan, either secured or unsecured, would be an appropriate type of loan to be
used to finance the need for a fixed amount of additional working capital during the busy
season. The collection of receivables from the selling season is the source of the
repayment of the loan, so the loan should be paid off about a month following the end of
the company's busy season. Therefore, the loan should have a maturity date of about one
month beyond the end of the company's busy season, so the company will have a chance
to collect the receivables from the selling season to use to pay off the loan. If the loan
cannot be paid off on its maturity date, then something is wrong, because it means the
company has used the collection of the receivables for something other than to pay off
the seasonal loan.
F. A transaction loan is a loan made for a specific purchase, such as a mortgage loan made
for the purchase of real estate or a term loan made for the purchase of equipment.
Usually, the disbursement check is made out to the seller of the item, so the lender can be
certain that the loan is being used for its designated purpose. A working capital loan such
as the one mentioned in this problem would not be called a transaction loan, because it
would not be used to make just a single purchase from a single supplier. It would
probably be used for multiple inventory purchases from multiple suppliers, possibly over
a period of several months. So it would not be practical for the loan to be disbursed by
means of checks made payable to each of the suppliers.
G. An installment loan is a type of long-term financing, and it would have a maturity date of
at least one year in the future. Installment loans are generally used for the purchase of
vehicles and other smaller fixed assets. Financing that provides additional working
capital to support the busy season is short-term financing, because it can be repaid when
the busy season is over and the receivables from the sales are collected. So an installment
loan would not be appropriate for this purpose.
H. An insurance company term loan, or any term loan no matter who the lender is, would be
used for long-term financing. A term loan is a loan made to a business for long-term
needs, such as purchase of fixed assets that has a maturity date of one year or more in the
future. Financing that provides additional working capital to support the busy season is
short-term financing, because it can be repaid when the busy season is over and the
receivables from the sales are collected. So a term loan would not be appropriate for this
purpose.

Question #49: McLean Inc. is considering the purchase of a new machine that will cost
$160,000. The machine has an estimated useful life of 3 years. Assume that 30% of the
depreciable base will be depreciated in the first year, 40% in the second year, and 30% in the
third year. The new machine will have a $10,000 resale value at the end of its estimated useful
life. The machine is expected to save the company $85,000 per year in operating expenses.
McLean uses a 40% estimated income tax rate and a 16% hurdle rate to evaluate capital projects.
Discount rates fora 16% rate are as follows: Present Value Present Value of an Ordinary
of $1 Annuity of $1 Year 1 0.862 0.862 Year 2 0.743 1.605 Year 3 0.641 2.246
What is the net present value of this project?
Correct Answer: D. $6.270
E. $5,842
F. $8,834
G. $30,910
H. $6,270
Explanation:
E. An answer of $5,842 results from subtracting the resale value of the equipment from the
purchase cost to calculate the depreciable base for tax purposes. For tax purposes, 100%
of an asset's cost is always depreciated.
F. An answer of $8,834 results from not calculating income tax due on the disposition of the
equipment at the end of 3 years. However, there will be a gain on the sale, since 100% of
the asset's cost will have been depreciated for tax purposes by the end of the third year.
G. An answer of $30,910 results from including only the initial investment and the gross
operating cash flows in the calculation of the net present value.
H. The cash flows are as follows:
The cash flows are as follows:

Year 0 Year 1 Year 2 Year 3


(160,000
Initial investment )
Depreciation 48,000 64,000 48,000
Depreciation Tax Sheild (Depr. ×
0.40) 19,200 25,600 19,200
Cash from disposition (after tax) 6,000
Operating cash flows 85,000 85,000 85,000
(34,000 (34,000 (34,000
Tax on operating cash flow at 40% ) ) )
Net cash flow 70,200 76,000 76,200
Discount factor:16% 0.862 0.743 0.641
(160,000
Discounted cash flow ) 60,512 56,914 48,844

The net present value is $(160,000) + $60,512 + $56,914 + $48,844 = $6,270

Question #50: A stock dividend


Correct Answer: • B. decreases future earnings per share.
E. Decreases the size of the firm.
F. Decreases future earnings per share.
G. Increases the debt-to-equity ratio of a firm.
H. Increases shareholders' wealth.
Explanation:
E. A stock dividend does not impact the size of the firm.
F. In a stock dividend more shares are issued to existing shareholders. Since there is no
increase in income from this event but there are more shares outstanding, future earnings
per share will decrease as a result of the stock dividend.
G. A stock dividend has no impact on the book value of the company's total equity.
Therefore, there is no effect on the debt-to-equity ratio.
H. A stock dividend does not in itself increase shareholder wealth. The stock dividend
provides more shares to each shareholder, but the total value of the shares remains
unchanged.

Question #51: The interest expense for a company is equal to its earnings before interest and
taxes (E B IT). The company's tax rate is 40%. The company's times-interest earned ratio is
equal to
Correct Answer: A. 1.0.
E. 2.0.
F. 0.6.
G. 1.0.
H. 1.2
Explanation:
E. This is not the correct answer. Please see the correct answer for an explanation. We have
been unable to determine how to calculate this incorrect answer choice. If you have
calculated it, please let us know how you did it so we can create a full explanation of why
this answer choice is incorrect. Please send us an email at admin@cmapass.com. Include
theTopic Name. Question Details or Screenshot and the actual incorrect answer choice --
not its letter, because that can change with every study session created.. Thank you in
advance for helping us to make your CMA PASS study materials better.
F. This answer results from reducing Earnings Before Interest and Taxes (EBIT) by income
tax due on it based on the tax rate, in calculating the times interest earned (interest
coverage) ratio. EBIT should not be reduced by income tax due on it.
G. The times interest earned ratio (interest coverage ratio) is EBIT ÷ Interest Expense. Since
EBIT and Interest Expense are the same, the company's times interest earned ratio must
be 1.0.
H. This is not the correct answer. Please see the correct answer for an explanation. We have
been unable to determine how to calculate this incorrect answer choice. If you have
calculated it, please let us know how you did it so we can create a full explanation of why
this answer choice is incorrect. Please send us an email at admin@cmapass.com. Include
the Topic Name. Question Details or Screenshot and the actual incorrect answer choice --
not its letter, because that can change with every study session created.. Thank you in
advance for helping us to make your CMA PASS study materials better.

Question #52: Lisa, Inc.


Statement of Financial Position

20 × 4 20 × 3
Assets
Current assets:
Cash $ 30 $ 25
Trading securities 20 15
Accounts receivable (net) 45 30
Inventories (at lower of cost of market) 60 50
Prepaid items 15 20
Total current assets $170 $140

Long-term assets:
Long-term investments:
Available-for-sale investments $ 25 $ 20
Property, plant & equipment
Land (at cost) 75 75
Building (net) 80 90
Equipment (net) 95 100
Intangible assets:
Patents (net) 35 17
Goodwill (net) 20 13
Total long-term assets $330 $315
Total Assets $500 $455

Liabilities and equity


Current liabilities:
Notes payable $23 $12
Accounts payable 47 28
Accrued interest $ 15 $ 15
Total current liabilities $ 85 $ 55

Long-term liabilities:
Long-term notes payable 10% due 12/31/20×6 $ 10 $ 10
Bonds payable 12% due 12/31/20×9 15 15
Total long-term debt $25 $25
Total liabilities $110 $80

Shareholders' equity
Preferred stock-5% cumulative, $100 par, nonparticipating
authorized, issued and outstanding $ 100 $ 100
Common stock-$10 par 20,000 shares authorized, 15,000
shares issued and outstanding 150 150
Additional paid-in capital-common 75 75
Retained earnings 65 50
Total equity $ 390 $ 375
Total liabilities & equity $500 455

Correct Answer: D.1.1


E. 1.8
F. 2.0
G. 0.6
H. 1.1
Explanation:
E. This answer incorrectly includes inventory in the calculation of the numerator.
F. This answer includes all current assets in the calculation, so it is the current ratio. not the
acid test (quick) ratio.
G. This answer does not include the receivables in the numerator of the calculation.
H. The acid test (or quick) ratio is calculated as follows: (Cash + Receivables + Trading
Securities) / Current Liabilities. Given the information in this question. we get ($30,000 +
$20,000 + $45,000) / $85.000. This is 1.1.

Question #53: In discounted cash flow techniques, which one of the following alternatives best
reflects the items to be incorporated in the initial net cash investment?

Capitalized Change in net Impact of spontaneous


expenditures (e.g., working Net proceeds from sale of old changes in current
shipping costs) capital asset in a replacement decision liabilities

I No Yes Yes Yes

II Yes No No No

III No Yes No No
IV Yes Yes Yes Yes

Correct Answer: • B. IV.


E. III.
F. IV.
G. II.
H. I.
Explanation:
E. Capitalized expenditures such as shipping costs are part of the cost of the fixed asset, and
so they are a part of the Year 0 cash outflow. If an old asset is being replaced, then the net
proceeds from the sale of the old asset are also a part of the Year 0 cash outflow. The
impact of a spontaneous change in current liabilities is a part of the Year 0 cash flow
because it is a part of the change in net working capital. An increase in accounts payable
is a spontaneous change in current liabilities. It is called "spontaneous" because it just
happens automatically. The company does not approach its vendors and ask them if it can
carry a larger balance due to them. But when the company increases its purchases that
automatically creates increased accounts payable. An increase in accounts payable would
function like a cash inflow. If accounts payable increase when inventory, a current asset,
increases that means that the full amount of the inventory increase has not been paid for
yet. Thus the apparent cash outflow caused by the increase in inventory is reduced by the
increase in accounts payable.
F. The cash outflow for the initial investment should include all of these things. Capitalized
expenditures such as shipping costs are part of the cost of the fixed asset, and so they are
a part of the Year 0 cash outflow. Any change in net working capital (usually an increase.
but it could also be a decrease) is also a part of the Year 0 cash outflow. If an old asset is
being replaced, then the net proceeds from the sale of the old asset are also a part of the
Year 0 cash outflow. The impact of a spontaneous change in current liabilities is a part of
the Year 0 cash flow because it is a part of the change in net working capital. An increase
in accounts payable is a spontaneous change in current liabilities. It is called
"spontaneous" because it just happens automatically. The company does not approach its
vendors and ask them if it can carry a larger balance due to them. But when the company
increases its purchases that automatically creates increased accounts payable. An increase
in accounts payable would function like a cash inflow. If accounts payable increase when
inventory, a current asset, increases, that means that the full amount of the inventory
increase has not been paid for yet. Thus the apparent cash outflow caused by the increase
in inventory is reduced by the increase in accounts payable.
G. Any change in net working capital (usually an increase. but it could also be a decrease) is
also a part of the Year 0 cash outflow. If an old asset is being replaced, then the net
proceeds from the sale of the old asset are also a part of the Year 0 cash outflow. The
impact of a spontaneous change in current liabilities is a part of the Year 0 cash flow
because it is a part of the change in net working capital. An increase in accounts payable
is a spontaneous change in current liabilities. It is called "spontaneous" because it just
happens automatically. The company does not approach its vendors and ask them if it can
carry a larger balance due to them. But when the company increases its purchases that
automatically creates increased accounts payable. An increase in accounts payable would
function like a cash inflow. If accounts payable increase when inventory, a current asset,
increases, that means that the full amount of the inventory increase has not been paid for
yet. Thus the apparent cash outflow caused by the increase in inventory is reduced by the
increase in accounts payable.
H. Capitalized expenditures such as shipping costs are part of the cost of the fixed asset, and
so they are a part of the Year 0 cash outflow.

Question #54: Leslie Corporation manufactures classroom desk chairs and tables. In the present
market, the company can sell as many units of product as it can manufacture, but it is constrained
by its availability of machine-hour capacity. Sales price and cost information for each unit of
product are shown below.
Desk chair Tables
Sales price $75 $180
Variable costs 60 155
Contribution margin $15 $25

Producing a desk chair requires 11/2 machine hours: producing a table requires 2% machine
hours. Which product, if any, is most profitable given the machine-hour constraints?
Correct Answer: • B. Both products are equally profitable.
E. There is not enough data to identify the most profitable product.
F. Both products are equally profitable.
G. Tables.
H. Desk chairs.
Explanation:
E. There is enough data to identify the product that is most profitable.
F. The product with the higher contribution margin per unit of the constrained resource is
the most profitable. The contribution margin per unit for desk chairs is $15 and desk
chairs require 1.5 machine hours each, so the contribution margin per machine hour for
desk chairs is $15 + 1.5, or 510. The contribution margin per unit for tables is $25 and
tables require 2.5 machine hours each, so the contribution margin per machine hour for
tables is $25 + 2.5, or $10. Therefore, desk chairs and tables are equally profitable, given
the machine-hour constraint.
G. The product with the higher contribution margin per unit of the constrained resource is
the most profitable. The contribution margin per machine hour for tables is not higher
than it is for desks.
H. The product with the higher contribution margin per unit of the constrained resource is
the most profitable. The contribution margin per machine hour for desks is not higher
than it is for tables.

Question #55: Which of the following is not a benefit of risk management?


Correct Answer: • B. Lower taxes.
E. Better contingency planning.
F. Lower taxes.
G. Improved cost control.
H. Increased shareholder value.
Explanation:
E. Contingency planning is the development of a "backup plan." or a plan that enables the
organization to respond appropriately to an unplanned event. It is used for emergency
response and post-disaster recovery. Contingency planning is particularly important in
information systems in order to insure that critical systems resources will be available
and operations can continue during an emergency situation. Better contingency planning
is a benefit of risk management because risk assessment and risk management enable
management to be aware of and prepared for more possible events that could prevent
them from achieving their objectives.
F. Lower taxes do not result from risk management, so they cannot be a benefit of risk
management.
G. Cost control is improved as a result of risk management because there are fewer
unpleasant surprises and fewer disruptions in the operations of the business.
H. Increased shareholder value is a benefit of risk management because risk management
can minimize losses and maximize opportunities.

Question #56: Bellcon is investigating the two projects shown below.

Required Return Holding Period


Investment A 5.75% 4 years
Investment B 6.25% 4 years
What is the most reasonable conclusion based on this limited information?
Correct Answer: • B. Investment B has greater risk than Investment A.
E. Investment B is the better choice for Bellcon.
F. Investment B has greater risk than Investment A.
G. Investment A has greater risk than Investment B.
H. Investment A is the better choice for Bellcon.
Explanation:
E. The problem gives only the required rates of return for both projects and the lengths of
both projects. Nothing in that information indicates that Investment B is a better choice
than Investment A.
F. From the limited information given, this is the only conclusion that can be drawn. A
higher required rate of return is used in capital budgeting when greater risk is perceived.
The required rate of return is higher for Investment B than it is for Investment A.
Therefore, we can conclude that Investment B has greater risk than Investment A.
G. A higher required rate of return is used in capital budgeting when greater risk is
perceived. Therefore, it is not correct to say that Investment A has greater risk than
Investment B. because Investment As required rate of return is lower than Investment B's,
not higher.
H. The problem gives only the required rates of return for both projects and the lengths of
both projects. Nothing in that information indicates that Investment A is a better choice
than Investment B.

Question #57: Datacomp Industries, which has no current debt, has a beta of 0.95 for its common
stock. Management is considering a change in the capital structure to 30% debt and 70% equity
This change would increase the beta on the stock to 1.05, and the after-tax cost of debt will be
7.5%. The expected return on equities is 16%, and the risk-free rate is 6%. Should Datacomp's
management proceed with the capital structure change?
Correct Answer: • D. Yes, because the weighted average cost of capital will decrease.
E. Yes, because there will be no effect on the weighted average cost of capital.
F. No, because the cost of equity capital will increase.
G. No, because the weighted average cost of capital will increase.
H. Yes, because the weighted average cost of capital will decrease.
Explanation:
E. Though the change should be made, this reason for making the change is incorrect. The
change should be made because the weighted average cost of capital will decrease. If the
weighted average cost of capital will remain the same, there is no need to make the
change.
F. The weighted average cost of capital will decrease if this change is made, so the change
should be made. See the correct answer for a complete explanation.
G. The weighted average cost of capital will decrease if this change is made, so the change
should be made. See the correct answer for a complete explanation.
H. Using the CAPM formula, which is r = r F β (r M – r F), we can determine that the
current cost of capital consisting only of equity for the firm is 15.5%:
r = 0.06 + 0.95(0.16 - 0.06) = 0.155.
The proposed capital structure will have an after tax cost equal to 13.8%:
(0.3 x 0.075) + (0.7 x [0.06 + 1.05(0.16 - 0.06)]) = 0.138.
So, the company should proceed with the capital change as it will reduce the WACC
from 15.5% to 13.8%.

Question #58: Please see the question below:


A company is considering the early retirement of its 10%, 10-year bonds payable. Before retiring
the bonds, the company's capital structure was
Correct Answer: • D. Financial leverage will decrease.
E. Asset turnover ratio will decrease.
F. Return on owner's equity will decrease.
G. Debt-equity ratio will increase.
H. Financial leverage will decrease.
Explanation:
E. Because the amount of debt will be decreased and their will be no change in equity, the
debt-equity ratio will decrease.
F. Because the bonds will be retired there will be less interest expense which will increase
net income. This increase in net income will cause the return on owners equity to
increase.
G. Because of the retirement of the bonds is done by spending cash, the total assets of the
company will decrease. This decrease in total assets will increase the asset turnover ratio.
H. Financial leverage relates to the amount of debt that a company uses for its financing. As
the amount of debt decreases, the leverage of the company will decrease. The fact that the
bonds will be retired at 103.5% of the face value is not relevant to this question.

Question #59: Given an acid test ratio of 2.0, current assets of $5,000, and inventory of $2,000,
the value of current liabilities is
Correct Answer: • C.51.500
E. $3,500
F. $2,500
G. $1,500
H. $6,000
Explanation:
E. This answer adds the value of inventory to the current assets figure instead of subtracting
it.
F. This answer incorrectly includes the value of the inventory in the numerator of the
calculation. Inventory is included in the current ratio, but not the acid test ratio.
G. The acid test ratio is calculated as follows: (Cash + Marketable Securities +
Receivables) / Current Liabilities. Inventory is a current asset, but it is not included in the
acid test ratio calculation. This means that based on the information given here, the
numerator is only $3,000 ($5,000 current assets - $2,000 inventory). Since the ratio is 2,
the current liabilities must be $1,500.
Note that the full information needed to calculate the numerator of the acid test ratio is
not given in this problem. We must assume that there are no pre-paids or other current
assets such as short-term deferred tax assets in current assets in order to calculate the
numerator this way. Pre-paids and other current assets are not included in the numerator
of the acid test, or quick, ratio. However, since total current assets and inventory are the
only information given for calculating the numerator, we must use what we have. But if
the individual components of current assets are given, the proper way to calculate the
numerator is Cash + Marketable Securities + Receivables, excluding any and all other
current assets.
H. This answer multiplies the numerator by 2 instead of dividing it by 2.

Question #60: Which one of the following items would likely increase earnings per share (EPS)
of a corporation?
Correct Answer: • Purchase of treasury stock.
E. An increase in the common stock shares authorized to be issued.
F. Declaration of a stock dividend.
G. Purchase of treasury stock.
H. Declaration of a stock split.
Explanation:
E. An increase in the number of authorized shares will have no impact on EPS because EPS
is based on the number of shares outstanding, not the number of shares authorized.
F. A stock dividend increases the number of shares outstanding and will therefore decrease
EPS.
G. The purchase of treasury stock decreases the number of shares that are outstanding. Since
treasury shares are not included in the EPS calculation, the purchase of treasury shares
will increase EPS by decreasing the number of shares to which the income must be
distributed.
H. A stock split increases the number of shares outstanding and will therefore decrease EPS.
Question #61: If the U.S. dollar declines in value relative to the currencies of many of the U.S.
trading partners. the likely result is that
Correct Answer: D. U.S. exports will tend to increase.
E. The U.S. balance of payments deficit will become worse.
F. Foreign currencies will depreciate against the dollar.
G. U.S. imports will tend to increase.
H. U.S. exports will tend to increase.
Explanation:
E. The decrease in the value of the dollar makes U.S. goods relatively less expensive
relative to foreign produced goods and this will increase U.S. exports. This will improve
the U.S. balance of payments deficit.
F. If the U.S. dollar declines in value (depreciates) relative to the currencies, the foreign
currencies must appreciate against the dollar.
G. The decrease in the value of the dollar makes U.S. goods relatively less expensive
relative to foreign produced goods and this will increase U.S. exports. Similarly, the
decline in the value of the dollar will make foreign goods relatively more expensive in
the U.S. and this will decrease imports.
H. The decrease in the value of the dollar makes U.S. goods relatively less expensive
relative to foreign produced goods and this will increase U.S. exports. This will improve
the U.S. balance of payments deficit.

Question #62: Oak Fine Furnishings manufactures a wide range of home furnishings. One of
their products is an oak headboard. The company currently sells 4,000 headboards at an average
price of $100 per unit. To manufacture the headboards, the variable costs are $55 per unit and the
total fixed costs assigned to the oak headboard are $150,000.1f the sale of headboards increases
by 50% and all else remains constant, this would result in
Correct Answer: • B. earnings before interest and taxes of $120,000.
E. A 50% increase in earnings before interest and taxes.
F. Earnings before interest and taxes of $120,000.
G. Fixed costs of $225,000.
H. A gross margin of 3380.000.
Explanation:
E. Earnings will not increase by the same percentage as revenues increase because of the
presence of fixed costs within the company's cost structure. Fixed costs do not change
with changes in volume as long as the volume remains within the relevant range. Because
fixed costs assigned to headboard will not increase when sales increase, the increase in
earnings attributable to headboards should be more than 50%.
F. The contribution margin per unit is $45 ($100 - $55). With a 50% increase in sales, sales
will become 6,000 units and earnings before interest and taxes will become (6,000 x $45)
- $150,000 = $120,000.
G. By definition, even if production increases, the fixed costs will remain the same, or
$150,000.
H. This answer results from increasing revenue by 50% without also increasing variable
costs by 50%. Variable costs must increase as well.

Question #63: Which of the following statements is not a responsibility for an IMA member
regarding the "Credibility" standard of ethical conduct?
Correct Answer: • A. To recognize and communicate professional limitations or other constraints
that would preclude responsible judgment or successful performance of an activity.
E. To recognize and communicate professional limitations or other constraints that would
preclude responsible judgment or successful performance of an activity.
F. To disclose all relevant information that could reasonably be expected to influence an
intended user’s understanding of the reports, analyses, or recommendations.
G. To disclose delays or deficiencies in information, timeliness, processing, or internal
controls in conformance with organization policy and/or applicable law.
H. To communicate information fairly and objectively.
Explanation:
E. To recognize and communicate professional limitations or other constraints that would
preclude responsible judgment or successful performance of an activity is a responsibility
of an IMA member with respect to the "Competence standard of ethical conduct.
F. With respect to the "Credibility" standard of ethical professional conduct, each IMA
member has a responsibility to:
4) Communicate information fairly and objectively.
5) Disclose all relevant information that could reasonably be expected to influence an
intended user's understanding of the reports, analyses, or recommendations.
6) Disclose delays or deficiencies in information, timeliness, processing, or internal
controls in conformance with organization policy and/or applicable law.
G. With respect to the "Credibility" standard of ethical professional conduct, each IMA
member has a responsibility to:
4) Communicate information fairly and objectively.
5) Disclose all relevant information that could reasonably be expected to influence an
intended user's understanding of the reports, analyses, or recommendations.
6) Disclose delays or deficiencies in information, timeliness, processing, or internal
controls in conformance with organization policy and/or applicable law.
H. With respect to the "Credibility" standard of ethical professional conduct, each IMA
member has a responsibility to:
4) Communicate information fairly and objectively.
5) Disclose all relevant information that could reasonably be expected to influence an
intended user's understanding of the reports, analyses, or recommendations.
6) Disclose delays or deficiencies in information, timeliness, processing, or internal
controls in conformance with organization policy and/or applicable law.

Question #64: Please see the Question Below:


Select information from a company’s year-end balance sheet is shown below.
Balance Sheet
As of December 31, Year 1

Cash $50,000
Accounts receivable 120,000
Inventory 75,000
Property, plant and equipment, net 250,000
Total assets $495,000

Accounts payable $35,000


Long-term debt 100,000
Total liabilities $135,000

Common stock $300,000


Retained earnings 100,000
Total equity $360,000
Total liabilities and equity $495,000

Based on the above information, a common-size balance sheet for the company will show
Correct answer: A. accounts receivable at 24%.
E. Accounts receivable at 24%
F. Property, plant and equipment, net at 69%
G. Long-term debt at 74%
H. Retained earnings at 17%
Explanation:
E. A vertical common-size financial statement expresses each component as a percentage of
a total. For example, fixed assets will not be stated as a dollar amount but rather will be
stated as a percentage of total assets. Accounts receivable of $120,000 divided by total
assets of $495,000 are 24.2% of total assets.
F. This is property, plant and equipment, net, of $250,000 divided by total equity of
$360,000. A vertical common-size financial statement expresses each component as a
percentage of a total. That total is usually total assets (for a balance sheet). It does not
need to be total assets, but unless a question says differently, we should assume the use of
total assets. For example, fixed assets will not be stated as a dollar amount but rather will
be stated as a percentage of total assets.
G. This is long-term debt of $100,000 divided by total liabilities of $135,000. A vertical
common-size financial statement expresses each component as a percentage of a total.
That total is usually total assets (for a balance sheet). It does not need to be total assets,
but unless a question says differently, we should assume the use of total assets. For
example, fixed assets will not be stated as a dollar amount but rather will be stated as a
percentage of total assets.
H. This is retained earnings of $60,000 divided by total equity of $360,000. A vertical
common-size financial statement expresses each component as a percentage of a total.
That total is usually total assets (for a balance sheet). It does not need to be total assets,
but unless a question says differently, we should assume the use of total assets. For
example, fixed assets will not be stated as a dollar amount but rather will be stated as a
percentage of total assets.

Question #65: Which one of the following describes a disadvantage to a firm that issues
preferred stock?
Correct Answer: • B. Preferred stock is usually sold on a higher yield basis than bonds.
E. Preferred stock typically has no maturity date.
F. Preferred stock is usually sold on a higher yield basis than bonds.
G. Most preferred stock is owned by corporate investors.
H. Preferred stock dividends are legal obligations of the corporation.
Explanation:
E. Preferred stock usually does not have a maturity date, but that is not a disadvantage to the
firm that issues it.
F. Because equity carries no guarantee of a return, investors typically demand a higher yield
to compensate for the higher risk.
G. Preferred stock can be owned by corporate investors, institutional investors, or private
investors.
H. The payment of a preferred stock dividends is discretionary. Missing a dividend payment
will not be a default on the part of the issuer. The board of directors can omit paying a
preferred stock dividend if it chooses. Therefore, preferred stock dividends are not legal
obligations of the corporation.

Question #66: After completing a marketplace analysis of Product Z, a company's accountant has
determined that a price change from $25 to $20 will result in a demand increase for Product Z
from 1,000 units to 1.500 units. Based on the information provided, what is the price elasticity of
demand for Product Z using the midpoint formula?
Correct Answer: • C. 1.80.
E. 1.33
F. 0.56.
G. 1.80.
H. 2.50.
Explanation:
E. This answer results from an attempt to calculate the price elasticity of demand using the
percentage method. It is incorrect for two reasons:
(3) The calculation is done incorrectly for the percentage method. For the percentage
method, the percentages of change in the quantity and the price should be calculated
by dividing the amount of change by the initial quantity and price, whereas the
percentages used in this answer were calculated by dividing the amount of change by
the changed quantity and price.
(4) The question says to use the midpoint method, not the percentage method, to
calculate the price elasticity of demand. Using the midpoint method. the formula for
the price elasticity of demand is:
(Q2 -Q1)/(Q2 +Q1)/2]
Ed= ……………………………
(P2 -P 1 )/[(P 2 +P 1)/2]
Where: Q 1 and 2 = First and second
quantity points
P 1 and 2 = First and second price
points
F. Using the midpoint method. the formula for the price elasticity of demand is:
(Q2 -Q 1)/[(Q2 +Q1)/2]
Ed=………………………….
(P2 -P1)/[(P 2 +P 1)/2]
Where: Q 1 and 2 = First and second
quantity points
P 1 and 2 = First and second price
points
This answer results from reversing the numerator and denominator in the formula, as follows:
(P2 -P 1)/[(P 2+P 1)/2]
Ed= …………………………….
(Q2 -Q1)/[(Q2 +Q1)/2]
G. Using the midpoint method, the formula for the price elasticity of demand is:
(Q2 -Q 1)/[(Q2 +Q1)/2]
Ed=…………………………………..
(P2 -P1)/[(P 2+P 1)/2]
Where: Q 1 and 2 = First and second
quantity points
P 1 and 2 = First and second price points
500 / 1.250 0.400
Ed=……………………………… 1.80
$5/ $22.50 0.222
H. This is the price elasticity of demand using the percentage method. The question says to
use the midpoint method, not the percentage method. Using the midpoint method. the
formula for the price elasticity of demand is:
(Q2 -Q1)/[(Q2 -Q1)/2] E d (P2 -P1)/[(P 2 -P 1)/2]
Where: Q 1 and 2 = First and second
quantity points
P 1 and 2 = First and second price points

Question #67: The management of a company is considering making a capital investment to


acquire a machine for its manufacturing facility at a total cost of €600.000 for equipment and
installation. The machine has a useful life of 5 years and a zero salvage value at the end of its
useful life. The management of the company uses the straight-line depreciation method for all
machinery acquired. How much would the company's annual tax savings be upon acquiring the
machine if the company's income tax rate is 30%?
Correct Answer: • B. €36.000.
E. €120000.
F. €36.000.
G. €180.000.
H. €84.000.
Explanation:
E. €120,000 is the amount of the annual depreciation on the equipment. The annual tax
savings for the company is the amount of the annual depreciation multiplied by the
company's tax rate.
F. The annual depreciation expense will be €600,000 + 5, or €120,000. At an income tax
rate of 30%, the depreciation tax shield will be 30% of €120,000, or €36,000.
G. €180,000 is 30% of the €600,000 investment. The annual tax savings for the company is
the amount of the annual depreciation on the investment multiplied by the company's tax
rate.
H. €84,000 is the amount of the annual depreciation less the amount of the depreciation tax
shield. The annual tax savings for the company is only the amount of the depreciation tax
shield.

Question #68: When is it appropriate to communicate potential ethical issues to authorities or


individuals not employed or engaged by your organization?
Correct Answer: • D. You believe there is a clear violation of the law.
E. Your immediate supervisor is involved and you are concerned about potential negative
consequences for yourself if you follow established company policy.
F. Your immediate supervisor, who is aware of the issue but is not involved, does not want
to take any action because he is concerned about potential negative consequences for
himself if the matter is addressed.
G. You have escalated the issue, as directed by company policy, but are not satisfied with
the company's response.
H. You believe there is a clear violation of the law.
Explanation:
E. If your immediate supervisor is involved, then you should present the issue to the next
level up, regardless of the potential for negative consequences.
F. If your immediate supervisor is not willing to take any action to resolve the ethical
conflict, then you should submit the issue to the next management level. Contact with
levels above the immediate superior should be initiated only with your superior's
knowledge, assuming he or she is not involved.
G. The highest acceptable reviewing authority may be a group such as the audit committee,
executive committee, board of directors, board of trustees, or owners. If you have taken
your concerns all the way up to this level without receiving any resolution of the ethical
conflict, it is still not appropriate to communicate the problem to authorities or
individuals outside the organization, unless you believe a clear violation of the law has
taken or is taking place.
H. If the law is clearly being violated, and if you have not been able to obtain satisfactory
resolution of the ethical conflict through other means, then it may be necessary to inform
authorities.
Question #69: Watson Corporation computed the following items from its financial records for
the year just ended: Price-earnings ratiol2 Payout ratio 0.6 Asset turnover 0.9 The dividend yield
on Watson's common stock is
Correct Answer: D. 5.0%
E. 7.5%
F. 7.2%
G. 10.8%
H. 5.0%
Explanation:
E. This is the asset turnover ratio (0.9) divided by the P-E ratio (12). The dividend yield is
calculated as the cash dividend per share divided by the market price per share. Since we
are not given either of these items directly, we need to calculate them from the
information that is given. See the correct answer for a complete explanation.
F. This is the P-E ratio (12) multiplied by the payout ratio (0.6). The dividend yield is
calculated as the cash dividend per share divided by the market price per share. Since we
are not given either of these items directly, we need to calculate them from the
information that is given. See the correct answer for a complete explanation.
G. This is the P-E ratio (12) multiplied by the asset turnover ratio (0.9). The dividend yield
is calculated as the cash dividend per share divided by the market price per share. Since
we are not given either of these items directly, we need to calculate them from the
information that is given. See the correct answer for a complete explanation.
H. The dividend yield is calculated as the cash dividend per share divided by the market
price per share. Since we are not given these items directly, we need to calculate them
from the information that is given. Let us assume that the P-E ratio is made up of a $12
market price and $1 of earnings. This gives us the P-E ratio of 12 that we are told the
company has. The payout ratio tells us what percentage of earnings were distributed as
cash, and since it is 0.6, in this example $0.60 would have been distributed to the
shareholders. We now know the necessary information to calculate the dividend yield.
The market price is $12 and the cash dividend was $0.60. $0.60 divided by $12 equals a
dividend yield of 5%.

Question #70: How can a management accountant use the Fraud Triangle to identify and manage
the risk of fraud?
Correct Answer: • C. The Fraud Triangle provides a model for explaining the pressures,
rationalizations, and opportunities that influence people to commit fraud.
E. The Fraud Triangle provides a model for explaining how persuasion, coercion, and
conviction influence people to commit fraud.
F. The Fraud Triangle provides a model for explaining the motives, means, and
opportunities that influence people to commit fraud.
G. The Fraud Triangle provides a model for explaining the pressures, rationalizations, and
opportunities that influence people to commit fraud.
H. The Fraud Triangle provides a SOX-compliant model for examining the company's
internal control environment in terms of its risk of fraud.
Explanation:
E. The Fraud Triangle does not provide a model for explaining how persuasion, coercion,
and conviction influence people to commit fraud.
F. The Fraud Triangle does not provide a model for explaining the motives, means, and
opportunities that influence people to commit fraud.
G. According to the Fraud Triangle model, three conditions need to be present for an
employee to commit a fraudulent act against an employer. The three conditions are
pressure, opportunity, and rationalization.
H. The Fraud Triangle does not provide a SOX-compliant model for examining the
company's internal control environment in terms of its risk of fraud.

Question #71: What variable is measured on the horizontal axis of the yield curve?
Correct Answer: • D. Years to maturity of the bonds.
E. Par value of the bonds.
F. Duration of the bonds.
G. Yield of the bonds.
H. Years to maturity of the bonds.
Explanation:
E. The par value of an individual bond is its stated amount or its face value. The par value of
bonds is not represented graphically on a yield curve graph.
F. The duration of an individual fixed income security is a weighted average of the times
until the receipt of both interest and principal, weighted according to the proportion of the
total present value of the bond represented by the present value of each cash flow to be
received. Duration is not represented graphically on a yield curve graph.
G. The yield curve shows the market rates for various maturities of bonds on a given date.
The market rates, or the yields, are on the vertical axis, not the horizontal axis.
H. The yield curve shows the market rates for various maturities of bonds on a given date.
The years to maturity of the bonds are on the horizontal axis and the market rates are on
the vertical axis.

Question #72: Donnelly Corporation manufactures and sells T-shirts imprinted with college
names and slogans. Last year, the shirts sold for $7.50 each, and the variable cost to manufacture
them was $2.25 per unit. The company needed to sell 20,000 shirts to break even. The net
income last year was $5.040. Donnelly's expectations for the coming year include the following:
-The sales price of the T-shirts will be $9 -Variable cost to manufacture will increase by one-
third -Fixed costs will increase by 10% -The income tax rate of 40% will be unchanged If
Donnelly Corporation wishes to earn $22,500 in net income for the coming year. the company's
sales volume in dollars must be
Correct Answer: • A. $229.500
E. $229,500
F. Some amount other than those given.
G. $257,625
H. $213,750
Explanation:
E. This question is asking for the sales revenue given a requirement for an after-tax net
income of $22,500. To solve it, we need to use the version of the breakeven formula for
determining the sales revenue required to result in a specific dollar amount of profit.
Target Sales Revenue = (FC + Target Pretax Income) /Contribution Margin Ratio In
order to use this formula, we need to calculate three things:
(4) the amount of fixed cost in the coming year,
(5) the amount of desired net income before tax for the coming year. and
(6) the contribution margin ratio for the coming year.
(4) We are told that fixed cost for the coming year will be 10% higher than the
previous year's fixed cost. So we need to find what the fixed cost was for last
year. We know the break-even point in units for last year (20,000) and we know
the unit contribution margin for last year ($7.50 - $2.25 = $5.25). So we can find
the fixed cost for last year by using the Break-Even Point in Units formula and
solving for FC: FC / Unit Contribution Margin = BEP in Units.
FC / 5.25 = 20,000 Solving for FC, we get FC = $105,000. Since fixed cost for
the coming year will be 10% higher than last year, fixed cost for the coming year
will be $105,000 x 1.10, which is $115,500.
(5) The formula to find before-tax net income when we know the after-tax net income
is After-Tax NI / (1 - tax rate). Therefore, the desired before tax net income is
$22,500 / (1- 0.40), which is $37,500.
(6) We are told that variable cost to manufacture will increase by one-third. Variable
cost last year was $2.25 per unit. Therefore, variable cost in the coming year will
increase by 1/3 of $2.25, which is $0.75, so variable cost will be $3 per unit. We
are told that the sales price will be $9. Therefore, the Contribution Margin Ratio
will be $6 / $9, which is 2/3 or 0.666667. Now, we can calculate the Target Sales
Revenue, because we have the fixed cost, the target pretax income, and the
contribution margin ratio for the coming year. Target Sales Revenue = ($115,500
+ $37.500) /0.666667 = $229,500.
F. The correct answer is given.
G. This answer results from calculating the desired before-tax net income by dividing the
after-tax net income by the tax rate. To calculate the desired before-tax net income, the
after-tax net income should be divided by (1- the tax rate).
H. This answer results from using fixed costs of $105.000 to calculate the target sales in
dollars. Fixed costs are expected to increase by 10% in the coming year.

Question #73: Ray Corporation has long-term debt of $1,200,000 and equity of $1.000.000. The
board of directors has set a goal of 1:1 for the company's debt-equity ratio. Which of the
following could the company employ to achieve this goal?
Correct Answer: • Issuing rights to purchase new common stock.
E. Paying a stock dividend to the existing shareholders.
F. Paying a dividend on its common stock.
G. Issuing new bonds.
H. Issuing rights to purchase new common stock.
Explanation:
E. Payment of a stock dividend would not change the debt-equity ratio. When a stock
dividend is declared, the corporation does not pay out assets or incur a liability. It issues
additional shares of stock to each stockholder and nothing more. Total equity remains the
same.
F. All dividends, except for stock dividends, reduce the total stockholders' equity in the
corporation, because the equity is reduced either through an immediate or promised
future distribution of assets. Therefore, payment of a dividend on its common stock
would increase the debt-equity ratio, since it would decrease equity. This is the opposite
of management's goal, because the debt-equity ratio now is 1.2:1, and management wants
to decrease it to 1:1.
G. Issuing new bonds would increase long-term debt, which would increase the debt-equity
ratio. This is the opposite of management's goal, because the debt-equity ratio now is
1.2:1, and management wants to decrease it to 1:1.
H. The debt-equity ratio measures how much long-term debt a company has compared to its
total equity. A ratio in excess of 1:1 indicates more reliance on long-term debt financing
than on equity financing. Issuing rights to purchase new common stock would result in
increased equity because some of the rights would be exercised to purchase the newly-
issued stock. That would increase equity, which could decrease the debt-equity ratio.

Question #74: A example of a hazard risk is


Correct Answer: • C. the risk of the death of the company's CEO.
E. the risk of a major customer going bankrupt while owing a large amount.
F. the risk of a lawsuit.
G. the risk of the death of the company's CEO.
H. the risk of a recession occurring.
Explanation:
E. The risk of a major customer going bankrupt while owing a large amount is a credit risk,
and credit risk is a type of financial risk Financial risks are risks that are connected with
the company's financial health.
F. The risk of a lawsuit is an operational risk. Operational risks are risks that result from
inadequate or failed internal processes, people or systems.
G. A hazard risk is the type of risk that can be insured against. For example, the risk of a
natural disaster such as a fire or flood can be managed with property insurance: the risk
of the death of a key employee can be managed with key person life insurance: and the
risk of a person getting injured on the premises can be managed with liability insurance.
H. The risk of a recession occurring is a strategic risk. Strategic risks are risks that occur on
a more macro level such as political risks, economic risks (i.e., the risk of a recession),
the risk of customers' needs changing, or even some risks that are specific to the company
itself, such as risk to the company's reputation or to its patents or trademarks.

Question #75: Jenson Copying Company is planning to buy a copying machine costing $25,310.
The net present values (NPV) of this investment, at various discount rates, are as follows.

Discount Rate NPV


4% $2,440
6% 1,420
8% 460
10% (440)

Jenson's approximate internal rate of return on this investment is


Correct Answer: • D. 9%.
E. 6%.
F. 10%.
G. 8%.
H. 9%.
Explanation:
E. The internal rate of return is the discount rate at which the NPV of a project is zero. At a
discount rate of 6%, the NPV of the project is $1.420; so 6% cannot be the internal rate of
return for this project.
F. The internal rate of return is the discount rate at which the NPV of a project is zero.
Therefore. the IRR of this project will be the discount rate in between the discount rate of
the lowest positive NPV (the positive amount closest to zero) which is $460 and the
discount rate of the lowest negative NPV (the negative amount closest to zero) which is
($440). 10% is the discount rate of the lowest negative NPV.
G. The internal rate of return is the discount rate at which the NPV of a project is zero.
Therefore. the IRR of this project will be the discount rate in between the discount rate of
the lowest positive NPV (the positive amount closest to zero) which is $460 and the
discount rate of the lowest negative NPV (the negative amount closest to zero) which is
($440). 8% is the discount rate of the lowest positive NPV.
H. The internal rate of return is the discount rate at which the NPV of a project is zero.
Therefore. the IRR of this project will be the discount rate in between the discount rate of
the lowest positive NPV (the positive amount closest to zero) which is $460 and the
discount rate of the lowest negative NPV (the negative amount closest to zero) which is
($440). The discount rate of the lowest positive NPV is 8% and the discount rate of the
lowest negative NPV is 10%. Therefore, the IRR is in between 8% and 10%. Since the
size of the positive NPV and the size of the negative NPV that go with the 8% and 10%
discount rates are approximately equal, we also know that the IRR must be almost
exactly in between 8% and 10%. The only answer choice between 8% and 10% is 9%.

Question #76: Locar Corporation had net sales last year of $18.600.000 (of which 20% were
installment sales). It also had an average accounts receivable balance (including the installment
receivables) of $1.380.000. Credit terms are 2/10, net 30. Based on a 360-day year. Locar's
average collection period last year was
Correct Answer: • A 26.7 days.
E. 26.7 days.
F. 27.3 days.
G. 26.2 days.
H. 33.4 days.
Explanation:
E. This is not the correct answer. Please see the correct answer for a complete explanation.
F. This answer results from reducing the accounts receivable balance by 2% (to reflect the
discount for early payment) before calculating the average collection period. The
accounts receivable balance should be used as given. Although the term net annual credit
sales" is used to calculate the average collection period and the accounts receivable
turnover period, "net sales" refers to gross sales less returns and allowances, not
receivables less the discount for prompt payment (and not sales net of uncollectible
accounts. either).
G. This can be calculated either by including the installment sales in both the sales and the
receivables amounts, or by excluding the installment sales from both the sales and the
receivables amounts. The number of days in receivables (average collection period) is
360 / accounts receivable turnover ratio. Using the sales and receivables including the
installment sales, the accounts receivable turnover ratio is $18,600,000 ÷ $1,380,000,
which is 13.48 times. The average collection period is 360 ÷ 13.48= 26.7 days. Using the
sales and receivables excluding the installment sales, sales would be $18.600.000 x 0.80
= $14,880,000 and receivables would be $1,380,000 x 0.80 = $1,104,000. The accounts
receivable turnover ratio is $14,880,000 ÷ $1,104,000 = 13.48 times. And the average
collection period is 360 ÷13.48 = 26.7 days.
H. This answer results from reducing the sales by the 20% that were installment sales but
not reducing the average accounts receivable balance by 20%. The average collection
period can be calculated either by including the installment sales in both the sales and the
receivables amounts, or by excluding the installment sales from both the sales and the
receivables amounts. But including the installment sales in one amount while excluding
them from the other amount does not work. Question 180sales and the receivables
amounts. But including the installment sales in one amount while excluding them from
the other amount does not work.

Question #77: Elements of project risk identification include which one of the following?
Correct Answer: • A. Interviews and observations.
E. Interviews and observations.
F. Cost estimates.
G. Actual risk events.
H. Activity duration estimates.
Explanation:
E. Interviews and observations are risk identification techniques. Risk identification
techniques include the following: -Event inventories, detailed lists of potential events or
an archive of events that have occurred: -Internal analysis, for example the company's
own experience with similar projects: -Escalation or threshold triggers to alert
management to areas of concern: -Facilitated workshops used to identify events by
drawing on accumulated knowledge and experience of management, staff and other
stakeholders: -Interviews, questionnaires, and surveys: -Process flow analysis using flow
diagrams to consider the inputs, tasks, responsibilities, and outputs of the project: -
Leading event indicators to identify conditions that could give rise to an event: -Loss
event data on past loss events: and -Brainstorming sessions.
F. Cost estimates are not risk identification techniques. Risk identification techniques
include the following: -Event inventories, detailed lists of potential events or an archive
of events that have occurred: -Internal analysis, for example the company's own
experience with similar projects: -Escalation or threshold triggers to alert management to
areas of concern: -Facilitated workshops used to identify events by drawing on
accumulated knowledge and experience of management, staff and other stakeholders: -
Interviews, questionnaires, and surveys: -Process flow analysis using flow diagrams to
consider the inputs, tasks, responsibilities, and outputs of the project: -Leading event
indicators to identify conditions that could give rise to an event: -Loss event data on past
loss events: and -Brainstorming sessions.
G. Actual risk events are not risk identification techniques, though experience with historical
events is used in risk identification techniques. Risk identification techniques include the
following: -Event inventories, detailed lists of potential events or an archive of events
that have occurred: -Internal analysis, for example the company's own experience with
similar projects: -Escalation or threshold triggers to alert management to areas of
concern: -Facilitated workshops used to identify events by drawing on accumulated
knowledge and experience of management, staff and other stakeholders: -Interviews,
questionnaires, and surveys: -Process flow analysis using flow diagrams to consider the
inputs, tasks, responsibilities, and outputs of the project: -Leading event indicators to
identify conditions that could give rise to an event: -Loss event data on past loss events:
and -Brainstorming sessions.
H. Activity duration estimates are not risk identification techniques. Risk identification
techniques include the following: -Event inventories, detailed lists of potential events or
an archive of events that have occurred: -Internal analysis, for example the company's
own experience with similar projects: -Escalation or threshold triggers to alert
management to areas of concern: -Facilitated workshops used to identify events by
drawing on accumulated knowledge and experience of management, staff and other
stakeholders: -Interviews, questionnaires, and surveys: -Process flow analysis using flow
diagrams to consider the inputs, tasks, responsibilities, and outputs of the project: -
Leading event indicators to identify conditions that could give rise to an event: -Loss
event data on past loss events: and -Brainstorming sessions.

Question #78: Financial risk management is


Correct Answer:
• B. the use of financial instruments to manage exposure to credit, market and other financial
risks to create economic value to the company.
E. deciding which risks must be addressed and prioritizing them.
F. the use of financial instruments to manage exposure to credit, market and other financial
risks to create economic value to the company.
G. managing risks connected with the day-to-day operations of the company.
H. the level of risk that resides with an event or process prior to management taking a
mitigating action.
Explanation:
E. Deciding which risks must be addressed and prioritizing those that must be addressed is
one of the steps in the management of any type of risk, not only financial risk
management.
F. The process of using financial instruments to manage exposure to credit, market and
other financial risks in order to create value for the company is financial risk
management.
G. This is the definition of operational risk management, not financial risk management.
H. This is the definition of inherent risk.

Question #79: Calvin Inc. is considering the purchase of a new state-of-the-art machine to
replace its hand-operated machine. Calvin's effective tax rate is 40%. and its cost of capital is
12%. Data regarding the existing and new machines are presented below.
Existing New
machine machine
Original cost $50,000 $90,000
Installation cost 0 4,000
Freight and insurance 0 6,000
Expected end salvage
value 0 0
Depreciation - -
Expected useful life 10 years 5 years

The existing machine has been in service for seven years and could be sold currently for
$25,000. If the new machine is purchased, Calvin expects to realize a $30,000 before-tax annual
reduction in labor costs.
If the new machine is purchased, what is the net amount of the initial cash outflow at Time 0 for
net present value calculation purposes?
Correct Answer: A. $79,000.
All Possible Answers:
E. $79.000.
F. $65,000.
G. $75.000.
H. $100.000.
Explanation:
E. The existing machine originally cost $50,000 and it has been in service for 7 years. Its
expected useful life was 10 years when it was purchased and it is being depreciated on
the straight line basis. Therefore, $5,000 is being depreciated each year ($50,000 + 10).
So the book value of the existing machine at the time of replacement would be $50.000 -
(7 x $5,000), which is $15,000. If it is sold for $25,000, there will be a taxable gain of
$10,000 on the sale ($25,000 - $15,000). The company's tax rate is 40%, so the tax on the
gain will be 40% of $10,000, which is $4.000. The cost of the new machine, the
installation and the freight and insurance on its shipment will all be capitalized, so the tax
rate will not affect those costs in Year O. Therefore, the Year 0 net cash outflow will be:
Outflows for capitalized equipment: ($90,000) + ($4,000) + ($6,000) = ($100.000) Inflow
from sale of existing equipment: $25,000 before tax Outflow for tax on gain on sale of
existing equipment: ($4,000) The net cash outflow is ($100,000) + $25,000 + ($4,000) =
($79,000)
F. This is the cost of the new machine minus the sale price of the existing machine.
However, it does not include the installation cost_ the freight and insurance, or the tax
due on the gain on the sale of the existing machine.
G. This is the cost of the new machine plus the installation cost and freight and insurance
cost minus the sale price of the existing machine. However, it does not include the tax
due on the gain on the sale of the existing machine.
H. This is the cost of the new machine plus the installation cost and the freight and insurance
cost However, it does not include the net after-tax cash to be received from the sale of the
existing machine.

Question #80: Austin Manufacturing, which is subject to a 40% income tax rate, had the
following operating data for the period just ended.
Selling price per unit $60
Variable cost per unit $22
Fixed costs $504,000
Management plans to improve the quality of its sole product by (1) replacing a component that
costs $3.50 with a higher-grade unit that costs $5.50, and (2) acquiring a $180,000 packing
machine. Austin will depreciate the machine over a 10-year life with no estimated salvage value
by the straight-line method of depreciation. If the company wants to earn after-tax income of
$172,800 in the upcoming period, it must sell
Correct Answer: C.22.500 units.
E. 23,800 units.
F. 21,316 units.
G. 22,500 units.
H. 19,300 units.
Explanation:
E. This answer results from adding the full cost of the new machine and the desired after-tax
net income to the present fixed costs to calculate the numerator of the calculation of the
number of units to be sold. The full cost of the new machine should not be used, only one
year of depreciation expense on it should be used. And the after-tax net income needs to
be converted to before-tax net income before using it in the calculation.
F. This answer results from dividing the new fixed cost plus the required pre-tax net income
by the current unit contribution margin. The unit contribution margin will change because
of replacing the current component that costs $3.50 with a higher-grade unit that costs
$5.50.
G. Replacing the component with a higher priced component will add $2 to variable costs.
The new variable cost will be $24 per unit and the new contribution margin will be $60 -
$24, or $36. Acquiring the packing machine will add $18,000 in new depreciation
expense to fixed costs ($180,000 + 10 years life), so the new fixed cost will be $522,000.
In order to earn after-tax income of $172,800, pre-tax income needs to be $288,000
($172,800 + (1- 40%). Fixed costs of $522,000 + desired pre-tax income of $288,000
divided by the unit contribution margin of $36 = 22,500 units the company needs to sell
to earn the desired after-tax net income of $172,800.
H. This answer results from using the after-tax desired profit as part of the numerator in the
calculation of the number of units of sales required. The after-tax desired profit needs to
be converted to before-tax profit before using it in the calculation.

Question #81: Moorehead Manufacturing Company produces two products for which the
following data have been tabulated. Fixed manufacturing cost is applied at a rate of $1.00 per
machine hour.
Per Unit XY-7 BD-4
Selling price $4.00 $3.00
Variable manufacturing cost $2.00 $1.50
Fixed manufacturing cost $0.75 $0.20
Variable selling cost $1.00 $1.00
The sales manager has had a $160,000 increase in the budget allotment for advertising and wants
to apply the money to the most profitable product. The products are not substitutes for one
another in the eyes of the company's customers.
Suppose Moorehead has only 100,000 machine hours that can be made available to produce
additional units of XY-7 and BD-4. If the potential increase in sales units for either product
resulting from advertising is far in excess of this production capacity, which product should be
advertised and what is the estimated increase in contribution margin earned?
Correct Answer: D. Product BD-4 should be produced, yielding a contribution margin of
$250,000.
E. Product BD-4 should be produced, yielding a contribution margin of $187,500.
F. Product XY-7 should be produced, yielding a contribution margin of $75,000.
G. Product XY-7 should be produced, yielding a contribution margin of $133,333.
H. Product BD-4 should be produced, yielding a contribution margin of $250,000.
Explanation:
E. This is not the correct answer. Please see the correct answer for an explanation. We have
been unable to determine how to calculate this incorrect answer choice. If you have
calculated it, please let us know how you did it so we can create a full explanation of why
this answer choice is incorrect. Please send us an email at admin@cmapass.com. Include
the Topic Name, Question Details or Screenshot and the actual incorrect answer choice --
not its letter, because that can change with every study session created.. Thank you in
advance for helping us to make your CMA PASS study materials better.
F. This is not the correct answer. Please see the correct answer for an explanation. We have
been unable to determine how to calculate this incorrect answer choice. If you have
calculated it, please let us know how you did it so we can create a full explanation of why
this answer choice is incorrect. Please send us an email at admin@cmapass.com. Include
the Topic Name, Question Details or Screenshot and the actual incorrect answer choice --
not its letter, because that can change with every study session created.. Thank you in
advance for helping us to make your CMA PASS study materials better.
G. This is not the correct answer. Please see the correct answer for an explanation. We have
been unable to determine how to calculate this incorrect answer choice. If you have
calculated it, please let us know how you did it so we can create a full explanation of why
this answer choice is incorrect. Please send us an email at admin@cmapass.com. Include
the Topic Name, Question Details or Screenshot and the actual incorrect answer choice --
not its letter, because that can change with every study session created.. Thank you in
advance for helping us to make your CMA PASS study materials better.
H. In this problem it is necessary to optimize the contribution margin of a scarce resource.
The scarce resource in this problem is machine hours. XY-7 BD-4 Sales price $4.00
$3.00 Variable costs $3.00 $2.50 Contribution margin $1.00 $0.50
Fixed manufacturing costs are applied based on machine hours. Therefore, the machine hours
required to produce one unit of product XY-7 are 0.75 ($0.75 + $1.00) and 0.20 for product BD-
4 ($0.20 + $1.00). This means that the CM per machine hour for XY-7 is $1.33 ($1.00 + 0.75),
and the CM per machine hour for BD-4 is $2.50 ($0.50 +0.20). Since BD-4 has the highest
contribution margin per machine hours used and since the potential increase in sales units
resulting from the advertising is far in excess of this production capacity, we know that all that
Moorhead can produce can be sold. Therefore, only BD-4 should be produced. Since 100,000
machine hours are available and each unit of BD-4 requires 0.20 hour to produce, the company
can produce 500,000 units (100,000 x 0.20). The total contribution margin will be $250,000
($0.50 UCM x 500,000 units sold). It is always best to optimize the contribution margin of the
scarce resource.
Question #82: Sands, Inc. uses a calendar year for financial reporting. The company is
authorized to issue 5,000.000 shares of $10 par common stock. At no time has Sands issued any
potentially dilutive securities. Listed below is a summary of Sands' common stock activities.
Number of common shares issued and outstanding at Dec. 31, 20X6:1,000,000 Shares issued as a
result of a 10% stock dividend on Sept. 30, 20X7:100,000 Shares issued for cash on March 31,
20X8:1.000,000 Number of common shares issued and outstanding at Dec. 31, 20X8:2,100,000
A two-for-one stock split of Sands' common stock took place on March 31, 20X9. The weighted-
average number of common shares used in computing earnings per common share for 20X8 on
the 20X9 comparative income statement was:
Correct Answer: D. 3.700.000.
E. 2,100,000.
F. 1,600,000.
G. 1,850,000.
H. 3,700,000.
Explanation:
E. This answer assumes that the shares that were issued in 20X8 were outstanding for the
entire period. In fact, they were outstanding for only 9 months and therefore should be
considered to be only 750,000 (1,000,000 x 9/12) shares for the purposes of the
weighted-average number of common shares outstanding calculation.
F. This answer incorrectly assumes that the shares issued on March 31. 20X8 were
outstanding for only 6 months of the year, instead of 9 months.
G. At the end of 20X7 there were 1,100,000 shares outstanding. To this, the company needs
to add the effect of any 20X8 share transactions. The only transaction was the issuance of
1,000,000 shares on March 31. Since these shares were outstanding for only 9 months of
the year, they will count as only 75% for the weighted-average share calculation. These
shares will become 750,000 shares for this calculation, giving the company 1,850,000
shares outstanding for the 20X8 EPS calculation.
H. This answer includes the effect of the 2-for-1 stock split that occurred on March 31.
20X9. However, the question asks for the weighted-average number of common shares
outstanding for 20X8 on the 20X8 comparative income statement. The stock split did not
take place until after the 20X8 comparative income statement was issued.
Question #84: A company has a current ratio of 2.0. Cash is 20%, accounts receivable is 40%,
and inventory is 40% of total current assets. What is the acid-test ratio for the company?
Correct Answer: B. 1.2.
E. 1.6.
F. 1.2.
G. 2.0.
H. 0.8.
Explanation:
E. This is Accounts Receivable + Inventory divided by Current Liabilities. The acid test (or
quick) ratio is Cash + Marketable Securities + Net Accounts Receivable divided by
Current Liabilities.
F. With a question like this, it is best to convert the percentages to currency a mounts. So
let's say cash is 20, accounts receivable is 40, inventory is 40, and total current assets
equal 100. Since the current ratio is 2.0, total current liabilities must be 50. The acid test
(or quick) ratio is Cash + Marketable Securities + Net Accounts Receivable divided by
Current Liabilities. Thus the acid test ratio is (20 + 0 + 40) + 50, or 1.2.
G. This is the current ratio. The acid test (or quick) ratio is Cash + Marketable Securities +
Net Accounts Receivable divided by Current Liabilities.
H. This is Accounts Receivable divided by Current Liabilities. The acid test (or quick) ratio
is Cash + Marketable Securities + Net Accounts Receivable divided by Current
Liabilities.

Question #85: Of the following, the working capital financing policy that would subject a firm to
the greatest level of risk is the one where the firm finances
Correct Answer: A. permanent current assets with short-term debt.
E. permanent current assets with short-term debt.
F. fluctuating current assets with short-term debt.
G. fluctuating current assets with long-term debt.
H. permanent current assets with long-term debt.
Explanation:
E. This question is about the maturity matching approach to financing. The maturity
matching approach to financing current assets (also called the hedging or the self-
liquidating approach) matches assets to be financed with financing having the same
maturity. Even though the assets being financed with short-term debt are current assets, if
they are permanent, they will not be liquidated the way other current assets would be.
Examples of permanent current assets are accounts receivable and inventory in a non-
seasonal business or in a growing business. Inventory is sold, but it is immediately
replaced with new inventory, so the level of investment in inventory remains the same.
Accounts receivable get collected, but they are immediately replaced by accounts
receivable for other sales, so the level of accounts receivable also remains the same. And
in a firm that is growing, accounts receivable and inventory don't just remain the same —
they increase. And they continue increasing until the firm's growth stops. In either a non-
seasonal business or in a growing business. the level of the accounts receivable and
inventory required to support sales will be permanent and they should be financed with
long-term debt (If the business is seasonal. at least a portion of their accounts receivable
and inventory will be liquidated after the selling season. so only a portion of their
accounts receivable and inventory will be permanent.) If a company finances permanent,
long-term assets with short-term debt, it may be able to lower its interest costs, if short-
term interest rates are lower than long-term fixed interest rates (which is usually the
case). However. it runs two risks: (1) of not being able to renew the short-term financing
when it matures and being required to pay it off when funds are not available because the
current assets have not been liquidated, potentially putting the firm into bankruptcy: and
(2) of being forced to renew the short-term debt at a higher interest rate in a period of
rising interest rates. If short-term interest rates increase enough, the firm may find itself
paying a higher rate of interest than it would have paid if it had originally financed the
permanent assets using long-term, fixed rate financing. Because of the risks involved,
using short-term financing for long-term, permanent assets is considered an aggressive,
risky approach to financing. The greater the proportion of permanent assets financed with
short-term debt, the more aggressive the financing is and the greater the level of risk the
firm faces.
F. This question is about the maturity matching approach to financing. The maturity
matching approach to financing current assets (also called the hedging or the self-
liquidating approach) matches assets to be financed with financing having the same
maturity. It is appropriate to finance fluctuating current assets with short-term debt. A
financing policy like this enables a firm to borrow only what it needs for only as long as
it needs it. This minimizes its interest costs because it does not have borrowed funds on
hand that it is not using.
G. This question is about the maturity matching approach to financing. The maturity
matching approach to financing current assets (also called the hedging or the self-
liquidating approach) matches assets to be financed with financing having the same
maturity. It is not appropriate to finance fluctuating current assets with long-term debt
because the firm could find at times that not all of its long-term borrowings are required
to finance the level of current assets. The result will be that the firm will be paying
interest on loan balances that it does not need. If the firm chooses to pay down the long-
term loan principal during a period when it does not need all of the borrowings, the next
time the current assets increase and the firm needs the financing for them, the financing
may not be there. However, this question asks for the working capital financing policy
that would subject a firm to the greatest level of risk. This policy is not the riskiest of the
answer choices and thus it is not the best answer choice.
H. It is appropriate to finance permanent current assets with long-term debt. This question is
about the maturity matching approach to financing. The maturity matching approach to
financing current assets (also called the hedging or the self-liquidating approach) matches
assets to be financed with financing having the same maturity. Examples of permanent
current assets are accounts receivable and inventory in a non-seasonal business or in a
growing business. Inventory is sold, but it is immediately replaced with new inventory, so
the level of investment in inventory remains the same. Accounts receivable get collected,
but they are immediately replaced by accounts receivable for other sales, so the level of
accounts receivable also remains the same. And in a firm that is growing, accounts
receivable and inventory don't just remain the same — they increase. And they continue
increasing until the firm's growth stops. In either a non-seasonal business or in a growing
business, the level of the accounts receivable and inventory required to support sales will
be permanent. (If the business is seasonal, at least a portion of their accounts receivable
and inventory will be liquidated after the selling season, so only a portion of their
accounts receivable and inventory will be permanent.)

Question #86: In foreign currency markets. the phrase "managed float" refers to the
Correct Answer: • A Discretionary buying and selling of currencies by central banks.
E. Discretionary buying and selling of currencies by central banks.
F. Tendency for most currencies to depreciate in value.
G. Necessity of maintaining a highly liquid asset, such as gold, to conduct international
trade.
H. Fact that actual exchange rates are set by private business people in trading nations.
Explanation:
E. Under the managed float system, the exchange rates are determined by the market, but
the government also sells and buys currency through its central bank in order to influence
the exchange rate.
F. Under the managed float system, the exchange rates are determined by the market, but
the government also sells and buys currency in order to influence the exchange rate. This
does not mean that most currencies will depreciate.
G. Under the managed float system, the exchange rates are determined by the market, but
the government also sells and buys currency in order to influence the exchange rate. This
does not require a nation to maintain highly liquid assets.
H. Under the managed float system, the exchange rates are determined by the market, but
the government also sells and buys currency in order to influence the exchange rate.

Question #87: In a make versus buy decision. the relevant costs include variable manufacturing
costs as well as
Correct Answer: • A avoidable fixed costs.
E. avoidable fixed costs.
F. general office costs.
G. factory management costs.
H. depreciation costs.
Explanation:
E. In the decision to make-versus-buy decision you want to look only at relevant costs that
differ among the options. Relevant costs include both avoidable fixed and variable costs.
F. General office costs will generally not differ among the options, therefore they are not
relevant.
G. Factory management costs will generally not differ among the options, therefore they are
not relevant.
H. Depreciation costs will generally not differ among the options, therefore they are not
relevant.

Question #88: In a decision analysis situation, which one of the following costs is generally not
relevant to the decision?
Correct Answer: • B. Historical cost.
E. Differential cost.
F. Historical cost
G. Incremental cost.
H. Avoidable cost.
Explanation:
E. Differential costs are costs that differ between two alternatives. These costs are relevant
to the decision making process.
F. Historical costs are sunk costs that are not relevant to the decision making process. They
are costs that have already been incurred, and nothing can be done to change them no
matter what decision is made. In the decision making process you want to look only at
costs that differ among the related options.
G. Incremental costs are additional costs that result when production, or some other factor is
increased. These cost are relevant to the decision making process.
H. Avoidable costs are relevant to decision making process because they are costs that can
be avoided depending upon the decision that is made. In the decision making process we
need to look at all avoidable (variable and fixed) costs that will vary among the related
options.

Question #89: A firm with an 18% cost of capital is considering the following projects (on
January 1, year1):
January 1, Year 1 December 31, Year 5

Cash Outflow Cash inflow Project Internal


(00's Omitted) (00's Omitted) Rate of return
Project A $3,500 $7,400 16%
Project B 4,000 9,950 ?

Correct answer: D. 20%


E. 15%
F. 18%
G. 16%
H. 20%
Explanation:
E. The internal rate of return is the discount rate at which the present value of the expected
cash inflows from a project equals the present value of the expected cash outflows, or the
discount rate at which the net present value is zero. A positive NPV would result from
using a discount rate of 15%, so that cannot be the project's IRR.
F. 18% is the company's cost of capital, which is given in the question.
G. 16% is approximately the internal rate of return for project A. but the question asks for
the internal rate of return for project B.
H. The internal rate of return is the discount rate at which the present value of the expected
cash inflows from a project equals the present value of the expected cash outflows, or the
discount rate at which the net present value is zero. To determine the internal rate of
return from the information given, we need to first know what discount factor for five
years would result in a present value of $9,950 that is equal to $4,000. To arrive at that
factor, we divide $4,000 by $9,950, and we get 0.402. We then look along the line of
factors for five years on the factor table given to locate a factor close to 0.402. That is
0.4019, which is in the 20% column. Thus, the internal rate of return is closest to 20%.

Question #90: Valley Inc. uses 400 lbs. of a rare isotope per year. The isotope costs $500 per lb.,
but the supplier is offering a quantity discount of 2% for order sizes between 30 and 79 lbs., and
a 6% discount for order sizes of 80 lbs. or more. The ordering costs are $200. Carrying costs are
$100 per year per lb. of material and are not affected by the discounts. If the purchasing manager
places eight orders of 50 lbs. each, the total cost of ordering and carrying inventory, including
discounts lost, will be
Correct Answer: • C. $12,100.
E. $1,600.
F. $4,100.
G. $12,100.
H. $6,600.
Explanation:
E. This is the ordering cost only. The total cost of ordering and carrying the inventory,
including discounts lost, will include the lost discount (from ordering at the 2% discount
level instead of at the 6% discount level), ordering costs and carrying costs.
F. This is the inventory ordering and carrying costs, but it does not include discounts lost.
The total cost of ordering and carrying the inventory, including discounts lost, will
include the lost discount (from ordering at the 2% discount level instead of at the 6%
discount level), ordering costs and carrying costs.
G. The total cost will include the lost discount (from ordering at the 2% discount level
instead of at the 6% discount level), ordering costs and carrying costs. Lost discount cost:
The company purchases 400 pounds at $500 per pound per year, at a total cost of
$200,000 (400 x $500). The cost of the lost discounts is the difference between 6% and
2% of $200,000, i.e., 4% multiplied by $200,000. $200,000 x 0.04 = $8,000. Ordering
cost: $200 per order x 8 orders per year = $1,600. Carrying cost: Each order consists of
50 pounds. It is assumed Valley Inc. will use those 50 pounds down to zero and then
receive another order of 50. We must also assume they will use the amount in each order
at a steady rate. Therefore, the average number of pounds in inventory throughout the
year will be the average of 50 and 0, which is (50 + 0) / 2, or 25 pounds. The carrying
cost of $100 per pound is multiplied by these 25 pounds, and the cost is $2,500. The total
cost will be $8,000+ $1,600 + $2,500 = $12,100.
H. This is the inventory ordering costs plus carrying costs, but the carrying costs are
calculated incorrectly. They are calculated using the number of units ordered each time
an order is placed. Furthermore, this answer does not include discounts lost. The
inventory carrying costs should be calculated using the average number of units in
inventory throughout the year, not the amount ordered each time an order is placed.

Question #91: If an entity's books of account are not maintained in its functional currency, the
FASB Accounting Standards Codification® requires remeasurement into the functional currency
prior to the translation process. An item that should be remeasured by use of the current
exchange rate is
Correct Answer: • an investment in bonds to be held until maturity
E. a plant asset and the associated accumulated depreciation.
F. inventories
G. the revenue from a long-term construction contract.
H. an investment in bonds to be held until maturity
Explanation:
E. Plant assets are nonmonetary assets and are therefore remeasured at the historical rates.
F. Inventories are nonmonetary items. Nonmonetary items are remeasured at the historical
exchange rate in effect when each transaction occurred.
G. Revenue items are usually remeasured at the historical rate that was in effect on the date
of the transaction.
H. An investment in bonds is a monetary asset and it is remeasured at the current exchange
rate. Most monetary assets are remeasured using the current exchange rate.
Question #92: Gleason Co. has two products, a frozen dessert and ready-to-bake breakfast rolls,
ready for introduction. However, plant capacity is limited, and only one product can be
introduced at present Therefore. Gleason has conducted a market study, at a cost of $26,000, to
determine which product will be more profitable. The results of the study follow.
Sales of Desserts at $1.80/unit Sales of Rolls at $1.20/unit

Volume Probability Volume Probability


250,000 0.30 200,000 0.20
300,000 0.40 250,000 0.50
350,000 0.20 300,000 0.20
400,000 0.10 350,000 0.10

The costs associated with the two products have been estimated by Gleason's cost accounting
department and are shown as follows.

Dessert Rolls
Ingredients per unit $0.40 $0.25
Direct labor per unit 0.35 0.30
Variable overhead per
unit 0.40 0.20
Production tooling 48,000 25,000
Adverting 30,000 20,000

'Gleason treats production tooling as a current operating expense rather than capitalizing it as a
fixed asset. According to Gleason's market study, the expected value of the sales volume of the
breakfast rolls is
Correct Answer: B. 260,000 units.
A. Some amount other than those given.
B. 260,000 units.
C. 275.000 units.
D. 125,000 units.
Explanation:
A. The correct answer is one of the answer choices given.
B. Expected sales volume is a weighted average of the possible sales volume, weighted
according to their probabilities. To find the expected sales volume, multiply each estimated sales
volume by its probability of occurring and sum the results. (200.000 x 0.2) + (250.000 x 0.5) +
(300.000 x 0.2) + (350.000 x 0.1) = 260.000. Expected sales volume is 260.000 units.
C. This calculation represents the average of the possible sales volumes of the rolls. The
expected value is a weighted average with the various probabilities of each sales volume as the
weights.
D. The answer of 125.000 was calculated by using the estimated sales volume of 250.000 times
its probability of 0.5. This is not the expected value of the sales volume of rolls.

Question #93: Frasier Products has been growing at a rate of 10% per year and expects this
growth to continue and produce earnings per share of $4.00 next year. The firm has a dividend
payout ratio of 35% and a beta value of 1.25. If the risk-free rate is 7% and the return on the
market is 15%, what is the expected current market value of Frasier's common stock?
Correct Answer: A. $20.00.
E. $20.00.
F. $28.00.
G. $16.00.
H. $14.00.
Explanation:
E. This requires the use of both the Capital Asset Pricing Model and the Dividend Growth
Model. First, we use the CAPM to find the investors' required rate of return:
r = r F + I3(r M - r F )
Where:
r F = the risk-free rate
r M = the market rate of return
β= the stocks beta coefficient
Plugging the numbers into the formula, we have:
r = 0.07 + 1.25(0.15 - 0.07) = 0.17
Next, we take the investors' required rate of return and use it as the r in the Dividend
Growth Model. stated to solve for P 0 . to find the expected current market value of a
share of stock:
d1 P 0 = r- g
Where:
P 0 = the fair value today of a share of stock
d 1= the next annual dividend to be paid
r = the investors' required rate of return
g = the expected growth rate of the dividend
Earnings per share next year are expected to be $4, and the dividend payout ratio is 0.35.
Therefore, the next dividend will be $4 x 0.35, or $1.40.
Plugging these numbers into the Dividend Growth Model, we get
P 0 = $1.40 / (0.17 - 0.10) = $20
F. This answer results from using the return on the market minus the growth rate in the
denominator of the Dividend Growth Model. However, the denominator of the Dividend
Growth Model is r - g, where r = the investors' required rate of return and g = the growth
rate.
G. This answer could result from using 10% minus the beta coefficient in the denominator of
the Dividend Growth Model. However, the denominator of the Dividend Growth Model
is r - g, where r = the investors' required rate of return and g = the growth rate.
H. This answer results from using just the growth rate in the denominator of the Dividend
Growth Model. However, the denominator of the Dividend Growth Model is r - g, where
r = the investors' required rate of return and g = the growth rate.

Question #94: When compared to a debt-to-assets ratio, a debt to equity ratio would
Correct Answer: D. Be higher than the debt to assets ratio.
E. Be about the same as the debt to assets ratio.
F. Have no relationship at all to the debt to assets ratio.
G. Be lower than the debt to assets ratio.
H. Be higher than the debt to assets ratio.
Explanation:
E. These two ratios could not be the same because the assets and equity of a company will
be different from each other (except in the unusual situation in which the company has no
debt).
F. There is a relationship between the magnitude of a company's debt-to-equity ratio and the
magnitude of the same company' debt-to-assets ratio. See the correct answer for a
complete explanation.
G. Debt to equity is total liabilities divided by total equity. Debt to assets is total liabilities
divided by total assets. Because the asset balance of a company is higher than its equity. a
debt-to-equity ratio will be higher than a debt-to-asset ratio because when equity is used
in the denominator, the denominator will be smaller than when assets are used in the
denominator.
H. Debt to equity is total liabilities divided by total equity. Debt to assets is total liabilities
divided by total assets. Because the asset balance of a company is higher than its equity. a
debt-to-equity ratio will be higher than a debt-to-asset ratio because when equity is used
in the denominator, the denominator will be smaller than when assets are used in the
denominator. We know that the assets of the company must be higher than the equity of
the company because of the accounting equation of assets = liabilities + equity.

Question #95: Jorelle Company's financial staff has been requested to review a proposed
investment in new capital equipment. Applicable financial data is presented below. There will be
no salvage value at the end of the investment's life and, due to realistic depreciation practices, it
is estimated that the salvage value and net book value are equal at the end of each year. All cash
flows are assumed to take place at the end of each year. For investment proposals. Jorelle uses a
12% after-tax target rate of return.
Investment proposal
Annual
Cost and book Annual net after-tax Cash net
Year Value flow income
0 $250,000 $0 $0
1 168,000 120,000 35,000
2 100,000 108,000 39,000
3 50,000 96,000 43,000
4 18,000 84,000 47,000
5 0 72,000 51,000

Discount factors for a 12% Rate of return:


Present value of $1 present value of annuity of $1 received at end of each
Year Received at end of period period
1 0.89 .89
2 0.80 1.69
3 0.71 2.40
4 0.64 3.04
5 0.57 3.61
6 0.51 4.12

The traditional payback period for the investment proposal is

Correct Answer: • A. 2.23 years.


E. 2.23 years.
F. 1.65 years.
G. 2.83 years.
H. Over 5 years.
Explanation:
E. When cash flows are not constant over the life of the project, we must find the
cumulative cash inflows for each year to determine when the inflows will equal the
outflows.
The cash flows are as follows: Year 0 Year 1 Year 2 Year 3 Year 4 Year 5
Net initial investment (250,000)
After-tax cash flows from operations 120,000 108,000 96,000 84,000 72,000
Cumulative cash flows (250,000) (130,000) (22,000) 74,000 158,000 230,000
The cumulative cash flow from the project becomes positive sometime during year 3. If the cash
flows are assumed to occur evenly throughout the year, the exact payback period is 2.23 years.
calculated as follows:
Number of the project year in the final year when cash flow is negative: 2
Plus: a fraction consisting of
Numerator = the positive value of the negative cumulative inflow amount from the final negative
year: 22,000
Denominator = cash flow for the following year: 96,000
OR: 2 + (22.000/96,000) = 2.23
The initial investment will be recouped after 2.23 years.
F. An answer of 1.65 years results from adding together the annual after-tax cash flow and
the annual net income for each year and using the total to determine the payback period.
However, the payback period should be determined using annual after-tax cash flow only.
G. An answer of 2.83 years results from using the discounted annual after-tax cash flows to
determine the payback period. However, the traditional payback period uses
undiscounted cash flows to calculate the payback period.
H. An answer of "over 5 years" could result from using the annual net income amounts
instead of the annual after-tax cash flows to calculate the payback period.

Question #96: The president of Reading Manufacturing. Inc. is establishing performance goals
for each of the company's manufacturing plants. The data below represent prior year results for
one of the plants. manufacturing plants. The data below represent prior year results for one of the
plants.
Revenue $ 400,000 Variable costs 100.000 Fixed costs 200,000 Average assets 1.000.000
Average liabilities 200,000
The plant's return on assets is
Correct Answer: C.10.0%
E. 12.5%
F. 37.5%
G. 10.0%
H. 30.0%
Explanation:
E. Return on assets is net income divided by average total assets. This answer uses average
net assets (assets minus liabilities. which is stockholders' equity).
F. Return on assets is net income divided by average total assets. This answer does not
include the fixed costs in the calculation of net income. Also, this answer uses average
net assets (assets minus liabilities. which is stockholders' equity) instead of total assets.
G. Return on assets is net income divided by average total assets. Net income is 5400.000
revenues - $100.000 variable costs -5200.000 fixed costs = S100.000. $100.000 divided
by average assets of $1.000.000 = 0.10 or 10%.
H. Return on assets is net income divided by average total assets. This answer does not
include the fixed costs in the calculation of net income.

Question #97: Which statement below best indicates the order of the components that should be
implemented in a new Enterprise Risk Management program?
Correct Answer: • C. Set risk management strategy and objectives, identify risks, and assess
risks.
E. Communicate and monitor risks, identify risks, and treat risks.
F. Assess risks, control risks, and treat risks.
G. Set risk management strategy and objectives, identify risks, and assess risks.
H. Control risks, set risk management strategy and objectives, and monitor risks.
Explanation:
E. Information and communication involves communicating all the relevant information that
needs to be communicated to the appropriate people, within a time frame that will allow
them to carry out their duties. Information and communication cannot be implemented
for an ERM until the company has identified the risks, assessed the risks, and decided
how to respond to each identified risk and the system has been put into place.
F. Control activities are developed after the company has determined how it will respond to
each risk that has been identified and assessed.
G. The components of an Enterprise Risk Management program, in order of their
implementation are: -Objective setting. The organization's strategic objectives and goals
for its operations, reporting and compliance activities must be determined and
established. -Event identification. Events are the internal and external events that affect
the organization's implementation of its ERM strategy or the achievement of its
objectives. -Risk assessment. Risk assessment is the process of analyzing and considering
risks from three perspectives: (1) the likelihood of the risk's occurring. (2) the potential
impact of the event if it does occur, and (3) the interrelationship of the risks on a unit-by-
unit or total organization basis. -Risk response. Risk response is what the company
decides to do with respect to each of the risks identified. Management must develop a
response for each of its identified risks. -Control activities. Control activities are the
policies and procedures implemented to ensure the risk responses are carried out -
Information and communication. This includes all the relevant information that needs to
be communicated to the appropriate people, within a time frame that will allow them to
carry out their duties. -Monitoring. The system put in place needs to be monitored to
ensure it continues to be appropriate and continues to be properly operated.
H. The organization's strategic objectives and goals for its operations, reporting and
compliance activities must be determined and established before control activities or
monitoring can take place.

Question #98: Colonie Inc expects to report net income of at least $10 million annually for the
foreseeable future. Colonie could increase its return on equity by taking which of the following
actions with respect to its inventory turnover and the use of equity financing?
Correct Answer: • A Increase inventory turnover: decrease use of equity financing.
E. Increase inventory turnover: decrease use of equity financing.
F. Increase inventory turnover: increase use of equity financing.
G. Decrease inventory turnover: decrease use of equity financing.
H. Decrease inventory turnover: increase use of equity financing.
Explanation:
E. Return on equity is net income divided by average total equity. Increasing inventory
turnover will decrease inventory, thereby decreasing inventory holding costs and
increasing net income. Decreasing use of equity financing, for example by repurchasing
treasury stock, will decrease total equity. The combination of increased net income along
with decreased equity will lead to increased return on equity.
F. Return on equity is net income divided by average total equity. Increasing inventory
turnover will decrease inventory, thereby decreasing inventory holding costs and
increasing net income. This would increase return on equity if equity were unchanged.
However, increasing use of equity financing, for example by issuing new common stock,
will increase equity. Whether return on equity would be increased or decreased by this
combination would depend on the magnitude of the two changes, so it is not possible to
state that return on equity would be either increased or decreased based on the
information given here.
G. Return on equity is net income divided by average total equity. Decreasing inventory
turnover will increase inventory, increasing the inventory holding costs and decreasing
net income. This would decrease return on equity if equity were unchanged. However,
decreasing use of equity financing, for example by repurchasing treasury stock, will
decrease equity. Whether return on equity would be increased or decreased by this
combination would depend on the magnitude of the two changes, so it is not possible to
state that return on equity would be either increased or decreased based on the
information given here.
H. Return on equity is net income divided by average total equity. Decreasing inventory
turnover will increase inventory, thereby increasing inventory holding costs and
decreasing net income. Increasing use of equity financing, for example by issuing
additional common stock, will increase total equity. The combination of decreased net
income along with increased equity will lead to decreased return on equity.

Question #99: Regis Company, which is subject to an effective income tax rate of 30%, is
evaluating a proposed capital project. Relevant information for the proposed project is
summarized below.
Initial investment $500,000
Annual operating cash inflows
for the first three years:
Year 1 $185,000
Year 2 175,000
Year 3 152,000
Depreciation will be calculated under the straight-line method using an 8-year estimated service
life and a terminal value of 850,000. In determining the estimated total after-tax cash flow in
Year 2 of the project. Regis should consider the after-tax operating cash
Correct Answer: B. inflow plus annual depreciation tax shield.
E. inflow only.
F. inflow plus annual depreciation tax shield.
G. inflow plus annual depreciation expense.
H. inflow plus the net impact of the annual depreciation expense and depreciation tax shield.
Explanation:
E. The total after-tax cash flow for Year 2 of the project will include the net after-tax
operating cash flow, but that is not the only thing it will include.
F. The total after-tax cash flow for Year 2 of the project will consist of the net after-tax
operating cash flow plus the depreciation tax shield for the year. Depreciation expense is
a non-cash transaction that is not included in the after-tax operating cash flow, and it
should not be added back in. Depreciation expense is used only to calculate the amount of
the depreciation tax shield, which is a cash flow item.
G. Depreciation expense is a non-cash transaction that is not included in the after-tax
operating cash flow, and it should not be added back in. Depreciation expense is used
only to calculate the amount of the depreciation tax shield, which is a cash flow item.
H. Depreciation expense is a non-cash transaction that is not included in the after-tax
operating cash flow, and it should not be added back in. Depreciation expense is used
only to calculate the amount of the depreciation tax shield. which is a cash flow item.
Question #100: A bill of lading is a document that
Correct Answer: • D. Is used to transfer responsibility for goods between the seller of goods and
a common carrier.
E. Is sent with the goods giving a listing of the quantities of items included in the shipment
F. Summarizes data relating to a disbursement and represents final authorization for
payment.
G. Reduces a customer's account for goods returned to the seller.
H. Is used to transfer responsibility for goods between the seller of goods and a common
carrier.
Explanation:
E. The description given is that of a packing slip, not a bill of lading.
F. The description given is related to a payment order, not a bill of lading.
G. The description is given is that of a credit memo, not a bill of lading.
H. A bill of lading is a document that transfers possession of goods from the seller to a
common carrier.

MOCK TEST 3
Question #1: A company has a December 31 year end. Which one of the following options to
increase net income during the last month of the company's fiscal year end would least likely
result in a violation of the IMAs Statement of Ethical Professional Practice?
Correct Answer: • Postpone planned marketing expenditures until January.
A. Persuade suppliers to postpone billing until January 1.
B. Delay the year-end closing until January 4 to capture sales over the New Year's holiday
in the current year.
C. Reduce the calculated allowance for bad debts and bad debt expenses.
D. Postpone planned marketing expenditures until January.
Explanation:
Answer (A) is incorrect.
Persuading suppliers to postpone billing is only necessary when the goods have been received
and the suppliers will soon send the bill. This action is a likely violation of the IMAs Statement
of Ethical Professional Practice because, under accrual accounting, when the goods are received,
the title of the goods has shifted ownership to the company, and accounts payable should be
recorded to recognize the liability.
Answer (B) is incorrect.
When a company has a set year-end date, postponing closing by 4 days violates the IMAs
Statement of Ethical Professional Practice because net income will not faithfully represent the
actual closing date. Since the financial statements are dated December 31 as the closing date,
financial statement users will assume that the sales from the beginning of January are not
included.
Answer (C) is incorrect.
Reducing the calculated allowance for bad debts and bad debt expense without justifiable reason
is a violation of the I MA's Statement of Ethical Professional Practice. Justification may include
data collected that indicates higher collections on account.
Answer (D) is correct.
Postponing planned marketing expenditures until a future date is least likely a violation of the
IMA's Statement of Ethical Professional Practice because as long as the marketing funds have
not been expended, the expense need not be recorded until the following period.

Question #2: A corporation manufactures two products that are considered joint products.
Common costs of $350,000, allocated using the physical measures method, yield 15,000 units of
Product A and 20,000 units of Product B. Product B incurs $50,000 of direct costs and sells for
$15 per unit. The corporation has the option of processing Product B further. This action would
increase the product's direct costs by $35,000 and would increase the unit selling price to $17. If
the corporation further processes Product B. its income will
Correct Answer: • Increase by $5.000.
A. Decrease by $45.000.
B. Increase by $5.000.
C. Increase by $55.000.
D. Increase by $105.000.
Explanation:
Answer (A) is incorrect.
The $50,000 of Product B's direct costs that are already included before the additional processing
are ignored for this calculation.
Answer (B) is correct.
The further processing will increase revenue by $2 per unit ($17 - $15). Therefore, total
increased revenues are $40,000 ($2 per unit x 20,000 units). Since this exceeds the $35,000 costs
of further processing by $5,000, the corporation's income will increase by $5,000.
Answer (C) is incorrect.
This is the total income of Product B if the further processing is added.
Answer (D) is incorrect.
This is the total income of Product B if the further processing is added and the original $50,000
of direct costs are excluded.

Question #3: If gross profit margin has remained fairly constant for the past several years, which
one of the following is the best explanation?
Correct Answer: The cost of goods sold and sales have decreased by the same percentage.
A. The cost of goods sold and sales have decreased by the same percentage.
B. The cost of goods sold and sales have decreased by the same dollar amount.
C. Net sales and net income have remained constant
D. The cost of goods has remained steady.
Explanation:
Answer (A) is correct.
Gross profit margin is the percentage of gross revenues that remains with the firm after paying
for merchandise. The key analysis with respect to the gross profit margin is whether it is keeping
up with the increase or decrease in sales. For example, a 10% increase in sales should be
accompanied by at least a 10% increase in the gross profit margin.
Answer (B) is incorrect.
The cost of goods sold and sales must have decreased by the same percentage, not dollar amount.
Answer (C) is incorrect.
The cost of goods sold must have also remained steady.
Answer (D) is incorrect.
Net sales must have also remained steady.

Question #4: Fact Pattern: Calvin, Inc., is considering the purchase of a new state-of-the-art
machine to replace its hand-operated machine. Calvin's effective tax rate is 40%, and its cost of
capital is 12%. Data regarding the existing and new machines are presented below.
Question: The existing machine has been in service for 7 years and could be sold currently for
$25.000. Calvin expects to realize a before-tax annual reduction in labor costs of $30,000 if the
new machine is purchased and placed in service.
If the new machine is purchased, the incremental cash flows for the fifth year would amount to
Existing
machine New machine
Original cost $50,000 $90,000
installation costs 0 4,000
Freight and insurance 0 6,000
Expected end salvage value 0 0
depreciation method Straight line straight line
Expected useful life 10 years 5 years

Correct answer: C. 26,000


A. $18,000
B. $24,000
C. $26,000
D. $30,000
Explanation:
Answer (A) is incorrect.
The amount of $18,000 results from failing to include the depreciation tax shield.
Answer (B) is incorrect.
The incremental cash flow during the first 3 years of the project, when the lost depreciation on
the old equipment is still relevant, is $24,000.
Answer (C) is correct.
The estimated incremental after-tax operating cash flows for each year of a capital project consist
of two components, the after-tax cash inflows from operations and the difference in depredation
tax shields between the old and new equipment. The first of these for Calvin can be calculated as
follows:
Net annual labor cost savings $30,000
Less: income tax expense ($30,000 x 40%) (12,000)
After-tax cash inflow from operations $ 18,000
The depredation tax shield on the new equipment is derived as follows:

Cost of new equipment


($90,000 + $4,000 + $6,000) $100,000
Divided by estimated useful life ÷5
Annual depreciation expense $ 20,000
Times: tax rate x 40%
Annual depreciation tax shield $ 8,000
The depredation tax shield in the fifth year consists only of the total shield generated by the new
equipment. Since the old equipment only has 3 years of service life remaining, it will have no
incremental effect on Years 4 and 5. Calvin's total incremental cash flow for the fifth year of this
project is therefore estimated at $26,000 ($18,000 + $8,000).
Answer (D) is incorrect.
The amount of $30,000 is merely the before-tax savings in labor cost.

Question #5: Which one of the following statements best characterizes the provisions of the
Foreign Corrupt Practices Act (FCPA)?
Correct Answer: A. The FCPA requires corporations to keep records and accounts in sufficient
detail to reflect transactions.
A. The FCPA requires corporations to keep records and accounts in sufficient detail to
reflect transactions.
B. The FCPA requires compliance with corporate codes of conduct to be reviewed and
reported on by external auditors.
C. The FCPA provides for treble damages in civil cases brought under the law.
D. The FCPA provides for criminal penalties for auditors who fail to report a corporation's
participation in bribery.
Explanation:
Answer (A) is correct.
The FCPA was enacted for the purpose of making it unlawful to make payments to foreign
government officials to assist in obtaining or retaining business. Corporations are thus required
to keep records to reflect all transactions so that they will not be covering up this transaction.
Answer (B) is incorrect.
This is not a requirement of FCPA.
Answer (C) is incorrect.
Bribery to foreign officials is considered a criminal case.
Answer (D) is incorrect.
Individuals and the corporation may be assessed fines (and imprisonment for individuals) if they
are in violation of the FCPA. This does not apply to auditors.

Question #6: A management accountant was recently tried and convicted in court for producing
and selling illegal narcotic drugs. None of the activity occurred during work hours, and he
performed duties as a management accountant without incident during the period of illegal
activity. The management accountant has argued that because the illegal activity was unrelated to
his service as a management accountant, no ethical violation had been committed. Which
provision of the I MAs Statement of Ethical Professional Practice is most likely to apply to the
management accountant's illegal actions?
Correct Answer: C. Integrity.
A. Competence.
B. Confidentiality.
C. Integrity.
D. Credibility.
Explanation:
Answer (A) is incorrect.
The competence standard would not be the most likely to apply to the management accountant's
illegal actions. Some of the responsibilities under the competence standard state that each
member has the responsibility to maintain an appropriate level of professional expertise by
continually developing knowledge and skills and to provide decision support information and
recommendations that are accurate, clear, concise, and timely. Even though the management
accountant was producing and selling illegal narcotics, he was still competent.
Answer (B) is incorrect.
The confidentiality standard would not be the most likely to apply to the management
accountant's illegal actions. Some of the responsibilities under the confidentiality standard state
that each member has the responsibility to keep information confidential except when disclosure
is authorized or legally required and to inform all relevant parties regarding appropriate use of
confidential information. The management accountant did not break any confidentiality rules by
producing and selling illegal drugs.
Answer (C) is correct.
The integrity standard states that each member has a responsibility to abstain from engaging in or
supporting any activity that might discredit the profession. Being convicted of producing and
selling illegal narcotic drugs is an activity that would discredit the profession.
Answer (D) is incorrect.
The credibility standard would not be the most likely to apply to the management accountant's
illegal actions. Some of the responsibilities under the credibility standard state that each member
has the responsibility to communicate information fairly and objectively and to disclose all
relevant information that could reasonably be expected to influence an intended user's
understanding. The management accountant's illegal activities did not apply to the credibility
standard.

Question #7: What is the most likely reason for management to overstate expenses?
Correct Answer: A. To minimize tax liability.
A. To minimize tax liability.
B. To earn a bonus.
C. To maximize net income.
D. To maximize cash on hand.
Explanation:
Answer (A) is correct.
The most likely reason for management to understate income by overstating expenses is to
minimize tax liability.
Answer (B) is incorrect.
Management may understate expenses to earn a bonus.
Answer (C) is incorrect.
Management may understate expenses to maximize net income.
Answer (D) is incorrect.
Understating expenses has no direct effect on cash.

Question #8: (please refer the fact pattern below)


Question: White's balance in accounts receivable at November 30, Year 2, is
[Fact Pattern #28] Selected data from White Corporation's financial statements for the year
ended November 30, Year 2, are as follows (all sales are on credit).

Current ratio 1.4


Quick ratio 0.86
Current liabilities $450,000
Accounts receivable turnover 3.65
Merchandise inventory turnover 3.30
Rate of return on assets 6.5%
Selected account balances at November30, Year 1
Accounts receivable $355,000
Merchandise inventory 237,000
Year 2 operations
Sales $1,241,000
Cost of goods sold 792,000
Correct Answer: A. $325,000
A. $325,000
B. $216,986
C. $78,973
D. $355,000

Explanation:
Answer (A) is correct.
The accounts receivable turnover (3.65) equals sales divided by the average balance in accounts
receivable. Thus, dividing the $1,241,000 of sales by 3.65 gives an average receivables balance
of $340,000. The average receivables balance is the average of the beginning and ending
balances. Since the beginning balance was $355,000, the ending balance must be $325,000
[($340,000 x 2) -$355,000].
Answer (B) is incorrect.
The accounts receivable balance is not $216,986.
Answer (C) is incorrect.
The accounts receivable balance is not $78,973.
Answer (D) is incorrect.
The beginning balance in accounts receivable was $355,000.

Question #9: Confidential negotiations between Company A and Company B were completed
this morning. It was decided that in 1 week, it will be publicly announced that Company A will
acquire Company B for a cash offer of a 30% premium over Company B's current market price.
If the stock price of Company B does not react at all today but rises by 30% with the public
announcement next week, this implies that the market is
Correct Answer: C. Semi-strong form efficient but is not strong form efficient.
A. Weak form efficient but is not semi-strong form efficient
B. Strong form efficient but is not semi-strong form efficient.
C. Semi-strong form efficient but is not strong form efficient.
D. Strong form efficient but is not weak form efficient.
Explanation:
Answer (A) is incorrect.
Weak form implies that only past information is reflected in the security price. Since the current
announcement of the 30% premium offer raises the security price by 30%, it is not weak form
efficient.
Answer (B) is incorrect.
Strong form implies that all public and private information is reflected in security prices. Since
the stock price did not change when the deal occurred privately, this is not strong form efficient.
Answer (C) is correct.
Semi-strong form implies that all publicly available information is reflected in security prices.
The 30% increase at the declaration of the merger shows that this is the case. Strong form
implies that all public and private information is reflected in security prices. Since the stock price
did not change when the deal occurred privately, this is not strong form efficient.
Answer (D) is incorrect.
Strong form implies that all public and private information is reflected in security prices. Since
the stock price did not change when the deal occurred privately, this is not strong form efficient.

Question #10: The cost of capital for foreign investment projects is higher because of all of the
following factors except
Correct Answer: D. Trigger pricing.
A. Exchange-rate risk.
B. Political risk arising from possible expropriation.
C. Laws requiring specific forms of financing.
D. Trigger pricing.
Explanation:
Answer (A) is incorrect.
They are all causes of higher costs of capital in foreign projects.
Answer (B) is incorrect.
They are all causes of higher costs of capital in foreign projects.
Answer (C) is incorrect.
They are all causes of higher costs of capital in foreign projects.
Answer (D) is correct.
The cost of capital is typically higher for foreign projects for a variety of reasons, including
exchange-rate risk, political risk, and limitations on sources of financing that often require a
certain percentage of domestic ownership. Trigger pricing is not a cause. Trigger pricing is a
means of managing exchange-rate risk by supplying foreign funds at an indexed price, but with
an option to convert to a futures-based fixed price when a specified basis differential exists
between the two prices.
Question #11: A firm must increase its acid test ratio above the current 0.9 level in order to
comply with the terms of a loan agreement. Which one of the following actions is most likely to
produce the desired results?
Correct Answer: B. Selling auto parts on account.
A. Expediting collection of accounts receivable.
B. Selling auto parts on account.
C. Making a payment to trade accounts payable.
D. Purchasing marketable securities for cash.
Explanation:
Answer (A) is incorrect.
Converting accounts receivable into cash has no net effect on the numerator of the acid test ratio.
Answer (B) is correct.
The acid test (quick) ratio consists of the quick assets (cash. marketable securities. and net
accounts receivable) divided by current liabilities. Exchanging merchandise inventory for
accounts receivable increases the numerator while having no effect on the denominator, resulting
in an increase in the overall ratio.
Answer (C) is incorrect.
Paying down accounts payable decreases both the numerator and denominator by the same dollar
amount, resulting in a decrease of the acid test ratio.
Answer (D) is incorrect.
Purchasing marketable securities for cash has no net effect on the numerator.

Question #12: An auto dealer employs 45 sales personnel to market its line of luxury
automobiles. The average car sells for $23,000, and a 6% commission is paid to the salesperson.
The auto dealer is considering a change to a commission arrangement that would pay each
salesperson a salary of $2,000 per month plus a commission of 2% of the sales made by that
salesperson. The amount of total monthly car sales at which the auto dealer would be indifferent
as to which plan to select is
Correct Answer: A. $2,250,000
A. $2,250,000
B. $3,000,000
C. $1,500,000
D. $1,250,000
Explanation:
Answer (A) is correct.
Given that X equals the cars sold, the indifference equation and its solution are as follows:
($23,000 x 6%) X = ($23,000 x 2%) X + (45 x $2,000)
$920X = $90,000
X = 97.8261
Cars At a price of $23,000 each, 97.8261 cars will sell for $2,250,000. Another approach is to
determine the sales per person at which $2,000 is equal to a 4% commission. This amount is
$50,000 ($2,000 +.04) per person, or $2,250,000 (45 x $50,000) for the entire sales force.
Answer (B) is incorrect.
At sales of $3,000,000, the auto dealer would prefer the salary plus commission plan.
Answer (C) is incorrect.
At sales of $1,500,000, the auto dealer would prefer the 6% commission plan.
Answer (D) is incorrect.
At sales of $1,250,000, the auto dealer would prefer the 6% commission plan.

Question #13: In discounted cash flow techniques, which one of the following alternatives best
reflects the items to be incorporated in the initial net cash investment?
Correct Answer: D. Yes (Capitalized expenditures (e.g. shipping costs)). (Changes in net
working capital) Yes. Yes (Net proceeds from sale of old asset in a replacement decision),
(Impact of spontaneous changes in current liabilities) Yes
A. No (Capitalized expenditures (e.g. shipping costs)). (Changes in net working capital )
Yes. Yes (Net proceeds from sale of old asset in a replacement decision). ( Impact of
spontaneous changes in current liabilities ) Yes
B. Yes (Capitalized expenditures (e.g. shipping costs)). (Changes in net working capital )
No. No (Net proceeds from sale of old asset in a replacement decision). ( Impact of
spontaneous changes in current liabilities )No
C. No (Capitalized expenditures (e.g. shipping costs)). (Changes in net working capital )
Yes. No (Net proceeds from sale of old asset in a replacement decision). ( Impact of
spontaneous changes in current liabilities ) No
D. Yes (Capitalized expenditures (e.g. shipping costs)). (Changes in net working capital )
Yes. Yes (Net proceeds from sale of old asset in a replacement decision). ( Impact of
spontaneous changes in current liabilities ) Yes
Explanation:
Answer (A) is incorrect.
Capitalized expenditures must be considered.
Answer (B) is incorrect.
Changes in working capital and proceeds from the sale of existing assets must be considered.
Answer (C) is incorrect.
Changes in working capital are not the only items that must be considered.
Answer (D) is correct.
All four of the items listed must be incorporated in a DCF analysis of a capital project.

Question #14: A manufacturer of printers is attempting to determine its cost of common equity
for cost of capital purposes. The manufacturer's long-term debt is rated AA by Standard &
Poor's. The manufacturer's common shares trade on the NASDAQ and the current market price is
$26.87. The most recent yearly common share dividend paid common shareholders was $1.04.
The consensus forecast of security analysts who follow the manufacturer's common shares is that
earnings growth will average 12.5% over the long term. The manufacturer's marginal income tax
rate is 40%. Using the dividend discount model, what is the manufacturer's cost of equity capital
for cost of capital purposes?
Correct Answer: C. 16.85%
A. 9.82%
B. 10.11%
C. 16.37%
D. 16.85%
Explanation:
Answer (A) is incorrect.
The figure of 9.82% subtracts the next dividend over the market price from the growth rate.
Furthermore, the market price should not be multiplied by 1 minus the tax rate.
Answer (B) is incorrect.
The figure of 10.11% fails to use to next dividend rather than the most recent dividend and
subtracts the dividend over the market price from the growth rate. Furthermore, the market price
should not be multiplied by 1 minus the tax rate.
Answer (C) is incorrect.
The next dividend. $1.17, must be used instead of $1.04.
Answer (D) is correct.
Under the dividend growth model, the cost of equity equals the expected growth rate plus the
quotient of the next dividend and the current market price. The next dividend is calculated as
$1.17 [$1.04 dividend x (1 + .125 growth)]. Thus, the cost of equity capital is 16.85% [12.5% +
51.17 ÷ $26.87)]. This model assumes that the payout ratio, retention rate, and the earnings per
share growth rate are all constant.

Question #15: The U.S. Foreign Corrupt Practices Act is particularly focused on the dealings of
financial institutions and the safeguarding of the global financial system. Financial institutions
must implement robust controls to ensure knowledge of their customers and the nature of their
business transactions and be in a position to prove to regulators a high level of due diligence.
These safeguards are required to minimize all of the following except
Correct Answer: B. Insider trading.
A. Money laundering.
B. Insider trading.
C. Terrorist financing.
D. Extortion and bribery.
Explanation:
Answer (A) is incorrect.
Money laundering is one focus of the safeguards of the global financial system relating to the
U.S. Foreign Corrupt Practices Act.
Answer (B) is correct.
The safeguards of the global financial system relating to the U.S. Foreign Corrupt Practices Act
deal with minimizing money laundering, terrorist financing, and extortion and bribery. Insider
trading is not a focus of the safeguards.
Answer (C) is incorrect.
Terrorist financing is one focus of the safeguards of the global financial system relating to the
U.S. Foreign Corrupt Practices Act.
Answer (D) is incorrect.
Extortion and bribery are focuses of the safeguards of the global financial system relating to the
U.S. Foreign Corrupt Practices Act.
Question #16: According to IMA's Statement of Ethical Professional Practice, a member has a
responsibility to recognize professional limitations. Under which standard of ethical conduct
would this responsibility be included?
Correct Answer: A. Competence.
A. Competence.
B. Confidentiality.
C. Integrity.
D. Credibility.
Explanation:
Answer (A) is correct.
The competence standard pertains to an IMA member's responsibility to "recognize and
communicate professional limitations or other constraints that would preclude responsible
judgment or successful performance of an activity."
Answer (B) is incorrect.
The confidentiality standard concerns an IMA member's responsibility not to disclose or use the
firm's confidential information.
Answer (C) is incorrect.
The integrity standard deals with conflicts of interest, avoidance of acts discreditable to the
profession, and refraining from activities that prejudice the ability to carry out duties ethically.
Answer (D) is incorrect.
Credibility is the fourth standard of IMA's Statement of Ethical Professional Practice. It requires
that information be communicated "fairly and objectively and that all information that could
reasonably influence users be disclosed.

Question #17: Sudden economic changes have forced a company to alter its business strategy.
The company is considering eliminating product lines, laying off production workers, reducing
advertising, and closing one of its factories. In taking these actions, which one of the following
costs should be considered sunk costs?
Correct Answer: C. Research and development costs of eliminated product lines.
A. Production workers' wages, severance, and advertising.
B. Utility costs at the closed factory and real estate taxes.
C. Research and development costs of eliminated product lines.
D. The costs of selling or demolishing the factory.
Explanation:
Answer (A) is incorrect.
Relevant costs must be made in the future and differ among the possible alternative courses of
action. By definition, a sunk cost is not relevant. Product workers' wages, severance, and
advertising are all costs that are to be made in the future and can differ among alternative courses
of action. Accordingly, these costs are relevant to the decision at hand and are not "sunk."
Answer (B) is incorrect.
Relevant costs must be made in the future and differ among the possible alternative courses of
action. By definition, a sunk cost is not relevant. Utility costs at the closed factory and real estate
taxes are both costs that are to be made in the future and can differ among alternative courses of
action. Accordingly, these costs are relevant the decision at hand and are not "sunk."
Answer (C) is correct.
Costs that have already been incurred or to which the organization is committed are called sunk
costs. These costs have no bearing on any future decisions. The research and development costs
of eliminated product lines have already been incurred and cannot be reversed as a result of
eliminating the product lines. Therefore, this answer choice correctly represents sunk costs.
Answer (D) is incorrect.
Relevant costs must be made in the future and differ among the possible alternative courses of
action. By definition, a sunk cost is not relevant. The cost of selling or demolishing the factory is
a cost that is to be made in the future and can differ among alternative courses of action.
Accordingly, this cost is relevant the decision at hand and is nor “sunk”.

Question #18: The CFO at a manufacturer of computer equipment learned last week that the
accounting department has not completed any bank reconciliations for the last 6 months due to
the implementation of a new accounting software package. What type of risk has been
identified?
Correct Answer: C. Operational risk.
A. Financial risk.
B. Hazard risk.
C. Operational risk.
D. Strategic risk.
Explanation:
Answer (A) is incorrect.
Financial risks encompass interest-rate risk, exchange-rate risk, commodity risk, credit risk,
liquidity risk, and market risk.
Answer (B) is incorrect.
Hazard risks are risks that are insurable. Examples include natural disasters, the incapacity or
death of senior officers, sabotage, and terrorism.
Answer (C) is correct.
Operational risks are the risks related to the enterprises ongoing, everyday operations.
Operational risk is the risk of loss from inadequate or failed internal processes, people, and
systems. These failures can relate to human resources (e.g. inadequate hiring or training
practices), business processes (poor internal controls), product failure (customer ill will.
lawsuits), occupational safety and health incidents, environmental damage, and business
continuity (power outages. natural disasters).
Answer (D) is incorrect.
Strategic risks include global economic risk, political risk, and regulatory risk.

Question #19: When only differential manufacturing costs are taken into account for special-
order pricing, an essential assumption is that
Correct Answer: C. Acceptance of the order will not affect regular sales.
A. Manufacturing fixed and variable costs are linear.
B. Selling and administrative fixed and variable costs are linear.
C. Acceptance of the order will not affect regular sales.
D. Acceptance of the order will not cause unit selling and administrative variable costs to
increase.
Explanation:
Answer (A) is incorrect.
The differential analysis of a special order considers total marginal costs. Thus, unit variable
costs and total fixed costs need not be constant, and any changes need not be in direct proportion
to the measure of activity.
Answer (B) is incorrect.
The assumption is that selling and administrative costs are not relevant.
Answer (C) is correct.
Granting a lower-than-normal price for a special order has potential ramifications for regular
sales because other customers may demand the same price. Thus, the decision to consider
differential manufacturing costs only should be based on a determination that all other costs are
not relevant, that is, that these other costs do not vary with the option chosen.
Answer (D) is incorrect.
The assumption is that acceptance of the order will not cause total selling and administrative
costs to change.

Question #20: Enterprise Risk Management (ERM) is closely aligned with corporate governance
because it
Correct Answer: B. Identifies which of the organizations' objectives is at greatest risk.
A. Focuses management's attention on the risks mitigated.
B. Identifies which of the organizations' objectives is at greatest risk.
C. Reduces the level of acceptable risks to be taken.
D. Identifies and isolates the silos in which risk exists.
Explanation:
Answer (A) is incorrect.
ERM focuses management's attention on all of the risks, especially ones that need to be
mitigated, not the ones that have already been mitigated.
Answer (B) is correct.
ERM recognizes risk management across the entire enterprise, so it identifies and responds to the
organization's greatest risks. Managing the risks of an organization is one of the goals of
corporate governance.
Answer (C) is incorrect.
ERM does not reduce the level of acceptable risk. but rather lowers the biggest risks of the
organization.
Answer (D) is incorrect.
ERM does not isolate risks: it assesses and responds to them.

Question #21: If gross profit margin has decreased substantially over the past 3 years. which one
of the following best explains this decrease?
Correct Answer: C. A physical count of merchandise inventory showed missing inventory higher
than expected. All Possible Answers:
A. The cost of merchandise inventory has decreased while sales prices have remained the
same.
B. Ending merchandise inventory is higher than expected.
C. A physical count of merchandise inventory showed missing inventory higher than
expected.
D. Cost of goods sold has remained steady while total expenses have increased.
Explanation:
Answer (A) is incorrect.
Gross profit margin is net sales minus cost of goods sold. Lower costs of merchandise will result
in a lower cost of goods sold, which increases, not decreases, gross profit margin.
Answer (B) is incorrect.
Gross profit margin is net sales minus cost of goods sold. Thus, a higher ending inventory results
in a lower cost of goods sold, which results in an increase of gross profit margin.
Answer (C) is correct.
A physical count showing missing inventory results in a higher cost of goods sold, which results
in a decrease of gross profit margin.
Answer (D) is incorrect.
Gross profit margin is net sales minus cost of goods sold. Thus, gross profit margin remains
unchanged if both sales and cost of goods sold remain steady.

Question #22: (Please refer the fact pattern below)


Question: The quick (acid-test) ratio at year end is
Broomall Corporation has decided to include certain financial ratios in its year-end annual report
to shareholders. Selected information relating to its most recent fiscal year is provided below.

Cash $ 10,000
Accounts receivable:
Beginning of year 24,000
End of year 20,000
Prepaid expenses 8,000
Inventory:
Beginning of year 26,000
End of year 30,000
Available-for-sale securities:
Historical cost 9,000
Fair value at year end 12,000
Accounts payable 15,000
Notes payable ( due in 90 days) 25,000
Bonds payable (due in 10 days) 35,000
Net credit sales for year 220,000
Cost of goods sold 140,000

Correct Answer: D. 1.05 to 1.


A. 2.00 to 1.
B. 1.925 to 1.
C. 1.80 to 1.
D. 1.05 to 1.

Explanation:
Answer (A) is incorrect.
The numerator should not include prepaid expenses or inventory. These assets are not very
liquid, so they are not considered in the acid test ratio.
Answer (B) is incorrect.
The numerator should include the available-for-sale securities at fair value at year end, and it
should not include prepaid expenses, inventory, or available-for-sale securities at cost. These
assets are not very liquid, so they are not considered in the acid test ratio.
Answer (C) is incorrect.
The numerator should not include inventory. Inventory is not very liquid, so it is not considered
in the acid test ratio.
Answer (D) is correct.
The acid test ratio is 1.05 as shown below. ($10,000 + $20,000 + $12.000) + ($15,000 +
$25,000) = 1.05

Question #23: A corporation is subject to a 30% effective income tax rate and uses the net
present value method to evaluate capital budgeting proposals. The capital budget manager
desires to improve the appeal of a marginally attractive proposal. To accomplish his goal, which
one of the following actions should be recommended?
Correct Answer: A. Postpone a fully deductible major overhaul from Year 4 to Year 5.
A. Postpone a fully deductible major overhaul from Year 4 to Year 5.
B. Decrease the project's estimated terminal salvage value.
C. Immediately pay the proposal's marketing program in its entirety rather than pay in five
equal installments.
D. Adjust the project's discount rate to reflect movement of the project from a low risk
category to an average risk category.
Explanation:
Answer (A) is correct.
Delaying cash outflows increases a project's net present value, making it more attractive.
Answer (B) is incorrect.
Decreasing the terminal salvage value of a project will result in greater depreciation expense
over the project's lifetime, which decreases taxable income and thus possibly makes the project
appear more attractive. However, the decrease in the terminal salvage value also results in a
smaller estimated cash inflow during the year of disposal, which would make the project appear
less attractive. The net effect of this change is therefore indeterminable, so it should not be
recommended as an action to take.
Answer (C) is incorrect.
Speeding up cash outflows will make the project less attractive.
Answer (D) is incorrect.
Reclassifying a project as average risk from low risk will increase the project's hurdle rate,
making it less attractive.

Question #24: Question: The Company has a fixed assets balance of


[Fact Pattern #14] A company has a current ratio of 1.4, a quick, or acid test, ratio of 1.2, and the
following partial summary balance sheet:

Cash $ 10
Accounts Receivable __
Inventory __
Fixed assets __
Total assets $ 100
Current liabilities $__
Long term liabilities 40
Stockholders' equity 30
Total liabilities and equity $__

Correct Answer: B. $58


A. $16
B. $58
C. $64
D. $84

Explanation:
Answer (A) is incorrect.
Neglecting to subtract the equity balance when calculating the current liability balance results in
$16.
Answer (B) is correct.
Total assets (given as $100) equals the sum of cash (given as $10), accounts receivable ($26 as
calculated using the quick ratio), inventory, and fixed assets. Inventory can be determined
because it is included in current, but not quick, assets, and the current and quick ratios are
known. Current assets equal $42 (1.4 current ratio x $30 current liabilities), and the quick assets
equal $36 (1.2 quick ratio x $30 current liabilities). Thus, inventory, which is the only difference
in this question between current and quick assets, equals $6 ($42 - $36). Fixed assets must then
equal $58 ($100 total assets - $10 cash - $26 accounts receivable - $6 inventory).
Answer (C) is incorrect.
Assuming that inventory is $0 results in $64.
Answer (D) is incorrect.
Ignoring accounts receivable results in $84.

Question #25: Which ethical standard is most clearly violated if an IMA member knows of a
problem that could mislead users but does nothing about it?
Correct Answer: C. Credibility.
A. Competence.
B. Legality.
C. Credibility.
D. Confidentiality.
Explanation:
Answer (A) is incorrect.
The competence standard pertains to the IMA member's responsibility to maintain his or her
professional skills and knowledge. It also pertains to the performance of activities in a
professional manner.
Answer (B) is incorrect.
Legality is not addressed in IMA's Statement of Ethical Professional Practice.
Answer (C) is correct.
Credibility is the fourth part of I MAs Statement of Ethical Professional Practice. It requires that
information be communicated 'fairly and objectively' and that all information that could
reasonably influence users be disclosed.
Answer (D) is incorrect.
The confidentiality standard concerns the IMA member's responsibility not to disclose or use the
firm's confidential information.

Question #26: If the price of apples declines and total revenue received by the firm increases. the
Correct Answer: A. Demand for apples is elastic.
A. Demand for apples is elastic.
B. Demand for apples is inelastic.
C. Elasticity of demand for apples is 1.0.
D. Elasticity of demand for apples is less than 1.0.
Explanation:
Answer (A) is correct.
A decline in price accompanied by an increase in total revenue indicates that quantity demanded
has increased by a greater percentage than the percentage price decrease. Hence, the price
elasticity of demand is greater than 1.0. Demand is elastic when it is greater than 1.0.
Answer (B) is incorrect.
The increase in revenue resulting from the price decrease does not indicate inelasticity.
Answer (C) is incorrect.
Elasticity must be greater than 1.0 if the total revenue increases as the result of a price decrease.
Answer (D) is incorrect.
Total revenues received by the firm decrease if the elasticity is less than 1.0.
Question #27: I MA's Statement of Ethical Professional Practice requires an I MA member to
follow the established policies of the organization when faced with an ethical conflict. If these
policies do not resolve the conflict, the member should
Correct Answer: • Contact the next higher managerial level if initial presentation to the
immediate superior does not resolve the conflict.
A. Consult the board of directors immediately.
B. Discuss the problem with the immediate superior if (s)he is involved in the conflict.
C. Communicate the problem to authorities outside the organization.
D. Contact the next higher managerial level if initial presentation to the immediate superior
does not resolve the conflict.
Explanation:
Answer (A) is incorrect.
This course of action would be appropriate only for the chief executive officer or for his or her
immediate subordinate when the CEO is involved in the conflict.
Answer (B) is incorrect.
The proper action would be to present the matter to the next higher managerial level.
Answer (C) is incorrect.
Such action is inappropriate unless legally prescribed.
Answer (D) is correct.
In these circumstances, the problem should be discussed with the immediate superior unless
(s)he is involved. In that case initial presentation should be to the next higher managerial level. If
the problem is not satisfactorily resolved after initial presentation, the question should be
submitted to the next higher level.

Question #28: Segregation of duties is a fundamental concept in an effective system of internal


control. Nevertheless, the internal auditor must be aware that this safeguard can be compromised
through
Correct Answer: B. Collusion among employees.
A. Lack of training of employees.
B. Collusion among employees.
C. Irregular employee reviews.
D. Absence of internal auditing.
Explanation:
Answer (A) is incorrect.
Lack of training by itself cannot negate effective separation of duties.
Answer (B) is correct.
By segregating duties, organizations make it more difficult for one person to perpetrate a fraud.
When custody of the asset and recordkeeping for the asset are invested in different persons, a
fraud generally cannot be executed by one of the two parties. However, if they collude, the
internal control aspect of the segregation is nullified.
Answer (C) is incorrect.
Irregular employee reviews may affect employee job performance, but they alone cannot negate
effective separation of duties.
Answer (D) is incorrect.
While the absence of an internal audit activity may lessen the chances that an organization will
maintain effective internal control over the long run, by itself it cannot negate effective
segregation of duties.

Question #29: If Company A has a higher rate of return on assets than Company B, the reason
may be that Company A has a profit margin on sales, a asset turnover ratio, or both.
Correct Answer: A. Higher (List A). (List B) Higher
A. Higher (List A). (List B) Higher
B. Higher (List A). (List B) Lower
C. Lower (List A). (List B) Higher
D. Lower (List A). (List B) Lower
Explanation:
Answer (A) is correct.
The DuPont model treats the return on assets as the product of the profit margin and the asset
turnover.
Return on assets - Profit margin + Asset turnover
Net income Net income Sales
= ×
Assets Sales Assets
If one company has a higher return on assets than another, it may have a higher profit margin, a
higher asset turnover, or both.
Answer (B) is incorrect.
A higher profit margin on sales or a higher asset-turnover ratio may explain a higher return on
assets.
Answer (C) is incorrect.
A higher profit margin on sales or a higher asset-turnover ratio may explain a higher return on
assets.
Answer (D) is incorrect.
A higher profit margin on sales or a higher asset-turnover ratio may explain a higher return on
assets.

Question #30: A company sells its single product for $30 per unit. The contribution margin ratio
is 45%, and fixed costs are $10,000 per month. The company has an effective income tax rate of
40%. If the company sells 1.000 units in the current month, the company's variable expenses
would be
Correct Answer: D. $16,500
A. $9,900
B. $12,000
C. $13,500
D. $16,500
Explanation:
Answer (A) is incorrect.
The amount of $9,900 results from improperly applying a variable cost ratio of 33% rather than
the correct ratio of (1- contribution margin ratio).
Answer (B) is incorrect.
The amount of $12,000 results from applying the tax rate rather than the variable cost ratio (1-
contribution margin ratio).
Answer (C) is incorrect.
The amount of $13,500 results from reversing the gross margin ratio.
Answer (D) is correct.
The company's contribution margin ratio reveals that 45% of the sales price of each product is
contribution margin. Thus, 55% of the sales price goes to variable costs (100% - 45%), making
the perunit variable cost $16.50 ($30 x 55%). Since the company sold 1,000 units in the current
month. total variable costs were $16,500 (1,000 x $16.50).

Question #31: Misappropriation of assets is most often perpetrated by


Correct Answer: A. Employees.
A. Employees.
B. Customers.
C. Management.
D. Auditors.
Explanation:
Answer (A) is correct.
Employees who have fiduciary responsibilities for assets are most likely to steal them.
Answer (B) is incorrect.
Customers may misappropriate assets, but most businesses are more vulnerable to wrongdoing
by employees.
Answer (C) is incorrect.
Management is most likely to misstate the financial statements.
Answer (D) is incorrect.
Independent auditors have little opportunity or motive to misappropriate assets.

Question #32: A manufacturer is considering introducing a new product that will require a
$250,000 investment of capital. The necessary funds would be raised through a bank loan at an
interest rate of 8%. The fixed operating costs associated with the product would be $122,500,
while the contribution margin percentage would be 42%. Assuming a selling price of $15 per
unit, determine the number of units (rounded to the nearest whole unit) the manufacturer would
have to sell to generate earnings before interest and taxes (EBIT) of 32% of the amount of capital
invested in the new product.
Correct Answer: B. 32,143 units.
A. 35,318 units.
B. 32,143 units.
C. 25,575 units.
D. 23,276 units.
Explanation:
Answer (A) is incorrect.
Improperly including interest as a fixed cost results in 35,318.
Answer (B) is correct.
The manufacturer has determined it must generate EBIT equal to 32% of the capital invested in
this project, or $80,000 ($250,000 x 32%). The number of units it must produce to achieve this
level of EBIT can be derived as follows: Breakeven point = (Fixed costs + EBIT) + Unit
contribution margin = ($122,500 + $80,000) + ($15 x 42%) = $202,500 + $6.30 = 32,142.86
units
Answer (C) is incorrect.
Improperly including interest as a fixed cost and using the cost percentage in the calculation
instead of the contribution margin percentage results in 25,575.
Answer (D) is incorrect.
Improperly using the complement of the contribution margin percentage instead of the
contribution margin percentage results in 23,276.

Question #33: A credit manager considering whether to grant trade credit to a new customer is
most likely to place primary emphasis on
Correct Answer: D. Liquidity ratios. All Possible Answers:
A. Profitability ratios.
B. Valuation ratios.
C. Growth ratios.
D. Liquidity ratios.
Explanation:
Answer (A) is incorrect.
Equity investors are concerned with profitability ratios.
Answer (B) is incorrect.
Equity investors are concerned with valuation ratios.
Answer (C) is incorrect.
Equity investors are concerned with growth ratios.
Answer (D) is correct.
Liquidity is a firm's ability to pay its short-term obligations as they come due. Trade payables are
the most common form of short-term obligation. Thus, a credit manager is most interested in
assessing a potential customer's liquidity.

Question #34: In assessing the financial prospects for a firm, financial analysts use various
techniques. An example of vertical, common-size analysis is
Correct Answer: D. Advertising expense for the current year is 2% of sales. All Possible
Answers:
A. An assessment of the relative stability of a firm's level of vertical integration.
B. A comparison in financial ratio form between two or more firms in the same industry.
C. Advertising expense is 2% greater compared with the previous year.
D. Advertising expense for the current year is 2% of sales.
Explanation:
Answer (A) is incorrect.
Vertical integration occurs when a corporation owns one or more of its suppliers or customers.
Answer (B) is incorrect.
Vertical, common-size analysis restates financial statements amounts as percentages.
Answer (C) is incorrect.
A statement that advertising expense is 2% greater than in the previous year results from
horizontal analysis.
Answer (D) is correct.
Vertical, common-size analysis compares the components within a set of financial statements. A
base amount is assigned a value of 100%. For example, total assets on a common-size balance
sheet and net sales on a common-size income statement are valued at 100%. Common-size
statements permit evaluation of the efficiency of various aspects of operations. An analyst who
states that advertising expense is 2% of sales is using vertical, common-size analysis.
Question #35: Fad Pattern: Excerpts from the statement of financial position for Markham
Corporation as of April 30 of the current year are presented as follows:

Cash $ 725,000
Accounts receivable (net) 1,640,000
Inventories 2,945,000
Total current assets $ 5,310,000
Accounts payable $1,236,000
Accrued liabilities 831,000
Total current liabilities $ 2,067,000

The board of directors of Markhammet on May 5 of the current year and declared a quarterly
cash dividend in the amount of $800,000 ($.50 per share). The dividend was paid on May 28 of
the current year to shareholders of record as of May 15 of the current year. Assume that the only
transactions that affected Markham during May of the current year were the dividend
transactions and that the closing entries have been made.
Question: Markham's working capital would be
Correct Answer: A. Decreased by the dividend declaration and unchanged by the dividend
payment.
A. Decreased by the dividend declaration and increased by the dividend payment.
B. Unchanged by either the dividend declaration or the dividend payment.
C. Decreased by the dividend declaration and unchanged by the dividend payment.
D. Increased by the dividend declaration and unchanged by the dividend payment.

Explanation:
Answer (A) is incorrect.
The dividend payment will not change working capital, but the declaration of the dividend
decreases working capital.
Answer (B) is incorrect.
The dividend payment will not change working capital, but the declaration of the dividend
decreases working capital.
Answer (C) is correct.
Working capital is the excess of current assets over current liabilities. The declaration of a
dividend reduces retained earnings and creates a new current liability. Thus, the declaration of a
dividend reduces working capital because current liabilities are increased without a
corresponding increase in current assets. The subsequent payment of the dividend has no effect
on working capital because current assets (cash) will be reduced by the same amount that current
liabilities (dividends payable) are reduced.
Answer (D) is incorrect.
The dividend payment will not change working capital, but the declaration of the dividend
decreases working capital.

Question #36: After a petition for bankruptcy liquidation has been filed and the court has issued
an order for relief,
Correct Answer: B. Creditors must immediately cease their collection activities.
A. The court usually appoints a permanent trustee to take control of the debtor's estate.
B. Creditors must immediately cease their collection activities.
C. The bankruptcy judge notifies creditors, collects the debtor's nonexempt property, and
distributes that property to the creditors.
D. A meeting is held by the creditors to vote on a plan of reorganization.
Explanation:
Answer (A) is incorrect.
The court usually appoints a temporary trustee, and the creditors usually elect a permanent
trustee.
Answer (B) is correct.
A debtor files a bankruptcy petition to obtain relief from creditors' collection efforts. Creditors
may file to ensure an equitable division of the debtor's estate, an outcome that may not be
reached if creditors are allowed to continue their individual collection activities. Thus, the order
for relief stays those activities.
Answer (C) is incorrect.
The trustee, not the judge, administers the estate.
Answer (D) is incorrect.
The creditors vote on a plan of reorganization under Chapter 11.

Question #37: Fad Pattern: Hobart Corporation evaluates capital projects using a variety of
performance screens, including a hurdle rate of 16%, payback period of 3 years or less, and an
accounting rate of return of
20% or more.
Question: Hobart's management is completing review of a project on the basis of the following
projections: Capital investment $200,000 Annual cash flows $65,000 Straight-line depredation 8
years Terminal value $20,000. The projected net present value is negative $2,000. Which one of
the following alternatives reflects the appropriate conclusions for the indicated evaluative
measures?
Correct Answer: D. Reject (Net Present Value). (Payback) Reject
A. Accept (Net Present Value). (Payback) Reject
B. Reject (Net Present Value). (Payback) Accept
C. Accept (Net Present Value). (Payback) Accept
D. Reject (Net Present Value). (Payback) Reject

Explanation:
Answer (A) is incorrect.
The appropriate decision under the net present value method is also to reject.
Answer (B) is incorrect.
The appropriate decision under the payback method is also to reject.
Answer (C) is incorrect.
The appropriate decision under both methods is to reject the project.
Answer (D) is correct.
A capital project is acceptable if the net present value of its cash flows is greater than zero. Since
the net present value of this project is negative, the appropriate decision is to reject it. Under the
payback method, the undiscounted cash inflows must exceed the undiscounted cash outflows
within a specified period. Since it takes more than 3 years for the inflows from this project
($65,000 x 3 = $195,000) to exceed the outflows ($200,000), the appropriate decision under this
method is also to reject.

Question #38: A corporation has a current ratio of 2 to 1 and a quick ratio (acid test) of 1 to 1. A
transaction that would change the quick ratio but not the current ratio is the
Correct Answer: A. Sale of inventory on account at cost.
A. Sale of inventory on account at cost.
B. Collection of accounts receivable.
C. Payment of accounts payable.
D. Purchase of a patent for cash
Explanation:
Answer (A) is correct.
The quick (acid test) ratio equals the quick assets (cash marketable securities. and accounts
receivable) divided by current liabilities. The current ratio is equal to current assets divided by
current liabilities. The sale of inventory (not a quick current asset) on account increases accounts
receivable (a quick asset), thereby changing the quick ratio. The sale of inventory on account,
however, replaces one current asset with another, and the current ratio is unaffected.
Answer (B) is incorrect.
Neither ratio is changed.
Answer (C) is incorrect.
The current, not the quick, ratio changes.
Answer (D) is incorrect.
Both decrease.

Question #39: Based on the fraud risk model, which of the following is the most likely motive
for employee theft?
Correct Answer: A. Gambling losses.
A. Gambling losses.
B. Ineffective internal controls.
C. Belief that rank has its privileges
D. Ineffective supervision.
Explanation:
Answer (A) is correct.
An employee's actual or perceived need for assets, for example, from gambling losses, can create
the motivation to commit fraud.
Answer (B) is incorrect.
Ineffective internal controls create the opportunity to commit fraud.
Answer (C) is incorrect.
The belief that rank has its privileges is a justification that some employees may use to justify
fraud.
Answer (D) is incorrect.
Ineffective supervision creates the opportunity to commit fraud.
Question #40: The reporting of accounting information plays a central role in the regulation of
business operations. The importance of sound internal control practices is underscored by the
Foreign Corrupt Practices Act of 1977, which requires publicly owned U.S. corporations to
maintain systems of internal control that meet certain minimum standards. Preventive controls
are an integral part of virtually all accounting processing systems, and much of the information
generated by the accounting system is used for preventive control purposes. Which one of the
following is not an essential element of a sound preventive control system?
Correct Answer: D. Implementation of state-of-the-art software and hardware.
A. Separation of responsibilities for the recording, custodial, and authorization functions.
B. Sound personnel practices.
C. Documentation of policies and procedures.
D. Implementation of state-of-the-art software and hardware.
Explanation:
Answer (A) is incorrect.
Segregation of functions makes it more difficult for one person both to perpetrate and conceal an
irregularity.
Answer (B) is incorrect.
Hiring honest and capable employees prevents many problems.
Answer (C) is incorrect.
Documentation provides a guide for conduct.
Answer (D) is correct.
Preventive controls are designed to prevent an error or irregularity from occurring. State-of-the-
art hardware and software would presumably incorporate the latest control features, but a less
advanced system could very well contain a sound preventive control structure. Hence, state-of-
the-art components are not essential for effective control.

Question #41: At the beginning of the year. a portfolio manager who manages a portfolio with a
mean annual return of 8% and annual standard deviation of 25% wants to estimate the worst-case
expected loss at an 80% confidence level. The value of the portfolio today is $5 million. Which
method would the portfolio manager use to estimate the probable maximum loss that may be
incurred at the end of the year?
Correct Answer: D. Value-at-risk. All Possible Answers:
A. Arbitrage pricing theory.
B. Capital asset pricing model.
C. Covariance.
D. Value-at-risk.
Explanation:
Answer (A) is incorrect.
The arbitrage pricing theory (APT) describes the price where a mispriced asset is expected to be.
It is not used to estimate probable loss.
Answer (B) is incorrect.
The capital asset pricing model is a model that describes the relationship between risk and
expected return and is used in the pricing of risky securities. It is not used to estimate probable
loss.
Answer (C) is incorrect.
Covariance is a measure of the degree to which returns on two risky assets move in tandem. It is
not used to estimate probable loss.
Answer (D) is correct.
Value-at-risk is a statistical technique used to measure and quantify the level of financial risk
within a firm or investment portfolio over a specific time frame.

Question #42: Which one of the following statements best represents the order of the steps in
developing target prices?
Correct Answer: C. Determine market price, calculate target cost, and use value engineering to
reduce costs.
A. Use value engineering and kaizen costing to reduce costs and determine desired price.
B. Use kaizen costing to reduce costs, determine desired mark-up, and set market price.
C. Determine market price, calculate target cost, and use value engineering to reduce costs.
D. Use value engineering to reduce costs, calculate target costs, and set desired price.
Explanation:
Answer (A) is incorrect.
The company must first determine the market price with target pricing and calculate target cost
before using value engineering.
Answer (B) is incorrect.
The company must first determine the market price and calculate target cost, then use value
engineering to reduce costs.
Answer (C) is correct.
The company must determine market price, calculate target cost, and then use value engineering
to reduce costs.
Answer (D) is incorrect.
The company must first determine the market price and then use value engineering to reduce
costs.

Question #43: A company is analyzing a capital investment proposal for new equipment to
produce a product over the next 8 years. The analyst is attempting to determine the appropriate
"end-of-life" cash flows for the analysis. At the end of 8 years, the equipment must be removed
from the plant and will have a net book value of zero, a tax basis of $75.000, a cost to remove of
$40.000, and scrap salvage value of $10,000. The effective tax rate is 40%. What is the
appropriate "end-of-life" cash flow related to these items that should be used in the analysis?
Correct Answer: C. $12.000 All Possible Answers:
A. $45,000
B. $27,000
C. $12,000
D. $(18,000)
Explanation:
Answer (A) is incorrect.
The amount of $45,000 ignores income taxes and assumes that the loss on disposal involves a
cash inflow.
Answer (B) is incorrect.
The amount of $27.000 assumes that the loss on disposal involves a cash inflow.
Answer (C) is correct.
The tax basis of $75,000 and the $40,000 cost to remove can be written off. However, the
$10,000 scrap value is a cash inflow. Thus, the taxable loss is $105,000 ($75,000 loss on
disposal + $40,000 expense to remove - $10,000 of inflows). At a 40% tax rate, the $105,000
loss will produce a tax savings (inflow) of $42,000. The final cash flows will consist of an
outflow of $40,000 (cost to remove) and inflows of $10,000 (scrap) and $42,000 (tax savings), or
a net inflow of $12,000.
Answer (D) is incorrect.
The amount of $(18,000) ignores the tax loss on disposal.

Question #44: A financial manager has discovered that her company is violating environmental
regulations. If her immediate superior is involved, the appropriate action is to
Correct Answer: C. Present the matter to the next higher managerial level.
A. Do nothing since she has a duty of loyalty to the organization.
B. Consult the audit committee.
C. Present the matter to the next higher managerial level.
D. Confront her immediate superior.
Explanation:
Answer (A) is incorrect.
Practitioners of management accounting and financial management have an obligation to the
public, their profession, the organization they serve, and themselves, to maintain the highest
standards of ethical conduct.
Answer (B) is incorrect.
The audit committee would be consulted first only if it were the next higher managerial level.
Answer (C) is correct.
To resolve an ethical problem, the financial manager/management accountant's first step is
usually to consult his or her immediate superior. If that individual is involved, the matter should
be taken to the next higher level of management.
Answer (D) is incorrect.
If the superior is involved, the next higher managerial level should be consulted first.

Question #45: The key difference between accounting profit and economic profit is that
economic profit
Correct Answer: D. Considers the opportunity cost of equity.
A. Highlights the historical cost concept.
B. Calculates changes in supply using EOQ models.
C. Excludes income tax and interest expense.
D. Considers the opportunity cost of equity.
Explanation:
Answer (A) is incorrect.
Both highlight the historical cost concept.
Answer (B) is incorrect.
Either can calculate changes in supply using EOQ models.
Answer (C) is incorrect.
Both include income tax and interest expense.
Answer (D) is correct.
Economic profit is the excess of revenues over the costs of land, labor, and equity. Accountants,
however, do not subtract the cost of investors' equity.
Question #46: Internal control cannot be designed to provide reasonable assurance regarding the
achievement of objectives concerning
Correct Answer: B. Elimination of all fraud.
A. Reliability of financial reporting.
B. Elimination of all fraud.
C. Compliance with applicable laws and regulations.
D. Effectiveness and efficiency of operations.
Explanation:
Answer (A) is incorrect.
Internal control can provide reasonable assurance regarding reliability of financial reporting.
Answer (B) is correct.
Internal control is a process designed to provide reasonable assurance regarding the achievement
of organizational objectives. Because of inherent limitations, however, no system can be
designed to eliminate all fraud.
Answer (C) is incorrect.
Internal control can provide reasonable assurance regarding compliance with applicable laws and
regulations.
Answer (D) is incorrect.
Internal control can provide reasonable assurance regarding effectiveness and efficiency of
operations.
Question #47: (Please refer the fact pattern below)
Question: The Bidwell Company's breakeven sales in units are
[Fact Pattern] The budget data for the Bidwell Company appear below.

Sales (100,000 units) $1,000,000


Costs: Fixed Variable
Direct materials $0 $300,000
Direct labor 0 200,000
Manufacturing overhead 100,000 150,000
Selling and administrative costs 110,000 50,000
Total costs $210,000 $700,000 910,000
Budgeted operating income $90,000

Correct Answer: D. 70,000 units.


A. 30,000 units.
B. 91,000 units.
C. 60,000 units.
D. 70,000 units.
Explanation:
Answer (A) is incorrect.
Fixed costs should be divided by the contribution margin, not unit variable costs.
Answer (B) is incorrect.
Total cost ÷ selling price is 91,000 units.
Answer (C) is incorrect.
Selling and administrative costs are included in the contribution margin.
Answer (D) is correct.
The breakeven point in units is found by dividing total fixed costs by the contribution margin per
unit. Variable costs are maw at 100,000 units, or $7 per unit. Selling price is $10 per unit,
resulting in a contribution margin per unit of $3. Dividing $210,000 of fixed costs by $3 per unit
results in a breakeven point of 70,000 units.
Question #48: A corporation sells two products with the following characteristics:
Product Product
1 2
Contribution margin ratio 40% 50%
Percentage of sales dollars 40% 60%
$240,00 $700,00
Fixed costs 0 0

Correct Answer: B. $2,043,478


A. $2,000,000
B. $2,043,478
C. $2,088,889
D. $2,136,364

Explanation:
Answer (A) is incorrect.
The amount of $2,000,000 results from incorrectly dividing Product 1's and Product 2's
individual fixed costs by their contribution margin ratio, failing to account for their respective
percentage of sales dollars.
Answer (B) is correct.
The breakeven point in dollars is obtained by calculating a weighted-average contribution ratio
and dividing it into fixed costs. The ratio is calculated as follows:
CM Ratio % of sales dollars
Product 140% x 40% = .16
Product 2 50% x 60% = .30
Weighted-average contribution ratio: .46
Total fixed costs are $940,000 ($700,000 + $240,000), so the breakeven point is equal to
$2,043,478 ($940,000 ÷ .46).
Answer (C) is incorrect.
The amount of $2,088,889 results from incorrectly dividing fixed costs by the average
contribution margin ratio of Product 1 and Product 2 instead of using the weighted-average
contribution margin.
Answer (D) is incorrect.
The amount of $2,136,364 results from incorrectly using 60% percentage of sales dollars for
Product 1 and 40% percentage of sales dollars for Product 2 when calculating the weighted-
average contribution margin.

Question #49: In an insourcing vs. outsourcing decision, the decision process favors the use of
total costs rather than unit costs. The reason is that
Correct Answer: D. All of the answers are correct.
A. Unit cost may be calculated based on different volumes.
B. Irrelevant costs may be included in the unit amounts.
C. Allocated costs may be included in the unit amounts.
D. All of the answers are correct.
Explanation:
Answer (A) is incorrect.
Reasons favoring total costs are that unit cost may be calculated based on different volumes costs
may be included in the unit amounts.
Answer (B) is incorrect.
Reasons favoring total costs are that unit cost may be calculated based on different volumes costs
may be included in the unit amounts.
Answer (C) is incorrect.
Reasons favoring total costs are that unit cost may be calculated based on different volumes costs
may be included in the unit amounts.
Answer (D) is correct.
Unit costs should be used with extreme care. In each situation, they may be calculated based
anticipated, so comparability may be lost. Irrelevant costs included in the unit cost should be
included in the analysis. Allocated costs should also be ignored, and only the relevant cost
should be considered, and that allocated costs and irrelevant and that allocated costs and
irrelevant and that allocated costs and irrelevant on a different volume level from that
disregarded: only relevant costs should s that will change with the option chosen.

Question #50: Fad Pattern: The Moore Corporation is considering the acquisition of a new
machine. The machine can be purchased for $90.000: it will cost $6,000 to transport to Moore's
plant and $9,000 to install. It is estimated that the machine will last 10 years, and it is expected to
have an estimated salvage value of $5.000. Over its 10-year life, the machine is expected to
produce 2.000 units per year, each with a selling price of $500 and combined material and labor
costs of $450 per unit. Federal tax regulations permit machines of this type to be depreciated
using the straight-line method over 5 years with no estimated salvage value. Moore has a
marginal tax rate of 40%.
Question: What is the net cash flow for the third year that Moore Corporation should use in a
capital budgeting analysis?
Correct Answer: A. $68,400
A. $68,400
B. $68,000
C. $64,200
D. $79,000
Explanation:
Answer (A) is correct.
The company will receive net cash inflows of $50 per unit ($500 selling price - $450 of variable
costs), or a total of $100.000 per year. This amount will be subject to taxation. But, for the first 5
years, there will be a depreciation deduction of $21,000 per year ($105,000 cost divided by 5
years). Therefore, deducting the $21,000 of depreciation expense from the $100,000 of
contribution margin will result in taxable income of $79,000. After income taxes of $31,600
($79,000 x 40%). the net cash flow in the third year is $68,400 ($100,000 - $31,600).
Answer (B) is incorrect.
The amount of $68,000 deducts salvage value when calculating depreciation expense, which is
not required by the tax law.
Answer (C) is incorrect.
The amount of $64,200 assumes depreciation is deducted for tax purposes over 10 years rather
than 5 years.
Answer (D) is incorrect.
The amount of $79,000 is taxable income.

Question #51: The Robo Division, which is part of a large company, has been approached to
submit a bid for a potential project for a customer, Robo Division has been informed by the
customer that they will not consider bids over $8,000,000, Robo Division purchases its materials
internally from the Cross Division. There would be no additional fixed costs for either the Robo
or Cross Divisions. Information regarding this project is as follows:
Cross Division Robo division
Variable costs $1,500,000 $4,800,000
transfer price 3,700,000 ___

If Robo Division submits a bid for $8,000,000, the amount of contribution margin recognized by
the Robo Division and GMT Industries, respectively, is
Correct Answer: C. $(500,000) and $1,700,000.

A. $(500,000) and $(2,000,000).


B. $3,200,000 and $(500,000).
C. $(500,000) and $1,700,000.
D. $3,200,000 and $1,700,000.

Explanation:
Answer (A) is incorrect.
The amount of $(2,000,000) for the company results from improperly subtracting the transfer
price.
Answer (B) is incorrect.
The amounts of $3,200,000 for Robo and $(500,000) for the company results from improperly
subtracting the transfer price from the company rather than from Robo, and from failing to
subtract the Cross Division's variable costs from the company.
Answer (C) is correct.
The contribution margins for the Robo Division and the company can be calculated as follows:

Robo GMT
Contract price $8,000,000 $8,000,000
Less: variable costs -4,800,000 -4,800,000
Less: variable costs __ -1,500,000
Less: transfer price -3,700,000 _________________
Contribution margin ($500,000) $1,700,000

Answer (D) is incorrect.


The amount of $3,200,000 for Robo results from failing to subtract the transfer price.

Question #52: A financial analyst has obtained the following data from financial statements:
Cash $200,000
Marketable securities 100,000
Accounts receivable, net 300,000
Inventories, net 480,000
Prepaid expenses 120,000
Total current assets $1,200,000

Accounts payable $250,000


Income taxes 50,000
Accrued liabilities 100,000
Current portion of long-term debt 200,000
Total current liabilities $600,000

In order to determine ability to pay current obligations, the financial analyst would calculate the
cash ratio as
Correct Answer: A. .50
A. .50
B. .80
C. 1.00
D. 1.20

Explanation:
Answer (A) is correct.
The cash ratio, a more conservative measure of liquidity than the quick ratio, is calculated as
follows:
Cash ratio = (Cash + Marketable securities) + Current liabilities
= ($200,000 + $100,000) + $600,000
= 0.5
Answer (B) is incorrect.
Improperly including only inventories in the numerator results in a ratio of .80.
Answer (C) is incorrect.
Improperly including accounts receivable in the numerator results in a ratio of 1.00.
Answer (D) is incorrect.
Improperly including accounts receivable and prepaid expenses in the numerator results in a ratio
of 1.20.

Question #53: Fact Pattern: Tosh Enterprises reported the following account information: (Refer
the image)
Question: What will happen to the ratios below if Tosh Enterprises uses cash to pay 25% of the
accounts payable?
Tosh Enterprises reported the following account information:

Accounts receivable $400,000


Accounts payable 260,000
Bonds payable, due in 10 years 600,000
Cash 200,000
Interest payable, due in 3 months 20,000
Inventory $800,000
Land 500,000
Short-term prepaid expense 80,000

Correct Answer: A. Increase (Current Ratio). (Quick Ratio) Increase All Possible Answers:
A. Increase (Current Ratio). (Quick Ratio) Increase
B. Decrease (Current Ratio). (Quick Ratio) Decrease
C. Increase (Current Ratio). (Quick Ratio) Decrease
D. Decrease (Current Ratio). (Quick Ratio) Increase

Explanation:
Answer (A) is correct.
Tosh's current ratio is 5.29 ($1,480,000 current assets ÷ $280,000 current liabilities), and its
quick ratio is 2.14 ($600,000 quick assets ÷ $280,000 current liabilities). Using cash to pay 25%
of the accounts payable decreases the numerator and denominator by $65,000 ($260,000
accounts payable x 25%). The new current ratio will be 6.58 ($1,415,000 $215,000), and the new
quick ratio will be 2.49 ($535,000+ $215,000). If a ratio exceeds 1.0, equal decreases in the
numerator and denominator increase the ratio.
Answer (B) is incorrect.
Given that both ratios initially exceeded 1.0, decreasing cash and accounts payable by equal
amounts will increase both ratios.
Answer (C) is incorrect.
Given that both ratios initially exceeded 1.0, decreasing cash and accounts payable by equal
amounts will increase both ratios.
Answer (D) is incorrect.
Given that both ratios initially exceeded 1.0, decreasing cash and accounts payable by equal
amounts will increase both ratios.

Question #54: Fad Pattern: Catfur Company has fixed costs of $300,000. It produces two
products, X and Y. Product X has a variable cost percentage equal to 60% of its $10 per unit
selling price. Product Y has a variable cost percentage equal to 70% of its $30 selling price. For
the past several years, unit sales of Product X were 40% of total unit sales. That ratio is not
expected to change.
Question: Assume that Catfur Company achieved its planned breakeven level of sales in dollars,
but the mix of products sold was one-to-one. Al I actual costs and unit selling prices equaled
budgeted amounts. What is the impact on profitability?
Correct Answer: B. The company earned a profit.
A. The company is operating at the breakeven point.
B. The company earned a profit.
C. The company sustained a loss.
D. Cannot be determined from the information given.
Explanation:
Answer (A) is incorrect.
The company is profitable due to the higher average contribution margin.
Answer (B) is correct.
The expected sales mix is 40% for Product X and 60% for Product Y. Weighted-average UCM
equals $7 [($10 - ($10 x 60%)] x 40%)+ [($30 - ($30 x 70%)] x 60%}. Weighted-average selling
price equals $22 [($10 x 40%) + ($30 x 60%)]. The weighted-average CMR therefore equals
0.3181818 ($7 ÷ $22), and the breakeven point in sales dollars equals $942,857 ($300,000 ÷
0.3181818). If actual sales were 50% Product X and 50% Product Y, weighted-average UCM
would equal $6.50 (($10 - ($10 x 60%)] x 50%) + ([$30 - ($30 x 70%)] x 50%}. Weighted-
average selling price would equal $20 [($10 x 50%) + ($30 x 50%)]. The weighted-average
CMR would therefore equal 0.325 ($6.50 ÷ $20), and the breakeven point in sales dollars would
equal $923,077 ($300,000 ÷ 0.325). Given that sales reached the budgeted breakeven point of
$942,857, Catfur must have made a profit of $19,780 ($942,857 - $923,077).
Answer (C) is incorrect.
The company is profitable due to the higher average contribution margin.
Answer (D) is incorrect.
The company is profitable due to the higher average contribution margin.

Question #55: (Please see the question below:)


Kim is thinking of organizing a fundraiser to support a local charity. She has planned to rent a
banquet hall and provide the guests with food, entertainment, and various party favors. She has
decided to charge $500 a person. After researching around town, Kim has discovered the
following costs:

Fixed costs
Rental fee of banquet hall $150,000
Advertising 50,000
Entertainment 4,000

Variable Costs Per guest


Food $12
Other miscellaneous costs 8

Correct Answer: D. 446 All Possible Answers:


A. 404
B. 425
C. 428
D. 446
Explanation:
Answer (A) is incorrect.
This number of guests is found by subtracting the $10,000 of desired income from the fixed
costs.
Answer (B) is incorrect.
This number of guests is the amount needed to break even.
Answer (C) is incorrect.
This number of guests is found by not subtracting the variable costs per person from the revenues
per person to get the contribution margin.
Answer (D) is correct.
The number of guests Kim must have to raise $10,000 can be calculated as follows:
Target unit sales = (Fixed costs + Target operating income) ÷ UCM
= {($150.000 + $50.000 + $4.000) + $10.000] ÷ ($500 - $12 - $8)
= $214.000 + $480
= 445.83

Question #56: A corporation's inventory expressed as a percentage of current assets increased


from 25% last July to 35% this July. The factor that is least likely to cause this increase is that
the corporation
Correct Answer: A. Is a seasonal company with traditionally higher activity in the summer
months. All Possible Answers:
A. Is a seasonal company with traditionally higher activity in the summer months.
B. Is beginning to experience high growth.
C. Has inventory that is becoming obsolete.
D. Used a material amount of cash from selling its short-term investments to purchase land.
Explanation:
Answer (A) is correct.
This statement is least likely to explain an increase in current assets from last July to this July. If
the corporation was a seasonal company with traditionally higher activity in the summer months,
it would budget similar amounts for each summer in expectation of the high activity. The sudden
increase in current assets for the following summer would not be explained by the fact that they
are a seasonal company.
Answer (B) is incorrect.
If the corporation was beginning to experience high growth, it would have to purchase more
inventory in order to meet the higher demand from the growth. This would cause the current
assets account to increase.
Answer (C) is incorrect.
Obsolete inventory refers to inventory held by a company that is at the end of its product life
cycle and has not seen any sales or usage for a set period of time. If the corporation has inventory
that is becoming obsolete, the inventory will be held by the company instead of being sold. This
would cause an increase in the current assets on the books.
Answer (D) is incorrect.
It can be assumed that the sale of short-term investments generated a gain, which caused a bigger
cash inflow than the outflow from the short-term investments. This means that the current assets
account increased. The fact that the company bought land with some of the cash is meant as a
distracter, as the question does not state how much of the cash was used to purchase the land.

Question #57: Question: Broomall's working capital at year end is


[Fact Pattern #16] Broomall Corporation has decided to include certain financial ratios in its
year-end annual report to shareholders. Selected information relating to its most recent fiscal
year is provided below.

Cash $10,000
Accounts receivable
Beginning of year 24,000
End of year 20,000
Prepaid expenses 8,000
Inventory:
Beginning of year 26,000
End of year 30,000
Available-for-sale securities
Historical cost 9,000
Fair value at year end 12,000
Accounts payable 15,000
Notes payable (due in 90 days) 25,000
Bonds payable (due in 10
years) 35,000
Net credit sales for sales 220,000
Cost of goods sold 140,000

Correct Answer: A. $40,000 All Possible Answers:


A. $40,000
B. $37,000
C. $28,000
D. $10,000
Explanation:
Answer (A) is correct.
Working capital consists of current assets net of current liabilities. Broomall's working capital at
year end can thus be calculated as follows:

Cash $10,000
Accounts receivable 20,000
Prepaid expenses 8,000
Inventory 30,000
Available-for-sale securities 12,000
Accounts payable -15,000
notes payable -25,000
Working capital $40,000
Answer (B) is incorrect.
The amount of $37,000 results from improperly including available-for-sale securities at cost
rather than at fair value.
Answer (C) is incorrect.
The amount of $28,000 results from improperly excluding available-for-sale securities from
current assets.
Answer (D) is incorrect.
The amount of $10,000 improperly treats only cash as working capital.

Question #58: Which of the outcomes represented in the following table would result from a
company's retirement of debt with excess cash?
Following Period's
Correct Answer: A. Increase (Total Assets Turnover Ratio), ( Times Interest Earned Ratio)
Increase
A. Increase (Total Assets Turnover Ratio), ( Times Interest Earned Ratio) Increase
B. Increase (Total Assets Turnover Ratio). ( Times Interest Earned Ratio) Decrease
C. Decrease (Total Assets Turnover Ratio). ( Times Interest Earned Ratio) Increase
D. Decrease (Total Assets Turnover Ratio). ( Times Interest Earned Ratio) Decrease
Explanation:
Answer (A) is correct.
Because total assets will decline without any impact on sales, the total assets turnover ratio (sales
+ total assets) will increase. In addition, a reduced debt level should cause a reduction in annual
interest payments, so the times interest earned ratio [(net income + interest + taxes) + interest]
should increase.
Answer (B) is incorrect.
The times interest earned ratio will increase and the total assets turnover ratio will increase.
Answer (C) is incorrect.
The times interest earned ratio will increase and the total assets turnover ratio will increase.
Answer (D) is incorrect.
The times interest earned ratio will increase and the total assets turnover ratio will increase.

Question #59: When using the net present value method for capital budgeting analysis, the
required rate of return is called all of the following except the
Correct Answer: A. Risk-free rate. All Possible Answers:
A. Risk-free rate.
B. Cost of capital.
C. Discount rate.
D. Cutoff rate.
Explanation:
Answer (A) is correct.
The rate used to discount future cash flows is sometimes called the cost of capital, the discount
rate, the cutoff rate, or the hurdle rate. A risk-free rate is the rate available on risk-free
investments such as government bonds. The risk-free rate is not equivalent to the cost of capital
because the latter must incorporate a risk premium.
Answer (B) is incorrect.
The rate used under the NPV method is the company's cost of capital.
Answer (C) is incorrect.
The NPV method discounts future cash flows to their present values.
Answer (D) is incorrect.
The cost of capital is often called a cutoff rate. Investments yielding less than the cost of capital
should not be made.
Question #60: A firm is interested in purchasing a $100 U.S. Treasury bill and was presented
with the following options:
Annual Discount Rate
Due date Yearly Rates
Option 1 180 days 6%
Option 2 360 days 3.50%
Option 3 120 days 8%
Option 4 240 days 4.50%

If the firm wishes to buy the Treasury bill at the lowest purchasing price, which option should be
chosen, assuming a 360-day year?
Correct Answer: B. Option 2.
A. Option 1.
B. Option 2.
C. Option 3.
D. Option 4.
Explanation:
Answer (A) is incorrect.
Option 1 has a purchase price of $97.00.
Answer (B) is correct.
To determine the amount of interest the lender will earn, the 3.5% discount rate is multiplied by
the face amount of the Treasury bill. The interest on this Treasury bill is $3.50 ($100 x 3.5% x 1
year). Thus, the purchase price is $96.50 ($100 - $3.5).
Answer (C) is incorrect.
Option 3 has a purchase price of $97.33.
Answer (D) is incorrect.
Option 4 has a purchase price of $97.00.

Question #61: Using the economic order quantity (EOQ) model, a decrease in which one of the
following variables would increase the EOQ?
Correct Answer: D. Carrying costs.
A. Annual sales.
B. Cost per order.
C. Safety stock level.
D. Carrying costs.
Explanation:
Click this link --> https://www.cmapass.com/wp-content/uploads/2016/11/Question-150.png

Question #62: The effective annual interest rate to the borrower of a $100.0001-year loan with a
stated rate of 7% and a 20% compensating balance is
Correct Answer: • 8.75%
A. 7.0%
B. 8.4%
C. 8.75%
D. 13.0%
Explanation:
Answer (A) is incorrect.
The rate of 7.0% results from failing to consider the effect of the compensating balance.
Answer (B) is incorrect.
The rate of 8.4% assumes a compensating balance percentage of 16.7%.
Answer (C) is correct.
The firm's effective rate on this loan can be calculated as follows:
Effective rate = Stated rate ÷ (1.0 - compensating balance %)
= 7% ÷ (100% - 20%)
= 7% ÷ 80%
= 8.75%
The amount of the loan is not needed to calculate the effective rate.
Answer (D) is incorrect.
The rate of 13.0% results from subtracting the stated rate from the compensating balance rate.

Question #63: A preferred stock is sold for $101 per share, has a face value of $100 per share.
Under writing fees of $5 per share, and annual dividends of $10 per share. If the tax rate is 40%,
the cost of funds (capital) for the preferred stock is
Correct Answer: D. 10.4% All Possible Answers:
A. 4.2%
B. 6.25%
C. 10.0%
D. 10.4%
Explanation:
Answer (A) is incorrect.
The figure of 4.2% results from improperly multiplying the dividends by the tax rate.
Answer (B) is incorrect.
The figure of 6.25% results from improperly multiplying the dividends by the tax rate.
Answer (C) is incorrect.
The figure of 10.0% results from improperly basing the calculation on par value funds received.
Answer (D) is correct.
The cost of capital for new preferred stock is equal to the dividend on the stock divided by the
net issue proceeds [$10 ÷ ($101 - $5) = 10.4%]. Because dividends on preferred stock are not
deductible for tax purposes, the income tax rate is irrelevant.

Question #64: A firm is being liquidated several months after its Chapter 7 bankruptcy filing.
The receiver has compiled the following information.

Assets $100,000
Common shares (at par) 22,000
Preferred shares (at par) 5,000
Secured bonds 50,000
Senior unsecured debt 27,000
Junior unsecured debt 18,000
Wage payable 7,000
Taxes owed 3,000
Credit from suppliers since filing 2,000
Court/trustee costs 1,500

Based on this information, what percentage of their claims will junior creditors receive?
Correct Answer: A. 53%
A. 53%
B. 64%
C. 81%
D. 100%
Explanation:
Answer (A) is correct.
Before the junior creditors are paid, secured bonds, court/trustee costs, credit from suppliers
since filing, wages payable, taxes owed, and senior creditors must all be paid (in that order).
After all of these debts are paid, the firm is left with $9,500 ($100,000 - $50,000 -$1,500 -
$2,000 - $7,000 - $3,000 - $27,000) that the junior creditor is entitled to. The junior creditors will
receive 53% of their claims ($9,500 ÷ $18,000). Please note that shareholders are last in line to
the claims.
Answer (B) is incorrect.
The credit from suppliers since filing claim must be paid before the claim for the unsecured debt.
The amount of 64% fails to take that claim into account.
Answer (C) is incorrect.
The credit from suppliers since filing claim and the taxes owed must be paid before the claim for
the unsecured debt. The amount of 81% fails to take both of these claims into account.
Answer (D) is incorrect.
The junior creditor has total claims of $18,000. Because there is only $9,500 left, (s)he will not
recover 100% of his/her claims.

Question #65: Which one of the following ethics-related actions by management is least
effective in encouraging acceptance by employees of an organization's code of ethics?
Correct Answer: A. Management appoints an ethics officer to monitor and report to management
on employee compliance.
A. Management appoints an ethics officer to monitor and report to management on
employee compliance.
B. Management follows ethical principles in decisions made on behalf of the organization.
C. Management keeps promises and commitments made to employees, customers, and
vendors.
D. Management supports employees in adhering to ethics standards.
Explanation:
Answer (A) is correct.
This approach does not involve employees at all and projects the image that employees need to
be supervised because they are likely to be unethical. This can discourage some employees.
Answer (B) is incorrect.
Management following ethical principles is an example of the "tone at the top' The 'tone at the
top" is one of the most effective ways of encouraging employees to behave ethically and accept
the organization's code of ethics.
Answer (C) is incorrect.
Management keeping promises and commitments is a sign that they behave ethically. The "tone
at the top" is one of the most effective ways of encouraging employees to behave ethically and
accept the organization's code of ethics.
Answer (D) is incorrect.
Management support can motivate employees because it shows that management is involved.

Question #66: A distributor is reviewing its inventory policy with respect to safety stocks of its
most popular product. Four safety stock levels were analyzed and annual stockout costs
estimated for each level.

Safety stock Stock out costs


1,000 units $3,000
1,250 units 2,000
1,500 units 1,000
2,000 units 0

The cost of this product is $20 per unit, holding costs are 4% per year, and the cost of short-term
funds is 10% per year. What is the optimal safety stock level?
Correct Answer: •C. 1,500 units.
A. 1,000 units.
B. 1,250 units.
C. 1,500 units.
D. 2,000 units.

Explanation:
Answer (A) is incorrect.
The amount of 1,000 units results in a higher cost than holding 1,500 units.
Answer (B) is incorrect.
The amount of 1,250 units results in a higher cost than holding 1,500 units.
Answer (C) is correct.
1000 1,250

purchase price $20 $20


times: safety stock level × 1,000 ×1,250
investment in safety stock $20,000 $25,000
times: cost of capital × 10% ×10%
interest cost of safety stock $2,000 $2,500
per-unit holding cost $0.80 $0.80
times: units held 1,000 ×1,250
total holding cost 800 1,000
Stock out cost 3,000 2,000
total cost of safety stock 5,800 $5,500

purchase price $20 $20


times: safety stock level ×1,500 ×2,000
Investment in safety stock $30,000 $40,000
times: cost of capital ×10% ×10%
interest cost of safety stock $3,000 $4,000
per-unit holding cost $0.80 $0.80
times: units held ×1,500 ×2,000
total holding cost 1,200 1,600
Stock out cost 1,000 0
total cost of safety stock $5,200 $5,600

The total cost ($5,200) is lowest when the safety stock is 1,500 units.
Answer (D) is incorrect.
The amount of 2,000 units results in a higher cost than holding 1,500 units.

Question #67: Fad Pattern: Jensen Corporation's board of directors met on June 3 and declared a
regular quarterly cash dividend of $.40 per share for a total value of $200.000. The dividend is
payable on June 24 to all stockholders of record as of June 17. Excerpts from the statement of
financial position for Jensen Corporation as of May 31 are presented as follows.

Cash $400,000
Accounts receivable (net) 800,000
Inventories 1,200,000
$2,400,00
Total current assets 0
Total current liabilities $1,000,00
0

Question: Jensen's working capital would be


Correct Answer: • Decreased by the dividend declaration and unchanged by the dividend
payment.
A. Unchanged by the dividend declaration and decreased by the dividend payment.
B. Decreased by the dividend declaration and increased by the dividend payment.
C. Unchanged by either the dividend declaration or the dividend payment
D. Decreased by the dividend declaration and unchanged by the dividend payment.

Explanation:
Answer (A) is incorrect.
The declaration of a dividend increases current liabilities and reduces working capital. The
subsequent payment has no effect on working capital.
Answer (B) is incorrect.
The declaration of a dividend increases current liabilities and reduces working capital. The
subsequent payment has no effect on working capital.
Answer (C) is incorrect.
The declaration of a dividend increases current liabilities and reduces working capital. The
subsequent payment has no effect on working capital.
Answer (D) is correct.
Working capital is defined as current assets minus current liabilities. The declaration of a
dividend increases current liabilities and thus reduces working capital. The subsequent payment
has no effect on working capital since current assets and current liabilities decrease by the same
amount.

Question #68: When none of the three fraud risk factors are present, an accountant
Correct Answer: C. Should not rule out the presence of fraud completely.
A. Can rule out the presence of fraud.
B. Should consider the likelihood of fraud to be high.
C. Should not rule out the presence of fraud completely.
D. Will likely search more diligently for fraud.
Explanation:
Answer (A) is incorrect.
The accountant would need more solid evidence to rule out the presence of fraud.
Answer (B) is incorrect.
When all three factors are present, fraud risk is high.
Answer (C) is correct.
Even when no factors are observed, an accountant cannot completely exclude the risk of fraud.
Factors may be present but hidden from the accountant.
Answer (D) is incorrect.
When fraud risk factors are present, the accountant will likely search more diligently for fraud.

Question #69: Question: Broomall's quick (acid test) ratio at year end is
[Fact Pattern #16] Broomall Corporation has decided to include certain financial ratios in its
year-end annual report to shareholders. Selected information relating to its most recent fiscal
year is provided below.

Cash $10,000
Accounts receivable:
Beginning of year 24,000
end of year 20,000
prepaid expenses 8,000
inventory:
Beginning of year 26,000
end of year 30,000
Available for sale securities:
historical cost 9,000
fair value at year end 12,000
Accounts payable 15,000
notes payable (due in 90 days) 25,000
Bonds payable (due in 10
years) 35,000
net credit sales for year 220,000
cost of goods sold 140,000

Correct Answer: D. 1.05 to 1.


A. 2.00 to 1.
B. 1.925 to 1.
C. 1.80 to 1.
D. 1.05 to 1.

Explanation:
Answer (A) is incorrect.
A ratio of 2.00 results from improperly including all current assets in the numerator.
Answer (B) is incorrect.
A ratio of 1.925 results from improperly including accounts receivable (beginning balance),
prepaid expenses, inventory, and available-for-sale securities (at cost) in the numerator. 1.925
can be calculated as follows:
A
( $ 10,000 cash+ $ 24,000 beg . bal.+ $ 8,000 proposal+ $ 26,000 inventory beg . bal. ¿ + $ 9,000 AFS at cost )
R
A
($ 15,000 + $ 25,000 Bonds payable)
P
Answer (C) is incorrect. A ratio of 1.80 results from improperly including accounts receivable
(beginning balance) and inventory in the numerator. 1.80 can be calculated as follows:
A
( $ 10,000 cash+ $ 24,000 beg . bal.+ $ 26,000 inventory beg . bal. ¿ + $ 12,000 bonds payable)
r
A
($ 15,000 + $ 25,000 Bonds payable)
p
Answer (D) is correct.
The quick (acid test) ratio consists of the quick assets (cash, marketable securities, and accounts
receivable) divided by current liabilities. Broomall's quick ratio at year end is thus 1.05
[($10,000 + $12,000 + $20,000) - ($15,000 + $25,000)].

Question #70: If a bond is rated below BBB, it is called


Correct Answer: C. A junk bond. All Possible Answers:
A. A zero-coupon bond.
B. An investment grade bond.
C. A junk bond.
D. An income bond.
Explanation:
Answer (A) is incorrect.
A zero-coupon bond pays no interest and is sold at a discount.
Answer (B) is incorrect.
An investment grade bond is rated A or BBB.
Answer (C) is correct.
AAA and AA are Standard & Poor's highest ratings. They signify the highest quality. Bonds
rated A and BBB are investment grade. Bonds rated below BBB are speculative high-yield or
low-grade bonds (junk bonds).
Answer (D) is incorrect.
An income bond pays interest only if the issuer earns income sufficient to pay the interest.

Question #71: Fad Pattern: For one of its divisions, Buona Fortuna Company has fixed costs of
$300,000 and a variable-cost percentage equal to 60% of its $10 per unit selling price. It would
like to earn a pre-tax income of $90,000 per year from the division.
Question: How many units will Buona Fortuna have to sell to earn a pre-tax income of $90,000
per year?
Correct Answer: D. 97,500 units.
A. 65,000 units.
B. 75,000 units.
C. 77,250 units.
D. 97,500 units.
Explanation:
Answer (A) is incorrect.
Using the unit variable cost in the denominator results in 65,000 units.
Answer (B) is incorrect.
This number of units is the breakeven point.
Answer (C) is incorrect.
This number of units would yield a profit less than the desired $90,000.
Answer (D) is correct.
Buona Fortuna's unit contribution margin is $4 ($10 unit price - $6 unit variable cost). By
treating desired profit as an additional fixed cost, the target unit sales can be calculated as
follows:
Target unit sales = (Fixed costs - Target operating income) ÷ UCM
= ($300,000 + $90,000) ÷ $4
= 97,500

Question #72: An entity is considering implementing a lockbox collection system at a cost of


$80.000 per year. Annual sales are $90 million, and the lockbox system will reduce collection
time by 3 days. If the entity can invest funds at 8%, should it use the lockbox system? Assume a
360-day year.
Correct Answer: C. No. producing a loss of $20.000 per year. All Possible Answers:
A. Yes, producing savings of $140,000 per year.
B. Yes, producing savings of $60,000 per year.
C. No. producing a loss of $20,000 per year.
D. No. producing a loss of $60,000 per year.
Explanation:
Answer (A) is incorrect.
This figure results from improperly adding, rather than subtracting, the cost of the lockbox.
Answer (B) is incorrect.
This figure results from failing to subtract the cost of the lockbox.
Answer (C) is correct.
The annual benefit (loss) from using the lockbox system is the excess (deficit) of interest earned
on the early deposits over (under) the cost of the service. If the plan is adopted, average cash
balance will increase by $750,000 00.000.000 x (3 days ÷ 360 days)].
Benefit (loss) = Interest earned - Cost
= ($750,000 x 8%) - $80,000
= $60,000 - $80,000
= $(20,000)
Answer (D) is incorrect.
This figure is the interest revenue that would be earned on the early deposits.
Question #73: A company manufactures a variety of shoes and has received a special one-time-
only order directly from a wholesaler. The company has sufficient idle capacity to accept the
special order to manufacture 15,000 pairs of sneakers at a price of $7.50 per pair. The company's
normal selling price is $11.50 per pair of sneakers. Variable manufacturing costs are $5.00 per
pair and fixed manufacturing costs are $3.00 a pair. The company's variable selling expense for
its normal line of sneakers is $1.00 per pair. What would the effect on the company's operating
income be if the company accepted the special order?
Correct Answer: C. Increase by $37.500.
A. Decrease by $60,000.
B. Increase by $22,500.
C. Increase by $37,500.
D. Increase by $52,500.
Explanation:
Answer (A) is incorrect.
A $60,000 decrease results from using the difference between the selling price for the special
order and the normal selling price.
Answer (B) is incorrect.
A $22,500 increase results from improperly subtracting the variable selling expense.
Answer (C) is correct.
The per-unit contribution margin earned from this special order will be the difference between
the selling price and the company's variable cost of manufacturing ($7.50 - $5.00 = $2.50).
Variable selling expenses usually arise from commission earned by the sales staff. Since we are
told that the order comes directly from the wholesaler, we can assume that no commission would
be earned on the order. The net effect on operating income is therefore an increase of $37,500
(15,000 x $2.50).
Answer (D) is incorrect.
A $52,500 increase results from using the normal selling price minus variable and fixed
manufacturing costs to arrive at contribution margin.

Question #74: Depreciation is incorporated explicitly in the discounted cash flow analysis of an
investment proposal because it
Correct Answer: C. Reduces the cash outlay for income taxes. All Possible Answers:
A. Is a cost of operations that cannot be avoided.
B. Is a cash inflow.
C. Reduces the cash outlay for income taxes.
D. Represents the initial cash outflow spread over the life of the investment.
Explanation:
Answer (A) is incorrect.
Depreciation is not a cost of operations in the capital budgeting model. Also, depreciation can be
avoided by not making investments.
Answer (B) is incorrect.
Depreciation is an allocation of historical cost and as such is not a cash inflow, but it may reduce
cash outflows for taxes.
Answer (C) is correct.
Depreciation is a noncash expense that is deductible for federal income tax purposes. Hence, it
directly reduces the cash outlay for income taxes and is explicitly incorporated in the capital
budgeting model.
Answer (D) is incorrect.
Periodic depreciation is determined by spreading the depreciation base. i.e., the cost of the asset
minus salvage value, not the initial cash outflow, over the life of the investment.

Question #75: The following financial information is given.

Year 1 Year2
Book value of assets $18,000 $26,000
Market value of equity 18,000 60,000
12 months ended 12 months ended
Year 1 Year2
Sales $1,000 $1,300
Cost of goods sold 500 700
Operating income 500 600
Depreciation expense 200 200
Interest expense 100 100
Pretax income 200 300
Income tax expense 80 120
Net income $120 $180

Using a common-size income statement, did operating income and net income increase or
decrease?
Correct Answer: C. Decreased (Operating income). (Net income) Increased
A. Increased (Operating income). (Net income) Increased
B. Increased (Operating income). (Net income) Decreased
C. Decreased (Operating income). (Net income) Increased
D. Decreased (Operating income). (Net income) Decreased
Explanation:
Answer (A) is incorrect.
Although total operating income increased, it decreased when restated in common size.
Answer (B) is incorrect.
Although total operating income increased, it decreased when restated in common size.
Furthermore, both actual net income and common-size net income increased.
Answer (C) is correct.
A common-size income statement restates line items as a percentage of net sales. The table
below provides these percentages.

Year 1 Year2
Sales 100.0% 100.00%
Cost of goods sold 50,00% 53.85%
Operating income 50.00% 46.15%
Depreciation expense 20.00% 15.38%
Interest expense 10.00% 7.69%
Pretax income 20.00% 23.08%
Income tax expense 8.00% 9.23%
Net income 12.00% 13.85%

The common-size operating income decreased, while net income increased.


Answer (D) is incorrect.
Both actual net income and common-size net income increased.

Question #76: Fad Pattern: The Frame Supply Company has just acquired a large account and
needs to increase its working capital by $100.000. The controller of the company has identified
the four sources of funds given below.
1. Pay a factor to buy the company's receivables, which average $125.000 per month and have an
average collection period of 30 days. The factor will advance up to 80% of the face value of
receivables at 10% and charge a fee of 2% on all receivables purchased. The controller estimates
that the firm would save $24.000 in collection expenses over the year. Assume the fee and
interest are not deductible in advance.
2. Borrow $110.000 from a bank at 12% interest. A 9% compensating balance would be
required.
3. Issue $110.000 of 6-month commercial paper to net $100.000. (New paper would be issued
every 6 months.)
4. Borrow $125.000from a bank on a discount basis at 20%. No compensating balance would be
required. Assume a 360-day year in all of your calculations.
Question: The cost of Alternative 3 to Frame Supply Company is
Correct Answer: D. 20.0%
A. 9.1%
B. 10.0%
C. 18.2%
D. 20.0%
Explanation:
Answer (A) is incorrect.
This percentage is the 6-month rate based on the face amount of the paper.
Answer (B) is incorrect.
The rate for 6 months is 10.0%.
Answer (C) is incorrect.
This percentage is based on the face amount of the commercial paper.
Answer (D) is correct.
By issuing commercial paper, the company will receive $100,000 and repay $110,000 every 6
months. Thus, for the use of $100,000 in funds, the company pays $10,000 in interest each 6-
month period, or a total of $20,000 per year. The annual percentage rate can therefore be
calculated as follows:
Effective rate = Interest expense ÷ Usable funds
= $20,000 ÷ $100,000
= 20.0%

Question #77: A firm can mitigate the risk of financial loss from the possible on-the-job injury of
one of its employees through
Correct Answer: D. Liability insurance.
A. Hazard insurance.
B. Workers' compensation insurance.
C. Key employee insurance.
D. Liability insurance.
Explanation:
Answer (A) is incorrect.
Hazard insurance is the same as homeowner's or automobile driver's insurance. It protects the
organization against damage caused to its facilities by accident or natural disaster.
Answer (B) is incorrect.
Workers' compensation insurance benefits the injured worker, not the organization.
Answer (C) is incorrect.
Key employee insurance benefits the organization only in case of the death of a critical member
of upper management.
Answer (D) is correct.
Liability insurance provides an organization with financial protection against damage caused to
consumers by faulty products or injury to persons suffered on the organization's premises.

Question #78: A firm is given terms of 2/10, net 45 by its suppliers. If the firm forgoes the cash
discount and instead pays the suppliers 5 days after the net due date with no penalty, what is the
annual interest rate cost (using a 360-day year)?
Correct Answer: B. 18.4%
A. 18.0%
B. 18.4%
C. 21.0%
D. 24.5%
Explanation:
Answer (A) is incorrect.
The rate of 18.0% results from failing to divide the discount rate by (1.0 - discount rate).
Answer (B) is correct.
The annualized cost of not taking a discount can be calculated with the following formula:
(Discount%/ (100 % - Discount%)) × (Days in year / (Total payment period - Discount Period))
Cost of not taking discount = [2% ÷ (100% - 2%)] x [360 days ÷ (50 days - 10 days)]
= (2% ÷ 98%) x (360 days ÷ 40 days)
= 2.04% x 9
= 18.36%
Answer (C) is incorrect.
The rate of 21.0% results from using the agreed-to discount period rather than the self-extended
discount period.
Answer (D) is incorrect.
The rate of 24.5% results from using a 30-day discount period.

Question #79: A retailer buys virtually all of its merchandise from manufacturers in a country
experiencing significant inflation. The retailer is considering changing its method of inventory
costing from first-in, first-out (FIFO) to last-in, first-out (LIFO). What effect would the change
from FIFO to LIFO have on the retailer's current ratio and inventory turnover ratio?
Correct Answer: C. The current ratio would decrease but the inventory turnover ratio would
increase.
A. Both the current ratio and the inventory turnover ratio would increase.
B. The current ratio would increase but the inventory turnover ratio would decrease.
C. The current ratio would decrease but the inventory turnover ratio would increase.
D. Both the current ratio and the inventory turnover ratio would decrease.
Explanation:
Answer (A) is incorrect.
The current ratio would decrease due to the lower inventory value under LIFO.
Answer (B) is incorrect.
The current ratio would decrease due to the lower inventory value under LIFO.
Answer (C) is correct.
During periods of high inflation, manufacturers and retailers often switch to LIFO inventory
valuation as a tax postponement tool. The higher costs attaching to more recent inventory pass
into cost of goods sold, reducing net income and tax liability. Since cost of goods sold is the
numerator of the inventory turnover ratio, turnover will increase. Also, inventory will be lower
under LIFO, which reduces the current ratio and increases the turnover ratio.
Answer (D) is incorrect.
The inventory turnover would increase due to higher cost of goods sold and lower inventory.
Question #80: The president of a company is establishing performance goals for each of the
company's manufacturing plants. The data below represent prior-year results for one of the
plants.

Revenues $400,000
Variable costs 100,000
Fixed costs 200,000
1,000,00
Average assets 0
Average liabilities 200,000

The plant's return on assets is


Correct Answer: D. 10.0%
A. 37.5%
B. 30.0%
C. 12.5%
D. 10.0%
Explanation:
Answer (A) is incorrect.
Return on assets is calculated as net income over average total assets. Net income is equal to
revenues less variable and fixed expenses. This answer choice fails to subtract fixed costs when
calculating net income. In addition. it incorrectly subtracts average liabilities from average assets
to calculate the denominator. Please note that the average assets are given and do not need to be
adjusted for liabilities.
Answer (B) is incorrect.
Return on assets is calculated as net income over average total assets. Net income is equal to
revenues less variable and fixed expenses. This answer choice fails to subtract fixed costs when
calculating net income.
Answer (C) is incorrect.
Return on assets is calculated as net income over average total assets. This answer choice
incorrectly subtracts average liabilities from average assets to calculate the denominator. Please
note that the average assets are given and do not need to be adjusted for liabilities.
Answer (D) is correct.
Return on assets is calculated as net income over average total assets. Net income is equal to
$100,000 ($400,000 revenues - $100,000 variable costs - $200,000 fixed costs). Average total
assets is stated as $1,000,000. Thus, return on assets is equal to 10.0% ($100,000 ÷ $1,000,000).
Question #81: I MA's Statement of Ethical Professional Practice includes a competence standard,
which requires an IMA member to
Correct Answer: B. Develop his or her professional proficiency on a continual basis.
A. Report information, whether favorable or unfavorable.
B. Develop his or her professional proficiency on a continual basis.
C. Discuss ethical conflicts and possible courses of action with an unbiased counselor.
D. Discuss, with subordinates, their responsibilities regarding the disclosure or information
about the firm.
Explanation:
Answer (A) is incorrect.
The credibility standard requires an IMA member to "communicate information fairly and
objectively."
Answer (B) is correct.
One of the responsibilities of an IMA member under the competence standard is to "maintain an
appropriate level of professional expertise by continually developing knowledge and skills:
Answer (C) is incorrect.
One of the suggestions from the "Resolution of Ethical Conflict" section is to-clarify relevant
ethical issues by initiating a confidential discussion with an IMA Ethics Counselor or other
impartial advisor to obtain a better understanding of possible courses of action.
Answer (D) is incorrect.
The confidentiality standard requires an IMA member to “inform all relevant parties regarding
appropriate use of confidential information. Monitor subordinates' activities to ensure
compliance.”

Question #82: Fad Pattern: Excerpts from the statement of financial position for Markham
Corporation as of April 30 of the current year are presented as follows:
Question: Markham's current ratio would be

Accounts receivable (net) 1,640,000


Inventories 2,945,000
total current assets $5,310,000
Accounts payable $1,236,000
Accrued liabilities 831,000
total current liabilities $ 2,067,000

The board of directors of Markhammet on May 5 of the current year and declared a quarterly
cash dividend in the amount of $800,000 ($.50 per share). The dividend was paid on May 28 of
the current year to shareholders of record as of May 15 of the current year. Assume that the only
transactions that affected Markham during May of the current year were the dividend
transactions and that the closing entries have been made.
Question: Markham's current ratio would be
Correct Answer: A. Decreased by the dividend declaration and increased by the dividend
payment.
A. Decreased by the dividend declaration and increased by the dividend payment.
B. Increased by the dividend declaration and unchanged by the dividend payment
C. Unchanged by either the dividend declaration or the dividend payment
D. Unchanged by the dividend declaration and decreased by the dividend payment.
Explanation:
Answer (A) is correct.
The current ratio equals current assets divided by current liabilities. The declaration of a
dividend results in a decrease in retained earnings and an increase in current liabilities. The effect
is to decrease the current ratio because current liabilities are increased at the time of the
declaration without a change in current assets. The April 30 current ratio is 2.57 ($5,310,000 ÷
$2,067,000). Following the declaration of an $800,000 dividend, the current ratio is 1.85
($5,310,000 ÷ $2,867,000). The subsequent payment of the dividend will increase the ratio
because the ratio is greater than 1.0 and both current assets and current liabilities will decline by
the same amount. The current ratio after the payment is 2.18 ($4,510,000 ÷ $2,067,000).
Answer (B) is incorrect.
The dividend declaration decreases the current ratio, and the payment increases the ratio.
Answer (C) is incorrect.
The dividend declaration decreases the current ratio, and the payment increases the ratio.
Answer (D) is incorrect.
The dividend declaration decreases the current ratio, and the payment increases the ratio.

Question #83: A company sells its single product for $30 per unit. The contribution margin ratio
is 45%, and fixed costs are $10,000 per month. Sales were 3,000 units in April and 4,000 units in
May. How much greater is the May income than the April income?
Correct Answer: B. $13,500
A. $10,000
B. $13,500
C. $16,500
D. $30,000
Explanation:
Answer (A) is incorrect.
The amount of fixed costs is $10,000.
Answer (B) is correct.
The contribution margin ratio reveals that 45% of the sales price of each product is contribution
margin ($30 x 45% = $13.50). Thus, the excess of May income over April is $13,500 (1,000 x
$13.50). Fixed costs do not vary between April and May.
Answer (C) is incorrect.
The amount of additional variable costs is $16,500.
Answer (D) is incorrect.
The difference in sales revenue is $30,000.

Question #84: All of the following are affected when merchandise is purchased on credit except
Correct Answer: B. Net working capital.
A. Total current assets.
B. Net working capital.
C. Total current liabilities.
D. Current ratio.
Explanation:
Answer (A) is incorrect.
Current assets are increased by the amount of inventory purchased.
Answer (B) is correct.
Working capital is the net of current assets and current liabilities. When merchandise is
purchased on credit, inventory (a current asset) is increased, and accounts payable (a current
liability) is increased by the same amount. Thus, no net change in working capital results.
Answer (C) is incorrect.
Current liabilities are increased by the amount of the credit purchase.
Answer (D) is incorrect.
The numerator (current assets) and the denominator (current liabilities) of the current ratio are
increased by the same amount.

Question #85: A company has had the following financial results for the last four years.

Year 1 Year 2 Year 3 Year 4


$1,250,00 $1,300,00 $1,359,00 $1,400,00
Sales 0 0 0 0
Cost of goods sold 750,000 785,000 825,000 850,000
Gross profit 500,000 515,000 534,000 550,000
Inflation factor 1.00 1.03 1.07 1.10

The company has analyzed these results using vertical common-size analysis to determine
trends. The performance of the company can best be characterized by which one of the following
statements?
Correct Answer: A. The common-size gross profit percentage has decreased as a result of an
increasing common size trend in cost of goods sold.
A. The common-size gross profit percentage has decreased as a result of an increasing
common size trend in cost of goods sold.
B. The common-size trend in sales is increasing and is resulting in an increasing trend in the
common-size gross profit margin.
C. The common-size trend in cost of goods sold is decreasing which is resulting in an
increasing trend in the common-size gross profit margin.
D. The increased trend in the common-size gross profit percentage is the result of both the
increasing trend in sales and the decreasing trend in cost of goods sold.

Explanation:
Answer (A) is correct.
The common-size gross profit percentage has decreased as a result of an increasing common-size
trend in cost of goods sold as shown below.

Year 1 Year 2 Year 3 Year 4


Sales 100% 100% 100% 100%
Cost of goods sold (÷Sales) 60.0% 60.3% 60.7% 60.7%
Gross profit (÷Sales) 40.0% 39.6% 39.2% 39.2%
Answer (B) is incorrect.
In a common-size income statement, sales revenue is used as a base so its percentage is always
100% on the common-size statement and does not increase.
Answer (C) is incorrect.
The common-size trend in cost of goods sold is increasing, not decreasing. As a result, the gross
profit as a percentage of sales is decreasing.
Answer (D) is incorrect.
In a common-size income statement, sales revenue is used as a base, so its percentage is always
100% on the common-size statement and does not increase. As a percentage of sales, cost of
goods sold is increasing. Therefore, the common-size gross profit percentage decreases over the
years.

Question #86: Which of the following is not an aspect of the Foreign Corrupt Practices Act of
1977?
Correct Answer: C. It requires the establishment of independent audit committees. All Possible
Answers:
A. It subjects management to fines and imprisonment.
B. It prohibits bribes to foreign officials.
C. It requires the establishment of independent audit committees.
D. It requires an internal control system to be developed and maintained.
Explanation:
Answer (A) is incorrect.
This is a provision of the Act.
Answer (B) is incorrect.
This is a provision of the Act.
Answer (C) is correct.
The Foreign Corrupt Practices Act of 1977 prohibits bribes to foreign officials and requires firms
to have adequate systems of internal control. Violation of the act subjects individual managers to
fines and/or imprisonment. The Act does not specifically require the establishment of audit
committees, but many firms have established audit committees as one means of dealing with the
internal control provisions of the act.
Answer (D) is incorrect.
This is a provision of the Act.

Question #87: Question: Tilghman Company's current liabilities at December 31 equal


(Fact Pattern #8] The selected data pertain to Tilghman Company at December 31:

Quick assets $208,000


Acid test ratio 2.6 to 1
Current ratio 3.5 to 1
Net sales for the year $1,800,000
Cost of sales for the year $990,000
Average total assets for the year $1,200,000

Correct Answer: B. $80,000 All Possible Answers:


A. $59,429
B. $80,000
C. $134,857
D. $187,200

Explanation:
Answer (A) is incorrect.
The current liabilities at year end are determined using the quick assets total and the acid test
ratio: Current liabilities equals the quick assets divided by the acid test ratio.
Answer (B) is correct.
Current liabilities can be calculated using the following relationship:
Acid test ratio = Quick assets + Current liabilities
2.6= $208,000 ÷ Current liabilities
Current liabilities x 2.6 =$208,000
Current liabilities = $208,000 ÷ 2.6
= $80,000
Answer (C) is incorrect.
The current liabilities at year end are determined using the quick assets divided by the acid test
ratio.
Answer (D) is incorrect.
The current liabilities at year end are determined using the quick assets divided by the acid test
ratio, total and the acid test ratio: Current total and the acid test ratio: Current liabilities equals
the quick liabilities equals the quick Assets divided by the acid test ratio.

Question #88: If an IMA member discovers unethical conduct in his or her organization and fails
to act. (s)he will be in violation of which of IMAs ethical standard(s)?
Correct Answer: D. All of the answers are correct.
A. "Refrain from engaging in any conduct that would prejudice carrying out duties
correctly?
B. "Communicate information fairly and objectively:
C. "Disclose all relevant information that could reasonably be expected to influence an
intended user's understanding of reporting analyses or recommendations?
D. All of the answers are correct.
Explanation:
Answer (A) is incorrect.
Each standard is violated by an IMA member who fails to act upon discovering unethical
conduct.
Answer (B) is incorrect.
Each standard is violated by an IMA member who fails to act upon discovering unethical
conduct.
Answer (C) is incorrect.
Each standard is violated by an IMA member who fails to act upon discovering unethical
conduct.
Answer (D) is correct.
An IMA member displays his or her competence and credibility and maintains integrity by
taking the appropriate action within the organization to resolve an ethical problem. All of these
activities should be a part of an IMA member's normal job processes.

Question #89: As a company becomes more conservative with respect to working capital policy,
it would tend to have a(n)
Correct Answer: D. Increase in the ratio of current assets to noncurrent assets. All Possible
Answers:
A. Increase in the ratio of current liabilities to noncurrent liabilities.
B. Decrease in the operating cycle.
C. Decrease in the quick ratio.
D. Increase in the ratio of current assets to noncurrent assets.
Explanation:
Answer (A) is incorrect.
An increase in current liabilities relative to noncurrent liabilities would increase liquidity risk.
Answer (B) is incorrect.
A decrease in the normal operating cycle permits a lower level of working capital. If assets can
be converted to cash more quickly, current assets can be reduced.
Answer (C) is incorrect.
A decrease in the quick ratio signifies that quick assets (cash. receivables. and marketable
securities) are decreasing relative to current liabilities.
Answer (D) is correct.
A conservative working capital policy results in an increase in working capital (current assets -
current liabilities). It is typified by a reduction in liquidity risk. Increasing the current ratio,
whether by decreasing current liabilities or increasing current assets, minimizes the risk that the
company will not be able to meet its obligations as they fall due. Thus, an increasing ratio of
current to noncurrent assets means that a company is forgoing the potentially higher returns on
long-term assets in order to guard against short-term cash flow problems.

Question #90: Question: If the selling price is $14 per unit, the breakeven point in units (rounded
to the nearest hundred) for surge protectors is
Bruell Electronics Co. is developing a new product, surge protectors for high-voltage electrical
flows. The cost information below relates to the product:

Unit costs
Direct materials $3.25
Direct labor 4.00
Distribution .75

The company will also be absorbing $120,000 of additional fixed costs associated with this new
product. A corporate fixed charge of S20,000 currently absorbed by other products will be
allocated to this new product.
Correct Answer: D. 20.000 units. All Possible Answers:
A. 8,500 units.
B. 10,000 units.
C. 15,000 units.
D. 20,000 units.
Explanation:
Answer (A) is incorrect.
A breakeven point of 8,500 units ignores variable costs.
Answer (B) is incorrect.
The breakeven point is 20,000 units when the contribution margin is $6 per unit.
Answer (C) is incorrect.
This number of units equals fixed costs divided by unit variable cost.
Answer (D) is correct.
The breakeven point in units fora new product equals total additional fixed costs divided by the
unit contribution margin. Unit variable costs total $8 ($3.25 + $4.00+ $.75). Thus. UCM is $6
($14 unit selling price - $8 unit variable cost), and the breakeven point is 20,000 units ($120,000
÷ $6).

Question #91: Integrity is an ethical requirement for all IMA members. One aspect of integrity
requires
Correct Answer: B. Avoidance of conflict of interest.
A. Performance of professional duties in accordance with relevant laws.
B. Avoidance of conflict of interest.
C. Refraining from using confidential information for unethical or illegal advantage.
D. Maintenance of an appropriate level of professional expertise.
Explanation:
Answer (A) is incorrect.
Performance of professional duties in accordance with relevant laws states an aspect of the
competence requirement.
Answer (B) is correct.
According to IMA's Statement of Ethical Professional Practice. IMA members must "mitigate
actual conflicts of interest. Regularly communicate with business associates to avoid apparent
conflicts of interest. Advise all parties of any potential conflicts:
Answer (C) is incorrect.
It states an aspect of the confidentiality requirement.
Answer (D) is incorrect.
Maintenance of an appropriate level of professional expertise states an aspect of the competence
requirement.

Question #92: Fad Pattern: Depoole Company is a manufacturer of industrial products that uses a
calendar year for financial reporting purposes. Assume that total quick assets exceeded total
current liabilities both before and after the transaction described. Further assume that Depoole
has positive profits during the year and a credit balance throughout the year in its retained
earnings account.
Question: Depoole's purchase of raw materials for $85,000 on open account will
Correct Answer: B. Decrease the current ratio.
A. Increase the current ratio.
B. Decrease the current ratio.
C. Increase net working capital.
D. Decrease net working capital.
Explanation:
Answer (A) is incorrect.
The current ratio is decreased.
Answer (B) is correct.
The purchase increases both the numerator and denominator of the current ratio by adding
inventory to the numerator and payables to the denominator. Because the ratio before the
purchase was greater than 1, the ratio is decreased.
Answer (C) is incorrect.
The purchase of raw materials on account has no effect on working capital (current assets and
current liabilities change by the same amount).
Answer (D) is incorrect.
The purchase of raw materials on account has no effect on working capital (current assets and
current liabilities change by the same amount).
Question #93: Fad Pattern: Stennet Company is considering two mutually exclusive projects.
The net present value (NPV) profiles of the two projects are as follows:
Discount Rate Net Present Value $(000)
(percent Project Project
) A B
0 $2,220 $1,240
10 681 507
12 495 411
14 335 327
16 197 252
18 77 186
20 (26) 128
22 (115) 76
24 (193) 30
26 (260) (11)
28 (318) (47)

Question: The approximate internal rates of return for Projects A and B. respectively, are
Correct Answer: C. 19.5% and 25.5%.
A. 0% and 0%.
B. 19.0% and 21.5%.
C. 19.5% and 25.5%.
D. 20.5% and 26.5%.
Explanation:
Answer (A) is incorrect.
Zero is the net present value, not the discount rate, associated with the internal rate of return.
Answer (B) is incorrect.
The internal rate of return for Project B is between 24% and 26%.
Answer (C) is correct.
A project's internal rate of return is the discount rate at which the net present value of its cash
flows equals zero. For Project A. this is somewhere between 18% and 20%, and for Project B. it
is between 24% and 26%.
Answer (D) is incorrect.
The internal rate of return of Project A is less than 20% and that for Project B is less than 26%.
Question #94: The requirement of the Foreign Corrupt Practices Act of 1977 to devise and
maintain adequate internal control is assigned in the act to the
Correct Answer: D. Company as a whole with no designation of specific persons or positions.
A. Chief financial officer.
B. Board of directors.
C. Director of internal auditing.
D. Company as a whole with no designation of specific persons or positions.
Explanation:
Answer (A) is incorrect.
Compliance with the FCPA is not the specific responsibility of the chief financial officer.
Answer (B) is incorrect.
Compliance with the FCPA is not the specific responsibility of the board of directors.
Answer (C) is incorrect.
Compliance with the FCPA is not the specific responsibility of the director of internal auditing.
Answer (D) is correct.
The accounting requirements apply to all public companies that must register under the
Securities Exchange Act of 1934. The responsibility is thus placed on companies, not individuals

Question #95: Preferred and common stock differ in that


Correct Answer: C. Preferred stock has a higher priority than common stock with regard to
earnings and assets in the event of bankruptcy. All Possible Answers:
A. Failure to pay dividends on common stock will not force the firm into bankruptcy, while
failure to pay dividends on preferred stock will force the firm into bankruptcy.
B. Common stock dividends are a fixed amount, while preferred stock dividends are not.
C. Preferred stock has a higher priority than common stock with regard to earnings and
assets in the event of bankruptcy.
D. Preferred stock dividends are deductible as an expense for tax purposes, while common
stock dividends are not.
Explanation:
Answer (A) is incorrect.
Failure to pay dividends will not force the firm into bankruptcy, whether the dividends are for
common or preferred stock. Only failure to pay interest will force the firm into bankruptcy.
Answer (B) is incorrect.
Preferred dividends are fixed.
Answer (C) is correct.
In the event of bankruptcy, the claims of preferred shareholders must be satisfied before common
shareholders receive anything. The interests of common shareholders are secondary to those of
all other claimants.
Answer (D) is incorrect.
Neither common nor preferred dividends are tax deductible.

Question #96: When evaluating a capital budgeting project, a company's treasurer wants to know
how changes in operating income and the number of years in the projects useful life will affect
its breakeven internal rate of return. The treasurer is most likely to use
Correct Answer: B. Sensitivity analysis.
A. Scenario analysis.
B. Sensitivity analysis.
C. Monte Carlo simulation.
D. Learning curve analysis.
Explanation:
Answer (A) is incorrect.
Scenario analysis is not used for quantitative calculations.
Answer (B) is correct.
Forecasts of many calculated NPVs under various assumptions are compared to see how
sensitive NPV is to changing conditions. Changing or relaxing the assumptions about a certain
variable or group of variables may drastically alter the NPV. Thus, the asset may appear to be
much riskier than was originally predicted. In summary, sensitivity analysis is simply an iterative
process of recalculated returns based on changing assumptions.
Answer (C) is incorrect.
Monte Carlo simulation is used to account for an element of randomness. The changes in
operating income and life of the project in this situation are under the control of the person
running the simulation and thus are not random.
Answer (D) is incorrect.
Learning curve analysis is used to anticipate the increased rate at which people perform tasks as
they gain experience: it is not an appropriate tool for capital budgeting.

Question #97: In an insourcing vs. outsourcing situation, which of the following qualitative
factors is usually considered?
Correct Answer: D. All of the answers are correct.
A. Special technology.
B. Skilled labor.
C. Special materials requirements.
D. All of the answers are correct.
Explanation:
Answer (A) is incorrect.
Special technology, skilled labor, and special materials requirements are all considered in an
insourcing vs. outsourcing situation.
Answer (B) is incorrect.
Special technology, skilled labor, and special materials requirements are all considered in an
insourcing vs. outsourcing situation.
Answer (C) is incorrect.
Special technology, skilled labor, and special materials requirements are all considered in an
insourcing vs. outsourcing situation.
Answer (D) is correct.
Special technology may be available either within or outside the firm that relates to the particular
product. The firm may possess necessary skilled labor or the supplier may. Special materials
requirements may also affect the decision process because one supplier may have monopolized a
key component. Another factor to be considered is that assurance of quality control is often a
reason for making rather than buying.

Question #98: Question: Assume that Stewart's idle capacity of 41,000 machine hours has a
traceable avoidable annual fixed cost of $44.000 that will continue if the capacity is not used.
The maximum price Stewart would be willing to pay a supplier for component B18 is
[Fact Pattern] Stewart Industries has been producing two bearings, components B12 and B18, for
use in production.

B12 B18
Machine hours required per units 2.5 3.0
Standard cost per unit:
Direct material $2.25 $3.75
Direct labor 4.00 4.50
Manufacturing overhead:
Variable (see note1) 2.00 2.25
Fixed (see note2) 3.75 4.50
$12.00 $15.00

Stewart's annual requirement for these components is 8,000 units of B12 and 11,000 units of
B18. Recently, Stewart's management decided to devote additional machine time to other
product lines resulting in only 41,000 machine hours per year that can be dedicated to the
production of the bearings. An outside company has offered to sell Stewart the annual supply of
the bearings at prices of $11.25 for B12 and $13.50 for B18. Stewart wants to schedule the
otherwise idle 41,000 machine hours to produce bearings so that the company can minimize its
costs (maximize its net benefits).
Note 1: Variable manufacturing overhead is applied on the basis of direct labor hours.
Note 2: Fixed manufacturing overhead is applied on the basis of machine hours.
Correct Answer: D. Some amount other than those given. All Possible Answers:
A. $10.50
B. $14.00
C. $14.50
D. Some amount other than those given.
Explanation:
Answer (A) is incorrect.
The maximum price Stewart would pay for 818 if it had sufficient idle capacity to produce its
annual requirement of both bearings is $10.50.
Answer (B) is incorrect.
The maximum price for B18 is $14.10 [($3.75 + $4.50 + $2.25 = $10.50) + ($1.20 x 3 hours)].
Answer (C) is incorrect.
The maximum price for B18 is $14.10 [($3.75 + $4.50 + $2.25 = $10.50) + ($1.20 x 3 hours)].
Answer (D) is correct.
If Stewart had sufficient idle capacity to manufacture its annual requirements of both bearings, it
would be willing to pay no more than $10.50 ($3.75 + $4.50 + $2.25) for a unit of B18. Since the
given fixed cost will continue if the idle capacity is not used, Stewart would increase its costs by
paying more than the unit variable cost ($3.75 + $4.50+ $2.25 = $10.50). However, Stewart must
purchase some bearings because it has insufficient idle capacity to produce its requirements. The
given suppliers' prices for B12 and 818 result in a loss per machine hour of $1.20 and $1.00,
respectively. At those prices, Stewart should manufacture all its requirements of B12 and
purchase some units of B18. Assuming the given price of B12 is held constant, Stewart would
benefit from purchasing B12 only if the loss per hour from buying B18 exceeded $1.20 per hour,
or $3.60 per bearing (3 hrs. x $1.20). The maximum price for 818 is thus $14.10 ($10.50 +
$3.60).

Question #99: The sales manager feels confident that, if the credit policy were changed. sales
would increase and, consequently, the company would utilize excess capacity. The two credit
proposals being considered are as follows:
Proposal Proposal
A B
Increase in sales $500,000 $600,000
Contribution margin 20% 20%
Bad debt percentage 5% 5%
Increased in operating
profits $75,000 $90,000
Desired return on sales 15% 15%

Currently, payment terms are net 30. The proposed payment terms for Proposal A and Proposal
B are net 45 and net 90, respectively. An analysis to compare these two proposals for the change
in credit policy would include all of the following factors except the
Correct Answer: B. Current bad debt experience.
A. Cost of funds.
B. Current bad debt experience.
C. Impact on the current customer base of extending terms to only certain customers.
D. Bank loan covenants on days' sales outstanding.

Explanation:
Answer (A) is incorrect.
The cost of funds is an obvious element in the analysis of any investment.
Answer (B) is correct.
All factors should be considered that differ between the two policies. Factors that do not differ,
such as the current bad debt experience, are not relevant. The company must estimate the
expected bad debt losses under each new policy.
Answer (C) is incorrect.
The impact on the current customer base of extending terms to only certain customers is relevant.
The current customers may demand the same terms.
Answer (D) is incorrect.
Existing loan agreements may require the company to maintain certain ratios at stated levels.
Thus, the company's ability to increase receivables and possible bad debt losses may be limited.

Question #100: A corporation has established a risk management process to help it create,
protect, and enhance shareholder value. Which of the following reflects the best order for that
risk process?
Correct Answer: A. Objective setting, event identification, risk assessment, risk response.
A. Objective setting, event identification, risk assessment, risk response.
B. Event identification, objective setting, risk assessment, risk response.
C. Risk assessment, risk response, objective setting, event identification.
D. Risk assessment, objective setting, event identification, risk response.
Explanation:
Answer (A) is correct.
The order of the Enterprise Resource Management program is outlined in the COSO Framework.
After the internal environment is established, objectives must be set, and events that can affect
those objectives must be identified. Once that is complete, the risks must be assessed, and then
possible responses to those risks should be identified and evaluated.
Answer (B) is incorrect.
Objectives must exist before management can identify potential events affecting their
achievement.
Answer (C) is incorrect.
Assessing and responding to risks must occur after objectives have been set, and events affecting
those objectives are identified.
Answer (D) is incorrect.
Assessing the risks must occur after objectives have been set, and events affecting those
objectives are identified.
MOCK TEST 4

Question #1: For the most recent fiscal period, Oakland Inc. paid a regular quarterly dividend of
$0.20 per share and had earnings of $3.20 per share. The market price of Oakland stock at the
end of the period was $40.00 per share. Oakland's dividend yield wa
Correct Answer: B. 2.00%.
A. 6.25%.
B. 2.00%.
C. 1.00%
D. 0.50%.
Explanation:
A. The dividend yield is Annual Dividends Per Common Share ÷ Current Market Price Per
Common Share. This is the quarterly dividend of $0.20 divided by earnings per share of $3.20.
B. The dividend yield is Annual Dividends Per Common Share ÷ Current Market Price Per
Common Share. The quarterly dividend is $0.20, and therefore the annual dividend is $0.20 x 4,
or $0.80 per share. The market price is $40. Therefore, the dividend yield is $0.80 ÷ $40, which
is 0.02 or 2%.
C. The dividend yield is Annual Dividends Per Common Share ÷ Current Market Price Per
Common Share. This is two times the quarterly dividend of $0.20 divided by the current market
price per share.
D. The dividend yield is Annual Dividends Per Common Share ÷ Current Market Price Per
Common Share. This is the quarterly dividend of $0.20 divided by the current market price per
share.
Question #2: At year-end. Appleseed Company reported net income of $588,000. The company
has 10,000 shares of $100 par value. 6% preferred stock and 120,000 shares of $10 par value
common stock outstanding and 5,000 shares of common stock in treasury. There are no dividend
payments in arrears, and the market price per common share at the end of the year was $40.
Appleseed's price-earnings ratio is
Correct Answer. C. 9.09.
A. 9.47
B. 8.50.
C. 9.09.
D. 8.16.
Explanation:
A The price/earnings ratio is the market price of the common stock divided by the company's
most recent annual basic earnings per share. The market price of the common stock is given as
$40 a share, but the basic earnings per share is not given, so it must be calculated. This answer
results from calculating basic earnings per share by dividing income available to common
stockholders by the total of the number of common shares outstanding and the number of shares
of common stock in treasury. Treasury stock is not outstanding common stock because it is not
owned by someone other than the company that issued it Therefore, it should not be included in
the denominator of the basic earnings per share calculation.
B. The price/earnings ratio is the market price of the common stock divided by the company's
most recent annual basic earnings per share. The market price of the common stock is given as
$40 a share, but the basic earnings per share is not given, so it must be calculated. This answer
results from making two mistakes in calculating earnings per share: (1) Using net income in the
numerator and (2) Using the number of common shares outstanding plus the number of treasury
shares in the denominator. Basic earnings per share is income available to common stockholders
divided by the weighted average number of common shares outstanding during the period.
Income available to common shareholders is net income minus the preferred dividend. And
treasury stock is not outstanding common stock because it is not owned by someone other than
the company that issued it Therefore, it should not be included in the denominator of the basic
earnings per share calculation.
C. The price learnings ratio is the market price of the common stock divided by the company's
most recent annual basic earnings per share. The market price of the common stock is given as
$40 a share, but the basic earnings per share is not given, so it must be calculated. Basic earnings
per share is income available to common stock holders divided by the weighted average number
of common shares outstanding during the period. Income available to common shareholders is
$588,000 minus the preferred dividend of $60,000 (1,000,000 x 0.06), or $528,000. Basic
earnings per share is therefore $528,000 ÷ 120,000 shares of common stock outstanding, or
$4.40 per share. (Treasury stock is not outstanding stock so it is not included.) The price-
earnings ratio is therefore $40 ÷ $4.40, which is 9.09.
D. The price/earnings ratio is the market price of the common stock divided by the company's
most recent annual basic earnings per share. The market price of the common stock is given as
$40 a share, but the basic earnings per share is not given, so it must be calculated. This answer
results from calculating basic earnings per share by dividing net income by the number of
common shares outstanding. Basic earnings per share is income available to common
stockholders divided by the weighted average number of common shares outstanding during the
period. Income available to common shareholders is net income minus the preferred dividend.

Question #3: Fad Pattern: Willis, Inc., has a desired rate of return of 15% and is considering the
acquisition of a new machine that costs $400,000 and has a useful life of 5 years. Willis projects
that earnings and cash flow will increase as follows:
After-tax Cash
Year Net Earnings Flow
1 $100,000 $160,000
2 100,000 140,000
3 100,000 100,000
4 100,000 100,000
5 200,000 100,000

15% interest Rate Factors

Period Present Value of $1 Present value of an annuity of $1


1 0.87 0.87
2 0.76 1.63
3 0.66 2.29
4 0.57 2.86
5 0.50 3.36

Correct Answer: C. Positive, $18,600.


A. Negative, $64,000.
B. Negative, $14,000.
C. Positive, $18,600.
D. Positive, $200,000.
Explanation:
Answer (A) is incorrect.
A negative NPV of $64,000 assumes net cash flows of $100,000 per year.
Answer (B) is incorrect.
A negative NPV of $14,000 results from discounting the net earnings.
Answer (C) is correct.
The NPV is calculated by discounting the after-tax cash flows by the desired rate of return. The
cash flows of this company constitute an annuity of $100,000 per year plus additional flows of
$60,000 in Year 1 and $40,000 in Year 2. Thus, the present value of these flows is $418,600
[($100,000 x 3.36 PV of an annuity of $1 for 5 periods at 15%) + ($60,000 x .87 PV of $1 for 1
period at 15%) + ($40,000 x .76 PV of $1 for 2 periods at 15%)]. The NPV is therefore $18,600
($418,600 - $400,000).
Answer (D) is incorrect.
The amount of $200,000 is the undiscounted excess of the cash flows over the investment.

Question #4: For a firm with a degree of operating leverage of 3.5, an increase in sales of 6%
will
Correct Answer: C. Increase pre-tax profits by 21%.
A. Increase pre-tax profits by 3.5%.
B. Decrease pre-tax profits by 3.5%.
C. Increase pre-tax profits by 21%.
D. Increase pre-tax profits by 1.71%.
Explanation:
\Answer (A) is incorrect.
The 3.5 is multiplied times the increase in sales.
Answer (B) is incorrect.
A firm with operating leverage will experience a rise in pre-tax profits when sales increase.
Answer (C) is correct.
A degree of operating leverage (DOL) of 3.5 means that operating income (EBIT) will increase
3.5 times greater than any sales increase. Multiplying 3.5 times the 6% sales increase results in a
pre-tax profit increase of 21%.
Answer (D) is incorrect.
The DOL is multiplied by the sales change, not divided into it.

Question #5: A change from one generally accepted accounting principle to another generally
accepted accounting principle should be accounted for in comparative reports by
Correct Answer: C. a cumulative adjustment to carrying amounts of assets and liabilities as of
the beginning of the first period presented, an offsetting adjustment to the opening balance of
retained earnings of the same period, and by adjusting prior periods' statements presented for the
effects of the change in each period.
A. a line item below extraordinary items on the current income statement
B. pro forma amounts for key figures shown supplementary on the income statement for all
periods presented.
C. a cumulative adjustment to carrying amounts of assets and liabilities as of the beginning
of the first period presented, an offsetting adjustment to the opening balance of retained
earnings of the same period, and by adjusting prior periods' statements presented for the
effects of the change in each period.
D. only a footnote disclosure in the current period.
Explanation:
A. No line item for any cumulative effect adjustment is to be used on the current or any prior
income statement.
B. Pro forma amounts are not to be used for reporting accounting changes.
C. A change of accounting principle is to be accounted for by adjusting carrying amounts of
assets and liabilities as of the beginning of the first period presented for the cumulative effect of
the change on periods prior to those presented in the financial statements. The effect of this prior
period adjustment is offset by adjusting the opening balance of retained earnings of the first
period presented. Financial statements for all periods presented are adjusted for the effects of the
change in each specific period, unless it is impracticable to do so.
D. The financial statements in the period the change is made must include disclosure of the
change, the reason for the change, and an explanation of why the company's management
believes the new accounting principle to be preferable from the perspective of financial reporting
(i.e.. not merely because it will result in favorable income tax consequences). However, footnote
disclosure is not the only requirement.

Question #6: Molar Inc. is evaluating three independent projects for the expansion of different
product lines. The Finance Department has performed an extensive analysis of each project and
the chief financial officer has indicated that there is no capital rationing in effect. Which of the
following statements are correct?
I. The project with the highest net present value (NPV) should be selected and the others
rejected. II. The project with the highest internal rate of return (IRR) exceeding the hurdle rate
should be selected and the others rejected. III. All projects with positive net present values
(NPVs) should be selected. IV. Molar should accept any projects with positive I RRs.
Correct Answer: C. III only.
A. I only.
B. II only.
C. III only.
D. III and IV only.
Explanation: Since there is no capital rationing, all projects with positive net present values
should be selected. Any projects with positive IRRs would still have to be higher than a required
hurdle rate.

Question #7: Fad Pattern: The data presented below show actual figures for selected accounts of
McKeon Company for the fiscal year ended May 31, Year 1, and selected budget figures for the
Year 2 fiscal year. McKeon's controller is in the process of reviewing the Year 2 budget and
calculating some key ratios based on the budget. McKeon Company monitors yield or return
ratios using the average financial position of the company. (Round all calculations to three
decimal places if necessary.)
Using a 365-day year, McKeon's inventory turnover is

5/31/Year 2 5/31/Year 1
Current assets $210,000 $180,000
Noncurrent assets 275,000 255,000
Current liabilities 78,000 85,000
Long-term debt 75,000 30,000
Common stock ($30 par
value) 300,000 300,000
Retained earnings 32,000 20,000

Year 2 Operations
Sales* $350,000
Cost of goods sold 160,000
Interest expense 3,000
Income taxes (40% rate) 48,000
Dividends declared and paid in Year2 60,000
Administrative expense 67,000
*All sales are credit sales

5/31/Year 2 5/31/Year 1
Cash $20,000 $10,000
Accounts receivable 100,000 70,000
Inventory 70,000 80,000
Prepaid expenses 20,000 20,000

Correct Answer: A. 171 days.


A. 171 days.
B. 160 days.
C. 183 days.
D. 78 days.
Explanation:
Answer (A) is correct.
Inventory turnover in terms of days is determined by dividing 365 by the inventory turnover
ratio. The inventory turnover ratio is equal to the $160,000 cost of goods sold divided by the
575,000 average balance in inventory [(580,000 beginning balance + 470,000 ending balance) ÷
2]. Hence, the inventory turnover ratio is 2.133 times per year. Dividing 365 by 2.133 results in
an inventory turnover of 171 days.
Answer (B) is incorrect.
Inventory turnover in terms of days is determined by dividing 365 days by the inventory turnover
ratio. The inventory turnover ratio equals the cost of goods sold divided by average inventory.
Answer (C) is incorrect.
Inventory turnover in terms of days is determined by dividing 365 days by the inventory turnover
ratio. The inventory turnover ratio equals the cost of goods sold divided by average inventory.
Answer (D) is incorrect.
Inventory turnover in terms of days is determined by dividing 365 days by the inventory turnover
ratio. The inventory turnover ratio equals the cost of goods sold divided by average inventory.

Question #8: Protective clauses set forth in an indenture are known as:
Correct Answer: B. covenants.
A. requirements.
B. covenants.
C. addenda.
D. provisions.
Explanation:
Protective covenants set limits (restrictions) on certain actions the company might be taking
during the term of the agreement. They are a particularly important feature in a bond indenture.

Question #9: When reviewing a credit application, the credit manager should be most concerned
with the applicant's:
Correct Answer: C. working capital and current ratio.
A. price-earnings ratio and current ratio.
B. working capital and return on equity.
C. working capital and current ratio.
D. profit margin and return on assets.
Explanation:
Liquidity measures, such as net working capital and the current ratio, help determine ability to
pay expenses on a timely basis. Therefore, the credit manager should be most concerned with
these measures in comparison to the others listed in the problem. Profit margin, price-earnings
ratio, and return on equity are profitability measures.

Question #10: ABC Company has revenues of $5,000,000, with a net profit margin of 15%. Its
operating cash flow is $1,300,000, its total assets are $6,000,000 (with current assets of
$1,500,000), and its total liabilities are $2,500,000 (with current liabilities of $500,000). This
leaves equity of $3,500,000, of which $500,000 is preferred shareholders' equity. ABC Company
has 1,000,000 shares of common stock outstanding. What is ABC Company's operating cash
flow to income ratio?
Correct Answer: D. 1.73.
A. 0.26.
B. 0.58.
C. 0.22.
D. 1.73.
Explanation:
The operating cash flow to income ratio is calculated by dividing operating cash flow by income.
Net profit can be computed as $5 million x 15% or $750,000. The operating cash flow to income
ratio is 1.73 ($1,300,000 operating cash flow ÷ $750,000 net income).
Question #11: What have public companies created to enable their employees to report improper
behavior anonymously?
Correct Answer: B. A confidential hotline that provides a means to help people overcome fear or
apprehension to speak openly when they see others not behaving in an appropriate manner.
A. Workers councils, under the supervision of company management, where employees can
discuss their issues with other employees.
B. A confidential hotline that provides a means to help people overcome fear or
apprehension to speak openly when they see others not behaving in an appropriate
manner.
C. Meetings with members of senior management where the employee can raise their
concerns to corporate decision makers.
D. A corporate ombudsman who brings together the conflicting parties to arbitrate a solution
to the conflict.
Explanation:
A. This choice is not correct because disclosure to a workers council would not be
anonymous.
B. This is correct because a hotline provides employees a method to disclose potential ethics
violations anonymously.
C. This choice is not correct because it does not offer a way to report potential violations
anonymously.
D. This choice is not correct because it does not offer a way to report potential violations
anonymously.

Question #12: When reporting a change in accounting principle,


Correct Answer: • the change is recognized by retrospectively adjusting the financial statements.
A. The pro forma effects of retroactive application of the new principle upon income before
extraordinary items and net income are not to be disclosed on the face of the income
statement or in the notes to the financial statements
B. The change is recognized by including the cumulative effect of the change in the net
income of the period of change.
C. The change is recognized by retrospectively adjusting the financial statements.
D. Income before extraordinary items is reported in the year of the change reflecting the
application of the new principle, but on a basis that includes the cumulative adjustment.
Explanation:
A. This is not the way a change in accounting principle is accounted for. See the correct
answer for a complete explanation.
B. This is not the way a change in accounting principle is accounted for. See the correct
answer for a complete explanation.
C. A change from one general accepted accounting principle to another generally accepted
accounting principle is recognized by including the cumulative effect of the change on
periods prior to those presented in the carrying amounts of assets and liabilities as of the
beginning of the first period presented and adjusting the opening balance of retained
earnings as of the beginning of the earliest period presented. Financial statements for
each prior period presented are adjusted to reflect the period-specific effects of applying
the new accounting principle.
D. This is not the way a change in accounting principle is accounted for. See the correct
answer for a complete explanation.

Question #13: Question: Assume sales and cost of goods sold for Year 2 were $300,000 and $
220,000, respectively. Lisa, Inc.’s inventory for Year 2 was
Lisa, Inc.
Statement of Financial Portion
December 31, Year 2
(00s)

Assets Year 2 Year 1


Current assets:
Cash $30 $25
Trading securities 20 15
Accounts receivables (net) 45 30
Inventories (at lower of cost or market) 60 50
Prepaid items 15 20
Total current assets 170 140
Long-term investments:
Securities (at cost) 25 20
Property, plant, & equipment
Land (at cost) 75 75
Building (net) 80 90
Equipment (net) 95 100
Intangible assets
Patents (net) 35 17
Goodwill (net) 20 13
Total long-term assets 330 315
Total assets $500 $455
Liabilities & Shareholders' Equity
Current liabilities:
Notes payable $23 $12
Accounts payable 47 28
Accrued interest 15 15
Total current liabilities 85 55
Long-term debt:
Notes payable 10% due 12/31/Year 9 10 10
Bonds payable 12% due 12/31/Year 8 15 15
Total long-term debt 25 25
Total liabilities $110 $80
Shareholders' equity:
Preferred-5%cumulative,$100 par, non-participating, 1000 shares authorized, issued
and outstanding $100 $100
Common-$10 par 20,000 shares authorized, 15,000 issued and outstanding shares 150 150
Additional paid-in-capital- common 75 75
Retained earnings 65 50
Total shareholders' equity $390 $375
Total liabilities & equity $500 $455

Correct Answer B. 4.0 times.


A. 3.7 times.
B. 4.0 times.
C. 4.4 times.
D. 5.0 times.
Explanation:
Answer (A) is incorrect.
The number 3.7 is based on the ending inventory.
Answer (B) is correct
The inventory turnover is computed by dividing cost of goods sold by average inventory.
Consequently the turnover is 4 times [$270,000 ÷ [($60,000 + $50,000) ÷ 2]).
Answer (C) is incorrect
The number 4.4 is based on the beginning inventory.
Answer (D) is incorrect.
Sales divided by ending inventory equal 5.0.
Question #14: Short-term interest rates are
Correct Answer: D. Usually lower than long-term rates.
A. Lower than long-term rates during periods of high inflation only.
B. Usually higher than long-term rates.
C. Not significantly related to long-term rates.
D. Usually lower than long-term rates.
Explanation:
A. During a period of high inflation, short-term interest rates are probably going to be higher
than long-term interest rates.
B. There is less risk in a short term situation because there is less time for something to
happen that would prevent the repayment of whatever amount was financed. Interest rates
incorporate the amount of repayment risk that the investor perceives. The higher the risk
the higher the rate. Therefore, short-term rates would not usually be higher than long-
term rates.
C. Short-term rates are related to long-term rates, although that relationship can change from
time to time. Term Structure of Interest Rates graphs describe the various relationships
that can exist at any given time between short-term rates and long-term rates. The yield
curve can be upsloping, downsloping, Flat, or humped. Yield curve theories attempt to
explain the slopes of the yield curves.
1. The Pure Expectations Theory states that the shape of the yield curve is determined
exclusively by expectations in the market of future interest rates.
2. Liquidity Preference Theory says that if investors increase their risk by holding long-
term bonds, they will require higher compensation in the form of a higher interest rate for
assuming that increased risk.
3. The Segmented Markets Theory focuses on cash needs of different groups of investors
and borrowers and maintains that each group chooses securities that meet its forecasted
cash needs.
4. The Preferred Habitat Theory is a compromise. It agrees with the Segmented Markets
Theory in saying that investors and borrowers normally concentrate on a particular
maturity market. However, it also recognizes, in agreement with the Pure Expectations
Theory, that expectations about future interest rate movements can cause investors and
borrowers to leave their preferred maturity markets for other maturity markets.

D. Usually short-term interest rates are lower than long-term interest rates. This is due
partially to the fact that there is less risk in a short term situation because there is less
time for something to happen that would prevent the repayment of whatever amount was
financed.
Question #15: The current market price of Action Pharmaceutical's common stock is $34, A 6-
month call option has been written on the stock. The option has an exercise price of $40 and a
market value of $4. A financial analyst estimates that, at the end of 6 months, the expected value
of the stock is $42.
What is the theoretical value of exercising the option on the date it is written?
Correct Answer: C. $0
A. $6.00
B. $8.00
C. $0
D. $4.00
Explanation:
A. This is the difference between the exercise price of the option and the current market
price of the stock. However, that is not the theoretical value of the option on the date it is
written.
B. This is the difference between the expected market value of the stock and the current
market value of the stock. However, this is not the theoretical value of the option on the
date it is written.
C. At the time the option was written, it had no theoretical value. This is because the option
exercise price ($40) was higher than the market price ($34). Because of this, no one
would use the option to buy a share since it would be cheaper to buy a share on the open
market.
D. This is the market value of the option. However, the market value is not the same as the
theoretical value of the option on the date it is written.

Question #16: Elgers' Company produces valves for the plumbing industry. Elgers' per unit sales
price and variable costs are as follows.
Sales price $12
Variable costs 8
Elgers' practical plant capacity is 40,000 units, Elgers' total fixed costs aggregate $48,000 and it
has a 40% effective tax rate. The maximum net profit that Elgers can earn is
Correct Answer: D. $67,200.
A. $112,000.
B. $48,000.
C. $96,000.
D. $67,200.
Explanation:
A. This is the net income before income tax.
B. This is the amount of Elgers' fixed costs.
C. This is the total contribution margin reduced by 40% for taxes. It does not include any
deduction for fixed costs.
D. Elgers' unit contribution margin is $4 ($12 - $8). The plant capacity is 40,000 units, so
the company's maximum total contribution margin is 40,000 x $4, or $160,000. Fixed
costs are $48,000, so the maximum net income before tax is $112,000. Taxes are 40%, or
$44,800, and net profit is $112,000 - $44,800, which is $67,200. This is the maximum net
profit Elgers can earn.

Question #17: The economic order quantity for a product is 500 units. However, new orders
require 4 working-days lead time during which 80 units will be used. Given this information, the
correct economic order quantity is
Correct Answer: B. 500 units.
A. 420 units.
B. 500 units.
C. 509 units.
D. 580 units.
Explanation:
Answer (A) is incorrect.
The lead time does not affect the amount of EOQ.
Answer (B) is correct.
The lead times does not affect the EOQ; it just means the order should be placed four days
earlier.
Answer (C) is incorrect
The lead time does not affect the amount of EOQ.
Answer (D) is incorrect.
The lead time does not affect the amount of EOQ.
Question #18: Breakeven quantity is defined as the volume of output at which revenues are equal
to
Correct Answer: A. total costs.
A. Total costs.
B. Fixed costs.
C. Variable costs.
D. Marginal costs.
Explanation:
A. The breakeven quantity is the number of units that must be sold in order to cover all
costs, both fixed and variable, so that the company does not have a loss.
B. Fixed costs are costs that do not change, regardless of the level of production or sales, as
long as the volume remains within the relevant range. The breakeven quantity is not the
volume of output at which revenues are equal to fixed costs. Variable costs also need to
be covered, or a loss equal to the amount of the variable costs will result.
C. Variable costs are costs that are incurred only when a product is made (or sold), such as
material or labor or sales commissions. The total variable cost increases as production or
sales increases, but the per unit variable cost remains unchanged as production or sales
volume increases or decreases, the breakeven quantity is not the volume where revenues
are equal to variable costs. Fixed costs also need to be covered, or a loss equal to the
amount of the fixed costs will result.
D. The marginal cost is the cost of the next unit produced or sold (whichever is appropriate).
The breakeven quantity is not the volume where revenues are equal to marginal cost.

Question #19: Verla Industries is trying to decide which one of the following two options to
pursue. Either option will take effect on January 1st of the next year.
Option One - Acquire a new finishing machine
The cost of the machine is $1,000,000 and will have a useful life of five years. Net pre-tax cash
flows arising from savings in labor costs will amount to $100,000 per year for five years.
Depreciation expense will be calculated using the straight-line method for both financial and tax
reporting purposes. As an incentive to purchase. Verla will receive a trade-in allowance of
$50.000 on their current fully depreciated finishing machine.
Option Two - Outsource the Finishing Work Verla can outsource the work to LM Inc. at a cost
of $200.000 per year for five years. If they outsource. Verla will scrap their current fully
depreciated finishing machine. Assume removal costs equal the machines scrap value.
Verla's effective income tax rate is 40%. The weighted-average cost of capital is 10%. The net
present value of acquiring the new finishing machine (i.e., Option One) is:
Correct Answer: A. - $434,424.
A. - $434.424.
B. - $369,260.
C. - $229,710.
D. - $267.620.
Explanation:
The cash inflow from the investment in the new machine for each of the five years is calculated
as follows:
Annual cash flow from investment = (annual savings - annual depreciation expense)(1 - tax rate)
+ (annual depreciation expense)
Annual depreciation expense = (cost - trade-in allowance) / 5 year life
Annual depreciation expense = ($1.000.000 - $50.000) / 5 years = $190,000
Annual cash flow from investment = ($100,000 - $190,000) (1-0.4) + $190,000
Annual cash flow from investment = $136,000
The net present value (NPV) of acquiring the new finishing machine can now be calculated:
NPV, new machine = (initial investment) + (annual cash flow) (PV annuity factor, i=10, n=5)
NPV, new machine = (-$950,000) + ($136,000)(3.791) = -$434,424.

Question #20: An analyst gathers the following data about a financial calculator produced and
sold by a company
• Unit selling price = $60
• Quantity sold = 100.000
• Unit variable costs = $35
• Fixed costs = $200.000
What is the contribution margin per unit and total contribution margin?
Correct Answer: B. Contribution margin per unit is $25 and total contribution margin is
$2,500,000.
A. Contribution margin per unit is $23 and total contribution margin is $2,500,000.
B. Contribution margin per unit is $25 and total contribution margin is $2,500,000.
C. Contribution margin per unit is $25 and total contribution margin is $2,300,000.
D. Contribution margin per unit is $23 and total contribution margin is $2,300,000.
Explanation: The contribution margin per unit is the unit selling price less the unit variable
costs. The total contribution margin is the contribution margin per unit times the quantity of
units sold. Thus, the contribution margin per unit is $60 - $35 = $25: and the total
contribution margin is $25 x 100,000 = $2,500,000.

Question #21: In the decision-making process, differential cost is a(n):


Correct Answer: B. cost that changes among alternative courses of action.
A. Fixed cost of alternative courses of action.
B. Cost that changes among alternative courses of action.
C. Sunk cost of alternative courses of action.
D. Opportunity cost of alternative courses of action.
Explanation: A differential cost is by definition a cost that differs for each decision
alternative. Thus, a decision maker considers only differential costs in the decision-making
process; the costs that do not change among alternative courses of action are considered
irrelevant for the decision.

Question #22: A service company's working capital at the beginning of January of the current
year was $70.000. The following transactions occurred during January:

Performed services on account $30,000


Purchased supplies on account 5,000
Consumed supplies 4,000
Purchased office equipment for cash 2,000
Paid short-term bank loan 6,500
Paid salaries 10,000
Accrued salaries 3,500

What is the amount of working capital at the end of January?


Correct answer: B. $80,500
A. $90,000
B. $80,500
C. $50,000
D. $47,000
Explanation:
Answer (A) is incorrect.
The amount of s90.000 ignores the consumed supplies, the cash purchase of office equipment,
and the accrued salaries.
Answer (B) is correct.
Working capital is the excess of total current assets (CA) over total current liabilities (CL). Thus,
working capital at the end of January equals $80,500 computed as follows:
Current Current
Assets* Liabilities*
Beginning working capital $70,000
Performed services on account 30,000 I N
Purchased supplies on account 0 I I
Consumed supplies -4,000 D N
Purchased office equipment -2,000 D N
Paid short-term back loan 0 D D
Paid salaries -10,000 D N
Accrued salaries -3,500 N I
Working capital, end of
January $80,500

*N = no effect; I = increase; D = decrease


Answer (C) is incorrect
The amount of $50,000 does not include the service performed on account.
Answer (D) is incorrect.
The amount of $47,500 omits the services performed on account and accrued salaries but
includes the payment of short-term loan.

Question #23: A company had $5 million in sales. $3 million in cost of goods sold, and $1
million in selling and administrative expenses during the last fiscal year. If the company's
income tax rate was 25%, what was the company's gross profit margin percentage?
Correct Answer: D. 40%
A. 20%.
B. 50%.
C. 30%.
D. 40%
Explanation:
A. This is net income before tax divided by sales revenue. The gross profit margin percentage is
gross profit divided by sales revenue. Gross profit is sales revenue minus cost of goods sold.
B. This is not the correct answer. The gross profit margin percentage is gross profit divided by
sales revenue. Gross profit is sales revenue minus cost of goods sold.
C. This is not the correct answer. The gross profit margin percentage is gross profit divided by
sales revenue. Gross profit is sales revenue minus cost of goods sold.
D. This is the correct answer. The gross profit margin percentage is gross profit divided by sales
revenue. Gross profit is sales revenue minus cost of goods sold. Gross profit equals $5,000,000 -
$3,000,000, which is $2,000,000. The gross profit margin percentage is $2,000,000 +
$5,000,000, which is 0.40 or 40%.

Question #24: IMA's Statement of Ethical Professional Practice includes an integrity standard,
which requires an IMA member to
Correct Answer: A. Decline to become a minority partner in a partnership that is a supplier of the
member's employer. All Possible Answers:
A. Decline to become a minority partner in a partnership that is a supplier of the member's
employer.
B. Report all information that could influence users of financial statements.
C. Disclose confidential information when authorized by his or her firm or required under
the law
D. Refuse gifts from anyone.
Explanation:
Answer (A) is correct.
One of the responsibilities of an [MA member under the integrity standard is to 'refrain from
engaging in any conduct that would prejudice carrying out duties ethically."
Answer (B) is incorrect.
The credibility standard requires an IMA member to "disclose all relevant information that could
reasonably be expected to influence an intended user's understanding of the reports, analyses,
and recommendations."
Answer (C) is incorrect.
The confidentiality standard requires an IMA member to "keep information confidential except
when disclosure is authorized or legally required."
Answer (D) is incorrect.
The integrity standard requires an IMA member to "mitigate actual conflicts of interest.
Regularly communicate with business associates to avoid apparent conflicts of interest. Advise
all parties of any potential conflicts."

Question #25: There are many strategies for risk response that an organization may choose from
to reduce risk to an acceptable level. Which action would not reduce risk for an organization?
Correct Answer: C. Adjusting the organization's capital structure to maximize the cost of capital.
A. Purchasing property and casualty insurance.
B. Requiring all employees to be bonded.
C. Adjusting the organization's capital structure to maximize the cost of capital.
D. Shifting costs from fixed to variable.
Explanation: This action would reduce risk by shifting risk to the bonding company.

Question #26: The days' sales in receivables ratio will be understated if the company
Correct Answer: B. Uses a natural business year for its accounting period.
A. Does not use average receivables in the ratio calculation.
B. Uses a natural business year for its accounting period.
C. Uses a calendar year for its accounting period.
D. Uses average receivables in the ratio calculation.
Explanation:
A. Not using average receivables in the days' sales-in-receivables ratio will not always
understate the ratio. The days' sales-in-receivables ratio is calculated as the days in the
year divided by the receivables turnover ratio. The receivables turnover ratio is calculated
as sales divided by average receivables. In order to understate the days' sales-in-
receivables ratio, either the sales need to be understated or the receivables turnover ratio
needs to be overstated.
B. A natural business year ends at the end of its main selling season when inventories have
been depleted and thus are low and accounts receivable from the selling season have been
collected and thus are also low. If the company uses the natural business year for its
accounting period, the accounts receivable balances will be understated because the
natural business year will start and end at the down cycle of the business. Because the
receivables will be understated, the receivables turnover will be higher than it would be
otherwise. This will lead to an understated days' sales-in-receivables ratio.
C. The use of the calendar year for the accounting period will not always understate the
ratio.
D. The use of average receivables in the days' sales-in-receivables ratio will not always
understate the ratio.

Question #27: Fact Pattern: Tosh Enterprises reported the following account information: (Refer
the image) Question: The current ratio for Tosh Enterprises is
The Enterprises reported the following account information:

Accounts receivable $400,000


Accounts payable 260,000
Bonds payable, due in 10 years 600,000
Cash 200,000
Interest payable, due in 3 months 20,000
Inventory $800,000
Land 500,000
Short-term prepaid expense 80,000

Correct Answer: D. 5.29


A. 1.68
B. 2.14
C. 5.00
D. 5.29
Explanation:
Answer (A) is incorrect.
The figure of 1.68 includes long-term bonds payable among the current liabilities.
Answer (B) is incorrect.
The figure of 2.14 is the quick ratio.
Answer (C) is incorrect
The figure of 5.00 excludes prepaid expenses from current assets.
Answer (D) is correct.
The current ratio equals current assets divided by current liabilities. Current assets consist of
cash, accounts receivable, inventory, and prepaid expenses, a total of $1,480,000 ($400,000 +
$200,000, $800,000+ $80,000). Current liabilities consist of accounts payable and interest
payable, a total of $280,000 ($260,000+ $20,000). Hence, the current ratio is 5.29 ($1,480,000 ÷
$280,000).
Question #28: A company invested 5300.000 in a new machine to produce cones for the textile
industry. Variable costs are 30% of the selling price and fixed costs are 5600.000. The company
has an effective income tax rate of 40%. The amount of sales required to earn an 8% after-tax
return on its investment would be
Correct Answer: B. $914.286
A. $891.429
B. $914,286
C. $2,080,000
D. $2.133.333
Explanation:
Answer (A) is incorrect.
The amount of $891,429 results from setting operating income, rather than net income, to the
target return on investment
Answer (B) is correct.
The company can calculate its target sales figure as follows:
Contribution margin - Fixed costs = Operating income
Sales - Variable costs - Fixed costs = Return on investment ÷ (1.0 - tax rate)
Sales - .3Sales - $600,000 = .08($300,000) + (1.0 - .40) .7
Sales - $600,000 = $24,000 ÷ .60 .7
Sales = $640,000
Sales = $914,286
Answer (C) is incorrect
The amount of $2,080,000 results from setting operating income, rather than net income, to the
target return on investment and reversing the variable cost ratio.
Answer (D) is incorrect.
The amount of $2,133,333 results from reversing the variable cost ratio.

Question #29: All of the following are popular reasons that companies may use off-balance sheet
financing except:
Correct Answer: B. To increase assets and debt on the balance sheet.
A. To improve certain financial ratios.
B. To increase assets and debt on the balance sheet.
C. To mitigate or transfer risk.
D. To make use of an asset without showing the corresponding liability on the balance sheet.
Explanation:
By definition, off-balance sheet financing activities are structured to minimize the recording of
assets and / or liabilities on a company's balance sheet which would make "to increase assets and
debt on the balance sheet contrary to their use.

Question #30: All of the following are legal rights of shareholders in U.S. publicly-traded
companies except the right to:
Correct Answer: A. vote on major management changes.
A. Vote on major management changes.
B. Receive annual financial reports.
C. Vote on charter and bylaw changes.
D. Vote on major mergers and acquisitions.
Explanation:
Changes in management (the CEO and below) are the responsibility of the board of directors.
The board members are elected by the shareholders. The board is responsible for "directing"
the corporation. Direction includes planning, organizing, staffing, coordinating, and
controlling.

Question #31: A corporation has a current ratio of 2 to 1 and a quick ratio (acid test) of 1 to
1. A transaction that would change the quick ratio but not the current ratio is the
Correct Answer: A. Sale of inventory on account at cost
A. Sale of inventory on account at cost
B. Collection of accounts receivable.
C. Payment of accounts payable.
D. Purchase of a patent for cash.
Explanation:
Answer (A) is correct.
The quick (acid test) ratio equals the quick assets (cash, marketable securities. and accounts
receivable) divided by current liabilities. The current ratio is equal to current assets divided
by current liabilities. The sale of inventory (not a quick current asset) on account increases
accounts receivable (a quick asset), thereby changing the quick ratio. The sale of inventory
on account, however, replaces one current asset with another, and the current ratio is
unaffected.
Answer (B) is incorrect.
Neither ratio is changed.
Answer (C) is incorrect
The current, not the quick ratio changes.
Answer (D) is incorrect.
Both decrease.
Question #32: The stage of the capital budgeting process that has the most risk is
Correct Answer: B. Forecasting cash flow.
A. Identifying alternative possible projects.
B. Forecasting cash flow.
C. Raising funds to initially support the project.
D. Evaluating performance and learning.
Explanation:
Answer (A) is incorrect.
Identifying alternative possible projects does not have the most risk in the capital budgeting
process. There are many different methods that can aid in accurately identifying different
projects. This reduces the risk of selecting alternatives.
Answer (B) is correct.
Forecasting cash flows has the most risk in the capital budgeting process. The economic
benefit or cost must be estimated period by period. In addition, the economic life, depreciable
life, and salvage value of the asset must be estimated. All of these estimates help forecast the
cash flows for the project. Because this step requires the most use of estimates, it is said to be
the riskiest in the capital budgeting process.
Answer (C) is incorrect.
Raising funds to initially support the project does not have the most risk in the capital
budgeting process. This step does not require many estimations and is something
management has control over, reducing the amount of risk involved.
Answer (D) is incorrect.
Evaluating performance and learning does not have the most risk in the capital budgeting
process. Evaluating performance is based on quantitative measures that can be evaluated
objectively, reducing the amount of risk involved.

Question #33: McLean Inc. is considering the purchase of a new machine that will cost
$160,000. The machine has an estimated useful life of 3 years. Assume that 30% of the
depreciable base will be depreciated in the first year. 40% in the second year, and 30% in the
third year. The new machine will have a $10,000 resale value at the end of its estimated
useful life. The machine is expected to save the company $85,000 per year in operating
expenses. McLean uses a 40% estimated income tax rate and a 16% hurdle rate to evaluate
capital projects.
Discount rates for a 16% rate are as follows:

Present value of an ordinary


Present value of $1 Annuity of $1
Year 1 0.862 0.862
Year 2 0.743 1.605
Year 3 0.641 2.246

What is the net present value of this project?


Correct Answer: D. $6.270
A. $5,842
B. $8,834
C. $30,910
D. $6,270
Explanation:
A. An answer of $5.842 results from subtracting the resale value of the equipment from the
purchase cost to calculate the depreciable base for tax purposes. For tax purposes. 100%
of an asset's cost is always depreciated.
B. An answer of $8.834 results from not calculating income tax due on the disposition of the
equipment at the end of 3 years. However, there will be a gain on the sale, since 100% of
the asset's cost will have been depreciated for tax purposes by the end of the third year.
C. An answer of $30.910 results from including only the initial investment and the gross
operating cash flows in the calculation of the net present value.
D. The cash flows are as follows:
The cash flows are as follows:

Year 0 Year 1 Year 2 Year 3


-
Initial investment 160,000
Depreciation 48,000 64,000 48,000
Depreciation tax sheild (depr. × 0.40) 19,200 25,600 19,200
Cash from disposition (after tax) 6,000
Operating cash flows 85,000 85,000 85,000
Tax on operating cash flow at 40% -34,000 -34,000 -34,000
Net cash flow 70,200 76,600 76,200
Discount factor: 16% 0.862 0.743 0.641
Discounted cash flow - 60,512 56,914 48,844
160,000

The net present value is $(160,000) + $60,512 + $56,914 + $48,844 = $6,270

Question #34: The quantity of a product is said to be relatively elastic when:


Correct Answer: A. people buy less at higher prices but the percent change in quantity
demanded is greater than the percent change in price.
A. People buy less at higher prices but the percent change in quantity demanded is greater
than the percent change in price.
B. People buy more at higher prices but the percent change in quantity demanded is less
than the percent change in price.
C. A price change has no effect on quantity demanded.
D. A price change leads to a price war.
Explanation:
A quantity demanded is said to be relatively elastic when the change in the amount people
buy is greater than the change in price, either higher or lower; a price change will cause an
even larger change in quantity demanded.

Question #35: A firm is constructing a risk analysis to quantify the exposure of its data center
to various types of threats. Which one of the following situations would represent the highest
annual loss exposure after adjustment for insurance proceeds?
Correct Answer: A. Frequency of Occurrence: 1 year Loss Amount: $15.000 Insurance
Coverage: 85%.
A. Frequency of Occurrence: 1 year Loss Amount: $15,000 Insurance Coverage: 85%.
B. Frequency of Occurrence: 100 years Loss Amount: $400,000 Insurance Coverage: 50%.
C. Frequency of Occurrence: 8 years Loss Amount: 575,000 Insurance Coverage: 80%.
D. Frequency of Occurrence: 20 years Loss Amount: $200,000 Insurance Coverage: 80%.
Explanation:
This situation would represent the highest annual loss exposure after the adjustment for
insurance. The expected annual loss would be 12,750 [15,000(.85)]. The expected annual
loss for Frequency of Occurrence of 8 years = (75,000/8) (.8) = 9,375(.8) = 7,500. The
expected annual loss for Frequency of Occurrence of 20 years = (200,000/20) (.8) =
100,000(.8) = 8.000. The expected annual loss for Frequency of Occurrence of 100 years =
(400,000/100) (.5) = 4,000(.5) = 2,000.

Question #36: Pearl Manufacturing Company has sales of $1,000,000, variable costs of
$500,000, and fixed costs of $250,000. What is the Degree of Operating Leverage?
Correct Answer: A.2.
A. 2.
B. 1.5.
C. 1.
D. 0.5.
Explanation:
Business risk is often measured by the degree of operating leverage. The formula to calculate
degree of operating leverage is DOL = Contribution Margin / Operating Income. Therefore.
DOL = ($1,000,000 - $500,000) / ($1,000,000 - $500,000 - $250,000) = 2.

Question #37: Fact Pattern: Assume the following information pertains to Ramer Company.
Matson Company, and for their common industry for a recent year.
Industry
Ramer Matson Average
Current ratio 3.50 2.80 3.00
Accounts receivable turnover 5.00 8.10 6.00
Inventory turnover 6.20 8.00 6.10
Times interest earned 9.00 12.30 10.40
Debt to equity ratio 0.70 0.40 0.55
Return on investment 0.15 0.12 0.15
Dividend payout ratio 0.80 0.60 0.55
Earnings per share $3.00 $2.00 __

Question: Some of the ratios and data for Ramer and Matson are affected by income taxes.
Assuming no inter period income tax allocation, which of the following items would be
directly affected by income taxes for the period?
Correct Answer: C. Return on investment and earnings per share.
A. Current ratio and debt to equity ratio.
B. Accounts receivable turnover and inventory turnover.
C. Return on investment and earnings per share.
D. Debt to equity ratio and dividend payout ratio.
Explanation:
Answer (A) is incorrect.
Neither ratio is based on net income.
Answer (B) is incorrect.
These turnover ratios are based on asset accounts and figures at the top of the income
statement not net income.
Answer (C) is correct.
Income taxes are an expense of the business and affect rates of return and earnings per share.
Any ratio that uses net income as a part of the calculation is affected. e.g., return on
investment. EPS, and dividend payout.
Answer (D) is incorrect.
The debt to equity ratio is not affected by taxes.

Question #38: Fact Pattern: Calamity Cauliflower Corporation is considering undertaking a


capital project. The company would have to commit $24,000 of working capital in addition
to an immediate outlay of $160,000 for new equipment. The project is expected to generate
$100,000 of annual income for 10 years. At the end of that time, the new equipment, which
will be depreciated on a straight-line basis, is expected to have a salvage value of $10,000.
The existing equipment that would be sold to make room for the project has a historical cost
of $220,000 and accumulated depreciation of $208,000. It has an estimated remaining useful
life of 2 years and the remaining carrying amount is being depreciated on a straight-line
basis. A scrap dealer has agreed to buy it for $8,000.
The company's effective tax rate is 40%.
Question: Calamity Cauliflower's expected depreciation tax shield for the final year of the
project is
Correct Answer: • $6,400
A. $6,400
B. $4,000
C. $2,400
D. $0
Explanation:
Answer (A) is correct.
The old equipment has a remaining useful life of two years, after which it will be fully
depreciated and no longer generating depreciation expense. Thus, the depreciation tax shield
in the project's last year consists only of the annual depreciation expense on the new
equipment ($160,000 historical cost =10 years = $16,000) times the tax rate (.40), or $6,400.
Answer (B) is incorrect.
The amount of $4,000 is the additional depreciation tax shield for the first two years of the
project.
Answer (C) is incorrect.
The amount of $2,400 is only the annual depreciation savings from disposing of the old
equipment.
Answer (D) is incorrect.
The company will have a depreciation tax shield every year of the project.

Question #39: Investors Group Ltd. holds an investment portfolio that has a daily value-at-
risk (VaR) of $500,000 at a 99% confidence level. This means that
Correct Answer:
B. the portfolio will probably lose more than $500,000 of its value on one day out of every
100 trading days, or two or three days out of each year.
A. The expected loss on the portfolio is $500,000.
B. The portfolio will probably lose more than $500,000 of its value on one day out of every 100
trading days, or two or three days out of each year.
C. The expected return on the portfolio is 99%.
D. The expected return on the portfolio is 99%.
Explanation:
A. VaR does not provide information about how much could be lost in the portfolio over a given
period of time but rather provides a worst case scenario and an estimation of how often that
worst case scenario will occur (called a confidence interval).
B. The Value-at-Risk model indicates the maximum loss over a given time period (here. one
day) such that there is a low probability (here. a 1% probability. or 100% minus 99%), that the
actual loss over the given period will be larger. The 1% probability translates to the event
occurring on one out of every 100 trading days, or two to three days out of each year. Therefore,
the portfolio will probably lose more than $500,000 of its value in a one-day period two or three
times each year.
C. VaR does not provide information about expected return on a portfolio, but rather provides a
worst case scenario and an estimation of how often that worst case scenario will occur (called a
confidence interval).
D. VaR does not provide information about the market value of a portfolio or how much could
be lost in the portfolio over a given period of time but rather provides a worst case scenario and
an estimation of how often that worst case scenario will occur (called a confidence interval).

Question #40: In evaluating a capital budget project, the use of the net present value (N PV)
model is generally not affected by the
Correct Answer: B. Method of funding the project.
A. Amount of added working capital needed for operations during the term of the project.
B. Method of funding the project.
C. Initial cost of the project.
D. Type of depreciation used.
Explanation:
A. Amount of added working capital needed for operations during the term of the project is
a component of a net present value capital budgeting analysis.
B. The method of funding the project is separate from the all methods of analyzing a capital
investment project including the net present value method.
C. The initial cost of the project is a component of a net present value capital budgeting
analysis.
D. The type of depreciation used for the projects fixed assets is a component of a net present
value capital budgeting analysis. The depreciation method used affects the depreciation
tax shield. which affects cash flow

Question #41: A power plant estimates that the cost to decommission its nuclear power plant in
today's dollars is $500 million. This cost is expected to escalate at 5% per year over the life of
the plant. The power plant must collect a constant amount each year from customers over the
remaining 20-year life of the plant and place the amounts in a fund that is expected to earn at a
rate of 7% per year. The fund currently has a balance of $100 million. How much must the
power plant collect from customers each of the next 20 years to cover the decommissioning
costs? Ignore income tax effects and round to millions.
Correct Answer: C. $23 million. All Possible Answers:
A. $38 million.
B. $26 million.
C. $23 million.
D. $20 million.
Explanation:
Answer (A) is incorrect.
The amount of $38 million fails to consider the growth of the $100 million already accumulated,
it also uses the wrong table.
Answer (B) is incorrect.
The amount of $26 million is based on the use of the wrong table.
Answer (C) is correct.
The first step is to determine the amount that will be needed in 20 years. If the $500 million
increases by 5% annually, the amount needed in 20 years would be $1,326,500,000 ($500
million x 2.653). Since there is already $100 million available in the fund, that amount will grow
at 7% annually for 20 years to equal $387 million. Subtracting the $387 million from
$1,326,500,000 leaves $939,500,000 to be raised. Use a present value table to find the present
value of 20 equal payments that will accumulate to $939.500.000 in 20 years at 7% interest.
Dividing the $939,500,000 by the future value factor of 40.995 (20 years at 7%) equals
$22,917,429 per year. or approximately $23 million.
Answer (D) is incorrect.
The amount of $20 million is based on the use of the wrong table.

Question #42: A company uses the payback method as part of its analysis of capital investments.
One of its projects requires a $140.000 investment and has the following projected before-tax
cash flows.
Year 1 $60.000
Year 2 60.000
Year 3 60,000
Year 4 80,000
Year 5 80,000
The company has an effective 40% tax rate. Based on these data, the after-tax payback period is
Correct Answer: • 3.7 years. All Possible Answers:
A. 1.5 years.
B. 2.3 years.
C. 3.4 years.
D. 3.7 years.
Explanation:
Answer (A) is incorrect.
The period of 1.5 years reduces the initial investment by a tax factor.
Answer (B) is incorrect.
The period of 2.3 years results from using before-tax, rather than after-tax, cash flows.
Answer (C) is incorrect
The period of 3.4 years results from using a before-tax inflow of $80,000 in Year 3.
Answer (D) is correct.
The payback period for this project can be calculated as follows:
Remaining
Before-Tax Tax After-Tax Initial
Cash Flow Effect Cash Flow Investment
Year 0 --------------$140.000
Year 1 $60,000 60% $36,000 104,000
Year 2 60,000 60% 36,000 68,000
Year 3 60,000 60% 36,000 32,000
Year 4 80,000 60% 48.000 --
The cumulative inflows exceed the total outflows between Year 3 and Year 4. The fraction of the
year can be interpolated by dividing the initial investment remaining unrecovered at the end of
Year 3 by the after-tax inflow from Year 4 ($32.000 + $48.000 = 0.6667), for a total payback
period of approximately 3.7 years.

Question #43: The CFO at a manufacturer of computer equipment learned last week that the
accounting department has not completed any bank reconciliations for the last 6 months due to
the implementation of a new accounting software package. What type of risk has been
identified?
Correct Answer: C. Operational risk
A. Financial risk
B. Hazard risk
C. Operational risk
D. Strategic risk
Explanation:
Answer (A) is incorrect.
Financial risks encompass interest-rate risk exchange-rate risk commodity risk credit risk
liquidity risk and market risk.
Answer (B) is incorrect.
Hazard risks are risks that are insurable. Examples include natural disasters, the incapacity or
death of senior officers, sabotage, and terrorism.
Answer (C) is correct.
Operational risks are the risks related to the enterprises ongoing, everyday operations.
Operational risk is the risk of loss from inadequate or failed internal processes, people, and
systems. These failures can relate to human resources (e.g., inadequate hiring or training
practices), business processes (poor internal controls), product failure (customer ill will.
lawsuits), occupational safety and health incidents, environmental damage, and business
continuity (power outages, natural disasters).
Answer (D) is incorrect.
Strategic risks include global economic risk, political risk and regulatory risk.

Question #44: Fact Pattern: Jennilyn Jasper, whose annual salary as a flight instructor is $40,000,
has just inherited $100,000 after taxes. She is considering quitting her job and opening a day-
care center. Certificates of deposit at the local bank are currently paying 6%, Jennilyn estimates
that she will have to pay $120,000 in salaries to employees per year. $20,000 to rent a building.
$9.000 each for furniture and supplies. $80,000 for insurance, and $7,000 for utilities.
Question: If Jennilyn's projections are accurate and she earns $250,000 in revenue from the
business, she will have incurred
Correct Answer: B. An accounting profit but not an economic profit.
A. Neither an accounting nor an economic profit.
B. An accounting profit but not an economic profit.
C. An economic profit but not an accounting profit.
D. Both an accounting profit and an economic profit.
Explanation:
Answer (A) is incorrect.
Jennilyn would earn an accounting profit of $5,000.
Answer (B) is correct.
An accounting profit is the excess of revenues over explicit costs, in this case ($250,000
revenue) -($120,000 salaries + $20,000 rent + $9,000 furniture + $9,000 supplies + $80,000
insurance + $7,000 utilities) = $5.000. An economic profit is a significantly higher hurdle. It is
not earned until the organization's income exceeds not only costs as recorded in the accounting
records, but the firm's implicit costs as well. In this case, the most important implicit costs are
Jennilyn's forgone salary ($40,000) and the interest she could have earned by simply investing
the inheritance instead of plowing it into the business ($100,000 x 6%). Since the combined
implicit costs of $46,000 exceed the accounting profit of $5,000. Jennilyn would incur an
accounting profit but an economic loss.
Answer (C) is incorrect
Jennilyn would earn an accounting profit but not an economic profit.
Answer (D) is incorrect.
While Jennilyn would earn an accounting profit, she would incur an economic loss.

Question #45: (Please refer the fact pattern below) Question: The marginal cost of producing the
ninth unit is
Total units of Average fixed Average variable Average total
product cost cost cost
6 $15.00 $25.00 $40.00
7 12.86 24.00 36.86
8 11.25 23.50 34.75
9 10.00 23.75 33.75

Correct Answer: C. $25.75


A. $23.50
B. $23.75
C. $25.75
D. $33.75
Explanation:
Answer (A) is incorrect.
This amount is the variable cost of the eighth unit.
Answer (B) is incorrect.
This amount is the variable cost of the ninth unit.
Answer (C) is correct.
Marginal cost is the incremental cost of producing one additional unit. Thus, the marginal cost of
the ninth unit is the increment over the total cost for eight units. The total cost for eight units at
$34.75 each is $278, and the total cost for nine units at $33.75 each is $303.75, so the total cost
for nine units is $25.75 greater than the total for eight units. This $25.75 is the marginal cost of
the ninth unit.
Answer (D) is incorrect.
This amount is the average cost per unit for nine units.

Question #46: A financial manager has discovered that her company is violating environmental
regulations. If her immediate superior is involved, the appropriate action is to
Correct Answer: C. Present the matter to the next higher managerial level.
A. Do nothing since she has a duty of loyalty to the organization.
B. Consult the audit committee.
C. Present the matter to the next higher managerial level.
D. Confront her immediate superior.
Explanation:
Answer (A) is incorrect.
Practitioners of management accounting and financial management have an obligation to the
public, their profession, the organization they serve, and themselves, to maintain the highest
standards of ethical conduct.
Answer (B) is incorrect.
The audit committee would be consulted first only if it were the next higher managerial level.
Answer (C) is correct.
To resolve an ethical problem, the financial manager/management accountant's first step is
usually to consult his or her immediate superior. If that individual is involved, the matter should
be taken to the next higher level of management.
Answer (D) is incorrect.
If the superior is involved, the next higher managerial level should be consulted first.
Question #47: If demand for a product is elastic, which one of the following would be true?
Correct Answer: A. A decrease in price would increase total revenue.
A. A decrease in price would increase total revenue.
B. An increase in price would be total revenue neutral.
C. A decrease in price would decrease total revenue.
D. An increase in price would increase total revenue.
Explanation:
Answer (A) is correct.
If the demand elasticity is greater than one demand for the product is elastic), a price decrease
will cause an increase in total revenue because the demand increases by a greater percentage than
the price decreases.
Answer (B) is incorrect.
A price increase on a product with elastic demand causes a decrease in total revenue.
Answer (C) is incorrect.
A price decrease on a product with elastic demand causes an increase in total revenue.
Answer (D) is incorrect.
A price increase on a product with elastic demand causes a decrease in total revenue.

Question #48: Which of the following is not a method for managing operational risks?
Correct Answer: C. Hedging strategies designed to reduce the risk of interest rate fluctuations.
A. Continuous reviewing of business processes.
B. Having good and functioning internal controls that are well maintained.
C. Hedging strategies designed to reduce the risk of interest rate fluctuations.
D. Regularly reviewing personnel.
Explanation:
A. Continuous reviewing of business processes is a method of managing operational risks.
B. Having good and functioning internal controls that are well maintained is a method of
managing operational risks.
C. Hedging strategies to reduce the risk of interest rate fluctuations is a method of managing
financial risks, not operational risks.
D. Regularly reviewing personnel is a method of managing operational risks.
Question #49: This year. Nelson Industries increased earnings before interest and taxes
(EBIT) by 17%. During the same period, net income after tax increased by 42%. The degree
of financial leverage that existed during the year is:
Correct Answer: B. 2.47.
A. 3.90.
B. 2.47.
C. 1.70.
D. 4.20.
Explanation:
A. This is not the correct answer. Please see the correct answer for an explanation.
B. Degree of financial leverage is calculated as the change in net income (after interest and
taxes) divided by the change in earnings before interest and taxes. This is 42% 17%, or 2.47.
C. This is not the correct answer. Please see the correct answer for an explanation.
D. This is not the correct answer. Please see the correct answer for an explanation.

Question #50: Relevant costs refer to


Correct Answer: C. Anticipated future costs that will differ among various alternative.
A. Past costs that are expected to be different in the future.
B. Costs that would be incurred within the relevant range of production.
C. Anticipated future costs that will differ among various alternatives.
D. All fixed costs.
Explanation:
A. Past costs are sunk costs and are not relevant to the decision at hand because they have
already been incurred and nothing can be done to change them.
B. Costs incurred within the relevant range of production may or may not be relevant in a
decision-making process. Relevant costs are expected future costs that differ among
alternatives. Thus some costs that are incurred within the relevant range of production may
be relevant to a decision if they would be different depending on what decision was made.
On the other hand, other costs that are incurred within the relevant range of production would
not be relevant if they would be the same no matter which decision was made.
C. Relevant costs are expected future costs that differ among alternatives. For example, if
there are two options available to a manager, the manager would consider in making the
decision only those costs that differ between the two options. Any costs that would be the
same under both options would not be considered because whichever choice is made will not
make a difference in those costs. They will be the same no matter which option is chosen,
hence they are not relevant to the decision.
D. in decision-making, relevant costs refer to those costs that vary among the options. Some
fixed costs may vary among the alternatives and would be relevant, but not all fixed costs are
relevant to a particular decision.

Question #51: Systematic risk is


Correct Answer: B. risk that cannot be diversified away by holding securities in a diversified
portfolio.
A. Risk that can be quantified.
B. Risk that cannot be diversified away by holding securities in a diversified portfolio.
C. The possibility that an investment cannot be sold (converted into cash) for its market
value.
D. Risk that can be diversified away by holding securities in a diversified portfolio.
Explanation:
A. Systematic risk cannot be quantified. (Quantify means to determine the amount of
something.)
B. Systematic risk also called market risk, is risk that cannot be diversified away. It is created
by the fact that economic cycles affect all businesses, and publicly-held investments are
traded in a market that can go up and down with economic news. Systematic, or market, risk
cannot be diversified away, and all investments are subject to it.
C. The possibility that an investment cannot be sold for its market value is liquidity risk
D. Systematic risk is not risk that can be diversified away.

Question #52: Which one of the following capital investment evaluation methods does not
take the time value of money into consideration?
Correct Answer: C. Accounting rate of return.
A. Discounted payback.
B. Internal rate of return.
C. Accounting rate of return.
D. Net present value.
Explanation:
A. The discounted payback method does take into consideration the time value of money,
because it is calculated using discounted cash flows.
B. The internal rate of return is the discount rate at which the net present value of an
investment project is zero. Therefore, it does take into consideration the time value of
money.
C. The accounting rate of return is the average annual increased accounting net income after
tax attributable to the investment divided by the net initial investment, or sometimes
divided by the average investment over the life of the project (Le.. the initial investment
amount divided by 2). The accounting rate of return does not take into consideration the
time value of money.
D. Net present value does take into consideration the time value of money, because it equals
the present value of future cash inflows minus the net initial investment.

Question #53: Last year, import of a European product costing €255 cost $242.25. This year, the
same import, which has not changed price in euros, costs $300.00. Which of the following is
true?
Correct Answer: D. The cost of the euro in U.S. dollars has increased from $0.95 to $1.176.
A. The cost of the euro in U.S. dollars has increased from $1.05 to 30.85.
B. The cost of the euro in U.S. dollars has decreased from $1.05 to $0.85.
C. The cost of the euro in U.S. dollars has decreased from $0.95 to $1.176.
D. The cost of the euro in U.S. dollars has increased from $0.95 to $1.176.
Explanation:
Notice that last year it cost fewer U.S. dollars than euros to buy the product. Therefore, the
exchange rate must be less than $1 U.S/euro. Calculating the exchange rate by dividing the U.S.
dollar amount by the euro amount, the exchange rate in the first year is $0.95 ($242.25/€255). In
the second year, it takes more U.S. dollars to purchase the goods, and the exchange rate is $1.176
($300 U.SJ €255).

Question #54: Green Company produces Product A and sells it for $18.00. The following cost
data apply:

Type of cost Per unit


Direct materials (3 lb. × $1.50) $ 4.50
Direct labor 6.45
Variable overhead 1.35
Fixed overhead 1.50
Variable selling expense 1.10
Fixed selling expense 2.20
$17.10

Green has thought of marketing a new Product B with the same cost structure as Product A
except that the price will be $15.60 Green Company currently has the plant capacity necessary
for this expansion Because of the cost structure. Green Company will find the production and
sale of Product B in the short run to be
Correct Answer: B. profitable to produce and sell Product Bin the short run at the price of
$15.60.
A. Not profitable unless the price can be raised to $17.10.
B. Profitable to produce and sell Product Bin the short run at the price of $15.60.
C. Not profitable at any price.
D. Not profitable at $15.60 because the fixed selling expense and fixed manufacturing
overhead will not be covered by the price.
Explanation:
A. It would be profitable in the short-term to produce and sell Product B at a price greater
than its variable costs $13.40.
B. Assuming the company has excess capacity the company will be better off if it is able to
sell Product B at any price above its incremental costs of $13.40 ($4.50 DM + $6.45 DL +
$1.35 variable overhead + $1.10 variable selling cost). At the price of $15.60 per unit.
Product B would contribute $2.20 per unit toward profit.
C. As long as the price for Product B is greater than its variable costs it would be profitable
in the short-term to produce and sell Product B.
D. Fixed selling and fixed manufacturing overhead do not need to be covered by the price of
Product B because they are already being covered by Product A.

Question #55: Selected financial data for Boyd Corporation are shown below.

January 1 December 31
Cash $48,000 $62,000
Accounts receivables (net) 68,000 47,000
Trading securities 42,000 35,000
Inventory 125,000 138,000
Plant & equipment (net) 325,000 424,000
Accounts payable 32,000 84,000
Accrued liabilities 14,000 11,000
Deferred taxes 15,000 9,000
Long-term bonds payable 95,000 77,000

Boyd's net income for the year was $96,000. Boyd's current ratio at the end of the year is
Correct Answer C.2.97.
A. 1.71
B. 1.56.
C. 2.97.
D. 2.71.
Explanation:
A. The current ratio is current assets divided by current liabilities. This answer results from
omitting inventory in calculating current assets and including only accounts payable as
current liabilities.
B. The current ratio is current assets divided by current liabilities. This answer results from
including deferred taxes and long-term bonds payable as current liabilities. Deferred taxes
cannot be included in either current assets or current liabilities in this question. Deferred
taxes could be an asset, or they could be a liability, if they are an asset, they could be either a
current asset or a long-term asset. If they area liability, they could be either a current liability
or a long-term liability. There is not enough information given in the question to enable us to
know what they are, so there is nothing we can do but ignore them. Long-term bonds payable
are long-term liabilities. Long-term liabilities are not included in the calculation of the
current ratio.
C. The current ratio is current assets divided by current liabilities. Trading securities are
usually classified as current assets, although they may be classified as non-current assets if
management does not consider them available to be used for current operations. Here, we
will assume they are a current asset Current assets = Cash $62,030 + A/R $ 47,000 + Trading
Securities $35,000+ Inventory $138,000 = $282,000. Current liabilities = Accounts Payable
$84,000+ Accrued Liabilities $11,000 = $95,000. The current ratio is $282,000 +$95,000 =
2.97. Deferred taxes are not included in either current assets or current liabilities in this
question. Deferred taxes could be an asset or they could be a liability, if they are an asset,
they could be either a current asset or a long-term asset. If they area liability, they could be
either a current liability or a long-term liability. There is not enough information given in the
question to enable us to know what they are, so there is nothing we can do but ignore them.
D. The current ratio is current assets divided by current liabilities. This answer results from
including deferred taxes as a current liability. Deferred taxes cannot be included in either
current assets or current liabilities in this question. Deferred taxes could be an asset, or they
could be a liability. If they are an asset, they could be either a current asset or a long-term
asset. If they area liability, they could be either a current liability or a long-term liability.
There is not enough information given in the question to enable us to know what they are, so
there is nothing we can do but ignore them.
Question #56: Katelyn is the Controller for Hobble Corners; a company that produces
manufactured homes. Hobbie Corners year-end is approaching and the forecasted profitability
report for the year is significantly lower than expected. Andrew. a senior Management
Accountant that reports to Katelyn. is responsible for forecasting the year-end profitability
reports. In an effort to lower costs, both Andrew and Katelyn have been asked to join a
committee to review operations. The committee selected several issues to address immediately in
an attempt to improve profitability. Andrew has been asked to work with the purchasing
department to select vendors that can offer raw materials at lower costs without compromising
quality. Hazel Enterprises can provide many of the key raw materials needed at a lower cost;
however, the owner of the company is related to Katelyn. After preparing a cost savings analysis.
Andrew recommends that Hazel Enterprises be selected as a new vendor. Katelyn has been asked
to review the percentage of completion calculations on the homes that are currently in process.
Katelyn has the ability to change the percentage of completion calculation which would directly
impact profitability for the company. Katelyn has modified the calculation and forwarded the
amended report to the auditors.
The IMA Statement of Ethical Professional Practice describes the overarching ethical principle
of Fairness. Identify an example of fairness relevant to the Hobbie Corners case above.
Correct Answer C Selecting vendors without bias, prejudice, or favoritism.
A. Ensuring information on reports and statements is accurate.
B. Conveying information at the appropriate time.
C. Selecting vendors without bias, prejudice, or favoritism.
D. Disclosing all necessary and relevant information to outside auditors.
Explanation:
Fairness requires empathetic, just consideration of the needs of others involved in a particular
situation and full disclosure of all necessary contextual information. The organization
deserves adequate and full disclosure that Hazel Enterprises can provide high quality
products at a lower cost however, the relationship with the controller must be disclosed.

Question #57: The risk associated with a project will increase in direct proportion to all of the
following except the:
Correct Answer D. Capital adequacy of the organization.
A. Duration of the project
B. Volatility of the cash flows associated with the project.
C. Uncertainty surrounding the impact of Federal regulation on the project.
D. Capital adequacy of the organization.
Explanation:
Answer (A) is incorrect.
Anytime uncertainty increases, risk increases. Thus, as the duration of a project or investment
increases, so does the associated risk.
Answer (B) is incorrect.
Anytime uncertainty increases, risk increases. Thus, as the volatility of a project or
investment increases, so does the associated risk.
Answer (C) is incorrect
Anytime uncertainty increases, risk increases.
Answer (D) is correct.
Capital adequacy is a term normally used in connection with financial institutions. A bank
must be able to pay those depositors that demand their money on a given day and still be able
to make new loans. Capital adequacy can be discussed in terms of solvency (the ability to
pay long-term obligations as they mature), liquidity (the ability to pay for day-to-day ongoing
operations) reserves (the specific amount a bank must have on hand to pay depositors), or
sufficient capital.

Question #58: Julie is the Senior Management Accountant for Hazelton Manufacturing: a
multi-national telecommunications company. In an effort to expand operations overseas.
Hazelton encourages senior management to offer payment to foreign officials to win
business. These payments are classified as normal commission expenses by the accounting
department Julie recently questioned whether these payments should be classified as
commission expense since they appear to be bribes to foreign officials. When Julie discussed
the situation with the Chief Financial Officer, he stated these payments were not illegal and
were expected within these foreign countries.
When all employees are hired, they are required to sign the company code of conduct. In
addition, the company provides annual ethics training to all employees and each employee is
evaluated based on compliance with operational goals and ethical expectations. The company
provides an anonymous whistleblower hotline for employees to report concerns to
management. Julie believes that the company she works for has an ethical organizational
culture.
Identify a method of monitoring ethical compliance commonly referred to as the human
performance feedback loop utilized in the Hazelton Manufacturing case.
Correct Answer C. Employees are evaluated on compliance with operational goals and
ethical expectations.
A. All employees are required to read and sign a company code of ethics.
B. A whistleblowing hotline is provided to report ethics concerns.
C. Employees are evaluated on compliance with operational goals and ethical expectations.
D. Employees complete annual ethics training.
Explanation:
Methods to monitor ethical compliance include human performance feedback loops and
survey tools. Annual employee reviews should evaluate the individual's compliance with
ethical expectations, along with operational goals.

Question #59: Universal Export wants to increase its return on equity (ROE). Which one of
the following strategies is most likely to increase the ROE?
Correct Answer C. Issue bonds, and use the money to buy back common stocks.
A. Issue common stock and use the money to increase sales.
B. Sign long-term contracts for raw materials.
C. Issue bonds, and use the money to buy back common stocks.
D. Offer salespeople commission-based compensation plans.
Explanation:
ROE can be calculated as:
ROE = net income /average equity, or
ROE = ROA x financial leverage, which can also be expanded as:
ROE = (net income / average total assets) x (average total assets / average equity)
The financial leverage component of the equation (average total assets /average equity) is
also referred to as the equity multiplier. The equity multiplier is a measure of financial
leverage (i.e., increasing ROE by substituting debt for equity). Issuing bonds and using the
funds to buy back common stock will decrease total equity without changing total assets.
Therefore. ROE will increase.

Question #60: Which one of the following provides a spontaneous source of financing for a
firm?
Correct Answer D. Accounts payable. AJI Possible Answers:
A. Debentures.
B. Accounts receivable.
C. Mortgage bonds.
D. Accounts payable.
Explanation:
A. Debentures are not a source of spontaneous financing. Spontaneous financing occurs
without special effort on the part of the borrower. The borrower must issue debentures.
B. Accounts receivable are not a source of spontaneous financing because in order to finance
through receivables, a factoring agreement must be entered into. Spontaneous financing
occurs without special effort on the part of the borrower.
C Mortgage bonds are not a source of spontaneous financing. Spontaneous financing occurs
without special effort on the part of the borrower. The borrower must issue mortgage bonds.
D. Accounts payable (or trade credit) is a spontaneous source of financing because it does not
need to be applied for before the transaction is entered into. It is created automatically at the
time of purchase.

Question #61: Which of the following is a primary difference between a cash outflow related
to the development of a new product and the expenditure made for the bulk purchase of raw
materials for existing products?
Correct Answer. C The number of accounting periods.
A. Potential profitability.
B. Contribution to working capital.
C. The number of accounting periods.
D. Effect of inflation.
Explanation:
Development of a new product exemplifies a capital investment; the bulk purchase of raw
materials is a current investment. A capital budgeting project spans more than one accounting
period whereas current investments can be written often in the same period in which the
expenses occur.

Question #62: (Please refer the fact pattern below) Question: If Richardson Motors purchases
the ten 1305 units from Simpson Castings, the capacity Richardson used to manufacture
these parts would be idle. Should Richardson decide to purchase the parts from Simpson. the
out-of-pocket cost per unit of T305 would
Richardson Motors uses 10 units of Part No. T305 each month in the production of large
diesel engines. The cost to manufacture one unit of 7305 is presented as follows:
Direct materials $2,000
Materials handling (20% of direct materials cost) 400
Direct labor 16,000
Manufacturing overhead (150% of direct labor) 24,000
Total manufacturing cost $42,400

Materials handling, which is not included in manufacturing overhead represents the direct
variable costs of the receiving department that are applied to direct materials and purchased
components on the basis of their cost Richardson's annual manufacturing overhead budget is
one-third variable and two-thirds fixed. Simpson Castings, one of Richardson's reliable
vendors, has offered to supply T305 at a unit price of $30,000.
Correct Answer C. Increase $9,600.
A. Decrease $6,400.
B. Increase $3,600.
C. Increase $9,600.
D. Decrease $12,400.
Explanation:
Answer (A) is incorrect.
Assuming all of the overhead invariable results in $6,400.
Answer (B) is incorrect.
Overlooking the $6,000 of receiving costs for purchased components results in $3,600.
Answer (C) is correct.
The out-of-pocket cost of making the part equals the total manufacturing cost minus the fixed
overhead, or $26,400 {$42,400 - [(2 ÷ 3) x $24,000]}. The cost of the component consists of
the $30,000 purchase price plus the $6,000 (20% of cost) of variable receiving costs, or a
total of $36,000. Thus, unit out-of pocket cost would increase by $9,600 if the components
were purchased.
Answer (D) is incorrect.
The difference between the full cost of making the part and the price of the component is
$12,400.

Question #63: The controller of OmniCorp asked a financial analyst to calculate common
size financial statements for the past four years. The controller is most likely looking for
which of the following?
Correct Answer C. Trends in expenses as a percentage of sales.
A. How the company is earning its profits.
B. The growth rate for sales.
C. Trends in expenses as a percentage of sales.
D. How efficiently the company is using assets.
Explanation:
Common size financial statements look at each element in the statement as a percentage of
another total amount. Common size income statements show expenses as a percent of sales,
while common size balance sheets show assets, liabilities, and equities as a percent of total
assets. A series of common size income statements will show trends in expenses as a
percentage of sales.

Question #64: In discounted cash flow techniques, which one of the following alternatives
best reflects the items to be incorporated in the initial net cash investment?
Net
proceeds
from sale of Impact of
Changes old asset in spontaneous
in net a changes in
Capitalized expenditure (e.g., shipping working replacement current
costs) capital decision liabilities
I No Yes Yes Yes
II Yes No No No
III No Yes No No
IV Yes Yes Yes Yes

Correct answer: B. IV
A. III
B. IV
C. II
D. I
Explanation:
A. Capitalized expenditures such as shipping costs are part of the cost of the fixed asset and
so they are a part of the Year 0 cash outflow. If an old asset is being replaced, then the net
proceeds from the sale of the old asset are also a part of the Year 0 cash outflow. The impact
of a spontaneous change in current liabilities is a part of the Year 0 cash flow because it is a
part of the change in net working capital. An increase in accounts payable is a spontaneous
change in current liabilities. It is called "spontaneous" because it just happens automatically.
The company does not approach its vendors and ask them if it can carry a larger balance due
to them. But when the company increases its purchases that automatically creates increased
accounts payable. An increase in accounts payable would function like a cash inflow. If
accounts payable increase when inventory, a current asset, increases, that means that the full
amount of the inventory increase has not been paid for yet. Thus the apparent cash outflow
caused by the increase in inventory is reduced by the increase in accounts payable.
B. The cash outflow for the initial investment should include all of these things. Capitalized
expenditures such as shipping costs are part of the cost of the fixed asset, and so they are a
part of the Year 0 cash outflow. Any change in net working capital (usually an increase. but
it could also be a decrease) is also a part of the Year 0 cash outflow. If an old asset is being
replaced, then the net proceeds from the sale of the old asset are also a part of the Year 0 cash
outflow. The impact of a spontaneous change in current liabilities is a part of the Year 0 cash
flow because it is a part of the change in net working capital. An increase in accounts payable
is a spontaneous change in current liabilities. It is called "spontaneous' because it just
happens automatically. The company does not approach its vendors and ask them if it can
carry a larger balance due to them. But when the company increases its purchases. that
automatically creates increased accounts payable An increase in accounts payable would
function like a cash inflow If accounts payable increase when inventory a current asset,
increases, that means that the full amount of the inventory' increase has not been paid for yet.
Thus the apparent cash outflow caused by the increase in inventory is reduced by the increase
in accounts payable.
C. Any change in net working capital (usually an increase. but it could also be a decrease) is
also a part of the Year 0 cash outflow. If an old asset is being replaced, then the net proceeds
from the sale of the old asset are also a part of the Year 0 cash outflow. The impact of a
spontaneous change in current liabilities is a part of the Year 0 cash flow because it is a part
of the change in net working capital. An increase in accounts payable is a spontaneous
change in current liabilities. It is called "spontaneous' because it just happens automatically.
The company does not approach its vendors and ask them if it can carry a larger balance due
to them. But when the company increases its purchases that automatically creates increased
accounts payable. An increase in accounts payable would function like a cash inflow. If
accounts payable increase when inventory, a current asset, increases, that means that the full
amount of the inventory increase has not been paid for yet. Thus the apparent cash outflow
caused by the increase in inventory is reduced by the increase in accounts payable.
D. Capitalized expenditures such as shipping costs are part of the cost of the fixed asset, and
so they are a part of the Year 0 cash outflow.

Question #65: Please see the question below:


Carlisle Company currently sells 400,000 bottles of perfume each year. Each bottle costs
$0.84 to produce and sells for $1.00. Fixed costs are $28,000 per year. The firm has annual
interest expense of $6,000, preferred stock dividends of $2,000 per year, and a 40% tax rate.
Carlisle uses the following formulas to determine the company's leverage.
Operating leverage = [Q (S - VC)] ÷ [Q (S - VC) – FC]
Financial leverage = EBIT ÷ {EBIT - I - [P / (1-T)]}
Total leverage = Q (S - VC) ÷ {Q (S - VC) - FC - I - [P 1/ (1 - T)]}
Where:
Q=Quantity
FC = Fixed Cost
VC =Variable Cost
S=Selling Price
I = Interest Expense
P= Preferred Dividends
T=Tax Rate
EBIT=Earnings before Interest and Taxes
The degree of financial leverage for Carlisle Company is
Correct Answer. D. 1.35
A. 1.78
B. 2.4
C. 2.3
D. 1.35
Explanation:
A. This is the degree of operating leverage, not financial leverage.
B. This is the degree of total leverage.
C. This is not the correct answer. Please see the correct answer for an explanation. We have
been unable to determine how to calculate this incorrect answer choice. If you have
calculated it please let us know how you did it soave can create a full explanation of why
this answer choice is incorrect. Please send us an email at admin@cmapass.com. Include
the Topic Name, Question Details or Screenshot and the actual incorrect answer choice -
not its letter, because that can change with every study session created. The Question ID
number appears at the top of the question. Thank you in advance for helping us to make
your CMA PASS study materials better.
D. Degree of financial leverage can be calculated as %Change in Net Income ÷ %Change in
Operating Profit (EBIT). When only one year of financial results is available. it can also
be calculated as EBIT ÷ (EBIT - Interest).When there is preferred stock the second
formula is modified as given in the problem: EMT ÷ (EBIT - I - [P/(1- T)]) Earnings
before interest and taxes equal $36,000 [$400,000 sales - ($0.84x 400,000 units) VC -
$28,000 FC]. Using the formula given in the problem, the calculation is as follows:
DFL = $36,000 ÷ [$36,000 - $6,000 - ($2,000 ÷ 0.6)]
= $36,000 ÷ $26,667
= 1.35

Question #66: The characteristics of venture capital include all of the following except
Correct Answer C. A minimum holding period of 5 years for new securities.
A. Initial private placement for the majority of issues.
B. A lack of liquidity for a period of time.
C. A minimum holding period of 5 years for new securities.
D. The use of common stock for most placements.
Explanation:
A. Venture capital placements are usually private placements, so this answer is incorrect
since we need to find the choice that is not a characteristic of venture capital.
B. Venture capital placements usually include a period of no liquidity after the
investment is made, so this answer is incorrect since we need to find the choice that is
not a characteristic of venture capital.
C. Venture capitalists do not have a minimum holding period because they will be
looking to sell their interest in the investment at a profit as soon as possible, even if it
is less than 5 years after the investment is made.
D. Venture capital placements usually use common shares, so this answer is incorrect
since we need to find the choice that is not a characteristic of venture capital.

Question #67: What is the role of gold in the present international monetary system?
Correct Answer C. Gold is like any other asset whose value depends upon supply and
demand
A. Gold is quoted in United States dollars only.
B. All of the major currencies of the world, except the United States dollar, have a fixed
value in terms of gold.
C. Gold is like any other asset whose value depends upon supply and demand.
D. Gold is the reserve asset of the International Monetary Fund.
Explanation:
Answer (A) is incorrect.
Although most exchanges quote the price of gold in U.S. dollars, the dollar value is not
linked to that of gold.
Answer (B) is incorrect.
Floating exchange rates have existed since about 1973. Tying currency values to a gold
standard, in effect, fixes exchange rates.
Answer (C) is correct.
Gold has no special role in the modern international monetary system. The present
system is based upon managed floating currency exchange rates. Consequently, gold is
treated as a commodity, the price of which depends upon supply and demand.
Answer (D) is incorrect.
The only reserves of the IMF are international currencies.

Question #68: Which one of the following actions may increase a company's return on
assets?
Correct Answer C. Replacement of capital equipment via an operating lease. All Possible
Answers:
A. Purchase of a new corporate headquarters.
B. An increase in inventory levels for a future store expansion.
C. Replacement of capital equipment via an operating lease.
D. Reduction of long-term debt through the issuance of common stock
Explanation:
Answer (A) is incorrect.
ROA is calculated by dividing a firm's net income by its average total assets. Since this
action causes total assets to increase. ROA would decrease overall.
Answer (B) is incorrect.
ROA is calculated by dividing a firm's net income by its average total assets. Since this
action causes total assets to increase. ROA would decrease overall.
Answer (C) is correct.
Return on assets, or ROA (also called return on total assets. or ROTA), is a
straightforward measure of how well management is deploying the firm's assets in the
pursuit of a profit. ROA is calculated by dividing a firm's net income by its average total
assets.
Answer (D) is incorrect.
This action would not affect ROA
Question #69: The cost-plus pricing approach is generally in what formula?
Correct Answer B. Unit cost x (1+ markup % on unit cost) = selling price.
A. Unit cost/ selling price = markup percentage.
B. Unit cost x (1+ markup %on unit cost) = selling price.
C. Variable cost + fixed cost + contribution margin - selling price.
D. Cost base + gross margin = selling price.
Explanation:
Cost-plus pricing involves setting the price of a product so as to earn a set markup
percentage on sales. The cost-plus pricing approach is expressed using the following
formula:
(Unit Cost) x (1- Markup% on unit cost) = Selling Price.

Question #70: Which of the following pricing approaches is generally used when some
level of product or service differentiation exists and a company can exercise modest
discretion in setting prices'
Correct Answer A. Cost-based pricing.
A. Cost-based pricing.
B. Peak-load pricing.
C. Target pricing
D. Market-based pricing.
Explanation:
By definition, the cost-based pricing (or cost-plus pricing) approach looks at the cost to
develop a product cc service and sets a price to recoup those costs and make a desired
profit. Cost-based pricing is appropriate when some level of product or service
differentiation exists and a company can exercise modest discretion in setting prices.

Question #71: Company A has a formal corporate code of ethics while company B does
not. The code of ethics covers such things as purchase agreements and relationships with
vendors as well as many other issues to guide individual behavior within the company.
Which of the following statements can be logically inferred?
I. Company A exhibits a higher standard of ethical behavior than does company B.
II. Company A has established objective criteria by which an employee's actions can
be evaluated.
III. The absence of a formal corporate code of ethics in company B would prevent a
successful audit of ethical behavior in that company.
Correct Answer B. II only.
A. I and II only.
B. II only.
C. III only.
D. II and III only.
Explanation:
A. The existence of a corporate code of ethics, by itself, does not ensure higher standards
of ethical behavior. It must be complemented by follow-up policies and monitoring
activities to ensure adherence to the code.
B. A formalized corporate code of ethics presents objective criteria by which actions can
be evaluated and would thus serve as criteria against which activities could be
evaluated. The existence of a corporate code of ethics, by itself, does not ensure
higher standards of ethical behavior. It must be complemented by follow-up policies
and monitoring activities to ensure adherence to the code. Standards which would
influence individual actions can occur in other places than the corporate code of
ethics. For example, there may be defined policies regarding purchasing activities that
may serve the same purpose as a code of ethics. These policies also serve as criteria
against which activities may be evaluated.
C. The absence of a formal corporate code of ethics in company B would not prevent a
successful audit of ethical behavior in that company. Standards which would
influence individual actions can be documented in places other than in a corporate
code of ethics. For example, there may be defined ponds regarding purchasing
activities that may serve the same purpose as a code of ethics. These policies also
serve as criteria against which activities may be evaluated.
D. Standards which would influence individual actions can be documented in places
other than in a corporate code of ethics. For example, there may be defined policies
regarding purchasing activities that may serve the same purpose as a code of ethics.
These policies also serve as criteria against which activities may be evaluated.

Question #72: In which product-mix pricing strategy is it appropriate for the seller to accept any
price that exceeds the storage and delivery costs for the product?
Correct Answer A. By-product pricing.
A. By-product pricing.
B. Captive-product pricing
C. Product-bundle pricing.
D. Optional-product pricing.
Explanation:
A. A by-product is a product of little value that was produced during the production process.
Any amount received above and beyond storage and delivery costs will be accepted
because it will improve profit.
B. An example of captive-product pricing would be a printer and printer ink. The ink is
called captive because it must be used along with the printer. It is often the case that the
seller will make more money on the captive product (the ink) than on the main product
(the printer). Therefore, the captive product would have to be priced above the costs of
storage and delivery.
C. An example of product-bundle pricing would be season tickets to the theater. The price of
the season ticket is lower than the price would be to purchase tickets to each production
separately. This is done to promote the sale of tickets. Al though the price of the season
ticket is lower than if the tickets were bought separately. it is high enough so the seller
still can make a profit on the sale. Therefore, the price would have to above the costs of
storage and delivery.
D. Optional-product pricing optional products, features or services that are offered along
with the main product as upgrades or options. The optional product or products have
costs, so they must be priced above their storage and delivery costs.

Question #73: Which of the following is an example of a horizontal merger?


Correct Answer D. Two or more firms who are within the same market
A. Two or more firms who work with each other in offering varying levels of the production
process.
B. Two or more firms who are in separate markets.
C. Two or more firms who work with each other as supplier and producer.
D. Two or more firms who are within the same market
Explanation:
A horizontal merger happens when tyro or more firms within the same market, also referred
to as competitors, join together. When a horizontal merger occurs, fewer competitors in the
market result; thus having the potential of leading towards a monopolistic circumstance.

Question #74: National Technology Corporation manufactures integrated computer


components. Its unit cost structure, based upon a volume of 300,000 units, is as follows.
Variable Fixed Total
Costs costs costs
Direct material $3.50 $3.50
Direct labor 9.00 9.00
Packaging 2.00 2.00
Manufacturing O/H 3.00 $6.50 9.50
Marketing Costs 2.50 8.00 10.50
Administrative costs 4.00 4.50 8.50
Total costs $24.00 $19.00 $43.00

A foreign company recently approached National with an order of 50,000 units of a specially
designed component at $35 per unit. The order will require specialized procurement costs of
$150,000 and only one-half of the variable costs associated with the administrative area will
be needed. Otherwise, cost behavior will remain the same.
Adequate capacity is available to handle this request what is the relevant unit cost to be
considered by National In making a decision on this offer?
Correct Answer C. $25.00.
A. $43.00.
B. $24.00.
C. $25.00.
D. $22.00.
Explanation:
A. The relevant unit cost will be the variable costs per unit plus the specialized procurement
costs per unit. This answer includes fixed costs as well.
B. The relevant unit cost will be the variable costs per unit plus the specialized procurement
costs per unit This answer does not include the specialized procurement costs per unit and
it also includes too much administrative costs per unit as we are told in the question that
administrative costs for this order are $2 less than usual.
C. The relevant unit cost will be the variable costs per unit plus the specialized procurement
costs per unit Variable costs per unit adjusted for the fact that only one-half of the
variable costs associated with the administrative area will be incurred, are:
Direct material $ 3.50
Direct labor 9.00
Packaging 2.00
Variable manufacturing O/H 3.00
Variable marketing costs 2.50
Variable administrative costs 2.00
Total variable costs $22.00
Specialized procurement costs will be $150,000 in total. On a per-unit basis, this equals
$3 per unit ($150 WO 50.000 units).
Therefore, the total relevant cost per unit to be considered by National in making a
decision on this offer is $22 variable costs plus $3 fixed costs, for a total of $25.
D. The relevant unit cost will be the variable costs per unit plus the specialized procurement
costs per unit This answer does not include the specialized procurement costs per unit.
Question #75: Atlantic Distributors is expanding and wants to increase its level of inventory to
support an aggressive sales target. They would like to finance this expansion using debt. Atlantic
currently has loan covenants that require the current ratio to be at least 1.2. The average cost of
the current liabilities is 12% and the cost of the long-term debt is 8%. Below is the current
balance sheet for Atlantic

Current assets $200,000 Current liabilities $165,000


Fixed assets 100,000 Long-term debt 100,000
Equity 35,000
Total assets $300,000 Total debt & equity $300,000
Which one of the following alternatives will provide the resources to expand the inventory while
lowering the total cost of debt and satisfying the loan covenant?
Correct Answer
D. Collect $25,000 accounts receivable; use $10.000 to purchase inventory and use the balance
to reduce short-term debt
A. Increase both accounts payable and inventory by $25,000.
B. Borrow short-term funds of $25,000, and purchase inventory of $25.000.
C. Sell fixed assets with a book value of $20,000 for $25,000 and use the proceeds to increase
inventory.
D. Collect $25,000 accounts receivable; use $10,000 to purchase inventory and use the balance
to reduce short-term debt
Explanation:
A. This will not provide the resources to expand the inventory while lowering the total cost
of debt and satisfying the loan covenant. Current assets would become $225,000 and
current liabilities would become $190,000. This would decrease the current ratio from
1.21 to 1.18, and the company would no longer be in compliance with its loan covenant it
would also not lower the total cost of debt because interest-incurring outstanding debt
would be unchanged.
B. This will not provide the resources to expand the inventory while lowering the total cost
of debt and satisfying the loan covenant. Current assets would become $225,000 and
current liabilities would become $190,000. This would decrease the current ratio from
1.21 to 1.18. and the company would no longer be in compliance with its loan covenant it
would also not lower the total cost of debt — in fact it would increase it — because
interest-incurring outstanding debt would increase.
C. This will not provide the resources to expand the inventory while lowering the total cost
of debt and satisfying the loan covenant. While it would increase inventory and increase
the current ratio from 1.21 to 1.36, it would not lower the cost of debt because interest-
incurring outstanding debt would be unchanged.
D. After collecting $25,000 in accounts receivable, using $10,000 of it to purchase inventory
and the balance to reduce short-term debt the current ratio would become 1.23, which
would be in compliance with the loan covenant
Collection of $25,000 in accounts receivable would not change current assets, because
the collection would increase cash by the same amount that that accounts receivable yeas
reduced. Use of $10,000 cash to purchase inventory would also not change current assets,
because cash would be reduced and inventory would be increased by the same amount.
However, use of the remaining $15,000 in cash to reduce short-term debt would reduce
current assets and current liabilities both by $15,000. Therefore current assets would
become $185,000 and current liabilities would become $150,000, resulting in a current
ratio of 1.23.
The question asks. "Which one of the following alternatives will provide the resources to
expand the inventory while lowering the total cost of debt and satisfying the loan
covenant? This will provide the resources to increase inventory by $10,000 while
lowering short-term debt by $15,000, which will mean lower interest cost. Furthermore,
the current ratio remains above 1.2, which satisfies the loan covenant.

Question #76: A corporation has $90 million in current assets. If the corporation has a
current ratio of 1.2 and a quick ratio of 0.9, what is net working capital?
Correct answer: B. $15 million.
A. $10 million.
B. $15 million.
C. $81 million.
D. $108 million.
Explanation:
Answer (A) is incorrect.
Current assets, not quick assets, are given, so the current, not quick ratio should be used
for finding current liabilities. Also, net working capital is current assets minus current
liabilities, not the other way around.
Answer (B) is correct.
Networking capital is Current assets - Current liabilities. The current ratio is Current
assets ÷ Current liabilities. Current liabilities can be found as follows:
Current ratio = Current assets ÷ Current liabilities
1.2 = $90 million ÷ Current liabilities
Current liabilities = $90 million ÷ 1.2
= $75 million
Therefore, net working capital is $15 million ($90 million - $75 million).
Answer (C) is incorrect
Multiplying current assets by the quick ratio does not give net working capital.
Answer (D) is incorrect.
Multiplying current assets by the current ratio does not give net working capital.

Question #77: A firm can mitigate the risk of financial loss from the possible on-the-job
injury of one of its employees through
Correct Answer D. Liability insurance.
A. Hazard insurance.
B. Workers' compensation insurance.
C. Key employee insurance.
D. Liability insurance.
Explanation:
Answer (A) is incorrect.
Hazard insurance is the same as homeowners or automobile driver's insurance. It protects
the organization against damage caused to its facilities by accident or natural disaster.
Answer (B) is incorrect.
Workers' compensation insurance benefits the injured worker, not the organization.
Answer (C) is incorrect
Key employee insurance benefits the organization only in case of the death of a critical
member of upper management.
Answer (D) is correct.
Liability insurance provides an organization with financial protection against damage
caused to consumers by faulty products or injury to persons suffered on the organization's
premises.

Question #78: Which of the following is not a common application for cost/volume/profit
(CVP) analysis?
Correct Answer B. Engaging in capital rationing.
A. Deciding whether to replace a piece of equipment
B. Engaging in capital rationing.
C. Pricing for a new product.
D. Deciding whether to outsource.
Explanation:
Cost/ Volume/ profit analysis data can be used in a wide variety of decision-making
situations such as raising or lowering existing prices, introducing a new product or
service, setting prices for new products and services, expanding product and service
markets, deciding whether to replace an existing piece of equipment, and deciding
whether to make or buy a product or service.
Question #79: Which statement below best indicates the order of the components that
should be implemented in a new Enterprise Risk Management program?
Correct Answer C. Set risk management strategy and objectives, identify risks, and assess
risks.
A. Communicate and monitor risks, identify risks, and treat risks.
B. Assess risks, control risks, and treat risks.
C. Set risk management strategy and objectives, identify risks, and assess risks.
D. Control risks, set risk management strategy and objectives, and monitor risks.
Explanation:
A. Information and communication involves communicating all the relevant information
that needs to be communicated to the appropriate people, within a time frame that
will allow them to carry out their duties. Information and communication cannot be
implemented for an ERM until the company has identified the risks, assessed the
risks, and decided how to respond to each identified risk and the system has been put
into place.
B. Control activities are developed after the company has determined how it will
respond to each risk that has been identified and assessed.
C. The components of an Enterprise Risk Management program, in order of their
implementation are: -Objective setting. The organization's strategic objectives and
goals for its operations, reporting and compliance activities must be determined and
established. -Event identification. Events are the internal and external events that
affect the organization's implementation of its ERM strategy or the achievement of its
objectives. -Risk assessment. Risk assessment is the process of analyzing and
considering risks from three perspectives: (1) the likelihood of the risk's occurring.
(2) the potential impact of the event if it does occur, and (3) the interrelationship of
the risks on a unit-by-unit or total organization basis. -Risk response. Risk response is
what the company decides to do with respect to each of the risks identified.
Management must develop a response for each of its identified risks. -Control
activities. Control activities are the policies and procedures implemented to ensure the
risk responses are carried out. -Information and communication. This includes all the
relevant information that needs to be communicated to the appropriate people, within
a time frame that will allow them to carry out their duties. -Monitoring. The system
put in place needs to be monitored to ensure it continues to be appropriate and
continues to be properly operated.
D. The organization's strategic objectives and goals for its operations, reporting and
compliance activities must be determined and established before control activities or
monitoring can take place.

Question #80: (please refer the fact pattern below) Question: Assuming a 360-day year and that
CyberAge continues paying on the last day of the credit period, the company's weighted-average
annual interest rate for trade credit (ignoring the effects of compounding) for these two vendors
is
(Fact pattern] CyberAge Outlet, a relatively new store, is a cafe that often customers the
opportunity to browse the Internet a play computer games at their tablet motile they drink coffee.
The customer pap a fee based on the amount of time spent signed as to the computer. The store
also sells toots, tee-shirts, and computer accessories. CyberAge has been paying all of its bills on
the last day of the payment period, thus forfeiting all supplier discounts. Shown below are data
on CyberAge's two major vendors, including average monthly purchases and add terms.
Average
monthly
Vendor Purchases Credit Terms
Web master $25,000 2/10, net 30
Softidee 50,000 5/10, net 90

Correct Answer B. 25.2%


A. 27.0%
B. 25.2%
C. 28.0%
D. 30.2%
Explanation:
Answer (A) is incorrect.
This percentage is based on weights of $25,000 and $50,000.
Answer (B) is correct.
The annualized costs of not taking the discounts offered by these two vendors can be calculated
as follows:
(Discount96/ (100%- Discount %)) × (Days in year / (Total payment period - Discount Period))
Web Master:
Cost of not taking discount = [2% + (100% - 2%)) x (360 days + (30 days -10 days)]
= (2% ÷ 98%) x (360days ÷ 20days)
= 2.0408% x 18
= 36.73%
Softidee:
Cost of not taking discount = [5% ÷ (100% - 5%)] x (360 days ÷ (90 days -10 days)]
= (5% ÷ 95%) x (360days ÷ 80days)
= 5.2632% x 4.5
= 23.68%
The average amount borrowed from Web Master is $16,333.33 [$24,500 x 1 month x (20days ÷
30 days)], and the average amount borrowed from Softidee is $126,666.67 [$47,500 x 3 months
x (80days ÷ 90days)]. Thus, the weighted average of these two rates based on average
borrowings is 25.2% [[($16,333.33 x 36.72%) + ($126,666.67 x 23.67%)] ÷ ($16,333.33+
$126,666.67)). This calculation, however, understates the true cost of not taking the discount
because it does not consider the effects of compounding.
Answer (C) is incorrect
This percentage is based on weights of $24,500 and $47,500.
Answer (D) is incorrect.
This percentage is an unweighted average of the two interest rates.

Question #81: The U.S. Foreign Corrupt Practices Act is particularly focused on the dealings of
financial institutions and the safeguarding of the global financial system. Financial institutions
must implement robust controls to ensure knowledge of their customers and the nature of their
business transactions and be in a position to prove to regulators a high level of due diligence.
These safeguards are required to minimize all of the following except
Correct Answer B. Insider trading.
A. Money laundering.
B. Insider trading.
C. Terrorist financing.
D. Extortion and bribery.
Explanation:
Answer (A) is incorrect.
Money laundering is one focus of the safeguards of the global financial system relating to the
US. Foreign Corrupt Practices Act.
Answer (B) is correct.
The safeguards of the global financial system relating to the U.S. Foreign Corrupt Practices
Act deal with minimizing money laundering, terrorist financing and extortion and bribery.
Insider trading is not a focus of the safeguards.
Answer (C) is incorrect
Terrorist financing is one focus of the safeguards of the global financial system relating to
the US. Foreign Corrupt Practices Act.
Answer (D) is incorrect.
Extortion and bribery are focuses of the safeguards of the global financial system relating to
the U.S. Foreign Corrupt Practices Act

Question #82: The COSO ERM framework includes eight interrelated components. Which
component in the framework implements policies and procedures to help ensure that risk
responses are effectively carried out?
Correct Answer B. Control activities.
A. Risk assessment
B. Control activities.
C. Objective setting.
D. Event identification.
Explanation:
Control activities include policies and procedures that are established and implemented to help
ensure that the risk responses are effectively carried out The COSO model lists six control
activities: the assignment of authority and responsibility, a system of transaction authorizations.
Adequate documentation and records, security of assets, independent verifications, and adequate
separation of duties.

Question #83: Katelyn is the Controller for Hobbie Corners; a company that produces
manufactured homes. Robbie Corners year-end is approaching and the forecasted profitability
report for the year is significantly lower than expected. Andrew. a senior Management
Accountant that reports to Katelyn, is responsible for forecasting the year-end profitability
reports. In an effort to lower costs, both Andrew and Katelyn have been asked to join a
committee to review operations. The committee selected several issues to address immediately in
an attempt to improve profitability.
Andrew has been asked to work with the purchasing department to select vendors that can offer
raw materials at lower costs without compromising quality. Hazel Enterprises can provide many
of the key raw materials needed at a lower cost: however, the owner of the company is related to
Katelyn. After preparing a cost savings analysis. Andrew recommends that Hazel Enterprises be
selected as a new vendor. Katelyn has been asked to review the percentage of completion
calculations on the homes that are currently in process. Katelyn has the ability to change the
percentage of completion calculation which would directly impact profitability for the company.
Katelyn has modified the calculation and forwarded the amended report to the auditors.
The IMA Statement of Ethical Professional Practice describes the overarching ethical principle
of Responsibility. Identify an example of responsibility relevant to the Hobbie Corners case
above.
Correct Answer C Ensuring information on reports and statements is accurate.
A. Disclosing all necessary and relevant information to outside auditors.
B. Providing information and feedback objectively.
C. Ensuring information on reports and statements is accurate.
D. Selecting vendors without bias, prejudice, or favoritism.
Explanation:
Responsibility requires actions to be performed with faithfulness and loyalty. Examples of
responsibility include conveying information at the appropriate time, ensuring information on
reports and statements is accurate, and gathering enough information to make an informed
decision. Katelyn has a responsibility to ensure that the information on the percentage of
completion report is accurate.

Question #84: Time drafts that result from commercial trade financing are:
\ Correct Answer C. Bankers' acceptances.
A. Federal trade securities.
B. Negotiable certificates of deposits.
C. Bankers' acceptances.
D. Commercial paper.
Explanation:
By definition, bankers' acceptances (BAs) are time drafts that result from commercial trade
financing. BAs involve a letter of credit accepted by a bank and typically imply that the BA
is backed by that bank.
Question #85: Company ABC is considering merging with Company XYZ. After analyzing
both companies, it is determined that the incremental after-tax free cash flow resulting from
the merger is estimated to be $3,500,000 and is expected to last for 20 years. Assuming a
required rate of return of 12% for the acquired company, using the discounted cash flow
method, what is the maximum amount Company ABC should offer to purchase Company
XYZ for?
Correct Answer D. $26,141,500.
A. $26,236,000.
B. $26,757,500.
C. $33,761,000.
D. $26,141,500.
Explanation:
Using the discounted cash flow method of valuation, the present value of the merger benefits
would be the S3.500.000 after-tax free cash flow multiplied by the Present Value of Annuity
Factor of 7.469, which equals 526.141,500. The price offered by the buyer should be less
than or equal to $26,141,500.

Question #86: The technique that recognizes the time value of money by discounting the
after-tax cash flows for a project over its life to time period zero using the company's
minimum desired rate of return is called the
Correct Answer C. Net present value method.
A. Accounting rate of return method.
B. Average rate of return method.
C. Net present value method.
D. Payback method.
Explanation:
A. The accounting rate of return is the ratio of the amount of increased book income to the
required investment. Since this method uses accrual accounting income, it includes
depreciation. However, it does not take into account the time value of money.
B. The average rate of return is not a technique that recognizes the time value of money by
discounting the after-tax cash flows for a project.
C. Net present value of a project is calculated by discounting the after-tax expected cash
flows for the project over its life to time period zero using the company’s minimum
required rate of return. The present value of the future expected cash inflows minus the
net initial investment equals the net present value.
D. The payback method explicitly does not recognize the time value of money.
Question #87: A business is showing an increase in receivables without a matching increase in
sales. This may indicate that the company has:
Correct Answer D. problems with collection of receivables.
A. An increased number of customers.
B. Become more efficient at delivering its products.
C. Decreased its terms of credit
D. Problems with collection of receivables.
Explanation:
An increase in receivables without a matching increase in sales indicates that there may be a
problem with collections. Since sales have not increased, there is no indication that there is
an increase in customers. There has been no indication given that the company has increased
its terms of credit or has become more efficient at delivering its products.

Question #88: What are the four standards of ethical conduct as expressed in the I MA Statement
of Ethical Professional Practice (2005)?
Correct Answer D. Competence. Confidentiality. Integrity and Credibility
A. Honesty, Fairness, Objectivity and Responsibility
B. Fairness, Confidentiality, Integrity and Credibility
C. Competence, Confidentiality, Objectivity and Responsibility
D. Competence, Confidentiality, Integrity and Credibility
Explanation:
A. These are overarching ethical principles.
B. Three of these are standards of ethical conduct but one is an overarching ethical principle
C. Two of these are standards of ethical conduct, but two are overarching ethical principles
D. The four standards of ethical conduct are Competence. Confidentiality. Integrity and
Credibility.

Question #89: Which one of the following is not a shortcoming of the payback method?
Correct Answer C. It offers no indication of a projects liquidity.
A. It ignores the time value of money.
B. It offers no consideration of cash flows beyond the expiration of the payback period.
C. It offers no indication of a projects liquidity.
D. It encourages establishing a short payback period.
Explanation:
It offers no indication of a projects liquidity. The payback method in capital budgeting
determines the number of years needed to recoup the net initial investment in a capital
budgeting project. It focuses on how fast the initial investment can be recovered (liquidity).
Payback ignores the time value of money the life of the project, and promotes short-term
thinking.

Question #90: Keller Products needs $150,000 of additional funds over the next year in order to
satisfy a significant increase in demand. A commercial bank has offered Keller a one-year loan at
a nominal rate of 8%, which requires a 15% compensating balance. How much would Keller
have to borrow, assuming it would need to cover the compensating balance with the loan
proceeds?
Correct Answer C. $176,471
A. $130,435.
B. $194,805.
C. $176,471.
D. $172,500.
Explanation:
In order to cover the loan needs and the compensating balance. Keller would have to borrow
Amount to borrow = ($150,000)/ (1- 0.15 compensating balance need)
Amount to borrow = ($150,000)/ (0.85)
Amount to borrow = $176,470.58, or roughly $176,471.

Question #91: A company makes a product that sells for $30. During the coming year, fixed
costs are expected to be $180,000, and variable costs are estimated at $26 per unit. How many
units must the company sell to break even?
Correct Answer C, 45,000 units.
A. 6,000 units.
B. 6,924 units.
C. 45,000 units.
D. 720,000 units.
Explanation:
Answer (A) is incorrect.
This number of units will cover the fixed costs for the year but not the variable costs
($180,000 ÷ $30 = 6,000).
Answer (B) is incorrect.
Dividing the total fixed costs by the variable cost per unit rather than the contribution margin
per unit results in 6,924 units.
Answer (C) is correct.
The contribution margin per unit is $4 ($30 selling price - $26 unit variable cost). Fixed costs
of $180,000 divided by the contribution of $4 per unit gives breakeven sales volume of
45,000 units.
Answer (D) is incorrect.
The total fixed costs should be divided (not multiplied) by the unit contribution margin.

Question #92: The owner of Woofie's Video Rental cannot decide how to project the real costs of
opening a rental store in a new shopping mall. The owner knows the capital investment required
but is not sure of the returns from a store in a new mall. Historically, the video rental industry
has had an inflation rate equal to the economic norm. The owner requires a real internal rate of
return of 10%. Inflation is expected to be 3% during the next few years. The industry expects a
new store to show a growth rate, without inflation. of 8%. First year revenues at the new store
are expected to be $400,000. The revenues for the second year, using both the real rate approach
and the nominal rate approach, respectively, would be:
Correct Answer D. $432,000 real and $444,960 nominal.
A. $440,000 real and $453,200 nominal.
B. $432,000 real and $452,000 nominal.
C. $440,000 real and $452,000 nominal.
D. $432,000 real and $444,960 nominal.
Explanation:
The real revenues for the second year are calculated as:
Real revenue. Year 2 = (Year 1 revenues) (1 + growth rate)
Real revenues. Year 2 = ($400,000) (1 +0.08) = $432,000
The nominal revenues for the second year are calculated as:
Nominal revenues. Year 2 = (real revenues) (1 + inflation rate)
Nominal revenues. Year 2 = ($432,000) (1 +0.03) = $444,960.

Question #93: The principal advantage of using commercial paper as a short-term financing
instrument is that it
Correct Answer C. Is usually cheaper than a commercial bank loan.
A. Is readily available to almost all companies.
B. Offers security. i.e., collateral, to the lender.
C. Is usually cheaper than a commercial bank loan.
D. Can be purchased without commission costs.
Explanation:
A. Commercial paper is an unsecured note that is able to be issued only by the largest, most
credit-worthy companies. Therefore, it is not a source of financing for most companies
B. Commercial paper is an unsecured note so there is no collateral available to the investor
in the case of default by the issuer.
C. Commercial paper is an unsecured note that is issued only by the largest, most stable and
credit-worthy companies. Commercial paper is sold at a discount and usually is very
short-term. Usually, commercial paper is cheaper than a bank loan because the nature of
the companies that are able to issue commercial paper greatly reduces the risk to the
investor.
D. Generally a commission is charged on commercial paper just as on other instruments.

Question #94: Capital Company has decided to discontinue a product produced on a machine
purchased four years ago at a cost of $70,000. The machine has a current book value of
$30,000. Due to technologically improved machinery now available in the marketplace the
existing machine has no current salvage value. The company is reviewing the various aspects
involved in the production of a new product. The engineering staff advised that the existing
machine can be used to produce the new product. Other costs involved in the production of
the new product will be materials of $20,000 and labor priced at 45,000. Ignoring income
taxes, the costs relevant to the decision to produce or not to produce the new product would
be:
Correct Answer D. $25,000.
A. $95,000.
B. $55,000.
C. $30,000.
D. $25,000.
Explanation:
The costs relevant to the decision to produce or not to produce the new product total $25,000,
and are comprised of the $20,000 cost of materials and the $5,000 cost of labor. The costs
associated with the old machine are irrelevant: they are sunk historical costs.
Question #95: Colonie Inc. expects to report net income of at least $10 million annually for
the foreseeable future. Colonie could increase its return on equity by taking which of the
following actions with respect to its inventory turnover and the use of equity financing?
Inventory Turnover Use of Equity Financing:
Correct Answer B. Inventory Turnover increase: Use of Equity Financing: Decrease.
A. Inventory Turnover: Decrease: Use of Equity Financing: Decrease.
B. Inventory Turnover: increase: Use of Equity Financing: Decrease.
C. Inventory Turnover: increase: Use of Equity Financing Increase.
D. Inventory Turnover: Decrease: Use of Equity Financing: Increase.
Explanation: To increase its return on equity (net income divided by average common
shareholders' equity. Colonie could increase its inventory turnover by liquidating inventory
and increasing net income. Colonie could also increase the return by decreasing the use of
equity financing. The decrease would lower the denominator in the formula without affecting
the numerator.
Question #96: Freely fluctuating exchange rates perform which of the following functions?
Correct Answer A. They automatically correct a lack of equilibrium in the balance of
payments.
A. They automatically correct a lack of equilibrium in the balance of payments.
B. They make imports cheaper and exports more expensive.
C. They impose constraints on the domestic economy.
D. They eliminate the need for foreign currency hedging.
Explanation:
Answer (A) is correct.
In a freely floating exchange rate system, the government steps aside and allows exchange
rates to be determined entirely by the market forces of supply and demand. The advantage of
such a system is that it tends to automatically correct any dis-equilibrium in the balance of
payments. The disadvantage is that a freely floating system makes a country vulnerable to
economic conditions in other countries.
Answer (B) is incorrect.
Forcing imports to be cheaper and exports more expensive can be accomplished with a fixed
exchange rate system.
Answer (C) is incorrect
Freely floating exchange rates impose no constraints on the domestic economy.
Answer (D) is incorrect.
A freely floating exchange rate system does not eliminate transaction risk (which must be
hedged).

Question #97: The expected maximum probable loss to a commercial building is


Correct Answer D. inversely related to the size of the building.
A. Directly related to the size of the building
B. Lower if the building is vacant
C. The complete destruction of the building and all of its contents.
D. Inversely related to the size of the building.
Explanation:
A. The maximum probable loss (also called the probable maximum loss, or PML) is the
largest loss that can occur under foreseeable circumstances. This is the largest amount of
damage that is likely to occur in a very bad year. When we say that something is directly
related to something else, it means they move in the same direction. When one thing
increases, the other thing also increases: and when one thing decreases, the other thing
also decreases. It is not accurate to say that the expected maximum probable loss to a
commercial building increases as the size of the building increases
The larger the building is, the less probability there is that the maximum probable loss
will occur. So even though the amount of the maximum probable loss may be greater for
a large building than it is fora smaller building, the probability of that amount of loss
occurring is much smaller fora large building than it is for a small building. When the
probability of the maximum probable loss occurring is multiplied by the amount of the
maximum probable loss, the expected maximum probable loss will generally be smaller
for a large building than fora small building.
B. The expected maximum probable loss increases if the building is vacant. A vacant
building is more vulnerable to destruction than a building that is occupied. When
occupants are in the building, they are aware when something happens that could cause
damage, and they do something about it For instance, if a fire starts in a building that is
occupied, the people get out and call the fire department But if a fire starts in a vacant
building the fi re could burn for a very longtime and cause a lot of damage before anyone
would notice and call the fi re department.
C. The complete destruction of the building and all of its contents is the definition of the
maximum possible loss, not the definition of the maximum probable loss. The definition
of the maximum probable loss is the largest loss that can occur under foreseeable
circumstances, or the largest amount of damage that is likely to occur in a very bad year.
Damage greater than the maximum probable loss could occur, but in the judgment of
management, it is very unlikely to occur.
D. The maximum probable loss (also called the probable maximum loss, or PML) is the
largest loss that can occur under foreseeable circumstances. This is the largest amount of
damage that is likely to occur in a very bad year. The expected maximum probable loss is
inversely related to the size of the building. When we say that something is inversely
related to something else, it means that the two things move in opposite directions. When
one thing increases, the other thing decreases, and when one thing decreases, the other
thing increases. The larger a building is, the less probability there is that the largest loss
that can occur under foreseeable circumstances will occur. So as the size of the building
increase, the probability of maximum probable loss decreases. Even though the amount
of the maximum probable loss may be greater for a large building than it is for a smaller
building, the probability of that amount of loss occurring is much smaller for a large
building than it is for a small building When the probability of the maximum probable
loss occurring is multiplied by the amount of the maximum probable loss, the expected
maximum probable loss will generally be smaller for a large building than fora small
building.

Question #98: Profits that are lost by moving an input from one use to another are referred to as
Correct Answer C. opportunity costs.
A. Replacement costs.
B. Cannibalization charges.
C. Opportunity costs.
D. Out-of-pocket costs.
Explanation:
A. Replacement costs are explicit future costs required to replace something used today.
Profits lost by moving an input from one use to another are implicit costs.
B. Cannibalization charges are related to competing products produced by the same
company. When one product "cannibalizes' another product. it takes sales away from the
other product Usually the product that cannibalizes the sales is a new product that takes
sales away from an existing product produced by the same company. However, this
question relates to profits sacrificed by not producing a product, not profits lost because
sales are taken away by another, competing product produced by the same company.
C. An opportunity cost is the contribution to income that is lost by not using a limited
resource in its best alternative use. Profits lost by moving an input from one use to
another is an example of an opportunity cost.
D. Out-of-pocket costs are actual, explicit costs that are paid directly. Profits lost by moving
an input from one use to another are implicit costs because there is no real expense paid
out The cost is the loss of an opportunity to earn a profit.
Question #99: Employee A observes that Employee B is improperly altering department records
to meet month end goals. These records are for internal use only and do not impact the
company’s financial records. Employee A notifies her supervisor of the impropriety, and the
supervisor advises Employee A that she instructed Employee B to alter the records and an
adjustment would be made the subsequent month to correct the records. Employee A should
Correct Answer A. follow the organization's established procedures on the resolution of such
conflict.
A. Follow the organization's established procedures on the resolution of such conflict.
B. Do nothing since the supervisor authorized the behavior.
C. Communicate the unethical behavior to external authorities.
D. Advise the supervisor that her behavior was unethical and do not communicate the
impropriety any further.
Explanation:
A. According to Resolution of Ethical Conflict" in the IMA's Statement of Ethical
Professional Practice, when faced with ethical issues, management accountants should
first follow their organization's established policies on the resolution of such conflict
B. This action does not conform to the guidance in 'Resolution of Ethical Conflict" in the
IMAs Statement of Ethical Professional Practice.
C. According to 'Resolution of Ethical Conflict in the IMAs Statement of Ethical
Professional Practice, communication of such problems to authorities or individuals not
employed or engaged by the organization is not considered appropriate, unless you
believe there is a clear violation of the law.
D. This action does not conform to the guidance in 'Resolution of Ethical Conflict in the
IMAs Statement of Ethical Professional Practice.

Question #100: Which of the outcomes represented in the following table would result from a
company's retirement of debt with excess cash?
Following Period's
Correct Answer A. Increase (Total Assets Turnover Ratio). (Times Interest Earned Ratio)
Increase
A. Increase (Total Assets Turnover Ratio). ( Times Interest Earned Ratio) Increase
B. Increase (Total Assets Turnover Ratio). ( Times Interest Earned Ratio) Decrease
C. Decrease (Total Assets Turnover Ratio), (Times Interest Earned Ratio) Increase
D. Decrease (Total Assets Turnover Ratio), (Times Interest Earned Ratio) Decrease
Explanation:
Answer (A) is correct.
Because total assets will decline without any impact on sales, the total assets turnover ratio
(sales =total assets) will increase. In addition, a reduced debt level should cause a reduction
in annual interest payments, so the times interest earned ratio ((net income + interest + taxes)
+ interest) should increase.
Answer (B) is incorrect
The times interest earned ratio will increase and the total assets turnover ratio will increase.
Answer (C) is incorrect
The times interest earned ratio will increase and the total assets turnover ratio will increase.
Answer (D) is incorrect.
The times interest earned ratio will increase and the total assets turnover ratio will increase.
MOCK TEST 5

Question #1: Jasper Company has a payback goal of 3 years on new equipment acquisitions.
A new sorter is being evaluated that costs $450,000 and has a 5-year life. Straight-line
depreciation will be used; no salvage is anticipated. Jasper is subject to a 40% income tax
rate. To meet the company's payback goal, the sorter must generate reductions in annual cash
operating costs of
Correct Answer: A. $190,000.
A. $190,000.
B. $150,000.
C. $114,000.
D. $100,000.
Explanation:
A. The payback period is the number of years it takes for the initial investment in a project
to be returned, without considering the time value of money. The new equipment will
cost $450,000, and the company needs that amount returned within three years. The
depreciation tax shield will be $36,000 per year ($450,000 + 5 x 0.40). Therefore, in three
years. $108.000 of the investment will have been returned by means of the depreciation
tax shield, leaving $342,000 to be returned in increased cash flow after taxes. Jasper's tax
rate is 40%. Therefore, the before-tax increased cash flow over the three year payback
period needs to be $342,000 + (1 - 0.40), or $570,000. The annual increase in cash flow
needed, then, is $570,000 ÷ 3, or $190,000.
B. $150,000 is the net after-tax cash flow increase required each year to achieve a payback
period of 3 years on a $450,000 investment. However, that is not what the question asks.
C. $114,000 is the amount of increased cash flow from operations net of tax that is required.
However, it is not the amount that the sorter must generate in annual reductions of cash
operating costs in order to achieve a three-year payback.
D. $100,000 in annual before-tax cash flow would equal $60,000 in annual after-tax cash
flow. When $60,000 is added to the depreciation tax shield of $36,000 per year, the
resulting $96,000 of annual net cash flow increase would not achieve a payback period of
3 years.

Question #2: A company has sales of $100,000, cost of sales of $40,000, interest expense of
$4,000, taxes of $18,000, and operating expenses of $15,000. What is the company's operating
profit margin?
Correct Answer: • 45%.
A. 60%
B. 45%
C. 41%
D. 23%
Explanation:
Answer (A) is incorrect.
Operating profit margin is equal to operating income divided by net sales. Operating income
includes COGS and operating expenses but not interest or taxes. The amount of 60% fails to
subtract the operating expenses of $15,000 from the net sales in order to calculate the correct
operating income of $45,000.
Answer (B) is correct.
Operating profit margin is equal to operating income divided by net sales. Operating income
includes COGS and operating expenses but not interest or taxes. Thus, the company's operating
income is equal to $45,000 ($100,000 sales - $40,000 COGS - $15,000 operating expenses). The
amount of $45,000 divided by $100,000 of net sales results in an operating profit margin of 45%.
Answer (C) is incorrect.
Operating profit margin is equal to operating income divided by net sales. Operating income
includes COGS and operating expenses but not interest or taxes. The amount of 41% of 41%
incorrectly includes the interest expense in the operating income calculation.
Answer (D) is incorrect.
Operating profit margin is equal to operating income divided by net sales. Operating income
includes COGS and operating expenses but not interest or taxes. The amount of 23% incorrectly
includes the interest expense and the taxes in the operating income calculation.
Question #3: Consider the following graph:

Correct Answer: B.12%


A. It is impossible to determine the return to the market from the information given.
B. 12%
C. 3%
D. 9%
Explanation:
A. It is not impossible to determine the return to the market from the graph.
B. The return to the market is the market return at the point where beta is 1.0.0n a graph of
the Security Market Line, betas are on the horizontal axis and expected returns are on the
vertical axis. At the point where beta is 1.0, the expected return is 12%.
C. 3% is the risk-free rate, according to this graph. The risk-free rate is the point where the
Security Market Line intersects the Y axis, where beta is zero.
D. 9% is the market risk premium, according to this graph. The market risk premium is the
difference between the risk-free rate and the return to the market.

Question #4: Which one of the following is not considered a key step in the risk management
process?
Correct Answer B. Reconsider current risks.
A. Prioritize risks.
B. Reconsider current risks.
C. Formulate risk responses.
D. Assess risks.
Explanation:
Answer (A) is incorrect.
Prioritizing risks is step 3. In large and/or complex organizations, top management may appoint
an ERM committee to review the risks identified by the various operating units and bring them
together in a coherent response plan.
Answer (B) is correct.
"Current" risks should not be considered in isolation. A risk management system must take a
broad view of the risks facing the organization.
Answer (C) is incorrect
Formulating risk responses is step 4. The ERM committee proposes adequate response strategies.
Answer (D) is incorrect.
Assessing risks is step 2. Every risk identified must be assessed as to its probability and potential
impact.

Question #5: A firm with a higher degree of operating leverage when compared to the industry
average implies that the
Correct Answer B. Firm's profits are more sensitive to changes in sales volume
A. Firm has higher variable costs.
B. Firm's profits are more sensitive to changes in sales volume
C. Firm is more profitable.
D. Firm is less risky.
Explanation:
Answer (A) is incorrect.
A firm with higher operating leverage has higher fixed costs and lower variable costs.
Answer (B) is correct.
Operating leverage is a measure of the degree to which fixed costs are used in the production
process. A company with a higher percentage of fixed costs (higher operating leverage) has
greater risk than one in the same industry that relies more heavily on variable costs.
However, such a firm is also able to expand production rapidly in times of higher product
demand. Thus, the more leveraged a firm is in its operations, the more sensitive operating
income is to changes in sales volume.
Answer (C) is incorrect
A firm with higher leverage will be relatively more profitable than a firm with lower leverage
when sales are high. The opposite is true when sales are low.
Answer (D) is incorrect
A firm with higher leverage is more risky. Its reliance on fixed costs is greater.

Question #6: Question: Dividendosaurus has a profit margin of


[Fact Pattern #44] The financial statements for Dividendosaurus, Inc., for the current year are
as follows:
Balance sheet

Cash $100
Accounts receivable 200
Inventory 50
Net fixed assets 600
Total $950
Accounts payable $140
Long-term debt 300
Capital stock 260
Retained earnings 250
Total $950

Sales $3,000
Cost of goods sold (1,600)
Gross profit $1,400
Operation expenses (970)
Operating income $430
Interest expense (30)
Income before tax $400
Income tax (200)
Net income $200
Add: Jan. 1 retained earnings 150
Less: dividends (100)
Dec. 31 retained earnings $250

Correct Answer • 6.67%


A. 6.67%
B. 13.33%
C. 14.33%
D. 46.67%
Explanation:
Answer (A) is correct
The profit margin is the ratio of net income to sales. For Dividendosaurus, it equals 6.67%
($200 net income ÷ $3,000 sales).
Answer (B) is incorrect
The ratio of income before tax to sales is 13.33%.
Answer (C) is incorrect
The ratio of income before interest and taxes to sales is 14.33%.
Answer (D) is incorrect
The ratio of gross profit to sales is 46.67%.

Question#7: The schedule below shows the demand and elasticity coefficients for computers.
Quality demanded (per Elasticity
week) Unit price Coefficient
140 900 0.8
120 1,100 0.9
100 1,200 1.0
80 1,300 1.1
60 1,400 1.3
40 1,500 2.0
A computer manufacturer has experienced a weekly demand of 100 computers, at a price of
$1,200 per computer, Weekly revenues, therefore have been $120,000. However, at that
price, the manufacturer has been experiencing a loss. At what price will the manufacturer see
the highest revenue?
Correct answer: C. $1,100.
A. $900
B. $1,400.
C. $1,100.
D. $1,300
Explanation: Unit price times number of units sold is total revenues. In this case, the unit price of
$1,100, although a lower price than before, will sell more computers and result in the highest
revenue, $132,000.
Quality demanded (per Elasticity Total revenues(quantity ×
week) Unit price Coefficient Price)
140 900 0.8 $126,000
120 1,100 0.9 132,000
100 1,200 1.0 120,000
80 1,300 1.1 104,000
60 1,400 1.3 84,000
40 1,500 2.0 60,000

Question #8: Which of the following actions will most likely result in a successful foreign
business venture in Islamic countries?
Correct Answer D. Behave in a manner that is consistent with Islamic ethics.
A. Adhere to Islamic beliefs.
B. Have property in an Islamic nation.
C. Employ Islamic people.
D. Behave in a manner that is consistent with Islamic ethics.
Explanation:
Successful operation by a company operating in a foreign country is a function of how well the
company adapts to the host country's culture. Successful adaptation includes behaving in a
manner that is consistent with host country's ethics.

Question #9: Question: Grimaldi's acid test ratio or quick ratio at the end of the year is
(Fact Pattern #171 Shown below are beginning and ending balances for certain of Grimaldi,
Inc.'s accounts.

January 1 December 31
Cash $48,000 $62,000
Marketable securities 42,000 35,000
Accounts receivable (net) 68,000 47,000
Inventory 125,000 138,000
Property, plant, and
equipment(net) 325,000 424,000
Accounts payable 32,000 84,000
Accrued liabilities 14,000 11,000
Deferred taxes 15,000 9,000
7% bonds payable 95,000 77,000
Grimaldi’s net income for the year was $96,000.
Correct answer: D. 1.52.
A. 0.83
B. 1.02
C. 1.15
D. 1.52
Explanation:
Answer (A) is incorrect.
A ratio of 0.83 results from improperly including bonds payable in the denominator.
Answer (B) is incorrect.
A ratio of 1.02 results from failing to include accounts receivable in the numerator.
Answer (C) is incorrect
A ratio of 1.15 results from failing to include marketable securities in the numerator.
Answer (D) is correct.
The acid test or quick ratio consists of the quick assets (cash. marketable securities. and accounts
receivable) divided by current liabilities. Grimaldi's quick ratio at year end is thus 1.516
[($62,000 - $35,000 + $47,000) ÷ ($84,000 + $11,000)].

Question #10: Webster Products is performing a capital budgeting analysis on a new product it is
considering. Annual sales are expected to be 50,000 units in the first year. 100,000 units in the
second year, and 125,000 units the year thereafter. Selling price will be $80 in the first year and
is expected to decrease by 5% per year. Annual costs are forecasted as follows.
Fixed costs $300,000 each year
Labor cost per unit $20 in year 1,
increasing 5% per
year thereafter
Material cost per unit $30 in year 1,
increasing 10% per
year thereafter
The investment of $2 million will be depreciated on a straight-line basis over 4 years for
financial reporting and tax purposes. Webster's effective tax rate is 40%. When calculating net
present value (NPV), the net cash flow for year 3 would be
Correct Answer C $1,058,750.
A. $858,750.
B. $1,070,000.
C. $1,058,750.
D. $558,750.
Explanation:
A. This is the net operating cash flow after tax. Net cash flow after tax includes this plus the
depredation tax shield.
B. This is not the correct answer. Please see the correct answer for an explanation
C.

D. This is accounting net income after tax it is not net cash flow after tax. Net cash flow
after taxis operating cash flowafter tax (excluding depreciation) p4us the depreciation tax
shield.

Question #11: Richardson Supply has a $100 invoice with payment terms of 2/10, net 60.
Richardson can either take the discount or place the funds in a money market account paying 6%
interest. Using a 360-day year. Richardson's cost of not taking the cash discount is
Correct Answer. D. 8.7%.
A. 6.4%.
B. 6.2%.
C. 12.2%.
D. 8.7%
Explanation:
A. This is not the correct answer. Please see the correct answer for an explanation.
B. This is ([360 /60) x (2 / [100 - 2])) - 0.06. The first part of the formula is incorrect. It
should be (360 / [60 - 10]), the Total Period -Period for Discount.
C. This is (360 / 60) x (2 / [100 - 2]) = 6 x 0.0204 = 0.1224. This is incorrect for two
reasons. One, the first part of the formula is incorrect. It should be (360 / [60 - 10]), the
Total Period - Period for Discount. And two, it does not take into consideration the 6%
return available in a money market account.
D. This question is a little bit more involved than the standard cash discount question
because we need to take into account the fact that Richardson can invest money at 6% if
they have it. If they pay within the discount period, they will save $2. However, if they
do not pay within the period, they will have an additional $98 available for 50 days that
they would not have had if they had paid early. That money will be able to earn 6%. Over
the 50days, that S98 will earn $0.817 ($98 x 0.06 ÷ 360 x 50). This means that they
would have an incremental cost of $1.183 for the 50 days covered by this receivable ($2 -
$0.817). Dividing this by the $98 that they have available from not paying early, we get a
cost of 1.2%. However, this is rate fora period of only 50 days so it needs to be multiplied
by 7.2 (calculated as 360 ÷ 50) in order to get an annual rate. The annual rate is 8.7%.

An alternative way of reaching the same answer is to calculate the cost of not taking the
discount and then subtract from that cost the annual rate that could be received by
investing the money in a money market account, as follows:
This question is a little bit more involved than the standard cash discount question
because we need to take into account the fact that Richardson can invest money at 6% if
they have it. If they pay within the discount period, they will save $2. However, if they
do not pay within the period, they will have an additional S98 available for 50 days that
they would not have had if they had paid early. That money will be able to earn 6%. Over
the 50days, that $98 will earn $0.817 ($98 x 0.06 + 360 x 50). This means that they
would have an incremental cost of $1.183 for the 50 days covered by this receivable ($2 -
$0.817). Dividing this by the $98 that they have available from not paying early, we get a
cost of 1.2%. However, this is rate fora period of only 50 days so it needs to be multiplied
by 7.2 (calculated as 360 ÷ 50) in order to get an annual rate. The annual rate is 8.7%. An
alternative way of reaching the same answer is to calculate the cost of not taking the
discount and then subtract from that cost the annual rate that could be received by
investing the money in a money market account, as follows:
(360/ [Total Period - Period for Discount]) x (Discount %/ [100 - Discount %]
(360/ [60 - 10]) x (2/ [100 - 2]) = 7.2 x 0.0204 = 0.1469
0.1469 - 0.06 = 0.0869 or 8.7%
Question #12: Please see the question below:
Refrigerator company manufactures ice-makers for installation in refrigerators. The costs
per unit, for 20,000 units of ice-makers, are as follows,
Direct materials $7
Direct labor 12
Variable overhead 5
Fixed overhead 10
Total costs $34
Correct Answer. A. $600,000.
A. $600,000.
B. $480,000.
C. $680,000.
D. $560,000.

Explanation:
The total relevant costs associated with the manufacture of ice-makers is as follows:

Total relevant costs = (unit variable manufacturing costs) (number of units) + (any
avoidable fixed costs)
Unit variable manufacturing costs = (direct materials + direct labor + variable overhead)
Unit variable manufacturing costs = ($7 + $12 + $5) = $24
Avoidable fixed costs are $6 per unit.
Therefore, the relevant costs to manufacture the ice-makers = ($24) (20,000 units) + ($6)
(20,000 units)
Total relevant costs = $480,000+ $120,000 = $600,000

Question #13: The tax impact of equipment depreciation affects capital budgeting decisions.
Currently, the Modified Accelerated Cost Recovery System (MACRS) is used as the
depreciation method for most assets for tax purposes.
The MACRS method of depreciation for assets with 3, 5, 7, and 10-year recovery periods is most
similar to which one of the following depreciation methods used for financial reporting
purposes?
Correct Answer B. 200%declining-balance.
A. Sum-of-the-years'-digits.
B. 200%declining-balance.
C. Units-of-production.
D. Straight-line.
Explanation:
A. The MACRS method of depreciation is not similar to the sum-of-the-years'-digits method
of depreciation.
B. The MACRS method of depreciation is most similar to the 200% declining balance
method (double-declining balance) of depredation.
C. The MACRS method of depreciation is not similar to the units-of-production method of
depreciation.
D. The MAC RS method of depreciation is not similar to straight-line depredation.

Question #14: Below is a statement of financial position of Martin Corporation:


Martin Corporation
Statement of financial position
(Dollars in millions)

Assets:
Current assets $75
Plant and equipment 250
Total Assets $325
Liabilities and shareholders' equity:
Liabilities:
Current liabilities $46
Long-term debt (12%) 64
Common equity:
Common stock, $1 par $10
Additional paid in capital 100
Retained earnings 105
Total liabilities and shareholders' equity $325

Additional data:
 The long term debt was originally issued at par ($1,000 per bond) and is currently trading
at $1,250 per bond.
 Martin Corporation can now issue debt 150 basis points over U.S. treasury bonds.
 The current risk-free rate (U.S. Treasury bonds) is 7%.
 The expected market return is currently 15%.
 The beta for martin is 1.25.
 Martin’s effective corporate income tax rate is 40%.
Using the capital Asset Pricing Model (CAPM), Martin Corporation’s current cost of common
equity is:
Correct Answer: A. 17.00%>
A. 17.00%.
B. 8.75%.
C. 15.00%.
D. 10.00%.
Explanation:
A. The CAPM formula is: R=R F + β(R M – R F). The difference between the expected
market return and the risk-free rate multiplied by the company’s beta is the individual
security’s risk premium. To that, we add the risk-free rate to get the cost of the
company’s common equity, which is the same thing as the investor’s required rate of
return for the stock (R).
In this question all we need to do is to put the information from the question into the
formula. The risk free rate is 7%, the market rate is 15% and the beta for the company is
1.25. This gives us: 0.07 + [1.25 (0.15 - 0.07)]. This is equal to 17%.
B. This is the risk-free rate multiplied by the beta. The CAPM incorporates more
information than this. The CAPM formula is R = R F+ β (R M – R F) . β (R M - R F) is
the individual security's risk premium. To that we add the risk-free rate to get the cost of
the company's common equity, which is the same thing as the investors' required rate of
return for the stock (R).
C. This is simply the expected market return. The CAPM formula is R = R F + β(R M - R
F ). The difference between the expected market return and the risk-free rate multiplied
by the company's beta is the individual security's risk premium. To that we add the risk-
free rate to get the cost of the company's common equity, which is the same thing as the
investors' required rate of return for the stock (R).
D. This answer does not add the risk free rate to the calculation of the individual security's
risk premium. The CAPM formula is R = R F+(R M - RF ).β (R M - R F) is the
individual security's risk premium. To that, we add the risk-free rate to get the cost of the
company's common equity, which is the same thing as the investors' required rate of
return for the stock (R).

Question #15: Following are the operating results of the two segments of Parklin Corporation.

Segment A Segment B Total


Sales $10,000 $15,000 $25,000
Variable costs of goods sold 4,000 8,500 12,500
Fixed costs of goods sold 1,500 2,500 4,000
Gross margin 4,500 4,000 8,500
Variable selling and administrative 2,000 3,000 5,000
Fixed selling and administrative 1,500 1,500 3,000
Operating income (loss) $1,000 $(500) $500

Correct Answer: D. A decrease of $2.000. All Possible Answers:


A. A decrease of $2,500.
B. An increase of $2,000.
C. An increase of $500
D. A decrease of $2,000.
Explanation:
To analyze this, one must compare the amount of sales foregone to the costs that will be avoided
if Segment B is closed. If Segment B is closed, opera ng income will be subject to the following
adjustments:
$(15,000
Sales )
Variable costs of goods sold $8,500
Fixed costs of goods sold $1,500
Variables selling and administrative expenses $3,000
Decrease in operating income $(2,000)

Question #16: On July 14, an investor goes short on a put option for 100 shares of OSC. Inc.
common stock with a strike price of $9.00, expiring on August 16, at an option premium of
$1.50 per share. The market price of OSC on July 14 is $8.00. On August 16, the market price of
OSC is $11.00. How much has the investor gained or lost on the option transaction? Disregard
any brokerage commissions involved.
Correct Answer: D. Gain of $150.
A. Gain of $200.
B. Loss of $50.
C. Loss of $150.
D. Gain of $150.
Explanation:
A. This is the market price of 100 shares on the option's expiration date ($1,100) minus the
strike price multiplied by 100 ($900).
B. This is the loss that would be sustained by the short party to a put transaction with the
same characteristics.
C. This is the loss that would be sustained by the long party to an option transaction with the
same characteristics.
D. The short party to an option is the one who sells the option and who must comply if the
buyer of the option chooses to exercise it. Here, the short party sold a put option to sell
the underlying stock at $9. The buyer of the option has the right to sell 100 shares of OSC
stock to the seller of the option at a price of $9.00. However, the market price of the stock
on the expiration date is greater than $9.00, and the buyer of the option will not sell stock
for $9.00 that he could sell on the open market for $11.00. So the buyer of the option lets
the option expire, and the seller of the option gets to keep the premium received for
selling the option of $1.50 per share. Multiplied by 100 shares, the option seller's gain is
$150.

Question #17: An enterprise is in the process of comparing its current financial performance for
Year 3 with the prior 2 years. The enterprise experienced exceptionally strong growth between
Years 1 and 2, with a slight decrease in sales between Years 2 and 3.

Year 1 Year 2 Year 3


Net sales $4,560,000 $30,980,400 $26,583,220
Cost of goods sold 2,378,900 24,655,340 21,444,985
Selling expenses
General and administrative
Expenses 290,500 500,000 600,000

Which one of the following statements is correct when using common-size analysis to compare
the results?
Correct Answer: C. The enterprises proportion of gross profit was lowest in Year 3 due to high
production costs. All Possible Answers:
A. The enterprises profitability increased each year due to more efficient production
processes.
B. The enterprise experienced the highest proportion of selling expenses in Year 2, which
led to the high net sales.
C. The enterprise increased the percentage of general and administrative expenses each year
in order to manage the company's growth.
D. The enterprises proportion of gross profit was lowest in Year 3 due to high production
costs.
Explanation:
Answer (A) is incorrect.
Although the enterprises profitability increased in Years 2 and 3 relative to Year 1, the gross
profit proportion in Years 2 and 3 was lower than Year 1.
Answer (B) is incorrect.
The enterprise did not experience the highest proportion of selling expenses in Year 2.
Answer (C) is incorrect.
Although general and administrative expenses increased, their proportion did not increase.
Answer (D) is correct.
Expressing financial statement items as percentages of corresponding base-year figures is a form
of common-size (percentage) analysis that is useful for evaluating trends. Net sales represents
100% on a common-size income statement. Although gross profit was higher in Year 3 than in
Year 1, its proportion relative to net sales and using common-size analysis is the lowest. Cost of
goods sold is the highest proportionally compared to Years 1 and 2.

Question #18: The net present value method of capital budgeting assumes that cash flows are
reinvested at
Correct Answer: B. The discount rate used in the analysis.
A. The risk-free rate.
B. The discount rate used in the analysis.
C. The rate of return of the project.
D. The cost of debt.
Explanation:
A. The risk-free rate is not a part of net present value analysis of a capital budgeting project.
B. The net present value method of capital budgeting involves the assumption that the
resulting cash flows will be able to be invested at the rate of return that is used as a
discount rate in the analysis.
C. The internal rate of return method of analyzing a capital investment project assumes that
the resulting cash flows will be able to be reinvested at the rate of return of the project,
but the net present value method does not.
D. The cost of debt for a capital investment is not the rate that the resulting cash flows are
assumed to be reinvested at in net present value analysis.

Question #19: The following selected data pertain to a 4-year project being considered by Metro
Industries: -A depreciable asset that costs $1,200,000 will be acquired on January 1. The asset,
which is expected to have a $200,000 salvage value at the end of 4 years, qualifies as 3-year
property under the Modified Accelerated Cost Recovery System (MACRS).
-The new asset will replace an existing asset that has a tax basis of $150,000 and can be sold on
the same January 1 for $180,000.
-The project is expected to provide added annual sales of 30,000 units at $20. Additional cash
operating costs are: variable, $12 per unit: fixed, $90,000 per year.
-A $50,000 working capital investment that is fully recoverable at the end of the fourth year is
required.
Metro is subject to a 40% income tax rate and rounds all computations to the nearest dollar.
Assume that any gain or loss affects the taxes paid at the end of the year in which it occurred.
The company uses the net present value method to analyze investments and will employ the
following factors and rates.
Present value of $1 Annuity at
Period Present value of $1 at 12% 12% MARCS
1 0.89 0.89 33%
2 0.80 1.69 45
3 0.71 2.40 15
4 0.64 3.04 7

The discounted, net-of-tax amount that relates to disposal of the existing asset is
Correct Answer: D. $169,320.
A. $190,680.
B. $180,000.
C. $168,000.
D. $169,320.
Explanation:
A. $190,680 results from adding the discounted tax liability to the cash inflow from the sale
of the existing asset. The discounted tax liability should be a deduction to the cash inflow
from the sale of the existing asset, not an addition to it.
B. $180,000 is the amount of cash to be received for the existing asset, but it does not
include the effect of taxes due on the gain. It also is not discounted.
C. $168,000 is the amount of cash to be received for the existing asset less the amount of tax
liability for the gain. However, the question asks for the discounted, net-of-tax amount
that relates to the disposal of the existing asset. In order to find the discounted amount, it
is necessary to discount the income tax liability since the tax is not due until year end.
D. Cash received from the sale of the existing asset on January 1 will be $180,000. Because
it takes place at the beginning of the year, it is considered a Year 0 cash flow. The tax
basis of the asset on that date will be $150,000. Therefore, income tax will be due on the
gain of $30,000 at the rate of 40%, so income tax will be $12,000. However, the problem
states that any gain or loss affects the taxes paid at the end of the year in which it
occurred. Therefore, tax on the January 1 gain will not be due until year end. Thus, we
discount the $12,000 income tax liability for one year at the 12% rate given in the Present
Value of $1 factor table. $12,000 x 0.89 = $10,680, so the present value of the income tax
liability is $10.680. The discounted, net-of-tax amount that relates to disposal of the
existing asset is $180,000 - $10,680.which equals $169,320.

Question #20: Which of the following would result in greater elasticity of resource demand?
Correct Answer: C. There are a great number of substitute resources available.
A. Demand for the product that the resource is producing is inelastic.
B. Marginal product declines rapidly.
C. There are a great number of substitute resources available.
D. The resource represents a small proportion of total production costs.
Explanation:
Elasticity of resources is affected by the following.
• Rate of decline in rate of marginal product: Rapid decline in marginal product indicates less
elasticity in resource demand.
• Ease of resource substitution: A large number of substitutes makes the demand for the resource
more elastic.
• Elasticity of product demand: Demand for resources is a derived demand. If the demand for the
product the resource produces is elastic, the demand for the resource will also be elastic.
• Resource cost/total cost ratio: The lower the proportion of total production costs the resource
represents, the lower its elasticity of demand.

Question #21: An entity has total assets of $7,500,000 and a current ratio of 2.3 times before
purchasing $750,000 of merchandise on credit for resale. After this purchase, the current ratio
will
Correct Answer: C. Be lower than 2.3 times.
A. Remain at 2.3 times.
B. Be higher than 2.3 times.
C. Be lower than 2.3 times.
D. Be exactly 2.53 times.
Explanation:
Answer (A) is incorrect.
The current ratio is the ratio of current assets to current liabilities. When the ratio is greater than
one, any change of equal dollar amount on both the numerator and denominator will result in a
lowering of the overall ratio (since the denominator will increase by a proportionally greater
amount). The purchase of merchandise on credit is an example of such a change: inventory
increases in the numerator and accounts payable increase in the denominator by an equal dollar
amount.
Answer (B) is incorrect.
The current ratio is the ratio of current assets to current liabilities. When the ratio is greater than
one, any change of equal dollar amount on both the numerator and denominator will result in a
lowering of the overall ratio (since the denominator will increase by a proportionally greater
amount). The purchase of merchandise on credit is an example of such a change: inventory
increases in the numerator and accounts payable increase in the denominator by an equal dollar
amount.
Answer (C) is correct.
The current ratio is the ratio of current assets to current liabilities. When the ratio is greater than
one, any change of equal dollar amount on both the numerator and denominator will result in a
lowering of the overall ratio (since the denominator will increase by a proportionally greater
amount). The purchase of merchandise on credit is an example of such a change: Inventory
increases in the numerator and accounts payable increases in the denominator by an equal dollar
amount.
Answer (D) is incorrect.
The current ratio is the ratio of current assets to current liabilities. When the ratio is greater than
one, any change of equal dollar amount on both the numerator and denominator will result in a
lowering of the overall ratio (since the denominator will increase by a proportionally greater
amount). The purchase of merchandise on credit is an example of such a change: inventory
increases in the numerator and accounts payable increase in the denominator by an equal dollar
amount.

Question#22: Fact pattern: The information below pertains to Devlin Company.


Statement of Financial position as of May 31
(in thousands)

Year 1 Year 2
Assets
Current assets
Cash $45 $38
trading securities 30 20
Accounts receivable (net) 68 48
Inventory 90 80
Prepaid expenses 22 30
Total current assets $255 $216
Investments, at equity 38 30
Property, plant, and equipment (net) 375 400
Intangible assets (net) 80 45
Total assets $748 $691
Liabilities
Current liabilities
Notes payable $35 $18
Accounts payable 70 42
Accrued expenses 5 4
Income taxes payable 15 16
Total current liabilities $125 $80
Long term debt 35 35
Deferred taxes 3 2
Total liabilities $163 $117
Equity
Preferred stock, 6%, $100 par value,
cumulative $150 $150
Common stock, $10 par value 225 195
Additional paid-in capital--common stock 114 100
Retained earnings 96 129
Total Equity $585 $574

Income statement for the year ended May 31


(in thousands)

Year 2 Year 1
Net sales $480 $460
Costs and expenses
Costs of goods sold 330 315
Selling, general, and administrative 52 51
Interest expense 8 9
Income before taxes $90 $85
Income taxes 36 34
Net income $54 $51

Question: Devlin Company’s rate of return on assets for the year ended May 31, Year 2, was
Correct Answer: B. 7.5%
A. 7.2%
B. 7.5%
C. 7.8%
D. 11.3%
Explanation:
Answer (A) is incorrect.
The figure of 7.2% uses ending total assets instead of average total assets.
Answer (B) is correct.
The rate of return on assets equals net income divided by average total assets. Accordingly, the
rate of return is 7.5% ($54 ÷ [($748 + $691) ÷ 2]}.
Answer (C) is incorrect.
Net income divided by beginning total assets equals 7.8%.
Answer (D) is incorrect.
The return on sales is 11.3%.

Question #23: Delphi Company has developed a new product that will be marketed for the first
time during the next fiscal year. Although the Marketing Department estimates that 35,000 units
could be sold at $36 per unit. Delphi's management has allocated only enough manufacturing
capacity to produce a maximum of 25,000 units of the new product annually. The fixed costs
associated with the new product are budgeted at $450,000 for the year which includes $60,000
for depreciation on new manufacturing equipment. Data associated with each unit of product are
presented below. Delphi is subject to a 40% income tax rate.

Variable costs
Direct material $7.00
Direct Labor 3.50
Manufacturing overhead 4.00
Total variable manufacturing cost 14.50
Selling expenses 1.50
Total variable cost $16.00

Delphi Company's management has stipulated that it will not approve the continued:
manufacture of the new product after the next fiscal year unless the after-tax profit is at least
$75,000 the first year. The unit selling price to achieve this target profit must be at least:
Correct Answer: A. $39.60.
A. $39.00.
B. $41.40.
C. $36.60.
D. $34.60.
Explanation:
After-tax profit = (1- tax rate) x [(unit price - unit variable costs)(volume) - (avoidable fixed
costs)]
$75,000 = (1- 0.40) x [(p - $16) (25,000 units) - ($450,000)] Where p = unit price
$75,000 = (0.60) [(25,000p - $400,000) - $450.000] (Divide both sides by 0.60 and add the
whole numbers on the right together.)
($75,000 / 0.60) = 25,000p - $850,000
$125,000 = 25,000p - $850.000 (Add $850,000 to each side.)
$975,000 = 25,000p (Divide both sides by 25,000.)
P = $39.00 per unit.

Question #24: Based on the fraud risk model, which of the following most likely is not an
opportunity to commit employee fraud?
Correct Answer: B. Living beyond ones means.
A. Lack of transaction authorizations.
B. Living beyond ones means.
C. Poor accounting records.
D. Lack of physical controls.
Explanation:
Answer (A) is incorrect.
Lack of transaction authorizations creates an opportunity to commit fraud.
Answer (B) is correct.
Living beyond one's means is pressure or motivation to commit fraud.
Answer (C) is incorrect.
Poor accounting records create an opportunity to commit fraud.
Answer (D) is incorrect.
Lack of physical controls creates an opportunity to commit fraud.

Question #25: Katelyn is the Controller for Hobbie Corners; a company that produces
manufactured homes. Hobbie Corners year-end is approaching and the forecasted profitability
report for the year is significantly lower than expected. Andrew, a senior Management
Accountant that reports to Katelyn, is responsible for forecasting the year-end profitability
reports. In an effort to lower costs, both Andrew and Katelyn have been asked to join a
committee to review operations. The committee selected several issues to address immediately in
an attempt to improve profitability. Andrew has been asked to work with the purchasing
department to select vendors that can offer raw materials at lower costs without compromising
quality. Hazel Enterprises can provide many of the key raw materials needed at a lower cost;
however, the owner of the company is related to Katelyn. After preparing a cost savings analysis.
Andrew recommends that Hazel Enterprises be selected as a new vendor. Katelyn has been asked
to review the percentage of completion calculations on the homes that are currently in process.
Katelyn has the ability to change the percentage of completion calculation which would directly
impact profitability for the company. Katelyn has modified the calculation and forwarded the
amended report to the auditors.
The IMA Statement of Ethical Professional Practice describes the first overarching ethical
principle as Honesty. Identify an example of honesty relevant to the Hobbie Corners case above.
Correct Answer: B. Disclosing all necessary and relevant information to outside auditors.
A. Ensuring information on reports and statements is accurate.
B. Disclosing all necessary and relevant information to outside auditors.
C. Selecting vendors without bias. Prejudice, or favoritism.
D. Providing information and feedback objectively.
Explanation:
The first principle, honesty, requires conscientious application to the task at hand and
truthfulness in all analyses and communications. Examples of honesty include: disclosing all
necessary and relevant information to outside auditors: refusing to record information that is
anything less than accurate: and providing factual information to others so that they can make
decisions based on truthful information. In this case. Katelyn has a responsibility to disclose all
relevant information to the outside auditors to justify the amendments to the percentage of
completion report.

Question #26: Given an acid test ratio of 2.0, current assets of $5,000, and inventory of $2,000,
the value of current liabilities is
Correct Answer: A. $1,500
A. $1,500
B. $2,500
C. $3,500
D. $6,000
Explanation:
Answer (A) is correct.
The acid test, or quick, ratio equals the quick assets (cash. marketable securities. and accounts
receivable) divided by current liabilities. Current assets equal the quick assets plus inventory and
prepaid expenses. (This question assumes that the entity has no prepaid expenses.) Given current
assets of $5,000, inventory of $2,000, and no prepaid expenses, the quick assets must be $3,000.
Because the acid test ratio is 2.0, the quick assets are double the current liabilities. Current
liabilities therefore are equal to $1,500 ($3,000 quick assets ÷ 2.0).
Answer (B) is incorrect.
Dividing the current assets by 2.0 results in $2,500. Current assets includes inventory, which
should not be included in the calculation of the acid test ratio.
Answer (C) is incorrect.
Adding inventory to current assets rather than subtracting it results in $3,500.
Answer (D) is incorrect.
Multiplying the quick assets by 2 instead of dividing by 2 results in $6,000.

Question #27: Foggy Products is evaluating two mutually exclusive projects, one requiring a $4
million initial outlay and the other a $6 million outlay. The Finance Department has performed
an extensive analysis of each project. Which of the following statements are correct?
I. Both projects should be rejected if their payback periods are longer than the company
standard.
II. The project with the higher Internal Rate of Return (IRR) should be selected
(assuming both IRRs exceed the hurdle rate).
III. The project with the higher positive net present value should be selected.
IV. Select the project with the smaller initial investment, regardless of which evaluation
method is used.
Correct Answer: B. I and III only.
A. I, II, and IV only.
B. I and III only.
C. I, II and III only.
D. II and III only.
Explanation:
A. II is not always true. A project may have a very high IRR, but the project may be a very
small project, and its NPV may not be as high as that of other projects. Other factors must
be considered besides the IRR.
IV is not true. Selecting the project with the smaller initial investment does not take into
consideration whether the project will benefit the company and its shareholders. The
project with the smaller initial investment might have a negative NPV.
B. I is true. If the payback period for a project is longer than the company standard for
payback periods, a project should be rejected. II is not always true. A project may have a
very high IRR, but the project may be a very small project, and its NPV may not be as
high as that of other projects. Other factors must be considered besides the IRR. III is
true. When two projects are mutually exclusive (i.e., the company can select one or the
other but not both), the project with the higher NPV should be selected, because that will
maximize the shareholders' wealth. IV is not true. Selecting the project with the smaller
initial investment does not take into consideration whether the project will benefit the
company and its shareholders. The project with the smaller initial investment might have
a negative NPV.
C. II is not always true. A project may have a very high IRR, but the project may be a very
small project, and its NPV may not be as high as that of other projects. Other factors must
be considered besides the IRR.
D. II is not always true. A project may have a very high IRR, but the project may be a very
small project, and its NPV may not be as high as that of other projects. Other factors must
be considered besides the IRR.

Question #28: A corporation's inventory expressed as a percentage of current assets increased


from 25% last July to 35% this July. The factor that is least likely to cause this increase is
that the corporation
Correct Answer: A. Is a seasonal company with traditionally higher activity in the summer
months.
A. Is a seasonal company with traditionally higher activity in the summer months.
B. Is beginning to experience high growth.
C. Has inventory that is becoming obsolete.
D. Used a material amount of cash from selling its short-term investments to purchase land.
Explanation:
Answer (A) is correct.
This statement is least likely to explain an increase in current assets from last July to this
July. If the corporation was a seasonal company with traditionally higher activity in the
summer months, it would budget similar amounts for each summer in expectation of the high
activity. The sudden increase in current assets for the following summer would not be
explained by the fact that they are a seasonal company.
Answer (B) is incorrect.
If the corporation was beginning to experience high growth, it would have to purchase more
inventory in order to meet the higher demand from the growth. This would cause the current
assets account to increase.
Answer (C) is incorrect.
Obsolete inventory refers to inventory held by a company that is at the end of its product life
cycle and has not seen any sales or usage for a set period of time. If the corporation has
inventory that is becoming obsolete, the inventory will be held by the company instead of
being sold. This would cause an increase in the current assets on the books.
Answer (D) is incorrect.
It can be assumed that the sale of short-term investments generated a gain, which caused a
bigger cash inflow than the outflow from the short-term investments. This means that the
current assets account increased. The fact that the company bought land with some of the
cash is meant as a distractor, as the question does not state how much of the cash was used to
purchase the land.

Question #29: Almelo Manpower Inc. provides contracted bookkeeping services. Almelo has
annual fixed costs of $100,000 and variable costs of $6 per hour. This year the company
budgeted 50,000 hours of bookkeeping services. Almelo prices its services at full cost and
uses a cost-plus pricing approach. The company developed a billing price of $9 per hour. The
company's mark-up level would be:
Correct Answer: B. 12.5%.
A. 50.0%.
B. 12.5%.
C. 33.3%.
D. 66.6%.
Explanation:
The unit cost at 50,000 hours are as follows:
Unit cost at 50,000 hours = (fixed costs/# hours) + variable cost per hour
Unit cost at 50,000 hours = ($100,000/50,000) + $6 = $2 + $6 = $8 per hour
Given the price of $9, the mark-up level on cost = (price - cost) / (cost)
Mark-up level on cost = ($9 - $8) / ($8) =1/8 = 0.125, or 12.5%.

Question #30: According to the DuPont formula, which one of the following will not
increase a profitable firm's return on equity?
Correct Answer: D. Lowering equity multiplier.
A. Increasing total asset turnover.
B. Increasing net profit margin.
C. Lowering corporate income taxes.
D. Lowering equity multiplier.
Explanation:
Answer (A) is incorrect.
Increasing total asset turnover would increase a profitable firm's return on equity.
Answer (B) is incorrect.
Increasing net profit margin would increase a profitable firm's return on equity.
Answer (C) is incorrect.
Lowering corporate income taxes would increase a profitable firm's return on equity.
Answer (D) is correct.
Lowering the equity multiplier would not increase a profitable firm's return on equity. The
DuPont model depicts return on assets as total asset turnover (sales divided by average total
assets) times the profit margin (net income divided by sales).

Question #31: A Whistle-blowing Framework or ethics helpline can assist in maintaining an


ethical organizational culture. Which of the following is not a benefit of a whistleblowing
framework or ethics helpline?
Correct Answer: C. Whistle-blowing helplines may be perceived as a snitch-line.
A. Whistle-blowing framework may enable the ability to collect, analyze, and summarize
existing and potential ethical problems.
B. Whistle-blowing helplines may help discover fraud within an organization.
C. Whistle-blowing helplines may be perceived as a snitch-line.
D. Whistle-blowing helplines may provide a confidential way to report ethical violations.
Explanation: If perceived as a snitch-line, employees may be reluctant to use them and would
then not be considered a benefit.

Question #32: Through the use of decision models, managers thoroughly analyze many
alternatives and decide on the best alternative for the company. Often the actual results
achieved from a particular decision are not what was expected when the decision was made.
In addition, an alternative that was not selected would have actually been the best decision
for the company. The appropriate technique to analyze the alternatives by using expected
inputs and altering them before a decision is made is
Correct Answer: A. Sensitivity analysis.
A. Sensitivity analysis.
B. Program Evaluation Review Technique (PERT).
C. Linear programming.
D. Expected value analysis.
Explanation:
A. Sensitivity analysis is a process of changing key variables to determine the possible
change in the optimal solution because of changes in the variables. It is used to define
how sensitive the project (sales for example) is to a change in those variables.
B. PERT is a project scheduling technique used to plan and control projects.
C. Linear programming is used to either maximize or minimize some quantity (called the
objective function). At the same time, this maximizing or minimizing must be
accomplished in the presence of constraints, or restrictions, such as limited quantities of
labor or materials. The maximization or minimization must be done without violating any
of the constraints.
D. Expected value analysis is used to determine expected return or cost. Expected value is
calculated by multiplying each projected outcome by its corresponding probability and
adding the products together. In other words, expected value is the weighted average of
the probable outcomes.

Question #33: In assessing the financial prospects for a firm, financial analysts use various
techniques. An example of vertical, common-size analysis is
Correct Answer: D. Advertising expense for the current year is 2% of sales.
A. An assessment of the relative stability of a firm's level of vertical integration.
B. A comparison in financial ratio form between two or more firms in the same industry.
C. Advertising expense is 2% greater compared with the previous year.
D. Advertising expense for the current year is 2% of sales.
Explanation:
Answer (A) is incorrect.
Vertical integration occurs when a corporation owns one or more of its suppliers or customers.
Answer (B) is incorrect. Vertical, common-size analysis restates financial statements amounts as
percentages.
Answer (C) is incorrect.
A statement that advertising expense is 2% greater than in the previous year results from
horizontal analysis.
Answer (D) is correct. Vertical, common-size analysis compares the components within a set of
financial statements. A base amount is assigned a value of 100%. For example, total assets on a
common-size balance sheet and net sales on a common-size income statement are valued at
100%. Common-size statements permit evaluation of the efficiency of various aspects of
operations. An analyst who states that advertising expense is 2% of sales is using vertical,
common-size analysis.

Question #34: Which of the following is a motive for fraudulent financial reporting?
Correct Answer: D. A manager's compensation is tied to reported financial results.
A. Oversight of management is lacking.
B. There is no internal auditing function.
C. The board of directors includes a number of related parties.
D. A manager's compensation is tied to reported financial results.
Explanation:
Answer (A) is incorrect.
When oversight of management is lacking, the opportunity for fraud increases.
Answer (B) is incorrect.
When there is no internal auditing function, the opportunity for fraud increases.
Answer (C) is incorrect.
When the board of directors includes a number of related parties, the opportunity for fraud
increases.
Answer (D) is correct.
When a manager's compensation is tied to reported financial results, the motive for fraud is
stronger.

Question #35: Which of the following economic policies would not tend to correct a balance of
payments deficit in the U.S.?
Correct Answer: D. Increase value of U.S. currency in relation to foreign currencies.
A. A reduction in the economic aid and humanitarian aid provided to other nations.
B. More effective use of monetary and fiscal policies to reduce inflation.
C. Increase productivity in the manufacturing of U.S. exports.
D. Increase value of U.S. currency in relation to foreign currencies.
Explanation:
A. A reduction of economic and humanitarian aid is a reduction of unilateral transfers out of
the U.S. thus reducing the balance of payments deficit.
B. A reduction of inflation will lower U.S. goods prices relative to foreign prices. As a
result. U.S. exports will increase, reducing the balance of payments deficit.
C. Increased productivity in the manufacture of U.S. exports would reduce the relative price
of U.S. export goods compared to other goods. This would decrease a balance of
payments deficit.
D. An increase in the value of U.S. currency will raise the comparative price of U.S.
products. This will reduce the volume of U.S. exports and increase imports as the relative
price of U.S. goods increases. Thus the balance of payments deficit will increase, not
decrease, if the value of the U.S. currency increases.

Question #36: When a company offers credit terms of 2/10, net 30, the annual interest cost,
based on a 360-day year, is
Correct Answer: B. 36.7%.
A. 35.3%.
B. 36.7%.
C. 24.0%.
D. 36.0%.
Explanation:
A. This is (360 / 20) x (2 / 102) = 0.3529 or 35.3%.
The correct formula is:
360 Discount %
×
Total period for payment −Period for discount payment 100 %−Discount %
B. This question is solved using the following formula:
360 Discount %
×
Total period for payment =period for discount payment 100 %−Discount %
Inputting the information from the question into the formula, we get 36.7%, as follows:
360 Discount %
×
Total period for payment −Period for discount payment 100 %−Discount %

Inputting the information from the question into the formula, we get 36.7%, as follows:
360 0.02
30 – 10 1.00 – 0.02
= 18 × 0.0204 = 0.367 or 36.7%
C.This is (360 / 30) x (2 / 100) = 0.24 or 24%.
The correct formula is:

360 Discount %
❑ ×
Total period for payment P eriodfor discount payment 100 %−Discount %

D. This is (360 / 20) x (2 / 100) = 0.36 or 36%.


The correct formula is:
360 Discount %
×
Total period for payment −Period for discount payment 100 %−Discount %

Question #37: As management accountants progress in the profession, they often have the
responsibility to supervise the work of less experienced workers. Which of the following is an
ethical responsibility of the supervisor?
Correct Answer: C. Ensure that workers handle confidential information appropriately.
A. Maximize the profit or minimize the cost of the department.
B. Encourage the workers to develop relations with customers.
C. Ensure that workers handle confidential information appropriately.
D. Hire new workers who will fit in socially with existing staff.
Explanation:
Per the IMA's "Statement of Ethical Professional Practice”, a management accountant has the
responsibility to keep information confidential except when disclosure is authorized or legally
required. A management accountant also has the responsibility to inform all relevant parties
regarding appropriate use of confidential information. This includes monitoring subordinates'
activities to ensure compliance.

Question #38: The slope of a Security Market Line is


Correct Answer: C. the market risk premium.
A. The graphical representation of the security's returns.
B. The graphical representation of the security's risk.
C. The market risk premium.
D. The beta.
Explanation:
A. The graph of the Capital Asset Pricing Model equation is a firm's Security Market Line.
However, the slope of a stock's Security Market Line is not the graphical representation
of the security's returns.
B. The Security Market Line tells us what investors' required rates of return are at each level
of risk as measured by the stock's beta. However, the slope of a stock's Security Market
Line is not the graphical representation of the security's risk.
C. Investors require a higher expected return for a stock with a higher beta. The Security
Market Line tells us what investors' required rates of return are at each level of risk as
measured by the stock's beta. It shows the linear relationship between the possible beta
coefficients for an individual investment and the required rate of return for the
investment. On an SML graph, the possible betas are on the x axis, and investors'
required rates of return are on the y axis. The slope of a stock's Security Market Line is
the market risk premium, which is RM-RF. The graph of the Capital Asset Pricing Model
equation is a firm's Security Market Line. Because the required rate of return by investors
is a firm's cost of capital, the firm's cost of equity capital will increase as its stock's beta
increases. The Security Market Line for an individual stock can be used to estimate the
firm's cost of debt capital and equity capital, based on investors' required rates of return at
each level of risk as measured by the stock's beta.
D. The Security Market Line tells us what investors' required rates of return are at each level
of risk as measured by the stock's beta. It shows the linear relationship between the beta
coefficient for an individual investment and the required rate of return for the investment.
However, the slope of a stock's Security Market Line is not the stock's beta, because the
possible betas are on the x axis on the graph.
Question #39: Finan Corporation's management is considering a plant expansion that will
increase its sales and have commensurate impact on its net working capital position. The
following information presents management's estimate of the impact the proposal will have on
Finan.

Current Proposal
Cash $100,000 $110,000
Accounts payable 400,000 470,000
Accounts receivable 560,000 690,000
Inventory 350,000 380,000
Marketable securities 200,000 200,000
Fixed assets 2,500,000 3,500,000
Net income 500,000 650,000

The impact of the plant expansion on Finan's working capital would be


Correct Answer: D. An increase of $100,000.
A. An increase of $950,000.
B. A decrease of $950,000.
C. A decrease of $100,000.
D. An increase of $100,000.
Explanation:
A. This answer results from including in working capital the increase in fixed assets and
deducting the increase in net income that will result from the plant expansion. Neither of
those items are components of working capital.
B. This answer results from including in working capital the increase in fixed assets and
deducting the increase in net income that will result from the plant expansion, and then
calculating the difference as a decrease instead of an increase. Neither fixed assets nor net
income are components of working capital.
C. This is the amount of the change in working capital, but the level of working capital will
increase, not decrease, if the proposal is implemented.
D. Working capital is calculated as current assets (cash. receivables. inventory and
marketable securities in this question) minus the current liabilities (accounts payable in
this question). Currently, the working capital is $810,000 ($100,000+ $560,000 +
$350,000 + $200,000 - $400,000). Under the new proposal it will be $910,000 (110,000
+ $690,000 + $380,000+ $200,000 – 470,000). This is an increase of $100,000 in the
working capital of the company.
Question #40: Katelyn is the Controller for Hobbie Corners; a company that produces
manufactured homes. Hobbie Corners year-end is approaching and the forecasted profitability
report for the year is significantly lower than expected. Andrew. a senior Management
Accountant that reports to Katelyn, is responsible for forecasting the year-end profitability
reports. In an effort to lower costs, both Andrew and Katelyn have been asked to join a
committee to review operations. The committee selected several issues to address immediately in
an attempt to improve profitability. Andrew has been asked to work with the purchasing
department to select vendors that can offer raw materials at lower costs without compromising
quality. Hazel Enterprises can provide many of the key raw materials needed at a lower cost;
however, the owner of the company is related to Katelyn. After preparing a cost savings analysis.
Andrew recommends that Hazel Enterprises be selected as a new vendor. Katelyn has been asked
to review the percentage of completion calculations on the homes that are currently in process.
Katelyn has the ability to change the percentage of completion calculation which would directly
impact profitability for the company. Katelyn has modified the calculation and forwarded the
amended report to the auditors.
The IMA Statement of Ethical Professional Practice describes the overarching ethical principle
of Responsibility. Identify an example of responsibility relevant to the Hobbie Corners case
above.
Correct Answer: C. Ensuring information on reports and statements is accurate.
A. Disclosing all necessary and relevant information to outside auditors.
B. Providing information and feedback objectively.
C. Ensuring information on reports and statements is accurate.
D. Selecting vendors without bias, prejudice, or favoritism.
Explanation:
Responsibility requires actions to be performed with faithfulness and loyalty. Examples of
responsibility include conveying information at the appropriate time, ensuring information on
reports and statements is accurate, and gathering enough information to make an informed
decision. Katelyn has a responsibility to ensure that the information on the percentage of
completion report is accurate.

Question #41: When the U.S. dollar is expected to rise in value against foreign currencies, a U.S.
company with foreign currency denominated receivables and payables should
Correct Answer: • A. Speed up collections and slow down payments.
A. Speed up collections and slow down payments.
B. Slow down collections and speed up payments.
C. Slow down collections and slow down payments.
D. Speed up collections and speed up payments.
Explanation:
A. If the U.S. dollar rises in value in the future. U.S. dollars will be worth more relative to
foreign currency and the foreign currency is worth less compared to the dollar. Therefore
the company will want to get as many dollars as possible now so that they will increase in
value while they are being held and the company will want to keep the dollars that they
have to use to pay the foreign denominated payables when the dollar is stronger.
Therefore, they should speed up collections and slow down payments.
B. If the U.S. dollar rises in value in the future. U.S. dollars will be worth more relative to
foreign currency and the foreign currency is worth less compared to the dollar. Therefore
the company will want to get as many dollars as possible now so that they will increase in
value while they are being held and the company will want to keep the dollars that they
have to use to pay the foreign denominated payables when the dollar is stronger.
Therefore, they should speed up collections and slow down payments.
C. If the U.S. dollar rises in value in the future. U.S. dollars will be worth more relative to
foreign currency and the foreign currency is worth less compared to the dollar. Therefore
the company will want to get as many dollars as possible now so that they will increase in
value while they are being held and the company will want to keep the dollars that they
have to use to pay the foreign denominated payables when the dollar is stronger.
Therefore, they should speed up collections and slow down payments.
D. If the U.S. dollar rises in value in the future. U.S. dollars will be worth more relative to
foreign currency and the foreign currency is worth less compared to the dollar. Therefore
the company will want to get as many dollars as possible now so that they will increase in
value while they are being held and the company will want to keep the dollars that they
have to use to pay the foreign denominated payables when the dollar is stronger.
Therefore, they should speed up collections and slow down payments.

Question #42: Grapevine Corporation produces two joint products. JP-1 and JP-2, and a single
by-product, BP-1, in Department 2 of its manufacturing plant. JP-1 is subsequently transferred to
Department 3 where it is refined into a more expensive, higher-priced product. JP-1R, and a by-
product known as BP-2. Recently, Santa Fe Company introduced a product that would compete
directly with JP-1R and, as a result, Grapevine must reevaluate its decision to process JP-1
further. The market for JP-1 will not be affected by Santa Fe's product, and Grapevine plans to
continue production of JP-1, even if further processing is terminated. Should this latter action be
necessary, Department 3 will be dismantled.
Which of the following items should Grapevine consider in its decision to continue or terminate
Department 3 operations?
1. The selling price per pound of JP-1.
2. The total hourly direct labor cost in Department 3.
3. Unit marketing and packaging costs for BP-2.
4. Supervisory salaries of Department 3 personnel who will be transferred elsewhere in the
plant, if processing is terminated.
5. Department 2 joint cost allocated to JP-1 and transferred to Department 3.
6. The cost of existing JP-1R inventory.
Correct Answer: 1,2,3.
A. 2, 3, 5, 6.
B. 1, 2, 3, 4, 5.
C. 2, 3, 4.
D. 1, 2, 3.
Explanation:
The decision to sell a product or process it further is dependent on the cost of the process and the
change in market value from the further processing. The further processing costs include the
direct labor costs in Department 3 and the marketing and packaging costs related to BP-2. The
change in market value in is the value of JP-1 less the value of J P1. These are items 1,2, and 3.

Question #43: Colt. Inc. is planning to use retained earnings to finance anticipated capital
expenditures. The beta coefficient for Colt's stock is 1.15, the risk-free rate of interest is 8.5%,
and the market return is estimated at 12.4%. If a new issue of common stock were used in this
model, the flotation costs would be 7%. By using the Capital Asset Pricing Model (CAPM)
equation:
R=R F + β (RM-RF)
The cost of using retained earnings to finance the capital expenditures is:
Correct Answer: D. 12.99%.
A. 14.71%.
B. 12.40%.
C. 13.96%.
D. 12.99%.
Explanation:
A. This answer results from using the flotation cost as R F in the Capital Asset Pricing
Model. The information on flotation in the question is not relevant to the calculation that
needs to be done.
B. This is the market rate of interest, not the cost of the retained earnings.
C. This is the cost of retained earnings divided by 100% minus the flotation cost of 7%. The
information on flotation costs is not relevant to the calculation that needs to be done.
Using retained earnings does not involve issuing new equity and thus no flotation costs
are incurred.
D. The information on flotation costs in the question is not relevant to the calculation that
needs to be done. All we need to do is put the information into the CAPM formula that is
given in the problem. This gives us: 8.5% + [1.15 x (12.4% - 8.5%)] = 12.99% as the cost
of retained earnings.

Question #44: Public record searches may be effective in certain instances. Which of the
following is a limitation on public record searches?
Correct Answer: D. Availability of records may be limited.
A. It is often very costly to search public records.
B. Very few types of information are available.
C. The information from public sources is most often incorrect.
D. Availability of records may be limited.
Explanation:
Answer (A) is incorrect.
Use of public sources of information is usually cost effective. In fact. Internet searches may be
virtually costless.
Answer (B) is incorrect.
A vast amount of information is available from public sources.
Answer (C) is incorrect.
In most cases, the information obtained is correct. However, care should be taken because
information may have changed since it was first collected and reported.
Answer (D) is correct.
Records for early periods may be limited or nonexistent.

Question #45: Diane Harper, Vice President of Finance for BGN Industries, is reviewing
material prepared by her staff prior to the board of directors meeting at which she must
recommend one of four mutually exclusive options for a new product line. The summary
information below indicates the initial investment required, the present value of cash inflows
(excluding the initial investment) at BGN's hurdle rate of 16%, and the internal rate of return
(IRR) for each of the four options.
Option Investment Present value of cash inflows at 16% IRR
X $3,950,000 $3,800,000 15.5%
Y 3,000,000 3,750,000 19.0%
Z 2,000,000 2,825,000 17.5%
W 800,000 1,100,000 18.0%

Which option should Harper recommend?


Correct Answer: D. Option Z.
A. Option Y.
B. Option W.
C. Option X.
D. Option Z.
Explanation:
A. Even though Option Y has the highest IRR. Option Y's NPV is not the highest among the
options. The option with the highest NPV is the option that will increase shareholder
wealth by the greatest amount, so Option Y is not the best recommendation.
B. The option with the highest NPV is the option that will increase shareholder wealth by
the greatest amount. Option W is a small project when compared with the rest of the
projects, and its NPV is correspondingly low. The option that will enrich shareholders the
most is the option that should be recommended.
C. The NPV for each project is the PV of the Cash Inflows at 16% minus the initial
investment. Since the NPV for Option X is negative ($3,800,000 - $3,950,000). Option X
is not acceptable.
D. Option Z's NPV is higher than any of the other options' NPVs. Thus. Option Z is the
option that will increase shareholders' wealth by the greatest amount, so it is the project
that should be recommended, even though two other options have higher I RRs. The NPV
for each project is the PV of the Cash Inflows at 16% minus the initial investment.
Therefore. the NPVs for the four projects are as follows:

Option Investment Present value of cash inflows at 16% NPV IRR


$(150,000
X $3,950,000 $3,800,000 ) 15.5%
Y 3,000,000 3,750,000 750,000 19.0%
Z 2,000,000 2,825,000 825,000 17.5%
W 800,000 1,100,000 300,000 18.0%

Option Z has the highest NPV. Even though Option Y has a higher IRR, the amount of the
investment is higher $3,000,000 versus $2,000,000) and in addition. Option Y's NPV is lower.
Option W is a small project when compared with the rest of the projects, and its NPV is
correspondingly lower than the others, even though its IRR is higher. Option X is not even one to
consider, because its NPV is negative and its IRR is below BGN's hurdle rate. The option that
will enrich shareholders the most is Option Z. so that is the option that should be recommended.

Question #46: Fact Pattern: Tosh Enterprises reported the following account information: (Refer
the image) Question: Tosh Enterprises' amount of working capital is
Total Enterprises reported the following account information:
$400,00
Accounts receivable 0
Accounts payable 260,000
Bonds payable, due in 10 years 600,000
Cash 200,000
Interest payable, due in 3 months 20,000
$800,00
Inventory 0
Land 500,000
Short-term prepaid expense 80,000

Correct Answer: C. $1,200,000


A. $600,000
B. $1,120,000
C. $1,200,000
D. $1,220,000
Explanation:
Answer (A) is incorrect.
The amount of $600,000 includes long-term bonds payable among the current liabilities.
Answer (B) is incorrect.
The amount of $1,120,000 excludes prepaid expenses from current assets.
Answer (C) is correct.
Working capital equals current assets minus current liabilities. For Tosh Enterprises, current
assets consist of cash, accounts receivable, inventory, and prepaid expenses, a total of
$1,480,000 ($400,000 + $200,000 + $800,000 + $80,000). Current liabilities consist of accounts
payable and interest payable for a total of $280,000 ($260,000 + $20,000). Accordingly, working
capital is $1,200,000 ($1,480,000 - $280,000).
Answer (D) is incorrect.
The amount of $1,220,000 excludes interest payable from current liabilities.
Question #47: When a multi-product plant operates at full capacity, quite often decisions must be
made as to which products to emphasize. These decisions are frequently made with a short-run
focus. In making such decisions, managers should select products with the:
Correct Answer: D. highest contribution margin per unit of the constraining resource.
A. Highest individual unit contribution margin.
B. Highest sales volume potential.
C. Highest sales price per unit.
D. Highest contribution margin per unit of the constraining resource.
Explanation:
The firm's short-run goal is to maximize profit. Maximizing short-run profit is equivalent to
maximizing contribution margin per unit of the constraining factor. The constraining factor here
is capacity. The firm maximizes profit by producing the products with the highest contribution
per unit of capacity.

Question #48: A company uses data obtained from customer listening surveys and customer
focus groups as a starting point for customer requirements. The ultimate goal is to refine the
requirements into technical requirements that will delight customers. This practice exemplifies
which value engineering practice?
Correct Answer: B. Quality function deployment (QFD).
A. Kaizen.
B. Quality function deployment (QFD).
C. Cause-and-effect analysis.
D. Process mapping.
Explanation:
By definition. QFD is a structured method in which customer requirements for a product or
service are translated into appropriate technical requirements at each stage of development and
production. The QFD process is often referred to as "listening to the voice of the customer."

Question #49: In evaluating independent capital investment projects, the best reason for a firm to
accept such projects is a(n)
Correct Answer: C. net present value greater than zero.
A. Initial investment greater than the present value of cash inflows.
B. Internal rate of return greater than the accounting rate of return.
C. Net present value greater than zero.
D. Accounting rate of return greater than zero.
Explanation:
A. If the initial investment were greater than the present value of the expected future cash
inflows, the project would have a negative net present value and would be unacceptable.
B. The internal rate of return and the accounting rate of return are different measurements
and provide very different information. Evaluating one against the other is not
meaningful.
C. When the net present value is greater than zero, that means the present value of the cash
inflows from the project is greater than the present value of the cash outflows from the
project. And that, in turn, means that the project will enhance the wealth of the
shareholders and is a project that should be accepted.
D. The accounting rate of return is the increase in the expected annual average after-tax
accounting net income divided by the net initial investment or the average investment.
The accounting rate of return is a percentage rate of return and zero is not a percentage
rate of return.

Question #50: A stockholder owns 10 shares of Shudo Corporation common stock at a current
market price of $10 per share. The corporation will allow each shareholder to buy proportional
new shares of stock at $9 per share. Currently, there are 5,000 shares outstanding and 500 new
shares will be issued. What is the value of one right (rounded to the nearest cent)?
Correct Answer: D. $.09.
A. $9.09.
B. $10.00.
C. $.91.
D. $.09.
Explanation:
A. This answer results from incorrectly interpreting the calculated answer. See the correct
answer for a complete explanation.
B. This is simply the current market price of the share of stock.
C. This is the value of the 10 rights that will be required to purchase a new share of stock.
D. In order to solve this question, we need to determine the value of the right when it is
selling rights-on. This is done using the following formula:
Vr = (P0 – Pn) / (r+1)
Where:
P0 = The value of a share with the rights still attached
Pn = The subscription (sales) price of a share
r = the number of rights needed to buy a new share
Vr = The value of the right
Since 5,000 shares are outstanding and 500 new shares will be issued, 10 shared will be required
to buy a new share (5,000 ÷ 500).
Putting the value into the formula, we calculate that the value of one right is
Vr = (10-9) / (10+1) = 0.0909.

Question#51: Jones Enterprises manufactures 3 products A, B, and C. During the month of May,
Jones’ Production, costs, and sales data were as follows.

A B C Totals
Units of production 30,000 20,000 70,000 120,000
Joint production costs to split-off $480,00
point 0
$140,00
Further processing costs $60,000 0
Unit sales price
At split-off 3.75 5.50 10.25
After further processing 8.00 12.50

Based on the above information, which one of the following alternatives should be recommended
to Jones’ management?
Correct Answer: B. Process Product C further but sell Product B at the split-off point
A. Process Product B further but sell Product C at the split-off point.
B. Process Product C further but sell Product B at the split-off point
C. Process both Products B and C further.
D. Sell both Product B and Product C at the split-off point.
Explanation:
A product should be processed further if the change in the market price from processing exceeds
the additional processing costs.
Product B can be sold at split-off for $5.50(20,000) = $110,000
Product B can be sold after further processing for $8(20,000) = $160,000
The cost increase is $160,000 - $110,000 = $50,000.
$50,000 is less than the additional processing costs of $60,000.
Therefore, Product B should be sold at split-off.
Product C can be sold at split-off for $10.25(70,000) = $717,500
Product C can be sold after further processing for $12.50(70,000) = $875,000
The cost increase is $875,000 - $717,500 = $157,500.
$157,500 is greater than the additional processing costs of $140,000.
Based on this information. Product C should be processed further.

Question #52: Please see the question below:


The treasury analyst for Garth Manufacturing has estimated the cash flows for the first half of
next year (ignoring any short-term borrowings) as follows.
Cash (millions)
Outflow
Inflows s
January $2 $1
February 2 4
March 2 5
April 2 3
May 4 2
June 5 3

Garth has a line of credit of up to $4 million on which it pays interest monthly at a rate of 1% of
the amount utilized. Garth is expected to have a cash balance of $2 million on January 1 and no
amount utilized on its line of credit. Assuming all cash flows occur at the end of the month,
approximately how much will Garth pay in interest during the first half of the year?
Correct Answer: C. $61,000
A. $80,000
B. $132,000
C. $61,000
D. Zero.
Explanation:
A. This is not the correct answer. Please see the correct answer for an explanation.
B. This is not the correct answer. Please see the correct answer for an explanation.
C. Garth will not need to borrow anything until the end of March because that is the first
date when the cash balance will be negative without borrowing.
At the end of March. Garth will need to borrow $2,000,000 to get its cash balance up to
zero. That $2,000,000 will be outstanding during the month of April, since the question
says that all cash flows occur at the end of each month.
For the month of April, that $2,000,000 outstanding on the line of credit will incur
$20,000 of interest, so that interest will become an additional cash outflow at the end of
April, making the required loan balance at the end of April $3,020,000. Cash in for April
is $2,000,000 and cash out for April is $3,000,000+ $20,000 interest: so Garth will need
to borrow another $1,000,000 as well as the amount due for interest. The ending cash
balance for April will be SO + $2D00D00 cash inflow from operations - $3,000,000 cash
outflow from operations - $20.000 interest = $(1.020.000). So Garth will add that to its
outstanding loan balance of $2,000,000. $3,020,000 will be outstanding on the loan
during the month of May, and interest due at the end of May on the outstanding balance
will be $30,200.
For the month of May, cash inflow is $4,000,000 from operations and cash outflow is
$2,000,000 from operations plus the cash paid in interest of $30,200. Ending cash
balance before any loans are repaid will be $1,969,800. So the company will be able to
pay down $1,969,800 on the line of credit at the end of May. The amount of the loan
outstanding at the end of May will be $1,050,200 ($3,020,000 - $1,969,800). So interest
accrued on the loan outstanding during the month of June will be $10,502. In June.
$5,000,000 of cash from operations comes in and only $3,000,000 cash from operations
goes out, plus the cash paid in interest of $10,502. At the end of the month, before any
loan repayment is made, the company will have $5,000,000 - $3,000,000 - $10,502, or
$1,989,498. The company will be able to pay off the entire principal outstanding on its
line of credit of $1,050,200 at the end of June and will have $939,298 in cash left. So the
total interest paid for the first half of the year will be: $20,000 paid at the end of April +
$30,200 paid at the end of May + $10,502 paid at the end of June, for a total of $60,700.
Since the question asks for the approximate amount of interest paid during the first half
of the year. $61,000 is the closest answer.
D. This answer could result from simply summing the beginning cash balance and the
expected inflows and subtracting the expected outflows. However, timing of the cash
flows is important in determining the need for financing, and financing will be needed, so
interest expense will be greater than zero.

Question #53: (Please refer the fact pattern below) Question: What will happen to the ratios
below if CPZ Enterprises uses cash to pay 50% of the accounts payable?
CPZ Enterprises had the following account information.
$200,00
Accounts receivable 0
Accounts payable 80,000
Bounds payable, due in 10 years 300,000
Cash 100,000
Interest payable, due in 3 months 10,000
Inventory 400,000
Land 250,000
Notes payable, due in 6 months 50,000
Prepaid expenses 40,000

Correct Answer: • Increase (Current Ratio), (Quick Ratio) Increase


A. Increase (Current Ratio), (Quick Ratio) Increase
B. Decrease (Current Ratio), (Quick Ratio) Decrease
C. Increase (Current Ratio), (Quick Ratio) Decrease
D. Decrease (Current Ratio), (Quick Ratio) Increase
Explanation:
Answer (A) is correct.
Using cash to pay accounts payable will affect both ratios in a positive way. For instance, before
the payment, current assets totaled $740,000 and current liabilities were $140,000, yielding a
current ratio of 5.29. Paying $40,000 of the accounts payable ($80,000 x 50%) would reduce
current assets to $700,000 and current liabilities to $100,000, for a new current ratio of 7.00. The
quick assets would decline from $300,000 to $260,000, and the current liabilities from $140,000
to $100,000, for a new quick ratio of 2.60, an increase over the old ratio of 2.14.
Answer (B) is incorrect.
Both ratios will increase as a result of using cash to pay 50% of the accounts payable.
Answer (C) is incorrect.
Both ratios will increase as a result of using cash to pay 50% of the accounts payable.
Answer (D) is incorrect.
Both ratios will increase as a result of using cash to pay 50% of the accounts payable.

Question #54: Which one of the following statements is most correct if a seller extends credit to
a purchaser for a period of time longer than the purchaser's operating cycle? The seller
Correct Answer: B. Is. in effect, financing more than just the purchaser's inventory needs.
A. Has no need for a stated cash discount rate or credit period.
B. Is, in effect, financing more than just the purchaser's inventory needs.
C. Will have a lower level of accounts receivable than those companies whose credit period
is shorter than the purchasers operating cycle.
D. Can be certain that the purchaser will be able to convert the inventory into cash before
payment is due.
Explanation:
A. The operating cycle of the buyer does not impact whether or not there is a need for a cash
discount rate and a credit period.
B. If the seller extends credit that is a longer period of time than the buyer's operating cycle,
the seller is providing longer financing than the buyer needs. Because the operating cycle
is shorter than the credit terms, the buyer will have sold the inventory that they purchased
on credit and collected the cash before the purchase needs to be paid for. Therefore, the
buyer will be able to use the money elsewhere for some period of time before needing to
repay the loan. This provides the buyer with financing for more than the inventory itself.
C. The operating cycle of the buyer does not impact the level of receivables that the seller
will hold.
D. While it is likely that the inventory will be converted into cash before the payment is due,
it is not certain. The operating cycle is based on averages and for this specific inventory,
maybe the actual time for collection will be longer than the operating cycle.

Question#55: Question: Lisa, Inc.’s acid test(quick) ratio at December 31, Year 2, was
Lisa, Inc.
Statement of Financial Position
December 31, Year 2
(00s)
Year Year
Assets 2 1
Current assets:
Cash $30 $25
Trading securities 20 15
Accounts receivable (net) 45 30
Inventories (at lower of cost or market) 60 50
Prepaid items 15 20
Total current assets 170 140
Long term investments
securities (at cost) 25 20
Property, plant and equipment:
Land (at cost) 75 75
Building (net) 80 90
Equipment (net) 95 100
Intangible assets
Patents (net) 35 17
goodwill (net) 20 13
Total long term assets 330 315
Total assets $500 $455
Liabilities & shareholders' equity
current liabilities
notes payable $23 $12
accounts payable 47 28
accrued interest 15 15
Total current liabilities 85 55
long term debt:
notes payable 10% due 12/31/year 9 10 10
bonds payable 12% due 12/31/year 8 15 15
total long term debt 25 25
total liabilities 110 80
shareholders' equity
preferred--5% cumulative, $100 par, non-participating, 1,000 shares authorized,
issued and outstanding $100 $100
common--$10 par 20,000 shares authorized,15,000 issued and outstanding shares 150 150
Additional paid-in-capital--common 75 75
Retained earnings 65 50
total shareholders' equity $390 $375
total liabilities & equity $500 $455
Correct answer: A. 1.1:1.0
A. 1.1:1.0
B. 1.8:1.0
C. 2.0:1.0
D. 2.5:1.0
Explanation:
Answer (A) is correct.
The acid test, or quick, ratio is calculated by dividing total quick assets by current liabilities.
Quick assets are those that can be quickly converted into cash. Besides cash, they include trading
securities and accounts receivable. Lisa's quick assets total $95,000 ($30,000 + $20,000 +
$45,000). Dividing $95,000 by the $85,000 of current liabilities results in a ratio of 1.1.
Answer (B) is incorrect.
Erroneously including inventories in the numerator results in 1.8.
Answer (C) is incorrect.
The ratio 2.0 is obtained by dividing total current assets by total current liabilities.
Answer (D) is incorrect.
The quick ratio must be less than the current ratio.

Question #56: At the beginning of last year, a manufacturing company increased its selling price
by $10 per unit. This price increase has no effect on the volume of sales. As a result, operating
profit margin will
Correct Answer: A. Increase as a result of the price increase.
A. Increase as a result of the price increase.
B. Decline as a result of the price increase.
C. Remain unchanged.
D. Change as a result of the price increase, but the direction of such change cannot be
determined.
Explanation:
Answer (A) is correct.
Operating profit margin is the percentage of revenues that remains with the firm after costs of
merchandise, selling expenses, and general and administrative expenses have been paid.
Increasing sales will increase this profit margin.
Answer (B) is incorrect.
Operating profit margin is the percentage of revenues that remains with the firm after costs of
merchandise, selling expenses, and general and administrative expenses have been paid.
Operating profit margin will not decline as a result of the price increase: instead, it will increase.
Answer (C) is incorrect.
Operating profit margin is the percentage of revenues that remains with the firm after costs of
merchandise, selling expenses, and general and administrative expenses have been paid.
Increasing sales will increase this profit margin.
Answer (D) is incorrect.
Operating profit margin is the percentage of revenues that remains with the firm after costs of
merchandise, selling expenses, and general and administrative expenses have been paid.
Increasing sales will increase this profit margin: the direction can be determined.

Question #57: A company's cash ratio will decrease if the company


Correct Answer: B. Purchases materials on account.
A. Purchases commercial paper.
B. Purchases materials on account.
C. Sells goods for cash at a selling price lower than cost
D. Receives cash by issuing a short-term note payable.
Explanation:
Answer (A) is incorrect.
The cash ratio can be expressed as cash and marketable securities divided by current liabilities.
The outflow of cash to purchase the commercial paper will cause a decrease in the numerator,
while the inflow of the commercial paper will cause an equal increase in the numerator. These
effects will cancel each other out, leaving the ratio unchanged.
Answer (B) is correct.
The cash ratio can be expressed as cash and marketable securities divided by current liabilities. If
the company purchases materials on account, it will cause current liabilities to increase (the
denominator of the cash ratio) while having no effect on cash or marketable securities (the
numerator of the cash ratio). Because the denominator is increasing while the numerator is
staying constant, the ratio will decrease.
Answer (C) is incorrect.
The cash ratio can be expressed as cash and marketable securities divided by current liabilities.
Selling goods for cash at a selling price lower than cost will cause an increase in the numerator
as cash is increasing. This transaction would have no effect on the denominator. Therefore, the
cash ratio would increase, not decrease, as the numerator is increasing while the denominator is
staying the same.
Answer (D) is incorrect.
The cash ratio can be expressed as cash and marketable securities divided by current liabilities.
Receiving cash by issuing a short-term note payable would cause an equal increase in both the
numerator and the denominator. This would cause the overall ratio to increase, not decrease.

Question #58: The phrase "self insurance" is often used to mean the same thing as
Correct Answer: A. Risk retention.
A. Risk retention.
B. Risk sharing.
C. Risk transfer.
D. Risk reduction.
Explanation:
Answer (A) is correct.
Risk retention is the acceptance of the risk of an activity by the organization. This term is
becoming synonymous with the phrase "self insurance."
Answer (B) is incorrect.
Risk sharing is the offloading of some loss potential to another party. Common examples are the
purchase of insurance policies, engaging in hedging operations, and entering into joint ventures.
It is synonymous with risk transfer.
Answer (C) is incorrect.
Risk transfer, synonymous with risk sharing, is the offloading of some loss potential to another
party. Common examples are the purchase of insurance policies, engaging in hedging operations,
and entering into joint ventures.
Answer (D) is incorrect.
Risk reduction (mitigation) is the act of lowering the level of risk associated with an activity. For
instance, the risk of systems penetration can be reduced by maintaining a robust information
security function within the organization.
Question #59: The frequency of the comparison of recorded accountability with assets (for the
purpose of safeguarding assets) should be determined by
Correct Answer: B. The nature and amount of the asset and the cost of making the comparison.
A. The amount of assets independent of the cost of the comparison.
B. The nature and amount of the asset and the cost of making the comparison.
C. The cost of the comparison and whether the susceptibility to loss results from errors or
fraud.
D. The auditor in consultation with client management.
Explanation:
Answer (A) is incorrect.
The costs of controls should be considered when making the comparison.
Answer (B) is correct.
Assets should be compared with the recorded accountability as frequently as the nature and
amount of the assets require, within the limits of acceptable costs of comparison. The costs of
safeguarding assets should not exceed the expected benefits.
Answer (C) is incorrect.
Whether the susceptibility to loss arises from errors or fraud should have little bearing on the
frequency of the comparison.
Answer (D) is incorrect.
Management, not the auditor, has responsibility for internal control.

Question #60: Carco, Inc. wants to use discounted cash flow techniques when analyzing its
capital investment projects. The company is aware of the uncertainty involved in estimating
future cash flows. A simple method some companies employ to adjust for the uncertainty
inherent in their estimates is to
Correct Answer: D. Adjust the minimum desired rate of return.
A. Prepare a direct analysis of the probability of outcomes.
B. Increase the estimates of the cash flows.
C. Use accelerated depreciation.
D. Adjust the minimum desired rate of return.
Explanation:
A. Although the preparation of a direct analysis of the probability of outcomes may be used
to address uncertainty, it is not a simple method, and this question asks for a simple
method.
B. Increasing estimates of future expected cash flows arbitrarily is not an acceptable method
of addressing uncertainty in the capital budgeting process.
C. The use of accelerated depreciation is not a technique used to address uncertainty in
capital budgeting.
D. A company adjusts for the uncertainty inherent in its estimates by increasing the required
rate of return used to discount the future expected cash flows. A higher discount rate will
require higher expected future cash flows in order for the investment to be acceptable. As
a result, fewer investments will be acceptable.

Question #61: A corporation's return on equity can be calculated if you know its
Correct Answer: A. Sustainable equity growth rate and dividend payout ratio.
A. Sustainable equity growth rate and dividend payout ratio.
B. Debt-equity ratio and market-to-book ratio.
C. Market-to-book ratio and equity multiplier.
D. Dividend yield and earnings yield.
Explanation:
Answer (A) is correct.
The sustainable equity growth rate can be found by multiplying return on equity by 1 minus the
dividend payout ratio. Thus, the return on equity can be derived given the sustainable growth rate
and the dividend payout ratio.
Answer (B) is incorrect.
The market-to-book ratio cannot be used to calculate return on equity.
Answer (C) is incorrect.
The equity multiplier and return on assets could calculate return on equity.
Answer (D) is incorrect.
These numbers would not provide the information needed to find the net income or average total
equity.
Question #62: John Moore was recently hired as assistant controller of a manufacturing
company. The company controller. Nancy Kay has forecasted a 16% increase in annual earnings
however, during the last quarter of the year. John estimates that the company will only report a
12% increase in earnings. When he reports this to Nancy, she tells him that meeting the numbers
won't be a problem. She explains that there are several jobs in production that will finish after the
end of the fiscal year and she will record the associated revenue in the accounting system for the
current year.
What is the first step that John Moore should take at this time?
Correct Answer: B. Follow his organization's established policies regarding the resolution of this
type of conflict.
A. Contact his lawyer to determine his rights.
B. Follow his organization's established policies regarding the resolution of this type of
conflict.
C. Notify the audit committee of the issue.
D. Discuss the issue with the CFO of another company, who does not know any employees
at John's company.
Explanation:
Before taking any steps. John Moore should check to see if his organization has established
policies regarding how to handle this type of conflict. If such policies exist, then he should
follow them.

Question #63: Hagar Company's bank requires a compensating balance of 20% on a $100,000
loan. If the stated interest on the loan is 7%, what is the effective cost of the loan?
Correct Answer: D. 8.75%
A. 8.40%
B. 5.83%
C. 7.00%
D. 8.75%
Explanation:
A. This answer results from increasing the amount of interest due for one year by 20% and
dividing the result by $100,000. This is not the way a compensating balance requirement
works. In a loan with a compensating balance requirement, the borrower does not
actually receive all of the monies that are loaned because they must keep on deposit with
the bank some of the money as the compensating balance. However, they must pay
interest on the full amount of the loan. The interest payable divided by the usable funds
equals the effective annual rate.
B. This is the interest on $100,000 at 7% divided by the amount of the loan plus the
compensating balance. In a loan with a compensating balance the borrower does not
actually receive all of the monies that are loaned because they must keep on deposit with
the bank some of the money as the compensating balance. However, they must pay
interest on the full amount of the loan. Because of this the effective interest rate on the
loan is higher than the stated (or nominal) rate of the loan. The interest payable divided
by the usable funds equals the effective annual rate.
C. This is the stated interest rate on the loan. In a loan with a compensating balance the
borrower does not actually receive all of the monies that are loaned because they must
keep on deposit with the bank some of the money as the compensating balance. However,
they must pay interest on the full amount of the loan. Because of this the effective interest
rate on the loan is higher than the stated (or nominal) rate of the loan. The interest
payable divided by the usable funds equals the effective annual rate.
D. In a loan with a compensating balance the borrower does not actually receive all of the
monies that are loaned because they must keep on deposit with the bank some of the
money as the compensating balance. However, they must pay interest on the full amount
of the loan. Therefore, Hagar will pay $7,000 in interest (7% of $100,000) but will have
use of only $80,000. The interest payable divided by the usable funds equals the effective
annual rate. The effective interest rate is 8.75% ($7,000 ÷ $80,000).

Question #64: All decisions by financial managers should be driven by the primary goal to
Correct Answer: B. Maximize stockholder wealth.
A. Minimize fixed costs and variable costs.
B. Maximize stockholder wealth.
C. Maximize revenues.
D. Stabilize growth.
Explanation:
A. The company exists in order to provide a return to its owners. Therefore, management of
the company should operate in such a way so as to maximize the wealth that is generated
for the shareholders. Minimizing fixed and variable costs may not automatically do that.
B. Ideally all decisions are made with the best interests of the owners of the company in
mind. This is done by making decisions that maximize shareholder wealth.
C. The company exists in order to provide a return to its owners. Therefore, management of
the company should operate in such a way so as to maximize the wealth that is generated
for the shareholders. The maximization of revenue does not automatically do that.
D. The company exists in order to provide a return to its owners. Therefore, management of
the company should operate in such a way so as to maximize the wealth that is generated
for the shareholders. Stable growth does not automatically do that.
Question #65: Enterprise Risk Management (ERM) is closely aligned with corporate governance
because it
Correct Answer: B. Identifies which of the organizations' objectives is at greatest risk.
A. Focuses management's attention on the risks mitigated.
B. Identifies which of the organizations' objectives is at greatest risk.
C. Reduces the level of acceptable risks to be taken.
D. Identifies and isolates the silos in which risk exists.
Explanation:
Answer (A) is incorrect.
ERM focuses management's attention on all of the risks, especially ones that need to be
mitigated, mitigated.
Answer (B) is correct.
ERM recognizes risk management across the entire enterprise, so it identifies and responds to the
Managing the risks of an organization is one of the goals of corporate governance.
Answer (C) is incorrect.
ERM does not reduce the level of acceptable risk, but rather lowers the biggest risks of the
organization.
Answer (D) is incorrect.
ERM does not isolate risks: it assesses and responds to them.

Question #66: Identify which items below may communicate to employees a corporate
responsibility for ethical conduct: I. Defining organizational values. I. Leading by example. III.
Communicating legal acts. IV. Improving ethical compliance.
Correct Answer: B. I, II and IV, only.
A. I, II and III, only.
B. I, II and IV, only.
C. II and IV, only.
D. I and III, only.
Explanation:
There are five primary categories for management to focus on in order to effectively maintain the
desired ethical atmosphere. These can communicate a corporate responsibility for ethical conduct
and are: defining values, leadership by example, ethics and internal controls, practical
application, and measuring and improving ethical compliance.

Question #67: The I MA Statement of Ethical Professional Practice Credibility standard states
that each member has a responsibility to:
Correct Answer: C. Disclose all relevant information that could reasonably be expected to
influence an intended users understanding of the information.
A. Maintain an appropriate level of professional expertise by continually developing
knowledge and skills.
B. Provide decision support information and recommendations that are accurate.
C. Disclose all relevant information that could reasonably be expected to influence an
intended users understanding of the information.
D. Refrain from using confidential information for unethical or illegal advantage.
Explanation:
The IMA Statement of Ethical Professional Practice Credibility standard states that each member
has a responsibility to: communicate information fairly and objectively: disclose all relevant
information that could reasonably be expected to influence an intended user's understanding of
the reports, analyses, or recommendations: and disclose delays or deficiencies in information,
timeliness, processing, or internal controls in conformance with organization policy and/or
applicable law.

Question#68: Fact pattern: The information below pertains to Devlin Company.


Statement of Financial position as of May 31
(in thousand)

Year 2 Year 1
Assets
Current assets
Cash $45 $38
Trading Securities 30 20
Accounts receivable (net) 68 48
Inventory 90 80
Prepaid expenses 22 30
total current assets $255 $216
Investments, at equity 38 30
Property, plant, and equipment (net) 375 400
Intangible assets (net) 80 45
Total assets $748 $691
Liabilities
Current liabilities
notes payable $35 $18
accounts payable 70 42
Accrued expenses 5 4
Income taxes payable 15 16
Total current liabilities $125 $80
Long term debt 35 35
deferred taxes 3 2
total liabilities $163 $117
Equity
Preferred stock, 6%, $100 par value cumulative $150 $150
Common stock, $10 par value 225 195
Additional paid-in capital--common stock 114 100
Retained earnings 96 129
Total equity $585 $574

Income statement for the year ended May 31


(in thousands)

Year 2 Year 1
Net sales $480 $460
Costs and expenses
Costs of goods sold 330 315
Selling, general, and administrative 52 51
Interest expense 8 9
Income before taxes $90 $85
Income taxes 36 34
Net income $54 $51

Question: Devlin Company’s times interest earned ratio for the year ended May 31, Year 2 was
Correct answer: C. 12.25 times.
A. 6.75 times.
B. 11.25 times.
C. 12.25 times.
D. 18.75 times.
Explanation:
Answer (A) is incorrect.
This figure uses after tax income in the numerator.
Answer (B) is incorrect.
This figure equals income before taxes divided by interest.
Answer (C) is correct.
The times interest earned ratio equals income available to pay interest (net income + income
taxes + interest expense) divided by interest expense. The ratio is therefore 12.25 [($54 + $36 +
$8) ÷ $8].
Answer (D) is incorrect.
This figure results from adding selling, general, and administrative expenses to the numerator.

Question #69: Given fixed costs of $10,000, variable costs per unit of $195, and a unit selling
price of $370, how many units must be sold to reach a target operating income of $60,000? (Use
the contribution margin method.)
Correct Answer: • C. 400 units.
A. 343 units.
B. 358 units.
C. 400 units.
D. 189 units.
Explanation:
The formula for the contribution margin method to earn a target profit level is:
¿ costs+target operating income
QT =
Unit contribution margin
$ 10,000+ $ 60,000 $ 70,000
= = = 400 units
$ 175 $ 175
Question #70: All of the following are included when calculating the acid test ratio except
Correct Answer: B. Prepaid insurance.
A. Six-month treasury bills.
B. Prepaid insurance.
C. Accounts receivable.
D. 60-day certificates of deposit.
Explanation:
Answer (A) is incorrect.
Six-month treasury bills are marketable securities and thus are included in the numerator of the
acid test ratio.
Answer (B) is correct.
The acid test (quick) ratio consists of the quick assets (cash. marketable securities. and net
accounts receivable) divided by current liabilities. Prepaid insurance is an illiquid current asset
and thus not appropriate to include in the numerator.
Answer (C) is incorrect.
Accounts receivable are included in the numerator of the acid test ratio.
Answer (D) is incorrect.
Sixty-day certificates of deposit are cash equivalents and thus are included in the numerator of
the acid test ratio.

Question #71: Long Inc. is analyzing a $1 million investment in new equipment to produce a
product with a $5 per unit margin. The equipment will last 5 years, be depreciated on a straight-
line basis for tax purposes, and have no value at the end of its life. A study of unit sales produced
the following data.
Probabilit
Annual Unit sales y
80,000 0.10
85,000 0.20
90,000 0.30
95,000 0.20
100,000 0.10
110,000 0.10
If long utilizes a 12% hurdle rate and is subject to a 40% effective income tax rate, the expected
net present value of the project would be
Correct Answer: D. $283,380.
A. $261,750.
B. $297,800.
C. $427,580.
D. $283,380.
Explanation:
A. This answer results from using the annual unit sales with the highest probability of
occurring (90,000) as the expected sales used in calculating the net present value of the
project. The expected sales should be a weighted average of the possible annual unit
sales, weighted according to probabilities.
B. This answer results from using a simple average of the various possible annual unit sales
as the expected sales used in calculating the net present value of the project. The expected
sales should be a weighted average of the possible annual unit sales, weighted according
to probabilities.
C. This answer results from multiplying the annual projected depreciation ($200,000) by 1-
the tax rate to calculate the annual depreciation tax shield. To calculate the depreciation
tax shield, the annual depreciation should be multiplied by the tax rate, not 1- the tax rate.
D. Using the annual unit sales and the probabilities given for each possible sales volume, we
first calculate the expected sales as a weighted average: (80,000 x 0.10) + (85,000 x 0.20)
+ (90,000 x 0.30) + (95,000 x 0.20) + (100,000 x 0.10) + (110,000 x 0.10) = 92,000 units.
Expected sales of 92,000 units x $5 per unit contribution margin = $460,000 net annual
cash flow before tax. $460,000 x (1 - 0.40) = $276,000 net annual cash flow after tax.
The depreciation tax shield is $1,000,000/ 5 = annual depreciation of $200,000, and
$200,000 x 0.40 = the depreciation tax shield of $80,000 per year. So the net annual after-
tax cash flow for each of the 5 years is $276,000 + $80,000, which is $356,000.
Discounting $356,000 using the PV of an annuity factor for 12% for 5 years, we have
$356,000 x 3.605, which is $1,283,380, $1,283,380 - the initial investment of $1,000,000
= NPV of $283,380.

Question #72: ABC Company has fixed costs of $300,000 per month. Total output per month is
150,000 units. Minimum pay for production line workers is $5.85 per hour, and total variable
costs are currently $275,000 per month. If variable costs increase to $350,000 per month and
production output increases to 250,000 per month, what is the marginal cost for increasing
production?
Correct Answer: D. $0.75 per unit marginal cost.
A. $1.83 per unit marginal cost.
B. $1.40 per unit marginal cost.
C. $3.83 per unit marginal cost.
D. $0.75 per unit marginal cost.
Explanation:
Marginal cost is calculated by dividing the changes in total costs ($350,000 variable cost -
$275,000 variable costs before the increase in production = $75,000 increased costs) by the
increase in production (100,000 units). $75,000 + 100,000 units = $0.75 per unit marginal cost.

Question #73: Value-based pricing:


Correct Answer: D. is an estimate based on the customer's perceived value of the product.
A. Is used for short-term pricing decisions.
B. Relevant costs are all variable costs.
C. Is one form of cost-based pricing.
D. Is an estimate based on the customer's perceived value of the product.
Explanation:
Value-based pricing is setting the price of a product or service based on what the price setter
believes regarding the customer's perceived value of the product.

Question #74: Yipann Corporation is reviewing an investment proposal. The initial cost as well
as other related data for each year are presented in the schedule below. All cash flows are
assumed to take place at the end of the year. The salvage value of the investment at the end of
each year is equal to its net book value, and there will be no salvage value at the end of the
investment's life.
Investment Proposal

Initial cost & book Annual net after tax cash Annual net
Year value flows income
0 $105,000 $0 $0
1 70,000 50,000 15,000
2 42,000 45,000 17,000
3 21,000 40,000 19,000
4 7,000 35,000 21,000
5 0 30,000 23,000

Yipann uses a 24% after-tax target rate of return for new investment proposals. The discount
figures for a 24% rate of return are given.

Present value of $1 received at the end of Present value of Annuity of $1


Year period received at end of each period
1 0.81 0.81
2 0.65 1.46
3 0.52 1.98
4 0.42 2.40
5 0.34 2.74
6 0.28 3.02
7 0.22 3.24

The average annual cash inflow at which Yipann would be indifferent to the investment (rounded
to the nearest dollar) is
Correct Answer: A. $38,321.
A. $38,321.
B. $46,667.
C. $21,000.
D. $40,000.
Explanation:
A. The question is asking for an average annual after-tax cash flow amount that will result in
a net present value of zero for the project, because that will be the average annual cash
flow level at which Yipann will be indifferent to the investment. We need to look at this
as a present value of an annuity problem, because since we are looking for an average
annual cash flow amount, all the annual cash flow amounts after year 0 will be the same
average amount. The annual cash flows given in the problem are irrelevant, because we
are looking for the average annual after-tax cash flow amount that will result in an NPV
of zero, given the initial investment in Year 0. Since the initial investment is $105,000
and the project's life is 5 years, we need to know what annuity amount will produce a
present value of $105,000 when discounted at 24% for 5 years. Recall that the present
value of an annuity is the annuity amount x PV of an annuity factor. We don't know the
annuity amount, but we do know the PV of an annuity factor and the present value
amount of $105,000. The PV of an annuity factor for 5 years at 24% is given in the
problem: 2.74. Thus, the formula is: Annuity Amount x 2.74 - $105,000. Therefore, the
Annuity Amount - $105,000 ÷ 2.74, which is equal to $38,321. This means that if the
annual after-tax cash flows are all the same and they are $38,321, the NPV will be zero
and the company will be indifferent indifferent to whether or not it makes the investment.
If it makes the investment, the investment will provide no additional value to the
shareholders, and the shareholders will gain nothing. If it doesn't make the investment,
the shareholders will lose nothing.
B. An answer of $46,667 results from dividing the initial investment amount of $105,000 by
the payback period.
C. An answer of $21,000 results from dividing the initial cost of the asset by the number of
years of the project's life.
D. $40,000 is the average of the five net after tax cash flows given in the problem.

Question #75: Specialty Cakes Inc. produces two types of cakes, a round cake and a heart-shaped
cake. Total fixed costs for the firm are $92.000. Variable costs and sales data for these cakes are
presented below.
Round
Cake Heart-shaped Cake
Selling price per unit $12 $20
Variable cost per unit $8 $15
Budgeted sales (units) 10,000 15,000

How many cakes will be required to reach the breakeven point?


Correct Answer: D. 8,000 round cakes and 12.000 heart-shaped cakes.
A. 9,000 round cakes and 11,000 heart-shaped cakes.
B. 10,000 round cakes and 10,000 heart-shaped cakes.
C. 23,000 round cakes and 18,400 heart-shaped cakes.
D. 8,000 round cakes and 12,000 heart-shaped cakes.
Explanation:
The breakeven point in units can be calculated by dividing fixed costs by contribution margin per
unit. Since there are two products, we calculate the weighted contribution margin by weighting
the products' individual contribution margins by budgeted sales.
Weighted contribution margin = ((10,000 / 25,000) x $4) + ((15,000 / 25,000) x $5) = $4.60
Breakeven point (total number of units. round and heart shaped cakes) = $92,000/$4.60 = 20,000
units (round and heart shaped cakes)
Round cakes: (10,000 /25,000) x 20,000 = 8,000 round cakes Heart-shaped cakes: (15,000 /
25,000) x 20,000) = 12,000 heart-shaped cakes.
Question #76: A firm uses the following model to determine the optimal average cash balance
(Q).

Q=
√ 2 × Annual cash disbursement ×cost per sale of T −bill
interst rate
An increase in which one of the following would result in a decrease in the optimal cash
balance?
Correct Answer: B. Return on marketable securities.
A. Cash requirements for the year.
B. Return on marketable securities.
C. Uncertainty of cash outflows.
D. Cost of a security trade
Explanation:
A. The formula given is the Baumol Cash Management Model. An increase in the cash
requirements for the year would cause an increase, not a decrease, in the optimal cash
balance according to the model. That is because an increase in the numerator of the
fraction under the radical sign will increase the value of the fraction, thereby increasing
the square root of the value and the optimal average cash balance.
Using the Baumol Cash Management Model, this can be illustrated by assigning some
values to it. Let's say the cost per sale of a T-Bill is $20 (that is the commission paid, or
the fixed cost per transaction to convert a T-Bill into cash). And let's say the annual cash
disbursement amount, or the total demand for cash for the period, is $50,000. And let's
say the interest rate on marketable securities is 2%, or 0.02. The optimal cash balance
will be the square root of [(2 x 20 x 50,000) / 0.02]:
(2 x 20 x 50,000) = 2,000,000. 2,000,000 /0.02 = 100,000,000. The square root of
100,000,000 = $10,000.
Now, let's increase the cash requirements for the year to $75,000. The optimal cash
balance will be the square root of [(2 x 20 x 75,000) /0.02]: (2 x 20 x 75,000) =
3,000,000. 3,000,000 /0.02 = 150,000,000. The square root of 150,000,000 = $12,247.
So because the cash requirements for the year have increased, the optimal cash balance
has also increased from $10,000 to $12,247. It has not decreased.
B. An increase in the return on marketable securities would increase the opportunity cost of
holding cash instead of investing it in marketable securities. When the opportunity cost of
holding cash increases, the amount of cash a business will want to hold will decrease.
That is because an increase in the denominator of the fraction under the radical sign will
decrease the value of the fraction, thereby decreasing the square root of the value and the
optimal average cash balance. The formula given is the Baumol Cash Management
Model. Using the Baumol Cash Management Model, this answer can be illustrated by
assigning some values to it. Let's say the cost per sale of a T-Bill is $20 (that is the
commission paid, or the fixed cost per transaction to convert a T-Bill into cash). And let's
say the annual cash disbursement amount, or the total demand for cash for the period, is
$50,000. And let's say the interest rate on marketable securities is 2%, or 0.02.
The optimal cash balance will be the square root of [(2 x 20 x 50,000) / 0.02]: (2 x 20 x
50,000) = 2,000,000. 2.000,000 / 0.02 = 100,000,000. The square root of 100,000,000 =
$10,000. Now, let's increase the interest rate on marketable securities to 3%, or 0.03: An
increase in the return on marketable securities would increase the opportunity cost of
holding cash instead of investing it in marketable securities. When the opportunity cost of
holding cash increases, the amount of cash a business will want to hold will decrease.
That is because an increase in the denominator of the fraction under the radical sign will
decrease the value of the fraction, thereby decreasing the square root of the value and the
optimal average cash balance. The formula given is the Baumol Cash Management
Model. Using the Baumol Cash Management Model, this answer can be illustrated by
assigning some values to it. Let's say the cost per sale of a T-Bill is $20 (that is the
commission paid, or the fixed cost per transaction to convert a T-Bill into cash). And let's
say the annual cash disbursement amount, or the total demand for cash for the period, is
$50,000. And let's say the interest rate on marketable securities is 2%, or 0.02. The
optimal cash balance will be the square root of [(2 x 20 x 50,000) / 0.02]: (2 x 20 x
50,000) = 2,000,000. 2,000,000 / 0.02 = 100,000,000. The square root of 100,000,000 =
$10,000. Now, let's increase the interest rate on marketable securities to 3%, or 0.03: An
increase in the return on marketable securities would increase the opportunity cost of
holding cash instead of investing it in marketable securities. When the opportunity cost of
holding cash increases, the amount of cash a business will want to hold will decrease.
That is because an increase in the denominator of the fraction under the radical sign will
decrease the value of the fraction, thereby decreasing the square root of the value and the
optimal average cash balance. The formula given is the Baumol Cash Management
Model. Using the Baumol Cash Management Model, this answer can be illustrated by
assigning some values to it. Let's say the cost per sale of a T-Bill is $20 (that is the
commission paid, or the fixed cost per transaction to convert a T-Bill into cash). And let's
say the annual cash disbursement amount, or the total demand for cash for the period, is
$50,000. And let's say the interest rate on marketable securities is 2%, or 0.02. The
optimal cash balance will be the square root of [(2 x 20 x 50,000) / 0.02]: (2 x 20 x
50,000) = 2,C00,000. 2.000,000 / 0.02 = 100,000,000. The square root of 100,000,000 =
$10,000. Now, let's increase the interest rate on marketable securities to 3%, or 0.03:
2,000,000 / 0.03 = 66,666,666.67. The square root of 66,666,666.67 = $8,165. So because
the interest rate has increased, the optimal cash balance has decreased from $10,000 to
$8,165.
C. The formula given is the Baumol Cash Management Model. The uncertainty of cash
outflows is a fact that limits the effectiveness of the Baumol Cash Management Model. It
does not result in either an increase or a decrease in the optimal cash balance calculated
using the model.
D. The formula given is the Baumol Cash Management Model. An increase in the cost of a
security trade would cause an increase, not a decrease, in the optimal cash balance
according to the model. That is because an increase in the numerator of the fraction under
the radical sign will increase the value of the fraction, thereby increasing the square root
of the value and the optimal average cash balance. Using the Baumol Cash Management
Model, this can be illustrated by assigning some values to it. Let's say the cost per sale of
a T-Bill is $20 (that is the commission paid, or the fixed cost per transaction to convert a
T-Bill into cash). And let's say the annual cash disbursement amount, or the total demand
for cash for the period, is $50,000. And let's say the interest rate on marketable securities
is 2%, or 0.02. The optimal cash balance will be the square root of [(2 x 20 x 50,000) /
0.02]: (2 x 20 x 50,000) = 2,000,000. 2,000,000 /0.02 = 100,000,000. The square root of
100,000,000 = $10,000. Now, let's increase the cost per sale of a T-Bill to $30. The
optimal cash balance will be the square root of [(2 x 30 x 50,000) / 0.02]: (2 x 30 x
50,000) = 3,000,000. 3,000,000 /0.02 = 150,000,000. The square root of 150,000,000 =
$12,247. So because the cost per sale of a T-Bill has increased, the optimal cash balance
has also increased from $10,000 to $12,247. It has not decreased.

Question #77: Which of the following is a primary benefit of involving cross-functional teams in
the target costing process?
Correct Answer: D. Better assurance that the proposed product or service generates the desired
profit margin
A. More accurate input and maintenance of cost table databases
B. More realistic market assessment of customer wants and needs
C. Improved analysis of how to gain and sustain competitive advantage
D. Better assurance that the proposed product or service generates the desired profit margin
Explanation:
The cross-functional participation of research and design, engineering, production, marketing,
and accounting are necessary to ensure that the proposed product or service when sold generates
the desired profit margin. Stated another way, the cross-functional team is given the
responsibility to design and develop the product or service so that it can be made for the target
cost.

Question #78: A widely used approach that is used to recognize uncertainty about individual
economic variables while obtaining an immediate financial estimate of the consequences of
possible prediction errors is
Correct Answer: C. Sensitivity analysis.
A. Expected value analysis.
B. Regression analysis.
C. Sensitivity analysis.
D. Learning curve analysis.
Explanation:
A. Expected value is the weighted average of all the possible values of a random variable,
with the probabilities of each of the values used as the weights. The expected value is the
mean value, also known as the average value. Expected value analysis does not yield an
immediate financial estimate of the consequences of possible prediction errors.
B. Regression analysis is a method of forecasting, using trend analysis. However, it does not
yield an immediate financial estimate of the consequences of possible prediction errors.
C. Sensitivity analysis can be used to determine how cash flows can be expected to vary
with changes in the underlying assumptions. Using expected cash flows, the NPV, IRR,
and PI of the project are determined. Then, the key assumptions that were used in making
the original expected cash flow projections are identified. One assumption at a time is
then changed, leaving the other assumptions unchanged, and the NPV, IRR and PI are
recalculated to determine what effect changing one assumption would have on those
measures.
D. Learning curves describe the fact that the more experience people have with something,
the more efficient they become in doing that task. Higher costs per unit early in
production are part of start-up costs. It is commonly accepted that new products and
production processes experience a period of low productivity followed by increased
productivity. However, learning curve analysis does not provide an immediate financial
estimate of the consequences of possible prediction errors.

Question #79: Capital Company has decided to discontinue a product produced on a machine
purchased four years ago at a cost of $70,000. The machine has a current book value of $30,000.
Due to technologically improved machinery now available in the marketplace the existing
machine has no current salvage value. The company is reviewing the various aspects involved in
the production of a new product. The engineering staff advised that the existing machine can be
used to produce the new product. Other costs involved in the production of the new product will
be materials of $20,000 and labor priced at $5,000. Ignoring income taxes, the costs relevant to
the decision to produce or not to produce the new product would be:
Correct Answer: D. $25,000.
A. $95,000.
B. $55,000.
C. $30,000.
D. $25,000.
Explanation:
The costs relevant to the decision to produce or not to produce the new product total $25,000,
and are comprised of the $20,000 cost of materials and the $5,000 cost of labor. The costs
associated with the old machine are irrelevant: they are sunk, historical costs.

Question #80: Little Baby Inc. market research indicates that the firm faces an elastic demand
curve for its infant products. What pricing strategy should the firm adopt if it wants to boost its
infant product revenue?
Correct Answer: A. Decrease prices.
A. Decrease prices.
B. Reduce its output.
C. Increase prices.
D. Do not change the prices.
Explanation:
For a firm that faces an elastic demand curve, lowering its price will lead to an increase in total
revenue. Revenue gained from the increase in quantity demanded will more than offset revenue
lost from the decrease in price.

Question #81: Rogers Inc. operates a chain of restaurants located in the Southeast. The company
has steadily grown to its present size of 48 restaurants. The board of directors recently approved
a large-scale remodeling of the restaurant, and the company is now considering two financing
alternatives.
The first alternative would consist of Bonds that would have a 9% effective annual rate and
would net $19.2 million after flotation costs Preferred stock with a stated rate of 6% that would
yield $4.8 million after a 4% flotation cost Common stock that would yield $24 million after a
5% flotation cost
The second alternative would consist of a public offering of bonds that would have an 11%
effective annual rate and would net $48 million after flotation costs.
Rogers' current capital structure, which is considered optimal. consists of 40% long-term debt.
10% preferred stock, and 50% common stock. The current market value of the common stock is
$30 per share, and the common stock dividend during the past 12 months was $3 per share.
Investors are expecting the growth rate of dividends to equal the historical rate of 6%. Rogers is
subject to an effective income tax rate of 40%. The interest rate on the bonds is greater for the
second alternative consisting of pure debt than it is for the first alternative consisting of both debt
and equity because
Correct Answer: B. The pure debt alternative carries the risk of increasing the probability of
default.
A. The combination alternative carries the risk of increasing dividend payments.
B. The pure debt alternative carries the risk of increasing the probability of default.
C. The diversity of the combination alternative creates greater risk for the investor.
D. The pure debt alternative would flood the market and be more difficult to sell.
Explanation:
A. Because the combination alternative maintains the same debt-equity mixture, which
would not warrant a rate increase in the cost of debt or equity.
B. As a larger proportion of an entity's capital is provided by debt, the debt becomes riskier
and more expensive. Hence, it requires a higher interest rate.
C. Because the diversity decreases, not increases, risk.
D. Because $50,000,000 is minuscule in the debt markets.

Question #82: A corporation has established a risk management process to help it create, protect,
and enhance shareholder value. Which of the following reflects the best order for that risk
process?
Correct Answer: A. Objective setting, event identification, risk assessment, risk response.
A. Objective setting, event identification, risk assessment, risk response.
B. Event identification, objective setting, risk assessment, risk response.
C. Risk assessment, risk response, objective setting, event identification.
D. Risk assessment, objective setting, event identification, risk response.
Explanation:
Answer (A) is correct.
The order of the Enterprise Resource Management program is outlined in the COSO Framework.
After the internal environment is established, objectives must be set, and events that can affect
those objectives must be identified. Once that is complete, the risks must be assessed, and then
possible responses to those risks should be identified and evaluated.
Answer (B) is incorrect.
Objectives must exist before management can identify potential events affecting their
achievement.
Answer (C) is incorrect.
Assessing and responding to risks must occur after objectives have been set, and events affecting
those objectives are identified.
Answer (D) is incorrect.
Assessing the risks must occur after objectives have been set, and events affecting those
objectives are identified.

Question #83: Joseph Krohn. Chairman of Quantum Financing Co., has directed Human
Resources to implement ongoing ethics training. Krohn considers this training critically
important as it:
Correct Answer: A. lessens the penalty if the corporation is convicted of improper conduct.
A. Lessens the penalty if the corporation is convicted of improper conduct.
B. Only applies to lower level employees most likely to behave unethically.
C. Will guarantee improper conduct will not occur.
D. Absolves the corporation from prosecution for improper conduct.
Explanation:
Section 406 of the 2002 Sarbanes/Oxley Act requires publicly held companies to disclose
whether the company has a code of ethics for is officers and senior executives. A benefit of such
a code is the possible lessening of penalties it the company is convicted of improper conduct.

Question #84: Crown Corporation has agreed to sell some used computer equipment to Bob
Parsons, one of the company's employees, for $5,000. Crown and Parsons have been discussing
alternative financing arrangements for the sale. Crown Corporation has offered to accept a
$1,000 down payment and set up a note receivable for Bob Parsons that calls for a $1,000
payment at the end of each of the next 4 years. If Crown uses a 6% discount rate, the present
value of the note receivable would be
Correct Answer: B. $3,465
A. $4,465
B. $3,465
C. $4,212
D. $2,940
Explanation:
A. An answer of $4,465 results from discounting the note receivable using the present value
of an annuity of $1,000 and a 6% rate for 4 years and then adding the $1,000 down
payment at its undiscounted value of $1,000. However, the question asks only for the
present value of the note receivable, not the present value of the entire transaction.
B. The note calls for four annual payments of $1,000. This is an ordinary annuity, since the
payments are due at the end of each period. Therefore, the factor in the present value of
an annuity table can be used as it is given, without adjustment. The present value of a
four-year ordinary annuity of $1,000, discounted at 6%, is $1,000 x 3.465, or $3,465.
C. An answer of $4,212 results from using the present value of an annuity factor for 6% for
5 years. However, the note is for four years.
D. An answer of $2,940 results from discounting the entire amount of the note ($4,000) at
8% for four years. This is incorrect for two reasons: (1) The full principle of the note
($4,000) is assumed to be paid at the maturity date in four years. However, the note calls
for annual principle payments of $1,000. (2) The discount rate used is 8%. However, the
discount rate to be used is 6%.

Question #85: Which practice would be most useful in a firm's attempt to close the gap between
current and allowable costs during the design process?
Correct Answer: D. Value engineering.
A. Quality function deployment
B. Reverse engineering.
C. Life-cycle costing.
D. Value engineering.
Explanation:
By definition, value engineering (or value analysis) is a principal technique in closing the gap
between current cost and allowable cost. It is the systematic analysis of product or service
design, materials, specifications, and production processes in the context of the customer
requirements.

Question #86: Given a spot rate of $1.8655 and a 90-day forward rate of $1.8723, the pound
sterling in the forward market is:
Correct Answer: D. Being quoted at a premium.
A. Overvalued.
B. Undervalued.
C. Being quoted at a discount.
D. Being quoted at a premium.
Explanation:
A. By definition, if the spot rate is less than the forward rate, the currency is said to be
selling at a premium in the forward market. This does not necessarily indicate anything
about the value of the currency.
B. By definition, if the spot rate is less than the forward rate, the currency is said to be
selling at a premium in the forward market. This does not necessarily indicate anything
about the value of the currency.
C. By definition, if the spot rate is less than the forward rate, the currency is said to be
selling at a premium in the forward market.
D. By definition, if the spot rate is less than the forward rate, the currency is said to be
selling at a premium in the forward market.

Question #87: The management of Giga Foods announced that it is dropping the price of its
product line from $15 to $10 per unit, and as a result of the price drop, sales volume will now be
120,000 units. Suppose the company is targeting an operating income of $1.20 per unit. What is
the company's target operating income in percentage?
Correct Answer: C. 12%.
A. 11%.
B. 15%.
C. 12%.
D. 10%.
Explanation:
The target income percentage = ((target income per unit) / (target sales price per unit)) multiplied
by 100
So, (1.2/10)(100) = 12%.

Question #88: A company uses the direct write-off method to account for uncollectible accounts
receivable. If the company subsequently collects an account receivable that was written off in a
prior accounting period, the effect of the collection of the account receivable on the current ratio
and total working capital would be
Correct Answer: C. Increase (Current Ratio). (Working Capital) Increase
A. None (Current Ratio). (Working Capital) None
B. Increase (Current Ratio). (Working Capital) Increase
C. Increase (Current Ratio). (Working Capital) None
D. None (Current Ratio). (Working Capital) Decrease
Explanation:
Answer (A) is incorrect.
The current ratio and working capital increase.
Answer (B) is correct.
Because the company uses the direct write-off method, the original entry involved a debit to a
bad debt expense account (closed to retained earnings). The subsequent collection required a
debit to cash and a credit to bad debt expense or retained earnings. Thus, only one current asset
account was involved in the collection entry, and current assets (cash) increased as a result. If
current assets increase and no change occurs in current liabilities, the current ratio and working
capital both increase.
Answer (C) is incorrect.
The current ratio and working capital increase.
Answer (D) is incorrect.
The current ratio and working capital increase.

Question #89: All of the following are potential benefits of risk management except
Correct Answer: D. Reduced inherent risk.
A. Lower cost of capital.
B. Efficient allocation of resources.
C. Flexibility in responding to unforeseen circumstances.
D. Reduced inherent risk.
Explanation:
Answer (A) is incorrect.
Reassuring investors is one of the benefits of risk management. Corporations with strong risk
management functions will probably have a lower cost of capital.
Answer (B) is incorrect.
Efficient use of resources is one of the benefits of risk management. Only after risks are
identified can resources be directed toward those with the greatest exposure.
Answer (C) is incorrect.
Encountering fewer surprises is one of the benefits of risk management. After a comprehensive,
organization-wide risk assessment has been performed, the odds of an incident arising that has
never been considered are greatly reduced.
Answer (D) is correct.
Inherent risk is the risk of an activity that arises from the activity itself. For example, uranium
prospecting is inherently riskier than retailing.

Question #90: Fact Pattern: The data presented below show actual figures for selected accounts
of McKeon Company for the fiscal year ended May 31, Year 1, and selected budget figures for
the Year 2 fiscal year. McKeon's controller is in the process of reviewing the Year 2 budget and
calculating some key ratios based on the budget. McKeon Company monitors yield or return
ratios using the average financial position of the company. (Round all calculations to three
decimal places if necessary.)

5/31/year 2 5/31/year 1
Current assets $210,000 $180,000
noncurrent assets 275,000 255,000
current liabilities 78,000 85,000
long term debt 75,000 30,000
common stock ($30 par value) 300,000 300,000
Retained earnings 32,000 20,000

year 2
oeparations
Sales * $350,000
costs o fgoods sold 160,000
interest expense 3,000
income taxes (40% rate) 48,000
dividends declared and paid in
year 2 60,000
administrative expense 67,000

*all sales are credit sale

Current assets
5/31/year 2 5/31/year 1
Cash $20,000 $10,000
Accounts receivable 100,000 70,000
inventory 70,000 80,000
Prepaid expenses 20,000 20,000

Question: McKeon Company's debt ratio for Year 2 is


Correct Answer: B. 0.315
A. 0.352
B. 0.315
C. 0.264
D. 0.237
Explanation:
Answer (A) is incorrect.
The debt ratio equals total debt at year-end divided by total assets at year-end.
Answer (B) is correct.
The debt ratio is equal to the total debt at year-end divided by total assets at year-end. Total debt
at year-end is $153,000 ($78,000 current liabilities + $75,000 long-term debt). Total assets equal
$485,000 ($210,000 current assets + $275,000 noncurrent assets). Thus, the debt ratio is .315
($153,000 ÷ $485,000).
Answer (C) is incorrect.
The debt ratio equals total debt at year-end divided by total assets at year-end.
Answer (D) is incorrect.
The debt ratio equals total debt at year-end divided by total assets at year-end.

Question #91: Fact Pattern: Depoole Company is a manufacturer of industrial products that uses
a calendar year for financial reporting purposes. Assume that total quick assets exceeded total
current liabilities both before and after the transaction described. Further assume that Depoole
has positive profits during the year and a credit balance throughout the year in its retained
earnings account.
Question: Depoole's purchase of raw materials for $85.000 on open account will
Correct Answer: B. Decrease the current ratio.
A. Increase the current ratio.
B. Decrease the current ratio.
C. Increase net working capital.
D. Decrease net working capital.
Explanation:
Answer (A) is incorrect.
The current ratio is decreased.
Answer (B) is correct.
The purchase increases both the numerator and denominator of the current ratio by adding
inventory to the numerator and payables to the denominator. Because the ratio before the
purchase was greater than 1. The ratio is decreased.
Answer (C) is incorrect.
The purchase of raw materials on account has no effect on working capital (current assets and
current liabilities change by the same amount).
Answer (D) is incorrect.
The purchase of raw materials on account has no effect on working capital (current assets and
current liabilities change by the same amount).

Question #92: Determining the appropriate level of working capital for a firm requires
Correct Answer: A. Offsetting the benefit of current assets and current liabilities against the
probability of technical insolvency.
A. Offsetting the benefit of current assets and current liabilities against the probability of
technical insolvency.
B. Maintaining a high proportion of liquid assets to total assets in order to maximize the
return on total investments.
C. Changing the capital structure and dividend policy of the firm.
D. Maintaining short-term debt at the lowest possible level because it is generally more
expensive than long-term debt.
Explanation:
A. Working capital management requires balancing the risk of insolvency with the benefits
of longer-term, higher-return investments.
B. Maintaining high levels of liquid assets would not maximize the return on total assets
because short-term assets have a lower return than longer-term assets.
C. The capital structure and dividend policy of a firm are not part of the working capital
management process.
D. Short-term debt is generally less expensive than long-term debt and working capital
management is more than just debt management.
Question #93: For a given level of sales and holding all other financial statement items constant,
a company's return on equity (ROE) will
Correct Answer: C. Decrease as their total assets increase.
A. Increase as their debt ratio decreases.
B. Decrease as their cost of goods sold as a percent of sales decrease.
C. Decrease as their total assets increase.
D. Increase as their equity increases.
Explanation:
Answer (A) is incorrect.
The debt ratio is total liabilities to total assets. If this ratio decreases, then either liabilities went
down or assets went up or both: since all other financial statement line items are held constant, it
will have no effect on the return on equity.
Answer (B) is incorrect.
If cost of goods sold as a percent of sales decreases, return on equity will not be affected.
Answer (C) is correct.
A firm's return on equity is a measure of how much equity capital is employed to generate its
level of earnings. In this case, an increase in total assets means an increase in equity (since all
other financial statement items are being held constant). Equity is the denominator, and an
increase in the denominator means a decrease in the overall ratio.
Answer (D) is incorrect. If equity increases, return on equity will decrease.

Question #94: Julie is the Senior Management Accountant for Hazelton Manufacturing; a multi-
national telecommunications company. In an effort to expand operations overseas. Hazelton
encourages senior management to offer payment to foreign officials to win business. These
payments are classified as normal commission expenses by the accounting department. Julie
recently questioned whether these payments should be classified as commission expense since
they appear to be bribes to foreign officials. When Julie discussed the situation with the Chief
Financial Officer, he stated these payments were not illegal and were expected within these
foreign countries. When all employees are hired, they are required to sign the company code of
conduct. In addition, the company provides annual ethics training to all employees and each
employee is evaluated based on compliance with operational goals and ethical expectations. The
company provides an anonymous whistleblower hotline for employees to report concerns to
management. Julie believes that the company she works for has an ethical organizational culture.
Identify a requirement of the U.S. Foreign Corrupt Practices Act (FCPA) relevant in the
Hazelton Manufacturing case.
Correct Answer: B. The Act forbids an American company to pay bribes to foreign government
officials.
A. The Act requires senior financial officers to follow a code of ethics.
B. The Act forbids an American company to pay bribes to foreign government officials.
C. The Act requires employee training for maintaining an ethical organizational culture.
D. The Act requires a company to provide a whistleblowing hotline to report ethics
concerns.
Explanation:
The FCPA forbids an American company doing business in another country to pay bribes to a
foreign government to obtain contracts or secure business.

Question #95: A market research analyst determined the following market data for a commodity.
Quantit
y quantity
supplie demande
Price d d
$25 250 750
50 500 500
75 750 250
100 1,000 0

Based on this information, which one of the following statements is correct?


Correct Answer: C. At a price of $30, there will be excess demand.
A. A market clearing price cannot be determined.
B. A market price of $50 cannot exist for very long.
C. At a price of $30, there will be excess demand.
D. At a price of $80, there will be insufficient supply.
Explanation:
The market clearing price of $50 occurs when supply equals demand (500 units). Any price less
than $50 will create excess demand.
Question #96: Which of the following, when considered individually, would generally have the
effect of increasing a firm's cost of capital?
I. The firm reduces its operating leverage.
II. The corporate tax rate is increased.
III. The firm pays off its only outstanding debt.
IV. The Treasury Bond yield increases.
Correct Answer: C. III and IV.
A. II and IV.
B. I, III and IV.
C. III and IV.
D. I and III.

Explanation:
A. An increase in the corporate tax rate would not cause the firm's cost of capital to increase,
if the corporate tax rate increases, this will decrease the net amount of interest expense
(after taxes) that the firm has to pay on its debt, and that would also decrease the firm's
cost of capital. To understand why this is so, suppose a company has net income before
interest and taxes of $100,000. The tax rate is 40%. Interest expense is $10,000. So net
income after subtracting interest expense is $90,000. Income tax is 40% of that, or
$36,000, so net income after interest and taxes are deducted is $54,000. If the company
had not had the interest expense, its net income before tax would have been $100,000. Its
tax would have been $40,000, and its net income would have been $60,000. Therefore,
the company's interest expense, net of taxes, is the difference between $60,000 net
income and $54,000 net income, which is only $6,000. That is less than the $10,000 that
the company actually paid in interest. The reason is because taxes on the company's net
income reduced its effective interest rate. Now, suppose the income tax rate increases to
45%. Net income after subtracting interest expense is still $90,000. But now, income tax
is 45% of that, or $40,500. Net income after tax is now $49,500. Let's compare that again
with what the company's net income would have been without the interest expense. Net
income would have been $100,000, and income tax would have been $45,000. The
company's net income would have been $55,000. So the company's interest expense, net
of taxes, is the difference between $55,000 net income without interest expense and
$49,500 net income with interest expense, or $5,500. The net amount of interest expense,
after taxes, for the company is actually less ($5,500 versus $6,000) than it was when the
tax rate was 40%.
B. Reducing its operating leverage would not cause the firm's cost of capital to increase.
Reducing its operating leverage would decrease the firm's cost of capital, because it
would decrease the amount of risk that investors would perceive in the company.
C. If the company pays off its only outstanding debt, its interest expense will go away
completely. Remember that the weighted average cost of capital is the weighted average
cost of all financing, debt and equity included. Debt generally has a lower cost of capital
than equity because of the effect of taxes, which reduce the effective interest rate of debt.
So if the debt goes away, all that will be left is equity, and equity has a higher cost of
capital than debt does. Therefore, the firm's cost of capital as a rate will increase. An
increase in the Treasury Bond yield will also cause the firm's cost of capital to increase.
The yield on Treasury securities is determined by market supply and demand for those
securities. When the market rate for Treasury securities (considered the risk-free rate)
increases, usually corporate bond market rates also increase in order to maintain the same
risk premium as previously. When market rates increase, the market price of existing
securities decreases in order to cause their rates of return to match the market rate of
return. So corporate bonds' market prices will decrease and their market rates will
increase. Remember that cost of capital calculations are based on market prices of the
stocks or bonds, not the issuing company's book value for the stocks or bonds. So when
the market prices of a firm's outstanding securities decrease, the firm's calculated cost of
capital for those securities will increase.
D. Reducing its operating leverage would not cause the firm's cost of capital to increase.
Reducing its operating leverage would decrease the firm's cost of capital, because it
would decrease the amount of risk that investors would perceive in the company.

Question #97: Question: The Company has a fixed assets balance of


[Fact Pattern #14] A company has a current ratio of 1.4, a quick, or acid test, ratio of 1.2, and the
following partial summary balance sheet:

cash $10
Accounts receivable
inventory
fixed assets
total assets $100
current liabilities
long term liabilities 40
stockholders' equity 30
total liabilities and
equity

Correct Answer: B. $58


A. $16
B. $58
C. $64
D. $84
Explanation:
Answer (A) is incorrect.
Neglecting to subtract the equity balance when calculating the current liability balance results in
$16.
Answer (B) is correct.
Total assets (given as $100) equals the sum of cash (given as $10), accounts receivable ($26 as
calculated using the quick ratio), inventory, and fixed assets. Inventory can be determined
because it is included in current, but not quick, assets, and the current and quick ratios are
known. Current assets equal $42 (1.4 current ratio x S30 current liabilities), and the quick assets
equal $36 (1.2 quick ratio x S30 current liabilities). Thus, inventory, which is the only difference
in this question between current and quick assets, equals $6 ($42 - $36). Fixed assets must then
equal $58 ($100 total assets - $10 cash - S26 accounts receivable - $6 inventory).
Answer (C) is incorrect.
Assuming that inventory is $0 results in $64.
Answer (D) is incorrect.
Ignoring accounts receivable results in $84.

Question #98: A farmers' cooperative has a large amount of grain that it has gathered from its
members and has stored in silos. Prices for grain are high, but none of the cooperative's
customers is prepared to purchase any for the next 3 months. In order to hedge against an
unfavorable change in grain prices over the next 3 months. the cooperative will employ a
financial risk management technique known as a
Correct Answer: A. Short hedge.
A. Short hedge.
B. Long hedge.
C. Naked option.
D. Interest rate swap.
Explanation:
Answer (A) is correct.
An extremely common form of financial risk management is called hedging. Hedging is the
process of using offsetting commitments to minimize or avoid the impact of adverse price
movements. A person who would like to sell an asset in the future has a long position in the asset
because (s)he benefits from a rise in value of the asset. To protect against a decline in value, the
owner can enter into a short hedge. i.e., obtain an instrument whose value will rise if the asset's
value falls.
Answer (B) is incorrect.
A long hedge is obtained by a party who fears a rise, not a fall, in the value of the underlying
asset.
Answer (C) is incorrect.
A naked option is one in which the seller of the option does not already possess the underlying
asset.
Answer (D) is incorrect.
An interest rate swap is a hedging tool for parties who are attempting to smooth a flow of interest
payments, not lock in the price of a commodity.

Question #99: Question: Broomall's quick (acid test) ratio at year end is
[Fact Pattern #16] Broomall Corporation has decided to include certain financial ratios in its
year-end annual report to shareholders. Selected information relating to its most recent fiscal
year is provided below.

Cash $10,000
Accounts receivable:
Beginning of year 24,000
end of year 20,000
prepaid expenses 8,000
inventory
Beginning of year 26,000
end of year 30,000
Available for sale securities
historical cost 9,000
fair value at year end 12,000
accounts payable 15,000
notes payable (due in 90 days) 25,000
Bonds payable (due in 10 years) 35,000
net credit sales for year 220,000
cost of goods sold 140,000

Correct answer: D. 1.05 to 1.


A. 2.00 to 1.
B. 1.925 to 1.
C. 1.80 to 1.
D. 1.05 to 1.

Explanation:
Answer (A) is incorrect
A ratio of 2.00 results from improperly including all current assets in the numerator.
Answer (B) is incorrect
A ratio 1.925 results from improperly including accounts receivable (beginning balance), prepaid
expenses, inventory, and available for sale securities (at cost) in the numerator, 1.925 can be
calculated as follows:
¿¿
¿

Answer (C) is incorrect A ratio 1.80 results from improperly including accounts receivable
(beginning balance) and inventory in the numerator 1.80 can be related as follows:
A
( $ 10,000 cash+ $ 24,000 beg . bal+$ 26,000 inventory beg . bal ¿ +$ 12,000 AFS at cost )
R
¿¿
Answer (D) is correct
The quick (acid test) ratio consists of the quick (cash, marketable securities, and accounts
receivable) divided by current liabilities, Broomall’s quick ratio at year end, 1.05 [($10,000 +
$12,000 + $20,000) ÷ ($15,000 + $25,000)]

Question #100: Shaw Corporation is considering a plant expansion that will increase its sales and
net income. The following data represent management's estimate of the impact the proposal will
have on the company:

current proposal
cash $100,000 $120,000
accounts payable 350,000 430,000
accounts receivable 400,000 500,000
inventory 380,000 460,000
marketable securities 200,000 200,000
mortage payable (current) 175,000 325,000
fixed assets 2,500,000 500,000
net income 500,000 650,000

The effect of the plant expansion on Shaw's working capital will be a(n)
Correct Answer: C. Decrease of $30,000.
A. Increase of $30,000.
B. Increase of $120,000.
C. Decrease of $30,000.
D. Decrease of $150.000.
Explanation:
A. This answer could result from omitting cash and accounts payable from the calculation of
the difference in working capital. Working capital consists of all current assets minus all
current liabilities. Cash is a current asset and accounts payable is a current liability, so
they should both be included.
B. This answer results from not including the current portion of the mortgage payable as a
current liability. Current maturities of long-term debt are current liabilities, so the
increase in the current portion of the mortgage payable decreases net working capital.
C. Currently, the company has working capital of $555,000: ($100,000 + 400,000 + 380,000
+ 200,000) - ($350,000 + $175,000). Under the proposed plan working capital would be
$525,000: ($120,000+ $500,000 + $460,000 + $200,000) - ($430,000 + $325,000). This
is a decrease of $30,000 from $555,000 to $525,000.
D. This answer could result from considering only the current portion of the mortgage
payable as equivalent to working capital. The current portion of the mortgage payable is a
current liability and so it is a component of working capital. However, working capital
consists of all current assets minus all current liabilities, so there are more components to
be included in calculating the change in working capital.

MOCK TEST 6

Question#1: Please refer to the information below


Devlin Company
Statement of financial position as of May 31
(In thousands)

Assets Year 2 Year 1


Current assets
Cash $45 $00
Trading securities 30 20
Accounts receivable (net) 68 48
Inventory 90 80
Prepaid expenses 22 30
Total current assets 255 216
Investment, at equity 38 30
Property, plant, and equipment (net) 375 400
Intangible assets (net) 80 45
Total assets $748 $691
Liabilities & shareholders' equity
Current liabilities
Notes payable $35 $18
Accounts payable 70 42
Accrued expenses 5 4
Income taxes payable 15 16
total current liabilities 125 80
Long term debt 35 35
Deferred taxes 3 2
Total liabilities 163 117
Shareholders' equity
Preferred stock, 6%, $100 par value, cumulative 150 150
Common stock, $10 par value 225 195
Additional paid-in capital--common stock 114 100
Retained earnings 96 129
Total shareholders' equity 585 574
Total liabilities and shareholders' equity $748 $691

Devlin Company
Income statement for the year ended May 31
(In thousands)
Year 2 Year 1
Net sales $480 $460
Costs and expenses
Cost of goods sold 330 315
Selling, general, and
administrative 52 51
Interest expense 8 9
Income before taxes 90 85
Income taxes 36 34
Net income $54 $51

Devlin Company’s rate of return on assets (ROA) for the year ended May 31, Year 2, was:
Correct Answer: D. 7.5%.
A. 7.8%.
B. 7.2%.
C. 11.2%.
D. 7.5%.
Explanation:
ROA is calculated by taking net income and dividing it by average total assets. Average total
assets is equal to the asset balance at the beginning of the period, plus the asset balance at the end
of the period, all divided by 2 (in order to arrive at an average for the year).
Average total assets = (beginning asset balance + ending asset balance) /2
Devlin's average total assets = ($748 + $691)/2 = $1439/2 = $719.50
Since net income is $54, then ROA is calculated as:
ROA = net income / average total assets
ROA = $54 / $719.50 = 0.075, or 7.5%
ROA is also called return on investment (ROI).

Question #2: Through the use of decision models, managers thoroughly analyze many
alternatives and decide on the best alternative for the company. Often the actual results achieved
from a particular decision are not what was expected when the decision was made. In addition,
an alternative that was not selected would have actually been the best decision for the company.
The appropriate technique to analyze the alternatives by using expected inputs and altering them
before a decision is made is
Correct Answer: A. Sensitivity analysis.
A. Sensitivity analysis.
B. Program Evaluation Review Technique (PERT).
C. Linear programming.
D. Expected value analysis.
Explanation:
A. Sensitivity analysis is a process of changing key variables to determine the possible
change in the optimal solution because of changes in the variables. It is used to define
how sensitive the project (sales for example) is to a change in those variables.
B. PERT is a project scheduling technique used to plan and control projects.
C. Linear programming is used to either maximize or minimize some quantity (called the
objective function). At the same time, this maximizing or minimizing must be
accomplished in the presence of constraints, or restrictions, such as limited quantities of
labor or materials. The maximization or minimization must be done without violating any
of the constraints.
D. Expected value analysis is used to determine expected return or cost. Expected value is
calculated by multiplying each projected outcome by its corresponding probability and
adding the products together. In other words, expected value is the weighted average of
the probable outcomes.

Question #3: Broomall Corporation has decided to include certain financial ratios in its year-end
annual report to shareholders. Selected information relating to its most recent fiscal year is
provided below.

Cash $10,000
Accounts receivable 20,000
Prepaid expenses 8,000
Inventory 30,000
Available for sale securities
At cost 9,000
Fair value at year end 12,000
Accounts payable 15,000
Notes payable (due in 90 days) 25,000
Bonds payable (due in 10 years) 35,000
Net credit sales for year 220,000
Cost of goods sold 140,000
Correct answer: D. $40,000
A. $37,000.
B. $10,000.
C. $28,000.
D. $40,000.
Explanation:
The term working capital as used by accountants is calculated by subtracting current liabilities
from current assets. Working capital = current assets - current liabilities Current assets include
cash, accounts receivable, prepaid expenses, inventories, and available-for-sale securities.
(Available-for-sale securities are carried at fair value. not cost.)
Current assets = $10,000 + $20,000 + $8,000 + $30,000 + $12,000 = $80,000
Current liabilities include accounts payable and notes payable due in 90 days.
Current liabilities = $15,000 + $25,000 = $40,000
Working capital = $80,000 - $40,000 = $40,000.

Question#4: The Dawson Corporation projects the following for the year.
$35
Earnings before interest and taxes million
Interest expense $5 million
Preferred stock dividends $4 million
Common stock dividend payout ratio 30%
Common shares outstanding 2 million
Effective corporate income tax rate 40%

If Dawson Corporation’s common stock is expected to trade at a price/earnings (P/E) ratio of


eight, the market price per share (to the nearest dollar) would be:
Correct answer: A. $56.
A. $56.
B. $125.
C. $68.
D. $72.
Explanation: The P/E ratio for a stock is the ratio of its market price to the corporation's earning
per share (EPS). The P/E ratio is given here, but it is necessary to determine the stock price using
the other available information.
The stock price, then, is $56, which is the EPS of $7, multiplied by the P/E of 8.
The EPS of $7 is calculated by taking the $14,000,000 in earnings to common shareholders and
dividing it by 2,000,000 common shares outstanding.
The earnings to common shareholders (E to CS) is calculated as follows:
EBIT = earnings before interest and taxes
E to CS = [(EBIT - Interest) (1 - Tax Rate)] - (Preferred Stock Dividend)
E to CS = [($35,000,000 - $5,000,000) (1 - 0.4)] -(S 4,000,000)
= [$30,000,000(0.6)] - ($ 4,000,000)
= $18,000,000 - $4,000,000 = $14,000,000.

Question #5: The amount of inventory that a company would tend to hold in stock would
increase as the
Correct Answer: C. Cost of carrying inventory decreases.
A. Variability of sales decreases.
B. Sales level falls to a permanently lower level.
C. Cost of carrying inventory decreases.
D. Cost of running out of stock decreases.
Explanation:
A. As the variability of sales decreases, the company will be able to hold lower levels of
inventory because there is less chance of a large demand that the company needs to be
prepared for.
B. If sales fall permanently, the company should hold less inventory. See the correct answer
for a complete explanation.
C. If the cost of holding inventory decreases, the company will be more likely to hold larger
levels of inventory. This is because there is a lower cost to hold the inventory, and by
having more inventory there is less risk of a stock out.
D. If the cost of running out of inventory decreases the company will hold less inventory
because of the lower cost of not having inventory.

Question #6: Which one of the following is not a characteristic of market-based costing?
Correct Answer: C. It is used by companies facing minimal competition.
A. It has a customer-driven external focus.
B. It is used by companies facing stiff competition.
C. It is used by companies facing minimal competition.
D. It starts with a target selling price and target profit.
Explanation:
Answer (A) is incorrect.
This is a characteristic of market-based costing.
Answer (B) is incorrect.
This is a characteristic of market-based costing.
Answer (C) is correct.
Market-based pricing is a strategy used by sellers in competitive markets, in other words, in
those markets where there is stiff competition.
Answer (D) is incorrect.
This is a characteristic of market-based costing.

Question #7: A corporation's net income as presented on its income statement is usually
Correct Answer: A. More than its economic profits because opportunity costs are not considered
in calculating net income.
A. More than its economic profits because opportunity costs are not considered in
calculating net income.
B. More than its economic profits because economists do not consider interest payments to
be costs.
C. Equal to its economic profits.
D. Less than its economic profits because accountants include labor costs, while economists
exclude labor costs.
Explanation:
Answer (A) is correct.
Economic (pure) profit equals total revenue minus economic costs. Economic costs are defined
by economists as total costs, which are the sum of outlay costs, and opportunity costs, which are
the values of productive resources in their best alternative uses. The return sufficient to induce
the entrepreneur to remain in business (normal profit) is an implicit (opportunity) cost. Net
income as computed under generally accepted accounting principles considers only explicit
costs, not such implicit costs as normal profit and the opportunity costs associated with not using
assets for alternative purposes. Thus, net income will be higher than economic profit because the
former fails to include a deduction for opportunity costs, for example, the salary forgone by an
entrepreneur who chooses to be self-employed.
Answer (B) is incorrect.
Both economists and accountants treat interest as a cost.
Answer (C) is incorrect.
Economic profits will be less than net income.
Answer (D) is incorrect.
Economic profits will be less than net income.

Question #8: Wearwell Company is considering three investment projects. Wearwell's president
asked the controller to prepare a report and recommend an appropriate investment decision. The
results of the controller's calculations for the three projects are as follows.

Project NPV IRR


A $20,680 12%
B $30,300 10%
C $15,000 13%

The company expects a minimum net present value (NPV) of $20.000 from accepted projects.
The projects are mutually exclusive and Wearwell's cost of capital is 8%. Which one of the
following options should the controller recommend to the president?
Correct Answer: D. Project B because it has the highest net present value (NPV).
A. Projects A and B because they exceed the minimum expected NPV.
B. Projects A. B. and C because each of the projects have an IRR greater than the cost of
capital.
C. Project C because it has the highest internal rate of return a (IRR).
D. Project B because it has the highest net present value (NPV).

Explanation:
A. This is not an option because these projects are mutually exclusive. "Mutually exclusive"
means if one project is chosen, the other or others cannot be chosen. An example of
mutually exclusive projects is a piece of land on which either a plant or an office building
can be constructed, but not both.
B. This is not an option because these projects are mutually exclusive. "Mutually exclusive
means if one project is chosen, the other or others cannot be chosen. An example of
mutually exclusive projects is a piece of land on which either a plant or an office building
can be constructed, but not both.
C. Project C does have the highest IRR, but that is probably because the amount of the
investment is small. Project C has an NPV of only $15,000, so it is well behind the other
projects in its ability to increase shareholder wealth.
D. The controller should recommend Project B because it has the highest net present value.
Project B will increase shareholder wealth by more than the other projects because its
NPV is higher than the others'. Project C has a higher IRR, but that is probably because
the amount of the investment is small. Project C has an NPV of only $15,000, so it is well
behind project B in its ability to increase shareholder wealth. All three projects are not an
option because these projects are mutually exclusive. "Mutually exclusive" means if one
project is chosen, the other or others cannot be chosen. Projects A and B are also not an
option for the same reason.

Question #9: IMA members are obligated to maintain the highest standards of ethical conduct.
Accordingly, I MAs Statement of Ethical Professional Practice explicitly requires that IMA
members
Correct Answer: B. Not condone violations by others.
A. Obtain sufficient competent evidence when expressing an opinion.
B. Not condone violations by others.
C. Comply with generally accepted auditing standards.
D. Adhere to generally accepted accounting principles.
Explanation:
Answer (A) is incorrect.
The expression of an opinion is a function of an external auditor.
Answer (B) is correct.
The principles section of IMA's Statement of Ethical Professional Practice, "IMA's overarching
ethical principles include: Honesty. Fairness. Objectivity, and Responsibility. Members shall act
in accordance with these principles and shall encourage others within their organizations to
adhere to them.”
Answer (C) is incorrect.
Compliance with GAAS is a requirement of external auditors.
Answer (D) is incorrect.
Adherence to GAAP is not expressly required by IMA's Statement of Ethical Professional
Practice.

Question #10: Question: Dimmell has a potential foreign customer that has offered to buy 1,500
tons at $450 per ton. Assume that all of Dimmell's costs would be at the same levels and rates as
in Year 2. What net income would Dimmell make if it took this order and rejected some business
from regular customers so as not to exceed capacity?
[Fact Pattern] The statement of income for Dimmell Co. presented below represents the
operating results for the fiscal year just ended. Dimmell had sales of 1,800 tons of product during
the current year. The manufacturing capacity of Dimmell's facilities is 3,000 tons of product.
Dimmell Co.
Statement of Income
For the Year Ended December 31, Year 2

$900,00
Sales 0
Variable costs:
$315,00
Manufacturing 0
(495,000
Selling costs 180,000 )
$405,00
Contribution margin 0
Fixed costs:
Manufacturing $90,000
Selling 112,500
(247,500
Administration 45,000 )
$157,50
Operating income 0
Income taxes (40%) (63,000)
Net income $94,500

Correct answer: C. $211,500.


A. $297,500.
B. $252,200.
C. $211,500.
D. $256,500.
Explanation:
Answer (A) is incorrect.
Failing to deduct selling expenses results in $297,500.
Answer (B) is incorrect.
Ignoring administrative expenses results in $252,000.
Answer (C) is correct.
Total sales equal $1,425,000 [(1,500 x $450) + (1,500 x $500)]. The unit variable cost of goods
sold is $275 (55% of $500), and total variable cost of goods sold is $825,000 (3,000 units x
$275). The contribution margin is $600,000, which equals $1,425,000 of sales minus $825,000
cost of goods sold. Operating income is $352,500, which is the contribution margin of $600,000
minus $247,500 fixed costs. Net income of $211,500 is calculated by subtracting tax from
operating income [$352,500 x (1.0 - .40)].
Answer (D) is incorrect.
This amount mixes variable costs and contribution margin.

Question #11: ROK Company has $900,000 in current assets and $1,800,000 in fixed assets,
including $500,000 in goodwill and patents. Its paid-in capital is $100,000 and retained earnings
are $2,100,000. Eighty percent of liabilities are long-term liabilities. What is its net tangible
assets to long-term debt ratio?
Correct Answer: D. 5.5.
A. 0.23.
B. 1.23.
C. 3.6.
D. 5.5.
Explanation: The net tangible assets to long-term debt ratio is calculated by dividing net tangible
assets by long-term debt. Net tangible assets for ROK Company are $2,200,000 ($900,000
current assets + $1,800,000 fixed assets - $500,000 intangible assets of goodwill and patents).
Total liabilities are assets less equity or S500.000 ($2.7 million - $2.2 million).
Since 80% of liabilities are long-term, total long-term liabilities are $400,000 (80% of
$500,000).
Therefore, the net tangible assets to long-term debt ratio is 5.5 ($2,200,000 net tangible assets +
$400,000 long-term liabilities).
Question #12: Consider the statements below regarding accounting treatments for goodwill
under IFRSs. Which statement describes the correct accounting treatment for goodwill under
IFRSs?
Correct Answer: A. IFRSs tests goodwill for impairment but goodwill is not amortized.
A. IFRSs tests goodwill for impairment but goodwill is not amortized.
B. IFRSs allows goodwill to be amortized for a period not to exceed 20 years.
C. IFRSs allows goodwill to be amortized for a period not to exceed 40 years.
D. IFRSs does not allow the amortization of goodwill.
Explanation:
Like U.S. GAAP goodwill is never amortized but it should be tested annually for impairment.

Question #13: A major difference between economic profit and accounting profit is that
economic profit
Correct Answer: C. Reduces profits by associated cost of capital.
A. Allows for more accurate expense accruals.
B. Minimizes the impact of accounting estimates.
C. Reduces profits by associated cost of capital.
D. Adjusts accounting profit by depreciation.
Explanation:
Answer (A) is incorrect.
Economic profit does not allow for more accurate expense accruals. The explicit costs
are accrued in the same manner for both accounting and economic profit. Economic
profits are more difficult to calculate because they include implicit costs, which are
difficult to accrue for.
Answer (B) is incorrect.
Economic profit maximizes the impact of accounting estimates as the implicit costs must
be estimated. These costs do not have to be estimated for accounting profit purposes.
Answer (C) is correct.
Economic profits are not earned until the organization's income exceeds not only costs
as recorded in the accounting records, but the firm's implicit costs as well. Economic
profit is equal to the accounting (book) profit less the implicit costs. Therefore,
economic profit reduces accounting profits by associated cost of capital.
Answer (D) is incorrect.
Economic profit adjusts accounting profit by the implicit costs. Depreciation is not an
implicit cost.

Question #14: Kielly Machines Inc. is planning an expansion program estimated to cost
$100 million. Kielly is going to raise funds according to its target capital structure
shown below.
Debt 0.30
Preferred stock 0.24
Equity 0.46
Kelly had net income available to common shareholders of $184 million last year of
which 75% was paid out in dividends The company has a marginal tax rate of 40%.
Additional data:
• The before-tax cost of debt is estimated to be 11%.
• The market yield of preferred stock is estimated to be 12%.
• The after-tax cost of common stock is estimated to be 16%.
What is Kiell’s weighted average cost of capital?
Correct Answer: B. 12.22%.
A. 13.00%.
B. 12.22%.
C. 13.54%.
D. 14.00%.

Explanation:
A. This is an un-weighted average of the costs given in the problem. This is incorrect
for two reasons: (1) the average should be a weighted average, weighted according to
each component's percentage of the total capital; and (2) the cost of debt used is the
before-tax cost of debt instead of the after-tax cost of debt.
B. The before-tax cost of debt is given as 11%. The company's marginal tax rate is 40%.
Therefore, the after-tax cost of debt is 0.11 -x (1 0.40), which is 0.066 or 6.6%. The
costs of the other components of the company's total capital to be raised are given in
the problem as 12% for preferred stock end 16% for common stock (retained
earnings). So the weighted average cost of capital is (0.30 0.066) + (0.24 0.12) +
(0.46 -x 0.16) = 0.1222 or 12.22%.
C. This answer results from using the before-tax cost of debt instead of the after-tax
cost of debt to calculate the weighted average cost of capital.
D. This is an un-weighted average of the costs of preferred stock and common stock.
This is incorrect for two reasons: (1) the average should be a weighted average,
weighted according to each component's percentage of the total capital; and (2) the
cost of debt is not included in the calculation.

Question #15: Corporations purchase their outstanding stock for all of the following reasons
except to
Correct Answer: B. Improve short-term cash flow.
A. Meet employee stock compensation contracts.
B. Improve short-term cash flow.
C. Increase earnings per share by reducing the number of shares outstanding.
D. Use the shares for a stock dividend.
Explanation:
A. This is one of the reasons that a company may purchase its own shares.
B. If a company buys its own shares that will require e cash outflow. This will therefore
not improve short-term cash flows.
C. This is one of the reasons that a company may purchase its own shares.
D. This is one of the reasons that a company may purchase its own shares.

Question #16: The level of safety stock in inventory management depends on all of the
following except the:
Correct Answer: A. Cost to reorder stock.
A. Cost to reorder stock.
B. Cost of running out of inventory.
C. Level of uncertainty of the sales forecast.
D. Level of customer dissatisfaction for back orders.
Explanation:
A. The amount of safety stock that a company is required to hold will be affected by: 1)
the variability of the lead time, 2) the variability of the demand for the product, and
3) the cost of stock out. The more variable either of the first two items are, the more
safety stock the company will need to carry to guard against stock outs in the case of
an unusually high demand or an unusually long lead time. If these items are more
consistent and predictable, the company can reduce the amount of its safety stock
because there is a smaller chance of needing so many items in stock. The greater the
cost of a stock out, the more safety stock the company will need to keep because the
potential loss from a stock out is higher.
B. The greater the cost of running out of inventory, the more safety stock that the
company must hold. See the correct answer for a complete explanation.
C. The more uncertain the sales forecast, the more safety stock the company will need
to keep. Therefore, the level of uncertainty of the future sales does impact the level
of safety stock. See the correct answer for a complete explanation.
D. The greater the dissatisfaction that customers have in the case of a stock out, the
more safety stock that the company must hold. See the correct answer for a complete
explanation.

Question #17: In a management decision process, the cost measurement of the benefits
sacrificed due to selecting an alternative use of resources is most often referred to as a(n)
Correct Answer: C. Opportunity cost.
A. Relevant cost.
B. Sunk cost.
C. Opportunity cost.
D. Differential cost.
Explanation:
Answer (A) is incorrect.
While opportunity costs are relevant costs, opportunity cost is the most precise
definition of the ones offered.
Answer (B) is incorrect.
A sunk cost is one already incurred.
Answer (C) is correct.
An opportunity cost is the cost of using a scarce resource for one purpose rather than
another.
Answer (D) is incorrect.
A differential cost is one that varies between alternatives.
Question #18: Which of the following best describes an important provision of the U.S.
Foreign Corrupt Practices Act?
Correct Answer: D. The internal accounting controls should be examined, and if material
weaknesses are found, controls must be strengthened.
A. Auditors cannot provide bookkeeping or other services related to the accounting
records or financial statements of the audit client.
B. Companies must follow the laws of the their home country as well as the laws of the
countries where any foreign subsidiaries are located.
C. The CEO end CFO must certify that they have no knowledge of any corrupt practices
occurring in any overseas subsidiaries of US companies.
D. The internal accounting controls should be examined, and if material weaknesses are
found, controls must be strengthened.
Explanation:
Answer (A) is incorrect.
This is an important provision of the Sarbanes-Oxley Act.
Answer (B) is incorrect.
This is not a provision of the U.S. Foreign Corrupt Practices Act.
Answer (C) is incorrect.
This is not a provision of the U.S. Foreign Corrupt Practices Act.
Answer (D) is correct.
The FCPA states that all public companies registered under the 1934 Act must devise
and maintain a system of internal accounting control sufficient to provide reasonable
assurance against material misstatements.

Question #19: Consider the following factors affecting a company as it is reviewing its
trade credit policy.
I. Operating at full capacity.
II. Low cost of borrowing.
III. Opportunity for repeat sales.
IV. Low gross margin per unit.
Which of the above factors would indicate that the company should liberalize its credit
policy?
Correct Answer: C. II and III only.
A. III and IV only.
B. I, II and III only.
C. II and III only.
D. I and II only.

Explanation:
A. Of these two factors, only Ill would be e factor that would indicate the company
should liberalize its credit policy. When a firm liberalizes its credit policy, it lowers
its requirements for extending credit. That means more of its customers will qualify
for credit. As a result, its sales will increase, and its outstanding receivables will also
increase. Its uncollectible accounts will also increase. The firm would probably also
need to increase its own short-term borrowing to support increases in accounts
receivable and also inventory that would be required. It would make sense for the
firm to liberalize its credit policy only if (a) the costs of doing so were lower than the
increased profit that would result from increased sales and (b) if it was able to fulfill
the increased demand that would result. If liberalizing its credit policy were to create
opportunities for repeat sales (III), this would increase profits because of the
increased sales. Because the increased sales would be coming from repeat customers,
those sales would not require the company to incur the expenses of gaining e new
customer and doing the necessary credit investigation. Therefore, sales from existing
customers should be more profitable than sales to new customers. If those sales
generated more profit than the increased costs associated with liberalizing its credit
policy, liberalizing the credit policy would be a good decision. So Ill is a factor that
would indicate the company should liberalize its credit policy. A low gross margin
per unit (IV) would indicate that the increased profits from increasing sales would
not be very great. Therefore, there is a greater possibility that the increased costs
associated with liberalizing its credit policy would be greater than the increased
profits that would result. So IV is not a factor that would indicate the company
should liberalize its credit policy.
B. Of these three factors, only II and III would be factors that would indicate the
company should liberalize its credit policy. When a firm liberalizes its credit policy,
it lowers its requirements for extending credit. That means more of its customers will
qualify for credit. As a result, its sales will increase, and its outstanding receivables
will also increase. Its uncollectible accounts will also increase. The firm would
probably also need to increase its own short-term borrowing to support increases in
accounts receivable and also inventory that would be required. It would make sense
for the firm to liberalize its credit policy only if (a) the costs of doing so were lower
than the increased profit that would result from increased sales and (b) if it was able
to fulfill the increased demand that would result. If the firm is already operating at
full capacity it would not be able to supply any more product. Therefore, it would not
want orders to increase, because it would not be able to fulfill them. So I is not a
factor that would indicate the company should liberalize its credit policy. If the
firm's cost of borrowing is low (l l), this would increase the possibility that the costs
of liberalizing its credit policy would be lower than the increased profits that would
be obtained because of increased sales. So II is a factor that would indicate the
company should liberalize its credit policy. If liberalizing its credit policy were to
create opportunities for repeat sales (Ill), this would increase profits because of the
increased sales. Because the increased sales would be coming from repeat customers,
those sales would not require the company to incur the expenses of gaining e new
customer and doing the necessary credit investigation. Therefore, sales from existing
customers should be more profitable than sales to new customers. If those sales
generated more profit than the increased costs associated with liberalizing its credit
policy, liberalizing the credit policy would be a good decision. So Ill is a factor that
would indicate the company should liberalize its credit policy.
C. II and Ill are factors that would indicate the company should liberalize its credit
policy. When a firm liberalizes its credit policy, it lowers its requirements for
extending credit. That means more of its customers will qualify for credit. As a
result, its sales will increase, and its outstanding receivables will also increase. Its
uncollectible accounts will also increase. The firm would probably also need to
increase its own short-term borrowing to support increases in accounts receivable
and also inventory that would be required. It would make sense for the firm to
liberalize its credit policy only if (a) the costs of doing so were lower than the
increased profit that would result from increased sales and (b) if it were able to fulfill
the increased demand that would result. If the firm's cost of borrowing is low (l l),
this would increase the possibility that the costs of liberalizing its credit policy
would be lower than the increased profits that would be obtained because of
increased sales. So II is a factor that would indicate the company should liberalize its
credit policy. If liberalizing its credit policy were to create opportunities for repeat
sales (Ill), this would increase profits because of the increased sales. Because the
increased sales would be coming from repeat customers, those sales would not
require the company to incur the expenses of gaining e new customer and doing the
necessary credit investigation. Therefore, sales from existing customers should be
more profitable than sales to new customers. If those sales generated more profit than
the increased costs associated with liberalizing its credit policy, liberalizing the
credit policy would be a good decision. So Ill is a factor that would indicate the
company should liberalize its credit policy.
D. Of these two factors, only II would be a factor that would indicate the company
should liberalize its credit policy. When a firm liberalizes its credit policy, it lowers
its requirements for extending credit. That means more of its customers will qualify
for credit. As a result, its sales will increase, and its outstanding receivables will also
increase. Its uncollectible accounts will also increase. The firm would probably also
need to increase its own short-term borrowing to support increases in accounts
receivable and also inventory that would be required. It would make sense for the
firm to liberalize its credit policy only if (a) the costs of doing so were lower than the
increased profit that would result from increased sales and (b) if it was able to fulfill
the increased demand that would result. If the firm is already operating at full
capacity it would not be able to supply any more product. Therefore, it would not
want orders to increase, because it would not be able to fulfill them. So I is not a
factor that would indicate the company should liberalize its credit policy. If the
firm's cost of borrowing is low (l l), this would increase the possibility that the costs
of liberalizing its credit policy would be lower than the increased profits that would
be obtained because of increased sales. So II is a factor that would indicate the
company should liberalize its credit policy.

Question #20: A corporation's goal is for operating income to equal 6% of sales. The
corporation estimates that the highest selling price the market will bear is $115 per unit. The
corporation expects to sell 100,000 units, expects to incur fixed costs of $3,500,000, and has
an effective income tax rate of 40%. To achieve these plans, the target variable cost per unit
must be
Correct Answer: B. $73.10
A. $108.10
B. $73.10
C. $68.50
D. $62.75
Explanation:
Answer (A) is incorrect.
The amount of $108.10 incorrectly ignores the fixed costs per unit.
Answer (B) is correct.
A target price is the expected market price for a product or service. Subtracting the unit
target operating income in addition to the fixed costs per unit from the target selling
price per unit produces the target variable cost per unit.
Target sales = $11,500,000 (100,000 units × $115 per unit)
Target operating income = $690,000 (.06 x $11,500,000)
Target operating income/unit = $6.90 ($690,000 ÷ 100,000 units)
Per unit fixed costs = $35/unit (3,500,000 ÷ 100,000 units)
Target variable cost = $115 - $35 - $6.9 = $73.10
Answer (C) is incorrect.
The amount of $68.50 incorrectly adjusts the target operating income/unit for tax effects.
Answer (D) is incorrect.
The amount of $62.75 is not meaningful in this context.

Question #21: Selected information regarding Dyle Corporation’s outstanding equity is


shown below.
$3,500,00
Common stock, $10 par value, 350,000 shares outstanding 0
Preferred stock , $100 par value 10,000 shares outstanding 1,000,000
Preferred stock dividend paid 60,000
Common stock dividend paid 700,000
Earnings per common share 3
Market price per common share 18

Dyle dividend yield on common stock is:


Correct answer: C. 11.11%.
A. 16.66%.
B. 20.00%.
C. 11.11%.
D. 16.88%.
Explanation:
The dividend yield on common stock is calculated as:
Dividend yield on common stock = (annual dividend per common share) / (market price of
common stock)
Dividend per common share = (total common stock dividend) / (number of common stock shares
outstanding)
Dividend per common share = ($700,000) / (350,000 shares) = $2
Dividend yield = $2 / $18 = 0.1111, or 11.11%.
Question #22: Jones & Company is considering the acquisition of scanning equipment to
mechanize its procurement process. The equipment will require extensive testing and debugging,
as well as user training prior to its operational use. Projected after-tax cash flows are as follows.

Time Period After-Tax Cash


Inflow/
Year (Outflow)
0 $(600,000)
1 $(500,000)
2 $450,000
3 $450,000
4 $350,000
5 $250,000

Management anticipates the equipment will be sold at the beginning of year 6 for $50.000 and its
book value is zero. Jones' internal hurdle and effective income tax rates are 14% and 40%,
respectively. Based on this information, a negative net present value was computed for the
project. Accordingly, it can be concluded that
Correct Answer:
A. The project has an internal rate of return (IRR) less than 14% since IRR is the interest
rate at which net present value is equal to zero.

A. The project has an internal rate of return (IRR) less than 14% since IRR is the interest
rate at which net present value is equal to zero.
B. Jones should examine the determinants of its hurdle rate further before analyzing any
other potential projects.
C. Jones should calculate the project payback to determine if it is consistent with the net
present value calculation.
D. The project has an IRR greater than 14% since IRR is the interest rate at which net
present value is equal to zero.

Explanation:
A. The IRR is the discount rate at which the net present value of the project is zero. If the
project's net present value computed using the company's hurdle rate of 14% is negative,
its I RR must be lower than 14%. Any project with a net present value of less than zero or
an IRR of less than the hurdle rate is not an acceptable project.
B. The purpose of the hurdle rate is to establish the minimum return the company expects
from a project. The fact that a project is unacceptable when that hurdle rate is used to
discount a project's future cash flows is not an indication that there is anything wrong
with the hurdle rate used. It is more likely an indication that the project does not meet the
company's requirements. Changing the company's requirements to make the project
acceptable is generally not the proper course of action.
C. The payback period method of capital budgeting should not be used to confirm a net
present value calculation for a project. The payback method can be helpful when the
company needs to recoup its investment quickly, perhaps because the project is in a
politically unstable area of the world or because it utilizes high technology that quickly
becomes obsolete. However, its usefulness is limited because it does not incorporate the
time value of money and it ignores the cost of capital and all cash flows beyond the
payback point.
D. The IRR is the discount rate at which the net present value of the project is zero. If the
project's net present value computed using the company's hurdle rate of 14% is negative,
its IRR must be lower than 14%. Any project with a net present value of less than zero or
an IRR of less than the hurdle rate is not an acceptable project.

Question #23: A company builds custom-designed machinery. A review of selected data and the
company's pricing policies revealed the following.
A 10% commission is paid on all sales orders.
Variable and fixed factory overheads total 40% and 20%, respectively, of direct labor.
Corporate administrative costs amount to 10% of direct labor.
When bidding on jobs, the company adds a 25% markup to the total of all factory and
administrative costs to cover income taxes and produce a profit.
The firm's income tax rate is 40%.
The company expects to operate at a maximum of 80% of practical capacity. The company
recently received an invitation to bid on the manufacture of some custom machinery for a
customer. For this project, the company's production accountants estimate the material and labor
costs will be $66,000 and $120,000, respectively. Accordingly, the company submitted a bid to
the customer in the amount of $375,000. Feeling the company's bid was too high, the customer
countered with a price of $280,000. Which one of the following options should be recommended
to the company's management?
Correct Answer: A. Accept the counteroffer because the order will increase operating income.
A. Accept the counteroffer because the order will increase operating income.
B. Accept the counteroffer even though the order will decrease operating income.
C. Reject the counteroffer even though the order will increase operating income.
D. Reject the counteroffer because the order will decrease operating income.
Explanation:
Answer (A) is correct.
Based on the information given. the bid price of $375,000 was apparently calculated as follows:
Materials (given) $ 66,000
Direct labor (given) 120.000
Sales commission (10% x $375,000) 37,500
Variable overhead (40% of DL) 48,000
Fixed overhead (20% of DL) 24,000
Corporate administrative (10% of DL) 12,000
Profit (25% of costs excluding sales commissions) 67,500
Total $375,000 ($66000 + $1,20,000 + 37,500 + 48,000 +24,000 + 12,000 + 67500)
The incremental cost of the project if the counteroffer is accepted is $262,000 [$66,000 materials
+ $120,000 direct labor + (10% x $280,000) sales commission + $48,000 variable overhead].
The counteroffer is $95,000 less than the bid price. However, since the amount of the counter
offer ($280,000) exceeds the incremental cost of the project ($262,000) by $18,000, the
counteroffer should be accepted because the order will increase operating income.
Answer (B) is incorrect.
The counteroffer should be accepted because it will result in increased operating income.
Answer (C) is incorrect.
The counteroffer should be accepted because it will result in increased operating income.
Answer (D) is incorrect.
The counteroffer should be accepted because it will result in increased operating income.

Question #24: The IASB has been working closely with the FASB to harmonize the international
standards with U.S. GAAP. Differences in accounting treatment exist for all of the following
except:
Correct Answer: C. Accounting for inventory using first-in-first out (FIFO).
A. Accounting for impairment of assets.
B. Accounting for inventory using last-in-first out (LIFO).
C. Accounting for inventory using first-in-first out (FIFO).
D. Accounting for development costs.
Explanation:
Both IAS and U.S. GAAP allow accounting for inventory using FIFO: therefore there is no
difference using this method.

Question #25: An entrepreneur wants to rent store space in a new shopping mall for the 3-month
holiday shopping season. The entrepreneur believes he has a new product available that has the
potential for good sales. The product can be obtained on consignment at the cost of $20 per unit,
and he expects to sell the item for $100 per unit. Due to other business ventures, the
entrepreneur's risk tolerance is low. He recognizes that, as the product is entirely new, there is an
element of risk. The mall management has offered the entrepreneur three rental options: (1) a
fixed fee of $8.000 per month. (2) a fixed fee of $3.990 per month plus 10% of the entrepreneur's
revenue, or (3) 30% of the entrepreneur's revenues. Which one of the following actions would
you recommend to the entrepreneur?
Correct Answer: D. Choose the third option no matter what the entrepreneur expects the
revenues to be.
A. Choose the first option no matter what the entrepreneur expects the revenues to be.
B. Choose the second option no matter what the entrepreneur expects the revenues to be
C. Choose the second option only if the entrepreneur expects revenues to exceed $5,700.
D. Choose the third option no matter what the entrepreneur expects the revenues to be.
Explanation:
Answer (A) is incorrect.
When the venture is risky and the risk for tolerance is low, fixed costs are to be avoided.
Answer (B) is incorrect.
When the venture is risky and the risk for tolerance is low, fixed costs are to be avoided.
Answer (C) is incorrect.
When the venture is risky and the risk for tolerance is low, fixed costs are to be avoided.
Answer (D) is correct.
The entrepreneur recognizes that trying to sell the new product is risky. For this reason, fixed
costs are to be avoided. If sales are low, relying on variable costs will keep total costs down.
Question #26: CPZ Enterprises had the following account information.
$200,00
Accounts receivable 0
Accounts payable 80,000
Bounds payable, due in ten years 300,000
Cash 100,000
Interest payable, due in three months 10,000
Inventory 400,000
Land 250,000
Notes payable, due in six months 50,000
Prepaid expenses 40,000

The company has an operating cycle of five months.


What will happen to the ratios below if CPZ Enterprises uses cash to pay 50% of the accounts
payable, and both ratios are greater than 1.0 prior to the payment?
Correct Answer: A. current ratio: increase: quick ratio: increase.
A. Current ratio: increase: quick ratio: increase.
B. Current ratio: decrease: quick ratio: increase.
C. Current ratio: increase: quick ratio: decrease.
D. Current ratio: decrease: quick ratio: decrease.
Explanation: The current ratio is the ratio of current assets to current liabilities. The acid-test
ratio (quick ratio) is the ratio of quick assets to current liabilities.
Current assets consist of cash, short-term marketable securities, accounts receivable, short-term
notes receivable, inventories, and prepaid expenses. Current liabilities are those due within one
year, or one business cycle, whichever is longer. Current liabilities consist of accounts payable,
accrued expenses payable, interest payable, taxes payable, short-term notes payable, and the
current portion of long-term debt.
The quick assets are cash short-term investments (marketable securities), and receivables. The
cash payment on accounts payable will decrease the current and quick assets by the same
amount. Since both the current ratio and the quick ratio are greater than one, the change will
increase both ratios.
For example, if prior to the payment, the current assets were equal to $200, the quick assets were
$150, and the current liabilities were $100, then the original current ratio would be $200/$100 =
2 and the original quick ratio would be $150/$100 = 1.5.
Assuming that the current liabilities are all accounts payable, the payment on 50% of the
accounts payable balance would reduce the current assets to $150, the quick assets to $100, and
the current liabilities to $50. The new current ratio would then be $150/$50 = 3. The new quick
ratio would be $100/$50 = 2. By using this example, one can see that both the current ratio and
quick ratio have increased as a result of CPZ Enterprises using cash to pay off 50% of the
account payable balance.

Question #27: Sarah Birdsong has prepared a net present value (NPV) analysis for a 15-year
equipment modernization program. Her initial calculations include a series of depredation tax
savings, which are then discounted. Birdsong is now considering the incorporation of inflation
into the NPV analysis. If the depreciation tax savings were based on original equipment cost,
which of the following options correctly shows how she should handle the program's cash
operating costs and the firm's required rate of return, respectively?

Cash operating costs Required rate of return


I Adjust for inflation Adjust for inflation
Do not adjust for
II Adjust for inflation inflation
III Do not adjust for inflation Adjust for inflation
Do not adjust for
IV Do not adjust for inflation inflation

Correct Answer: • D. I.
A. III.
B. II.
C. IV.
D. I.
Explanation:
A. Future cash operating costs need to be adjusted for inflation, and the required rate of
return needs to be adjusted for inflation as well. If we adjust one, we must adjust the
other.
B. Future cash operating costs need to be adjusted for inflation, and the required rate of
return needs to be adjusted for inflation as well. If we adjust one, we must adjust the
other.
C. Future cash operating costs need to be adjusted for inflation, and the required rate of
return needs to be adjusted for inflation as well. If we adjust one, we must adjust the
other.
D. Future cash operating costs need to be adjusted for inflation, and the required rate of
return needs to be adjusted for inflation as well. If we adjust one, we must adjust the
other.
Question #28: A new management accountant is concerned about complying with the ethical
standard of competence in the IMAs Statement of Ethical Professional Practice. Which one of
the following is not required under the standard of competence?
Correct Answer: A. Maintain expertise in all areas of accounting.
A. Maintain expertise in all areas of accounting.
B. Continually develop knowledge and skills.
C. Perform duties in accordance with relevant regulations and standards.
D. Provide recommendations that are accurate and timely.
Explanation:
Answer (A) is correct.
Maintaining expertise in all areas of accounting would be a difficult task. According to the
ethical standard of competence in the IMA's Statement of Ethical Professional Practice, a CMA
only needs to recognize and communicate professional limitations or other constraints that would
preclude responsible judgment or successful performance of an activity.
Answer (B) is incorrect.
According to the ethical standard of competence in the IMAs Statement of Ethical Professional
Practice, a CMA should maintain an appropriate level of professional expertise by continually
developing knowledge and skills.
Answer (C) is incorrect.
According to the ethical standard of competence in the IMAs Statement of Ethical Professional
Practice, a CMA should perform professional duties in accordance with relevant laws,
regulations, and technical standards.
Answer (D) is incorrect.
According to the ethical standard of competence in the IMAs Statement of Ethical Professional
Practice, a CMA should provide decision support information and recommendations that are
accurate. clear. Concise, and timely.

Question #29: An organization is evaluating whether their risk management strategy is effective.
This organization is at which step in the Risk Management Process (RMP)?
Correct Answer: B. Monitor the risk exposure and the strategy.
A. Determine the company's tolerance for risk.
B. Monitor the risk exposure and the strategy.
C. Implement an appropriate risk management strategy.
D. Evaluate the risk exposure.
Explanation:
Periodic monitoring assesses the status quo or any unexpected changes in the risk exposure (as a
result of market volatility. etc.). This step also considers whether the risk management strategy
selected is effective. Strategy adjustments may be necessary.

Question #30: The level of risk that an organization will face can increase as a result of all of the
following except:
Correct Answer: D. Decreasing the expected time to complete a project.
A. The cash reserves the company holds.
B. Increasing the expected time to complete a project.
C. Variability of expected returns.
D. Decreasing the expected time to complete a project.
Explanation:
Risks are a function of volatility (variability) and time. Increased volatility, such as variability in
expected returns, translates into increased risk. The longer the time frame, the more the
uncertainty, and consequently, the higher the risk. Shorter time frames imply lower risk.

Question #31: Foster Manufacturing is analyzing a capital investment project that is forecasted to
produce the following cash flows and net income.
after tax
cash net
year flows income
$(20,000
0 ) $0
1 6,000 2,000
2 6,000 2,000
3 8,000 2,000
4 8,000 2,000

If Foster's cost of capital is 12%. the net present value for this project is
Correct Answer: D. $924
A. $6.074.
B. $6.998.
C. $(1.600).
D. $924
Explanation:
A. This is the present value of the future net incomes. The net present value is found by
discounting the after-tax cash flows to the present (Year 0) using the 12% cost of capital
and subtracting the Year 0 cash outflow.
B. This is not the correct answer. Please see the correct answer for an explanation.
C. This is not the correct answer. Please see the correct answer for an explanation.
D. The net present value is found by discounting the after-tax cash flows to the present
(Year 0) using the 12% cost of capital and subtracting the Year 0 cash outflow. The
$6,000 for the first two years can be discounted using the PV of an annuity factor for
12% for 2 years. The Year 3 and Year 4 cash flows can be discounted using the PV of $1
factors for 3 and 4 years at 12%. ($6,000 x 1.69) + ($8,000 x 0.712) + ($8,000 x 0.636) -
$20,000 = $924.

Question #32: An organization can mitigate operational risk by:


Correct Answer: C. Modifying cost structures to make more costs variable costs.
A. Modifying cost structures to make more costs differential costs.
B. Modifying cost structures to make more costs step costs.
C. Modifying cost structures to make more costs variable costs.
D. Modifying cost structures to make more costs fixed costs.
Explanation:
Operational risks may be lessened by shifting the organization's costs from fixed to variable. For
example, a company can outsource parts or activities rather than producing or performing them
internally.

Question #33: Integrity is an ethical requirement for all IMA members. One aspect of integrity
requires
Correct Answer: B. Avoidance of conflict of interest.
A. Performance of professional duties in accordance with relevant laws.
B. Avoidance of conflict of interest.
C. Refraining from using confidential information for unethical or illegal advantage.
D. Maintenance of an appropriate level of professional expertise.
Explanation:
Answer (A) is incorrect.
Performance of professional duties in accordance with relevant laws states an aspect of the
competence requirement.
Answer (B) is correct.
According to IMAs Statement of Ethical Professional Practice. IMA members must 'mitigate
actual conflicts of interest. Regularly communicate with business associates to avoid apparent
conflicts of interest. Advise all parties of any potential conflicts."
Answer (C) is incorrect.
It states an aspect of the confidentiality requirement.
Answer (D) is incorrect.
Maintenance of an appropriate level of professional expertise states an aspect of the competence
requirement.

Question #34: (Please Refer the question below:)


A Company currently manufactures all component pans used in the manufacture of various hand
tools. A handle is used in three different tools. The unit cost budget for 20,000 handles is

Direct material $0.60


Direct labor 0.40
variable overhead 0.10
fixed overhead 0.20
total unit cost $1.30

A parts manufacturer has offered to supply 20,000 handles to Cohasset for S1.25 each, delivered
If Company currently has idle capacity that cannot be used, accepting the offer will
Correct Answer: B. Increase the handle unit cost by $.15.
A. Decrease the handle unit cost by $.05.
B. Increase the handle unit cost by $.15.
C. Decrease the handle unit cost by $.15.
D. Increase the handle unit cost by $.05.
Explanation:
Answer (A) is incorrect.
The firm will incur the fixed costs whether it buys the parts or not
Answer (B) is correct.
Since the fixed cost will be incurred whether the company makes or buys the part the relevant
unit cost of making the part is the $1.10 variable cost ($1.30 - $.20 fixed overhead). The
existence of idle capacity indicates that the firm has no opportunity cost to be considered in the
calculation. Thus, accepting the offer would increase costs by $.15 per unit.
Answer (C) is incorrect
The firm will incur the fixed costs whether it buys the parts or not
Answer (D) is incorrect.
The firm will incur the fixed costs whether it buys the parts or not

Question #35: Please refer the question below:


A manufacturer has been approached by a new customer who wants to place a one-time order for
a component similar to one that the manufacturer makes for another customer. Existing sales will
not be affected by acceptance of this order. The manufacturer has a policy of setting its targeted
selling price at 60% over full manufacturing cost. The manufacturing costs and the targeted
selling price for the existing product are presented as follows.

Direct material $2.30


Direct labor 3.60
variable manufacturing overhead (applied at 75% of direct labor
cost) 2.70
fixed manufacturing overhead (applied at 150% of direct labor
cost) 5.40
total manufacturing unit cost $14.00
markup (60% of full manufacturing cost) 8.40
Targeted selling price $22.40

The manufacturer has excess capacity to produce the quantity of the component desired by the
new customer. The direct materials used in the component for the new customer would cost the
manufacturer 50.25 less than the component currently being made. The variable selling expenses
(packaging and shipping) would be the same, or $0.90 per unit. Under these circumstances, the
minimum unit price at which the manufacturer would accept the special order is one exceding
Correct Answer: B. $9.25
A. $8.35
B. $9.25
C. $14.00
D. $14.80

Explanation:
Answer (A) is incorrect.
This amount does not consider the variable selling expenses.
Answer (B) is correct.
Because the manufacturer has excess capacity and existing sales will be unaffected, the
minimum price the manufacturer should be willing to accept is anything above the total variable
cost of the unit ($2.05 + $3.60 + $2.70 $0.90 = $9.25), an amount that includes the variable
manufacturing cost and the variable selling expenses. The fixed costs are not relevant.
Answer (C) is incorrect
The total manufacturing cost is $14.00, which includes the fixed manufacturing overhead and
direct materials at $2.30 per unit, but not the variable selling expenses. Additional fixed
manufacturing overhead costs will not be incurred because the manufacturer is below full
capacity. The fixed manufacturing overhead is a sunk cost that is not relevant to this decision.
Answer (D) is incorrect.
This amount does not consider that the manufacturer is below full capacity and that the customer
is placing a one-time order. Under these circumstances, the manufacturer would not use its
targeted selling price formula.

Question #36: Kell Inc. is analyzing an investment for a new product expected to have annual
sales of 100.000 units for the next 5 years and then be discontinued. New equipment will be
purchased for $1,200,000 and cost $300,000 to install. The equipment will be depreciated on a
straight-line basis over 5 years for financial reporting purposes and 3 years for tax purposes. At
the end of the fifth year, it will cost $100,000 to remove the equipment, which can be sold for
$300,000. Additional working capital of $400,000 will be required immediately and needed for
the life of the product. The product will sell for $80, with direct labor and material costs of $65
per unit. Annual indirect costs will increase by $500,000, Kell's effective tax rate is 40%.
In a capital budgeting analysis, what is the expected cash flow at time = 3 (3rd year of operation)
that Kell should use to compute the net present value?
Correct Answer: B. $800.000.
A. $720,000.
B. $800,000.
C. $300,000.
D. $760,000.
Explanation:
A. This answer results from depreciating the equipment over 5 years. The question says
“The equipment will be depreciated on a straight-line basis over 5 years for financial
reporting purposes and 3 years for tax purposes." In capital budgeting, we always use the
depreciation method that is used for tax purposes to calculate the depreciation tax shield.
B. The cash flows in the third year will be: Annual Cash Flow from Operations:
Contribution Margin = $15/unit. For 100,000 units it will be $1,500,000. After tax, it will
be $1,500,000 x 0.60, which is $900,000. Depreciation Tax Shield: Tax depreciation will
be $500,000 in each of Years 1, 2 and 3 ($1,500,000 ÷ 3): so the Depreciation Tax Shield
for Year 3 will be 40% of that, or $200.000. Indirect costs each year (after tax) will be
($500,000) x 0.60, or ($300,000). Net cash flow for Year 3 will be: $200,000 + $900,000
- $300,000 = $800,000.
C. This is the incremental indirect costs after tax. The net cash flow for the third year
consists of that plus other components.
D. This answer results from subtracting the salvage value ($300,000) from the initial cost
($1,500,000) and dividing the net amount ($1,200,000) by 3 to calculate the amount of
annual straight-line depreciation and the depreciation tax shield. For capital budgeting
purposes. 100% of the asset's initial cost must be depreciated. We do that because we are
calculating the amount of tax savings that results from the annual depreciation. Tax
regulations in the U.S. require that 100% of an asset's cost be depreciated on the tax
return (this is different from U.S. GAAP requirements to use the net of the cost less the
salvage value). Since we are calculating the tax savings, we must use the depreciation
method required on the tax return.

Question #37: Oakes Inc. manufactured 40,000 gallons of Mononate and 60,000 gallons of
Beracyl in a joint production process, incurring $250,000 of joint costs. Oakes allocates joint
costs based on the physical volume of each product produced. Mononate and Beracyl can each
be sold at the split-off point in a semi-finished state or, alternatively processed further.
Additional data about the two products are as follows.

Mononate Beracyl
Sales price per gallon at split-off $7 $15
Sales price per gallon if processed further $10 $18
$115,00
Variable production costs if processed further $125,000 0
An assistant in the company's cost accounting department was overheard saying that "....when
both joint and separable costs are considered, the firm has no business processing either product
beyond the split-off point. The extra revenue is simply not worth the effort "Which of the
following strategies should be recommended for Oakes?
Correct Answer: B. Sell at split-off (Mononate). (Beracyl) Process further.
A. Sell at split-off (Mononate), (Beracyl) Sell at split-off
B. Sell at split-off (Mononate). (Beracyl) Process further
C. Process further (Mononate). (Beracyl) Sell at split-off
D. Process further (Mononate). (Beracyl) Process further
Explanation:
Answer (A) is incorrect.
Beracyl should be processed further.
Answer (B) is correct.
The additional revenue to be received from processing each product further should be compared
with the costs of processing the product further. The joint costs are irrelevant to this decision
because they have already been incurred. Mononate: 40,000 gallons x $3 additional revenue =
$120,000 additional revenue if processed further Variable cost to process further = 125,000 Loss
from processing further = $(5,000) Beracyl: 60,000 gallons x $3 additional revenue = $180,000
additional revenue if processed further Variable cost to process further = $115,000 Profit from
processing further = $65,000 Mononate should not be processed further but should be sold at the
split-off point, because to process it further would mean the company would lose $5,000.
However, Beracyl should be processed further because the company can earn additional
operating income of $65,000 by processing it further.
Answer (C) is incorrect
Mononate should be sold at split-off and Beracyl should be processed further.
Answer (D) is incorrect.
Mononate should be sold at split-off.

Question #38: All of the following are potential benefits of risk management except:
Correct Answer: D. Decreased inherent risk.
A. Reduced cost of capital.
B. More effective resource allocations.
C. Increased understanding of entity objectives.
D. Decreased inherent risk.
Explanation:
Inherent risk is the risk that exists before any controls are implemented to mitigate such risk. It is
the probability of a threat occurring.

Question #39: The accounting rate of return


Correct Answer: D. Focuses on income as opposed to cash flows.
A. Is synonymous with the internal rate of return.
B. Is inconsistent with the divisional performance measure known as return on investment.
C. Recognizes the time value of money.
D. Focuses on income as opposed to cash flows.
Explanation:
A. The accounting rate of return is very different from the internal rate of return. The
internal rate of return is the discount rate at which the net present value of an investment
is zero. The accounting rate of return does not incorporate present value concepts at all.
B. The accounting rate of return is not inconsistent with the divisional performance measure
known as Return on Investment (ROI). In fact, the accounting rate of return is quite
consistent with Return on Investment.
C. The accounting rate of return does not take into account the time value of money.
D. The accounting rate of return uses accrual accounting income, including depreciation,
rather than cash flows. It does not take into account the time value of money, and for that
reason it is also called the unadjusted rate of return model.

Question #40: Selected financial data for ABC Company is presented below.
• For the year just ended ABC has net income of $5,300,000.
• $5,500,000 of 7% convertible bonds were issued in the prior year at a face value of $1,000.
Each bond is convertible into 50 shares of common stock. No bonds were converted during the
current year.
• 50,000 shares of 10% cumulative preferred stock par value $100, were issued in the prior year.
Preferred dividends were not declared in the current year, but were current at the end of the prior
year.
• At the beginning of the current year 1,060,000 shares of common stock were outstanding. On
June 1 of the current year 60,000 shares of common stock were issued and sold.
• ABC's average income tax rate is 40%. ABC Company's basic earnings per share for the
current fiscal year is:
Correct Answer: B. $4.38.
A. $4.29.
B. $4.38.
C. $3.67.
D. $4.73.
Explanation:
The shares outstanding is calculated as 1,095,000 = 1,060,000(5/12) + 1,120,000(7/12) =
441,667 + 653,333. The EPS formula = (Net income - Preferred Dividends) / Weighted Average
shares outstanding or (5,300,000 net income – 500,000 [50,000(100) (.1)] preferred dividend)/
1,095,000 weighted shares outstanding = 4,800,000/1,095,000 = $4.38 basic earnings per share.

Question #41: Devlin Company’s acid test ratio at May 31, Year 2, was
Devlin Company
Statement of financial position as of May 31
(In thousands)

Assets Year 2 Year 1


Current assets
Cash $45 $38
Trading securities 30 20
Accounts receivable (net) 68 48
Inventory 90 80
Prepaid expenses 22 30
Total current assets 255 216
Investment, at equity 38 30
Property, Plant & equipment (net) 375 400
Intangible assets (net) 80 45
Total assets $748 $691
Liabilities and shareholders' equity
Current liabilities
Notes payable $35 $18
Accounts payable 70 42
Accrued expenses 5 4
Income taxes payable 15 16
Total current liabilities 125 80
Long term debt 35 35
Deferred taxes 3 2
Total liabilities 163 117
Shareholders' equity
Preferred stock, 6%, $100 par value,
cumulative 150 150
Common stock, $10 par value 225 195
Additional paid in capital--common stock 114 100
Retained earnings 96 129
Total shareholders' equity 585 574
Total liabilities and shareholders' equity $748 $691

Devlin Company
Income Statement for the year ended May 31
(In thousands)

Year 2 Year 1
Net sales $480 $460
Costs and expenses
Cost goods sold 330 315
Selling, general &
administrative 52 51
Interest expense 8 9
Income before taxes 90 85
Income taxes 36 34
Net income $54 $51

Correct Answer: D. 1.14 to 1.


A. 2.04 to 1.
B. 1.86 to 1.
C. 0.60 to 1.
D. 1.14 to 1.
Explanation:
The acid-test ratio (also known as the quick ratio) is the ratio of quick assets to current liabilities.
Quick assets are those easily converted to cash without significant loss. The quick assets are
cash, short-term investments (trading securities), and net receivables.
Devlin's acid-test ratio for Year 2 is calculated:
Acid-test ratio = (cash + marketable securities + net receivables) /current liabilities
Acid-test ratio = ($45 cash + $30 trading securities + $68 accounts receivable (net)) / ($125
current liabilities)
Acid-test ratio = $143 / $125 = ratio of 1.14 to 1.

Question #42: The best reason for financial managers to use shelf registration for debt or equity
offerings is because:
Correct Answer: A. Issuers are spared the expense of filing several registrations by registering
offerings in advance and then issuing them quickly under favorable market conditions.
A. Issuers are spared the expense of filing several registrations by registering offerings in
advance and then issuing them quickly under favorable market conditions.
B. Corporations do not have to register new securities with the Securities and Exchange
Commission.
C. This technique provides businesses more flexibility in selection of an investment banker.
D. Smaller, regional underwriters are used more frequently to distribute the new issues,
saving firms issuance costs.
Explanation:
A. A shelf registration enables a company to register shares but then not immediately issue
them. A company that qualifies for a shelf registration is able to make one large
registration and then sell the shares as they are needed instead of making many smaller
registrations as there is a need to sell the shares.
B. Shares registered under a shelf registration must still be registered with the SEC.
C. The use of a shelf registration does not impact the decision about the investment banker
that will be used in the issuance.
D. Whether a shelf registration is used or not does not impact the size of the underwriter
used to distribute the issuance. That is determined by the size and scope of the issuance.

Question #43: The effective annual interest rate to the borrower of a $100.000 one-year loan with
a stated rate of 7% and a 20% compensating balance is
Correct Answer: B. 8.75%.
A. 8.4%.
B. 8.75%.
C. 7.0%.
D. 13.0%.
Explanation:
A. This is 7% x 1.20. This is not the correct way to find the answer to this question. The
correct way to calculate the effective annual rate on this loan is to first calculate the
annual interest that will be due on $100,000 at a stated rate of 7% for one year. Then, find
the net funds from the loan the borrower will have available after deducting the
compensating balance requirement. The effective annual interest rate on a loan with a
compensating balance when no interest is received on the compensating balance is the
annual interest due divided by the net funds the borrower will have available to use.
B. The annual interest on $100,000 at a stated interest rate of 7% is $7000 ($100,000 x
0.07). The net available funds to the borrower if a 20% compensating balance is required
is 80% of $100,000, or $80,000. The effective annual interest rate on a loan with a
compensating balance when no interest is received on the compensating balance is the
annual interest due divided by the net funds the borrower will have available to use. Thus
the effective annual interest rate on the loan is $7,000 + $80,000, which equals 0.0875 or
8.75%.
C. 7.0% is the stated interest rate on the loan. Because the borrower has a compensating
balance requirement, not all of the loan proceeds will be available to the borrower to use.
The effective annual interest rate on a loan with a compensating balance when no interest
is received on the compensating balance is the annual interest due divided by the net
funds the borrower will have available to use, and it is different from the stated interest
rate.
D. This is 20% minus 7%. That is not the correct way to find the answer to this question.
The correct way to calculate the effective annual rate on this loan is to first calculate the
annual interest that will be due on $100,000 at a stated rate of 7% for one year. Then, find
the net funds from the loan the borrower will have available after deducting the
compensating balance requirement. The effective annual interest rate on a loan with a
compensating balance when no interest is received on the compensating balance is the
annual interest due divided by the net funds the borrower will have available to use.

Question #44: The following information concerning Arnold Company's common stock was
included in the company's financial reports for the last two years.

Year 2 Year 1
Market price per share on December 31 $60 $50
Par value per share 10 10
Earnings per share 3 3
Dividents per share 1 1
Books value per share on December 31 36 34
Based on the price-earnings information, investors would most likely consider Amold's common
stock to:
Correct Answer: B. show a positive trend in growth opportunities in Year 2 compared to Year 1.
A. Be overvalued at the end of Year 2.
B. Show a positive trend in growth opportunities in Year 2 compared to Year 1.
C. Show a decline in growth opportunities in Year 2 compared to Year 1.
D. Indicate inferior investment decisions by management in Year 2.
Explanation:
The company's P/E (price/earnings) ratio increased from year 1 to year 2. The P/E ratio is
calculated by taking the market price per share and dividing it by the earnings per share.
P/E ratio = (market price per share)/ (earnings per share)
P/E ratio, year 1 = 350 / 33 = 16.67
P/E ratio, year 2 = 360 / 33 = 20
Since the P/E ratio is increasing from year 1 to year 2, it is showing a positive trend in
growth opportunities in year 2 compared to year 1.

Question #45: A firm has $10 million in equity and $30 million in long-term debt to finance
its operations. The firm's beta is 1.125, the risk-free rate is 6%, and the expected market
return is 14%. The firm issued long-term debt at the market rate of 9%. Assume the firm is at
its optimal capital structure. The firm's effective income tax rate is 40%. What is the firm's
weighted average cost of capital?
Correct Answer: A. 7.8%.
A. 7.8%.
B. 8.6%.
C. 9.5%.
D. 10.5%.
Explanation:
A. The cost of equity is calculated using the Capital Asset Pricing Model:
R = R F + β (R M - R F).
R = 0.06 + 1.125(0.14 - 0.06)
R = 0.15 The after-tax cost of debt is the interest rate on the debt multiplied by 1 - the tax
rate: 0.09(1 - 0.40) = 0.054
Total capital is $40 million ($10 million equity plus $30 million debt). Debt is 75% of
total capital ($30 million ÷ $40 million) and equity is 25% of total capital ($10 million ÷
$40 million). Therefore, the firm's weighted average cost of capital is (0.75 x 0.054) +
(0.25 x 0.15) = 0.078 or 7.8%.
B. This answer results from weighting the firm's equity at 33-1/3% and weighting the firm's
debt at 66-2/3% in calculating the weighted average cost of capital. The firm has $10
million in equity and $30 million in long-term debt, for total capital of $40 million. $10
million in equity is 25% of total capital while $30 million in debt is 75% of total capital.
C. This is not the correct answer. Please see the correct answer for an explanation.
D. This answer results from using the firm's cost of debt at its gross amount in calculating
the weighted average cost of capital. The firm's cost of debt should by reduced to its tax-
equivalent rate by multiplying it by 1- the tax rate.

Question #46: Marble Savings Bank has received loan applications from three companies in the
auto parts manufacturing business and currently has the funds to grant only one of these requests.
Specific data, shown below, has been selected from these applications for review and
comparison with industry averages.

Bailey Nutron Sonex Industry


Total sales (millions) $4.27 $3.91 $4.86 $4.30
Net profit margin 9.55% 9.85% 10.05% 9.65%
Current ratio 1.82 2.02 1.96 1.95
Return on assets 12.0% 12.6% 11.4% 12.4%
Debt / equity ratio 52.5% 44.6% 49.6% 48.3%
Financial leverage 1.30 1.02 1.56 1.33

Based on the information above, select the strategy that should be most beneficial to Marble
Savings.
Correct Answer: C. Grant the loan to Nutron as both the debt/equity (D/E) ratio and degree of
financial leverage (DFL) are below the industry average.
A. Grant the loan to Bailey as all the company's data approximate the industry average.
B. Marble Savings Bank should not grant any loans as none of these companies represents a
good credit risk.
C. Grant the loan to Nutron as both the debt/equity (D/E) ratio and degree of financial
leverage (DFL) are below the industry average.
D. Grant the loan to Sonex as the company has the highest net profit margin and degree of
financial leverage.
Explanation:
The D/E ratio and the DFL both measure an organization's risk. The lower the debt/equity
ratio, the lower the risk. Similarly, the lower the degree of financial leverage, the lower the
risk. In the case of Nutron, both the D/E ratio and DFL measures are below the industry
averages. Therefore, it is the least risky of the three choices.

Question #47: A new accounting clerk at a firm that had recently terminated several
employees due to budgetary cutbacks accidentally viewed a supervisor's biweekly paycheck.
Not realizing that the paycheck included an annual bonus, the accounting clerk erroneously
multiplied the gross pay by 26 to find annual earnings. The accounting clerk was amazed that
this supervisor appeared to earn more than twice the local average for employees in an
accounting supervisory position. The accounting clerk discussed this situation with a friend, a
recently terminated employee of the company who now worked for a local newspaper. As a
result of this discussion, the supervisor's "outrageous* salary was made public. Which one of
the standards of the IMA's Statement of Ethical Professional Practice did the accounting
clerk's actions violate?
Correct Answer: B. Confidentiality.
A. Competence.
B. Confidentiality.
C. Integrity.
D. Credibility.
Explanation:
Answer (A) is incorrect.
The standard of competence relates to a member's responsibility to (1) maintain an
appropriate level of professional expertise by continually developing knowledge and skills;
(2) perform professional duties in accordance with relevant laws, regulations, and technical
standards; (3) provide decision support information and recommendations that are accurate,
clear, concise, and timely; and (4) recognize and communicate professional limitations or
other constraints that would preclude responsible judgment or successful performance of an
activity.
Answer (B) is correct.
The standard of confidentiality states each member has a responsibility to (1) keep
information confidential except when disclosure is authorized or legally required. (2) inform
all relevant parties regarding appropriate use of confidential information and monitor
subordinate? activities to ensure compliance, and (3) refrain from using confidential
information for unethical or illegal advantage.
Answer (C) is incorrect
The standard of integrity relates to a member's responsibility to (1) mitigate actual conflicts
of interest, regularly communicate with business associates to avoid apparent conflicts of
interest, and advise all parties of any potential conflicts: (2) refrain from engaging in any
conduct that would prejudice carrying out duties ethically: and (3) abstain from engaging in
or supporting any activity that might discredit the profession.
Answer (D) is incorrect.
The standard of credibility relates to a member's responsibility to (1) communicate
information fairly and objectively; (2) disclose all relevant information that could reasonably
be expected to influence an intended user's understanding of the reports, analyses, or
recommendations; or (3) disclose delays or deficiencies in information, timeliness,
processing, or internal controls in conformance with organization policy and/or applicable
law.

Question#48: Assuming that Lisa Inc.’s net income for Year 2 was $35,000 and there are no
preferred stock dividends in arrears, Lisa return on common equity (ROE) for Year 2 was:

Assets Year 2 Year 1


Current assets $30 $25
Marketable securities (at lower of cost or market) 20 15
Accounts receivable (net) 45 30
Invetories (at lower of cost or market) 60 50
Prepaid items 15 20
Total current assets 170 140
Long term investments securities (at cost) 25 20
Property, plant, and equipment )
Land (at cost) 75 75
Building (net) 80 90
Equipment (net) 95 100
Intangible assets (net)
Patents (net) 35 17
Goodwill (net) 20 13
Total long term assets 330 315
total assets $500 $455
Liabilities and shareholders' equity
Current liabilities
Notes payable $23 $12
Accounts payable 47 28
Accrued interest 15 15
Total current liabilities 85 55
Long term debt
Notes payable 10% 10 10
Bonds payable 12% 15 15
Total long-term debt 25 25
Total Liabilities 110 80

Shareholders' equity
Preferred--5% cumulative, $100 par, non-participating, authorized, issued and
outstanding, 1,000 shares 100 100
Common--$10 par 20,000 shares authorized, 15,000 issued and outstanding shares 150 150
Additional paid-in capital--common 75 75
Retained earnings 65 50
Total shareholders' equity 390 375
Total liabilities and equity $500 $455

Assuming that Lisa Inc.’s net income for Year 2 was $35,000 and there are no preferred
stock dividends in arrears, Lisa return on common equity (ROE) for Year 2 was:
Correct answer: B. 10.6%.
A. 12.4%.
B. 10.6%.
C. 7.8%.
D. 10.9%.

Explanation: ROE is calculated as:


ROE = (net income - preferred stock dividends) / (average common shareholders' equity)
The average common shareholders' equity is calculated by taking the beginning balance of
common shareholders' equity less the preferred stock equity, plus the ending balance of common
shareholders' equity less the preferred stock equity, and dividing that total by 2. Dividing by two
calculates the average for the period.
Average common shareholders' equity = [(beginning balance common shareholders' equity -
preferred stock equity) + (ending balance common shareholders' equity - preferred stock equity)]
/2
Lisa's average common shareholders' equity = [($390 - $100) + ($375 - $100)] / 2 = ($565) / 2 =
$282.50
Lisa's preferred stock dividend is 5% of $100, or 0.05($100) = $5. Net income is $35. Therefore.
ROE = ($35 - $5) / ($282.50) = 430/$282.50 = 0.106. or 10.6%.
Question #49: Computer Services is an established firm that sells computer hardware, software
and services. The firm is considering a change in its credit policy. It has been determined that
such a change would not change the payment patterns of the current customers. To determine
whether such a change would be beneficial, the firm has identified the proposed new credit
terms, the expected additional sales, the expected contribution margin on the sales, the expected
bad debt losses, and the investment in additional receivables and the period of the investment.
What additional information, if any, does the firm require to determine the profitability of the
proposed new policy as compared to the current credit policy?
Correct Answer: A. The opportunity cost of funds.
A. The opportunity cost of funds.
B. No additional information is needed.
C. The new credit standards.
D. The credit standards that presently exist.
Explanation:
A. The Company will need to borrow more money because its outstanding receivables
balance as well as its inventory will be higher due to the additional sales expected. The
opportunity cost of funds along with information on the investment that would be
required in increased receivables and inventory will tell the company what its increased
interest cost will be because of needing to carry the increased receivables.
B. Some additional information is needed in order to determine the profitability of the
proposed new policy as compared to the current credit policy.
C. The new credit standards are already known. The question asks what additional
information is needed.
D. The credit standards that presently exist are already known. The question asks what
additional information is needed.

Question #50: Question: The variance of actual contribution margin from budgeted contribution
margin attributable to sales price is
Xerbert Co.
Budget and Actual Income Statements
For the year Ending December 31
(00s omitted)
Xenox Xeon Total Xenox Xeon Total
Unit sales 150 100 250 130 130 260
Net dollar sales $900 $1,000 $1,900 $780 $1,235 $2,015
Variable expenses (450) (750) (1,200) (390) (975) (1,365)
Contribution margin $450 $250 $700 $390 $260 $650
Fixed expenses:
Manufacturing $153 $140
Marketing 95 90
Other fixed expenses 200 190
Total fixed expenses $448 $420
Income before taxes $252 $230

Correct Answer: C. $65,000 unfavorable.


A. $115,000 unfavorable.
B. $115,000 favorable.
C. $65,000 favorable.
D. $65,000 unfavorable.

Explanation:
Answer (A) is incorrect.
The total variance between actual and budgeted sales is $115,000 (favorable). The sales price
variance is $65,000 (unfavorable), and the sales quantity variance is $180,000 (favorable), for a
total favorable variance of $115,000. The sales price variance is unfavorable because actual sales
were less than budgeted sales, and the sales quantity variance is favorable because actual
quantity exceeded budgeted quantity.
Answer (B) is incorrect.
The total variance between actual and budgeted sales is $115,000 (favorable). The sales price
variance is $65,000 (unfavorable), and the sales quantity variance is $180,000 (favorable), for a
total favorable variance of $115,000. The sales price variance is unfavorable because actual sales
were less than budgeted sales, and the sales quantity variance is favorable because actual
quantity exceeded budgeted quantity.
Answer (C) is incorrect
The actual sales price for Xeon was less than the budgeted sales price, creating a $65,000
unfavorable sales price variance.
Answer (D) is correct.
The budgeted prices of Xenox and Xeon were $6 and $10 per unit, respectively. Actual sales in
units multiplied by the budgeted prices equal total sales of $2,080,000. Because actual sales were
only $2,015,000, the variance of $65,000 is unfavorable (lower sales than budgeted).

Question #51: A firm's total assets are $3,000,000 and its total equity is $2,000,000. If current
assets represent 50% of total assets and current liabilities represent 30% of total liabilities, then
what is the firm's working capital?
Correct Answer: D. $1,200,000.
A. $500,000.
B. $4,000,000.
C. $1,800,000.
D. $1,200,000.
Explanation:
Working capital is current assets minus current liabilities: $1,500,000 - $300,000 = $1,200,000.

Question #52: The DCL Corporation is preparing to evaluate the capital expenditure proposals
for the coming year. Because the firm employs discounted cash flow methods of analyses, the
cost of capital for the firm must be estimated. The following information for DCL Corporation is
provided:
-Market price of common stock is $50 per share. -The dividend next year is expected to be $2.50
per share. -Expected growth in dividends is a constant 10%. -New bonds can be issued at face
value with a 13% coupon rate. -The current capital structure of 40% long-term debt and 60%
equity is considered to be optimal. -Anticipated earnings to be retained in the coming year are $3
million. -The firm has a 40% marginal tax rate.
If the firm must assume a 10% flotation cost on new stock issuances, what is the cost of new
common stock?
Correct Answer: B. 15.56%.
A. 15.00%.
B. 15.56%.
C. 16.11%.
D. 15.05%.
Explanation:
A. This answer does not include the 10% flotation costs. See the correct answer for a
complete explanation.
B. The cost of funds from the sale of new shares of stock can be calculated with the
following formula: dl C ns = +g P n
Where:
C ns = Cost of the new issuance of common stock
d 1= The next dividend to be paid
P n = Net proceeds of the issue (selling price minus issuance costs)
g = Annual expected %growth in dividends
Note that the dividend given as $2.30 per share is for the next year. Since it is for next
year and the formula calls for the next dividend to be paid, the dividend does not need to
be adjusted upward by one year's growth before it is used in the formula.
Inputting the information into this equation we get:

2.50
Cns = +0.10=0.1556 ,∨15.56 %
50−(0.10 ×50)

C. This answer applies the dividend growth to next year's dividend, making next year's
dividend $2.75 instead of $2.50 as it is stated. See the correct answer for a complete
explanation.
D. This answer uses the present value of the next year's dividend, instead of the cash value.
See the correct answer for a complete explanation.

Question #53: Within a financial risk management context, the term Value at Risk (VAR) is
defined as the:
Correct Answer: C. maximum loss within a certain time period at a given level of confidence.
A. Maximum value a company can lose.
B. Most likely negative outcome.
C. Maximum loss within a certain time period at a given level of confidence.
D. Worst possible outcome given the distribution of outcomes.
Explanation:
VAR is defined as the maximum loss within a given period of time and given a specified
probability level (level of confidence).
Question #54: Datacomp Industries, which has no current debt, has a beta of 0.95 for its common
stock. Management is considering a change in the capital structure to 30% debt and 70% equity.
This change would increase the beta on the stock to 1.05, and the after-tax cost of debt will be
7.5%. The expected return on equities is 16%, and the risk-free rate is 6%. Should Datacomp's
management proceed with the capital structure change?
Correct Answer: D. Yes, because the weighted average cost of capital will decrease.
A. Yes, because there will be no effect on the weighted average cost of capital.
B. No, because the cost of equity capital will increase.
C. No, because the weighted average cost of capital will increase.
D. Yes, because the weighted average cost of capital will decrease.
Explanation:
A. Though the change should be made, this reason for making the change is incorrect. The
change should be made because the weighted average cost of capital will decrease. If the
weighted average cost of capital will remain the same, there is no need to make the
change.
B. The weighted average cost of capital will decrease if this change is made, so the change
should be made. See the correct answer for a complete explanation.
C. The weighted average cost of capital will decrease if this change is made, so the change
should be made. See the correct answer for a complete explanation.
D. Using the CAPM formula, which is r = r F + β (r M - r F), we can determine that the
current cost of capital consisting only of equity for the firm is 15.5%:
r = 0.06 0.95(0.16 - 0.06) = 0.155.
The proposed capital structure will have an after tax cost equal to 13.8%:
(0.3 x 0.075) + (0.7 x (0.06 1.05(0.16 - 0.06))) = 0.138.
So, the company should proceed with the capital change as it will reduce the WACC
from 15.5% to 13.8%.

Question #55: A proposed transfer price may be based upon the full-cost price. Full-cost
price is the price
Correct Answer: C. Usually set by an absorption-costing calculation.
A. On the open market
B. Representing the cash outflows of the supplying division plus the contribution to the
supplying division from an outside sale.
C. Usually set by an absorption-costing calculation.
D. Set by charging for variable costs plus a lump sum or an additional markup, but less
than full markup.
Explanation:
Answer (A) is incorrect.
The market price is the price on the open market.
Answer (B) is incorrect.
The outlay cost plus opportunity cost is the price representing the cash outflows of the
supplying division plus the contribution to the supplying division from an outside sale.
Answer (C) is correct.
Full-cost price is the price usually set by an absorption-costing calculation and includes
materials, labor, and a full allocation of manufacturing OM. This full-cost price may lead
to dysfunctional behavior by the supplying and receiving divisions, e.g., purchasing from
outside sources at a slightly lower price that is substantially above the variable costs of
internal production.
Answer (D) is incorrect.
The variable-cost-plus price is the price set by charging for variable costs plus a lump
sum or an additional markup, but less than full markup.

Question #56: Which of the following statements describe the importance of a


whistleblowing framework in maintaining an ethical organizational culture? I. It provides
measurable feedback for determining whether employees are following a code of ethics.
II. It creates opportunities to enhance and improve internal controls. III. It empowers
management to become better role models for employees. IV. It helps to identify
potential errors or risks at each task level within the organization.
Correct Answer: A. I and II only.
A. I and II only.
B. II and IV only.
C. III and IV only.
D. I, III, and IV only.
Explanation:
Answer (A) is correct
The collection, analysis, and summarization of ethics issues can provide insight into the
operation of its code of ethics and the degree to which employees are following it, in
addition, tracking and monitoring issues raised through a whistleblowing framework
creates opportunities to enhance and improve internal control. For instance, a large
number of fraud cases are reported through a whistleblowing framework which can
highlight areas where internal controls were lacking.
Answer (B) is incorrect.
Each task level will not be analyzed by a whistleblowing framework because not all tasks
are at risk of fraud or weak controls.
Answer (C) is incorrect.
The whistleblowing framework would scare management into becoming better role
models, not empower them. Also, each task level will not be analyzed by a
whistleblowing framework because not all tasks are at risk of fraud or weak controls.
Answer (D) is incorrect.
The whistleblowing framework would scare management into becoming better role
models, not empower them. Also, each task level will not be analyzed by a
whistleblowing framework because not all tasks are at risk of fraud or weak controls.

Question #57: The currency of the primary environment in which the entity operates is
commonly called the:
Correct Answer: D. Functional currency.
A. Historical currency.
B. Reporting currency.
C. Floating currency.
D. Functional currency.
Explanation:
For foreign subsidiaries the firms are required to identify a "functional currency" as the
currency of the primary environment in which the entity operates.

Question #58: An investor is deciding whether to invest in either of two companies


(Company A and Company B) within the same industry. Both companies have a 15%
profit margin and total assets of $15.000.000. Which of the following is true when
measuring these companies based on their Return on Assets (ROA)?
Correct Answer: B. The Company with the higher sales per year will be more attractive.
A. The company smith the lower sales per year will be more attractive.
B. The company with the higher sales per year will be more attractive.
C. Each company is equally attractive as their assets are equal.
D. Both are equally profitable, therefore they are equally attractive.
Explanation:
Given the calculation for ROA = Net Income /Average Total Assets, and the profit
margin % and asset value given is the same for both companies, the company with the
higher sales per year will generate a higher amount of net income dollars resulting in the
higher ROA.

Question #59: There are many strategies for risk response that an organization may
choose from to reduce risk to an acceptable level. Which action would not reduce risk for
an organization?
Correct Answer: C. Adjusting the organization's capital structure to maximize the cost of
capital.
A. Purchasing property and casualty insurance.
B. Requiring all employees to be bonded.
C. Adjusting the organization's capital structure to maximize the cost of capital.
D. Shifting costs from fixed to variable.
Explanation:
This action would reduce risk by shifting risk to the bonding company.

Question #60: A company is in the process of evaluating a major product line expansion.
Using a 14% discount rate, the firm has calculated the present value of both the projects
cash inflows and cash outflows to be $15.8 million. The company will likely evaluate this
project further by
Correct Answer: D. comparing the internal rate of return versus the company's cost of
capital and hurdle rate.
A. Comparing the internal rate of return versus the discount rate used.
B. Taking a closer look at the expansion's contribution margin.
C. Comparing the internal rate of return versus the accounting rate of return.
D. Comparing the internal rate of return versus the company's cost of capital and hurdle
rate.
Explanation:
A. The IRR is the discount rate at which the NPV of the project is zero. Since the present
value of both the project's cash inflows and cash outflows are the same ($15.8
million) when a discount rate of 14% is used, the projects NPV is zero at that
discount rate. Thus, the internal rate of return is the same as the discount rate, and
there is no new knowledge to be gained by comparing the two.
B. The contribution margin of a project is not a part of a capital budgeting analysis.
C. When the accounting rate of return method is used for capital budgeting, management
sets a required accounting rate of return, and projects whose returns exceed that rate
are considered acceptable. Comparison with the internal rate of return of the project
would not be meaningful, because the internal rate of return is based on cash flow
whereas the accounting rate of return is based on accounting income.
D. Since the present values of both the project's cash inflows and cash outflows are the
same ($15.8 million), the NPV of this project is zero, when a discount rate of 14% is
used. If the discount rate the company has used as a hurdle rate is truly representative
of its cost of capital and the anticipated risk in the project this is not a project the
company should embark upon, because it will not earn any profit for the shareholders.
So the company should evaluate whether the 14% that it used as its hurdle rate is
reasonable. To do that, it should compare the company's cost of capital with the
hurdle rate it used. This is the only answer choice that includes comparing the
company's cost of capital with the hurdle rate.

Question #61: The management team attended an ethics training session at the IMA Annual
Conference and subsequently made plans to enhance their ethics program. The president plans to
chair a committee of employees to review the company's behavioral values, while the CFO
intends to review the ethical standards applicable to the Finance Department. The manager of the
Human Resources Department will investigate the feasibility of establishing a whistleblowing
framework that includes a "hotline for reporting ethics violations. These activities exemplify
Correct Answer: B. Leadership by example.
A. Organizational transparency.
B. Leadership by example.
C. Measurement of ethical compliance.
D. Alignment of internal controls with ethical standards.
Explanation:
Answer (A) is incorrect.
These activities exemplify leadership by example.
Answer (B) is correct.
"Tone at the top" plays an important role in determining an organization's ethical environment.
For a code of ethics to be effective, its application must be demonstrated by those in positions of
power and leadership. Leaders must be seen living and managing by the code of ethics.
Answer (C) is incorrect.
These activities exemplify leadership by example.
Answer (D) is incorrect.
These activities exemplify leadership by example.
Question #62: All of the following may reduce the coupon rate on a bond issued at par except a
Correct Answer: B. Call provision.
A. Conversion option.
B. Call provision.
C. Sinking fund.
D. Change in rating from Aa to Aaa.
Explanation:
A. A conversion option is considered to be a benefit by the investor because it enables the
investor to convert the bonds to shares if that would be beneficial. Therefore, bonds that
have a conversion feature will be able to issued at a lower interest rate.
B. A call provision is considered to be detrimental by the bondholder because this gives the
issuer the right to retire the bonds at any time prior to maturity. This option will be
exercised if interest rates fall and the issuer is able to find other, cheaper sources of
financing. In this environment, the investor may not be able to find an investment with
the same rate of return. To cover for this increased level of risk, the investor will require
that callable bonds pay a higher rate of interest.
C. Investors prefer bonds to have sinking funds since it provides some guarantee that the
bonds will be able to be paid when they mature. Because investors like sinking funds,
bonds with sinking funds will be able to be issued at a lower rate of interest.
D. This change in rating is an increase in the rating, meaning that there is less risk associated
with the bonds. This will enable the issuer to decrease the interest rate of the bonds.

Question #63: Systematic risk is


Correct Answer: B. risk that cannot be diversified away by holding securities in a diversified
portfolio.
A. Risk that can be quantified.
B. Risk that cannot be diversified away by holding securities in a diversified portfolio.
C. The possibility that an investment cannot be sold (converted into cash) for its market
value.
D. Risk that can be diversified away by holding securities in a diversified portfolio.
Explanation:
A. Systematic risk cannot be quantified. (Quantify means to determine the amount of
something.)
B. Systematic risk also called market risk, is risk that cannot be diversified away, it is
created by the fact that economic cycles affect all businesses, and publicly-held
investments are traded in a market that can go up and down with economic news.
Systematic, or market, risk cannot be diversified away, and all investments are subject to
it.
C. The possibility that an investment cannot be sold for its market value is liquidity risk
D. Systematic risk is not risk that can be diversified away.

Question #64: The Enterprise Risk Management (ERM)-Integrated Framework recently released
by COSO expands upon the 1992 COSO model and provides for a more robust and extensive
focus on enterprise risk management. When compared with the 1992 Internal Control-Integrated
Framework issued by COSO, which of the following elements is or are new to the ERM
framework?
I. Control activities, information and communication, and monitoring
II. Internal environment, objective setting, and event identification
III. Risk assessment and risk response
Correct Answer: C.II and III.
A. II only.
B. III only.
C. II and III.
D. I only.
Explanation:
The ERM framework issued by COSO in September. 2004 added five new components to
the 1992 Internal Control-Integrated Framework. The five components are internal
environment, objective setting, event identification, risk assessment, and risk response.

Question #65: Grand Savings Bank has received loan applications from three companies in
the plastics manufacturing business and currently has the funds to grant only one of these
requests. Specific data, shown below, has been selected from these applications for review
and comparison with industry averages.

Springfield Reston Valley Industry


Total sales (millions) $4.27 $3.91 $4.86 $4.30
Net profit margin 9.55% 9.85% 10.05% 9.65%
Current ratio 1.82 2.02 1.96 1.95
Return on assets 12.0% 12.6% 11.4% 12.4%
Debt / Equity ratio 52.5% 44.6% 49.6% 48.3%
Financial leverage 1.30 1.02 1.56 1.33

Correct Answer:
C. Grant the loan to Reston as both the debt/equity (D/E) ratio and degree of financial
leverage (DFL) are below the industry average.
A. Grand should not grant any loans as none of these companies represents a good credit
risk.
B. Grant the loan to Springfield as all the company's data approximate the industry average.
C. Grant the loan to Reston as both the debt/equity (D/E) ratio and degree of financial
leverage (DFL) are below the industry average.
D. Grant the loan to Valley as the company has the highest net profit margin and degree of
financial leverage.
Explanation:
The D/E ratio and the DFL both measure an organization's risk. The lower the DIE ratio, the
lower the risk .Similarly, the lower the DFL, the lower the risk In the case of Reston, both the
D/E ratio and DFL measures are below the industry averages. Therefore, it is the least risky
of the three choices.

Question #66: Firms subject to the reporting requirements of the Securities Exchange
Act of 1934 are required by the Foreign Corrupt Practices Act of 1977 to maintain
satisfactory internal control. The role of the independent auditor relative to this act is to
Correct Answer: D. Attest to the financial statements.
A. Report clients with unsatisfactory internal control to the SEC.
B. Provide assurances to users as part of the traditional audit attest function that the
client is in compliance with the present legislation.
C. Express an opinion on the sufficiency of the client's internal control to meet the
requirements at the Act
D. Attest to the financial statements.
Explanation:
Answer (A) is incorrect.
The auditor is not required to report violations of the act to the SEC, although e duty to
disclose outside the client may exist in same circumstances; e.g., the client's failure to
take remedial action regarding an illegal act may constitute a disagreement that it must
report on Form 8-K (AU 317)
Answer (B) is incorrect.
The traditional attest function does not involve compliance auditing.
Answer (C) is incorrect
The FCPA contains no requirement that an auditor express an opinion on internal
control.
Answer (D) is correct
Whether a client is in conformity with the Foreign Corrupt Practices Act is a legal
question. Auditors cannot be expected to provide clients or users of the financial
statements with legal advice. The role at the auditor is to assess control risk in the course
of an engagement to attest to the fair presentation of the financial statements.

Question #67: MON Corporation has total assets of $7,000,000 .Its common equity is
$3,000,000, including paid-in capital and retained earnings. The company's net profit
margin is 5% with revenues of $13,050,000 and its preferred dividends are $25,000.
What is MON Corporation's return on common equity ratio?
Correct Answer: D. 0.21.
A. 0.31.
B. 0.36.
C. 0.28.
D. 0.21.
Explanation:
The return on common equity ratio is calculated by dividing net income minus preferred
dividends by common equity. Net income is computed as 5% of $13,050,000 = $652,500
Income available for common shareholders (net income minus preferred dividends) for
MON Corporation is $627,500 ($652,500 net income - $25,000 preferred dividends). Its
return on common equity ratio is 0.209 ($627,500 net income ÷ $3,000,000 common
equity).
Question #68: Williams, Inc. is interested in measuring its overall cost of capital and has
gathered the following data. Under the terms described as follows, the company can sell
unlimited amounts of all instruments.
Williams can raise cash by selling $1,000, 8%, 20-year bonds with annual interest
payments. In selling the issue, an average premium of $30 per bond would be received, and
the firm must pay flotation costs of $30 per bond. The after-tax cost of funds is estimated to
be 4.8%.
Williams can sell $8 preferred stock at par value, $100 per share. The cost of issuing and
selling the preferred stock is expected to be SS per share.
Williams' common stock is currently selling for $100 per share. The firm expects to pay
cash dividends of $7 per share next year, and the dividends are expected to remain constant.
The stock will have to be underpriced by $3 per share, and flotation costs are expected to
amount to $5 per share.
Williams expects to have available $100,000 of retained earnings in the coming year. Once
these retained earnings are exhausted, the firm will use new common stock as the form of
common stock equity financing.
The capital structure that Williams would like to use for any future financing is:
-Long-term debt: 30%
-Preferred stock: 20%
-Common stock: 50%
The cost of funds from retained earnings for Williams, Inc. is
Correct Answer: B. 7.0%
A. 7.6%.
B. 7.0%.
C. 7.4%.
D. 8.1%.
Explanation:
A. This is the cast of new common stock, not the cost of retained earnings.
B. The three elements required to calculate the cost of retained earnings are the
dividends per share, (2) the expected dividend growth rate, and (3) the market price
of the stack. Because growth in the dividend is not expected, the calculation is
simply to divide the dividend of $7 by the $100 market price of the stock to arrive at
a cost of retained earnings of 7%.
C. This would be the cost at new common stock if the underpricing required were not
included in the calculation of the cost. However, the question asks for the cost of
retained earnings.
D. This answer could result from using an anticipated growth rate in the dividend of
1.1%. However, the dividends are expected to remain constant.

Question #69: For each of the next six years Atlantic Motors anticipates net income of $10,000,
straight-line tax depreciation of $20,000, a 40% tax rate, a discount rate of 10%, and cash sales
of $100,000. The depreciable assets are all being acquired at the beginning of year 1 and will
have a salvage value of zero at the end of six years.
The present value of the total depreciation tax savings would be
Correct Answer: B. $34,840.
A. $27,072.
B. $34,840.
C. $8,000.
D. $87,100.
Explanation:
A. This is $8,000 (the annual depreciation tax shield) x 6 years x 0.564 (the present value of
$1 factor for 6 years at 10%). The annual depreciation tax shield is an annuity, and this is
not the correct way to calculate the present value of an annuity. The present value of an
annuity factor should be used.
B. The depreciation is $20,000 and the tax rate is 0.40. Therefore, the annual tax savings due
to the depreciation will be $20,000 x 0.40, or $8,000. The tax savings due to the
depreciation expense is called the depreciation tax shield. The present value of the total
depreciation tax shield would be $8,000 discounted using the PV of an annuity factor for
10% for 6 years.
C. This is $8,000 (the annual depreciation tax shield) x 6 years x 0.564 (the present value of
$1 factor for 6 years at 10%). The annual depreciation tax shield is an annuity, and this is
not the correct way to calculate the present value of an annuity. The present value of an
annuity factor should be used.
D. The depreciation is $20,000 and the tax rate is 0.40. Therefore, the annual tax savings due
to the depreciation will be $20,000 x 0.40, or $8,000. The tax savings due to the
depreciation expense is called the depreciation tax shield. The present value of the total
depreciation tax shield would be $8,000 discounted using the PV of an annuity factor for
10% for 6 years.
E. This is $8,000 (the annual depreciation tax shield) x 6 years x 0.564 (the present value of
$1 factor for 6 years at 10%). The annual depreciation tax shield is an annuity, and this is
not the correct way to calculate the present value of an annuity. The present value of an
annuity factor should be used.
F. The depreciation is $20.000 and the tax rate is 0.40. Therefore. the annual tax savings due
to the depreciation will be $20.000 x 0.40. or $8.000. The tax savings due to the
depreciation expense is called the depreciation tax shield. The present value of the total
depreciation tax shield would be $8.000 discounted using the PV of an annuity factor for
10% for 6 years. A. This is $8.000 (the annual depreciation tax shield) x 6 years x 0.564
(the present value of $1 factor for 6 years at 10%). The annual depreciation tax shield is
an annuity. and this is not the correct way to calculate the present value of an annuity.
The present value of an annuity factor should be used. B. The depreciation is 520.000 and
the tax rate is 0.40. Therefore. the annual tax savings due to the depreciation will be
$20.000 x 0.40. or $8.000. The tax savings due to the depreciation expense is called the
depreciation tax shield. The present value of the total depreciation tax shield would be
$8.000 discounted using the PV of an annuity factor for 10% for 6 years. 48.000 x 4.355
= $34.840. C. This is the annual tax savings due to the depreciation. The tax savings due
to the depreciation expense is called the depreciation tax shield. The question asks for the
present value of the depreciation tax shield. D. This is the present value of the
depreciation expense. The question asks for the present value of the tax savings due to the
depreciation expense. The tax savings due to the depreciation expense is called the
depreciation tax shield.

Question #70: An organization can mitigate the risk of financial loss from consumer product
injuries by purchasing:
Correct Answer: B. Liability insurance.
A. Health insurance.
B. Liability insurance.
C. Hazard insurance.
D. Workers compensation insurance.
Explanation:
Liability insurance is a part of the general insurance system of risk financing to protect the
purchaser (the Insured") from the risks of liabilities imposed by lawsuits and similar claims. It
protects the insured in the event he or she is sued for claims that come within the coverage of the
insurance policy.
Question #71: Corporations have the responsibility to issue financial statements that are timely.
accurate. and transparent. reflecting all the transactions of the company. Which of the following
documents refer to this responsibility?
I. IMAs Statement of Ethical Professional Practice
II. SOX Section 406: Code of Ethics for Senior Financial Officers.
III. IMAs Statement on Management Accounting "Values and Ethics: From Inception to
Practice"
IV. U.S. Foreign Corrupt Practices Act
Correct Answer: D. II and IV only.
A. I and II only.
B. I and III only.
C. II and III only.
D. II and IV only.
Explanation:
Answer (A) is incorrect.
The IMAs Statement of Ethical Professional Practice discusses ethical principles and standards
that should be followed by members of the IMA. This does not refer to the responsibility to issue
financial statements that are timely, accurate, and transparent, reflecting all the transactions of
the company.
Answer (B) is incorrect.
The IMAs Statement on Management Accounting 'Values and Ethics: From Inception to Practice
is a useful document for understanding ethical concepts in an organizational context. This does
not refer to the responsibility to issue financial statements that are timely, accurate, and
transparent, reflecting all the transactions of the company. The IMA's Statement of Ethical
Professional Practice discusses ethical principles and standards that should be followed by
members of the IMA. This does not refer to the responsibility to issue financial statements that
are timely, accurate, and transparent, reflecting all the transactions of the company.
Answer (C) is incorrect.
The IMAs Statement on Management Accounting 'Values and Ethics: From Inception to
Practice" is a useful document for understanding ethical concepts in an organizational context.
This does not refer to the responsibility to issue financial statements that are timely, accurate,
and transparent, reflecting all the transactions of the company.
Answer (D) is correct.
SOX Section 406: Code of Ethics for Senior Financial Officers and the U.S. Foreign Corrupt
Practices Act both refer to the corporate responsibility to issue financial statements that are
timely, accurate, and transparent, reflecting all the transactions of the company.
Question #72: Consider the statements below regarding accounting treatments under U.S. GAAP
and IFRSs. Which statements are correct?
I. U.S. GAAP permits the recording of extraordinary items on the income statement.
II. IFRS does not permit the use of LIFO to account for inventory.
III. Under IFRS, fair value accounting for property, plant and equipment is only allowed
when fair value is reliably measurable.
IV. Under U.S. GAAP research and development costs are capitalized as incurred.
Correct Answer: • A. I, II and III, only. All Possible Answers:
A. I, II, and III, only.
B. I and III, only
C. II, III and IV, only.
D. II and IV, only.
Explanation:
Answers I, II, and III are all true statements. Answer IV is not a correct statement Under U.S.
GAAP research and development costs are expensed as incurred, not capitalized.

Question #73: Shown below are beginning and ending balances for certain of Grimaldi Inc.’s
accounts.
January December
1 31
Cash $48,000 $62,000
Marketable
securities 42,000 35,000
Accounts receivable 68,000 47,000
Inventory 125,000 138,000
Plant & equipment 325,000 424,000
Accounts payable 32,000 84,000
Accrued liabilities 14,000 11,000
7% bonds payable 95,000 77,000

Grimaldi's acid test ratio or quick ratio at the end of the year is:
Correct Answer: B. 1.52.
A. 1.15.
B. 1.52.
C. 0.83.
D. 1.02.
Explanation:
The quick ratio is calculated as: Quick ratio = (current assets - inventories - prepayments) /
(current liabilities)
Current assets = (cash + marketable securities + receivables + inventory)
Current Assets = $62,000 + $35,000+ $47,000 + $138,000 = $282,000
Current liabilities = (accounts payable + accrued liabilities)
Current liabilities = $84,000+ $11,000 = $95,000
Quick Ratio = (Current assets - Inventory - Prepayments) / Current Liabilities
Quick Ratio = ($282,000 - $138,000 - $0) / $95,000
Quick ratio = $144,000 / $95,000 = 1.516, which rounds to 1.52.

Question #74: The net present value profiles of projects A and B are as follows.
Discount Net Present Value
Rate $(000)
(percent) Project A Project B
0 $2,220 $1,240
10 681 507
12 495 411
14 335 327
16 197 252
18 77 186
20 (26) 128
22 (115) 76
24 (193) 30
26 (260) (11)
28 (318) (47)

The approximate internal rates of return for Projects A and B respectively, are
Correct Answer: B. 19.5% and 25.5%.
A. 20.5% and 26.5%.
B. 19.5% and 25.5%.
C. 19.0% and 21.5%.
D. 0% and 0%.
Explanation:
A. At a discount rate of 20.50%, Project A has a Net Present Value of approximately
(48.25). At a discount rate of 26.5%. Project B has a Net Present Value of approximately
(20). These NPVs were calculated by interpolating from the data given. The internal rate
of return is the discount rate at which the NPV is equal to zero; so 20.5% cannot be the
internal rate of return for Project A and 26.5% cannot be the internal rate of return for
Project B. Look for the rates where the NPV of each project changes from a positive
number to a negative number. The rate at which the NPV is zero will be somewhere in
between those two rates.
B. The internal rate of return is the discount rate at which the NPV is equal to zero. We are
given several NPVs for both projects, each one calculated using a different discount rate.
As the discount rate increases, the NPVs decrease. For each project, the rate at which the
NPV changes from a positive number to a negative number will be approximately where
the NPV will be zero. The IRR will be a rate in between the last positive rate in the table
and the first negative rate. We can estimate that from the table. For Project A, at a
discount rate of 18%, its NPV is 77. At a discount rate of 20%, its NPV is (26).
Therefore, the discount rate at which its NPV will be zero must be somewhere in between
18% and 20% but closer to 20% because the negative amount (26) is a smaller amount
than the positive amount (77). For Project B, at a discount rate of 24%, its NPV is 30. At
a discount rate of 26%, its NPV is (11). Therefore, the discount rate at which its NPV will
be zero must be somewhere in between 24% and 26% but closer to 26% because the
negative amount (11) is a smaller amount than the positive amount (20). The only answer
choice that fulfills both of those ranges is 19.5% and 25.5%. With the answer choices
given, there is no need to do any further calculation to interpolate the exact IRR for each
project.
C. At a discount rate of 19.0%, Project A has a Net Present Value of approximately 25.5. At
a discount rate of 21.5%. Project B has a Net Present Value of approximately 89. These
NPVs were calculated by interpolating from the data given. The internal rate of return is
the discount rate at which the NPV is equal to zero; so 19.0% cannot be the internal rate
of return for Project A and 21.5% cannot be the internal rate of return for Project B. Look
for the rates where the NPV of each project changes from a positive number to a negative
number. The rate at which the NPV is zero will be somewhere in between those two
rates.
D. At a discount rate of 0% for both projects. Project A has a Net Present Value of $2,220
and Project B has a Net Present Value of $1,240. The internal rate of return is the
discount rate at which the NPV is equal to zero; so 0% cannot be the internal rate of
return for either project Look for the rates where the NPV of each project changes from a
positive number to a negative number. The rate at which the NPV is zero will be
somewhere in between those two rates.
Question #75: Following is an excerpt from Albion Corporation's balance sheet.
$30,000,00
Long-term debt (9% interest rate) 0
Preferred stock (100,000 shares. 12%
dividend) 10,000,000
Common stock (5,000,000 shares
outstanding) 60,000,000

Albion's bonds are currently trading at $1,083.34, reflecting a yield to maturity of 8%. The
preferred stock is trading at $125 per share. Common stock is selling at $16 per share, and
Albion's treasurer estimates that the firm's cost of equity is 17%. If Albion's effective income tax
rate is 40%, what is the firm's cost of capital?
Correct Answer: D. 13.1%
A. 12.6%
B. 13.9%
C. 14.1%
D. 13.1%
Explanation:
A. This answer results from using book values to calculate the weighted average cost of
capital. Market values, not book values, should be used.
B. This answer results from using the market interest rate of 8% for debt without adjusting it
for the tax deductibility of interest by multiplying it by (1 - the tax rate).
C. This answer results from using the market interest rate of 8% for debt without adjusting it
for the tax deductibility of interest by multiplying it by (1 - the tax rate) and from using
the nominal dividend rate of 12% for the preferred stock instead of a dividend rate
calculated using the actual dividends paid per year divided by the market value of the
preferred stock
D. The weighted average cost of capital is calculated using market values of the components
of the capital. Bonds are always issued in increments of $1,000 face value. So if a
company has $30,000,000 in bonds on its balance sheet, it has 30,000 bonds outstanding
($30,000,000+ 1,000). The market value of the company's bonds is 30,000 bonds x the
market price of $1,083.34, or $32,500,200. The market value of the preferred stock is
$125 x 100,000 shares, or $12,500,000. The market value of the common stock is $16 x
5,000,000 shares outstanding, or $80,000,000. Thus, the company's total capital for the
purpose of calculating its weighted average cost of capital is $32,500,200+ $12,500,000
+ $80,000,000, which equals $125,000,200. The cost of debt at market value is given as
8%. Therefore, we will use 8%, adjusted for the tax deductibility of interest as the cost of
debt. The cost of the preferred stock is 12% of $10,000,000, or $1,200,000. At a market
value of $12,500,000 (calculated above), the cost of the preferred stock is $1,200,000+
$12,500,000, or 0.096 (9.6%).
The cost of the common stock is given as 17%.
Therefore, the calculation of the company's weighted average cost of capital, based upon
the information given, is as follows:
Long-term debt: $32,500,200 ÷ $125,000,200 x 0.08 x (1- 0.40) =0.01248
Preferred stock: $12,500,000 ÷ $125,000,200 x 0.096 =0.00959
Common stock: $80,000,000 ÷ $125,000,200 x 0.17 =0.10879
Weighted average cost of capital 0.13086
The rounded cost of capital is 13.1%.

Question #76: The current market price of Action Pharmaceutical's common stock is $34. A 6-
month call option has been written on the stock. The option has an exercise price of $40 and a
market value of $4. A financial analyst estimates that, at the end of 6 months, the expected value
of the stock is $42.
What is the value just prior to expiration of the option if the stock closes at $42 at the end of 6
months?
Correct Answer: D. $2.00
A. $4.00
B. $6.00
C. $0
D. $2.00
Explanation:
A. $4 is the market value of the option at the beginning of the 6 months. The question is
about the value of the option at the end of the six months.
B. This answer is incorrect because at a price of the option of more than $2, the potential
shareholder would be better off buying the share in the market for $42 than paying $6 for
the option and then paying $40 for the share with the option. See the correct answer for a
complete explanation.
C. Since the market price is higher than the call price of the option, the option does have
some value. See the correct answer for a complete explanation.
D. Just prior to its expiration the option has no time value left. Its only value will be its
intrinsic value, which is the amount by which the call option's strike price is lower than
the market price of the stock. Since the share can be purchased with the option for $40
and the expected market price at the end of the option period will be $42, the value of the
option is the difference between these amounts, or $2. If the price of the option is more
than $2, a person would be better off buying the share for $42 than buying the option and
then paying $40 for the share with the option.
Question #77: Which one of the following statements concerning cash flow determination for
capital budgeting purposes is not correct?
Correct Answer: C. Book depreciation is relevant because it affects net income.
A. Net working capital changes should be included in cash flow forecasts.
B. Tax depreciation must be considered because it affects cash payments for taxes.
C. Book depreciation is relevant because it affects net income.
D. Sunk costs are not incremental flows and should not be included.
Explanation:
A. It is correct to say that net working capital changes should be included in cash flow
forecasts. Net working capital changes as a result of a capital budgeting project are cash
inflows and outflows for that project
B. It is correct to say that tax depreciation must be considered because it affects cash
payments for taxes. The tax depreciation reduces taxable income which reduces income
taxes due.
C. It is not correct to say that book depreciation is relevant because it affects net income.
Book depreciation is the depreciation expense recorded under generally accepted
accounting principles, and it does affect the net income reported on the financial
statements. However, the depreciation used in capital budgeting analyses is the type of
depreciation used for tax purposes, which is different from the type of depreciation used
for book purposes. The type of depreciation for tax purposes is relevant to a capital
budgeting analysis because it impacts the amount of income tax due, and the income tax
due is a relevant cash flow item.
D. It is correct to say that sunk costs are not incremental flows and should not be included.
The only cash flows that are relevant in a capital budgeting analysis are those that will
occur as a direct result of the project under consideration. Sunk costs by definition are
those which have already occurred and cannot be changed.

Question #78: Question: The percentage difference between the actual and the budgeted
breakeven point in units was that actual was
Xerbert Co.
Budget and Actual Income Statements
For the Year Ending December 31
(000s omitted)

Budget Actual
Xenox Xeon Total Xenon Xeon Total
Unit sales 150 100 250 130 130 260
Net dollar sales $900 $1,000 $1,900 $780 $1,235 $2,015
Variable expenses (450) (750) (1,200) (390) (975) (1,365)
Contribution
margin $450 $250 $700 $390 $260 $650
Fixed expenses:
Manufacturing $153 $140
Marketing 95 90
Other fixed expenses 200 190
Total fixed expenses $448 $420
Income before taxes $252 $230

Correct Answer: A. 5.00% above budget.


A. 5.00% above budget.
B. 6.67% below budget.
C. 6.67% above budget.
D. 5.00% below budget.
Explanation:
Answer (A) is correct
According to the budget, sales of 250 units would produce a contribution margin of $700, or
$2.80 per unit. Dividing the $448 of budgeted fixed costs by $2.80 gives a breakeven point of
160 units. The 260 actual units sold produced a contribution margin of $650, or $2.50 per unit.
Dividing the $420 of fixed costs by $2.50 gives a breakeven point of 168 units. Consequently,
the actual breakeven point is 5% (8 ÷ 160) above the budget.
Answer (B) is incorrect.
The percentage difference between the actual and the budgeted fixed costs is 6.67% [(420 - 448)
÷ 448].
Answer (C) is incorrect
The percentage difference between the actual and the budgeted fixed costs is 6.67% [(420 - 448)
÷ 448].
Answer (D) is incorrect.
The percentage difference between the actual and the budgeted breakeven point was 5.00%
above (not below) the budget.
Question #79: Which one of the following is a true statement regarding organizational ethics?
Correct Answer: D. Paying attention to "whistleblowers" plays a significant role in maintaining
an effective ethical atmosphere.
A. As long as officer and employee behavior meet the requirements of the law, the
organization can be considered to have a functioning system of ethical behavior.
B. A strong sense of ethics on the part of employees who are in the best position to
appropriate cash and other assets is the most vital part of a functioning system of ethical
behavior.
C. If an organization has a strong code of ethical conduct in place, the role of employee
training can be downplayed.
D. Paying attention to "whistleblowers" plays a significant role in maintaining an effective
ethical atmosphere.
Explanation:
Answer (A) is incorrect.
A sense of ethics requires an ability to distinguish between ethical and merely legal behavior.
"Values and Ethics: From Inception to Practice" states, in part. "Many individuals at the center of
corporate scandals (of the late 20th and early 21st Century] have professed the belief that they
were innocent of any wrongdoing, including Kenneth Lay of Enron or Conrad Black of
Hollinger. The problem is that these individuals did not define their behavior by what most of
society would see as 'reasonable but rather they followed their own particular code-in some
cases, limiting the definition of ethical behavior to require compliance with the law and nothing
more." (II. Introduction.)
Answer (B) is incorrect.
"Values and Ethics: From Inception to Practice states, in part, "Ethical behavior is not something
that applies to someone else-every single individual is responsible for behaving ethically.
Nowhere is this more important than the demonstration of ethical behavior that managers and
supervisors exhibit in the way they execute their day-to-day work..." This phenomenon is
referred to as the "tone at the top." (VI. Leadership by Example.)
Answer (C) is incorrect.
Employee training is important to maintaining an ethical organizational culture. "Values and
Ethics: From Inception to Practice" states, in part. "Every existing member of staff should
receive ongoing training, starting at the board level and cascading down throughout the
organization ... Ethics training for employees should focus on covering ethical concepts, the
organization's code, and compliance. To achieve this, training should include: ethical concepts
and thinking: What is 'behind' the issue of ethical action?; (and] the organization's code of ethics
and any supporting 'rules:" (VIII. Practical Application: Converting Intent into Operational
Reality.)
Answer (D) is correct.
"Values and Ethics: From Inception to Practice states, in part, "A whistleblowing framework
(e.g.. an ethics helpline) is an important component in maintaining an ethical organizational
culture. An effective feedback system includes having a confidential framework for employees
to report possible violations of the organization's code of ethics and to receive advice on the
ethical aspects of challenging decisions. Statistics show that a large number of occupational
fraud cases are detected through an employee "hotline" or other reporting method ..." (IX.
Measuring and Improving Ethical Compliance.)

Question #80: All of the following are forms of off-balance sheet financing except:
Correct Answer: A. Completing a horizontal merger.
A. Completing a horizontal merger.
B. Creating a special purpose entity.
C. Factoring accounts receivable.
D. Forming a joint venture.
Explanation:
The four common techniques used by companies to engage in off-balance sheet financing are;
factoring of receivables, creating a special purpose entity, operating leases and joint ventures.

Question #81: Paulson Incorporated acquired all of the common stock of Sampson Company on
January 1, 20x1 for $80,000 LCU, which was equal to the fair value of the company. Paulson
continued to operate Sampson as a subsidiary in the foreign country. On January 1, 20x1
Sampson borrowed $200,000 LCU and signed a 5-year note agreeing to pay 10% annual interest
beginning on January 1, 20x2. Sampson purchased a building for $280,000 LCU and estimated
its useful life as 20 years with no salvage value. The building will be depreciated using the
straight-line method. The subsidiary rents the building for $10,000 LCU per month and as of
December 31, 20x1, they received 11 payments for the year. During the year, $8,000 in
maintenance expenses were paid evenly throughout the year to maintain the building. Sampson
issued at $7,500 cash dividend to Paulson at the end of the year. The functional currency for the
subsidiary is the LCU. Exchange rates for 1 LCU were as follows:
January 1, 20x1 $1.90 December 31, 20x1 $2.00 Average for 20x1 $1.95 The accountant in the
local country prepared the following income statement, balance sheet, and statement of retained
earnings for the year in LCU and forwarded it to Paulson Incorporated.
Income Statement

LCU
Rent revenue $120,000
Interest expense 20,000
Depreciation expense 14,000
Maintenance expense 8,000
Net income $78,000

Statement of Retained Earnings

LCU
Retained earnings, 1/1 $-
Net income $78,000
Dividends paid $7,500
Retained earnings, 12/31 $70,500

Balance Sheet

LCU
Cash $94,500
Accounts receivable $10,000
Building $280,000
Accumulated $(14,000
depreciation )
Total assets $370,500
Interest payable $20,000
Note payable $200,000
Common stock $80,000
Retained earnings $70,500
Total liabilities and
equities $370,500

What is the impact on return on assets when Paulson translates Sampson's accounts into U.S.
dollars?
Correct Answer: D. Return on assets decreases from 21.1% to 20.5%.
A. Return on assets increases from 20.5% to 22.6%.
B. There is no effect: return on assets remains the same.
C. Return on assets increases from 20.5% to 21.1%.
D. Return on assets decreases from 21.1% to 20.5%.
Explanation:
Return on Assets in LCU = 21.1% (78,000 LCU / 370,500 LCU) and decreases to 20.5%
($152,100 / $741,000) when the financial statements are translated. Net Income changes from
78,000 LCU times 1.95 to U.S. $152,100 and assets change from 370,500 LCU times 2.0 to U.S.
$741,000.

Question #82: (Please refer the fact pattern below) Question: Assume the rental opportunity does
not exist and Richardson Motors could use the idle capacity to manufacture another product that
would contribute $104,000 per month. If Richardson chooses to manufacture the ten T305 units
in order to maintain quality control. Richardson's opportunity cost is
Richardson Motors uses 10 units of Part No. T305 each month in the production of large diesel
engines. The cost to manufacture one wit of T305 is presented as follows:

Direct materials $2,000


Materials handling (20% of direct materials
cost) 400
Direct labor 16,000
Manufacturing overhead (150% of direct labor) 24,000
Total manufacturing cost $42,000
Materials handling, which is not included in manufacturing overhead, represents the direct
variable costs of the receiving department that are applied to direct materials and purchased
components on the basis of their cost. Richardson's annual manufacturing overhead budget is
one-third variable and two-thirds fixed Simpson Castings, one of Richardson's reliable vendors,
has offered to supply T305 at a unit price of $30,000.
Correct Answer: C. $8.000
A. $68,000
B. $88,000
C. $8,000
D. $(96,000)
Explanation:
Answer (A) is incorrect.
This amount overlooks the $6,000 per unit of receiving costs for purchased components.
Answer (B) is incorrect.
This amount assumes only one-third of the overhead is fixed.
Answer (C) is correct.
The out-of-pocket cost of making the part equals the total manufacturing cost minus the fixed
overhead, or $26,400 [$42,000 - [(2 ÷ 3) x $24,000)). The cost of the component consists of the
$30,000 purchase price plus the $6,000 (20% of cost) of variable receiving costs, or a total of
$36.000. Thus, unit out-of pocket cost would increase by $9,600 if the components were
purchased. For 10 units, the additional cost of purchasing is $96,000. However, the net effect of
purchasing is a gain of $8,000 ($104,000 contribution from making another product - $96,000).
Opportunity cost is the benefit from the next best alternative use of the resources. Hence, the
company's opportunity cost of making the part is $8,000.
Answer (D) is incorrect.
This amount ignores the $104,000 income from alternative production.

Question #83: Consider the following transactions:


I. A firm receives cash on account.
II. A firm sells goods on account (cost of goods sold (COGS) is less than sales price).
III. A firm makes a payment on account.
IV. A firm purchases inventory on account.
If a firm has a current ratio greater than one, which of the transactions above would cause its
current ratio to increase?
Correct Answer: D.II and III.
A. I, II, III, and IV.
B. I, II, and III.
C. II, III, and IV.
D. II and III.
Explanation:
The current ratio is defined as current assets divided by current liabilities. Examine each
transaction for its effect on either current assets or current liabilities:
I. Debit to cash, credit to accounts receivable: the net result is no change in either
current assets or current liabilities. Therefore, there is no change to the current ratio.
II. Debit to accounts receivable, credit to sales for an amount greater than a debit to
COGS and a credit to inventory. The result will be an increase in current assets.
Therefore, there is an increase in the current ratio.
III. Debit to accounts payable, credit to cash. The result of this transaction is an equal
decrease in current assets and current liabilities. Given that the current ratio is greater
than 1, this transaction will result in an increase in the current ratio.
IV. Debit to inventory and credit to accounts payable. The result of this transaction is an
equal increase in current assets and current liabilities. Given that the current ratio is
greater than 1, this transaction will result in a decrease in the current ratio.
Question #84: The Financial Analysis Department of Stover Inc. has analyzed a proposed capital
investment and calculated the appropriate incremental cash flows as follows.

Year Cash
(100,000)
0 outflow
1 80,000 inflow
2 80,000 inflow
3 80,000 inflow
(100,000)
4 outflow

A net present value (NPV) of approximately 525.000 and an internal rate of return (IRR) of
minus 29% were calculated for the project and the project was submitted to the board of directors
for approval. Which one of the following statements is correct?
Correct Answer: B. The project has another IRR in addition to the minus 29% rate.
A. The IRR calculation must have contained an error.
B. The project has another IRR in addition to the minus 29% rate.
C. The board of directors should not approve the project.
D. In the NPV calculation, the project's cash flows are assumed to be reinvested at Stover's
cost of capital.
Explanation:
A. The IRR is the discount rate at which the NPV of an investment will be equal to O. This
is the discount rate at which the present value of the expected cash inflows from a project
equals the present value of the expected cash outflows. The error occurred in submitting
the negative IRR to the board. Whenever a project has a negative cash flow or flows in
any subsequent year(s) after Year 0, it can have more than one IRR, because more than
one discount rate will cause the project's NPV to be zero. The Financial Analysis
Department should have determined what the correct (positive) IRR was before
submitting anything to the board.
B. The IRR is the discount rate at which the NPV of an investment will be equal to O. This
is the discount rate at which the present value of the expected cash inflows from a project
equals the present value of the expected cash outflows. Whenever a project has a negative
cash flow or flows in any subsequent year(s) after Year 0, it can have more than one IRR,
because more than one discount rate will cause the project's NPV to be zero. The number
of IRRs will be equal to the number of sign changes in the cash flows. Furthermore, one
or more of the I RRs will frequently be negative. That is obviously what has occurred
here. The project has a negative cash flow in Year 4. Therefore, the project will have two
IRRs: one due to the normal sign change from negative in Year 0 to positive in Year 1,
and one due to the sign change from positive in Year 3 to negative in Year 4.
C. Whether or not the board of directors should approve the project depends upon a great
many factors. That is a judgment that cannot be made on the basis of the limited facts
given.
D. The IRR calculation assumes that the cash inflows from the project can be reinvested at
the Internal Rate of Return, not the company's cost of capital.

Question #85: When a company desires to increase the market value per share of common stock,
the company will
Correct Answer: A. Implement a reverse stock split.
A. Implement a reverse stock split.
B. Sell treasury stock
C. Sell preferred stock
D. Split the stock
Explanation:
A. In a reverse stock split the company reduces the number of shares outstanding. For
example, in a 1-for-2 reverse stock split, every two shares that are held by someone
become one share. This one share, however, has a value that is twice as high as an
individual share before the reverse stock split. Therefore, a reverse stock split will
increase the market value of a common share.
B. If the company sells treasury shares there are more shares outstanding to which the value
of the company needs to be divided. A sale of treasury shares will reduce the value of an
individual common share.
C. Selling preferred stock will have only an indirect effect on the market value of common
stock and it would decrease the common stocks market value, not increase it. Preferred
stock reduces the income available to common shareholders which in turn reduces basic
and diluted earnings per share. The reduction of earnings per share would reduce the
value of the common stock in the judgment of the market, so the market value of the
common stock could decline. It would not increase.
D. In a stock split each share is split into some number of more shares. For example, in a 2-
for-1 split, each share that is held by someone becomes two shares. However, the market
value of each of those shares is half what it had been before the split. A stock split
reduces the market value of each common share. It does not increase it.
Question #86: A company is analyzing the opportunity to expand into a new market. The
expansion would require an initial investment of $261,600. Cash flows for the new market
expansion are forecasted to be $120,000 for each of the next 3 years. The company has a cost of
capital of 8%. The discounted payback period for the new market expansion would be
Correct Answer: B. 2.5 years.
A. 2.0 years.
B. 2.5 years.
C. 2.8 years.
D. 2.2 years.
Explanation:
A. This is not the correct answer. Please see the correct answer for an explanation.
B. The Discounted Payback Method uses the present value of cash flows instead of
undiscounted cash flows to calculate the payback period. Each year's cash flow is
discounted using an appropriate interest rate, usually the company's cost of capital, and
then those discounted cash flows are used to calculate the payback period. The
discounted cash flows and cumulative cash flows of the project discounted for each year
individually using the company's cost of capital of 8%, areas follows:
Cumula
tive
Discoun
Undiscou Discou Discount ted
nted Cash nted ed Cash Cash
Flow Factor Flow Flow
Year $(261,600 =$(261,6 $(261,6
0 ) ×1.000 00) 00)
Year =111,12 (150,00
1 120,000 ×0.926 0 0)
Year =102,84 (47,640
2 120,000 ×0.857 0 )
Year
3 120,000 ×0.794 =95,280 47,640

The payback period is the number of the final year in which the projected cash flow is negative
(Year 2), plus a fraction consisting of:
-Numerator: The positive value of the negative cumulative cash flow amount from the final
negative year: ($47,640)
-Denominator: Cash flow for the following year: $95,280. Thus, the payback period is 2 >
(47,640/95,280), or 2.5 years. Actually, it is not be necessary to do the final calculation of the
payback period. Because the negative cumulative cash flow in Year 2 is exactly the same as the
positive cumulative cash flow in Year 3, it is easy to see that the point where cumulative cash
flow becomes zero is exactly half-way between Year 2 and Year 3.
C. This is not the correct answer. Please see the correct answer for an explanation.
D. This is the payback period using undiscounted cash flows. The Discounted Payback Method
uses the present value of cash flows instead of undiscounted cash flows to calculate the payback
period. Each year's cash flow is discounted using an appropriate interest rate, usually the
company's cost of capital, and then those discounted cash flows are used to calculate the payback
period.

Question #87: The controller of Omni Corp asked a financial analyst to calculate common size
financial statements for the past four years. The controller is most likely looking for which of the
following?
Correct Answer: C. Trends in expenses as a percentage of sales.
A. How the company is earning its profits.
B. The growth rate for sales.
C. Trends in expenses as a percentage of sales.
D. How efficiently the company is using assets.
Explanation:
Common size financial statements look at each element in the statement as a percentage of
another total amount. Common size income statements show expenses as percent of sales,
while common size balance sheets show assets, liabilities, and equities as a percent of total
assets. A series of common size income statements will show trends in expenses as a
percentage of sales.

Question #88: When a company offers credit terms of 2/10, net 30, the annual interest cost,
based on a 360-day year, is
Correct Answer: B. 36.7%.
A. 35.3%.
B. 36.7%.
C. 24.0%.
D. 36.0%.
Explanation:
A. This is (360 / 20) x (2 / 102) = 0.3529 or 35.3%.
The correct formula is:
360 Discount %
×
Total period for payment −Period for discount payment 100 %−Discount %
B. This question is solved using the following formula:
360 Discount %
×
Total period for payment −Period for discount payment 100 %−Discount %

Inputting the information from the question into the formula, we get 36.7%, as follows:
360 Discount %
×
Total period for payment −Period for discount payment 100 %−Discount %
Inputting the information from mthe question into the formula, we get 36.7%, as follows:
3600.02
30 – 101.00 – 0.02
=18× 0.0204 = 0.367 or 36.7%
C. This is (360/ 30) x (2 / 100) = 0.24 or 24%.
The correct formula is:
360 Discount %
×
Total period for payment −Period for discount payment 100 %−Discount %
D. This is (360 / 20) × (2 / 100) = 0.36 or 36%.
The correct formula is:
360 Discount %
×
Total period for payment −Period for discount payment 100 %−Discount %

Question #89: Risk assessment is a function of Managements attitude toward risk. A risk averse
management team typically:
Correct Answer: B. Is more likely to spend money to mitigate risk.
A. Does not prioritize mitigation of risks.
B. Is more likely to spend money to mitigate risk.
C. Is not willing to spend time to mitigate risk
D. Is not willing to spend money to mitigate risk
Explanation:
Risk assessment is a function of the management attitude toward risk as well as the estimate
of potential risk For example, the more risk averse a management team is, the more they will
be willing to spend on mitigating the risk Likewise, the greater the potential risk is perceived
to be, the more time and money management will be willing to spend to minimize or mitigate
the risk
Question #90: Which of the following ratios does not measure a firm's ability to use its assets
efficiently?
Correct Answer: A. Current ratio.
A. Current ratio.
B. Days' sales in inventory.
C. Asset turnover rate.
D. Average collection period.
Explanation:
The current ratio measures liquidity while all the other ratios are measuring the firm's ability to
use its assets efficiently.

Question #91: A company plans to sell 12,000 units of product XT and 8,000 units of
product RP. The Company has a capacity of 12,000 productive machine hours. The unit cost
structure and machine hours required for each product are as follows:

Unit Costs XT RP
Materials $37 $24
Direct labor 12 13
Variable overhead 6 3
Fixed overhead 37 38
Machine Hours required 1.0 1.5

The company can purchase 12,000 units of XT at 560 and/or 8,000 units of RP at $45.
Based on the above, which one of the following actions should be recommended to the
company's management?
Correct Answer: A. Produce XT internally end purchase RP.
A. Produce XT internally end purchase RP
B. Produce RP internally and purchase XT.
C. Purchase bath XT and RP.
D. Produce both XT and RP
Explanation:
Answer (A) is correct
Relevant unit cost to product XT is $55 ($37 + $12 + $6) and cost for RP is $40 ($24 +
$13 + $3). Only enough machine hours are available to produce one product or the other
(XT: 12,000 × 1.0 hour per unit = 12,000 hours, RP: 8,000 units x 1.5 hours per unit =
12,000 hours). Cost to produce XT in-house would be $660,000 (12,000 x $55) and cost
to produce RP would be $320,000 (8,000 x $40). Cost to purchase XT from outside
would be $720,000 (12,000 x $60) end cost to purchase RP be $360,000 (8,000 × $45).
The possible combination with the lowest cost is to produce XT and purchase RP
($660,000 + $360,000 = $1,020,000)
Answer (B) is incorrect.
Producing RP and purchasing XT would result in a total cost of $1,040,000.
Answer (C) is incorrect.
Purchasing bath products would result in e total cost of $1,080,000.
Answer (D) is incorrect
Only enough machine hours are available to produce one product or the other.

Question #92: On July 14, an investor goes short on a put option for 100 shares of OSC, Inc.
common stock with a strike price of $9.00, expiring on August 16. at an option premium of
$1.50 per share. The market price of OSC on July 14 is $8.00. On August 16, the market
price of OSC is S 11.00. How much has the investor gained or lost on the option
transaction? Disregard any brokerage commissions involved.
Correct Answer: D. Gain of $150.
A. Gain of $200.
B. Loss of $50.
C. Loss of $150.
D. Gain of $150.
Explanation:
A. This is the market price of 100 shares on the option's expiration date ($1,100) minus
the strike price multiplied by 100 ($900).
B. This is the lass that would be sustained by the short party to a put transaction with
the same characteristics.
C. This is the loss that would be sustained by the long party to an option transection
with the same characteristics.
D. The short party to an option is the one who sells the option end who must comply if
the buyer of the option chooses to exercise it Here, the short party sold put option to
sell the underlying stock at The buyer of the option has the right to sell 100 shares of
OSC stock to the seller of the option at e price of $900. However, the market price of
the stock on the expiration date is greater than $9.00, and the buyer at the option will
not sell stock for $9.00 that he could sell on the open market for $1,100. So the buyer
of the option lets the option expire, end the seller at the option gets to keep the
premium received far selling the option at $1.50 per share. Multiplied by 100 shares,
the option seller's gain is $150.

Question #93: Synergy Inc. produces a component that is popular in many refrigeration
systems. Data on three of the five different models of this component are as follows.

A B C
Volume needed 5,000 6,000 3,000
Manufacturing costs
Variable Direct costs $10 $24 $20
Variable overhead 5 10 15
Fixed overhead 11 20 17
Total Manufacturing
Costs $26 $54 $52
Cost if purchased $21 $42 $39
Synergy applies variable overhead machine hours at the rate of $2.50 per hour. Models A
and B are manufactured in the Freezer Department, which has a capacity of 28,000 machine
processing hours. Which one of the following options should be recommended to Synergy's
management?
Correct Answer: The Freezer Department's manufacturing plan should include 5,000 units
of Model A and 4,500 units of Model B.
A. Purchase all three products in the quantities required.
B. Manufacture all three products in the quantities required.
C. The Freezer Department's manufacturing plan should include 5,000 units of Model A
and 4,500 units of Model B.
D. The Freezer Department's manufacturing plan should include 2,000 units of Model A
and 6,000 units of Model B.
Explanation:
Answer (A) is incorrect.
There is idle capacity in the freezer department that can be profitably used.
Answer (B) is incorrect.
This would exceed the department's production capacity.
Answer (C) is correct.
The variable costs of producing all of the units of A end 4,500 units of a would be
calculated as follows:
Variable costs of A: 5,000 × $15 = $ 75,000
Variable costs at B: 4,500 x $34 = 153,000
Purchase B: 1,500 × $42 = 63,000
Purchase C: 3,000 × $39 = 117,000
Total $408,000
If the company produced 2,000 units of Model A and 6,000 units of Model B, the
calculations would be:
Variable costs of A: 2,000 x $15 = $ 30,000
Variable costs of B: 6,000 × $34 = 204,000
Purchase A: 3,000 x $21 = 63,000
Purchase C: 3,000 × $39 = 117,000
Total $414,000
Therefore, the first option is better than the second option.
Answer (D) is incorrect.
It is more costly than alternative C.

Question #94: A company sells its single product for $30 per unit. The contribution margin
ratio is 45%, and the company has fixed costs of $10,000 per month. If 3,000 units are sold
in the current month, the company's income would be
Correct Answer: A. $30,500
A. $30,500
B. $49,500
C. $40,500
D. $90,000
Explanation:
Answer (A) is correct
The company's contribution margin ratio reveals that of the sales price of each product is
contribution margin ($30 x 45% = $13.50). Thus, sales at 3,000 units will produce total
revenues at $40,500 (3,000 x $13.50). Since fixed costs are $10,000, income for the
month is $30,500 ($40,500 - $10,000).
Answer (B) is incorrect.
Variable cost for the month is $49,500.
Answer (C) is incorrect.
Contribution margin for the month is $40,300
Answer (D) is incorrect
Revenue for the month is $90,000

Question #95: Kell Inc. is analyzing an investment for a new product expected to have
annual sales of 100,000 units for the next 5 years and then be discontinued. New equipment
will be purchased for $1,200,000 and cost $300,000 to install. The equipment will be
depreciated one straight-line basis over 5 years for financial reporting purposes and 3 years
for tax purposes. At the end of the fifth year, it will cost $100,000 to remove the equipment,
which can be sold for $300,000. Additional working capital of $400,000 will be required
immediately and needed for the life of the product. The product will sell for $80, with direct
labor and material costs of $65 per unit. Annual indirect costs will increase by $500,000.
Kell's effective tax rate is 40%.
In a capital budgeting analysis, what is the cash outflow at time O (initial investment) that
Kell should use to compute the net present value?
Correct Answer: B. $1,900,000.
A. $1,300,000.
B. $1,900,000.
C. $1,500,000.
D. $1,700,000.
Explanation:
A. This answer results from two errors:
(1) The increase in working capital required is not included.
(2) The salvage value minus the cost to remove at the equipment's end of life is included
as part of the calculation of the net cash outflow for the initial investment at Year 0. The
cash flows from the sale at the equipment at the end of its life are cash flows taking
place in the final year of the project, not at the beginning of the project
B. Purchase of equipment of plus installation cost of ($300,000) plus increase in
working capital of ($400,000) = ($1,900,000) cash out flows.
C. This answer omits the increase in working capital. The required increase in working
capital is a cash outflow at the beginning of the project.
D. This answer results from including the salvage value minus the cost to remove at the
equipment's end of life as part of the calculation of the net cash outfox.' for the initial
investment at Year 0. The cash flows from the sale of the equipment at the end of its life
ere cash flows taking place in the final year of the project, not at the beginning at the
project.

Question #96: Which one of the following pricing methods takes into consideration a
product's entire life cycle?
Correct Answer: A. Target pricing.
A. Target pricing.
B. Transfer pricing.
C. Market-based pricing.
D. Cost-based pricing.
Explanation:
Answer (A) is correct
A target price is the expected market price for a product or service, given the company’s
knowledge of its consumers' perceptions of value and competitors' responses.
Subtracting the unit target operating income determines the long-term unit target cost.
Relevant costs are future value-chain costs, whether variable or fixed.
Answer (B) is incorrect.
Transfer pricing does not take into consideration a product's entire life cycle.
Answer (C) is incorrect.
Market-based pricing does not take into consideration e product's entire life cycle.
Answer (D) is incorrect.
Cost-based pricing does not take into consideration e product's entire life cycle.
Question #97: Question: Assuming sufficient demand exists. N-Air could sell all the
products at the prices previously mentioned at either the split-off point or after further
processing. To maximize its profits, N-Air Corporation should
[Fact Pattern] N-Air Corporation uses a joint process to produce three products: A, B and C,
all derived from one input. The company can sell these products at point of split-off (end of
the Joint process) or process them further .The joint production Costs during October were
$10,000. N-Air allocates joint costs to the products in proportion to the relative physical
volume of output. Additional information is presented in the opposite column.
Produc Units Unit sales Price at Split- Unit Sales Unit Additional
t Produced off Price Cost
A 1,000 $4.00 $5.00 $.75
B 2,000 2.25 4.00 1.20
C 1,500 3.00 3.75 .90

Correct Answer: C. Sell product C at split-off and perform additional processing on


products A and B.
A. Sell product A at split-off and perform additional processing on products B and C.
B. Sell product B at split-off and perform additional processing on products C and A.
C. Sell product C at split-off and perform additional processing on products A and B.
D. Sell products A, B, and C at split-off
Explanation:
Answer (A) is incorrect.
N-Air should further process products A and B and sell product C at split-off
Answer (B) is incorrect.
N-Air should further process products A and B and sell product C at split-off
Answer (C) is correct.
To maximize profits, it must be determined whether each product's incremental revenues
will exceed its incremental costs. Joint costs are irrelevant because they are sunk costs.

A B C
Unit sales price if processed
further $5.00 $4.00 $3.75
Minus: unit sales price at split-off (4.00) (2.25) (3.00)
Incremental revenue per unit $1.00 $1.75 $0.75
Minus: incremental unit cost (.75) (1.20) (.90)
Excess unit revenue over unit cost $0.25 $0.55 $(0.15)
It is most profitable for N-Air to process products A and B farther and to 3 ell product C
at the split-off point.
Answer (D) is incorrect
N-Air should further process products A and B end sell product C at split-off.

Question #98: Which one of the following is least likely to be a reason why U.S.
multinational corporations utilize the foreign exchange market?
Correct Answer: D. To improve the return an it-vestments of a foreign subsidiary.
A. To offset accounts payable transection exposure to foreign firms.
B. To counter some of the currency risk at dividend payments from foreign subsidiaries
to the U.S. parent.
C. To hedge the currency risk at accounts receivable transactions in foreign currencies.
D. To impure the return on investments of a foreign subsidiary.
Explanation:
A. U.S. multinational companies use the foreign exchange market for many reasons.
One of those reasons is so that they can hedge their payables that are due in e foreign
currency. By using foreign currency markets the company can reduce, or eliminate,
the chance of great lasses due to the fluctuation of the exchange rates
B. U.S. multinational companies use the foreign exchange market for many reasons.
One of those reasons is to reduce the risk of devaluation of dividend payments as a
result of fluctuations of the exchange rate.
C. U.S. multinational companies use the foreign exchange market for many reasons.
One of those reasons is so that they can hedge their receivables that are going to be
collected in a foreign currency. By using foreign currency markets the company can
reduce, or eliminate, the chance of great losses due to the fluctuation of the exchange
rates
D. Though U.S. multinationals use the foreign currency market for a number of reasons,
this is not one of the reasons. The foreign currency markets will not enable the
company to improve the return on the investment of e foreign subsidiary. Assuming
that the foreign subsidiary's return on investments would involve cash inflows and
outflows in the foreign subsidiary's currency only, those cash flow would not be
subject to exchange rate risk.

Question #99: All of the following are valid reasons for expansion of international business
by U.S. multinational corporations, except to:
Correct Answer: B. Protect their domestic market from competition from foreign
manufacturers.
A. Secure new sources for raw materials.
B. Protect their domestic market from competition from foreign manufacturers.
C. Minimize their casts of production.
D. Find additional areas where their products can be successfully marketed.
Explanation:
Protecting the domestic market from foreign competition is not a valid reason for
expansion overseas; all the other options are valid.

Question #100: Jackson Corporation uses net present value techniques in evaluating its
capital investment projects. The company is considering a new equipment acquisition that
will cost $100,000, fully installed, and have a zero salvage value at the end of its five-year
productive life. Jackson will depreciate the equipment on a straight-line basis for both
financial and tax purposes. Jackson estimates $70,000 in annual recurring operating cash
income end $20,000 in annual recurring operating cash expenses. Jackson's cost of capital is
12% and its effective income tax rate is 40%. What is the net present value of this
investment on an after-tax basis?
Correct Answer: B. $36,990
A. $80,250.
B. $36,990.
C. $28,840.
D. $8,130.
Explanation:
A. This answer uses only the before-tax operating cash flow to calculate the net present
value. This ignores bath the depreciation tax shield and tax on the operating cash
flow.
B. Annual after-tax cash flows will be [($70,000-$20,000) × (1 - 0.40)], which equals
$30,000. The annual depreciation tax shield is ($100,000 ÷ 5) × 0.40, which is
$8,000. Therefore, the total annual after-tax cash flow is $38,000. The present value
of these net inflows for a 5-year period is $136,990 ($38,000x 3.605 present value of
an ordinary annuity for 5 years at 12%), and the NPV of the investment is $36,990
($136,990 - $100,000 initial investment).
C. $23,840 is the present value of the depreciation tax savings.
D. $8,150 ignores the depreciation tax savings.
MOCK TEST 7

Question #1: Green Company produces Product A and sells it for $18.00. The following cost
data apply:

Type of cost Per unit


Direct materials $4.50
Direct labor 6.45
Variable overhead 1.35
Fixed overhead 1.50
Variable selling expense 1.10
Fixed selling expense 2.20
$17.10

Green has thought of marketing a new Product B with the same cost structure as Product A
except that the price will be $15.60 Green Company currently has the plant capacity necessary
for this expansion Because of the cost structure. Green Company will find the production and
sale of Product B in the short run to be
Correct Answer: B. profitable to produce and sell Product Bin the short run at the price of
$15.60.
A. Not profitable unless the price can be raised to $17.10.
B. Profitable to produce and sell Product B in the short run at the price of $15.60.
C. Not profitable at any price.
D. Not profitable at $15.60 because the fixed selling expense and fixed manufacturing
overhead will not be covered by the price.
Explanation:
A. It would be profitable in the short-term to produce and sell Product B at a price greater
than its variable costs $13.40.
B. Assuming the company has excess capacity the company will be better off if it is able to
sell Product B at any price above its incremental costs of $13.40 ($4.50 DM + $6.45 DL
+ $1.35 variable overhead + $1.10 variable selling cost). At the price of $15.60 per unit.
Product B would contribute $2.20 per unit toward profit.
C. As long as the price for Product B is greater than its variable costs it would be profitable
in the short-term to produce and sell Product B.
D. Fixed selling and fixed manufacturing overhead do not need to be covered by the price of
Product B because they are already being covered by Product A.

Question #2: Management accountants are frequently asked to analyze various decision
situations, including the following:
I. The cost of a special device that is necessary if a special order is accepted.
II. The cost proposed annually for the plant service for the grounds at corporate
headquarters.
III. Joint production costs incurred, to be considered in a sell-at-split versus a process-
further decision.
IV. The costs associated with alternative uses of plant space, to be considered in a
make/buy decision.
V. The cost of obsolete inventory acquired several years ago, to be considered in a keep-
versus-disposal decision.
The cost described in situation II is a
Correct Answer: A. discretionary cost.
A. Discretionary cost.
B. Prime cost.
C. Imputed cost.
D. Relevant cost.
Explanation:
A. Discretionary costs are costs that can be deferred to future periods without creating a
significant impact in the current period. They have no relationship to input and output.
This is best described in situation II since the annual cost of service for the grounds at
corporate headquarters would have no effect on plant production or sales volume.
B. Prime costs include direct labor, and direct materials. This is not described in situation II.
C. An imputed cost is an opportunity cost. It is a benefit that is given up as a result of using
the company's resources elsewhere. It is a cost that is not stated and must be calculated in
some way. Imputed costs are those costs that are not paid in cash, but are implied. The
best example of an imputed cost is the interest charge on equity capital.
D. Relevant costs are those costs that will differ among the decision options. Situation II
would not be considered a relevant cost.

Question #3: Which one of the following capital budgeting techniques would result in the same
project selection as the net present value method?
Correct Answer: C. Profitability index.
A. Discounted payback.
B. Internal rate of return.
C. Profitability index.
D. Accounting rate of return.
Explanation:
Answer (A) is incorrect.
Discounted payback will not necessarily yield the same decision as net present value.
Answer (B) is incorrect.
The internal rate of return will not necessarily yield the same decision as net present value.
Answer (C) is correct.
The net present value of a capital project is derived by subtracting the discounted cash outflows
from the discounted cash inflows. The profitability index is the ratio of the same two numbers
(inflows to outflows). Thus, they will yield the same decision (if the net present value is positive,
the profitability index will be > 1).
Answer (D) is incorrect.
The accounting rate of return will not necessarily yield the same decision as net present value.

Question #4: Profits that are lost by moving an input from one use to another are referred to as
Correct Answer: C. Opportunity costs.
A. Replacement costs.
B. Cannibalization charges.
C. Opportunity costs.
D. Out-of-pocket costs.
Explanation:
A. Replacement costs are explicit future costs required to replace something used today.
Profits lost by moving an input from one use to another are implicit costs.
B. Cannibalization charges are related to competing products produced by the same
company. When one product °cannibalizes" another product, it takes sales away from the
other product. Usually the product that cannibalizes the sales is a new product that takes
sales away from an existing product produced by the same company. However, this
question relates to profits sacrificed by not producing a product, not profits lost because
sales are taken away by another, competing product produced by the same company.
C. An opportunity cost is the contribution to income that is lost by not using a limited
resource in its best alternative use. Profits lost by moving an input from one use to
another is an example of an opportunity cost.
D. Out-of-pocket costs are actual, explicit costs that are paid directly. Profits lost by moving
an input from one use to another are implicit costs because there is no real expense paid
out. The cost is the loss of an opportunity to earn a profit.
Question #5: Which of the following is not an aspect of the Foreign Corrupt Practices Act of
1977?
Correct Answer: C. It requires the establishment of independent audit committees.
A. It subjects management to fines and imprisonment.
B. It prohibits bribes to foreign officials.
C. It requires the establishment of independent audit committees.
D. It requires an internal control system to be developed and maintained.
Explanation:
Answer (A) is incorrect.
This is a provision of the Act.
Answer (B) is incorrect.
This is a provision of the Act.
Answer (C) is correct.
The Foreign Corrupt Practices Act of 1977 prohibits bribes to foreign officials and requires
firms to have adequate systems of internal control. Violation of the act subjects individual
managers to fines and/or imprisonment. The Act does not specifically require the
establishment of audit committees, but many firms have established audit committees as one
means of dealing with the internal control provisions of the act.
Answer (D) is incorrect.
This is a provision of the Act.

Question #6: Peggy Monahan, controller, has gathered the following information regarding
Lampasso Company.
Beginning of the End of the
year year
Inventory $ 6,400 $7,600
Accounts
receivable 2,140 3,060
Accounts payable 3,320 3,680

Total sales for the year were $85,900, of which $62,400 were credit sales. The cost of goods
sold (COGS) was $24,500.
Lampasso's inventory turnover ratio for the year was:
Correct Answer: D.3.5 times.
A. 8.2 times.
B. 3.2 times.
C. 8.9 times.
D. 3.5 times.
Explanation:
The inventory turnover per year is calculated as:
Inventory turnover per year = (COGS for year) / (average inventory balance for the year)
Average inventory balance for the year = (beginning balance + ending balance) /2
Average inventory balance for the year = ($6,400 + $7,600) / 2 = $14,000 / 2 = $7,000
Turnover per year = $24.500 / $7,000 = 3.5 times.

Question #7: All of the following are affected when merchandise is purchased on credit
except:
Correct Answer: C. Net working capital.
A. Current ratio.
B. Total current assets.
C. Net working capital.
D. Total current liabilities.
Explanation:
When merchandise is purchased on credit, total current assets increase and total current
liabilities increase by the same amount ... therefore, the net working capital (current assets -
current liabilities remains the same when merchandise is purchased on credit. The current
ratio, however, will change.

Question #8: Consider the statements below regarding accounting treatments for goodwill
under I FRSs. Which statement describes the correct accounting treatment for goodwill under
IFRSs?
Correct Answer: A. IFRSs tests goodwill for impairment but goodwill is not amortized.
A. IFRSs tests goodwill for impairment but goodwill is not amortized.
B. IFRSs allows goodwill to be amortized for a period not to exceed 20 years.
C. IFRSs allows goodwill to be amortized for a period not to exceed 40 years.
D. IFRSs does not allow the amortization of goodwill.
Explanation:
Like U.S. GAAP goodwill is never amortized but it should be tested annually for
impairment.

Question #9: A business entity that is owned, operated, and controlled by a small group of
investors with a specific business purpose, common goal, and is created specifically to keep
the liabilities associated with a specific project off the parent company's books is commonly
called a:
Correct Answer: B. Special purpose entity.
A. Joint venture.
B. Special purpose entity.
C. Horizontal merger.
D. Conglomerate.
Explanation:
A Special purpose entity is an entity created with a special sometimes undisclosed business
purpose where the entity is often times created specifically to keep the liabilities associated
with a specific project off the parent company's books.

Question #10: Fact Pattern: A company is evaluating the possible introduction of a new
version of an existing product that will have a 2-year life cycle. At the end of 2 years, this
version will be obsolete, with no additional cash flows or salvage value. The initial and sole
outlay for the modified product is $6 million, and the company's desired rate of return is
10%. Following are the potential cash flows (assumed to occur at the end of each year) and
their probabilities if the product is marketed:
The following interest factors for the present value of $1 at 10% are relevant:
Period 1.909
2.826
Question: The projects net present value is

Correct Answer: A. $878,050


A. $878,050
B. $3,242,050
C. $3,636,000
D. $6,000,000
Explanation:
Answer (A) is correct
The expected value of the cash flows at the end of Year 1 is $4 million [(.3 x $2 million) +
(.4 x $4 million) + (.3 x $6 million)], and the present value of this amount is $3,636,000
(.909 x $4 million). The expected value of the cash flows at the end of Year 2 is $3,925,000
[(.3 x .5 x $0) + (.3 x .5 x $4 million) + (.4 x .25 x $6.4 million) + (.4 x .75 x $3.2 million) +
(.3 x .4 x $6.875 million) + (.3 x .6 x $5 million)], and the present value of this amount is
$3,242,050 (.826 x $3,925,000). Hence, the NPV is $878,050 [($3,636,000 + $3,242,050) -
$6 million initial outlay].
Answer (B) is incorrect.
The amount of $3,242,050 equals the expected present value of the Year 2 cash flows.
Answer (C) is incorrect.
The amount of $3,636,000 equals the expected present value of the Year 1 cash flows.
Answer (D) is incorrect.
The amount of $6,000,000 equals the initial outlay.

Question #11: A company produces ready-to-bake pie crusts. In deciding whether to process
this product further into complete ready-to-bake pies by adding filling, relevant dollar
amounts to consider would include all of the following except the
Correct Answer: C. Cost to manufacture the crusts.
A. Selling price of the crusts.
B. Selling price of the complete pies.
C. Cost to manufacture the crusts.
D. Cost to add the filling.
Explanation:
A. The difference between the additional revenue to be received (by processing the crust
further into ready-to-bake pies) and the additional cost (to add the filling) is relevant to
the decision as to whether to process the product further. Therefore, the selling price of
the crusts is relevant to the decision because it is needed to determine the additional
revenue to be received.
B. The difference between the additional revenue to be received (by processing the crust
further into ready-to-bake pies) and the additional cost (to add the filling) is relevant to
the decision as to whether to process the product further. Therefore, the selling price of
the complete pies is relevant to the decision because it is needed to determine the
additional revenue to be received.
C. The difference between the additional revenue to be received (by processing the crust
further into ready-to-bake pies) and the additional cost (to add the filling) is relevant to
the decision as to whether to process the product further into complete ready-to-bake pies
by adding filling. The cost to manufacture the crusts is a sunk cost. It cannot be changed
by the decision whether to process the crusts further, so it is not relevant to the decision.
D. The difference between the additional revenue to be received (by processing the crust
further into ready-to-bake pies) and the additional cost (to add the filling) is relevant to
the decision as to whether to process the product further. Therefore, the cost to add the
filling is relevant to the decision because it is needed to determine the difference between
the additional revenue to be received and the additional cost.

Question #12: If inventories are expected to change, the type of costing that provides the best
information for breakeven analysis is
Correct Answer: C. variable (direct) costing.
A. Joint costing.
B. Absorption (full) costing.
C. Variable (direct) costing.
D. Job order costing.
Explanation:
A. To use breakeven analysis, variable costs need to be segregated from fixed costs.
Variable costs are not segregated from fixed costs in joint costing.
B. To use breakeven analysis, variable costs need to be segregated from fixed costs.
Variable costs are not segregated from fixed costs in absorption costing.
C. Variable costing provides the best information for breakeven analysis whether inventories
are expected to change or not, because variable costs are segregated from fixed costs. To
use the formula for the breakeven point in CVP analysis, fixed costs need to be separated
from variable costs.
D. To use breakeven analysis, variable costs need to be segregated from fixed costs.
Variable costs are not segregated from fixed costs in job order costing.

Question #13: A company produces and sells 2.000 units of finished goods and incurs
$60.000 of fixed costs annually. The contribution margin is $60 per unit. and variable cost is
$40 per unit. If the company expects sales quantities to increase by 10% next year. the
operating profit will be
Correct Answer: A. $72,000.
A. $72,000.
B. $120,000.
C. $60,000.
D. $132,000.
Explanation:
A. If the company expects sales quantities to increase by 10% next year, the quantity sold
will become 2,200 units (2,000 units x 1.10). The contribution margin per unit is $60, so
the contribution margin will be 2,200 x $60, or $132.000. Fixed costs will remain the
same, at $60,000, so operating income will be $132,000 - $60,000, or $72,000.
B. $120,000 is the contribution margin at the current sales volume (2,000 units x $60
contribution margin per unit = $120,000). However, this answer does not include any
increase in the contribution margin to reflect the expected 10% increase in sales volume,
nor does it include any deduction for fixed costs.
C. $60,000 is the current operating income at the current sales volume. It does not include
any adjustment for the expected 10% increase in sales volume.
D. $132,000 is the contribution margin if sales increase by 10% next year. Since the
contribution margin per unit is 460, the contribution margin at a sales level of 2,200 units
(2,000 units x 1.10) will be 2,200 x $60, or $132,000. However, this answer does not
include any deduction for fixed costs.

Question #14: When reviewing a credit application, the credit manager should be most
concerned with the applicant's:
Correct Answer: C. working capital and current ratio.
A. Price-earnings ratio and current ratio.
B. Working capital and return on equity.
C. Working capital and current ratio.
D. Profit margin and return on assets.
Explanation:
Liquidity measures, such as net working capital and the current ratio, help determine ability
to pay expenses on a timely basis. Therefore, the credit manager should be most concerned
with these measures in comparison to the others listed in the problem. Profit margin, price-
earnings ratio, and return on equity are profitability measures.

Question #15: A financial analyst has obtained the following data from Kryton Industries'
financial statements.

Cash $200,000
Marketable securities 100,000
Accounts receivable, net 300,000
Inventories, net 480,000
Prepaid expenses 120,000
1,200,00
Total current assets 0
Accounts payable 250,000
Income taxes 50,000
Accrued liabilities 100,000
Current portion of long term
debt 200,000
Total current liabilities $600,000

In order to determine Kryton's ability to pay current obligation, the financial analyst would
calculate Kryton's cash ratio as:
Correct Answer: A. 0.50.
A. 0.50.
B. 1.20.
C. 0.80.
D. 1.00
Explanation:
Cash ratio = (Cash + Cash Equivalents p Marketable securities) / Current liabilities
= ($200,000 + 0 + $100,000) / $600,000
= 0.50.

Question #16: Daily sales and cost data for Crawford Industries are shown below.

Sales
Total
Units $ costs
20 $2,000 $1,200
21 2,090 1,250
22 2,170 1,290
23 2,240 1,330
24 2,300 1,380
25 2,350 1,440

The marginal cost of the 23rd unit is


Correct Answer: A. $40.00.
A. $40.00.
B. $30.00.
C. $57.83.
D. $50.00.
Explanation:
A. Marginal cost is the addition to total cost by increasing production by one unit. The total
cost of 23 units is $1,330, and the total cost of 22 units is $1,290. $1.330 - $1.290 = $40.
This is the marginal cost, or the addition to total cost by increasing production from 22
units to 23 units.
B. This is not the correct answer. Please see the correct answer for an explanation. We have
been unable to determine how to calculate this incorrect answer choice. If you have
calculated it, please let us know how you did it so we can create a full explanation of why
this answer choice is incorrect. Please send us an email at admin@cmapass.com. Include
the Topic Name. Question Details or Screenshot and the actual incorrect answer choice --
not its letter, because that can change with every study session created. Thank you in
advance for helping us to make your CMA PASS study materials better.
C. This is the total cost of producing 23 units ($1,330) divided by 23 units. Thus it is the
average total cost per unit to produce 23 units. It is not the marginal cost of the 23rd unit.
Marginal cost is the addition to total cost by increasing production by one unit.
D. Marginal cost is the addition to total cost by increasing production by one unit. This is the
difference between total costs at the production level of 24 units and the total costs at the
production level of 23 units. Thus it is the marginal cost of the 24th unit. not the marginal
cost of the 23rd unit.

Question #17: Which ethical standard is most clearly violated if an IMA member knows of a
problem that could mislead users but does nothing about it?
Correct Answer: C. Credibility.
A. Competence.
B. Legality.
C. Credibility.
D. Confidentiality.
Explanation:
Answer (A) is incorrect.
The competence standard pertains to the IMA member's responsibility to maintain his or her
professional skills and knowledge. It also pertains to the performance of activities in a
professional manner.
Answer (B) is incorrect.
Legality is not addressed in IMAs Statement of Ethical Professional Practice.
Answer (C) is correct.
Credibility is the fourth part of IMAs Statement of Ethical Professional Practice. It requires that
information be communicated "fairly and objectively.' and that all information that could
reasonably influence users be disclosed.
Answer (D) is incorrect.
The confidentiality standard concerns the IMA member's responsibility not to disclose or use the
firm's confidential information.

Question #18: Which one of the following provide a spontaneous source of financing for a firm?
Correct Answer: B. Accounts payable.
A. Debentures.
B. Accounts payable.
C. Mortgage bonds.
D. Preferred stock.
Explanation:
Accounts payable are trade payables not requiring the issuance of a formal document such as a
promissory note. Trade payables are backed by the general credit worthiness of the buyer.

Question #19: Which of the following provisions are covered in the U.S Foreign Corrupt
Practices Act?
I. Illegal payments to foreign officials to assist in obtaining business.
II. Transparency of accounting records reflecting all transactions.
III. Payments to agents for the purpose of influencing foreign officials.
IV. Maintenance of an adequate system of internal controls.
Correct Answer: D. I. II. III. and IV.
A. I only.
B. I and III only.
C. II, III and IV only.
D. I, II, III and IV.
Explanation:
Answer (A) is incorrect.
The U.S. Foreign Corrupt Practices Act also covers transparency of accounting records reflecting
all transactions, payments to agents for the purpose of influencing foreign officials, and
maintenance of an adequate system of internal controls.
Answer (B) is incorrect.
The U.S. Foreign Corrupt Practices Act also covers transparency of accounting records reflecting
all transactions and maintenance of an adequate system of internal controls.
Answer (C) is incorrect.
The U.S. Foreign Corrupt Practices Act also covers illegal payments to foreign officials to assist
in obtaining business.
Answer (D) is correct.
The U.S. Foreign Corrupt Practices Act covers illegal payments to foreign officials to assist in
obtaining business, transparency of accounting records reflecting all transactions, payments to
agents for the purpose of influencing foreign officials, and maintenance of an adequate system of
internal controls.

Question #20: There is a market for both product X and product Y. Which of the following costs
and revenues would be most relevant in deciding whether to sell product X or process it further
to make product Y?
Correct Answer: A. Additional cost of making Y. given the cost of making X and additional
revenue from Y.
A. Additional cost of making Y. given the cost of making X and additional revenue from Y.
B. Total cost of making X and the revenue from sale of X and Y.
C. Total cost of making Y and the revenue from sale of Y.
D. Additional cost of making X given the cost of making Y. and additional revenue from Y.
Explanation:
A. Relevant revenues and costs are those that will differ among the related options. The
relevant revenue and cost in this problem are the additional cost of making and selling
product Y and the additional revenue (over and above the revenue from X) to be earned
from selling product Y.
B. The cost of making product X is a sunk cost and not relevant.
C. Only incremental (differential) costs are considered relevant.
D. Product Y can only be processed after product X is completed.
Question #21: A review of the inventories of Cedar Grove Company shows the following cost
data for entertainment centers.
$400.00 per
Invoice price unit
Freight & insurance on 20.00 per
shipment unit
15.00 per
Insurance on inventory unit
140.00 per
Unloading unit
10.00 per
Cost of placing orders order
Cost of capital 25%

What are the total unit carrying costs of inventory for an entertainment center?
Correct Answer: B. $120.
A. $105.
B. $120.
C. $115.
D. $420.
Explanation: The total unit carrying costs of the inventory are calculated as:
Total unit carrying costs of the inventory = (insurance cost per unit) + (lost interest on
investment)
Total unit carrying costs of the inventory = ($15) + (lost interest on investment)
The lost interest is calculated as follows:
Lost interest = (cost of capital)[(invoice price) + ($20 freight & insurance on shipment))
Lost interest = (0.25)0400) + ($20))
Lost interest = (0.25) ($420) = $105
Total unit carrying costs of the inventory = $15 + $105 = $120.

Question #22: Fact Pattern: Kell. Inc.. is analyzing an investment fora new product expected to
have annual sales of 100,000 units for the next 5 years and then be discontinued. New equipment
will be purchased for $1,200,000 and cost $300,000 to install. The equipment will be depreciated
on a straight-line basis over 5 years for financial reporting purposes and 3 years for tax purposes.
At the end of the fifth year, it will cost $100,000 to remove the equipment, which can be sold for
$300,000. Additional working capital of $400,000 will be required immediately and needed for
the life of the product. The product will sell for $80, with direct labor and material costs of $65
per unit. Annual indirect costs will increase by $500,000, Kell's effective tax rate is 40%.
Question: In a capital budgeting analysis, what is the cash flow at time = 0 (initial investment)
that Kell should use to compute the net present value?
Correct Answer: D. $1,900,000
A. $1,30,.000 •
B. $1,500,000
C. $1,700,000
D. $1,900,000
Explanation:
Answer (A) is incorrect.
The amount of $1,300,000 results from improperly subtracting the salvage value and failing to
include the increase in working capital.
Answer (B) is incorrect.
The amount of $1,500,000 results from failing to include the increase in working capital.
Answer (C) is incorrect.
The amount of $1,700,000 results from improperly including the salvage value and failing to
include the increase in working capital.
Answer (D) is correct.
The net initial investment for a capital project consists of three components: the purchase of new
equipment (including installation costs), the increase in working capital, and the after-tax
proceeds from the disposal of old equipment For Kell, the first of these is $1,500,000 ($120,000
+ $300,000), and the second is $400,000. No proceeds will be received on disposal because no
existing equipment is being removed. Therefore, the net cash flow at time = 0 is calculated as
follows:
Full cost of new equipment $1,500,000
Increase in working capital 400,000
Net initial investment $1,900,000.

Question #23: Which of the following statements describe the importance of a whistleblowing
framework in maintaining an ethical organizational culture?
I. It provides measurable feedback for determining whether employees are following a
code of ethics.
II. It creates opportunities to enhance and improve internal controls.
III. It empowers management to become better role models for employees.
IV. It helps to identify potential errors or risks at each task level within the organization.
Correct Answer: A. I and II only.
A. I and II only.
B. II and IV only.
C. III and IV only.
D. I, III and IV only.
Explanation:
Answer (A) is correct
The collection, analysis, and summarization of ethics issues can provide insight into the
operation of its code of ethics and the degree to which employees are following it, in addition,
tracking and monitoring issues raised through a whistleblowing framework creates opportunities
to enhance and improve internal control. For instance, a large number of fraud cases are reported
through a whistleblowing framework which can highlight areas where internal controls were
lacking.
Answer (B) is incorrect.
Each task level will not be analyzed by a whistleblowing framework because not all tasks are at
risk of fraud or weak controls.
Answer (C) is incorrect.
The whistleblowing framework would scare management into becoming better role models, not
empower them. Also, each task level will not be analyzed by a whistleblowing framework
because not all tasks are at risk of fraud or weak controls.
Answer (D) is incorrect.
The whistleblowing framework would scare management into becoming better role models, not
empower them. Also, each task level will not be analyzed by a whistleblowing framework
because not all tasks are at risk of fraud or weak controls.

Question #24: Opportunity costs are


Correct Answer: C. Relevant to decision making.
A. Equal to historical costs.
B. Not used for decision making.
C. Relevant to decision making.
D. The same as variable costs.
Explanation:
A. Historical costs are those cost that have been incurred in the past.
B. Opportunity cost are relevant to the decision making process.
C. Opportunity cost is relevant to decision making. Opportunity cost is the benefit to income
(inflow) that is lost by not using a limited resource for its best alternative use.
D. Opportunity cost is the benefit to income (inflow) that is lost by not using a limited
resource for its best alternative use. Variable costs are those costs that vary with the level
of production (outflow).

Question #25: Power Systems. Inc. manufactures jet engines for the United States armed forces
on a cost-plus basis. The cost of a particular jet engine the company manufactures is shown as
follows.

$200,00
Direct materials 0
Direct labor 150,000
Overhead:
Supervisr's salary 20,000
Fringe benefits on direct
labor 15,000
Depreciation 12,000
Rent 11,000
$408,00
Total cost 0
If production of this engine were discontinued, the production capacity would be idle, and the
supervisor would be laid off. When asked to bid on the neat contract for this engine, the
minimum unit price that Power Systems should bid is
Correct Answer: A. $385,000.
A. $385,000.
B. $397,000.
C. $365,000.
D. $408,000.
Explanation:
A. In this problem want to calculate all avoidable cost. Therefore, the minimal unit price that
Power Systems should bid is $385,000 ($200,000 DM + $150,000 DL + $20,000
Supervisors salary + $15,000 Fringe benefits on DL).
B. This answer includes depreciation which is an unavoidable cost, and should not be
included.
C. This answer does not include the salary of the supervisor who would be laid off if
production of the engine were discontinued. This is an avoidable cost and should be
included in the calculation.
D. This answer includes unavoidable cost (depreciation and rent) that should not be
included.

Question #26: Maple Motors buys axles in order to produce automobiles. Maple carries an
average credit balance of $25,000,000 with its axle supplier. The axle supplier provides credit
terms of 1/10 net 25. The nominal annual cost of Maple not taking the trade discount is closest to
which one of the following? Assume a 365-day year.
Correct Answer: B. 24.6%.
A. 24.0%.
B. B. 24.6%.
C. 14.4%.
D. 14.5%.
Explanation:
Nominal annual cost = [% discount / (100% - %discount)] x [365 / (payment date - discount
period)]
Nominal annual cost = (1% / 99%) x (365 / 15) = 24.6%.

Question #27: Which one of the following is a debt instrument that generally has a maturity of
ten years or more?
Correct Answer: C. A bond.
A. A financial lease.
B. A chattel mortgage.
C. A bond.
D. A treasury note.
Explanation:
A bond is a promise to pay a specified amount of interest over time and to repay principal at
maturity. These instruments generally have long-term maturities. Treasury notes have maturities
of one to ten years. Treasury bill maturities are less than one year. A financial lease is a contract.

Question #28: J J Motors. Inc. employs 45 sales personnel to market its line of luxury
automobiles. The average car sells for $23,000, and a 6% commission is paid to the salesperson.
J.J Motors is considering a change to a commission arrangement that would pay each salesperson
a salary of $2,000 per month plus a commission of 2% of the sales made by that salesperson. The
amount of total monthly car sales at which .1.1Motors would be indifferent as to which plan to
select is
Correct Answer: D. $2,230,000.
A. $3,000,000.
B. $1,500,000.
C. $1,250,000.
D. $2,230,000.
Explanation:
A. A monthly total sales level of $3,000,000 would result in total commissions paid of
$180,000 if J J Motors continues paying commissions at the rate of 6% of sales
($3,000,000 x 0.06). If J J Motors adopts the new commission formula of $2,000 per
month per salesperson plus 2% of sales, the total commissions paid would be $130,000
($2,000 x 45) + (0.02 x $3,000,000). At a sales level of $3,000,000. J J Motors would
prefer the new commission formula because the commission payout would be lower.
B. A monthly total sales level of $1,500,000 would result in total commissions paid of
$90,000 if J J Motors continues paying commissions at the rate of 6% of sales (1,500,000
x 0.06). If J J Motors adopts the new commission formula of $2,000 per month per
salesperson plus 2% of sales, the total commissions paid would be $120,000 ($2,000 x
45) + (0.02 x $1,500,000). At a sales level of $1,500,000, J J Motors would prefer the
existing commission formula because the commission payout would be lower.
C. A monthly total sales level of $1,250,000 would result in total commissions paid of
$75,000 if J J Motors continues paying commissions at the rate of 6% of sales (1,250,000
x 0.06). If J J Motors adopts the new commission formula of $2,000 per month per
salesperson plus 2% of sales, the total commissions paid would be $115,000 ($2,000 x
45) + (0.02 x $1,230,000). At the sales level of $1,250,000, J J Motors would prefer the
existing commission formula because the commission payout would be lower.
D. There is more than one correct way to calculate the answer to this problem. One solution
is to calculate the sales volume where J J Motors will indifferent as to which plan is
selected. Remember that the total sales volume will be the sales made per salesperson
multiplied by the number of sales personnel.
The solution can be calculated algebraically as follows:
Let X be the monthly sales per salesperson. Create two equations, each containing one of
the possible commission payment methods.
The two equations are:
C = 0.06X and C = $2,000 x 0.06X, where X = the monthly car sales.
Next, set the right sides of the two commission payment methods equal to one another,
since we are looking for the total monthly sales at which the same commission amounts
would be paid under both methods. Solve for X. and then multiply the value you get for
X by 45 to get the total sales for all salespeople per month.
0.06X = $2,000 + 0.02X
Simplify by subtracting 0.02X from both sides of the equation:
0.04X = $2,000
Solve for X by dividing both sides of the equation by 0.04:
X = $50,000, which is the sales per salesperson at which the payment under both plans
will be equal.
Multiply $50,000 by 45 salespeople to get the total sales per month at which both
payment plans will result in the same payments to salespeople:
$50,000 x 45 = $2,250,000
To check: At a sales volume of $2,250,000, a straight commission of 6% will result in a
total payment of $2,250,000 x 0.06, or $135,000.
At a sales volume of $2,250,000, a salary of $2,000 per month paid to each salesperson
plus a commission of 2% will result in a total payment of:
($2,000 x 45) + (0.02 x $2,250,000) = $135,000
If you calculated the answer a different way but got the same answer, then your method is
also correct.

Question #29: A toothbrush manufacturer has noticed a shift of customer preferences in


its growing Asian sales market towards an electronic battery operated toothbrush from a
manual toothbrush. This shifting of customer tastes best represents what type of risk to
the toothbrush manufacturer?
Correct Answer: D. Strategic risk.
A. Business risk.
B. Financial risk
C. Operational risk
D. Strategic risk.
Explanation:
A. A shifting of customer tastes does not represent business risk to the toothbrush
manufacturer. Business risk for a firm is the risk of changes in its earnings before
interest or taxes. Business risk depends on a variety of factors, including the
variability of demand, sales price, and the price of inputs as well as the amount of the
company's operating leverage. The more stable all of these variables are, the less
business risk a company will experience.
B. A shifting of customer tastes does not represent financial risk to the toothbrush
manufacturer. Financial risk is risk connected to the financial health of the company.
Examples include volatility of foreign currencies, volatility of interest rates, volatility
of commodity prices (inputs), credit risk liquidity risk and market risk Furthermore,
when a company borrows, it obligates itself to pay interest charges and to repay the
principal, creating financial risk Financial risk related to borrowing includes two
aspects: 1. the risk that the firm will not be able to pay its interest and other
obligations when they become due because of lack of cash flow, and 2. The increased
variability in earnings per share caused by the use of debt and the accompanying
requirement to pay interest on the debt.
C. A shifting of customer tastes does not represent operational risk to the toothbrush
manufacturer. Operational risk is risk that results from inadequate or failed internal
processes, people or systems. Some examples of operational risks are technology,
business continuity, customer satisfaction, and the risk of product or service failure.
Operational risk includes legal risk and compliance risk Legal risk is the risk that is
associated with uncertainty due to legal actions or uncertainty in the applicability or
interpretation of contracts, laws or regulations where the company operates.
Compliance risk is the current or future risk to profits or the company's assets as a
result of violations of, or nonconformance with, laws, rules, regulations, required
practices, internal policies and procedures, or ethical standards.
D. A shifting of customer tastes best represents strategic risk to the toothbrush
manufacturer. Strategic risk is risk such as reputation risk brand risk (patent and
trademark protection), leadership risk and the risk of customers' needs changing.
Strategic risks also include risks related to actions of competitors and changes in the
regulations businesses are subject to, as regulatory changes could cause significant
increases in compliance expense. Capital availability is another strategic risk.

Question #30: If an IMA member has a problem in identifying unethical behavior or resolving an
ethical conflict, the first action (s)he should normally take is to
Correct Answer: B. Discuss the problem with his or her immediate superior.
A. Consult the board of directors.
B. Discuss the problem with his or her immediate superior.
C. Notify the appropriate law enforcement agency.
D. Resign from the company.
Explanation:
Answer (A) is incorrect.
The board would be consulted initially only if the immediate superior is the chief executive
officer and that person is involved in the ethical conflict.
Answer (B) is correct.
IMA's Statement of Ethical Professional Practice states that the member should first discuss an
ethical problem with his or her immediate superior. If the superior is involved, the problem
should be taken initially to the next higher managerial level.
Answer (C) is incorrect.
An IMA member should keep information confidential except when disclosure is authorized or
legally required.
Answer (D) is incorrect.
Resignation is a last resort.

Question #31: Which of the following statements accurately characterizes economic order
quantity (EOQ) and safety stock principles?
Correct Answer: A. EOQ analysis assumes demand and lead time are known with certainty;
safety stock uses a model involving probability.
A. EOQ analysis assumes demand and lead time are known with certainty: safety stock uses
a model involving probability.
B. EOQ analysis involves a probability model: safety stock assumes demand varies but lead
time is known with certainty.
C. Both models are based on the premise that there is no true measure of certainty in
inventory control.
D. Both models advocate reducing stock to a predetermined reorder level.
Explanation:
While EOQ analysis is based on the theoretical assumption that demand and lead time are
known with certainty, safety stock uses a model involving probability. Safety stock analysis
attempts to address the uncertainty found in inventory systems, particularly the real-world
uncertainty in demand and lead time.
Question #32: On its year-end financial statements. Caper Corporation showed sales of
$3,000,000, net fixed assets of $1,300,000, and total assets of $2,000,000. The company's fixed
asset turnover is:
Correct Answer: A. 2.3 times.
A. 2.3 times.
B. 1.5 times.
C. 63.0%.
D. 43.3%.
Explanation:
The fixed asset turnover is equal to net sales divided by the average net fixed assets for the year.
Fixed asset turnover = (net sales) / (average net fixed assets for the year)
Fixed asset turnover = 53.000.000 / 31.300.000 = 2.3 times.

Question #33: A company has just completed the final development of its only product, general
recombinant bacteria, which can be programmed to kill most insects before dying themselves.
The product has taken 3 years and $6,000,000 to develop. The following costs are expected to be
incurred on a monthly basis for the normal production level of 1.000.000 pounds of the new
product:
Correct Answer: C. 15,000,000 pounds.
A. 25,600,000 pounds.
B. 13,017,000 pounds.
C. 15,000,000 pounds.
D. 14,000,000 pounds.
Explanation:
A. This is not the correct answer. Please see the correct answer for an explanation. We have
been unable to determine how to calculate this incorrect answer choice. If you have
calculated it, please let us know how you did it so we can create a full explanation of why
this answer choice is incorrect. Please send us an email at admin@cmapass.com. Include
the Topic Name. Question Details or Screenshot and the actual incorrect answer choice --
not its letter, because that can change with every study session created.. Thank you in
advance for helping us to make your CMA PASS study materials better.
B. This is not the correct answer. Please see the correct answer for an explanation. We have
been unable to determine how to calculate this incorrect answer choice. If you have
calculated it, please let us know how you did it so we can create a full explanation of why
this answer choice is incorrect. Please send us an email at admin@cmapass.com. Include
theTopic Name. Question Details or Screenshot and the actual incorrect answer choice --
not its letter, because that can change with every study session created.. Thank you in
advance for helping us to make your CMA PASS study materials better.
C. The cost information given is for one month's production. The unit contribution margin is
$3 per pound, calculated as follows:
Selling price per pound $5.90
Less: Variable costs per lb. based on 1,000,000 lbs:
Direct materials: $300,000 ÷ 1,000,000 lbs 0.30
Direct labor: $1,250,000 ÷ 1,000,000 lbs 1.25
Variable factory overhead: $450,000 + 1,000,000 lbs 0.45
Variable selling, general, and adm. expense: $900,000 ÷ 1,000,000 lbs 0.90
Contribution margin per pound $3.00
Fixed factory overhead for one month is $2,000,000, and fixed selling, general and
administrative expense for one month is $1,500,000. Since the company's income tax rate is not
given, we can assume the question is asking for before-tax profit. The required profit given is for
a year, whereas the costs given are for one month. Therefore, we must adjust either the fixed
costs or the required profit amount so they are both for the same duration. Since the question
asks for an annual sales volume, we will adjust the costs by multiplying them by 12. Fixed
factory overhead for one year is $24,000,000 and fixed selling, general and administrative
expense is $18,000,000. The formula to find the required number of units to sell to earn a
specific profit is: (Fixed Costs + Required Profit) / Contribution Margin Per Unit ($24,000,000 +
$18,000,000+ $3,000,000) / $3 = 15,000,000 pounds
D. This is the breakeven volume, not the volume required to earn a $3,000,000 profit. The
required profit should be included as another fixed cost in the calculation.

Question #34: Which one of the following ethics-related actions by management is least
effective in encouraging acceptance by employees of an organization's code of ethics?
Correct Answer: A. Management appoints an ethics officer to monitor and report to management
on employee compliance.
A. Management appoints an ethics officer to monitor and report to management on
employee compliance.
B. Management follows ethical principles in decisions made on behalf of the organization.
C. Management keeps promises and commitments made to employees, customers, and
vendors.
D. Management supports employees in adhering to ethics standards.
Explanation:
Answer (A) is correct
This approach does not involve employees at all and projects the image that employees need to
be supervised because they are likely to be unethical. This can discourage some employees.
Answer (B) is incorrect.
Management following ethical principles is an example of the "tone at the top." The "tone at the
top' is one of the most effective ways of encouraging employees to behave ethically and accept
the organization's code of ethics.
Answer (C) is incorrect
Management keeping promises and commitments is a sign that they behave ethically. The "tone
at the top" is one of the most effective ways of encouraging employees to behave ethically and
accept the organization's code of ethics.
Answer (D) is incorrect.
Management support can motivate employees because it shows that management is involved.

Question #35: If a company uses off-balance-sheet financing, assets have been acquired:
Correct Answer: A. with operating leases.
A. With operating leases.
B. With a line of credit.
C. For cash.
D. With financing leases.
Explanation:
Companies make use of an asset without showing the corresponding obligation. Four of the
common techniques employed to achieve off-balance sheet financing are: factoring of
accounts receivables; special purpose entities: operating leases; and joint ventures.

Question #36: Shown below are selected data from Fortune Company's most recent financial
statements.

Marketable securities $10,000


Accounts receivable 60,000
Inventory 25,000
Supplies 5,000
Accounts payable 40,000
Short term debt payable 10,000
Accruals 5,000

What is Fortune's net working capital?


Correct Answer: A. $45.000.
A. $45,000.
B. $35,000.
C. $50,000.
D. $80,000.
Explanation:
Net working capital is calculated as total current assets minus current liabilities. Therefore
(10,000 + 60,000 + 25,000 + 5,000) - (40,000 + 10,000 + 5,000) = $45,000.

Question #37: Condensed monthly operating income data for Korbin Inc. for May follows:
Urban Suburba
store n store Total
$80,00 $120,00 $200,00
Sales 0 0 0
Variable costs 32,000 84,000 116,000
Contribution margin 48,000 36,000 84,000
Direct fixed costs 20,000 40,000 60,000
$28,00
Store segment margin 0 $(4,000) $24,000
Common fixed cost 4,000 6,000 10,000
$24,00 $(10,000
Operating income 0 ) $14,000

Additional information regarding Korbin's operations follows:


• One-fourth of each store's direct fixed costs would continue if either store were closed.
• Korbin allocates common fixed costs to each store on the basis of sales dollars.
• Management estimates that closing the Suburban Store would result in a 10% decrease in the
Urban Store's sales, while closing the Urban Store would not affect the Suburban Store's sales.
• The operating results for May are representative of all months.
Korbin is considering a promotional campaign at the Suburban Store that would not affect the
Urban Store. Increasing annual promotional expense at the Suburban Store by $60,000 in order
to increase this store's sales by 10% would result in a monthly increase (decrease) in Korbin's
operating income during the year (rounded) of
Correct Answer: C. $(1.400)
A. $(5,000)
B. $487
C. $(1,400)
D. $7,000
Explanation:
A. The answer of a $5,000 decrease to monthly operating income does not include the
increase in the contribution margin from the increased sales resulting from the promotion.
B. This is not the correct answer. Please see the correct answer for an explanation. We have
been unable to determine how to calculate this incorrect answer choice. If you have
calculated it, please let us know how you did it so we can create a full explanation of why
this answer choice is incorrect. Please send us an email at admin@cmapass.com. Include
theTopic Name. Question Details or Screenshot and the actual incorrect answer choice --
not its letter, because that can change with every study session created.. Thank you in
advance for helping us to make your CMA PASS study materials better.
C. The monthly cost for the advertising is $5,000 ($60,000 ÷12). The advertising will
increase sales by 10%, so the contribution margin will increase by $3,600 ($36,000
contribution margin x 1.10). Therefore, the advertising promotion will cost the company
an additional $1,400 ($3,600 increase in contribution margin minus $5,000 advertising
cost).
D. The answer of a $7,000 increase to monthly operating income results from correctly
considering the increase in sales revenue (a $12,000 increase to monthly operating
income) and the increase in advertising expense (a $5.000 decrease to monthly operating
income). However, the increased sales will result in a 10% increase in variable costs, as
well, which will reduce operating income. That reduction is not considered in this
answer.

Question #38: Large firms often seek to control risk through allocating or rationing capital
among divisions. When capital is rationed, managers are most likely to choose among
prospective investments based on their
Correct Answer: D. Profitability index rankings.
A. Net present value (NPV) rankings.
B. Internal rate of return (IRR) rankings.
C. Payback periods.
D. Profitability index rankings.
Explanation:
Answer (A) is incorrect.
Managers will most likely not choose their investments based solely on the NPV of the projects
when capital is rationed. This method does not take into consideration the limited resources
available. The net initial investment is also a factor that must be considered when selecting
among prospective projects.
Answer (B) is incorrect.
Managers will most likely not choose their investments based solely on the IRR of the project
when capital is rationed. The internal rate of return expresses a project's return in percentage
terms. The IRR of an investment is the discount rate at which the investment's NPV equals zero.
This method does not take into consideration the fact that resources may be limited.
Answer (C) is incorrect
Managers will most likely not choose their investments based solely on the payback periods of
the project when capital is rationed. The payback period is the number of years required to return
the original investment, that is, the time necessary for a new asset to pay for itself. Note that no
consideration is made for the time value of money under this method. In addition, this method
does not take into consideration the fact that resources may be limited.
Answer (D) is correct.
The profitability index is a method for ranking projects to ensure that limited resources are
placed with the investments that will return the highest NPV. This can be calculated by dividing
the NPV of the future cash flows by the net investment.

Question #39: Elements of project risk identification include which one of the following?
Correct Answer: A. Interviews and observations.
A. Interviews and observations.
B. Cost estimates.
C. Actual risk events.
D. Activity duration estimates.
Explanation:
A. Interviews and observations are risk identification techniques. Risk identification
techniques include the following: -Event inventories, detailed lists of potential events or
an archive of events that have occurred; -Internal analysis, for example the company's
own experience with similar projects: -Escalation or threshold triggers to alert
management to areas of concern: -Facilitated workshops used to identify events by
drawing on accumulated knowledge and experience of management. staff and other
stakeholders; -Interviews, questionnaires, and surveys; -Process flow analysis using flow
diagrams to consider the inputs, tasks, responsibilities, and outputs of the project: -
Leading event indicators to identify conditions that could give rise to an event: -Loss
event data on past loss events: and -Brainstorming sessions.
B. Cost estimates are not risk identification techniques. Risk identification techniques
include the following: -Event inventories, detailed lists of potential events or an archive
of events that have occurred; -Internal analysis, for example the company's own
experience with similar projects: -Escalation or threshold triggers to alert management to
areas of concern: -Facilitated workshops used to identify events by drawing on
accumulated knowledge and experience of management, staff and other stakeholders; -
Interviews, questionnaires, and surveys; -Process flow analysis using flow diagrams to
consider the inputs, tasks, responsibilities, and outputs of the project: -Leading event
indicators to identify conditions that could give rise to an event: -Loss event data on past
loss events: and -Brainstorming sessions.
C. Actual risk events are not risk identification techniques, though experience with historical
events is used in risk identification techniques. Risk identification techniques include the
following: -Event inventories. detailed lists of potential events or an archive of events
that have occurred; -Internal analysis, for example the company's own experience with
similar projects: -Escalation or threshold triggers to alert management to areas of
concern; -Facilitated workshops used to identify events by drawing on accumulated
knowledge and experience of management, staff and other stakeholders; -Interviews,
questionnaires, and surveys; -Process flow analysis using flow diagrams to consider the
inputs, tasks, responsibilities, and outputs of the project: -Leading event indicators to
identify conditions that could give rise to an event: -Loss event data on past loss events:
and -Brainstorming sessions.
D. Activity duration estimates are not risk identification techniques. Risk identification
techniques include the following: -Event inventories. detailed lists of potential events or
an archive of events that have occurred; -Internal analysis, for example the company's
own experience with similar projects: -Escalation or threshold triggers to alert
management to areas of concern; -Facilitated workshops used to identify events by
drawing on accumulated knowledge and experience of management, staff and other
stakeholders; -Interviews, questionnaires, and surveys; -Process flow analysis using flow
diagrams to consider the inputs, tasks, responsibilities, and outputs of the project: -
Leading event indicators to identify conditions that could give rise to an event: -Loss
event data on past loss events: and -Brainstorming sessions.
Question #40: A firm's financial risk is a function of how it manages and maintains its debt.
Which one of the following sets of ratios characterizes the firm with the greatest amount of
financial risk?
Correct Answer: D. High debt-to-equity ratio, low interest-coverage ratio, volatile return on
equity.
A. High debt-to-equity ratio, high interest-coverage ratio, volatile return on equity.
B. Low debt-to-equity ratio, low interest-coverage ratio, volatile return on equity.
C. High debt-to-equity ratio, high interest-coverage ratio, stable return on equity.
D. High debt-to-equity ratio, low interest-coverage ratio, volatile return on equity.
Explanation:
A. A company with the greatest amount of financial risk would have a low interest coverage
ratio. See the correct answer for a complete explanation.
B. A company with the greatest amount of financial risk would have a high debt-to-equity
ratio. See the correct answer for a complete explanation.
C. A company with the greatest amount of financial risk would have a low interest coverage
ratio and a volatile return on equity. See the correct answer for a complete explanation.
D. A company that has the greatest amount of financial risk will have a high debt-to-equity
ratio, a low interest-coverage ratio (meaning that their profits are not much greater than
their interest payments) and a volatile return on equity (which indicates that profits
fluctuate greatly from one period to the next).

Question #41: Consider the statements below regarding accounting treatments for business
combinations. Which statement is incorrect?
Correct Answer: C. Under [FRS. a subsidiary must be consolidated when the parent owns a
majority of voting interest in the subsidiary.
A. Under IFRS, a subsidiary must be consolidated when it is under control of the parent.
B. Under U.S. GAAP, a subsidiary must be consolidated when the parent owns a majority of
voting interest in the subsidiary.
C. Under IFRS, a subsidiary must be consolidated when the parent owns a majority of
voting interest in the subsidiary.
D. Under U.S. GAAP. a subsidiary must be consolidated when it is under control of the
parent
Explanation:
Accounting treatments for business combinations include the requirements of consolidation of
subsidiaries under the control of a parent (IFRS and U.S. GAAP).and majority ownership by a
parent (IFRS).

Question #42: The controller of a manufacturer is evaluating two projects and wishes to do a
cash flow analysis of each of the projects. Both projects have positive cash inflows starting in
Year 1 and have similar initial investments. The cost of capital is expected to fluctuate during the
life of the projects, and the controller has selected the net present value method for her analysis.
Did the controller select the most appropriate method for her analysis?
Correct Answer: A. Yes, she should have selected the net present value method because it can
properly consider the fluctuating cost of capital.
A. Yes, she should have selected the net present value method because it can properly
consider the fluctuating cost of capital.
B. No, she should have selected the payback method to properly consider the initial
investments and time value of money.
C. No, she should have selected the accounting rate of return since it will properly consider
the time value of money.
D. No, she should have selected the internal rate of return method to properly consider the
fluctuating cost of capital.
Explanation:
Answer (A) is correct
A project's NPV can easily be determined using different desired required rates of return for
different periods, so it would be able to properly consider the fluctuating cost of capital.
Answer (B) is incorrect.
The accounting rate of return ignores the time value of money and long-term profitability, and
therefore the two projects cannot be compared.
Answer (C) is incorrect.
The accounting rate of return ignores the time value of money, and therefore the two projects
cannot be compared.
Answer (D) is incorrect.
The IRR is limited to a single summary rate for the entire project and therefore cannot properly
consider the fluctuating cost of capital.
Question #43: Markowitz Company increased its allowance for uncollectable accounts. This
adjustment will:
Correct Answer: A. reduce the current ratio.
A. Reduce the current ratio.
B. Increase the acid test ratio.
C. Increase working capital.
D. Reduce debt-to-asset ratio.
Explanation:
The current ratio is calculated as current assets divided by current liabilities. By increasing the
allowance for doubtful accounts, the next effect is to reduce the current asset accounts
receivable. Therefore, this adjustment would reduce the current ratio.

Question #44: Sudden economic changes have forced the AutoFacsimilie Co. to alter its business
strategy. The company is considering eliminating product lines, laying off production workers,
reducing advertising, and closing one of its factories. In taking these actions, which one of the
following costs should be considered sunk costs?
Correct Answer: C. Research and development costs of eliminated product lines.
A. Production workers’ wages, severance, and advertising.
B. The costs of selling or demolishing the factory.
C. Research and development costs of eliminated product lines.
D. Utility costs at the closed factory and real estate taxes.
Explanation:
A. Sunk costs are costs that have already been spent and cannot be changed no matter which
decision is made today. As none of the production worker's wages, severance and
advertising costs have been incurred yet, all of them could be avoided depending on the
decision that is made today. Therefore, they are not sunk costs.
B. Sunk costs are costs that have already been spent and cannot be changed no matter which
decision is made today. The cost of selling the factory or demolishing it have not yet been
incurred and could therefore be avoided in the future by deciding to continue to operate
the factory. Therefore, they are not sunk costs.
C. Sunk costs are costs that have already been incurred and cannot be changed no matter
which decision is made today. Research and development costs of eliminated product
lines have already been spent, and no decision made now can change them. Therefore,
those costs are sunk costs.
D. Sunk costs are costs that have already been spent and cannot be changed no matter which
decision is made today. As long as the company continues to own the factory, it will
continue to have expenses related to its ownership, even if the factory is closed. Real
estate taxes will continue to be due and payable, because they are based on the value of
the property, whether or not the property is occupied. It will be necessary to keep some
utilities connected in order to preserve the value of the factory, even if it is not being
used. For example, if the factory is located in a cold climate, they will need to keep it
heated at least somewhat so that frost damage does not occur in the building. If the
factory is located in a hot climate, they will need to keep it cooled at least somewhat so
that mold does not grow in the building. So those expenses will continue in the future,
until they sell the building. Therefore, utility costs and real estate taxes for the closed
factory are not sunk costs because they are not costs that have already been spent. They
are future costs.

Question #45: For a profitable company, the amount by which sales can decline before
losses occur is known as the
Correct Answer: C. Margin of safety.
A. Hurdle rate.
B. Marginal income rate.
C. Margin of safety.
D. Sales volume variance.
Explanation:
A. The hurdle rate is a capital budgeting term. It is the rate of return that must be
received on a specific project under consideration before it will be acceptable to
management.
B. The marginal income rate is the rate at return that is gained from making one more
sale or one more investment.
C. The margin of safety is the amount by which sales can decrease before losses can
occur (budgeted/actual sales — sales level at breakeven).
D. The Sales volume variance measures the impact of differences in sales volume. The
calculation is [(Actual Sales volume - Budgeted Sales volume) × Standard
contribution per unit].

Question #46: Nelson Industries increased earnings before interest and taxes by 17%.
During the same period, net income after tax increased by 42%. The degree of financial
leverage (DFL) that existed during the year is:
Correct Answer: A.2.47.
A. 2.47.
B. 1.70.
C. 1.68.
D. 4.20
Explanation:
The DFL is defined as the percent change in net income after tax given e percent change
in operating income (earnings before interest and taxes, or EBIT).
Nelson's degree of financial leverage is calculated as:
DFL = (% change in net income after tax) / (% change in operating income, or EBIT)
Nelson's DFL = (42%) (17%) = 2.47.

Question #47: Risk assessment is a process that involves identifying the risks and
vulnerabilities that an organization is exposed to in order to identify events that may
occur and affect the entity negatively. How is a particular risk assessed or measured?
Correct Answer: B. By the loss frequency and the loss severity
A. The amount of a loss, if a loss does occur
B. By the amount of the expected loss
C. By the probability of a loss occurring
D. By the lass frequency and the loss severity
Explanation:
A. The amount of a loss, if a loss does occur, is used in calculating the expected loss,
and expected loss is one of the terms used to express the measurement of a potential
loss from e specific risk. However, the amount of a loss if it occurs is not the only
input into calculating expected loss and assessing risk.
B. The expected loss, given set at probabilities, is the amount that management expects
to be lost to a given risk on average in one year. Expected lass is one at the terms
used to express the measurement of a potential loss from e specific risk, but it is not
the way risk is assessed.
C. The probability or probabilities of a loss occurring are used in calculating the
expected loss. The expected loss, given a set of probabilities, is the amount that
management expects to be lost to e given risk an average in one year. The probability
of a loss occurring is used in calculating the expected loss, and expected loss is one
of the terms used to express the measurement of a potential loss from a specific risk.
However, the probability at loss occurring is not the only input into calculating
expected loss and assessing risk.
D. Risk assessment is the process of analyzing and considering risks from two
perspectives: (1) the likelihood of the risk's occurring and (2) the potential impact of
the event if it does occur. The likelihood of the risk's occuring is called the loss
frequency. The potential impact of the event if it does occur is called loss severity.
Loss frequency is the measurement of how often the loss occurs, on average Loss
frequency is expressed in relation to time period such as a year. A loss frequency of
0.23 year means there is a probability that a lass will take place in any given year,
and on average, a loss occurs once every four years Loss severity measures how
serious a loss is when it occurs, in terms of cost For example, historically when a
loss has occurred, the average cost of the loss is $30,000. Both the loss frequency
and the loss severity are used in measuring the potential loss that could occur from a
specific risk and thus assessing the risk.

Question #48: The use of an accelerated method instead of the straight-line method of
depreciation in computing the net present value of a project has the effect of
Correct Answer: C. Increasing the present value of the depreciation tax shield.
A. Raising the hurdle rate necessary to justify the project.
B. Lowering the net present value of the project.
C. Increasing the present value of the depreciation tax shield.
D. Increasing the cash outflows at the initial point of the project
Explanation:
Answer (A) is incorrect.
The hurdle rate can be reached more easily as a result at the increased present value at
the depreciation tax shield.
Answer (B) is incorrect
The greater depreciation tax shield increases the NPV.
Answer (C) is correct.
Accelerated depreciation results in greater depreciation in the early years at an asset's
life compared with the straight-line method. Thus, accelerated depreciation results in
lower income tax expense in the years of e project and higher income tax expense in the
oater years. By effectively deferring taxes, the accelerated method increases the present
value of the depreciation tax shield.
Answer (D) is incorrect.
Greeter initial depreciation reduces the cash outflows for the taxes, but has no effect on
the initial cash outflows.
Question #49: Which of the following is a benefit of a well developed and implemented
Enterprise Risk Management (ERM) system?
Correct Answer: B. Identification end management at cross-enterprise risks.
A. There is less probability of events occurring that would impact the company's ability
to achieve its objectives.
B. Identification end management at cross-enterprise risks.
C. Higher expected cash can be used in capital budgeting analyses because of less
uncertainty.
D. Reliable information for use in decision making.
Explanation:
A. The probabilities of events occurring that would impact the company's ability to
achieve its objectives are the same after implementation at an ERV system as they
were before.
B. Identification and management of multiple and cross-enterprise risks is e benefit of
an effective ERM system. Enterprise Risk Management is different from traditional
risk management where the heads of the various units identify end manage the risks
they perceive as effecting their own unit's ability to achieve its objectives. With
ERM, risks that affect the whole enterprise cross- enterprise risks that may be missed
by individual department or division heads are recognized and managed.
C. An effective ERM system does not result in higher expected cash flows being used in
capital budgeting analyses.
D. Reliable information for use in decision making is a benefit of a strong internal
control system. It is not a benefit of e good ERM system.

Question #50: Donnelly Corporation manufactures and sells T-shirts imprinted with college
names and slogans. Last year, the shirts sold for $7.50 each, and the variable cost to manufacture
them was $2.25 per unit. The company needed to sell 20.000 shirts to break even. The net
income last year was $5,040. Donnelly's expectations for the coming year include the following:
-The sales price of the T-shirts will be $9
-Variable cost to manufacture will increase by one-third
-Fixed costs will increase by 10%
-The income tax rate of 40% will be unchanged If Donnelly Corporation wishes to earn $22,500
in net income for the coming year, the company's sales volume in dollars must be
Correct Answer: A. $229,500
A. $229,500
B. Some amount other than those given.
C. $257,625
D. $213,750
Explanation:
A. This question is asking for the sales revenue given a requirement for an after-tax net
income of $22,500. To solve it, we need to use the version of the breakeven formula for
determining the sales revenue required to result in a specific dollar amount of profit.
Target Sales Revenue = (FC + Target Pretax Income) / Contribution Margin Ratio
In order to use this formula, we need to calculate three things: (1) the amount of fixed
cost in the coming year. (2) The amount of desired net income before tax for the coming
year, and (3) The contribution margin ratio for the coming year.
(1) We are told that fixed cost for the coming year will be 10% higher than the previous
years fixed cost. So we need to find what the fixed cost was for last year. We know the
break-even point in units for last year (20,000) and we know the unit contribution margin
for last year ($7.50 - $2.25 = $5.25). So we can find the fixed cost for last year by using
the Break-Even Point in Units formula and solving for FC: FC / Unit Contribution
Margin = BEP in Units.
FC / 5.25 = 20,000
Solving for FC, we get FC = $105,000
Since fixed cost for the coming year will be 10% higher than last year, fixed cost for the
coming year will be $105,000 x 1.10, which is $115,500. (2) The formula to find before-
tax net income when we know the after-tax net income is After-Tax N I / (1- tax rate).
Therefore, the desired before tax net income is $22,500 / (1- 0.40), which is $37,500. (3)
We are told that variable cost to manufacture will increase by one-third. Variable cost last
year was $2.25 per unit. Therefore, variable cost in the coming year will increase by 1/3
of $2.25, which is $0.75, so variable cost will be $3 per unit. We are told that the sales
price will be $9. Therefore, the Contribution Margin Ratio will be $6 / $9, which is 2/3 or
0.666667.
Now, we can calculate the Target Sales Revenue, because we have the fixed cost, the
target pretax income, and the contribution margin ratio for the coming year.
Target Sales Revenue = ($115,500 + $37,500) / 0.666667 = $229.500.
B. The correct answer is given.
C. This answer results from calculating the desired before-tax net income by dividing the
after-tax net income by the tax rate. To calculate the desired before-tax net income, the
after-tax net income should be divided by (1- the tax rate).
D. This answer results from using fixed costs of $105,000 to calculate the target sales in
dollars. Fixed costs are expected to increase by 10% in the coming year.
Question #51: For a firm engaged in risk management. Value at Risk is defined as the
Correct Answer: C. maximum loss within a certain time period at a given level of confidence.
A. Most likely negative outcome at a given level of confidence.
B. Maximum value a company can lose.
C. Maximum loss within a certain time period at a given level of confidence.
D. Worst possible outcome given the distribution of outcomes.
Explanation:
A. This is not the definition of Value at Risk Value at Risk (VaR) measures the potential
loss in value of a risky asset or event over a defined period for a given confidence
interval. It is based on the assumption that the possible outcome of the event is
represented by a normal distribution (bell curve). With a normal distribution, we know
that 95% of the results will lie within 1.96 standard deviations of the mean, and that 99%
of the results will lie within 2.57 standard deviations of the mean. Using this information,
we can predict what the range of results will be with a measured level of confidence.
B. This is not the definition of Value at Risk Value at Risk (VaR) measures the potential
loss in value of a risky asset or event over a defined period for a given confidence
interval. It is based on the assumption that the possible outcome of the event is
represented by a normal distribution (bell curve). With a normal distribution, we know
that 95% of the results will lie within 1.96 standard deviations of the mean, and that 99%
of the results will lie within 2.57 standard deviations of the mean. Using this information,
we can predict what the range of results will be with a measured level of confidence.
C. Value at Risk (VaR) measures the potential loss in value of a risky asset or event over a
defined period for a given confidence interval. It is based on the assumption that the
possible outcome of the event is represented by a normal distribution (bell curve). With a
normal distribution, we know that 95% of the results will lie within 1.96 standard
deviations of the mean, and that 99% of the results will lie within 2.57 standard
deviations of the mean. Using this information, we can predict what the range of results
will be with a measured level of confidence.
D. This is not the definition of Value at Risk Value at Risk (VaR) measures the potential
loss in value of a risky asset or event over a defined period for a given confidence
interval. It is based on the assumption that the possible outcome of the event is
represented by a normal distribution (bell curve). With a normal distribution, we know
that 95% of the results will lie within 1.96 standard deviations of the mean, and that 99%
of the results will lie within 2.57 standard deviations of the mean. Using this information,
we can predict what the range of results will be with a measured level of confidence.
Question #52: What is the effective annual interest rate on a $5 million loan with an interest rate
1
of 8%, a commitment fee of %, and a compensating balance of 10%?
4
Correct Answer: B. 9.17%.
A. 11.11%.
B. 9.17%.
C. 8.64%.
D. 8%.
Explanation:
The formula for effective annual rate of interest is:
PR +CF 0.0825
EI = = =9.17 %
1−CB 0.9
Where:
• EI = effective annual rate of interest
• PR = principal interest charge (%)
• CB = compensating balance (%)
• CF = commitment fee (%)

Question #53: The technique that measures the estimated performance of a capital investment by
dividing the project's annual after-tax net income by the average investment cost is called the
Correct Answer: D. Accounting rate of return method.
A. Bail-out payback method.
B. Internal rate of return method.
C. Profitability index method.
D. Accounting rate of return method.
Explanation:
Answer (A) is incorrect.
The bail-out payback method measures the length of the payback period when the periodic cash
inflows are combined with the salvage value.
Answer (B) is incorrect.
The internal rate of return method determines the rate at which the NPV is zero.
Answer (C) is incorrect
The profitability index is the ratio of the present value of future net cash inflows to the initial
cash investment.
Answer (D) is correct.
The accounting rate of return (also called the unadjusted rate of return or book value rate of
return) measures investment performance by dividing the accounting net income by the average
investment in the project. This method ignores the time value of money.

Question #54: The U.S. Foreign Corrupt Practices Act is particularly focused on the dealings of
financial institutions and the safeguarding of the global financial system. Financial institutions
must implement robust controls to ensure knowledge of their customers and the nature of their
business transactions and be in a position to prove to regulators a high level of due diligence.
These safeguards are required to minimize all of the following except
Correct Answer: B. Insider trading.
A. Money laundering.
B. Insider trading.
C. Terrorist financing.
D. Extortion and bribery.
Explanation:
Answer (A) is incorrect.
Money laundering is one focus of the safeguards of the global financial system relating to the
U.S. Foreign Corrupt Practices Act.
Answer (B) is correct.
The safeguards of the global financial system relating to the U.S. Foreign Corrupt Practices Act
deal with minimizing money laundering, terrorist financing, and extortion and bribery. Insider
trading is not a focus of the safeguards.
Answer (C) is incorrect.
Terrorist financing is one focus of the safeguards of the global financial system relating to the
U.S. Foreign Corrupt Practices Act.
Answer (D) is incorrect.
Extortion and bribery are focuses of the safeguards of the global financial system relating to the
U.S. Foreign Corrupt Practices Act

Question #55: Fact Pattern: A firm with an 18% desired rate of return is considering the
following projects (on January 1. Year 1):
January 1,
Year 1 Cash December 31,
Outflow Year 5 Cash Project
(000's Inflow (000's internal Rate
Omitted) Omitted) of Return
Project A $3,500 $7,400 16%
Project B 4,000 9,950 ?

Correct Answer: C. $(265.460)


A. $316,920
B. $23,140
C. $(265,460)
D. $(316,920)
Explanation:
Answer (A) is incorrect.
The amount of $316,920 discounts the cash inflow over a 4-year period.
Answer (B) is incorrect.
The amount of $23,140 assumes a 16% discount rate.
Answer (C) is correct.
The cash inflow occurs 5 years after the cash outflow, and the NPV method uses the firm's
desired rate of return of 18%. The present value of $1 due at the end of 5 years discounted at
18% is .4371. Thus, the NPV of Project A is $(265,460) [($7,400,000 cash inflow x .4371) -
$3,500,000 cash outflow].
Answer (D) is incorrect.
The amount of $(316,920) discounts the cash inflow over a 4-year period and also subtracts the
present value of the cash inflow from the cash outflow.
Question #56: Which of the following is not considered a weakness of ratio analysis?
Correct Answer: D. Markets change from year to year, and comparing ratios over time is
problematic
A. A firm in multiple industries has difficulty comparing its ratios to any one industry.
B. Firms may use different valuation methods on inventory (FIFO. LIFO. Weighted-
average, etc.).
C. Firms may use different depreciation methods (straight-line. double-declining balance.
etc.).
D. Markets change from year to year, and comparing ratios over time is problematic.
Explanation: While markets change from year to year, ratio analysis can still be valuable
in analyzing how the firm has changed with the markets. It is clear that conglomerates
have difficulty comparing ratios to a single industry and that accounting assumptions
affect financial statements and ratio analysis results.
Explanation: While markets change from year to year, ratio analysis can still be valuable in
analyzing how the firm has changed with the markets. It is clear that conglomerates have
difficulty comparing ratios to a single industry and that accounting assumptions affect financial
statements and ratio analysis results.

Question #57: When a company desires to increase the market value per share of common stock,
the company will implement:
Correct Answer: B. A reverse stock split.
A. A stock dividend.
B. A reverse stock split.
C. The sale of treasury stock.
D. A stock split.
Explanation:
A stock split does not affect the value of the firm. A reverse stock split would increase the
stock price in the same proportion as the decrease in the number of shares outstanding. For
example, a one-for-two split would double the price of the stock and decrease the number of
shares outstanding by one half.

Question #58: Which of the following is a benefit of international diversification?


Correct Answer: A. A company that is diversified internationally does not suffer as much from
adverse interest rate changes in only one country.
A. A company that is diversified internationally does not suffer as much from adverse
interest rate changes in only one country.
B. A company that is diversified internationally can choose in which country to pay income
taxes.
C. A company that is diversified internationally does not have to pay tariffs.
D. A company that is diversified internationally is able to employ more people.
Explanation:
A company that is diversified internationally lessens the risk in one country of adverse
interest rate changes or political upheavals. In some cases, they may save shipping costs,
such as a Japanese car maker opening a plant in another country so vehicles do not need to be
shipped. However, international diversification does not exempt any company from paying
tariffs, and a company cannot simply choose which country in which to pay income taxes.

Question #59: Which of the following are elements of earnings quality?


I. Management's discretion in choosing from among accepted accounting principles.
II. Management compensation in relation to net earnings.
III. The degree to which assets are maintained
IV. The effect of cyclical and other economic forces on the stability of earnings
Correct Answer: D. II and IV only.
A. I and III only.
B. I, III and IV only.
C. I, II, III and IV.
D. II and IV only.
Explanation: The basic factors of earnings quality are management and accountants' discretion in
choosing accounting principles, the degree to which maintenance of assets has been provided for,
and the effect of cyclical and other economic forces on the stability of earnings.

Question #60: Fact Pattern: Henderson. Inc.. has purchased a new fleet of trucks to deliver its
merchandise. The trucks have a useful life of 8 years and cost a total of $500.000. Henderson
expects its net increase in after-tax cash flow to be $150,000 in Year 1. $175,000 in Year 2.
$125,000 in Year 3, and $100,000 in each of the remaining years.
Question: Based on a 6% annual interest rate, what is the discounted payback period for
Correct Answer: C. 4.25 years.
A. 3.5 years.
B. 3.98 years.
C. 4.25 years.
D. 5.0 years.
Explanation:
Answer (A) is incorrect.
This number of years is the undiscounted payback period.
Answer (B) is incorrect.
This number of years results from not discounting the cash flows in the fourth year.
Answer (C) is correct.
The discounted payback period for an investment, assuming a 6% discount, can be found by
accumulating each year's discounted net cash flows until the initial investment is recovered.
$150,000 x .94339 = $141,508.50
175,000 x .88999 = 155,748.25
125,000 x .83962 = 104,952.50
100,000 x .79209 = 79,209.00
$481,418.25
Thus, the answer is something greater than four years. After four years, an additional $18,581.75
($500,000 - $481,418.25) is needed.
The calculation for the fifth year is $74,726 ($100,000 x .74726). Consequently, the discounted
payback period is approximately 4.25 years [4 + ($18,581.75 ÷ $74,726)].
Answer (D) is incorrect.
The full fifth year is not necessary.
Question #61: In evaluating a capital budget project, the use of the net present value (NPV)
model is generally not affected by the
Correct Answer: A. Method of funding the project.
A. Method of funding the project.
B. Initial cost of the project.
C. Amount of added working capital needed for operations during the term of the project.
D. Project's salvage value.
Explanation:
Answer (A) is correct
The NPV method computes the present value of future cash inflows to determine whether they
are greater than the initial cash outflow. Future cash inflows include any salvage value on
facilities. Included in the initial investment are the cost of new equipment and other facilities,
and additional working capital needed for operations during the term of the project. The discount
rate (cost of capital or hurdle rate) must be known to discount the future cash inflows. If the NPV
is positive, the project should be accepted. The method of funding a project is a decision separate
from that of whether to invest.
Answer (B) is incorrect.
The initial costs of the project are necessary to determine the NPV.
Answer (C) is incorrect
Additional working capital needs are necessary to determine the NPV.
Answer (D) is incorrect.
The project's salvage value is a future cash inflow to be discounted.

Question #62: A firm has been disappointed by previous capital budgeting decisions using the
payback method. A new requirement has been implemented that requires discounted cash flow
analysis to be used to compute the net present value (NPV) of proposed purchases over
$300,000. The Processing Department of the firm is considering the acquisition of a new
machine that will reduce labor costs by a pre-tax amount of $175,000 per year. Other
information regarding the possible acquisition is as follows:
 The machine will cost $450,000. Installation charges will amount to an additional
$25,000.
 The machine will have a useful life of 3 years, with no salvage value. Depreciation rates
for tax purposes are 25%. 38%, and 37% for Years 1, 2 and 3, respectively.
 The firm's cost of capital. 12%, is considered the appropriate discount rate.
 The income tax rate is 40%.
 Cash flows are assumed to occur at the end of the calendar year, which coincides with the
firm's fiscal year end.
Which of the following best indicates the net present value of the proposed investment and
the appropriate acquisition decision?
Correct Answer: B. Approximately $(73.000); recommend not making the investment.
A. Approximately $73,000; recommend making the investment.
B. Approximately $(73,000): recommend not making the investment.
C. Approximately $55,000; recommend making the investment.
D. Approximately $(55,000); recommend not making the investment.
Explanation:
Answer (A) is incorrect.
The correct NPV is negative, not positive, so the investment should not be recommended.
Answer (B) is correct.
The net initial investment in Year 0 will be $475,000 ($450,000 + $25,000 installation charges).
The annual pre-tax savings per year are stated as $175,000. Therefore, the annual after-tax
savings will be $105,000 [$175,000 x (1 - .40)]. The depreciation tax shield for each year is
calculated as follows:
Year 1: $475,000 x .25 = $118,750 x .40 = $47,500
Year 2: $475,000 x .38 = $180,500 x .40 = $72,200
Year 3: $475,000 x .37 = $175,750 x .40 = $70,300
Now solve for the NPV using the PV factors at 12%:
($475,000) + [($105,000 + $47,500) x .893] [($105,000 + $72,200) x .797] [($105,000 +
$70,300) x .712] = -$72,776
Because the NPV is negative, the firm should not make the investment.
Answer (C) is incorrect
The net initial investment in Year 0 will be $475,000 ($450,000 + $25,000 installation charges).
This answer choice incorrectly uses the $450,000 as the net initial investment in Year 0. This is
not correct as the $25,000 installation charges must be considered. Additionally, the correct NPV
is negative, not positive, so the investment should not be recommended.
Answer (D) is incorrect.
The net initial investment in Year 0 will be $475,000 ($450,000 + $25,000 installation charges).
This answer choice incorrectly uses the $450,000 as the net initial investment in Year 0. This is
not correct as the $25,000 installation charges must be considered. Even though the investment
should not be recommended, this is not the correct NPV.

Question #63: Kator Co. is a manufacturer of industrial components. One of their products that is
used as a subcomponent in auto manufacturing is KB-96. This product has the following
financial structure per unit:

Selling price $150


Direct materials $20
Direct labor 15
Variable manufacturing
overhead 12
Fixed manufacturing overhead 30
Shipping and handling 3
Fixed selling and administrative 10
Total costs $90

During the next year, KB-96 sales are expected to be 10,000 units. All of the costs will remain
the same except that fixed manufacturing overhead will increase by 20% and direct materials
will Increase by 10%. The selling price per unit for next year will be $160. Based on this data,
the contribution margin from KB-96 for next year will be
Correct Answer: D. $1,080,000
A. $620,000
B. $1,110,000
C. $750,000
D. $1,080,000
Explanation:
A. This answer results from two errors: 1. The direct materials cost per unit was not
increased to reflect the 10% increase expected for the next year. 2. A fixed cost value of
$480,000 was deducted, calculated by multiplying the current years fixed manufacturing
cost per unit and fixed selling and administrative costs per unit ($40 in total) by the next
year’s forecasted sales of 10,000 units, and then increasing the resulting amount by 20%.
The question asks for the expected contribution margin for the next year. The
contribution margin is sales less all variable costs. It does not include fixed costs, so fixed
costs should not be deducted. It should be noted also that there are three problems with
trying to calculate next year's forecasted fixed costs from the information given.
1. It is not possible to calculate a forecasted amount for fixed costs from the information
given. The current year's fixed costs per unit are derived from the current year's total
fixed costs divided by the current year’s activity. For example, to calculate the current
years fixed manufacturing costs, it would be necessary to know the current year’s
production level. The current year's fixed manufacturing costs could be "backed into
by multiplying the current year's per unit fixed manufacturing cost by the current
year's production volume, if the current year's production volume were known. Since
the current year's production volume is not given, it is not possible to calculate the
current year's fixed manufacturing costs. Therefore, it is also not possible to
determine forecasted fixed manufacturing costs for next year of 20% more than this
years fixed manufacturing costs.
2. The fixed selling and administrative cost per unit is also a derived amount. It is
derived from the total fixed selling and administrative costs divided by the total sales
in units for the current year. Since the total sales in units for the current year is not
given, it is not possible to calculate total fixed selling and administrative costs for the
current year, so those costs cannot be used as a forecast for fixed selling and
administrative costs for the next year.
3. Furthermore, the fixed selling and administrative costs are not expected to increase
next year, so including them in the amount that was increased by 20% for next year is
incorrect.
B. This answer results from omitting the shipping and handling cost from the calculation of
the contribution margin. Shipping and handling costs are variable costs and should be
deducted in the calculation of the contribution margin.
C. This answer results from two errors:
1. The shipping and handling cost was omitted from the calculation of the contribution
margin. The contribution margin is sales less all variable costs, and shipping and
handling cost is a variable cost that should be deducted in calculating the contribution
margin for the next year.
2. A fixed cost value of $360,000 was deducted, calculated by multiplying the current
year's fixed manufacturing cost of $30 per unit by the next year's forecasted sales of
10,000 units, and then increasing the resulting amount by 20%. The question asks for
the expected contribution margin for the next year. The contribution margin is sales
less all variable costs. It does not include fixed costs, so fixed manufacturing costs
should not be deducted. It should be noted also that it is not possible to calculate a
forecasted amount for fixed manufacturing costs from the information given. The
current year's fixed manufacturing costs per unit are derived from the current year's
total fixed manufacturing costs divided by the current year's production. It would be
necessary to know the current year's production level in order to determine the current
year's fixed manufacturing costs. The current year's fixed manufacturing costs could
be backed into" by multiplying the current year's per unit fixed manufacturing cost by
the current year's production volume, if the current year's production volume were
known. Since the current year's production volume is not given, it is not possible to
calculate the current year's fixed manufacturing costs. Therefore, it is also not
possible to determine forecasted fixed manufacturing cost for next year of 20% more
than this year's fixed manufacturing costs.
D. The unit contribution margin formula is sales price per unit - all variable costs per unit.
The sales price next year will be $160. Variable cost include direct material, direct labor,
variable overhead and shipping & handling. Direct material next year will be ($20 x 10%
increase in cost) = $22. Direct labor = $15. Variable overhead = $12. Shipping and
Handling = $3. Total variable cost = $52. The unit contribution margin = $160 - $52 =
$108. The company expects to sell 10,000 units during the year. Therefore, the total
contribution margin for the next year is expected to be $108 x 10,000, or $1,080,000.

Question #64: A primary benefit a firm expects to gain from lengthening a credit period is:
Correct Answer: B. Increased profitability resulting from increased sales.
A. Improved inventory control resulting from relatively stable demand.
B. Increased profitability resulting from increased sales.
C. Fewer collections procedures necessary for past due accounts.
D. Increased revenue from interest charges on past due accounts.
Explanation:
A firm extends a credit period with the expectation of increased profitability from increased
sales.

Question #65: Maydale Inc:s financial statements show the following information.
Accounts receivable, end of Year 1 $ 320,000
Credit sales for Year 2 3,600,000
Accounts receivable, end of Year 2 400,000
Maydale's accounts receivable turnover ratio is:
Correct Answer: D.10.
A. 11.25.
B. 0.10.
C. 9.
D. 10.
Explanation:
The accounts receivable turnover per year is calculated as:
Accounts receivable turnover per year = (Net credit sales for year) / (average accounts receivable
balance for the year)
Average accounts receivable balance for the year = (beginning balance + ending balance) / 2
Average accounts receivable balance for the year = ($320,000 + $400,000) / 2 = $720,000/ 2 =
$360,000
Turnover per year = $3,600,000 / $360,000 = 10 times.

Question #66: Gleason Co. has two products, a frozen dessert and ready-to-bake breakfast rolls,
ready for introduction. However, plant capacity is limited, and only one product can be
introduced at present. Therefore. Gleason has conducted a market study, at a cost of $26,000, to
determine which product will be more profitable. The results of the study follow.
Sales of desserts at $1.80/unit Sales of rolls at $1.20/unit

Volume Probability Volume Probability


250,000 0.30 200,000 0.20
300,000 0.40 250,000 0.50
350,000 0.20 300,000 0.20
400,000 0.10 350,000 0.10

The costs associated with the two products have been estimated by Gleason's cost accounting
department and are shown as follows

Desserts Rolls
Ingredients per unit $0.40 0.25
Direct labor per unit 0.35 0.30
Variable overhead per
unit 0.40 0.20
Production tooling* 48,000 25,000
Advertising 30,000 20,000

'Gleason treats production tooling as a current operating expense rather than capitalizing it as a
fixed asset According to Gleason's market study, the expected value of the sales volume of the
breakfast rolls is
Correct Answer: B. 260,000 units.
A. Some amount other than those given.
B. 260,000 units.
C. 275,000 units.
D. 125,000 units.
Explanation:
A. The correct answer is one of the answer choices given.
B. Expected sales volume is a weighted average of the possible sales volume, weighted
according to their probabilities. To find the expected sales volume, multiply each
estimated sales volume by its probability of occurring and sum the results. (200,000 x
0.2) + (250,000 x 0.5) + (300,000 x 0.2) + (350,000 x 0.1) = 260,000.
Expected sales volume is 260,000 units.
C. This calculation represents the average of the possible sales volumes of the rolls. The
expected value is a weighted average with the various probabilities of each sales volume
as the weights.
D. The answer of 125,000 was calculated by using the estimated sales volume of 250,000
times its probability of 0.5. This is not the expected value of the sales volume of rolls.

Question #67: A European company provides annual reports for U.S. investors purchasing ADRs
of the company's stock in the United States. The company reports €1,500,000 net income. The
exchange rate between the euro and the U.S. dollar is €1.19/$1. Which of the following
statements is true?
Correct Answer: C. Annual statements sent to U.S. investors will show net income as
€1,500,000.
A. Annual statements sent to U.S. investors will show net income as $1,260,504.
B. Annual statements sent to U.S. investors will show net income as $1,500,000.
C. Annual statements sent to U.S. investors will show net income as €1,500,000.
D. Annual statements sent to U.S. investors will show net income as $1,785,000.
Explanation:
Financial statements generally do not make adjustments for foreign currency exchange rates, as
this would show wild fluctuations due to the exchange rate rather than company performance.

Question #68: A long-term call option to buy common stock directly from a corporation is a:
Correct Answer: D. Warrant.
A. Forward contract.
B. Convertible security.
C. Futures contract.
D. Warrant.
Explanation:
By definition, a warrant is a long-term call option to buy common stock directly from a
corporation. It gives bond or preferred stockholders the right to purchase shares of common
stock at a given price.

Question #69: Which of the following is an example of a horizontal merger?


Correct Answer: D. Two or more firms who are within the same market. Al Possible Answers:
A. Two or more firms who work with each other in offering varying levels of the production
process.
B. Two or more firms who are in separate markets.
C. Two or more firms who work with each other as supplier and producer.
D. Two or more firms who are within the same market.
Explanation:
A horizontal merger happens when two or more firms within the same market, also referred to as
competitors, join together. When a horizontal merger occurs, fewer competitors in the market
result; thus having the potential of leading towards a monopolistic circumstance.

Question #70: When determining the amount of dividends to be declared, the most important
factor to consider is the:
Correct Answer: A. future planned uses of cash.
A. Future planned uses of cash.
B. Impact of inflation on replacement costs.
C. Future planned uses of retained earnings.
D. Expectations of the shareholders.
Explanation:
When determining the amount of dividends to be declared, the most important factor to consider
is the future planned uses of cash.

Question #71: Suppose a company has three foreign subsidiaries: Subsidiary A is located in a
country with a 40% corporate tax rate. Subsidiary B is located in a country with a 30% corporate
tax rate, and Subsidiary C is located in a country with a 35% corporate tax rate. If allowed by
relevant laws, how would the company improve its combined after-tax earnings using transfer
pricing?
Correct Answer: A. Reduce the price that Subsidiary A charges to Subsidiary B.
A. Reduce the price that Subsidiary A charges to Subsidiary B.
B. Changes in transfer pricing policy have no impact on combined after-tax earnings.
C. Increase the price that Subsidiary A charges to Subsidiary B.
D. Reduce the price that Subsidiary B charges to Subsidiary C
Explanation:
Since the tax rate in the country Subsidiary A operates in is 40% and is higher than the tax
rate in the country Subsidiary B operates in (30%). the transfer price should be set so as to be
taxed in the country that Subsidiary B operates in. Therefore, the transfer price should be set
low (reduced). The price is a taxable revenue in the country Subsidiary A operates in: it is a
deductible expense in the country Subsidiary B operates in.

Question #72: A large multinational company currently has its information technology
department located in Germany. In order to reduce the risk of system failure, the company
has decided to split up the information technology department into two geographically
separate locations and set up a new location in Singapore The company can still face a
catastrophic system failure, but the risk will be greatly reduced. The risk that remains after
the company sets up the second information technology department in Singapore is best
described as
Correct Answer: D. residual risk.
A. Hazard risk.
B. Inherent risk.
C. Business risk.
D. Residual risk.
Explanation:
A. Hazard risk is the type of risk that is can be insured against. Common examples are
natural disasters (property insurance), death of a key employee (key person life
insurance), personal injury that takes place on the premises of the business (liability
insurance), and any other unexpected event that can be insured against. The risk that
remains after the company sets up the second information technology department in
Singapore is not hazard risk
B. Inherent risk is the level of risk in each event before any mitigation action is taken. Since
action has been taken to mitigate the risk by splitting up the information technology
department into two geographically separate locations, the risk remaining is not inherent
risk
C. Business risk for a firm is the risk of changes in its earnings before interest or taxes.
Business risk depends on a variety of factors, including the variability of demand, sales
price, and the price of inputs as well as the amount of the company's operating leverage.
The more stable all of these variables are, the less business risk a company will
experience. The risk that remains after the company sets up the second information
technology department in Singapore is not business risk
D. Residual risk is the level of risk that remains after management has taken action to
mitigate the risk. Thus the risk that remains after the company sets up the second
information technology department in Singapore is best described as residual risk.

Question #73: Pearl Manufacturing Company has sales of $1,000,000, variable costs of
$500,000, and fixed costs of $250,000. In addition. Pearl has interest payments of $50,000,
preferred stock dividends of $60,000, and faces an effective tax rate of 40%. What is the Degree
of Financial Leverage (DFL)?
Correct Answer: A. 1.25.
A. 1.25.
B. 2.5.
C. 1.5.
D. 1.35.
Explanation:
The Degree of Financial Leverage is computed as follows: DFL = EBIT/EBT. Placing the
numbers into the formula. DFL = $(1,000,000 - $500,000 - $250,000) / $(1,000,000 - $500,000 -
$250,000 - $50,000), which equals 1.25. A DFL of 1.25 implies that every 1% of increase in
operating income will result in a 1.25% increase in Earnings per share.

Question #74: If Dexter Industries has a beta value of 1.0, then its:
Correct Answer: C. expected return should approximate the overall market.
A. Price is relatively stable.
B. Volatility is low.
C. Expected return should approximate the overall market.
D. Return should equal the risk-free rate.
Explanation:
Beta is a measure of the movement of the price of a particular stock compared with the
movement of the market as a whole during the same period. Therefore, a beta of 1 would imply
the expected return should approximate the overall market.

Question #75: A firm's total assets are $10,000,000, its total liabilities are $4,000,000, its current
assets are $2,800,000 million ($600,000 cash, $1,000,000 money market investments, $700,000
inventory, and $500,000 receivables), its current liabilities are $900,000 ($400,000 accounts
payable, $500,000 notes payable), and its equity is $6,000,000. What is the firm's cash ratio?
Correct Answer: D. 1.78.
A. 2.5.
B. 3.11.
C. 1.5.
D. 1.78.
Explanation:
The cash ratio is cash and cash equivalents and marketable securities divided by current
liabilities. ($600,000 cash + $1,000,000 money market) ÷ $900,000 = $1,600,000/$900,000 =
1.78.

Question #76: Fact Pattern: Stennet Company is considering two mutually exclusive projects.
The net present value (NPV) profiles of the two projects are as follows:
Discount rate Project A Project B
(%) (NPV) (NPV)
0 $2,220 $1,240
10 681 507
12 495 411
14 335 327
16 197 252
18 77 186
20 (26) 128
22 (115) 76
24 (193) 30
26 (260) (11)
28 (318) (47)

Question: The approximate internal rates of return for Projects A and B, respectively, are
Correct Answer: C. 19.5% and 25.5%.
A. 0% and 0%.
B. 19.0% and 21.5%.
C. 19.5% and 25.5%.
D. 20.5% and 26.5%.
Explanation:
Answer (A) is incorrect.
Zero is the net present value, not the discount rate, associated with the internal rate of return.
Answer (B) is incorrect.
The internal rate of return for Project B is between 24% and 26%.
Answer (C) is correct.
A projects internal rate of return is the discount rate at which the net present value of its cash
flows equals zero. For Project A this is somewhere between 18% and 20%, and for Project B. it
is between 24% and 26%.
Answer (D) is incorrect.
The internal rate of return of Project A is less than 20% and that for Project B is less than 26%.

Question #77: Many countries have eliminated double taxation of foreign income as a way
to encourage multinational firms to undertake foreign direct investment. As a result,
multinational firms no longer have to worry about:
Correct Answer: B. Taxation of income in bath the host country end the home country.
A. Taxation that must be paid to bath the host country end the International Monetary
Fund (IMF)
B. Taxation of income in bath the host country end the home country.
C. Double taxation of income in the host country.
D. Double taxation of income in the home country.
Explanation:
A company's foreign income is taxed by either the home country or the host country, but
not by both.
Question #78: The basis for the components of an Enterprise Risk Management system
as defined by COSO in Enterprise Risk Management - Integrated Framework is
Correct Answer: B. The internal environment of the organization.
A. The organization's control activities.
B. The internal environment of the organization.
C. Assessment of risk.
D. Identification of the organization's strategic objectives.
Explanation:
A. Control activities are the policies and procedures implemented to ensure that risk
responses are effectively implemented. They are not the basis for the other
components.
B. The internal environment of the organization-the attitude in the organization toward
risk and risk management - is the basis for ell the other components of an Enterprise
Risk Management (ERM) system.
C. Risk assessment - the process of analyzing risks from the perspective of the
likelihood of the risk's occurring end the potential impact of the event if it does occur
- is the core of an Enterprise Risk System (ERM). However, it is not the basis for the
other components.
D. Identification of the organization's strategic objectives is an important component of
an Enterprise Risk Management (ERM) system. However, it is not the basis for the
other components.

Question #79: The cross-rates for foreign exchange are shown below.

FCU 1 FCU 2
FCU 3 4 1.60
FCU 2 2 1.00

FCU = foreign currency unit


These spot rates present an opportunity for:
Correct Answer: A. Triangular arbitrage.
A. Triangular arbitrage.
B. Exchange rate hedging.
C. Purchasing power parity.
D. Interest rate parity.
Explanation:
Triangular or triangle arbitrage involves taking advantage of the imbalance among three foreign
exchange markets. A combination of deals is made that exploits the imbalance, resulting in a
profit.

Question #80: In which of the following circumstances would financial leverage be likely to
increase?
Correct Answer: A. Firm issues bonds to repurchase some of its own common stock
A. Firm issues bonds to repurchase some of its own common stock
B. Firm purchases assets with cash.
C. Firm signs a contract to rent a new manufacturing site.
D. Firm accepts a large order from a new customer. Excess capacity exists to fill the order.
Explanation:
Financial leverage relates to the fixed financing costs of a firm. Of the choices given, issuing
new bonds is the only one that affects fixed financing costs.

Question #81: Underhall Incas common stock is currently selling for $108 per share. Underhall
is planning a new stock issue in the near future and would like to stimulate interest in the
company. The Board, however, does not want to distribute capital at this time. Therefore.
Underhall is considering whether to offer a 2-for-1 common stock split or a 100% stock dividend
on its common stock. The best reason for opting for the stock split is that:
Correct Answer: B. it will not impair the company's ability to pay dividends in the future.
A. The impact on earnings per share will not be as great
B. It will not impair the company's ability to pay dividends in the future.
C. It will not decrease shareholders' equity.
D. The par value per share will remain unchanged.
Explanation:
The best reason for opting for the stock split is that it will not impair the company's ability to pay
dividends in the future.

Question #82: A firm's total assets are $10,000,000, its total liabilities are $4,000,000, its current
assets are $2,800,000 ($600,000 cash. $1,000,000 money market investments. $700,000
inventory, and $500,000 receivables), its current liabilities are $900,000 ($400,000 accounts
payable. $500,000 notes payable), and its equity is $6,000,000. Its operating cash flow is
$1,500,000. What is the firm's cash flow ratio?
Correct Answer: C. 1.67.
A. 1.5.
B. 3.75.
C. 1.67.
D. 3.
Explanation:
Cash flow ratio = Operating Cash Flow/ Current Liabilities = $1,500,000/ $900,000 = 1.67.

Question #83: In the 20X3 fiscal year. Newman Manufacturing's gross profit margin remained
unchanged from the 20X2 fiscal year. But, in 20X3, the company's net profit margin declined
from the level reached in 20X2. This could have happened because in 20X3:
Correct Answer: C. corporate tax rates increased.
A. Common share dividends increased.
B. Cost of goods sold increased relative to sales.
C. Corporate tax rates increased.
D. Sales increased at a faster rate than operating expenses.
Explanation:
Increasing taxes will reduce net income after tax and thus the net profit margin.

Question #84: BDU Company has net income of $500,000 and average assets of $2,000,000 for
the current year. If its asset turnover is 1.25 times, then what is its profit margin?
Correct Answer: D.0.2.
A. 0.31.
B. 0.25.
C. 0.36.
D. 0.2.
Explanation:
Profit Margin = Net Income/Sales
Calculate Sales by rearranging the following formula:
Asset Turnover = Sales / Assets
Sales = (Asset Turnover) (Assets)
Sales = (1.25) ($2,000,000) = $2,500,000
Profit Margin = $500,000 / $2,500,000 = 0.2.

Question #85: A company uses the internal rate of return {IRR) method to evaluate capital
projects. The company is considering four independent projects with the following IRRs:
Project IRR
I 10%
II 12%
III 14%
IV 15%
The company's cost of capital is 13%. Which one of the following project options should the
company accept based on IRR?
Correct Answer: B. Projects II I and IV only.
A. Projects I and II only.
B. Projects III and IV only.
C. Project IV only.
D. Projects I, II, III and IV.
Explanation:
Answer (A) is incorrect.
Projects I and II have rates of return lower than the company's cost of capital.
Answer (B) is correct.
When sufficient funds are available, any capital project whose internal rate of return (IRR)
exceeds the company's cost of capital should be accepted.
Answer (C) is incorrect
The rate of return for Project III also exceeds the company's cost of capital.
Answer (D) is incorrect.
Projects I and II should be rejected; their rates of return are lower than the company's cost of
capital.

Question #86: Which of the following is not a benefit of risk management?


Correct Answer: B. Lower taxes.
A. Better contingency planning.
B. Lower taxes.
C. Improved cost control.
D. Increased shareholder value.
Explanation:
A. Contingency planning is the development of a "backup plan;' or a plan that enables the
organization to respond appropriately to an unplanned event. It is used for emergency
response and post-disaster recovery. Contingency planning is particularly important in
information systems in order to insure that critical systems resources will be available
and operations can continue during an emergency situation. Better contingency planning
is a benefit of risk management because risk assessment and risk management enable
management to be aware of and prepared for more possible events that could prevent
them from achieving their objectives.
B. Lower taxes do not result from risk management, so they cannot be a benefit of risk
management.
C. Cost control is improved as a result of risk management because there are fewer
unpleasant surprises and fewer disruptions in the operations of the business.
D. Increased shareholder value is a benefit of risk management because risk management
can minimize losses and maximize opportunities.

Question #87: IMA's Statement on Management Accounting. "Values and Ethics: From
Inception to Practice recommends a defined code of conduct and ethical behavior for all
organizations. One advantage of having such a code is that it
Correct Answer: A. Provides employees with guidance for handling unfamiliar situations.
A. Provides employees with guidance for handling unfamiliar situations.
B. Ensures ethical behavior by all employees
C. Shields the organization from liability in cases of loss of stockholder value due to fraud.
D. Eases the investigative process performed by police and prosecutors in cases of suspected
fraud.
Explanation:
Answer (A) is correct
"Values and Ethics: From Inception to Practice states, in part."...what does an employee do when
unplanned events occur? What reference does an individual look to for help in making
decisions?...This is why it is important to have a defined set of organizational values and code of
ethics - they create the "touchstone against which every unanticipated decision must be judged.
Failure to have every individual in the organization know and understand these values and
ethical code leads to inconsistency and. in the worst cases, unethical or fraudulent behavior."
(IV. Values. Ethics, and Accounting.)
Answer (B) is incorrect.
A code of conduct cannot guarantee ethical behavior by employees.
Answer (C) is incorrect
A code of conduct cannot guarantee that an organization will be shielded from liability in cases
of fraud.
Answer (D) is incorrect.
A code of conduct does not ease law enforcement's investigative process.

Question #88: A new management accountant is concerned about complying with the ethical
standard of competence in the I MA's Statement of Ethical Professional Practice. Which one of
the following is not required under the standard of competence?
Correct Answer: A. Maintain expertise in all areas of accounting.
A. Maintain expertise in all areas of accounting.
B. Continually develop knowledge and skills.
C. Perform duties in accordance with relevant regulations and standards.
D. Provide recommendations that are accurate and timely.
Explanation:
Answer (A) is correct
Maintaining expertise in all areas of accounting would be a difficult task. According to the
ethical standard of competence in the IMAs Statement of Ethical Professional Practice, a CMA
only needs to recognize and communicate professional limitations or other constraints that would
preclude responsible judgment or successful performance of an activity.
Answer (B) is incorrect.
According to the ethical standard of competence in the IMA's Statement of Ethical Professional
Practice, a CMA should maintain an appropriate level of professional expertise by continually
developing knowledge and skills.
Answer (C) is incorrect.
According to the ethical standard of competence in the IMA's Statement of Ethical Professional
Practice, a CMA should perform professional duties in accordance with relevant laws,
regulations, and technical standards.
Answer (D) is incorrect.
According to the ethical standard of competence in the IMA's Statement of Ethical Professional
Practice, a CMA should provide decision support information and recommendations that are
accurate, clear, concise, and timely.

Question #89: Firms with high degrees of financial leverage would be best characterized as
having:
Correct Answer: C. high debt-to-equity ratios.
A. High fixed-charge coverage.
B. Zero coupon bonds in their capital structures.
C. High debt-to-equity ratios.
D. Low current ratios.
Explanation:
Financial leverage is the use of debt (fixed cost funds) to increase returns to owners
(stockholders). A high degree of financial leverage means the benefits from tax-deductibility of
interest (from additional debt) is more than offset by the increase in the payments to repay the
debt.

Question #90: A firm is constructing a risk analysis to quantify the exposure of its data center to
various types of threats. Which one of the following situations would represent the highest
annual loss exposure after adjustment for insurance proceeds?
Frequency of Occurrence Loss Insuranc
(years) Amount e
I 1 $15,000 85
II 8 75,000 80
III 20 200,000 80
IV 100 400,000 50

Correct Answer: D. I.
A. II.
B. IV.
C. III.
D. I.
Explanation:
A. The question asks for the highest annual loss exposure after adjustment for insurance
proceeds. The way to calculate that is to (1) calculate the loss after insurance
reimbursement for each situation by multiplying the loss amount by (1 - insurance
coverage rate), and (2) divide each loss after reimbursement by the frequency of
occurrence in years to calculate the annual loss amount for each.
B. The question asks for the highest annual loss exposure after adjustment for insurance
proceeds. The way to calculate that is to (1) calculate the loss after insurance
reimbursement for each situation by multiplying the loss amount by (1- insurance
coverage rate), and (2) divide each loss after reimbursement by the frequency of
occurrence in years to calculate the annual loss amount for each.
C. The question asks for the highest annual loss exposure after adjustment for insurance
proceeds. The way to calculate that is to (1) calculate the loss after insurance
reimbursement for each situation by multiplying the loss amount by (1- insurance
coverage rate), and (2) divide each loss after reimbursement by the frequency of
occurrence in years to calculate the annual loss amount for each.
D. The question asks for the highest annual loss exposure after adjustment for insurance
proceeds. The way to calculate that is to (1) calculate the loss after insurance reimbursement
for each situation by multiplying the loss amount by (1 - insurance coverage rate). and (2)
divide each loss after reimbursement by the frequency of occurrence in years to calculate the
annual loss amount for each, as follows:
Frequenc Loss
y of Loss Insurance after
Occurrenc Amoun (%coverag ins. Annual
e (years) t e) Reimb. loss
$15,00
I 1 0 85 $2,250 $2,250
II 18 75,000 80 15,000 1,875
200,00
III 20 0 80 40,000 2,000
400,00 200,00
IV 100 0 50 0 2,000

The highest annual loss is 1, with an annual loss exposure of $2,250.


Question #91: Protective clauses set forth in an indenture are known as:
Correct Answer: B. Covenants.
A. Requirements.
B. Covenants.
C. Addenda.
D. Provisions.
Explanation:
Protective covenants set limits (restrictions) on certain actions the company might be taking
during the term of the agreement. They are a particularly important feature in a bond indenture.

Question #92: In discounted cash flow techniques, which one of the following alternatives best
reflects the items to be incorporated in the initial net cash investment?
Correct Answer: D. Yes (Capitalized expenditures (e.g.. shipping costs)). (Changes in net
working capital) Yes. Yes (Net proceeds from sale of old asset in a replacement decision).
(Impact of spontaneous changes in current liabilities ) Yes
A. No (Capitalized expenditures (e.g.. shipping costs)). (Changes in networking capital )
Yes. Yes (Net proceeds from sale of old asset in a replacement decision). ( Impact of
spontaneous changes in current liabilities ) Yes
B. Yes (Capitalized expenditures (e.g.. shipping costs)). (Changes in net working capital )
No. No (Net proceeds from sale of old asset in a replacement decision). ( Impact of
spontaneous changes in current liabilities )No
C. No (Capitalized expenditures (e.g.. shipping costs)). (Changes in networking capital )
Yes. No (Net proceeds from sale of old asset in a replacement decision). ( Impact of
spontaneous changes in current liabilities ) No
D. Yes (Capitalized expenditures (e.g.. shipping costs)). (Changes in net working capital )
Yes. Yes (Net proceeds from sale of old asset in a replacement decision). ( Impact of
spontaneous changes in current liabilities ) Yes
Explanation:
Answer (A) is incorrect.
Capitalized expenditures must be considered.
Answer (B) is incorrect.
Changes in working capital and proceeds from the sale of existing assets must be considered.
Answer (C) is incorrect.
Changes in working capital are not the only items that must be considered.
Answer (D) is correct.
All four of the items listed must be incorporated in a DCF analysis of a capital project.

Question #93: Devlin Company's acid-test ratio at May 31. Year 2, was:
Devlin Company
Statement of Financial Position as of May 31
(In thousands)

Assets Year 2 Year 1


Current assets
Cash $45 $38
Trading securities 30 20
Accounts receivable (net) 68 48
Inventory 90 80
Prepaid expenses 22 30
Total current assets 255 216
Investments, at equity 38 30
Property, plant & equipment (net) 375 400
Intangible asset (net) 80 45
Total assets $748 $691
Liabilities and shareholders' equity
Current liabilities
Notes payable $35 $18
Accounts payable 70 42
Accrued expenses 5 4
Income taxes payable 15 16
Total current liabilities 125 80
Long term debt 35 35
Deferred taxes 3 2
Total liabilities 163 117
Shareholders' equity
Preferred stock, 6%, $100 par value,
cumulative 150 150
Common stock, $10 par value 225 195
Additional paid-in capital--common stock 114 100
Retained earnings 96 129
Total shareholders' equity 585 574
Total liabilities and shareholders' equity $748 $691

Devlin Company
Income statement for the Year Ended May 31
(In thousands)

Year 2 Year 1
Net sales $480 $460
Costs and expenses
Cost of goods sold 330 315
Selling, general, and
administrative 52 51
Interest before taxes 8 9
Income before taxes 90 85
Income taxes 36 34
Net income $54 $51

Correct Answer: D. 1.14 to 1.


A. 2.04 to 1.
B. 1.86 to 1.
C. 0.60 to 1.
D. 1.14 to 1.
Explanation: The acid-test ratio (also known as the quick ratio) is the ratio of quick assets to
current liabilities. Quick assets are those easily converted to cash without significant loss. The
quick assets are cash, short-term investments (trading securities), and net receivables.
Devlin's acid-test ratio for Year 2 is calculated:
Acid-test ratio = (cash + marketable securities + net receivables) / current liabilities
Acid-test ratio = ($45 cash + $30 trading securities + $68 accounts receivable (net)) / ($125
current liabilities)
Acid-test ratio = $143 / $125 = ratio of 1.14 to 1.
Question #94: A steady drop in a firm's price/earnings (P/Es) ratio could indicate that:
Correct Answer: C. earnings per share has been increasing while the market price of the stock
has held steady.
A. Both earnings per share and the market price of the stock are rising.
B. Earnings per share has been steadily decreasing.
C. Earnings per share has been increasing while the market price of the stock has held
steady.
D. The market price of the stock has been steadily rising.
Explanation:
The P/Es ratio is the price of the stock divided by the earnings per share.

Question #95: Starlight Theater stages a number of summer musicals at its theater in northern
Ohio. Preliminary planning has just begun for the upcoming season, and Starlight has developed
the following estimated data.
Number of Average Attendance per Ticket Variable Fixed
Production performances performance price costs costs
Mr. $165,00
Wonderful 12 3,500 $18 $3 0
That's Life 20 3,000 15 1 249,000
All that Jazz 12 4,000 20 0 316,000

Represent payments to production companies and are based on tickets sold. :


Costs directly associated with the entire run of each production for costumes, sets, and artist fees.
Starlight will also incur $565,000 of common fixed operating charges (administrative overhead.
facility costs, and advertising) for the entire season, and is subject to a 30% income tax rate. If
Starlight's schedule of musicals is held as planned, how many patrons would have to attend for
Starlight to break even during the summer season?
Correct Answer: C. 79,938.
A. 79,302.
B. 81,390.
C. 79,938.
D. 77,918.
Explanation:
A. This answer results from using a simple average (un-weighted) of the unit contribution
margins for the three performances to calculate the breakeven volume. The average unit
contribution margin used should be a weighted unit contribution margin. The weighting
should be based on total attendance at the performances, which incorporates both the
number of performances of each musical and the average attendance per performance.
B. This answer results from using the number of performances of each production to
calculate the weights for the weighted average unit contribution margin. The weighting
should be based on total attendance at the performances, which incorporates both the
number of performances of each musical and the average attendance per performance.
C. This is a break-even analysis when more than one product is sold.
The weighted average unit contribution margin is based on total attendance, as follows:
# Avg. Att.
Production Perf Per Perf.
Mr. Wonderful 12 x 3,500 =
That's Life 20 x 3,000 =
All That Jazz 12 X 4,000 =
Total % of
Attendance Total UCM
42,000 0.28 $15
60,000 0.40 14
48,000 0.32 20
150,000
The weighted average unit contribution margin is: (0.28 x $15) + (0.40 x $14) + (0.32 x
20) = $16.20
Total fixed costs are $730,000 + $565,000, or $1,295,000.
Therefore, the break-even point in total units for this 3-product firm is: $1,295,000 ÷
$16.20, which is 79.938.
D. This answer results from using the average attendance per performance to calculate the
weights for the weighted average unit contribution margin. The weighting should be
based on total attendance at the performances, which incorporates both the number of
performances of each musical and the average attendance per performance.

Question #96: A firm is evaluating six investment opportunities (projects). The following table
reflects each project's net present value (NPV) and the respective initial investments required. All
of these projects are independent.
Investme
Project NPV nt
$5,00
R 0 $10,000
S 5,000 5,000
T 8,000 40,000
15,00
U 0 60,000
15,00
V 0 75,000
W 3,000 15,000

Lewis has an investment constraint of $100,000. Which combination of projects would represent
the optimal investment that should be recommended to Lewis Services management.
Correct Answer: A. R, S, U and W.
A. R, S, U and W.
B. R, V and W.
C. R, S and V.
D. T and U.

Explanation:
Answer (A) is correct.
The profitability lit is an efficient means for ranking capital projects. It is the ratio of the net
present value of a project's cash-flows to the net investment. The indexes for these projects can
be calculated as follows:
Investme Profitabilit
Project NPV nt y Index
$5,00
R 0 $10,000 0.50
S 5,000 5,000 1.00
T 8,000 40,000 0.20
15,00
U 0 60,000 0.25
15,00
V 0 75,000 0.20
W 3,000 15,000 0.20

They can now be ranked in order of desirability, and those projects fitting within the $100,000
capital constraint can be selected. The projects with the three highest profitability indexes, S, R
and U, can all be undertaken ($5,000 + $10,000 + $60,000 = $75,000). One more project from
among the others can be undertaken if its initial investment is less that $25,000 ($100,000 -
$75,000). Project W fits this criterion.
Answer (B) is incorrect.
The combination of Projects R, V and W (total NPV of $23,000) is not the most profitable use of
the $100,000 investment constraint.
Answer (C) is incorrect
The combination of Projects R, S and V (total NPV of $25,000) is not the most profitable use of
the $100,000 investment constraint.
Answer (D) is incorrect.
The combination of Projects T and U (total NPV of $13,000) is not the most profitable use of the
$100,000 investment constraint.

Question #97: Which of the following statements describes an option that is in-the-money?
Correct Answer: A. Payment must be made to the owner if the contract is exercised.
A. Payment must be made to the owner if the contract is exercised.
B. The owner of the contract decides not to sell the underlying asset.
C. The strike price exceeds the price of the underlying asset.
D. The owner can exercise the option at any time before maturity.
Explanation:
Different payoffs are possible with options. An option generally referred to as being in-the-
money requires immediate payment to the owner if the contract is exercised.

Question #98: Lazar Industries produces two products. Crates and Boxes. Per unit selling prices,
costs, and resource utilization for these products are as follows.

Crates Boxes
Selling price $20 $30
Direct material costs $5 $5
Direct labor costs 8 10
Variable overhead
costs 3 5
Variable selling costs 1 2
Machine hours per
unit 2 4

Production of Crates and Boxes involves joint processes and use of the same facilities. The total
fixed factory overhead cost is $2,000,000 and total fixed selling and administrative costs are
$840,000 Production and sales are scheduled for 500,000 units of Crates and 700,000 units of
Boxes. Lazar maintains no direct materials, work-in-process, or finished goods inventory
Lazar can reduce direct material costs for Crates by 50% per unit, with no change in direct labor
costs. However, it would increase machine-hour production time by 1-1/2 hours per unit. For
Crates, variable overhead costs are allocated based on machine hours. What would be the effect
on the total contribution margin if this change was implemented?
Correct Answer: A. $125,000 increase.
A. $123,000 increase.
B. $300,000 increase.
C. $230,000 decrease.
D. $1,250,000 increase.
Explanation:
A. The present contribution margin for crates is $3 per unit ($20 - 5 - 8 - 3 - 1).
If the change is made, variable overhead applied will change as follows:
Machine hours/unit currently required for Crates = 2 hours.
Variable overhead currently applied per unit = $3.
Therefore, the variable OH application rate is $1.50 per machine hour used ($3 ÷ 2).
If the machine hours/unit are increased by 1-1/2 hours per unit, each unit will require 3.5
hours of machine time (2 hours presently + 1-1/2 hour increase).
At an application rate of $1.50 per machine hour. 3.5 hours machine time per unit will
result in $5.25 VOH being applied per unit (3.5 x $1.50).
Now, we can calculate the contribution margin per unit for crates that would result from
this change:
Selling price $20.00
Direct materials (reduced by 50%) 2.50
Direct labor (unchanged) 8.00
Variable overhead applied 5.25
Variable selling costs (unchanged) 1.00
Contribution margin per unit $3.25
Since the current contribution margin per unit is $3.00, the contribution margin per unit
will increase by $0.25. 500.000 crates will be produced and sold. Therefore, the total
increase in the contribution margin as a result of this change will be $0.25 x 500,000, or
$125,000.
B. This is not the correct answer. Please see the correct answer for a complete explanation.
We have been unable to determine how to calculate this incorrect answer choice. If you
have calculated it, please let us know how you did it so we can create a full explanation
of why this answer choice is incorrect.
C. This answer results from adjusting the variable selling cost per crate on the assumption
that variable selling costs are part of overhead and would thus be applied to production at
a higher rate due to more machine hours being required per unit. Selling costs are not
applied to production because they are period costs and are expensed as they are incurred.
Therefore, the variable selling cost per unit will not change as a result of the increased
number of machine hours required to produce each crate.
D. This answer results from incorporating the decrease in direct materials costs but not the
increase in the amount of variable overhead applied. The amount of variable overhead
applied will increase because variable overhead is allocated based on machine hours, and
the number of machine hours required per unit will increase.

Question #99: The requirement of the Foreign Corrupt Practices Act of 1977 to devise and
maintain adequate internal control is assigned in the act to the
Correct Answer: D. Company as a whole with no designation of specific persons or positions.
A. Chief financial officer.
B. Board of directors.
C. Director of internal auditing.
D. Company as a whole with no designation of specific persons or positions.
Explanation:
Answer (A) is incorrect.
Compliance with the FCPA is not the specific responsibility of the chief financial officer.
Answer (B) is incorrect.
Compliance with the FCPA is not the specific responsibility of the board of directors.
Answer (C) is incorrect.
Compliance with the FCPA is not the specific responsibility of the director of internal auditing.
Answer (D) is correct.
The accounting requirements apply to all public companies that must register under the
Securities Exchange Act of 1934. The responsibility is thus placed on companies, not
individuals.

Question #100: Which of the following gives minority shareholders more choice in corporate
governance?
Correct Answer: B. Cumulative voting.
A. Traditional voting.
B. Cumulative voting.
C. Proxy-voting.
D. Preemptive voting.
Explanation:
By definition, a cumulative voting system allows shareholders to cast different numbers of
votes for different candidates. Cumulative voting attempts to give minority shareholders
more voice in corporate governance by increasing their chances to elect a certain number of
directors.

MOCK TEST 8

Question #1: Johnson Company manufactures a variety of shoes and has received a special
one-time-only order directly from a wholesaler. Johnson has sufficient idle capacity to accept
the special order to manufacture 15,000 pairs of sneakers at a price of $7.50 per pair.
Johnson's normal selling price is $11.50 per pair of sneakers. Variable manufacturing costs
are $5.00 per pair and fixed manufacturing costs are $3.00 a pair. Johnson's variable selling
expense (not including shipping-out expense) to obtain an order for its normal line of
sneakers is $1.00 per pair. What would the effect on Johnson's operating income be if the
company accepted the special order?
Correct Answer D. Increase by $37,500.
A. Decrease by $60,000.
B. Increase by $52,500.
C. Increase by $22,500.
D. Increase by $37,500.
Explanation:
A. This is the difference between Johnson's normal selling price of $11.50 per pair and the
special order price of $7.50 per pair, multiplied by 15,000 pairs. This answer assumes the
company will have a loss equal to the amount of the discount on each pair of sneakers
because it could have sold them for $11.50 instead of $7.50. That is not the case, because
the company's facilities would be idle if it does not accept this order. In other words, it
cannot sell these sneakers for $11.50. Thus there is no opportunity cost in accepting this
order. Furthermore, the company's normal fixed costs and normal variable selling costs
do not apply to this order. (1) Fixed manufacturing costs are irrelevant, because they will
be the same whether the order is accepted or not accepted. (2) The variable selling
expense for this special order would not be the same as the selling expense for the
company's normal line of sneakers. The question states that the $1.00 variable selling
expense does not include shipping-out expense. Other typical variable selling expenses
are sales representative's commissions, advertising etc., .But in this question it is stated
that the order was received directly from the wholesaler, so those other selling expenses
for the company's normal line of sneakers would not apply.
B. This is the special order price of $7.50 per unit minus the fixed cost per unit of $3.00 per
pair minus the normal variable selling cost of $1.03 per pair, the difference multiplied by
15,000 units. This is incorrect because (1) Fixed manufacturing costs are irrelevant,
because they will be the same whether the order is accepted or not accepted: (2) The
variable manufacturing cost per unit has not been taken into consideration: and (3) The
variable selling cost for the company's normal line of sneakers has been included. The
selling expense for this special order would not be the same as the selling expense for the
company's normal line of sneakers. The question states that the $1.00 variable selling
expense does not include shipping-out expense. Other typical variable selling expenses
are sales representatives commissions, advertising, etc., . But in this question it is stated
that the order was received directly from the wholesaler, so those other selling expenses
for the company's normal line of sneakers would not apply.
C. This answer results from including the $1.03 per unit variable selling expense for the
normal line of sneakers as a cost of the order. The key to getting this question right is to
not assume that the variable selling expense for this special order would be the same as
the variable selling expense for the company's normal line of sneakers. The question
states that the $1.00 variable selling expense does not include shipping-out expense.
Other typical variable selling expenses are sales representative's commissions,
advertising, etc., . But in this question it is stated that the order was received directly from
the wholesaler, so those other selling expenses for the company's normal line of sneakers
would not apply.
D. The incremental contribution margin per pair of sneakers is $7.50 selling price minus
$5.00 variable manufacturing cost or $2.50. Fixed manufacturing costs are irrelevant,
because they will be the same whether the order is accepted or not accepted. The variable
selling expense for this special order would not be the same as the variable selling
expense for the company's normal line of sneakers. The question states that the S1.03
variable selling expense does not include shipping-out expense. Other typical variable
selling expenses are sales representative's commissions, advertising, etc., . But in this
question it is stated that the order was received directly from the wholesaler, so those
other selling expenses for the company's normal line of sneakers would not apply.
Therefore, the effect on operating income if the company accepts the special order will be
$2.50 x 15,000 pairs, or $37,500.

Question #2: Which of the following is a benefit of a well developed and implemented
Enterprise Risk Management (ERM) system?
Correct Answer B. Identification and management of cross-enterprise risks.
A. There is less probability of events occurring that would impact the company’s ability to
achieve its objectives.
B. Identification and management of cross-enterprise risks.
C. Higher expected cash flows can be used in capital budgeting analyses because of less
uncertainty.
D. Reliable information for use in decision making.
Explanation:
A. The probabilities of events occurring that would impact the company's ability to achieve
its objectives are the same after implementation of an ERM system as they were before.
B. Identification and management of multiple and cross-enterprise risks is a benefit of an
effective ERM system. Enterprise Risk Management is different from traditional risk
management where the heads of the various units identify and manage the risks they
perceive as affecting their own unit's ability to achieve its objectives. With ERM, risks
that affect the whole enterprise -- cross-enterprise risks -- that may be missed by
individual department or division heads are recognized and managed.
C. An effective ERM system does not result in higher expected cash flows being used in
capital budgeting analyses.
D. Reliable information for use in decision making is a benefit of a strong internal control
system. It is not a benefit of a good ERM system.
Question #3: A company has decided to self-insure for its employees' medical insurance. This is
an example of
Correct Answer A. Retaining the risk
A. Retaining the risk.
B. Reducing the risk
C. Transferring the risk
D. Exploiting the risk.
Explanation:
A. Retaining risk means bearing the risk such as when a firm chooses to self-insure by
budgeting for and paying the cost of any losses out of its own funds. Self-insuring
employees' medical costs is an example of retaining a risk. A company would choose to
do this if it believed that the cost to pay the expenses and to administer the payouts would
be less than the cost to purchase insurance to cover the expenses.
B. Reducing or mitigating the risk includes activities such as risk diversification, for
example splitting the IT function into two separate geographical areas so that one area
can back up the other one if a natural disaster strikes and destroys one of the IT centers.
Self-insuring employees' medical expenses does not reduce the risk
C. Transferring or sharing risk usually means purchasing insurance. Self-insuring is not the
same thing as purchasing insurance.
D. Exploiting a risk involves exposing the firm to risk in order to take advantage of a
situation and thus increase the value of the firm. Self-insuring against the risk of
employees' medical expenses does not increase the value of the firm.

Question #4: A company issued a 15-year. $1,000 par value bond. The coupon rate on this bond
is 9% annually, with interest being paid each 6 months. The investor who purchased the bond
expects to earn a 12% nominal rate of return. The cash proceeds received by the company from
the investor totaled
Correct Answer B. $793.43
A. $619.43
B. $793.43
C. $875.38
D. $950.75
Explanation:
Answer (A) is incorrect.
Failing to consider the maturity value of the bond results in $619.43.
Answer (B) is correct.
The cash flows consist of interest of $45 every 6 months for 15 years (30 periods), and
$1,000 at the end of the 30th interest period. The 12% discount rate translates to 6% every 6
months. Thus, the calculation is as follows:
Periodic interest 13,765(30 periods @ 6%) x $45 = $619.43
Maturity amount .174(6%) x $1,000 = 174.00
Total proceeds $793.43($619.43 + $ 174)
Answer (C) is incorrect
Using 15 annual periods instead of 30 half-year periods results in $875.38.
Answer (D) is incorrect.
Using the wrong semi-annual interest results in $950.75.

Question #5: A firm earned $10.000 before interest and taxes, has a 36% tax rate, and has the
following debt outstanding:
First mortgage bond.
9.0% $5,000
Debenture, 10.2% 10,000
Subordinated bond, 12.0% 6,000
Total long-term debt $21,000

The annual coverage of the firm's debt is


Correct Answer A. 4.57 times.
A. 4.57 times.
B. 2.92 times.
C. 11.85 times.
D. 3.57 times.
Explanation:
Answer (A) is correct
The times interest earned (interest coverage) ratio is computed by dividing the income available
for paying interest (pretax pre-interest income) by the annual interest expense. The first step is to
determine the annual interest expense:

First mortgage bond. 9.0%× $5,000= $450


Debenture 10.2%× 10,000= 1,020
Subordinated bond 12.0%× 6,000= 720
Total long-term debt $2,190

Answer (B) is incorrect


After-tax, pre-interest income of $6,400 divided by the interest expense equals 2.92 times.
Answer (C) is incorrect
Pre-tax, pre-interest income should be divided by interest expense to find the times interest
earned ratio.
Answer (D) is incorrect.
Pre-tax, after-interest income ($7,810) divided by $2.190 equals 3.57 times.

Question #6: Kator Co. is a manufacturer of industrial components. One of their products that is
used as a subcomponent in auto manufacturing is KB-96. This product has the following
financial structure per unit:

Selling price $150


Direct materials $20
Direct labor 15
Variable manufacturing
overhead 12
Fixed manufacturing overhead 30
Shipping and handling 3
Fixed selling and
administrative 10
Total costs $90

Kator Co. has received a special, one-time, order for 1,000 KB-96 parts. Assume that Kator is
operating at full capacity and that the contribution margin of the output that would be displaced
by the special order is $10,000. The minimum price that is acceptable, using the original data, for
this one-time special order is in excess of
Correct Answer: C. $60
A. $87
B. $70
C. $60
D. $100
Explanation:
A. An answer of $87 includes fixed manufacturing, but does not include shipping and
handling.
B. This answer is including fixed selling and administrative cost which are unavoidable and
should not be included.
C. If the company is operating at full capacity, then in order to produce the special order the
company will lose $10,000 of potential contribution, or $10 per unit ($10,000 lost
contribution + 1,000 units in the special order). This lost opportunity has to be reflected
in the price that will be charged for the special order, which will be $20 DM + $15 DL +
$12 VOH+ $3 shipping and handling + $10 opportunity cost for a total of $60. $60 is the
minimum price that is acceptable.
D. The answer $100 is calculated by the use of the full cost ($90 per unit) plus the
opportunity cost of $10 per unit to calculate the minimum acceptable price ($90 full cost
of KB-96. $10 opportunity cost). The answer should include only incremental cost plus
the opportunity cost

Question #7: A company needs to borrow $500,000 to meet its working capital requirements for
next year. A bank has offered the company a 9.5% simple interest loan that has a 16%
compensating balance requirement. Determine the effective interest rate for the loan.
Correct Answer • 11.31%
A. 11.02%
B. 11.31%
C. 12.75%
D. 19.00%
Explanation:
Answer (A) is incorrect.
The effective interest rate is equal to the net interest expense over the usable funds. The amount
of 11.02% incorrectly multiples the simple interest with the compensating balance requirement
and then adds it to the simple interest in order to calculate the effective interest rate.
Answer (B) is correct.
The effective interest rate is equal to the net interest expense over the usable funds. The
company's net interest expense is equal to $47,500 ($500,000 x 9.5%). Because the bank is
requiring a compensating balance of 16%, the company's usable funds are equal to $420,000
[$500,000 - ($500,000 x 16%)]. Thus, the effective interest rate is 11.31% ($47,500 + $420,000).
Answer (C) is incorrect
The effective interest rate is equal to the net interest expense over the usable funds. The amount
of 12.75% incorrectly averages the 9.5% simple interest and the 16% compensating balance
requirement to calculate the effective interest rate.
Answer (D) is incorrect
The effective interest rate is equal to the net interest expense over the usable funds. The amount
of 19.00% incorrectly uses the 16% compensating balance requirement to calculate the net
interest expense in the numerator of the equation.

Question #8: Oakes Inc. manufactured 40,000 gallons of Mononate and 60,000 gallons of
Beracyl in a joint production process, incurring $250,000 of joint costs. Oakes allocates joint
costs based on the physical volume of each product produced. Mononate and Beracyl can each
be sold at the split-off point in a semi-finished state or, alternatively, processed further.
Additional data about the two products are as follows.
Monorat
e Beracyl
Sales price per gallon at split-off $7 $ 15
Sales price per gallon if processed further $ 10 $18
$115,00
Variable production costs if processed further $125,000 0

An assistant in the company's cost accounting department was overheard saying that "....when
both joint and separable costs are considered, the firm has no business processing either product
beyond the split-off point. The extra revenue is simply not worth the effort" Which of the
following strategies should be recommended for Oakes?
Correct Answer C. Sell Mononate at split-oft process Beracyl further.
A. Sell Mononate at split-off sell Beracyl at splitoff.
B. Process Mononate further, sell Beracyl at split-off.
C. Sell Mononate at split-off process Beracyl further.
D. Process Mononate further process Beracyl further.
Explanation:
A. This would not result in the highest operating income.
B. This would not result in the highest operating income.
C. The additional revenue to be received from processing each product further should be
compared with the costs of processing the product further. The joint costs are irrelevant
to this decision because they have already been incurred.
Mononate:
40,000 gallons x $3 additional revenue = $120,000 additional revenue if processed
further
Variable cost to process further = 125,000
Loss from processing further = $(5,000)
Beracyl:
60,000 gallons x $3 additional revenue = $180,000 additional revenue If processed
further
Variable cost to process further = $115,000
Profit from processing further = $65,000
Mononate should not be processed further but should be sold at the split-off point,
because to process it further would moan the company would lose $5,000. However,
Beracyl should be processed further because the company can earn additional operating
income of $65,000 by processing it further.
D. This would not result in the highest operating income.

Question#9: Tendulkar Inc. has a project that requires a $40,000,000 initial investment and is
expected to generate annual after-tax cash flows of $6,000,000 for 12 years. Tendulkar’s
weighted average cost of capital is 14%. This project’s net present value (NPV) and the
approximate internal rate of return (IRR) are:
(Use one or both of the following Present Value tables to calculate your answer.)
Present value of $1

Periods 6% 7% 8% 9% 10% 12% 14% 16% 18% 20%


1 0.943 0.935 0.926 0.917 0.909 0.893 0.877 0.862 0.847 0.833
2 0.890 0.873 0.857 0.842 0.826 0.797 0.769 0.743 0.718 0.694
3 0.840 0.816 0.772 0.772 0.772 0.712 0.675 0.641 0.609 0.579
4 0.792 0.763 0.708 0.708 0.708 0.683 0.592 0.552 0.516 0.482

5 0.747 0.713 0.681 0.650 0.621 0.567 0.519 0.476 0.437 0.402
6 0.705 0.666 0.630 0.596 0.564 0.507 0.456 0.410 0.370 0.335
7 0.665 0.623 0.583 0.547 0.513 0.452 0.400 0.354 0.314 0.279
8 0.627 0.582 0.540 0.502 0.467 0.404 0.351 0.305 0.266 0.233

9 0.592 0.544 0.500 0.460 0.424 0.361 0.308 0.263 0.255 0.194
10 0.558 0.508 0.463 0.422 0.386 0.322 0.270 0.227 0.191 0.162
11 0.527 0.475 0.429 0.388 0.350 0.287 0.237 0.195 0.162 0.135
12 0.497 0.444 0.397 0.356 0.319 0.257 0.208 0.168 0.137 0.112

Present value of an Ordinary Annuity of $1

Periods 6% 7% 8% 9% 10% 12% 14% 16% 18% 20%


1 0.943 0.935 0.926 0.917 0.909 0.893 0.877 0.862 0.847 0.833
2 1.833 1.808 1.783 1.759 1.736 1.690 1.647 1.605 1.566 1.528
3 2.673 2.624 2.577 2.531 2.487 2.402 2.322 2.246 2.174 2.106
4 3.465 3.387 3.312 3.240 3.170 3.037 2.914 2.798 2.690 2.589

5 4.212 4.100 3.993 3.890 3.791 3.605 3.433 3.274 3.127 2.991
6 4.917 4.767 4.623 4.486 4.355 4.111 3.889 3.685 3.498 3.326
7 5.582 5.389 5.206 5.033 4.868 4.584 4.288 4.039 3.812 3.605
8 6.210 5.971 5.747 5.535 5.335 4.968 4.639 4.344 4.078 3.837

9 6.802 6.515 6.247 5.995 5.759 5.328 4.946 4.607 4.303 4.031
10 7.360 7.024 6.710 6.418 6.145 5.650 5.216 4.833 4.494 4.192
11 7.887 7.499 7.139 6.805 6.495 5.938 5.453 5.029 4.656 4.327
12 8.384 7.943 7.536 7.161 6.814 6.194 5.660 5.197 4.793 4.439

Correct answer: C. NPV: ($6,040,000); IRR: 10%.


A. NPV: ($232,000); IRR; 10%.
B. NPV: ($6,040,000); IRR: 12%.
C. NPV: ($6,040,000); IRR: 10%.
D. NPV: ($232,000); IRR: 12%.
Explanation:
To calculate NPV
Cash Present
Item Year(s) flow(s) Factor value
$ $
(40,000,000 (40,000,000
Initial investment 0 ) 1.0 )
Annual after-tax cash
flows 1.12 6,000,000 5.66 33,960,000
$
Net present value (6,040,000)

IRR is used to estimate the discount rate that makes the present value of net cash inflows equal to
the initial investment. In other words, the IRR is the discount rate that makes the net present
value equal to zero.
To calculate IRR:
IRR factor = Initial investment / Annual cash inflows
IRR factor = $(40,000,000) / 6,000,000
IRR factor = 6.667
Once the IRR factor has been calculated, go to the PV of annuity of $1 table. Since this
investment has a 12 year life, look across the 12 year row to find the discount factor that is
closest to 6.667. It’s closest to the factor that appears in the 10% column (6.814), so the IRR is
approximately 10%.

Question #10: Specialty Inc. has a limited supply of 1.2001tis of raw materials which can be
used to produce either product X or Y. details of which are given below.
Product X Product Y
Selling price per
unit $200 $250
Variable costs
per unit 176 200
Raw materials
used per unit 8 lbs. 10 lbs.

Which one of the following should Specialty produce in order to maximize contribution margin?
A. 100 units of product X and 40 units of product Y.
B. 120 units of product Y
C. 100 units of product X and 80 units of product Y.
D. 150 units of product X.
A. When operating at capacity, operating income is maximized by maximizing contribution
margin per unit of the resource that is limiting either the production or the sale of products.
While it is possible for the company to produce 100 units of Product X and 40 units of Product Y
this would not maximize contribution margin
Correct Answer: B.120 units of product Y.
A. 100 units of product X and 40 units of product Y.
B. 120 units of product Y.
C. 100 units of product X and 80 units of product Y.
D. 150 units of product X.
Explanation:
A. When operating at capacity, operating income is maximized by maximizing contribution
margin per unit of the resource that is limiting either the production or the sale of
products. While it is possible for the company to produce 100 units of Product X and 40
units of Product Y. this would not maximize contribution margin.
B. When operating at capacity, operating income is maximized by maximizing contribution
margin per unit of the resource that is limiting either the production or the sale of
products.
The contribution margin per unit of Product X is $24. Product X requires 8 lbs. of raw
materials per unit Therefore, the contribution margin per unit of raw materials for Product
X is $24 ÷ 8, or $3.
The contribution margin per unit of Product Y is $50. Product Y requires 10 lbs. of raw
materials per unit. Therefore, the contribution margin per unit of raw materials for
Product Y is $50 ÷ 10, or $5.
Since the contribution margin per unit of raw materials used is higher for Product Y than
for Product X, the company should use all of its raw materials to produce Product Y,
assuming demand is sufficient to enable the company to sell all of the Product Y it
produces. Since 1,200 pounds of raw material are available and each unit of Product Y
requires 10 pounds of material. Specialty Inc. will be able to produce 120 units of
Product Y (1200 ÷ 10)
C. This combination of production for Products X and V is not possible given the amount of
raw material that is available.
D. When operating at capacity, operating income is maximized by maximizing contribution
margin per unit of the resource that is limiting either the production or the sale of
products. While it is possible for the company to produce 150 units of X with 1,200
pounds of raw materials, this would not maximize contribution margin.

Question #11: IMAs Statement of Management Accounting "Values and Ethics: From Inception
to Practice identifies that ongoing training should include all of the following except:
Correct Answer B. Requiring each employee to sign the code of conduct
A. How ethics affects specific jobs and processes.
B. Requiring each employee to sign the code of conduct.
C. What action is taken when an ethical issue is identified.
D. Penalties when noncompliance with the code of ethics is proven.
Explanation:
The SMA specifies that ongoing training should include the following expectations:
• General employee behavior and personal conduct
• How ethics are built into work management methods.
• How ethics affects specific jobs, processes, activities, and relationships.
• How the organization monitors compliance with code
• What routes are open to employees who have compliance issues.
• What action is taken when a complaint or issue is identified.
• The actions and penalties once noncompliance is proven.

Question#12: A firm has gathered financial statement data from three companies applying for
credit as new customers. The company extends credit to customer on the credit terms 2/10,
net 30. Prior to accepting the customers, a financial analyst with the firm performs a liquidity
analysis. Summary data shown below:
Company F Company G Company H
Cash $20,000 $12,850 $130,000
Accounts receivable 40,000 74,500 100,000
inventory 170,000 42,240 354,300
Current assets 230,000 129,590 584,300
Total assets 567,888 260,400 780,560
Current liabilities 175,000 63,800 142,100
Total liabilities 487,120 97,680 364,760
Total shareholders' equity 80,768 162,720 415,800

Correct answer: C. Company F has a low ratio, so the firm should not accept Company F as a
new customer.
A. Company H has the highest current ratio but the lowest acid-test ratio, so the firm should
not accept. Company H as a new customer.
B. Company G has a low debt to total assets ratio, so the firm should accept Company G as
a new customer.
C. Company F has a low current ratio, so the firm should not accept Company F as a new
customer.
D. Company F has a high long-term debt to equity ratio, so the firm should accept Company
F as a new customer.
Explanation:
Answer (A) is incorrect.
Although Company H has the highest current ratio of 4.11 (584,300 ÷ 142,100), its acid-test ratio
of 1.62 (230,000 ÷ 142,100) is not the lowest. Company F has the lowest acid test ratio of 0.34
(60,003 ÷ 175,000).
Answer (B) is incorrect.
Although it does have a low debt to total assets ratio, this is not the only indicator the firm should
look at in order to determine whether it should accept the company as a customer.
Answer (C) is correct.
Low current and add-test ratios indicate that a company may not be able to repay its debts.
Therefore, the firm would not want to accept it as a credit customer.
Answer (D) is incorrect.
A high long-term debt to equity ratio is a bad indicator, therefore, the firm would not accept
Company F for this reason.

Question #13: A company has a $100 invoice with payment terms of 2/10, net 60. The company
can either take the discount or place the funds in a money market account paying 6% interest.
Using a 360day year, the company's cost of not taking the cash discount is
Correct Answer. B. 8.7%
A. 12.2%
B. 8.7%
C. 6.4%
D. 6.2%
Explanation:
Answer (A) is incorrect.
The cost of not taking the cash discount is 8.7%. The company will initially lose $2 by not taking
the discount. This amount is partially offset by interest earned on $98 for 50 days of $.817. Thus,
the net cost is $1.183 ($2.00 - $.817). Because a 360-day year has 7.2 periods of 50 days each,
the total annualized cost is $8.52 (7.2 x $1.183). The loss rate is about 8.7% ($8.52 ÷ $98).
Answer (B) is correct.
The payment terms are 2/10, net 60. This means that the company will get a 2% discount, which
would equal $2 ($100 x 2%). if it pays the within invoice 10 days. This means that not taking the
discount will cost $2. This amount is partially offset by interest earned on $98 for 50 days of
$.817. Thus, the net cost is $1.183 ($2.00 - $.817). Since a 360-day year has 7.2 fifty-day
periods, the total annualized cost is $8.52 (7.2 x $1.183). The loss rate is about 8.7% ($8.52 ÷
$98).
Answer (C) is incorrect
The cost of not taking the cash discount is 8.7%. The company will initially lose $2 by not taking
the discount. This amount is partially offset by interest earned on $98 for 50 days of $.817. Thus,
the net cost is $1.183 ($2.03 - $.817). Because a 360-day year has 7.2 periods of 50 days each,
the total annualized cost is $8.52 (7.2 x $1.183). The loss rate is about 8.7% ($8.52 ÷ 598).
Answer (D) is incorrect.
The cost of not taking the cash discount is 8.7%. The company will initially lose $2 by not taking
the discount. This amount is partially offset by interest earned on $98 for 50 days of $.817. Thus,
the net cost is $1.183 ($2.00 - $.817). Because a 360-day year has 7.2 periods of 50 days each the
total annualized cost is $8.52 (7.2 x $1.183). The loss rate is about 8.7% ($8.52 ÷ $98).

Question#14: Question: RST’s acid test (or quick) ratio at the end of Year 6 is (please refer the
fact pattern below)
[Fact pattern #15]
RST Corporation comparative Income
Statement for the Year 5 and 6

Year 6 Year 5
$285,00 $200,00
Sales (all are credit) 0 0
Cost of goods sold 150,000 120,000
$135,00
Gross profit 0 $80,000
Selling and administrative expenses 65,000 36,000
Income before interest and income
taxes $70,000 $44,000
Interest expenses 3,000 3,000
Income before income taxes $67,000 $41,000
Income tax expense 27,000 16,000
Net income $40,000 $25,000

RST Corporation
Comparative Balance Sheets
End of Year 5 and 6

Assets Year 6 Year 5


Current assets:
Cash $5,000 $4,000
Short term marketable
investments 3,000 2,000
Accounts receivable (net) 16,000 14,000
Inventory 30,000 20,000
Total current assets $54,000 $40,000
Noncurrent assets:
Long term investments 11,000 11,000
Property, plant and equipment 80,000 70,000
Intangibles 3,000 4,000
$148,00 $125,00
Total assets 0 0

Liabilities and stockholders' equity


Current liabilities:
$11,0 $7,00
Accounts payable 00 0
Accrued payables 1,000 1,000
Total current $12,0 $8,00
liabilities 00 0
Long term liabilities:
0% Bonds payable, 30,00 30,00
due in Year 12 0 0
42,00 $38,0
Total liabilities 0 00
Stockholders' equity:
Common stock, 2,400 $24,0 24,00
shares, $10 par 00 0
82,00 63,00
Retained earnings 0 0
Total stockholders' $106, $87,0
equity 000 00
Total liabilities and $148, $125,
stockholders' equity 000 000

The market value of RST’s common stock at the end of Year 6 was $100.00 per share.

V Correct Your Answer: • 2.00 to 1.


Correct Answer • 2.00 to 1. All Possible Answers:
• 2.40 to 1. • 2.18 to 1. • 2.00 to 1. • 1.50 to 1.
Explanation:
Answer (A) is incorrect.
Dividing total current liabilities ($12,000) by the amount of cash ($5,000) results in 2.4, which
produces no meaningful ratio.
Answer (B) is incorrect.
Dividing quick assets ($5,000+$3,000+ $16,000) by accounts payable ($11,000) results in 2.18.
The denominator should include all current liabilities.
Answer (C) is correct.
Liquidity ratios measure the ability of a company to meet its short-term obligations. A
commonly used liquidity ratio is the acid test or quick ratio, which equals the sum of the quick
assets (net accounts receivable, short-term marketable securities. and cash) divided by current
liabilities. This ratio at the end of Year Six is 2.0 (($5,000 + $3,000 - $16,000) ÷ $12,000].
Answer (D) is incorrect.
Dividing total current liabilities ($12,000) by the sum of cash ($5,000) and short-term
marketable investments ($3,000) results in 1.5, which produces no meaningful ratio.

Question #15: Systematic risk explains why


Correct Answer A. Stock value tend to move in the same direction.
A. Stock value tend to move in the same direction.
B. Diversification reduces overall risk
C. Stock values move in different directions.
D. Diversification increases overall risk
Explanation:
Answer (A) is correct.
Systematic risk also called market risk is the risk faced by all firms. Changes in the economy as a
whole, such as the business cycle, affect all players in the market. Since all firms are affected by
systematic risk, all of their stock values move somewhat in the same direction.
Answer (B) is incorrect.
Systematic risk is also known as un-diversifiable risk Therefore, diversification has no effect on
systematic risk.
Answer (C) is incorrect
Systematic risk affects all firms equally. Therefore, it causes the stock values to move in the
same general direction.
Answer (D) is incorrect.
Systematic risk is also known as un-diversifiable risk Therefore, diversification has no effect on
systematic risk.

Question #16: In estimating °after-tax incremental cash flows:' for capital project evaluations,
which one of the following options reflects the items that should be included in the analyses?
Correct Answer A. Sunk Costs: No: Change in Net Working Capital: Yes: Estimated Impacts of
Inflation: Yes.
A. Sunk Costs: No: Change in Net Working Capital: Yes: Estimated Impacts of Inflation:
Yes.
B. Sunk Costs: No: Change in Net Working Capital: No: Estimated Impacts of Inflation:
Yes
C. Sunk Costs: No: Change in Net Working Capital: Yes: Estimated Impacts of Inflation:
No.
D. Sunk Costs: Yes: Change in Net Working Capital: No: Estimated Impacts of Inflation:
No.
Explanation: In estimating "after-tax incremental cash flows:' under discounted cash flow
analyses for capital project evaluations, project related changes in net working capital and
estimated impacts of inflation should be included in the analysis. Sunk costs are past costs
that do not change as a result of a future decision and should not be included in the analysis.

Question #17: A major difference between risk in investing and other types of risk is
Correct Answer A. Risk in investing has the potential for either a positive or negative event
whereas other types of risk have the potential only for a negative event.
A. Risk in investing has the potential for either a positive or negative event whereas other
types of risk have the potential only for a negative event.
B. Other types of risk can be managed with insurance, whereas it is not possible to manage
risk in investing.
C. Risk in investing is operational risk whereas other types of risk are static risks.
D. Risk in investing has the potential for great losses, whereas other types of risk have the
potential for either great losses or great gains.
Explanation:
A. Risk in investing is defined as the variability of actual returns from expected returns, and
this variability may be either a gain or a loss. So risk in investing is the potential for
either a positive event (a gain in excess of the expected return) or a negative event (a loss
or a gain that is lower than the expected return). Enterprise risk is defined as "any event
or action that can keep an organization from achieving its objectives." Thus, types of
enterprise risk other than investing risks have the potential only for a negative event to
occur.
B. Some but not all types of other risk can be managed with insurance: and risk in investing
can be managed and reduced, though not eliminated, through proper investment
diversification and through hedging activities.
C. Risk in investing is one type of financial risk which in turn is a type of enterprise risk
Strategic risk is another type of enterprise risk, but not the only type. The four types of
enterprise risk are strategic risk financial risk operational risk and hazard risk
D. Risk in investing is defined as the variability of actual returns from expected returns, and
this variability may be either a gain or a loss. Therefore, risk in 'misting is not limited to
the potential for losses but it also includes the potential for gains above the expected
return. Enterprise risk is defined as "any event or action that can keep an organization
from achieving its objectives." Therefore, types of enterprise risk other than investing
risks do not involve the potential for gains.

Question #18: The following transactions occurred during a company's first year of operations:
I. Purchased a delivery van for cash
II. Borrowed money by issuance of short-term debt
III. Purchased treasury stock
Which of the items above caused a change in the amount of working capital?
Correct Answer D. I and III only.
A. I only.
B. I and II only.
C. II and III only.
D. I and III only.
Explanation:
Answer (A) is incorrect.
The purchases of the van and treasury stock affect working capital.
Answer (B) is incorrect.
The purchases of the van and treasury stock but not the issuance of short-term debt affect
working capital.
Answer (C) is incorrect
The purchases of the van and treasury stock but not the issuance of short-term debt affect
working capital.
Answer (D) is correct.
Working capital is computed by deducting total current liabilities from total current assets.
The purchase of a delivery van for cash reduces current assets and has no effect on current
liabilities. The borrowing of cash by incurring short-term debt increases current assets by the
same amount as it increases current liabilities: hence, it will have no effect on working
capital. The purchase of treasury stock decreases current assets but has no effect on current
liabilities. Thus, the purchases of the van and treasury stock affect working capital.

Question #19: In equipment-replacement decisions, which one of the following does not affect
the decision-making process?
Correct Answer D. original fair market value of the old equipment.
A. Cost of the new equipment
B. Operating costs of the new equipment
C. Operating costs of the old equipment.
D. Original fair market value of the old equipment.
Explanation:
The original fair market value of the old equipment is a sunk cost Sunk costs are costs that
have already occurred and cannot be recovered in the future. Therefore, they are irrelevant
for decision making.

Question #20: The amount of cash that an investor would have to receive to be indifferent
between the payoff and a given gamble is called the:
Correct Answer D. Certainty equivalent.
A. Investment beta.
B. Covariance.
C. Variance.
D. Certainty equivalent.
Explanation:
By definition. a certainty equivalent is the amount of cash an investor requires at a given point in
time to make that investor indifferent between that certain amount and the amount expected
(assuming risk) at the same point in time.

Question #21: Gleason Co. has two products, a frozen dessert and ready-to-bake breakfast rolls,
ready for introduction. However, plant capacity is limited, and only one product can be
introduced at present. Therefore. Gleason has conducted a market study, at a cost of $26,000, to
determine which product will be more profitable. The results of the study follow.
Sales of desserts Sales of Rolls
at $1.80/unit at $1.20/unit
probabili volum probabili
volume ty e ty
250,00 200,00
0 0.30 0 0.20
300,00 250,00
0 0.40 0 0.50
350,00 300,00
0 0.20 0 0.20
400,00 350,00
0 0.10 0 0.10

The costs associated with the two products have been estimated by Gleason's cost accounting
department and are shown as follows.

Dessert Rolls
Ingredients per unit $0.40 $0.25
Direct labor per unit 0.35 0.30
Variable overhead per
unit 0.40 0.20
Production tooling" 48,000 25,000
Advertising 30,000 20,000

'Gleason treats production tooling as a current operating expense rather than capitalizing it as a
fixed asset.
According to Gleason's market study, the expected value of the sales volume of the breakfast
rolls is
Correct Answer B. 260,000 units.
A. Some amount other than those given.
B. 260,000 units.
C. 275,000 units.
D. 125,000 units.
Explanation:
A. The correct answer is one of the answer choices given.
B. Expected sales volume is a weighted average of the possible sales volume, weighted
according to their probabilities. To find the expected sales volume, multiply each
estimated sales volume by its probability of occurring and sum the results. (200,000 x
0.2) + (250,000 x0.5) + (300,000 x0.2) + (350,000 x0.1) = 260,000.
Expected sales volume is 260,000 units.
C. This calculation represents the average of the possible sales volumes of the rolls. The
expected value is a weighted average with the various probabilities of each sales volume
as the weights.
D. The answer of 125,000 was calculated by using the estimated sales volume of 250,000
times its probability of 0.5. This is not the expected value of the sales volume of rolls.

Question #22: The most likely benefit of a multinational company to its host country is
Correct Answer C. Increased tax revenues.
A. Net capital outflow.
B. Formation of cartels.
C. Increased tax revenues.
D. Establishment of transfer prices to minimize taxes.
Explanation:
Answer (A) is incorrect.
Remittance of royalties, dividends, and profits can result in a net capital outflow, an adverse
effect.
Answer (B) is incorrect.
Formation of cartels (international monopolies) has an anticompetitive effect.
Answer (C) is correct.
Benefits to the host country include (1) new investment of capital, technology, and
management abilities: (2) improvements in output and efficiency along with the resulting
stronger balance of payments: and (3) stimulation of competition, increased tax revenues, and
a higher standard of living.
Answer (D) is incorrect.
The multinational may charge economically unreasonable transfer prices. The purpose is to
earn profits where (1) taxes are lowest, or (2) the capital outflow restrictions are weakest.

Question #23: Richardson Motors uses 10 units of Part No. T305 each month in the
production of large diesel engines. The cost to manufacture one unit of T305 is presented as
follows.

Direct materials $2,000


Materials handling (20% of direct materials
cost) 400
Direct labor 16,000
Manufacturing overhead (150% of direct 24,000
labor)
Total manufacturing cost $42,000

Materials handling, which is not included in manufacturing overhead, represents the direct
variable costs of the receiving department that are applied to direct materials and purchased
components on the basis of their cost. Richardson's annual manufacturing overhead budget is
one-third variable and two-thirds fixed. Simpson Castings, one of Richardson's reliable
vendors, has offered to supply T305 at a unit price of $30,000.
Assume Richardson Motors is able to rent all idle capacity for $50,000 per month. If
Richardson decides to purchase the 10 units from Simpson Castings, Richardson's monthly
cost for T305 would
Correct Answer B. Increase $46,000.
A. Increase $96.000.
B. Increase $46,000.
C. Decrease $64,000.
D. Decrease $14,000.
Explanation:
A. This answer overlooks the rent of the idle capacity.
B. Total per unit manufacturing cost for the T305 is $42,400. Since the overhead of $24,000
is 1/3 variable and 2/3 fixed. $16,000 of that would be fixed and would continue if
Richardson purchases the T305 outside and $8,000 of it would be variable and would be
avoidable. Thus, the total avoidable variable manufacturing cost per unit is $26,400
($2,000 DM, $16,000 DL $400 materials handling, and $8,000 variable manufacturing
OH).
If Richardson Motors purchases the units from Simpson Castings then the per unit
variable cost will be $36,000 ($30,000 purchase price + $6,000 material handling cost a
pp4ied [20% hand ling cost x $30,000 per unit]. Therefore, by purchasing frail Simpson
Castings the per unit cost of the T305 component will increase $9,600 ($36,000 -
$26,400). By purchasing 10 units per month total monthly cost would increase by
$96,000 ($9,600 x 10). But, if the idle capacity can be rented for $50,000 per month, then
the monthly net cost would be reduced to $46,000 ($96,000 - $50,000).
C. An answer of $64,000 is the difference between the total cost to manufacture the T305
and the total cost to purchase the T305. However, it does not consider the fixed
manufacturing cost which will continue whether or not the T305 is being manufactured.
It also does not include the rental income.
D. An answer of $14,000 does not include the materials handling cost for the purchased
components.
Question #24: What is the significance of the criterion (hurdle) rate in internal rate of return
(IRR) calculations?
Correct Answer • C. Projects below the criterion rate should be rejected
A. The criterion rate is used to find the discount rate in a present value annuity table.
B. Projects above the criterion rate should be rejected.
C. Projects below the criterion rate should be rejected.
D. The criterion rate is used to find the discount rate in a present value interest factor table.
Explanation:
The IRR method evaluates a capital investment by comparing the estimate's internal rate of
return to a predetermined criterion. This criterion is based on whatever rate the firm uses to
evaluate investments such as its minimum rate of return. the rate from another desirable
alternate investment or an industry average The criterion rate serves as a cutoff point Projects
below this cutoff rate should be rejected.

Question #25: The strong form of the efficient markets hypothesis (EMH) states that current
market prices of securities reflect
Correct Answer B. All information whether it is public or private
A. All publicly available information.
B. All information whether it is public or private.
C. No relevant information.
D. Only information found in past price movements.
Explanation:
Answer (A) is incorrect.
The semi-strong form of EMH states that only publicly available information is reflected in
current market prices.
Answer (B) is correct
The EMH states that stock prices reflect all relevant information, so the market is
continuously adjusting to new information. Stock prices are in equilibrium, so investors
cannot earn abnormal returns. The strong form of the EMH states that all public and private
information is instantaneously reflected in current market prices of securities. Thus, investors
cannot earn abnormal returns.
Answer (C) is incorrect
The EMH states that current market prices at least reflect past price movements.
Answer (D) is incorrect
The weak form of the EMH states that only past price movements are reflected in current
market prices.

Question #26: Sam Smith has been offered a pair of tickets to the pro football team if Smith
purchases a computerized inventory control system from a specific vendor. Which steps
should Smith take?
Correct Answer: A. Review his company's policies on gifts from vendors.
A. Review his company's policies on gifts from vendors.
B. Refuse any further conversations with the vendor.
C. Sign the contract for the system if the price of the ticket is less than $50.
D. Consult with the Audit Committee of the Board of Directors.
Explanation:
According to IMAs "Statement of Ethical Professional Practice' when faced with ethical
issue, an individual should follow their organization's established policies on the resolution
of such a conflict.

Question #27: A company's breakeven point in sales volume may be affected by equal
percentage increases in both selling price and variable cost per unit (assume all other factors
are constant within the relevant range). The equal percentage changes in selling price and
variable cost per unit will cause the breakeven point in sales revenue to
Correct Answer: C Remain unchanged.
A. Decrease by more than the percentage increase in the selling price.
B. Increase by the percentage change in variable cost per unit
C. Remain unchanged.
D. Decrease by less than the percentage increase in selling price.
Explanation:
A. Equal percentage increases in both selling price and variable cost per unit will not cause
the breakeven sales revenue to decrease by more than the percentage increase in the
selling price. Because the variable costs increase by the same percentage as the sales
revenue increases, the contribution margin ratio remains the same after the increases.
B. Equal percentage increases in both selling price and variable cost per unit will not cause
the breakeven sales revenue to increase by the percentage change in variable cost per
unit. Because the sales revenue increases by the same percentage as the variable costs
increase, the contribution margin ratio remains the same after the increases.
C. The breakeven point in sales revenue is equal to fixed cost divided by the contribution
margin ratio, if the sales price and the variable costs increase by the same percentage,
then the contribution margin ratio will remain unchanged and the breakeven point in sales
revenue will also remain unchanged. Example: A company with $10.000 in fixed costs
has a product that sells for $10 per unit with a variable cost of $8 per unit. The
contribution margin is $2, and the contribution margin ratio is $2 ÷ $10, or 0.20. The
breakeven point in revenue is $10,000 ÷ 0.20, or $50,000. If the sales price increases by
5% and the variable costs increase by 5% the new selling price will be $10.50 ($10 x
1.05) and the new variable costs will be $8.40 ($8 x 1.05). The new contribution margin
will be $2.10, and the new contribution margin ratio will be $2.10 ÷ $10.50, or 0.20. The
breakeven point in revenue will remain $10,000 ÷ 0.20, or $50,000.
D. Equal percentage increases in both selling price and variable cost per unit will cause the
breakeven sales volume to decrease by a lesser percentage than the percentage increase in
the selling price. However, this question asks for the effect on the breakeven point in
sales revenue, not the effect on the breakeven point in sales volume. Because the variable
costs increase by the same percentage as the sales revenue increases, the contribution
margin ratio remains the same after the increases.

Question #28: Aspen Company plans to sell 12.000 units of product XT and 8.000 units of
product RP. Aspen has a capacity of 12.000 productive machine hours. The unit cost structure
and machine hours required for each product is as follows.

Unit Costs XT RP
Materials $37 $24
Direct Labor 12 13
Variable overhead 6 3
Fixed overhead 37 38
Machine hours
required 1.0 1.5

Aspen can purchase 12,000 units of XT at $60 and/or 8,000 units of RP at $45. Based on the
above, which one of the following actions should be recommended to Aspen's management?
Correct Answer C Produce XT internally and purchase RR
A. Produce both M and RP.
B. Purchase both XT and RR
C. Produce XT internally and purchase RR
D. Produce RP internally and purchase XT.
Explanation:
A. This is not the most cost-effective solution. See correct answer for an explanation.
B. This is not the most cost-effective solution. See correct answer for an explanation.
C. Usually with a problem like this, we calculate the contribution margin per machine hour,
and the product with the highest contribution margin per machine hour required to
produce it is the one we should produce first with whatever available production capacity
we have. Here we don't have any sales prices, so we can't calculate any contribution
margins. However, we do have the cost to purchase each unit externally. So we can use
the difference between the cost to purchase externally and the variable cost to produce
internally in dace of the contribution margin, because the cost to purchase externally can
be looked at as the amount saved by producing the units internally. Furthermore, the sales
price veil I be the same regardless of how the product is obtained, so revenue is really
irrelevant because it does not differ between the alternatives. The cost to purchase XT
externally is $60, and the variable cost to produce it internally is $55. So the savings from
producing it internally are $5 per unit, and that $5 divided by the number of machine
hours required to produce one unit (1 hour) is the savings per machine hour, which is $5.
The cost to purchase RP externally is $45, and the variable cost to produce it internally is
$40. So the savings from producing it internally are also $5 per unit. But it takes 15 hours
to produce one unit of RP internally. Therefore, the savings per machine hour are $5
divided by 15, or $3,333.
Since the savings per machine hour for producing XT internally are $5 and the savings
per machine hour for producing RP internally are only $3.33, the company should give
priority to producing XT internally. The company has 12,000 machine hours available. It
requires 12,000 units of XT, and each unit of XT requires 1 machine hour. Therefore, the
company should manufacture all 12,000 units of XT that it requires using its 12,000
available machine hours. It will then need to purchase all of the RP it needs externally.
D. This is not the most cost-effective solution. See correct answer for an explanation.

Question #29: The following information regarding inventory policy was assembled. The
company uses a 50-week year in all calculations.

Correct Answer : B. 2,700 units.


A. 5,500 units.
B. 2,700 units.
C. 1,200 units.
D. 240 units.
Explanation:
Answer (A) is incorrect.
This number of units equals the order size plus the safety stock
Answer (B) is correct.
The reorder point is the inventory level at which an order should be placed. It can be
quantified using the following equation:
Reorder point = (Average weekly demand x Lead time) + Safety stock
= [(12,000 units + 50 weeks) x 5 weeks) + 1,500 units
= 1,200 units + 1,500 units
= 2,700 units
Answer (C) is incorrect.
This number of units omits safety stock
Answer (D) is incorrect.
This number of units is the average weekly usage.

Question #30: Fact pattern: The information below pertains to Devlin Company.
Statement of Financial Position as of May 31
(In Thousands)

Year2 Year1
Assets
Current assets
Cash $45 $38
Trading securities 30 20
Accounts receivable (net) 68 48
Inventory 90 80
Prepaid expenses 22 30
Total current assets $255 $216
Investment, at equity 38 30
Property, plant, and equipment (net) 375 400
Intangible assets (net) 80 45
Total assets $748 $691
Liabilities
Current liabilities
Notes payable $35 $18
Accounts payable 70 42
Accrued expenses 5 4
Income taxes payable 15 16
Total current liabilities $125 $80
Long term debt 35 35
Deferred taxes 3 2
Total liabilities $163 $117
Equity
Preferred stock, 6%, $100 par Value,
cumulative $150 $150
Common stock, $10 par value 225 195
Additional paid-in capital--common stock 114 100
Retained earnings 96 129
Total equity $585 $574

Income Statement for the year ended May 31


(In Thousands)

Year2 Year1
Net sales $480 $460
Costs and expenses
Costs of goods sold 330 315
Selling, general and
administrative 52 51
Interest expense 8 9
Income before taxes $90 $85
Income taxes 36 34
Net income $54 $51

Question: Devlin Company’s rate of return on assets for the year ended May 31, Year 2, was
Correct Answer: B. 7.5%
A. 7.2%
B. 7.5%
C. 7.8%
D. 11.3%
Explanation:
Answer (A) is incorrect.
The figure of 7.2% uses ending total assets instead of average total assets.
Answer (B) is correct.
The rate of return on assets equals net income divided by average total assets. Accordingly,
the rate of return is 7.5% ($54 ÷ (($748 + $691) ÷ 2)).
Answer (C) is incorrect
Net income divided by beginning total assets equals 7.8%.
Answer (D) is incorrect.
The return on sales is 11.3%.

Question #31: Last year, a corporation had a total asset turnover ratio of 1.5, a profit margin
of 10%, and an equity multiplier of 2. This year, if the profit margin is 8%, but the return on
equity stays the same, then what could be true?
Correct Answer D. The equity multiplier increases to 3.0, and the total asset turnover
decreases to 1.25.
A. The equity multiplier remains 2.0, and the total asset turnover increases to 1.7.
B. The equity multiplier remains 2.0, and the total asset turnover increases to 3.5.
C. The equity multiplier increases to 2.2, and the total asset turnover remains 1.5.
D. The equity multiplier increases to 3.0, and the total asset turnover decreases to 1.25.
Explanation:
Answer (A) is incorrect.
This would yield a return on equity of 27.2% (8% x 1.7 x 2.0).
Answer (B) is incorrect
This would yield a return on equity of 56% (8% x 3.5 x 2.0).
Answer (C) is incorrect
This would yield a return on equity of 26.4% (8% x 1.5 x 2.2).
Answer (D) is correct
The return on equity (ROE) for last year is 30% (10% x 15 x 2). Given these facts, this year's
ROE is also 30% (8% x 1.25 x 3.0).

Question #32: A company wants to evaluate the purchase of a new machine.


• The machine costs $48,000 plus $2,000 in shipping and installation charges.
• Purchase of the machine would require an increase in net working capital of $5,000.
• The company depreciates this asset according to a Modified Acceleration Cost Recovery
System (MACRS) depreciation schedule for a five-year asset (Year 1= 0.20, Year 2 = 0.32,
Year 3 = 0.19, Year 4 = 0.12, Year 5 = 0.11 and Year 6 = 0.06).
• The company expects that the computer would increase its before-tax revenues by $30,000
a year, but would also increase operating costs by 10,000 a year.
• The company expects to sell the machine after five years for $8,000.
• The firm's marginal tax rate is 40%.
What is the initiation cash flow in Year 0 for the capital budgeting project?
Correct Answer. C. ($55,000).
A. ($48,000).
B. ($50,000).
C. ($55,000).
D. ($53,000).
Explanation:
The calculation of the incremental cash flows at initiation is
• Direct effect for the cost of the new asset ($48,000)
• Additional capitalized expenditures ($2,000)
• Increase in net working capital ($5.000)
• Initiation cash flow ($55,000)

Question #33: Which of the following will reduce average production costs following a
merger?
Correct Answer B. The existence of economies of scale.
A. A conglomerate merger.
B. The existence of economies of scale.
C. A vertical merger.
D. Net operating losses of an acquired firm.
Explanation:
Answer (A) is incorrect.
A conglomerate merger is the combination of firms in unrelated industries.
Answer (B) is correct
A reason to merge exists if the value of the combined firm exceeds the sum of the values of
the separate firms. The combined firm may operate more efficiently. Following a merger, if
the average cost of production falls as a result of production level increases, then there are
economies of scale.
Answer (C) is incorrect
A vertical merger is when a firm combines with a supplier or customer.
Answer (D) is incorrect.
A net operating loss of an acquired firm disappears in taxable reorganizations.

Question #34: A large arithmetic error was made in the preparation of its year-end financial
statements by improper placement of an extra digit in the calculation of bad debt expense
allowance. The error caused the net income to be reported at almost half of the proper
amount. In accordance with GAAP, correction of the error when discovered in the next year
should be treated as
Correct Answer D. A prior-period adjustment to the opening retained earnings balance.
A. An increase in bad debt expense for the year in which the error is discovered.
B. A component of income for the year in which the error is discovered, but separately listed
on the income statement and fully explained in a note to the financial statements.
C. An extraordinary item for the year in which the error was made.
D. A prior-period adjustment to the opening retained earnings balance.
Explanation:
Answer (A) is incorrect.
Correction of the error when discovered in the next year should be treated as a prior-period
adjustment to the opening retained earnings balance, not simply as an increase in bad debt
expense in the next year.
Answer (B) is incorrect.
Correction of the error when discovered in the next year should be treated as a prior-period
adjustment to the opening retained earnings balance. It is not a component of this year's
income and thus should be included in retained earnings.
Answer (C) is incorrect.
Correction of the error when discovered in the next year should be treated as a prior-period
adjustment to the opening retained earnings balance, as it is not an extraordinary item.
Answer (D) is correct.
An accounting error results from (1) a mathematical mistake. (2) a mistake in the application
of GAAP, or (3) an oversight or misuse of facts existing when the statements were prepared.
An accounting error related to a prior period is reported as a prior-period adjustment by
restating the prior-period statements.
Question #35: A corporation's inventory expressed as a percentage of current assets increased
from 25% last July to 35% this July. The factor that is least likely to cause this increase is
that the corporation
Correct Answer A. Is a seasonal company with traditionally higher activity in the summer
months.
A. Is a seasonal company with traditionally higher activity in the summer months.
B. Is beginning to experience high growth.
C. Has inventory that is becoming obsolete.
D. Used a material amount of cash from selling its short-term investments to purchase land.
Explanation:
Answer (A) is correct.
This statement is least likely to explain an increase in current assets from last July to this
July. If the corporation was a seasonal company with traditionally higher activity in the
summer months, it would budget similar amounts for each summer in expectation of the high
activity. The sudden increase in current assets for the following summer would not be
explained by the fact that they are a seasonal company.
Answer (B) is incorrect.
If the corporation was beginning to experience high growth, it would have to purchase more
inventory in order to meet the higher demand from the growth. This would cause the current
assets account to increase.
Answer (C) is incorrect
Obsolete inventory refers to inventory held by a company that is at the end of its product life
cycle and has not seen any sales or wage for a set period of time. If the corporation has
inventory that is becoming obsolete, the inventory will be held by the company instead of
being sold. This would cause an increase in the current assets on the books.
Answer (D) is incorrect.
It can be assumed that the sale of short-term investments generated a gain, which caused a
bigger cash inflow than the outflow from the short-term investments. This means that the
current assets account increased. The fact that the company bought land with some of the
cash is meant as a distractor, as the question does not state how much of the cash was used to
purchase the land.

Question #36: A company issued long-term bonds and used the proceeds to repurchase 40%
of the outstanding shares of its stock This financial transaction will likely cause the
Correct Answer C. Times interest earned ratio to decrease.
A. Total assets turnover ratio to increase.
B. Current ratio to decrease.
C. Times interest earned ratio to decrease.
D. Fixed charge coverage ratio to increase.
Explanation:
Answer (A) is incorrect.
The total assets turnover ratio is unaffected.
Answer (B) is incorrect.
The current ratio is unaffected.
Answer (C) is correct.
The times interest earned ratio equals earnings before interest and taxes divided by interest
expense. If bonds replace some equity in the capital structure interest expense will increase in
the denominator, which has the effect of reducing the ratio.
Answer (D) is incorrect.
The fixed charge coverage ratio will decrease.

Question #37: A company has 7.000 obsolete toys carried in inventory at a manufacturing
cost of $6 per unit. If the toys are reworked for $2 per unit, they could be sold for $3 per unit.
If the toys are scrapped, they could be sold for $1.85 per unit. Which alternative is more
desirable (rework or scrap) and what is the total dollar amount of the advantage of that
alternative?
Correct Answer A. Scrap, $5,950.
A. Scrap. $5,950.
B. Scrap. $47,950.
C. Rework $36,050.
D. Rework. $8,050.
Explanation:
A. The manufacturing cost of $6.00 is a sunk cost and not relevant to the decision. The net
revenue from reworking the obsolete toys is $7,000 (($3.00 - $2.00) x 7,000)]. Revenue if
the toys are scrapped is $12,950 ($1.85 x 7,000). The dollar advantage for selling as scrap
is $5,950 ($12,950 - $7,000).
This could also be solved by multiplying the per unit difference by 7,000. The net
advantage to reworking and selling the inventory is $1 per unit ($3 sales price minus $2
rework cost). The net advantage to selling the inventory for scrap is $1.85 per unit.
Selling the inventory for scrap will net the company $0.85 per unit more than reworking
it to sell, $0.85 x 7,000 = $5,950.
B. This is the rework cost ($2) plus the sale price of the reworked units ($3) plus the revenue
from selling them for scrap ($185) multiplied by 7,000. That is not the total dollar
amount of the advantage of one of the alternatives.
C. This is the present inventory cost of $6 plus the net realizable value of $1 minus the scrap
revenue of $1.85, the quantity multiplied by 7,000. That is not the total dollar amount of
the advantage of one of the alternatives.
D. This answer results from subtracting $1.85 from $3.00 and multiplying the difference by
7,000. This ignores the cost of the rework.

Question #38: A summary of an income statement is shown below.


Sales $15,000,000
Cost of goods sold (9,000,000)
Operating expenses (3,000,000)
Interest expense (800,000)
Taxes (880,000)

Based on the above information, the degree of financial leverage is


Correct Answer B. 1.36
A. 0.96
B. 1.36
C. 1.61
D. 2.27
Explanation:
Answer (A) is incorrect.
The degree of financial leverage is calculated by dividing earnings before interest and taxes by
earnings before taxes.
Answer (B) is correct.
The degree of financial leverage (DFL) equals earnings before interest and taxes (EBIT) divided
by earnings before taxes, which can be calculated as follows:
Sales $ 15,000,000
Cost of goods sold (9,000,000)
Operating expenses (3,000,000)
EBIT S 3,000,000
Interest expense (800,000)
Earnings before taxes $ 2,200,000
Taxes (880,000)
Net income $ 1,320,000
DFL = EBIT÷ Earnings before taxes
= $3,000,000+ $2,200,000
= 1.36
Answer (C) is incorrect
The degree of financial leverage is calculated by dividing earnings before interest and taxes by
earnings before taxes.
Answer (D) is incorrect.
The figure 2.27 results from improperly using net income in the denominator.

Question #39: Average daily collection of checks for a firm is $40,000. The firm also writes on
the average $35,000 of checks daily. If the collection period for checks is 5 days, calculate the
net float
Correct Answer A. $25,000
A. $25,000
B. $40,000
C. $175,000
D. $200,000
Explanation:
Answer (A) is correct
The difference between collections and payables is $5,000 daily. Five days' worth amounts to
$25,000 of float.
Answer (B) is incorrect.
The amount of $40,000 ignores the impact of payable and the 5-day float period.
Answer (C) is incorrect
The amount of $175,000 ignores the impact of receivables.
Answer (D) is incorrect
The amount of $200,000 ignores the impact of payables.
Question #40: With respect to a given risk an expected loss is
Correct Answer A. An amount that management expects to be lost per year on average over a
period of several years.
A. An amount that management expects to be lost per year on average over a period of
several years.
B. The amount expected to be lost every year to that risk.
C. The amount that management thinks could be lost to the risk in a very bad year.
D. A loss that management knows will occur.
Explanation:
A. An amount that management expects to be lost per year on average over a period of
several years is the definition of expected loss. The expected loss to a given risk can be
calculated as a weighted average of the possible losses to that risk weighted according to
their probabilities of occurring.
B. An expected loss is not the amount expected to be lost every year to a given risk.
C. The amount that management thinks could be lost to the risk in a very bad year is the
maximum probable loss.
D. If management knows without a doubt that a loss veil I occur, there is no risk involved.

Question #41: When calculating ratios involving income, an adjustment is most likely to be
made for
Correct Answer C. Nonrecurring gains and losses.
A. Gross profit.
B. Selling expenses.
C. Nonrecurring gains and losses.
D. Fixed overhead costs.
Explanation:
Answer (A) is incorrect.
Income is less likely to be adjusted for recurring costs.
Answer (B) is incorrect.
Income is less likely to be adjusted for recurring costs.
Answer (C) is correct.
Nonrecurring gains and losses are sometimes added to or subtracted from income to arrive at
income from continuing operations. Because ratios are used to predict the future, non-recurring
items not likely to recur should not be considered.
Answer (D) is incorrect.
Income is less likely to be adjusted for recurring costs.

Question #42: The yield curve shown implies that the

Correct Answer C. Long-term interest rates have a higher annualized yield than short-term rates.
A. Credit risk premium of corporate bonds has increased.
B. Credit risk premium of municipal bonds has increased.
C. Long-term interest rates have a higher annualized yield than short-term rates.
D. Short-term interest rates have a higher annualized yield than long-term rates.
Explanation:
Answer (A) is incorrect.
The yield curve does not reflect the credit risk premium of bonds.
Answer (B) is incorrect.
The yield curve does not reflect the credit risk premium of bonds.
Answer (C) is correct.
The term structure of interest rates is the relationship between yield to maturity and time to
maturity. This relationship is depicted by a yield curve. Assuming the long-term interest rate
is an average of expected future short-term rate, the curve will be upward sloping 'Mien
future short-term interest rates are expected to rise. Furthermore, the normal expectation is
for long-term investments to pay higher rates because of their higher risk Thus, long-term
interest rates have a higher annualized yield than short-term rates.
Answer (D) is incorrect.
Long-term interest rat.es should be higher than short-term rates.
Question #43: A company has extra cash at the end of the year and is analyzing the best way
to invest the funds. The company should invest in a project only if the
Correct Answer A. Expected return on the project exceeds the return on investments of
comparable risk
A. Expected return on the project exceeds the return on investments of comparable risk
B. Return on investments of comparable risk exceeds the expected return on the project
C. Expected return on the project is equal to the return on investments of comparable risk
D. Return on investments of comparable risk equals the expected return on the project.
Explanation:
Answer (A) is correct.
Investment risk is analyzed in terms of the probability that the actual return on an investment
will be lower than the expected return. Comparing a project's expected return with the return
on an asset of similar risk helps determine whether the project is worth investing in. If the
expected return on a project exceeds the return on an asset of comparable risk, the project
should be pursued.
Answer (B) is incorrect
A project should be pursued only if its expected return exceeds the return on investments of
similar risk
Answer (C) is incorrect
A project should be pursued only if its expected return exceeds the return on investments of
similar risk.
Answer (D) is incorrect.
A project should be pursued only if its expected return exceeds the return on investments of
similar risk.

Question #44: Verla Industries is trying to decide which one of the following two options to
pursue. Either option will take effect on January 1st of the next year.
Option One - Acquire a New Finishing Machine.
The cost of the machine is $1,000,000 and will have a useful life of five years. Net pre-tax
cash flows arising from savings in labor costs will amount to $100,000 per year for five
years. Depreciation expense will be calculated using the straight-line method for both
financial and tax reporting purposes. As an incentive to purchase. Verla will receive a trade-
in allowance of $50,000 on their current fully depreciated finishing machine.
Option Two - Outsource the Finishing Work
Verla can outsource the work to LM Inc. at a cost of $200,000 per year for five years. If they
outsource. Verla will scrap their current fully depreciated finishing machine.
Verla's effective income tax rate is 40%. The weighted-average cost of capital is 10%.
When comparing the two options, the $50,000 trade-in allowance would be considered
Correct Answer • C relevant because it is a decrease in cash outflow.
A. Relevant because it is an increase in cash outflows.
B. Irrelevant because it does not affect cash.
C. Relevant because it is a decrease in cash outflow.
D. Irrelevant because it does not affect taxes.
Explanation:
A. The trade-in allowance is relevant because it differs between alternatives and it does have
an impact on the cash flows. However, the trade-in allowance will reduce the cash
outflow under Option One, not increase it, because it offsets the purchase price.
B. The trade-in allowance is relevant because it differs between alternatives and it does have
an impact on the cash flows. The trade-in allowance will reduce the cash outflow under
Option One, since it offsets the purchase price. However, it will not be available under
Option Two.
C. Relevant cash flows are those expected future cash flows that differ between alternatives.
The trade-in allowance will reduce the cash outflow under Option One, since it offsets
the purchase price. However, it will not be available under Option Two. Therefore, it
differs between alternatives and is a relevant cash flow.
D. Relevant cash flows are expected future cash flows that differ between alternatives. Since
the trade-in allowance applies to only one of the options, it differs between alternatives
and is a relevant cash flow.

Question #45: Susan Hines has developed an estimate of the earnings per share for her firm, a
reseller of lawn equipment, for the next year using the following parameters.

Sales $20 million


Cost of goods sold 70%of sales
General & administrative
expenses $300,000
$100,000 plus 10% of
Selling expense sales
$5 million @ 8% interest
Debt outstanding rate
Effective tax rate 35%
Common shares outstanding 2 million
She is now interested in the sensitivity of earnings per share to sales forecast changes. A 10%
sales increase would increase earnings per share by
Correct Answer: A. 13.0 cents per share.
A. 13.0 cents per share.
B. 7.0 cents per share.
C. 10.4 cents per share.
D. 20.0 cents per share.
Explanation:
A. Since the firm is a reseller, all of its costs in cost of goods sold are variable costs because
they represent the cost to purchase the product that was resold. The current gross profit
margin is $6,000,000 ($20,000,000 x 0.30). If sales increase by 10%, the gross profit
margin will increase by 10% of $6,000,000, or $600,000.
Selling expense has a fixed component and a variable component. The variable
component will increase by 10% of the increased sales. Sales will increase by $2,000,000
(10% of $20,000,000), so selling expense will increase by $2,000,000 x 0.10, or
$200,000. Net income before tax will therefore increase by $600,000 - $200,000, or
$400,000. The increase in net income after tax will be $400.000 x (1- 0.35), which is
$260,000. There are 2,000,000 shares outstanding so the increased net income after tax
per share will be $260,000 ÷ 2,000,000, which is $0.13 per share.
B. This is not the correct answer. Please see the correct answer for an explanation We have
been unable to determine how to calculate this incorrect answer choice. If you have
calculated it please let us know how you did it so we can create a full explanation of why
this answer choice is incorrect.
C. This answer results from increasing earnings per share by 10%to reflect the 10% increase
in sales. Only revenue and variable expenses will increase as a result of the increase in
sales. Fixed expenses will be unchanged. For that reason, the percentage increase in
earnings per share will not be the same as the percentage of increase in sales.
D. This answer results from using the amount of increase in net income before tax. The
increase in net income after tax should be used instead.

Question #46: As related to relevant cash flow categories, direct effect refers to the:
Correct Answer C. immediate effect that a cash inflow, outflow, or commitment has on cash
flows.
A. Total outflows to fund an investment and initiate a project
B. Net increase or decrease in tax depreciation charges.
C. Immediate effect that a cash inflow, outflow, or commitment has on cash flows.
D. Change in a firm's taxpaying obligations.
Explanation: By definition, direct effect refers to the immediate effect that a cash inflow,
outflow, or commitment has on cash flows.

Question #47: Which type of real option would a firm most likely choose if uncertainty about the
initial investment is high and the potential loss of immediate cash flows are small?
Correct Answer D. Postpone
A. Adapt.
B. Abandon.
C. Expand.
D. Postpone
Explanation:
The postpone (wait-and-see) option is the equivalent of owning a call option on an
investment project. The call option is exercised when a firm commits to the investment
project. Deferral is desirable when uncertainty about the investment is high and the
immediate cash inflows lost or postponed are small.

Question #48: Katelyn is the Controller for Hobbie Corners; a company that produces
manufactured homes. Hobbie Corners year-end is approaching and the forecasted
profitability report for the year is significantly lower than expected. Andrew. a senior
Management Accountant that reports to Katelyn. is responsible for forecasting the year-end
profitability reports, in an effort to lower costs, both Andrew and Katelyn have been asked to
join a committee to review operations. The committee selected several issues to address
immediately in an attempt to improve profitability.
Andrew has been asked to work with the purchasing department to select vendors that can
offer raw materials at lower costs without compromising quality. Hazel Enterprises can
provide many of the key raw materials needed at a lower cost: however, the owner of the
company is related to Katelyn. After preparing a cost savings analysis. Andrew recommends
that Hazel Enterprises be selected as a new vendor.
Katelyn has been asked to review the percentage of completion calculations on the homes
that are currently in process. Katelyn has the ability to change the percentage of completion
calculation which would directly impact profitability for the company. Katelyn has modified
the calculation and forwarded the amended report to the auditors.
The IMA Statement of Ethical Professional Practice describes the overarching ethical
principle of Fairness. Identify an example of fairness relevant to the Hobbie Corners case
above.
Correct Answer C. Selecting vendors without bias, prejudice, or favoritism.
A. Ensuring information on reports and statements is accurate.
B. Conveying information at the appropriate time.
C. Selecting vendors without bias, prejudice, or favoritism.
D. Disclosing all necessary and relevant information to outside auditors.
Explanation:
Fairness requires empathetic just consideration of the needs of others involved in a particular
situation and full disclosure of all necessary contextual information The organization
deserves adequate and full disclosure that Hazel Enterprises can provide high quality
products at a lower cost: however, the relationship with the controller must be disclosed.

Question #49: Fact Pattern: Assume the following information pertains to Ranier Company.
Matson Company, and for their common industry for a recent year.
Industry
Ramer Matson Average
Current ratio 3.50 2.80 3.00
Accounts receivable
turnover 5.00 8.10 6.00
Inventory turnover 6.20 8.00 6.10
Times interest earned 9.00 12.30 10.40
Debt to equity ratio 0.70 0.40 0.55
Return on investment 0.15 0.12 0.15
Dividend payout ratio 0.80 0.60 0.55
Earnings per share $3.00 $2.00 __

Question: The attitudes of both Ramer and Matson concerning risk are best explained by the
Correct Answer D. Debt to equity ratio and times interest earned.
A. Current ratio, accounts receivable turnover, and inventory turnover.
B. Dividend payout ratio and earnings per share.
C. Current ratio and earnings per share.
D. Debt to equity ratio and times interest earned.
Explanation:
Answer (A) is incorrect.
These are liquidity ratios that do not concern risk incurrence.
Answer (B) is incorrect.
EPS does not indicate management's intent with respect to risk.
Answer (C) is incorrect
The current ratio and EPS are not indicators of the level of risk accepted.
Answer (D) is correct.
Matson is the more conservative company because it is less highly leveraged (lower debt to
equity ratio and a higher interest coverage). Moreover, it also pays out a smaller portion of its
earnings in the form of dividends (lower dividend payout ratio). These ratios reflect
management intent.

Question #50: Given an acid test ratio of 2.0, current assets of $5,000, and inventory of
$2,000, the value of current liabilities is
Correct Answer A. $1,500
A. $1,500
B. $2,500
C. $3,500
D. $6,000
Explanation:
Answer (A) is correct
The add test, or quick ratio equals the quick assets (cash, marketable securities, and accounts
receivable) divided by current liabilities. Current assets equal the quick assets plus inventory
and prepaid expenses. (This question assumes that the entity has no prepaid expenses.) Given
current assets of $5,000, inventory of $2,000, and no prepaid expenses, the quick assets must
be $3,000. Because the acid test ratio is 2.0, the quick assets are double the current liabilities.
Current liabilities therefore are equal to $1,500 ($3,000 quick assets ÷ 2.0).
Answer (B) is incorrect
Dividing the current assets by 2.0 results in $2,500. Current assets includes inventory, which
should not be included in the calculation of the acid test ratio.
Answer (C) is incorrect
Adding inventory to current assets rather than subtracting it results in $3,500.
Answer (D) is incorrect.
Multiplying the quick assets by 2 instead of dividing by 2 results in $6,000.
Question #51: The expected maximum probable loss to a commercial building is
Correct Answer D. inversely related to the size of the building.
A. Directly related to the size of the building
B. Lower if the building is vacant.
C. The complete destruction of the building and all of its contents.
D. Inversely related to the size of the building.
Explanation:
A. The maximum probable loss (also called the probable maximum loss, or PML) is the
largest loss that can occur under foreseeable circumstances. This is the largest amount of
damage that is likely to occur in a very bad year. When we say that something is directly
related to something else, it means they move in the same direction. When one thing
increases. The other thing also increases: and when one thing decreases, the other thing
also decreases. It is not accurate to say that the expected maximum probable loss to a
commercial building increases as the size of the building increases. The larger the
building is, the less probability there is that the maximum probable loss will occur. So
even though the amount of the maximum probable loss may be greater for a large
building than it is for a smaller building, the probability of that amount of loss occurring
is much smaller fora large building than it is for a small building. When the probability of
the maximum probable loss occurring is multiplied by the amount of the maximum
probable loss, the expected maximum probable loss will generally be smaller for a large
building than fora small building.
B. The expected maximum probable loss increases if the building is vacant. A vacant
building is more vulnerable to destruction than a building that is occupied. When
occupants are in the building, they are aware 'Mien something happens that could cause
damage, and they do something about it For instance, if a fire starts in a building that is
occupied, the people get out and call the fire department. But if a fire starts in a vacant
building, the fire could burn for a very longtime and cause a lot of damage before anyone
would notice and call the fired department.
C. The complete destruction of the building and all of its contents is the definition of the
maximum possible loss, not the definition of the maximum probable loss. The definition
of the maximum probable loss is the largest loss that can occur under foreseeable
circumstances, or the largest amount of damage that is likely to occur in a very bad year.
Damage greater than the maximum probable loss could occur, but in the judgment of
management it is very unlikely to occur.
D. The maximum probable loss (also called the probable maximum loss, or PML) is the
largest loss that can occur under foreseeable circumstances. This is the largest amount of
damage that is likely to occur in a very bad year. The expected maximum probable loss is
inversely related to the size of the building. When we say that something is inversely
related to something else it means that the two things move in opposite directions. When
one thing increases, the other thing decreases, and when one thing decreases, the other
thing increases. The larger a building is, the less probability there is that the largest loss
that can occur under foreseeable circumstances will occur. So as the size of the building
increases, the probability of maximum probable loss decreases. Even though the amount
of the maximum probable loss may be greater for a large building than it is for a smaller
building, the probability of that amount of loss occurring is much smaller for a large
building than it is for a small building. When the probability of the maximum probable
loss occurring is multiplied by the amount of the maximum probable loss, the expected
maximum probable loss will generally be smaller for a large building than fora small
building.

Question #52: A financial analyst calculated the company's degree of financial leverage as
1.5. If income before interest increases by 5%, earnings to shareholders will increase by
Correct Answer D. 7.50%
A. 1.50%
B. 3.33%
C. 5.00%
D. 7.50%
Explanation:
Answer (A) is incorrect.
The percentage of 1.5 results from equating the degree of financial leverage with its effect on
net earnings.
Answer (B) is incorrect.
The percentage of 3.33 results from improperly identifying the difference between EBIT
before the increase and EBIT after the increase (7.5% - 5% = 2.5%) as the numerator to
determine the degree of the effect (2.5% ÷ 7.5%= 3.33%).
Answer (C) is incorrect
Five percent results from equating the amount of the increase in EBIT with its effect on net
earnings.
Answer (D) is correct.
The degree of financial leverage (DFL) is the multiple of operating income (or earnings
before interest and taxes, called EBIT) over earnings before taxes (EBT).A high multiple
indicates heavy use of fixed costs in the firms capital structure, revealed by high interest
payments on debt. This firm's EBIT is 1.5 times EBT. Thus, a given percentage change in
EBIT will result in a change one-and-a-half times as great in EBT (1.5 x 5% = 7.5%).
Question #53: A company wants to evaluate the purchase of a new machine.
• The machine costs $48,000 plus $2,000 in shipping and installation charges.
• Purchase of the machine would require an increase in net working capital of $5,000.
• The company depreciates this asset according to a Modified Acceleration Cost Recovery
System (MACRS) depreciation schedule for a five-year asset (Year 1= 0.20, Year 2 = 0.32,
Year 3 = 0.19, Year 4 = 0.12, Year 5 =0.11, and Year 6 = 0.06).
• The company expects that the computer would increase its before-tax revenues by $30,000
a year, but would also increase operating costs by na000 a year.
• The company expects to sell the machine after five years for $8,000.
• The firm's marginal tax rate is 40%.
What is the incremental cash flow at disposal in Year 5?
Correct Answer C. $20,200.
A. $18,000.
B. $14,200.
C. $20,200.
D. $16,700.

Explanation:
Total acquisition cost for the new machine (including shipping and installation) = $48,000 +
$2,000 = $50,000
The income statement for year 5 prior to the sale of the machine is as follows:

Income Statement Prior to sale of machine:


$30,00
Operating Revenue 0
Operating Expenses 10,000
Operating Income before Depreciation 20,000
Depreciation expense (year 5: $50,000 ×
0.11) 5,500
Income before taxes 14,500
Taxes (40%) 5,800
Income after taxes $8,700
The book value on the machine at the end of year 5 is computed by taking the Original Cost
divided by the percentage of cost that has already been depreciated. Since the company is
using a MACRS as their depreciation method. 94% of the cost of the machine has already
been depreciated, therefore, $50,000 x 94% = $47,000. The book value of the machine at
time of sale is $3,000 ($50,000 — $47,000). The problem tells us that the machine is sold at
the end of year 5 for $8,000, providing a profit on the sale of $5,000 ($8,000 sale price —
$3,000 book value). Tax must be paid on the $5,000 profit, therefore an additional $2,000 in
tax expense (cash outflow) is incurred.
Therefore, the flow of physical cash in this problem is:

Cash in from new revenue generated $30,000


(10,000
Cash out from new expenses generated )
Cash received from sale of equipment 8,000
Cash out for tax expense (per income
statement) (5,800)
Cash out for tax expense on profit from
machine (2,000)
Incremental Cash Flow, Year 5 20,200

Question #54: There is a market for both product X and product Y. Which of the following
costs and revenues would be most relevant in deciding whether to sell product X or process it
further to make product Y?
Correct Answer. A. Additional cost of making Y. given the cost of making X and additional
revenue from Y.
A. Additional cost of making Y, given the cost of making X and additional revenue from Y.
B. Total cost of making X and the revenue from sale of X and Y.
C. Total cost of making Y and the revenue from sale of Y.
D. Additional cost of making X given the cost of making Y, and additional revenue from Y.
Explanation:
A. Relevant revenues and costs are those that will differ among the related options. The
relevant revenue and cost in this problem are the additional cost of making and selling
product Y and the additional revenue (over and above the revenue from X) to be earned
from selling product Y.
B. The cost of making product X is a sunk cost and not relevant.
C. Only incremental (differential) costs are considered relevant.
D. Product Y can only be processed after product X is completed.
Question #55: Please see the question below:
Averag
Averag e Averag
e fixed variable e total
Units of product cost cost cost
6 $15.00 $25.00 $40.00
7 12.86 24.00 36.86
8 11.25 23.50 34.75
9 10.00 23.75 33.75

The marginal cost of producing the ninth unit is


Correct Answer C $25.75
A. $23.75
B. $33.75
C. $25.75
D. $23.50
Explanation:
A. This is the average variable cost of producing 9 units. Marginal cost refers to the
additional cost of production from producing one additional unit. To find the marginal
cost of producing the ninth unit one must find the total cost for producing both 8 units
and 9 units of the good. The difference in total cost is the marginal cost of producing the
ninth unit
B. This is the average cost per unit of producing 9 units. Marginal cost refers to the
additional cost of production from producing one additional unit To find the marginal
cost of producing the ninth unit one must find the total cost for producing both 8 units
and 9 units of the good. The difference in total cost is the marginal cost of producing the
ninth unit.
C. Marginal cost refers to the additional cost of production from producing one additional
unit. To find the marginal cost of producing the ninth unit, one must find the total cost for
producing both 8 units and 9 units of the good. The difference in total cost is the marginal
cost of producing the ninth unit. This chart gives average total costs, not total costs.
When doing this calculation, the marginal cost of producing the 9th unit is $25.75. The
marginal cost of the 9th unit = Total cost of 9 units - Total cost of 8 units = ($33.75 x 9) -
($34.75 x 8) = $303.75 - $278.00, and the marginal cost of producing the 9th unit =
$25.75.
D. This is the average variable cost of producing 8 units. Marginal cost refers to the
additional cost of production from producing one additional unit. To find the marginal
cost of producing the ninth unit one must find the total cost for producing both 8 units
and 9 units of the good The difference in total cost is the marginal cost of producing the
ninth unit

Question #56: Smithco is considering the acquisition of scanning equipment to mechanize its
procurement process. The equipment will require extensive testing and debugging, as well as
user training prior to its operational use. Projected after-tax cash flows are shown below.
After-tax
Cash
inflows/
(outflows
Time period (year ) )
$
0 (550,000)
1 (500,000)
2 450,000
3 350,000
4 250,000
5 150,000

Management anticipates the equipment will be sold at the beginning of year 6 for $50,000 when
its book value is zero. Smithco's internal hurdle and effective tax rates are 14% and 40%,
respectively. The project's payback period will be:
Correct Answer. D. 4 years.
A. 3 years.
B. 2.3 years.
C. 3.5 years.
D. 4 years.
Explanation:
The payback is the length of time that it takes to recover the initial investment or, in this case,
the investments. The cumulative cash flow after 4 years is calculated as: Cumulative cash
flow after 4 years = (-$550,000 -$500,000 + $450,000 + $350,000 + $250,000) = 0.
Because the cash outflows are recovered by the fourth year, the payback is therefore 4 years.

Question #57: John Moore was recently hired as assistant controller of a manufacturing
company. The company controller. Nancy Kay, has forecasted a 16% increase in annual
earnings however, during the last quarter of the year. John estimates that the company will
only report a 12% increase in earnings. When he reports this to Nancy, she tells him that
meeting the numbers won't be a problem. She explains that there are several jobs in
production that will finish after the end of the fiscal year and she will record the associated
revenue in the accounting system for the current year.
What is the first step that John Moore should take at this time?
Correct Answer B. Follow his organization's established policies regarding the resolution of
this type of conflict
A. Contact his lawyer to determine his rights.
B. Follow his organization's established policies regarding the resolution of this type of
conflict
C. Notify the audit committee of the issue.
D. Discuss the issue with the CFO of another company, who does not know any employees
at John's company.
Explanation:
Before taking any steps. John Moore should check to see if his organization has established
policies regarding how to handle this type of conflict. If such policies exist, then he should
follow them.

Question #58: Risk assessment is a process that involves identifying the risks and
vulnerabilities that an organization is exposed to in order to identify events that may occur
and affect the entity negatively. How is a particular risk assessed or measured?
Correct Answer: D. By the loss frequency and the loss severity
A. By the amount of a loss, if a loss does occur
B. By the amount of the expected loss
C. By the probability of a loss occurring
D. By the loss frequency and the loss severity
Explanation:
A. The amount of a loss, if a loss does occur, is used in calculating the expected loss. and
expected loss is one of the terms used to express the measurement of a potential loss from
a specific risk However, the amount of a loss if it occurs is not the only input into
calculating expected loss and assessing risk
B. The expected loss, given a set of probabilities, is the amount that management expects to
be lost to a given risk on average in one year. Expected loss is one of the terms used to
express the measurement of a potential loss from a specific risk but it is not the way risk
is assessed.
C. The probability or probabilities of a loss occurring are used in calculating the expected
loss. The expected loss, given a set of probabilities, is the amount that management
expects to be lost to a given risk on average in one year. The probability of a loss
occurring is used in calculating the expected loss, and expected loss is one of the terms
used to express the measurement of a potential loss from a specific risk. However, the
probability of a loss occurring is not the only input into calculating expected loss and
assessing risk
D. Risk assessment is the process of analyzing and considering risks from two perspectives:
(1) the likelihood of the risk's occurring and (2) the potential impact of the event if it does
occur. The likelihood of the risks occurring is called the loss frequency. The potential
impact of the event if it does occur is called loss severity. Loss frequency is the
measurement of how often the loss occurs, on average. Loss frequency is expressed in
relation to a time period such as a year. A loss frequency of 0.25 per year means there is a
25%probability that a loss will take place in any given year, and on average, a loss occurs
once every four years. Loss severity measures how serious a loss is when it occurs, in
terms of cost. For example, historically when a loss has occurred, the average cost of the
loss is $50,000. Both the loss frequency and the loss severity are used in measuring the
potential loss that could occur from a specific risk and thus assessing the risk.

Question #59: Cervine Corporation makes two types of motors for use in various products.
Operating data and unit cost information for its products are presented below.
Product Product
A B
Annual unit capacity 10,000 20,000
Annual unit demand 10,000 20,000
Selling price $100 $80
Variable manufacturing cost 53 45
Fixed manufacturing cost 10 10
Variable selling $ administrative 10 11
Fixed selling & aldministrative 5 4
Fixed other administrative 2 0
Unit operating profit $20 $10

Machine hours per unit 2.0 1.5

Cervine has 40,000 productive machine hours available. The relevant contributions per machine
hour for each product to be utilized in making a decision on product priorities for the coming
year are
Correct Answer: C Product A $18.50. Product B $16.03.
A. Product A $17.00, product B $14.00
B. Product A $20.00, product B $10.00
C. Product A $18.50, product B $16.00
D. Product A $37.00, product B $24.00.
Explanation:
A. These are the selling prices minus the variable manufacturing cost, fixed manufacturing
cost and variable selling& administrative cost for each product. What is needed are the
unit contribution margins per machine hour. The unit contribution margins per machine
hour are the unit selling prices minus the unit variable costs (only variable costs) for each
product and the difference divided by the number of machine hours required for each
unit.
B. These are the unit operating profits, not the unit contribution margins per machine hour.
The unit contribution margins per machine hour are the unit contribution margins for
each product divided by the number of machine hours required for each unit
C. The unit contribution margin per machine hour is the unit contribution margin for each
product divided by the number of machine hours required for the production of each unit
of product. Product As unit contribution margin is $37 ($103 - $53 - $10), and it requires
2 machine hours per unit Therefore, its UCM per machine hour is $18.50. Product B's
unit contribution margin is $24 ($80 - $45 - $11), and it requires 1.5 machine hours per
unit Therefore, its UCM per machine hour is $16.
D. These are the unit contribution margins for each product. However, the unit contribution
margins per machine hour are needed. The unit contribution margins per machine hour
are the unit contribution margins for each product divided by the number of machine
hours required for each unit

Question #60: An economist determined the following market data for a commodity.
Quantit
y Quantity
Supplie Demande
Price d d
$25 250 750
$50 500 500
$75 750 250
$100 1,000 0

Based on this information, which one of the following statements is correct?


Correct Answer: D. In the long-run, if producers' costs per unit increase, then a reasonable
market clearing price could be $70.
A. In the short-term, there would be excess demand at a price of $70.
B. In the long-run, if producers' costs per unit decline, then a reasonable market clearing
price could be $65.
C. In the short-term, there would be excess supply at a price of $40.
D. In the long-run, if producers' costs per unit increase, then a reasonable market clearing
price could be $70.
Explanation:
A. At a price higher than the market clearing price, there is excess supply, not excess
demand.
B. The market clearing price is determined by the supply and demand cunt. While the cost
of production does impact the amount that the suppliers are willing to provide at a
specific price, a decrease in the cost of production would lead to a lower market clearing
price, not a higher market clearing price.
C. At a price of $40, there will be excess demand. This is because at a price below the
equilibrium price, there will be more people who want to buy the product than sell at that
price.
D. The market clearing price is determined by the supply and demand curves. It is the
equilibrium price where the supply curve crosses the demand curve. One of the factors
that will impact the amount supplied to the market is the cost of production. An increase
in the cost of production will cause the supply curve to shift to the left. When the supply
curve shifts to the left, at every market price, suppliers will be willing to supply a lower
amount of the good. The result would be a higher equilibrium price and a lower
equilibrium quantity. Suppliers will require a higher price in order to recover their costs
— and therefore be willing to sell their goods.

Question #61: A bank has received loan applications from three companies in the plastics
manufacturing business and currently has the funds to grant only one of these requests. Specific
data shown below has been selected from these applications for review and comparison with
industry averages.
Springfiel Herndo Industr
d Reston n y
Total sales
(millions $4.27 $3.91 $4.86 $4.30
Net profit margin 9.55% 9.85% 10.05% 9.65%
Current ratio 1.82 2.02 1.96 1.95
return on assets 12.0% 12.6% 11.4% 12.4%
Debt / equity
ratio 52.5% 44.6% 49.6% 48.3%
Financial
leverage 1.30 1.02 1.56 1.33
Based on the information above, select the strategy that druid be the most beneficial to Grand
Savings.
Correct Answer D. Grant the loan to R. as both the debt/equity ratio and degree of financial
leverage are below the industry average.
A. The bank should not grant any loans. as none of these companies represents a good credit
risk
B. Grant the loan to S, as all the Company's data approximate the industry average.
C. Grant the loan to R. as both the debt/equity ratio and degree of financial leverage are
below the industry average.
D. Grant the loan to H. as the company has the highest net profit margin and degree of
financial leverage.
Explanation:
Answer (A) is incorrect.
R is a good credit risk.
Answer (B) is incorrect.
Debt makes up more than half of S's capital structure: "approximating industry averages' is
meaningless when just a few percentage points can mean the difference between a good
credit risk and a poor one
Answer (C) is correct.
The bank's primary concern is the customer's ability to pay a loan back Crucial in deciding
the likelihood of payback is how much of the customer's capital structure is made up of debt
currently, that is, before the loan is made. R's is well below the industry average (a few
percentage points can mean the difference between a good credit risk and a poor one) and is
the lowest of the three potential customers. Also. R is clearly the least leveraged of the three
by far, as revealed by its low degree of financial leverage.
Answer (D) is incorrect.
While a high profit margin may be indicative of the ability to pay back a loan, a high degree
of financial leverage indicates the opposite, and H's is well above the industry average.

Question #62: Ticker Company sells two products. Product A provides a contribution margin
of $3 per unit, and Product B provides a contribution margin of $4 per unit. If Tickers sales
mix shifts toward Product A. which one of the following statements is correct?
Correct Answer A. Operating income will decrease if the total number of units sold remains
constant.:
A. Operating income will decrease if the total number of units sold remains constant
B. The total number of units necessary to break even will decrease.
C. The contribution margin ratios for Products A and B will change.
D. The overall contribution margin ratio will increase.
Explanation:
A. If Ticker's sales mix shifts toward Product Awhile the total number of units sold remains
the same, that means Ticker is selling more of Product A and less of Product B. Product
A has a lower contribution margin than Product B. If the company is selling more of the
lower contribution margin product and less of the higher contribution margin product the
total contribution margin will decrease. Because the total contribution margin decreases
while the total fixed costs remain the same, operating income will also decrease.
B. If Tickers sales mix shifts toward Product Awhile the total number of units sold remains
the same, that means Ticker is selling more of Product A and less of Product B. Product
A has a lower contribution margin than Product B. This change in the product mix will
cause the weighted average contribution margin to decrease, because more weight will be
put on the product with the lower contribution margin. The decrease in the weighted
average contribution margin will cause the total number of units necessary to break even
to increase, not decrease.
C. The individual contribution margin ratios for Products A and B will not change.
D. If Tickers sales mix shifts toward Product Awhile the total number of units sold remains
the same, that means Ticker is selling more of Product A and less of Product B. Product
A has a lower contribution margin than Product B. If the company is selling more of the
lower contribution margin product and less of the higher contribution margin product, the
total contribution margin will decrease, not increase.

Question #63: Katelyn is the Controller for Hobbie Corners; a company that produces
manufactured homes. Robbie Corners year-end is approaching and the forecasted profitability
report for the year is significantly lower than expected. Andrew, a senior Management
Accountant that reports to Katelyn, is responsible for forecasting the year-end profitability
reports. In an effort to lower costs, both Andrew and Katelyn have been asked to join a
committee to review operations. The committee selected several issues to address immediately in
an attempt to improve profitability. Andrew has been asked to work with the purchasing
department to select vendors that can offer raw materials at lower costs without compromising
quality. Hazel Enterprises can provide many of the key raw materials needed at a lower cost:
however, the owner of the company is related to Katelyn. After preparing a cost savings analysis.
Andrew recommends that Hazel Enterprises be selected as a new vendor. Katelyn has been asked
to review the percentage of completion calculations on the homes that are currently in process.
Katelyn has the ability to change the percentage of completion calculation which would directly
impact profitability for the company. Katelyn has modified the calculation and forwarded the
amended report to the auditors. The IMA Statement of Ethical Professional Practice describes the
overarching ethical principle of Responsibility. Identify an example of responsibility relevant to
the Hobbie Corners case above.
Correct Answer C. Ensuring information on reports and statements is accurate.
A. Disclosing all necessary and relevant information to outside auditors.
B. Providing information and feedback objectively.
C. Ensuring information on reports and statements is accurate.
D. Selecting vendors without bias, prejudice, or favoritism.
Explanation:
Responsibility requires actions to be performed with faithfulness and loyalty. Examples of
responsibility include conveying information at the appropriate time, ensuring information on
reports and statements is accurate, and gathering enough information to make an informed
decision. Katelyn has a responsibility to ensure that the information on the percentage of
completion report is accurate.

Question #64: Which of the following is not true about financial markets?
Correct Answer: C. In perfectly competitive markets, financial intermediaries act as price
setters to clear the market
A. Financial markets are the total supply and demand for securities.
B. Financial markets facilitate borrowing and lending of financial assets and obligations.
C. In perfectly competitive markets, financial intermediaries act as price setters to clear the
market
D. Financial markets change over time, causing people to adjust their pattern of
consumption.
Explanation:
Answer (A) is incorrect.
It is a true statement about financial markets.
Answer (B) is incorrect
It is a true statement about financial markets.
Answer (C) is correct.
Financial markets bring entities that have funds to invest together with entities that have
financing needs. They facilitate the transfer of assets and obligations. Due to this activity,
financial markets cause people to adjust their consumption patterns. Financial intermediaries
increase the efficiency of financial markets through better allocation of financial resources,
not by clearing the market.
Answer (D) is incorrect.
It is a true statement about financial markets.

Question #65: Edwards Products has just developed a new product with a manufacturing cost
of $30. The Marketing Director has identified three marketing approaches for this new
product.
Approach X Set a selling price of $36 and have the firm's sales staff sell the product at a 10%
commission with no advertising program. Estimated annual sales would be 10,000 units.
Approach Y Set a selling price of $38, have the firm's sales staff sell the product at a 10%
commission, and back them up with a $30,000 advertising program. Estimated annual sales
would be 12,000 units.
Approach Z Rely on wholesalers to handle the product Edwards would sell the new product
to the wholesalers at $32 per unit and incur no selling expenses. Estimated annual sales
would be 14,000 units.
Correct Answer: A. Z, X, Y.
A. Z, X, Y.
B. Y, Z, X
C. X, Y, Z.
D. Z, Y, X.
Explanation:
A. The best way to solve this is to create income statements for each of the three approaches.
The sales values are calculated based on selling price multiplied by units sold.
Manufacturing costs are the $30 given in the problem multiplied by the number of units
sold. Commissions under Approach X and Y are 10% of sales, and only Approach Y has
advertising expenses. Here are the income statements:
XYZ
Sales $360,000 $456.030 $448,000
Manufacturing Costs 300,000 360,000 420,000
Commissions 36,000 45,600 0
Advertising 0 30,000 0
Operating Income $ 24,000 $ 20,400 $ 28,000
The highest operating income is with Approach Z, next highest is Approach X and the
lowest is Approach Y.
B. The best way to solve this is to create income statements for each of the three approaches.
This answer could result from any error in calculations or in evaluating the results of the
calculations. See correct answer for details.
C. The best way to this is to create income statements for each of the three approaches. This
answer could result from any error in calculations or in evaluating the results of the
calculations. See correct answer for details.
D. The best way to solve this is to create income statements for each of the three approaches.
This answer could result from any error in calculations or in evaluating the results of the
calculations. See correct answer for details.
Question #66: A firm has decided to make an additional investment in its operating assets that
are financed by debt. Assuming all other factors remain constant this increase in investment will
have which one of the following effects?
Correct Answer B. No change (Operating Income Margin). Decrease (Operating Asset
Turnover). Decrease (Return on Operating Assets)
A. Increase (Operating Income Margin). No change (Operating Asset Turnover), Increase
(Return on Operating Assets)
B. No change (Operating Income Margin). Decrease (Operating Asset Turnover). Decrease
(Return on Operating Assets)
C. No change (Operating Income Margin), increase (Operating Asset Turnover), Decrease
(Return on Operating Assets)
D. Decrease (Operating Income Margin). Decrease (Operating Asset Turnover). Decrease
(Return on Operating Assets)
Explanation:
Answer (A) is incorrect.
Asset turnover will decrease.
Answer (B) is correct
If all else remains constant an additional investment in operating assets will not change
income but will affect the balance sheet Asset turnover will decrease because an uncharged
numerator (sales) will be divided by an increased amount of fixed assets. Return on operating
assets will decline because an unchanged numerator (income) will be divided by greater
assets.
Answer (C) is incorrect
Asset turnover will decrease.
Answer (D) is incorrect.
Operating income will not change.
Question #67: A firm is constructing a risk analysis to quantify the exposure of its data center to
various types of threats. Which one of the following situations would represent the highest
annual loss exposure after adjustment for insurance proceeds?

Frequenc Insuranc
y of Loss e (%
Occurrenc Amoun coverag
e (years) t e)
$15,00
I 1 0 85
II 8 75,000 80
200,00
III 20 0 80
400,00
IV 100 0 50

Correct Answer: D. I.
A. II.
B. IV.
C. III.
D. I.
Explanation:
A. The question asks for the highest annual loss exposure after adjustment for insurance
proceeds. The way to calculate that is to (1) calculate the loss after insurance
reimbursement for each situation by multiplying the loss amount by (1- insurance
coverage rate), and (2) divide each loss after reimbursement by the frequency of
occurrence in years to calculate the annual loss amount for each
B. The question asks for the highest annual loss exposure after adjustment for insurance
proceeds. The way to calculate that is to (1) calculate the loss after insurance
reimbursement for each situation by multiplying the loss amount by (1 - insurance
coverage rate), and (2) divide each loss after reimbursement by the frequency of
occurrence in years to calculate the annual loss amount for each.
C. The question asks for the highest annual loss exposure after adjustment for insurance
proceeds. The way to calculate that is to (1) calculate the loss after insurance
reimbursement for each situation by multiplying the loss amount by (1 - insurance
coverage rate), and (2) divide each loss after reimbursement by the frequency of
occurrence in years to calculate the annual loss amount for each.
D. The question asks for the highest annual loss exposure after adjustment for insurance
proceeds. The way to calculate that is to (1) calculate the loss after insurance
reimbursement for each situation by multiplying the loss amount by (1 - insurance
coverage rate), and (2) divide each loss after reimbursement by the frequency of
occurrence in years to calculate the annual loss amount for each, as follows:
Frequency Insurance Loss
of (% after
Occurrence Loss coverage Ins. Annual
(years) Amount ) Reimb. loss
I 1 $15,000 85 $2,250 $2,250
II 8 75,000 80 15,000 1,875
III 20 200,000 80 40,000 2,000
IV 100 400,000 50 200,000 2,000
The highest annual loss is I. with an annual loss exposure of $2,250.

Question #68: Despite its shortcomings, the traditional payback period continues to be a popular
method to evaluate investments because, in part, it:
Correct Answer: B. provides some insight into the risk associated with a project
A. furnishes information about an investment's lifetime performance.
B. provides some insight into the risk associated with a project
C. ignores the time value of money.
D. Focuses on income rather than cash flow.
Explanation:
Provides some insight into the risk associated with a project The payback method in capital
budgeting determines the number of years needed to recoup the net initial investment in a capital
budgeting project.

Question #69: The term "underwriting spread' refers to the


Correct Answer: C. Difference between the price the investment banker pays for a new security
issue and the price at which the securities are resold.
A. Commission percentage an investment banker receives for underwriting a security issue.
B. Discount investment bankers receive on securities they purchase from the issuing
company.
C. Difference between the price the investment banker pays for a new security issue and the
price at which the securities are resold.
D. Commission a broker receives for either buying or selling a security on behalf of an
investor.
Explanation:
Answer (A) is incorrect.
The underwriting spread is not based on a commission. The underwriter actually buys the
new securities and resells them at a price that is expected to result in a profit
Answer (B) is incorrect
The underwriting spread is not a genuine discount: it is simply the difference between the
price paid and the price received for a new security.
Answer (C) is correct.
An investment banker performs an underwriting or insurance function when it purchases an
issue of securities and then resells them. The risk of price fluctuations during the distribution
period is borne entirely by the investment banker. Investment banking is also an efficient
vehicle for marketing the securities because investment bankers are specialists in such
activities. The profit earned is the underwriting spread, or the difference between the
purchase and resale prices of the securities (effectively, the wholesale and retail prices).
Answer (D) is incorrect.
The underwriting spread is not based on a commission. The underwriter actually buys the
new securities and resells them at a price that is expected to result in a profit

Question #70: The Liabilities and Shareholders' Equity section of a Statement of Financial
Position is shown below.
January Decembe
1 r 31
Accounts payable $32,000 $84,000
Accrued liabilities 14,000 11,000
7% bonds payable 95,000 77,000
Common stock ($10 par value) 300,000 300,000
Reserve for bond retirement 12,000 28,000
Retained earnings 155,000 206,000
Total liabilities and shareholders' $608,00
equity 0 $706,000

Correct Answer: C. 32.2%.


A. 25.1%.
B. 25.6%.
C. 32.2%.
D. 33.9%.
Explanation:
Answer (A) is incorrect.
Accounts payable, accrued liabilities, and bonds payable are debt for debt-to-equity purposes.
Common stock reserve for bond retirement, and retained earnings are equity for debt-to-equity
purposes.
Answer (B) is incorrect.
Accrued liabilities and the bond payable are debt. Retained earnings is equity that should be
included in the calculation of the debt-to-equity ratio.
Answer (C) is correct.
The debt-to-equity ratio is 32.2% as shown below.
Debt-to-equity ratio = Total debt ÷ Equity
= ($84,000 + $11,000 + $77,000) ÷ ($300,000 + $28,000 + $206,000)
= $172,000 ÷ $534,000
= 32.2%
Answer (D) is incorrect.
The reserve for bond retirement needs to be included as equity.

Question #71: The U.S. dollar has a free-floating exchange rate. When the dollar has fallen
considerably in relation to other currencies, the
Correct Answer D. Cheaper dollar helps U.S. exporters of domestically produced goods.
A. Trade account in the U.S. balance of payments is neither in a deficit nor in a surplus
because of the floating exchange rates.
B. Capital account in the U.S. balance of payments is neither in a deficit nor in a surplus
because of the floating exchange rates.
C. Fall in the dollar's value cannot be expected to have any effect on the U.S. trade balance.
D. Cheaper dollar helps U.S. exporters of domestically produced goods.
Explanation:
Answer (A) is incorrect.
The trade account still has a deficit despite the cheaper dollar, imports continue to exceed
exports because patterns of consumption are slant° change.
Answer (B) is incorrect.
The capital account benefits from the cheaper dollar. Foreigners can buy more dollars with
fewer yen, marks, etc. Moreover, foreign capital inflow increases because of the federal
government's budget deficits. Hence, the U.S. experiences a net capital inflow.
Answer (C) is incorrect
The fall in the dollar has a positive effect on the nation's trade deficit Exports increase and
imports decrease.
Answer (D) is correct
A decline in the value of the dollar relative to other currencies lowers the price of US, goods
to foreign consumers. Thus, exporters of goods produced in the U.S. benefit. Simultaneously,
a low value for the dollar decreases imports by making foreign goods more expensive.

Question #72: in Year 3, gross profit margin remained unchanged from Year 2. But, in Year
3, the company's net profit margin declined from the level reached in Year 2. This could have
happened because, in Year 3.
Correct Answer: A. Corporate tax rates increased.
A. Corporate tax rates increased.
B. Cost of goods sold increased relative to sales.
C. Sales increased at a faster rate than operating expenses.
D. Common share dividends increased.
Explanation:
Answer (A) is correct.
Gross profit margin is net sales minus cost of goods sold. Net profit margin is gross profit
margin minus all remaining expenses and losses, one of which is income taxes. If corporate
tax rates increased, net profit margin would decrease, leaving gross profit margin unchanged.
Answer (B) is incorrect.
A change in cost of goods sold would have affected gross profit margin.
Answer (C) is incorrect
Sales increasing faster than operating expenses would have resulted in an increase, not a
decrease, to net profit margin.
Answer (D) is incorrect.
Any impact on dividends cannot be determined from the information given.
Question #73: Ten years ago, perpetual preferred shares with a par value of $50 and an
annual dividend rate of 6%were issued. Currently, there are no dividends in arrears. Since the
issue date, interest rates have risen, and the shares are now selling at $38. The markets
current required rate of return on these shares is
Correct Answer: C. 7.89%
A. 4.56%
B. 6.03%
C. 7.89%
D. 15.79%
Explanation:
Answer (A) is incorrect.
The markets current required rate of return is 7.89%.
Answer (B) is incorrect
The figure of 6.00% is the company's annual dividend rate.
Answer (C) is correct.
The required rate of return on these shares is calculated by dividing the dividend by the issue
price. Thus. $3 (6% x S50) must be divided by $38 to yield 7.89%.
Answer (D) is incorrect.
The figure of 15.79% results from incorrect using the dividend rate instead of calculating the
dividend to divide.

Question #74: The following information pertains to the year ended December 31:

Sales $720,000
Net income 120,000
Average total assets 490,000

Which one of the following formulas depicts the use of the DuPont model to calculate
Andrew's return on assets?
Correct Answer: C. (720,000 ÷ 480,000) × (120,000 ÷ 720,000)
A. (720,000 ÷ 480,000) x (720,000 ÷ 120,000)
B. (480,000 ÷ 720,000) x (720,000 ÷ 120,000)
C. (720,000 ÷ 480,000) x (120,000 ÷ 720,000)
D. (480,000 ÷ 720,000) x (120,000 ÷ 720,000)
Explanation:
Answer (A) is incorrect.
The DuPont model depicts return on assets as total asset turnover (sales divided by average
total assets) times the profit margin (net income divided by sales).
Answer (B) is incorrect.
The DuPont model depicts return on assets as total asset turnover (sales divided by average
total assets) times the profit margin (net income divided by sales).
Answer (C) is correct.
The DuPont model depicts return on assets as total asset turnover (sales divided by average
total assets) times the profit margin (net income divided by sales). Therefore, the ROA
calculation uses the formula [($720,000 ÷ $480,000) x ($120,000 + $720,000)].
Answer (D) is incorrect.
The DuPont model depicts return on assets as total asset turnover (sales divided by average
total assets) times the profit margin (net income divided by sales).

Question #75: Katelyn is the Controller for Hobbie Corners; a company that produces
manufactured homes. Hobbie Corners year-end is approaching and the forecasted
profitability report for the year is significantly lower than expected. Andrew, a senior
Management Accountant that reports to Katelyn. is responsible for forecasting the year-end
profitability reports, in an effort to lower costs, both Andrew and Katelyn have been asked to
join a committee to review operations. The committee selected several issues to address
immediately in an attempt to improve profitability. Andrew has been asked to work with the
purchasing department to select vendors that can offer raw materials at lower costs without
compromising quality. Hazel Enterprises can provide many of the key raw materials needed
at a lower cost: however, the owner of the company is related to Katelyn. After preparing a
cost savings analysis. Andrew recommends that Hazel Enterprises be selected as a new
vendor. Katelyn has been asked to review the percentage of completion calculations on the
homes that are currently in process. Katelyn has the ability to change the percentage of
completion calculation which would directly impact profitability for the company. Katelyn
has modified the calculation and forwarded the amended report to the auditors. The IMA
Statement of Ethical Professional Practice describes the first overarching ethical principle as
Honesty, identify an example of honesty relevant to the Hobbie Corners case above.
Correct Answer B. Disclosing all necessary and relevant information to outside auditors.
A. Ensuring information on reports and statements is accurate.
B. Disclosing all necessary and relevant information to outside auditors.
C. Selecting vendors without bias, prejudice, or favoritism.
D. Providing information and feedback objectively.
Explanation:
The first principle, honesty, requires conscientious application to the task at hand and
truthfulness in all analyses and communications. Examples of honesty include: disclosing all
necessary and relevant information to outside auditors: refusing to record information that is
anything less than accurate: and providing factual information to others so that they can make
decisions based on truthful information. In this case. Katelyn has a responsibility to disclose
all relevant information to the outside auditors to justify the amendments to the percentage of
completion report.

Question #76: Fact Pattern: Depoole Company is a manufacturer of industrial products that
uses a calendar year for financial reporting purposes. Assume that total quick assets exceeded
total current liabilities both before and after the transaction described. Further assume that
Depoole has positive profits during the year and a credit balance throughout the year in its
retained earnings account
Question: Depoole's payment of a trade account payable of $64,500will
Correct Answer: C. Increase both the current and quick ratios.
A. Increase the current ratio, but the quick ratio would not be affected.
B. Increase the quick ratio, but the current ratio would not be affected.
C. Increase both the current and quick ratios.
D. Decrease both the current and quick ratios.
Explanation:
Answer (A) is incorrect
The current ratio and the quick ratio will increase.
Answer (B) is incorrect
The current ratio and the quick ratio will increase.
Answer (C) is correct.
Current assets consist of more assets than quick assets; thus, if quick assets exceed current
liabilities, then current assets do also. It can also be concluded that both ratios are greater
than 1. An equal reduction in the numerator and the denominator, such as a payment of a
trade payable will cause each ratio to increase.
Answer (D) is incorrect
The current ratio and the quick ratio will increase.

Question #77: The Foreign Corrupt Practices Act is a U.S. law that prohibits U.S. companies
from:
Correct Answer: A. making 'corrupt" payments to foreign officials for the purpose of
obtaining or retaining business.
A. Making "corrupt" payments to foreign officials for the purpose of obtaining or retaining
business.
B. Exporting to countries that do not comply with US human rights regulations.
C. Selling products for corrupt unethical, or illegal purposes.
D. Making products in overseas markets that do not comply with the same safety and
environmental regulations as for domestically produced products.
Explanation:
The 1977 Foreign Corrupt Practices Act is a U.S. law that forbids U.S. companies from
obtaining contracts or business through the payment of bribes.

Question #78: A company has 480 million in current assets, comprised of $30 million in
inventory and $50 million in cash and marketable securities. The company's current liabilities
total $50 million. If the company purchases an additional $10 million in inventory with $10
million in cash, the effect of this transaction on the company would be to
Correct Answer B. Decrease the quick ratio while the current ratio remains unchanged.
A. Decrease the current ratio and increase the quick ratio.
B. Decrease the quick ratio while the current ratio remains unchanged.
C. Leave both the current ratio and the quick ratio unchanged.
D. Decrease the current ratio and decrease the quick ratio.
Explanation:
Answer (A) is incorrect.
Since the decrease in cash is offset by an increase in inventory, the current ratio remains
unchanged. Also, the decrease in cash decreases the quick ratio.
Answer (B) is correct.
The quick ratio decreases from 1.00 (S50 million + $50 million) to .80 ($40 million a S50
million) due to the $10 million decrease in cash. The current ratio remains unchanged
because the $10 million decrease in cash is offset by the $10 million increase in inventory.
Answer (C) is incorrect
The decrease in cash lowers the numerator in the quick ratio, which decreases the quick ratio.
Answer (D) is incorrect
Since the decrease in cash is offset by an increase in inventory, the current ratio remains
unchanged.

Question #79: Calvin Inc. is considering the purchase of a new state-of-art machine to
replace its hand-operated machine. Calvin's effective tax rate is 40%, and its cost of capital is
12%. Data regarding the existing and new machines are presented below.
Existing New
machine machine
Original cost $50,000 $90,000
Installation costs 0 4,000
Freight and insurance 0 6,000
Expected end salvage
value 0 0
straight- straight-
Depreciation method line line
Expected useful life 10 years 5 years

The existing machine has been in service for seven years and could be sold currently for
$25,000. If the new machine is purchased Calvin expects to realize a $30,000 before-tax
annual reduction in labor costs.
If the new machine is purchased, what is the net amount of the Initial cash outflow at Time 0
for net present value calculation purposes?
Correct Answer D. $79,000.
A. $65,000.
B. $75,000.
C. $100,000.
D. $79,000.
Explanation:
The initial cash outflow at time 0 is calculated as follows:
Initial cash outflow= (cost of machine) + (installation costs) + (freight and insurance) -
(proceeds from sale of existing machine) + (tax from sale of machine)
Tax from sale of machine = (tax rate) (proceeds from sale - net book value)
Net book value, end of year 7 = (original cost) (%of life remaining)
Net book value, end of year 7 = ($50,000) (0.3) = $15,000
Tax from sale of machine = (0.4) ($25,000 - $15,000) = $4,000
Initial cash outflow= ($90,000) + ($4,000) + ($6,000) - ($25,000) + ($4,000).
Initial cash outflow= $79,000.

Question #80: A corporation experienced the following year-over-year changes: Net profit
margin Increased 25% Total asset turnover Increased 40% Total assets Decreased 10% Total
equity Increased 40% Using DuPont analysis, what is the year-over-year change in return on
equity (ROE)?
Correct Answer: C. Increased 12.5%.
A. Increased 95.0%.
B. Increased 63.0%.
C. Increased 12.5%.
D. Increased 10.0%.
Explanation:
Answer (A) is incorrect.
The DuPont Model for ROE is as follows: Net profit margin x Total asset turnover x Equity
multiplier (Total assets + Total equity). The year-over-year change is not calculated by
simply adding and subtracting the increases and decreases from last year to this year. The
incorrect amount of 95% results from adding and subtracting the year-over-year changes
(25% + 40% -10% + 40%).
Answer (B) is incorrect.
The year-over-year change is not calculated by simply dividing the increase in the net profit
margin by the increase in the total asset turnover.
Answer (C) is correct.
The ROE using the DuPont analysis is calculated as follows: Net profit margin x Total asset
turnover x Equity multiplier (Total assets + Total equity). The best way to solve this problem
is to use actual numbers for the return on equity comparison of this year to last year.
Assuming that last year the corporation had a net profit margin of .025, total asset turnover of
1.05, total assets of $500,000, and total equity of $200,000, last year's ROE is equal to 6.56%
[.025 x 1.05 x ($500,000 + $200,000)]. By using the information given in the prob4em, the
current-year amounts can be calculated, resulting in a net profit margin of .03125 (increased
by 25%). total asset turnover of 1.47 (increased by 40%). total assets of 5450.000 (decreased
by 10%). and total equity of $280.000 (increased by 40%). Therefore, this year's ROE is
equal to 7.38% [.03125 x 1.47 x (450,000 + 280,000)]. The increase in ROE from last year to
this year can now be calculated as 12.5% [(7.38 - 6.56) ÷ 6.56].
Answer (D) is incorrect.
This answer choice incorrectly multiples the year-aver-year change for the net profit margin
by the year over-year change for the total asset turnover to get an increase in ROE of 10.0%.

Question #81: The management of a company is attempting to reduce the cost for Product X
by analyzing the trade-offs between different types of product features and total product cost.
What type of cost reduction strategy is the company using?
Correct Answer: B. Value engineering.
A. Total quality management.
B. Value engineering.
C. Activity-based costing.
D. Kaizen.
Explanation:
A. Total Quality Management describes an approach that is committed to customer
satisfaction and continuous improvement of products or services. The basic premise of
TQM is that quality improvement is a way of increasing revenues and decreasing costs,
but it is not a cost reduction strategy that attempts to reduce the cost for a product by
analyzing the trade-offs between different types of product features and total product
cost.
B. Value engineering is a cost-reduction strategy that attempts to reduce the cost for a
product by analyzing the trade-offs between different types of product features and total
product cost Value engineering involves evaluation of all the business functions in the
value chain with the objective of reducing costs while satisfying customer needs. This
evaluation may lead to design improvements, materials specification changes or
modifications in manufacturing methods. When performing value engineering
management distinguishes between a value-added cost and a non-value-added cost. If a
value-added cost were eliminated, it would reduce the products value, or usefulness, to
customers. Since value-added costs cannot be eliminated, value engineering seeks to
reduce their costs by improving efficiency. On the other hand, if a non-value-added cost
were eliminated, it would not reduce the value or utility of the product. A non-value-
added cost is a cost the customer is not willing to pay for. Examples of non-value-added
costs are costs for expediting, re-work and repair: and these are costs that can be reduced
through improvements to the manufacturing process.
C. Activity-based costing is a method of allocating manufacturing overhead costs to
products based on cost drivers. ABC identifies the causal relationship between the
incurrence of cost and activities, determines the underlying driver of activities,
establishes cost pools related to individual drivers, develops costing rats, and applies cost
to product on the basis of resources consumed (drivers). It is not a cost reduction strategy
that attempts to reduce the cost for a product by analyzing the trade-offs between
different types of product features and total product cost.
D. The term kaizen is a Japanese word that means 'improvement'. As used in business, it
implies "continuous improvement' or slow but constant incremental improvements being
made in all areas of business operations. It is not a cost reduction strategy that attempts to
reduce the cost for a product by analyzing the trade-offs between different types of
product features and total product cost.

Question #82: Fact Pattern: Depoole Company is a manufacturer of industrial products that uses
a calendar year for financial reporting purposes. Assume that total quick assets exceeded total
current liabilities both before and after the transaction described. Further assume that Depoole
has positive profits during the year and a credit balance throughout the year in its retained
earnings account
Question: Depooles early liquidation of a long-term note with cash affects the
Correct Answer: B. Quick ratio to a greater degree than the current ratio.
A. Current ratio to a greater degree than the quick ratio.
B. Quick ratio to a greater degree than the current ratio.
C. Current and quick ratio to the same degree.
D. Current ratio but not the quick ratio.
Explanation:
Answer (A) is incorrect
The quick ratio is affected to a greater degree than the current ratio.
Answer (B) is correct
The numerators of the quick and current ratios are decreased when cash is expended. Early
payment of a long-term liability has no effect on the denominator (current liabilities). Since
the numerator of the quick ratio, which includes cash, net receivables, and marketable
securities, is less than the numerator of the current ratio, which includes all current assets, the
quick ratio is affected to a greater degree.
Answer (C) is incorrect.
The quick ratio is affected to a greater degree than the current ratio.
Answer (D) is incorrect
The quick ratio is affected to a greater degree than the current ratio.

Question #83: A company plans to tighten its credit policy. The new policy will decrease the
average number of days in collection from 75 to 50 days and will reduce the ratio of credit
sales to total revenue from 70% to 60%. The company estimates that projected sales will be
5% less if the proposed new credit policy is implemented. If projected sales for the coming
year are $50 million, calculate the dollar impact on accounts receivable of this proposed
change in credit policy. Assume a 360-day year.
Correct Answer: C. $3,333,334 decrease.
A. $6,500,000 decrease.
B. $3,819,445 decrease.
C. $3,333,334 decrease.
D. $18,749,778 increase.
Explanation:
Answer (A) is incorrect.
This figure is the decrease in credit sales.
Answer (B) is incorrect.
This figure is calculated using total, not credit sales.
Answer (C) is correct.
Projected credit sales for the year under the old credit policy were $35 million ($50,000,000
x 70%). The level of average receivables was calculated as follows:
Receivables turnover = Days in year ÷ Average collection period
= 360 days + 75 days
= 4.8 times per year
Average receivables = Net credit sales ÷ Receivables turnover
= $35,000,000 ÷ 4.8 times
= $7,291,667
Under the new policy, total sales will be $47.5 million ($50,000,000 x 95%), and credit sales
will be $28.5 million ($47,500,000 x 60%). The new level of average receivables is
calculated as follows:
Receivables turnover = Days in year ÷ average collection period
= 360 days + 50 days
= 7.2 times per year
Average receivables = Net credit sales ÷ Receivables turnover
= $28,500,000 ÷ 7.2 times = $3,958,333
The average receivables balance will therefore be reduced by $3,333,334 ($7,291,667 -
$3,958,333).
Answer (D) is incorrect.
Receivables will decrease.
Question #84: In order to increase production capacity. Gunning Industries is considering
replacing an existing production machine with a new technologically improved machine
effective January 1. The following information is being considered by Gunning Industries.
• The new machine would be purchased for $160,000 in cash. Shipping, installation, and
testing would cost an additional $30,000.
• The new machine is expected to increase annual sales by 20,000 units at a sales price of $40
per unit Incremental operating costs are comprised of $30 per unit in variable costs and total
fixed costs of $40,000 per year.
• The investment in the new machine will require an immediate increase in working capital
of $35,000.
• Gunning uses straight-line depreciation for financial reporting and tax reporting purposes.
The new machine has an estimated useful life of five years and zero salvage value.
• Gunning is subject to a 40% corporate income tax rate.
Gunning uses the net present value method to analyze investments and will employ the
following factors and rates.
Present value of an
Present value of $1 at Ordinary Annuity $1 at
Period 10% 10%
1 0.909 0.909
2 0.826 1.736
3 0.751 2.487
4 0.683 3.170
5 0.621 3.791

Gunning Industries' discounted annual depreciation tax shield for the first year of operation
would be:
Correct Answer: B. $13,817.
A. $22,800.
B. $13,817.
C. $15,200.
D. $20,725.
Explanation:
The depreciation tax shield for a period is calculated by taking the depredation for the period and
multiplying it by the relevant tax rate Using straight-line depreciation, the annual depreciation
charge is calculated as: Annual depreciation charge, straight-line depreciation = (depreciable
base) / (estimated service life) For Gunning, the depreciable base will include the initial cost of
the machine and the shipping, installation and testing. Therefore, the depreciable base will be
calculated as follows:
Depreciable base = $160,000+$30,000 = $190,000
Annual depreciation charge, straight-line depreciation = ($160,000 + $30,000) / (5 years)
Annual depreciation charge, straight-line depreciation = $190,000/ 5 years = $38,000
The annual depreciation tax shield is calculated by taking the annual depreciation and
multiplying it by the tax rate. as follows:
Annual depreciation tax shield = (annual depreciation) (tax rate)
Annual depreciation tax shield = ($38,000) (0.40) = $15,200
The discounted annual depreciation tax shield is calculated by taking the annual depreciation tax
shield and discounting it by the appropriate present value of $1 factor, as follows:
Discounted annual depreciation tax shield = (annual depreciation tax shield) (present value of $1
factor)
The present value of $1 factor for 10% at the end of year 1 is 0.909.
Discounted annual depreciation tax shield = ($15,200) (0.909) = $13,817.

Question #85: Which one of the following items is least likely to directly impact an equipment
replacement capital expenditure decision?
Correct Answer: B. The net present value (NPV) of the equipment that is being replaced.
A. The amount of additional accounts receivable that will be generated from increased
production and sales.
B. The net present value (NPV) of the equipment that is being replaced.
C. The depreciation rate that will be used for tax purposes on the new asset.
D. The sales value of the asset that is being replaced.
Explanation:
The NPV of the equipment that is being replaced is a sunk cost and would not be considered
in the decision to replace the equipment.
Question #86: Which one of the following actions may increase a company's return on
assets?
Correct Answer C. Replacement of capital equipment via an operating lease.
A. Purchase of a new corporate headquarters.
B. An increase in inventory levels for a future store expansion.
C. Replacement of capital equipment via an operating lease.
D. Reduction of long-term debt through the issuance of common stock
Explanation:
Answer (A) is incorrect.
ROA is calculated by dividing a firm's net income by its average total assets. Since this
action causes total assets to increase. ROA would decrease overall.
Answer (B) is incorrect.
ROA is calculated by dividing a firm's net income by its average total assets. Since this
action causes total assets to increase. ROA would decrease overall.
Answer (C) is correct.
Return on assets, or ROA (also called return on total assets. or ROTA), is a straightforward
measure of how well management is deploying the firm's assets in the pursuit of a profit.
ROA is calculated by dividing a firm's net income by its average total assets.
Answer (D) is incorrect.
This action would not affect ROA.

Question #87: An automobile company that uses the futures market to set the price of steel to
protect a profit against price increases is an example of
Correct Answer: B. A long hedge.
A. A short hedge.
B. A long hedge.
C. Selling futures to protect the company from loss.
D. Selling futures to protect against price declines.
Explanation:
Answer (A) is incorrect.
A short hedge is a futures contract that is sold to protect against price declines. The
automobile company wishes to protect itself against price increases.
Answer (B) is correct
A change in prices can be minimized or avoided by hedging. Hedging is the process of using
offsetting commitments to minimize or avoid the impact of adverse price movements. The
automobile company desires to stabilize the price of steel so that its cost to the company will
not rise and cut into profits. Accordingly, the automobile company uses the futures market to
create a long hedge, which is a futures contract that is purchased to protect against price
increases.
Answer (C) is incorrect
The automobile company needs to purchase futures in order to protect itself from loss, not
sell futures. Selling futures protects against price declines.
Answer (D) is incorrect.
It is the definition of a short hedge, which is used for avoiding price declines. The automobile
company wants to protect itself against price increases.

Question #88: A toothbrush manufacturer has noticed a shift of customer preferences in its
growing Asian sales market towards an electronic battery operated toothbrush from a manual
toothbrush. This shifting of customer tastes best represents what type of risk to the
toothbrush manufacturer?
Correct Answer D. Strategic risk
A. Business risk
B. Financial risk
C. Operational risk
D. Strategic risk
Explanation:
A. A shifting of customer tastes does not represent business risk to the toothbrush
manufacturer. Business risk for a firm is the risk of changes in its earnings before interest
or taxes. Business risk depends on a variety of factors, including the variability of
demand, sales price, and the price of inputs as well as the amount of the company's
operating leverage. The more stable all of these variables are, the less business risk a
company will experience.
B. A shifting of customer tastes does not represent financial risk to the toothbrush
manufacturer. Financial risk is risk connected to the financial health of the company.
Examples include volatility of foreign currencies, volatility of interest rates, volatility of
commodity prices (inputs), credit risk liquidity risk and market risk Furthermore. when a
company borrows, it obligates itself to pay interest charges and to repay the principal,
creating financial risk Financial risk related to borrowing includes two aspects: 1. the risk
that the firm will not be able to pay its interest and other obligations when they become
due because of lack of cash flow, and 2. the increased variability in earnings per share
caused by the use of debt and the accompanying requirement to pay interest on the debt
C. A shifting of customer tastes does not represent operational risk to the toothbrush
manufacturer. Operational risk is risk that results from inadequate or failed internal
processes, people or systems. Some examples of operational risks are technology,
business continuity, customer satisfaction, and the risk of product or service failure.
Operational risk includes legal risk and compliance risk Legal risk is the risk that is
associated with uncertainty due to legal actions or uncertainty in the applicability or
interpretation of contracts, laws or regulations where the company operates. Compliance
risk is the current or future risk to profits or the company's assets as a result of violations
of, or nonconformance with, laws, rules, regulations, required practices, internal policies
and procedures, or ethical standards.
D. A shifting of customer tastes best represents strategic risk to the toothbrush manufacturer.
Strategic risk is risk such as reputation risk brand risk (patent and trademark protection),
leadership risk and the risk of customers' needs charging. Strategic risks also include
risks related to actions of competitors and changes in the regulations businesses are
subject to, as regulatory changes could cause significant increases in compliance expense.
Capital availability is another strategic risk.

Question #89: A corporation has 200,000 shares of common stock outstanding. Net income for
the recently ended fiscal year was $500,000, and the stock has a price-earnings ratio of eight.
The board of directors has just declared a three-for-two stock split. For an investor who owns
100 shares of stock before the split, the approximate value (rounded to the nearest dollar) of the
investment in the corporation's stock immediately after the split is
Correct Answer: $2,000
A. $250
B. $1,333
C. $2,000
D. $3,000
Explanation:
Answer (A) is incorrect.
The amount of $250 represents the annual earnings on 100 shares.
Answer (B) is incorrect
The amount of $1,333 assumes that the value of the total investment declines after the split
Answer (C) is correct.
EPS equals $2.50 ($500,000 NI ÷ 200,000 pre-split shares). Thus, 100 shares had a value of
$2,000 (100 shares x $2.50 EPS x 8 P/E ratio) before the split. This value is unchanged by
the stock split. Although the stockholder has more shares, the total value of the investment is
the same.
Answer (D) is incorrect.
The amount of $3,000 assumes that the value of the investment as well as the number of
shares increases by 50%.
Question #90: Julie is the senior management accountant for Hazelton Manufacturing, a
multinational telecommunications company. In an effort to expand operations overseas.
Hazelton encourages senior management to offer payment to foreign officials to win
business. These payments are classified as normal commission expenses by the accounting
department. Julie recently questioned whether these payments should be classified as
commission expense since they appear to be bribes to foreign officials. When Julie discussed
the situation with the chief financial officer, he stated these payments were not illegal and
were expected within these foreign countries. When all employees are hired, they are
required to sign the company code of conduct. In addition, the company provides annual
ethics training to all employees and each employee is evaluated based on compliance with
operational goals and ethical expectations. The company provides an anonymous
whistleblower hotline for employees to report concerns to management. Julie believes that
the company she works for has an ethical organizational culture.
Identify a requirement of Section 406 of the Sarbanes-Oxley Act relevant in the Hazelton
Manufacturing case.
Correct Answer C. The act requires senior financial officers to follow a code of ethics.
A. The act requires a company to provide a whistleblowing hot line to report ethics
concerns.
B. The act requires employee training for maintaining an ethical organizational culture.
C. The act requires senior financial officers to follow a code of ethics.
D. The act forbids an American company to pay bribes to foreign government officials.
Explanation: Section 406 of the Sarbanes-Oxley Act requires companies to adopt (or explain
why they have not adopted) a code of ethics for senior financial officers.

Question #91: The I MA Statement of Ethical Professional Practice Integrity standard states
that each member has a responsibility for all of the following except:
Correct Answer D. Communicate information fairly and objectively.
A. Abstain from engaging in or supporting any activity that might discredit the profession.
B. Mitigate actual conflicts of interest
C. Regularly communicate with business associates to avoid apparent conflicts of interest
D. Communicate information fairly and objectively.
Explanation:
The IMA Statement of Ethical Professional Practice Integrity standard states that each
member has a responsibility to: mitigate actual conflicts of interest, regularly communicate
with business associates to avoid apparent conflicts of interest advise all parties of any
potential conflicts, refrain from engaging in any conduct that would prejudice carrying out
duties ethically, and abstain from engaging in or supporting any activity that might discredit
the profession. Communicating information fairly and objectively is one of the Statement's
Credibility standards.

Question #92: What is the major difference between traditional risk management and
Enterprise Risk Management (ERM)?
Correct Answer: C. In traditional risk management, each individual business unit does its
own risk management, whereas with ERM, the major risk areas for the organization as a
whole are identified, and the individual units use that as a basis to define the specific risks
they face.
A. Traditional risk management considers only risks that could have a negative effect on the
organization, whereas ERM includes risks that could have either a negative or a positive
effect on the organization.
B. Traditional risk management focuses on risks that can be mitigated by insurance, whereas
ERM includes risks that cannot be mitigated by insurance but can be mitigated in other
ways.
C. In traditional risk management, each individual business unit does its own risk
management, whereas with ERM, the major risk areas for the organization as a whole are
identified, and the individual units use that as a basis to define the specific risks they face.
D. Traditional risk management prepares an organization to respond to specific recognized
events that could prevent the organization from achieving its objectives. ERM prepares
the organization to respond to any event that could prevent it from achieving its
objectives whether or not the event has been previously recognized as a risk
Explanation:
A. Both traditional risk management and ERM focus on risks that could prevent the
organization from achieving its objectives, so both focus on risks that could have a
negative effect.
B. Both traditional risk management and ERM focus on risks that can be mitigated in
various ways, one of which is by purchasing insurance.
C. In traditional risk management, each individual business unit does its own risk
assessment and risk management taking into consideration the risks it faces individually.
With ERM, the organization as a whole identifies the major risk areas that may affect
various operating units in different ways, and this general model serves as a basis for
individual unit managers to define the specific risks facing their units.
D. ERM does not prepare an organization to respond to any event that could prevent it from
achieving its objectives but only to events that have been identified in advance as having
the potential to prevent it from achieving its objectives.

Question #93: Julie is the Senior Management Accountant for Hazelton Manufacturing: a multi-
national telecommunications company. In an effort to expand operations overseas. Hazelton
encourages senior management to offer payment to foreign officials to win business. These
payments are classified as normal commission expenses by the accounting department Julie
recently questioned whether these payments should be classified as commission expense since
they appear to be bribes to foreign officials. When Julie discussed the situation with the Chief
Financial Officer, he stated these payments were not illegal and were expected within these
foreign countries. When all employees are hired, they are required to sign the company code of
conduct. In addition, the company provides annual ethics training to all employees and each
employee is evaluated based on compliance with operational goals and ethical expectations. The
company provides an anonymous whistleblower hotline for employees to report concerns to
management. Julie believes that the company she works for has an ethical organizational culture.
Identify a requirement of the U.S. Foreign Corrupt Practices Act (FCPA) relevant in the
Hazelton Manufacturing case.
Correct Answer B. The Act forbids an American company to pay bribes to foreign government
officials.
A. The Act requires senior financial officers to follow a code of ethics.
B. The Act forbids an American company to pay bribes to foreign government officials.
C. The Act requires employee training for maintaining an ethical organizational culture.
D. The Act requires a company to provide a whistleblowing hotline to report ethics
concerns.
Explanation:
The FCPA forbids an American company doing business in another country to pay bribes to
a foreign government to obtain contracts or secure business.
Question #94: An investment project requires an initial investment of $100,000. The project is
expected to generate net cash inflows of $28,000 per year for the next five years. Assuming a
12% cost of capital, the project's payback period is:
Correct Answer C. 3.57 years.
A. 3.36years.
B. 0.28 years.
C. 3.57years.
D. 1.4 years.
Explanation:
A project's payback (PB) is the length of time it takes for the cash flows (CFs) generated by
the initial investment (I) to equal the initial investment. Assuming the cash flows occur
uniformly throughout the year, the project's PB is 3.57years The PB = I/CF =
$100,000/28,000 = 3.57 years.

Question #95: Financial risk management is


Correct Answer: B. the use of financial instruments to manage exposure to credit market and
other financial risks to create economic value to the company.
A. Deciding which risks must be addressed and prioritizing them.
B. The use of financial instruments to manage exposure to credit market and other financial
risks to create economic value to the company.
C. Managing risks connected with the day-to-day operations of the company.
D. The level of risk that resides with an event or process prior to management taking a
mitigating action.
Explanation:
A. Deciding which risks must be addressed and prioritizing those that must be addressed is
one of the steps in the management of any type of risk, not only financial risk
management.
B. The process of using financial instruments to manage exposure to credit, market and
other financial risks in order to create value for the company is financial risk
management.
C. This is the definition of operational risk management not financial risk management.
D. This is the definition of inherent risk
Question #96: A company's capital structure consists entirely of long-term debt and common
equity. The cost of capital for each component is shown below.
Long-term debt: 8%
Common equity: 15%
The company pays taxes at a rate of 40%. If the weighted average cost of capital is 10.41%, what
proportion of the company's capital structure is in the form of long-term debt?
Correct Answer: B. 45%
A. 34%
B. 45%
C. 55%
D. 66%
Explanation:
Answer (A) is incorrect.
This percentage results from simply dividing the 8% cost of debt by the 15%cost of common
stock.
Answer (B) is correct.
The effective rate for debt is the after-tax cost [8% x (1.0 -.40 tax rate) = 4.8%]. The formula for
weighted-average cost of capital can be solved as follows:
(Debt weight x Cost of debt) + (Equity weight x Cost of equity) = WACC
(Debt weight x.048) + (Equity weight x .15) = .1041
[(1 - Equity weight) x .048] + (Equity weight x .15) = .1041
.048 - (.048 x Equity weight) - (Equity weight x .15) = .1041 –
(.048 x Equity weight) + (Equity weight x .15) = .0561
Equity weight x .102 = .0561
Equity weight = .55
Since equity is 55% of the capital structure, debt makes up 45%.
Answer (C) is incorrect
This percentage is the proportion of WACC in the form of common equity.
Answer (D) is incorrect.
This percentage results from improperly subtracting the percentage of common equity rather than
the tax rate [8%x (1.0 - .15) = 6.8%] and (6.8 ÷ 10.41) = 65.32%.
Question #97: In which product-mix pricing strategy is it appropriate for the seller to accept any
price that exceeds the storage and delivery costs for the product?
Correct Answer: A. By-product pricing
A. By-product pricing.
B. Captive-product pricing
C. Product-bundle pricing.
D. Optional-product pricing.
Explanation:
A. A by-product is a product of little value that was produced during the production process.
Any amount received above and beyond storage and delivery costs will be accepted
because it will improve profit.
B. An example of captive-product pricing would be a printer and printer ink. The ink is
called captive because it must be used along with the printer. It is often the case that the
seller will make more money on the captive product (the ink) than on the main product
(the printer). Therefore, the captive product would have to be priced above the costs of
storage and delivery.
C. An example of product-bundle pricing would be season tickets to the theater. The price of
the season ticket is lower than the price would be to purchase tickets to each production
separately. This is done to promote the sale of tickets. Although the price of the season
ticket is lower than if the tickets were bought separately, it is high enough so the seller
still can make a profit on the sale. Therefore, the price would have to above the costs of
storage and delivery.
D. Optional-product pricing optional products, features or services that are offered along
with the main product as upgrades or options. The optional product or products have
costs, so they must be priced above their storage and delivery costs.

Question #98: Bruell Electronics Co. is developing a new product, surge protectors for high-
voltage electrical flows. The following cost information relates to the product:
Unit
Costs
Direct
materials $3.25
Direct labor 4.00
Distribution 0.75
The company will also be absorbing $120,000 of additional fixed costs associated with this new
product. A corporate fixed charge of $20,000 currently absorbed by other products will be
allocated to this new product. How many surge protectors (rounded to the nearest hundred) must
Bruell Electronics sell at a selling price of $14 per unit to gain $30,000 additional income before
taxes?
Correct Answer: D. 25,000 units.
A. 10,700 units.
B. 20,000 units.
C. 12.100 units.
D. 25,000 units.
Explanation:
A. This is not the correct answer. Please see the correct answer for an explanation.
B. This is not the correct answer. Please see the correct answer for an explanation
C. This is not the correct answer. Please see the correct answer for an explanation.
D. The volume needed to earn the desired pre-tax income is calculated by dividing the
additional fixed costs phis the desired pretax income by the contribution margin. Fixed
costs + desired pre-tax additional profit = $120,000 + $30,000, or $150,000. $150.000+
[$14 - ($3.25 + $4.00 + $0.75)] = 25,000 units.

Question #99: A chief financial officer has been tracking the activities of the company's nearest
competitor for several years. Among other trends, the CFO has noticed that this competitor is
able to take advantage of new technology and bring new products to market more quickly than
the CFO's company. In order to determine the reason for this, the CFO has been reviewing the
following data regarding the two companies:

Company Competitor
Accounts receivable turnover 6.85 7.35
Return on assets 15.34 14.74
Times interest earned 15.65 12.45
Current ratio 2.11 1.23
Debt/equity ratio 42.16 55.83
Degree of financial leverage 1.06 1.81
Price/earnings ratio 26.56 26.15

On the basis of this information, which one of the following is the best initial strategy for the
CFO to follow in attempting to improve the Flexibility of the company?
Correct Answer: C. Seek additional sources of outside financing for new product introductions.
A. Seek cost cutting measures that would increase the company's profitability.
B. Investigate ways to improve asset efficiency and turnover times to improve liquidity.
C. Seek additional sources of outside financing for new product introductions.
D. Increase the company's investment in short-term securities to increase the current ratio.
Explanation:
Answer (A) is incorrect.
Cutting costs makes it harder to take advantage of new opportunities or to innovate. Cost
cutting is a last resort and the company's return on assets is already better than its
competitors.
Answer (B) is incorrect
The receivables turnover is not much different than that of the competitor.
Answer (C) is correct.
The company's times interest earned, debt/equity ratio, and degree of financial leverage all
reveal that the company is less leveraged than its competitor. The two firms' price-earnings
ratios are comparable, so the company should be able to raise new capital fairly easily, either
debt or equity. Thus, the company should seek additional sources of outside financing for
new product introductions.
Answer (D) is incorrect.
Increasing investment in short-term securities would not change the current ratio.

Question #100: Direct foreign investment allows firms to avoid


Correct Answer: C. Trade restrictions imposed on foreign companies in the customers'
market.
A. Exposure to political risk
B. The cost of exchange rate fluctuations.
C. Trade restrictions imposed on foreign companies in the customers' market.
D. Domestic regulations on the use of foreign technology.
Explanation:
Answer (A) is incorrect.
Direct foreign investment increases exposure to political risk
Answer (B) is incorrect.
Direct foreign investment increases exposure to exchange rate risk
Answer (C) is correct.
Reasons for international business expansion, known as direct foreign investment, can be
both revenue oriented (seeking new markets or avoiding trade restrictions) and cost-oriented
(seeking cheaper inputs or favorable exchange rates.
Answer (D) is incorrect.
A multinational company is subject to its home country’s regulations on the use of foreign
technology.

MOCK TEST 9

Question#1: A retail company has experienced rapid growth in sales during the current year.
An analyst has calculated the following ratios for this company.
Prior year current year
Inventory turnover 5.4 9.3
Receivables turnover 4.2 3.5
Fixed asset turnover 2.4 3.6
Quick ratio 1.5 1.2

Based on the above, the analyst may conclude that sales increased due to more open in
current year.
Correct Answer: C. favorable credit policies.
A. Control over inventory levels.
B. Competitive pricing.
C. Favorable credit policies.
D. Stores open in current year
Explanation:
A. Sales would not increase as a result of more control over inventory levels.
B. There is no information in the question that would indicate any changes in pricing.
C. The receivables turnover ratio decreased, which indicates that receivables are being
collected more slowly in the current year. If sales increased, that increase could be
due to more favorable credit policies which could mean that more customers with
slower payment records received credit.
D. There is no information given that would lead to a conclusion that more stores were
open.
Question #2: Which one of the following would cause the demand curve for bagels to shift
to the left?
Correct Answer: B. A decrease in the cost of muffins.

A. An increase in the population.


B. A decrease in the cost of muffins.
C. An increase in the supply of bagels.
D. A decrease in the price of bagels.

Explanation:
A decrease in the cost of muffins would cause the demand curve for bagels to shift to
the left as demand for muffins would become greater. Substitutes can affect demand for
a product or service, as can complements. If the price of one product drops and it can be
substituted for a higher priced product, the demand for the higher priced product will
decrease and the demand curve will shift to the left.

Question #3: The correct priority of claims in a bankruptcy liquidation is

Correct Answer: A. Administrative expenses, wage claims, taxes due, claims of general
or unsecured creditors, and shareholder claims.

A. Administrative expenses, wage claims, taxes due, claims of general or unsecured


creditors, and shareholder claims.
B. Administrative expenses, wage claims, taxes due, shareholder claims, and debt
holder claims.
C. All wage claims, administrative expenses, debt holder claims, taxes due, and
shareholder claims.
D. All wage claims, administrative expenses, debt holder claims, shareholder claims,
and taxes due.

Explanation:
Answer (A) is correct.
After secured creditors receive the proceeds of the sale of specific collateral, the
other assets are distributed according to the following scheme: (1 ) administrative
expenses, (2) claims of gap creditors, (3) wages of no more than a certain amount,
(4) unpaid contributions to employee benefit plans, (5) customer deposits, (6) taxes,
(7) certain unfunded pension plan liabilities, (8) claims of general or unsecured
creditors, (9) claims of preferred shareholders, and (10) claims of common
shareholders.

Answer (B) is incorrect.


Debt holders receive payment before shareholders.

Answer (C) is incorrect.


Administrative expenses come first and taxes come before debt holder claims.

Answer (D) is incorrect.


Administrative expenses come first and taxes come before debt holders.

Question #4: A company has decided to self-insure for its employees' medical
insurance. This is an example of

Correct Answer: A. retaining the risk.

A. Retaining the risk.


B. Reducing the risk.
C. Transferring the risk.
D. Exploiting the risk.

Explanation:

A. Retaining risk means bearing the risk, such as when a firm chooses to self-insure
by budgeting for and paying the cost of any losses out of its own funds. Self-
insuring employees' medical costs is an example of retaining a risk. A company
would choose to do this if it believed that the cost to pay the expenses and to
administer the pay outs would be less than the cost to purchase insurance to
cover the expenses.
B. Reducing or mitigating the risk includes activities such as risk diversification,
for example splitting the IT function into two separate geographical areas so that
one area can back up the other one if a natural disaster strikes and destroys one
of the IT canter’s. Self-insuring employees' medical expenses does not reduce
the risk.
C. Transferring or sharing risk usually means purchasing insurance. Self-insuring is
not the same thing as purchasing insurance.
D. Exploiting a risk involves exposing the firm to risk in order to take advantage of
a situation and thus increase the value of the firm. Self-insuring against the risk
of employees' medical expenses does not increase the value of the firm.
Question #5: Which one of the following inventory cost flow assumptions will result
in a higher inventory turnover ratio in an inflationary economy?
Correct Answer: D. LIFO
A. Specific identification
B. Weighted average.
C. FIFO
D. LIFO
Explanation:
A. The inventory turnover ratio is calculated as cost of goods sold divided by
average inventory Therefore; the inventory method that gives the lowest value of
inventory will create a higher inventory turnover. Even under specific identification;
the inventory turnover will not be higher than LIFO. This is because LIFO assumes
that all of the items in inventory are the oldest whereas specific identification will
have some old and some new units in ending inventory.
B. The inventory turnover ratio is calculated as cost of goods sold divided by average
inventory Therefore; the inventory method that gives the lowest value of inventory
will create the highest inventory turnover ratio. Weighted average will give an
ending inventory value that is between that of LIFO and FIFQ so it will not be either
the lowest or the highest inventory balance.
C. The inventory turnover ratio is calculated as cost of goods sold divided by average
inventory Therefore; the inventory method that gives the lowest value of inventory
will create a higher inventory turnover ratio. In an inflationary economy, FIFO
provides the highest value for the inventory since it assumes that the oldest
(cheapest) units are sold first and the newest (most expensive) units are n inventory
at the end of the period.
D. The inventory turnover ratio is calculated as cost of goods sold divided by
average inventory Therefore; the inventory method that gives the lowest value of
inventory will create the highest inventory turnover ratio In an inflationary economy,
LIFO provides the lowest value for the Inventory since it assumes that the newest
(most expensive) units are sold first and the oldest (cheapest) units are in inventory
at the end of the period.

Question #6: A company's cash ratio will decrease if the company


Correct Answer: C. purchases materials on account
A. Purchases commercial paper.
B. Receives cash by issuing a short-term note payable.
C. Purchases materials on account.
D. Sells goods for cash at a selling price lower than cost.
Explanation:
A. The cash ratio is Cash + Cash Equivalents + Marketable Securities
Current Liabilities
Commercial paper is a cash equivalent. If the company purchases commercial paper
with cash, the purchase will cause a decrease in cash and an equal increase in cash
equivalents. Neither the numerator nor the denominator of the cash ratio will change
and thus the cash ratio will not change.
B. The cash ratio is Cash + Cash Equivalents + Marketable Securities
Current Liabilities
Commercial paper is a cash equivalent. If the company purchases commercial paper
with cash, the purchase will cause a decrease in cash and an equal increase in cash
equivalents. Neither the numerator nor the denominator of the cash ratio will change
and thus the cash ratio will not change.
B. The cash ratio is Cash + Cash Equivalents + Marketable Securities
Current Liabilities
The numerator and the denominator of the ratio will increase by equal amounts as a
result of the short-term loan. Whether the cash ratio would increase or decrease
depends upon whether the numerator or the denominator of the ratio had been greater
before the money was borrowed.
If the numerator had been greater than the denominator, an equal increase in the
numerator and denominator will cause a decrease in the cash ratio. For example, if
the numerator were 100 and the denominator were 75, the cash ratio would be 1.33.
If the amount of the loan were 20, the numerator would increase to 120 and the
denominator would increase to 95. The cash ratio would decrease to 1.26.
If the denominator had been greater than the numerator, an equal increase in the
numerator and denominator will cause an increase in the cash ratio. If the numerator
were 75 and the denominator were 100, the cash ratio would be 0.75. An equal
increase of 20 would cause the numerator to increase to 95, the denominator to
increase to 120, and the cash ratio to increase to 0.79.
This occurs because an equal increase in the numerator and the denominator of any
ratio will represent a smaller proportional increase to the larger number than to the
smaller number. If the larger number is in the numerator, it will cause the ratio to
decrease. If the larger number is in the denominator, it will cause the ratio to
increase.
C. The cash ratio is Cash + Cash Equivalents + Marketable Securities
Current Liabilities
If the company purchases materials on account, current liabilities will increase. The
numerator of the ratio will remain the same while the denominator increases, which
will cause the ratio to decrease.
D. The cash ratio is Cash + Cash Equivalents + Marketable Securities

Current Liabilities

Selling goods for cash will cause the numerator of the ratio to increase because cash
will increase. That will be true whether the goods are sold at a price lower than cost
or higher than cost. The denominator will not change as a result of the sale. Thus the
cash ratio will increase, not decrease.

Question #7: Which one of the following is a permitted transaction under the U.S.
Foreign Corrupt Practices Act?

Correct Answer: A. Payments to expedite routine governmental action.

A. Payments to expedite routine governmental action.


B. Payments to government officials to circumvent importation rules in countries
where such payments are a customary business practice by multinational
competitors.
C. Payments to close friends of government officials to obtain an exception to a
regulation.
D. Payments to customs officials to enable the release of an oil drilling rig and other
equipment.

Explanation:
Payments to customs officials to enable the release of an oil drilling rig and other
equipment.

Question #8: ABC Company has fixed costs of $300,000 per month. Total output per
month is 150,000 units. Minimum pay for production line workers is $5.85 per hour,
and total variable costs are currently $275,000 per month. If variable costs increase
to $350,000 per month and production output increases to 250,000 per month, what
are the average total costs before and after the increase in production?
Correct Answer: A. $3.83 per unit before and $2.60 per unit after the increase in
production.
A. $3.83 per unit before and $2.60 per unit after the increase in production.
B. $1.83 per unit before and $1.40 per unit after the increase in production.
C. $2 per unit before and $1.20 per unit after the increase in production.
D. $3.83 per unit before and after the increase in production.
Explanation:
Average total cost is calculated by dividing total costs by total output. Before the
increase in production, average total costs per unit are $3.83 per unit [($275,000
variable costs + $300,000 fixed costs) /150,000 units]. After the increase, average
total costs are $2.60 per unit [($350,000 variable costs + $300,000 fixed costs) /
250,000 units].

Question #9: A company enters into an agreement with a firm that will factor the
company's accounts receivable. The factor agrees to buy the company's receivables,
which average $100,000 per month and have an average collection period of 30 days.
The factor will advance up to 80% of the face value of receivables at an annual rate
of 10% and charge a fee of 2% on all receivables purchased. The controller of the
company estimates that the company would save $18,000 in collection expenses over
the year. Fees and interest are not deducted in advance. Assuming a 360- day year,
what is the annual cost of financing?
Correct Answer: D.17.5%
A. 10.0%
B. 12.0%
C. 14.00%
D. 17.5%
Explanation:
Answer (A) is incorrect. This percentage overlooks the factor fee.
Answer (B) is incorrect. This percentage overlooks the fact that the 2% fee recurs
every month.
Answer (C) is incorrect. This percentage miscalculates the factor fee and the savings
from reduced collection costs.
Answer (D) is correct. The first step is to calculate the amount the firm will receive
from the factoring transaction:

Amount of receivables $100,000

Times: advance percentage *80%


Amount received $80,000

This amount is the basis for the calculation of interest expense

Amount advanced $80,000


Times: annual finance charge *10%
Annualized interest expense $8,000

The next step is to calculate the net outlay


Amount of receivables $100,000
Times :factor fee percentage *2%
Monthly factor fee $2000
Times: months *12
Annual factor fee $24,000
Less: annual savings (18000)
Net outlay $6000

Now the net cost in dollar terms can be determined:

Annualized interest expense $8,000


Net outlay 6000
Annual net cost $14,000

As with all financing arrangements, the effective rate is the ratio of the amount the
firm must pay to the amount the firm gets use of:
Effective rate= Net cost+ Usable funds
=$14,000/$80,000
=17.5%

Question #10: Please see the question below:


What is the profit or loss of a decision to sell or process a product further given the
following information?
Unit production cost for a product= $8,000
Unit selling price for a product = $12,000
Incremental processing cost per unit = $1,000
New unit selling price = $14,000
Correct Answer: B. $1,000.
A. -$1,000.
B. $1,000.
C. -$2,000.
D. $2,000.
Explanation:
In this case, the product should be processed further. The incremental revenue is
$2,000 ($14,000 - $12,000). The incremental revenue exceeds the incremental cost to
process further ($1,000) and would result in a $1,000 gain from further processing
($2000 - $1,000 = $1,000).

Question #11: An electronics manufacturer has subsidiaries in several international locations and
is concerned about its exposure to foreign exchange risk. In countries where currency values are
likely to fall, the manufacturer should encourage all of the following policies except
Correct Answer: A. Granting trade credit whenever possible.
A. Granting trade credit whenever possible.
B. Investing excess cash in inventory or other real assets.
C. Purchasing materials and supplies on a trade credit basis.
D. Borrowing local currency funds if an appropriate interest rate can be obtained.
Explanation:
Answer (A) is correct.
Extension of credit in a foreign currency would result in receiving payment in less valuable
dollars if the foreign currency became less valuable. Thus one would not want to encourage
granting trade credit in a foreign country when the country's currency is expected to lose value.
Answer (B) is incorrect.
Investing monetary assets into nonmonetary assets is advantageous when the monetary unit is
going to lose value.
Answer (C) is incorrect.
It is advantageous to become a debtor when the monetary unit is losing value.
Answer (D) is incorrect.
It is advantageous to become a debtor when the monetary unit is losing value.

Question #12: In the product life cycle classifications (PLC) of industry evolution, shakeout
occurs:
Correct Answer: A. Between growth and maturity.
A. Between growth and maturity.
B. Between embryonic and growth.
C. After maturity.
D. After decline.
Explanation:
The progression of PLC classifications is: embryonic, growth, shakeout, maturity, and decline.
Shakeout occurs when the level of customer sophistication increases due to exposure and first-
hand use of a new product. Suppliers and customers concentrate around market leaders, forcing
marginal players to drop out of the market.

Question #13: In a joint manufacturing process, joint costs incurred prior to a decision as to
whether to process the products after the split-off point should be viewed as:
Correct Answer: C. Sunk costs.
A. Relevant costs.
B. Standard costs.
C. Sunk costs.
D. Differential costs.
Explanation:
In a joint manufacturing process, joint costs incurred prior to a decision as to whether to process
the products after the split-off point should be viewed as sunk costs. They are past costs, already
incurred, and are, therefore, irrelevant for decision making.

Question #14: Sylvan Corporation has the following capital structure. Debenture bonds:
$10,000,000 Preferred equity: $1,000,000 Common equity: $39,000,000
The financial leverage of Sylvan Corporation would increase as a result of:
Correct Answer: C. Financing its future investments with a higher percentage of bonds
A. Maintaining the same dollar level of cash dividends as the prior year, even though
earnings have increased by 7%.
B. Financing its future investments with a higher percentage of equity funds.
C. Financing its future investments with a higher percentage of bonds
D. Issuing common stock and using the proceeds to retire preferred stock.
Explanation:
A. Using more bonds in the future would increase the financial leverage. Financial leverage
is the use of debt to increase earnings. Financial leverage ratios measure a company's use
of debt to finance its assets and operations. Financial leverage also can be defined as the
percentage of fixed cost financing in a firm's overall cost structure, because the increased
amount of debt causes the company's financial costs (interest expense) to increase. Since
financial leverage is the use of debt, anything that increases the company's debt will
increase its financial leverage. Of the four answer choices, only "financing its future
investments with a higher percentage of bonds" involves increasing the company's debt.
B. Financial leverage is the use of debt to increase earnings. Financial leverage ratios
measure a company's use of debt to finance its assets and operations. Financial leverage
also can be defined as the percentage of fixed cost financing in a firm's overall cost
structure, because the increased amount of debt causes the company's financial costs
(interest expense) to increase. Since financial leverage is the use of debt, anything that
increases the company's debt will increase its financial leverage. Maintaining the same
dollar level of cash dividends as the prior year when earnings increase would not increase
debt, so financial leverage would not increase. In fact, all other things being equal,
financial leverage would decrease. One of the measures of financial leverage, the
financial leverage ratio, is total assets divided by total equity. If earnings increase but the
dividend remains the same, the dividend payout ratio will decrease. If the dividend
payout ratio is decreased, retained earnings will increase and the financial leverage of the
company will decrease.
C. Financial leverage is the use of debt to increase earnings. Financial leverage ratios
measure a company's use of debt to finance its assets and operations. Financial leverage
also can be defined as the percentage of fixed cost financing in a firm's overall cost
structure, because the increased amount of debt causes the company's financial costs
(interest expense) to increase. Since financial leverage is the use of debt, anything that
increases the company's debt will increase its financial leverage. Financing its future
investments with a higher percentage of equity funds would not cause the company's
financial leverage to increase. The use of equity as the source of capital would lead
instead to decreased financial leverage. One of the measures of financial leverage, the
financial leverage ratio, is total assets divided by total equity. If total equity increases, all
other things being equal, financial leverage will decrease.
D. Financial leverage is the use of debt to increase earnings. Financial leverage ratios
measure a company's use of debt to finance its assets and operations. Financial leverage
also can be defined as the percentage of fixed cost financing in a firm's overall cost
structure, because the increased amount of debt causes the company's financial costs
(interest expense) to increase. Since financial leverage is the use of debt, anything that
increases the company's debt will increase its financial leverage. Issuing common stock
and using the proceeds to retire preferred stock would not increase debt, so it would not
cause the company's financial leverage to increase.

Question #15: Question: Smithco's payback period for the project will be (please refer the fact
pattern below)
[Fact Pattern #158) Smithco is considering the acquisition of scanning equipment to mechanize
its procurement process. The equipment will require extensive testing and debugging, as well as
user training, prior to its operational use. Projected after-tax cash flows are shown below.
After-tax cash
Inflows/(outflow
Year s)
0 $(550,000)
1 (500,000)
2 450,000
3 350,000
4 350,000
5 350,000

Management anticipates the equipment will be sold at the beginning of Year 6 for 550,000 when
its book value is zero. Smithco's internal hurdle and effective tax rates are 14% and 40%,
respectively.
Correct Answer: D. 3.75 years.
A. 2.3 years.
B. 3.0 years.
C. 3.5 years.
D. 3.75 years.
Explanation:
Answer (A) is incorrect.
The period of 2.3 years fails to consider the outflow in Year 1.
Answer (B) is incorrect.
The period of 3.0 years fails to consider the outflow in Year 1.
Answer (C) is incorrect.
The period of 3.5 years fails to consider the outflow in Year 1.
Answer (D) is correct.
Payback is the measure of the period it takes to recover the initial investment in a project. It will
take almost 4 years to recover the initial investment of $550,000. Because of the outflow of
$500,000 during the first year, the company will be trying to recover $1,050,000 by the end of
Year 1. After recovering $450,000 in Year 2 and $350,000 in Year 3, the company will still need
to recover an additional $250,000 (1,050,000 — 450,000 — 350,000) in Year 4. Thus, it will
take about 5/7 ($250,000/$350,000) of Year 4 to recover the initial investment.

Question #16: When managing cash and short-term investments, a corporate treasurer is
primarily concerned with
Correct Answer: D. Liquidity and safety.
A. Maximizing rate of return.
B. Minimizing taxes.
C. Investing in Treasury bonds since they have no default risk.
D. Liquidity and safety.
Explanation:
Answer (A) is incorrect.
Most companies are not in business to earn high returns on liquid assets (i.e., they are held to
facilitate operations).
Answer (B) is incorrect.
The holding of cash and cash-like assets is not a major factor in controlling taxes.
Answer (C) is incorrect.
Investments in Treasury bonds do not have sufficient liquidity to serve as short-term assets.
Answer (D) is correct.
Cash and short-term investments are crucial to a firm's continuing success. Sufficient liquidity
must be available to meet payments as they come due. At the same time, liquid assets are subject
to significant control risk. Therefore, liquidity and safety are the primary concerns of the
treasurer when dealing with highly liquid assets. Cash and short-term investments are held
because of their ability to facilitate routine operations of the company. These assets are not held
for purposes of achieving investment returns.

Question #17: The use of debt in the capital structure of a firm


Correct Answer: B. Increases its financial leverage
A. Increases its operating leverage.
B. Increases its financial leverage
C. Decreases its financial leverage.
D. Decreases its operating leverage
Explanation:
A. Leverage generally refers to the ability to receive a high level of return relative to the
amount of cost expended. Operating leverage measures the use of operating fixed cost to
generate greater operating profit. Operating leverage refers to the fact that, for a given
level of fixed expenses, a given percentage change in sales will result in a higher
percentage of change in profits than the percentage change in sales. The use of debt in the
capital structure of a firm does not increase its operating leverage, because the cost of
debt is not an operating fixed cost.
B. Leverage generally refers to the ability to receive a high level of return relative to the
amount of cost expended. Financial leverage is the use of debt to increase earnings. The
cost of using debt to finance operations is interest. Interest is a fixed charge because
unlike dividends, interest must be paid whether the firm is profitable or not. The use of
financing that carries a fixed charge is called financial leverage. The use of debt in the
capital structure of a firm increases its financial leverage.
C. Leverage generally refers to the ability to receive a high level of return relative to the
amount of cost expended. Financial leverage is the use of debt to increase earnings. The
cost of using debt to finance operations is interest. Interest is a fixed charge because
unlike dividends, interest must be paid whether the firm is profitable or not. The use of
financing that carries a fixed charge is called financial leverage. The use of debt in the
capital structure of a firm changes its financial leverage, but it does not decrease its
financial leverage.
D. Leverage generally refers to the ability to receive a high level of return relative to the
amount of cost expended. Operating leverage measures the use of operating fixed cost to
generate greater operating profit. Operating leverage refers to the fact that, for a given
level of fixed expenses, a given percentage change in sales will result in a higher
percentage of change in profits than the percentage change in sales. The use of debt in the
capital structure of a firm does not decrease its operating leverage, because the cost of
debt is not an operating fixed cost.

Question #18: For a firm engaged in risk management, Value at Risk is defined as the
Correct Answer: C. maximum loss within a certain time period at a given level of confidence.
A. Most likely negative outcome at a given level of confidence.
B. Maximum value a company can lose.
C. Maximum loss within a certain time period at a given level of confidence.
D. Worst possible outcome given the distribution of outcomes.
Explanation:
A. This is not the definition of Value at Risk. Value at Risk (VaR) measures the potential
loss in value of a risky asset or event over a defined period for a given confidence
interval. It is based on the assumption that the possible outcome of the event is
represented by a normal distribution (bell curve). With a normal distribution, we know
that 95% of the results will lie within 1.96 standard deviations of the mean, and that 99%
of the results will lie within 2.57 standard deviations of the mean. Using this information,
we can predict what the range of results will be with a measured level of confidence.
B. This is not the definition of Value at Risk. Value at Risk (VaR) measures the potential
loss in value of a risky asset or event over a defined period for a given confidence
interval. It is based on the assumption that the possible outcome of the event is
represented by a normal distribution (bell curve). With a normal distribution, we know
that 95% of the results will lie within 1.96 standard deviations of the mean, and that 99%
of the results will lie within 2.57 standard deviations of the mean. Using this information,
we can predict what the range of results will be with a measured level of confidence.
C. Value at Risk (VaR) measures the potential loss in value of a risky asset or event over a
defined period for a given confidence interval. It is based on the assumption that the
possible outcome of the event is represented by a normal distribution (bell curve). With a
normal distribution, we know that 95% of the results will lie within 1.96 standard
deviations of the mean, and that 99% of the results will lie within 2.57 standard
deviations of the mean. Using this information, we can predict what the range of results
will be with a measured level of confidence.
D. This is not the definition of Value at Risk. Value at Risk (VaR) measures the potential
loss in value of a risky asset or event over a defined period for a given confidence
interval. It is based on the assumption that the possible outcome of the event is
represented by a normal distribution (bell curve). With a normal distribution, we know
that 95% of the results will lie within 1.96 standard deviations of the mean, and that 99%
of the results will lie within 2.57 standard deviations of the mean. Using this information,
we can predict what the range of results will be with a measured level of confidence.

Question #19: The purchase of treasury stock with a firm's surplus cash
Correct Answer: D. Increases a firm's financial leverage.:
A. Increases a firm's equity.
B. Increases a firm's interest-coverage ratio.
C. Increases a firm's assets.
D. Increases a firm's financial leverage.
Explanation:
A. Treasury stock is the corporation's own shares that it has repurchased on the market. The
purchase of treasury stock reduces owners' equity because those shares are no longer
outstanding.
B. The purchase of treasury shares does not affect the firm's interest-coverage ratio.
Treasury stock is the corporation's own shares that it has repurchased on the market. The
purchase of treasury stock reduces owners' equity because those shares are no longer
outstanding. The interest coverage ratio is earnings before interest and taxes divided by
interest expense.
C. Treasury stock is the corporation's own shares that it has repurchased on the market.
Treasury stock is not an asset. The purchase of treasury stock reduces owners' equity
because those shares are no longer outstanding. The purchase of the treasury stock will
reduce assets because cash will be reduced while reducing owners' equity.
D. Treasury stock is the corporation's own shares that it has repurchased on the market. The
purchase of treasury stock reduces owners' equity because those shares are no longer
outstanding. The decrease in equity causes the financial leverage of the company to
increase. The financial leverage ratio will also increase. The financial leverage ratio is
Total Assets divided by Total Equity. Assets and equity will decrease by the same
amount when treasury stock is purchased with cash, but the proportionate reduction of
equity will be greater than the proportionate reduction of assets, since assets are greater
than equity, so the financial leverage ratio will increase.

Question #20: Fact Pattern: Morton Company needs to pay a supplier's invoice of $50,000 and
wants to take a cash discount of 2/10, net 40. The firm can borrow the money for 30 days at 12%
per annum plus a 10% compensating balance.
Question: Assuming Morton Company borrows the money on the last day of the discount period
and repays it 30 days later, the effective interest rate on the loan is
Correct Answer: B. 13.33%
A. 12.00%
B. 13.33%
C. 13.20%
D. 13.48%
Explanation:
Answer (A) is incorrect.
This percentage is the contract rate.
Answer (B) is correct.
Morton's effective rate on this loan can be calculated as follows:
Effective rate = Stated rate ÷ (1.0 — Compensating balance %)
= 12% ÷ (100% — 10%)
= 12% ÷ 90%
= 13.33%
Answer (C) is incorrect.
This percentage assumes that the company has access to loan funds of $50,000 and is calculated
by determining interest based on a loan total of $55,000.
Answer (D) is incorrect.
This percentage is calculated by determining interest on a loan amount of $55,056.
Question #21: A depreciation tax shield is
Correct Answer: B. A Reduction in income taxes.
A. An after-tax cash outflow.
B. A reduction in income taxes.
C. The cash provided by recording depreciation.
D. The expense caused by depreciation.
Explanation:
Answer (A) is incorrect.
A tax shield is not a cash flow; but a means of reducing outflows for income taxes.
Answer (B) is correct.
A tax shield is something that will protect income against taxation. Thus; a depreciation tax
shield is a reduction in income taxes due to a company's being allowed to deduct depreciation
against otherwise taxable income.
Answer (C) is incorrect.
Cash is not provided by recording depreciation; the shield is a result of deducting depreciation
from taxable revenues.
Answer (D) is incorrect.
Depreciation is recognized as an expense even if it has no tax benefit.

Question #22: In a decision analysis situation, which one of the following costs is generally not
relevant to the decision?
Correct Answer: C. Historical cost.
A. Differential cost.
B. Incremental cost.
C. Historical cost.
D. Avoidable cost.
Explanation:
Historical costs are sunk costs. They have already been incurred and cannot be recovered in the
future. Therefore; they are irrelevant for decision making.

Question #23: DRP Insurance Company wants to be "best in class" in terms of Enterprise Risk
Management (ERM) implementation. To achieve this goal, the company plans to identify events
that affect the implementation of strategy and achievement of objectives. Which of the following
best reflects an analysis that would help its identification process?
Correct Answer: A. Review of incidents and new market conditions.
A. Review of incidents and new market conditions.
B. Analysis of default histories and dispersion.
C. Review of accidents and operational measures.
D. Summary of driving records and age.
Explanation:
A. Enterprise risk management (ERM) is defined as "a process, effected by an entity's board
of directors, management and other personnel, applied in strategy setting and across the
enterprise, designed to identify potential events that may affect the entity, and manage
risk to be within its risk appetite, to provide reasonable assurance regarding achievement
of entity objectives." A review of incidents and new market conditions is a strategy
designed to identify potential events that may affect the entity and to manage risk.
B. Enterprise risk management (ERM) is defined as "a process, effected by an entity's board
of directors, management and other personnel, applied in strategy setting and across the
enterprise, designed to identify potential events that may affect the entity, and manage
risk to be within its risk appetite, to provide reasonable assurance regarding achievement
of entity objectives." Analysis of default histories and dispersion is not a strategy
designed to identify potential events that may affect the entity and to manage risk.
C. Enterprise risk management (ERM) is defined as "a process, effected by an entity's board
of directors, management and other personnel, applied in strategy setting and across the
enterprise, designed to identify potential events that may affect the entity, and manage
risk to be within its risk appetite, to provide reasonable assurance regarding achievement
of entity objectives." A review of accidents and operational measures is not a strategy
designed to identify potential events that may affect the entity and to manage risk.
D. Enterprise risk management (ERM) is defined as "a process, effected by an entity's board
of directors, management and other personnel, applied in strategy setting and across the
enterprise, designed to identify potential events that may affect the entity, and manage
risk to be within its risk appetite, to provide reasonable assurance regarding achievement
of entity objectives." A summary of driving records and age is not a strategy designed to
identify potential events that may affect the entity and to manage risk.

Question #24: CPZ Enterprises had the following account information.

Accounts receivable $200,000

Accounts payable 80,000

Bonds payable, due in ten years 10,000

Cash 100,000

Interest payable, due in three months 10,000

Inventory 400,000

Land 250,000

Notes payable, due in six months 50,000

Prepaid expenses 40,000

The company has an operating cycle of five months. What is the company's acid test (quick)
ratio?
Correct Answer: D. 2.14
A. 2.31
B. 0.68
C. 1.68
D. 2.14
Explanation:
A. The quick ratio is calculated as quick assets (current assets excluding inventory and
prepaid expenses) divided by current liabilities. This answer does not include the
interest payable as a current liability.
B. The quick ratio is calculated as quick assets (current assets excluding inventory and
prepaid expenses) divided by current liabilities. This answer includes bonds payable
as a current liability.
C. The quick ratio is calculated as quick assets (current assets excluding inventory and
prepaid expenses) divided by current liabilities. This answer includes inventory and
prepaid expenses as quick assets and includes bonds payable as a current liability
D. The quick ratio is calculated as quick assets (current assets excluding inventory and
prepaid expenses) divided by current liabilities. Quick assets include accounts
receivable ($200,000) and cash ($l00,000). Current liabilities include accounts
payable ($80,000), interest payable ($10,000) and notes payable ($50,000). Note that
even though the notes payable are due after the operating cycle is oven the
distinction for a current asset or liability is that it will be converted or settled within
12 months or the operating cycle, whichever is longer. Thus the 12-month limit
governs what is considered a current liability, not the 5-month length of the firm's
operating cycle. The total current assets are $300,000 and current liabilities are
$140,000. This gives us a quick ratio of 2.14.

Question #25: A management accountant is leading the effort to completely identify all of
the risks that face her company. Which one of the following best identifies a technique that
she should consider using?
Correct Answer: A. Self-assessment workshops
A. Self-assessment workshops
B. Customer surveys.
C. Customer surveys.
D. Internal audits.
Explanation:
A. Facilitated workshops can be useful as a technique to identify risks by drawing on
the knowledge and experience of management staff7 and other stakeholders. By
combining the knowledge and experience of team members7 important risks can be
identified that otherwise might be missed
B. Information might be gained from customer surveys that could be helpful in
identifying any situations that might represent risks, but customer surveys are not a
method of completely identifying all of the risks a company could face.
C. In business process re-engineering, management starts with a clean sheet of paper
and redesigns processes to accomplish its objectives Business process re-engineering
is not a method of completely identifying all of the risks a company could face.
D. Internal auditing is defined as an independent objective assurance and consulting
activity designed to add value and improve an organization's operations. It helps an
organization accomplish its objectives by bringing a systematic, disciplined approach
to evaluate and improve the effectiveness of risk management; control and
governance processes. However, internal audits are not a method of completely
identifying all of the risks a company could face.

Question #26: Donovan Corporation recently declared and issued a 50% stock dividend.
This transaction will reduce the company's
Correct Answer: A. Book value per common share.
A. Book value per common share.
B. Current ratio.
C. Return on operating assets
D. Debt-to-equity ratio.
Explanation:
A. A stock dividend is paid in shares of newly-issued stock; not in cash. A stock
dividend increases the number of common shares outstanding but does not increase
common equity on the balance sheet The book value per share is total assets minus
all liabilities and claims of securities that are senior to the common stock (i.e, take
priority over common stockholders' claims) such as preferred stock7 divided by the
number of common shares outstanding. When the divisor; the number of common
shares, increases, the book value per common share will decrease.
B. A stock dividend is paid in shares of newly-issued stock, not in cash. A stock
dividend increases the number of common shares outstanding but does not increase
common equity on the balance sheet. Since a stock dividend is not paid in cash, it
cannot affect the current ratio
C. A stock dividend is paid in shares of newly-issued stock, not in cash A stock
dividend increases the number of common shares outstanding but does not increase
common equity on the balance sheet Payment of a stock dividend does not affect
operating assets on the balance sheet and it does not affect net income Therefore; it
could not affect return on operating assets
D. A stock dividend is paid in shares of newly-issued stock, not in cash A stock
dividend increases the number of common shares outstanding but does not increase
common equity on the balance sheet Since equity does not change; the debt-to-equity
ratio will not change

Question #27: Fact Pattern: The Frame Supply Company has just acquired a large account and
needs to increase its working capital by $100,000. The controller of the company has identified
the four sources of funds given below.
1. Pay a factor to buy the company's receivables, which average $125,000 per month and have an
average collection period of 30 days. The factor will advance up to 80% of the face value of
receivables at 10% and charge a fee of 2% on all receivables purchased. The controller estimates
that the firm would save $24,000 in collection expenses over the year. Assume the fee and
interest are not deductible in advance.
2. Borrow $110,000 from a bank at 12% interest. A 9% compensating balance would be
required.
3. Issue $110,000 of 6-month commercial paper to net $100,000. (New paper would be issued
every 6 months.)
4. Borrow $125,000 from a bank on a discount basis at 20%. No compensating balance would be
required. Assume a 360-day year in all of your calculations.
Question: The cost of Alternative 4 to Frame Supply Company is
Correct Answer: B. 25.0%
A. 20.0%
B. 25.0%
C. 40.0%
D. 50.0%
Explanation:
Answer (A) is incorrect.
The effective rate must exceed the contract rate of 20%.
Answer (B) is correct.
The company will receive $100,000 ($125,000 x 80%) at an annual cost of $25,000 ($125,000
— $100,000). The effective interest rate on this loan can thus be calculated as follows:
Effective rate = Interest expense ÷ Usable funds
= $25,000 ÷ $100,000
= 25.0%
Answer (C) is incorrect.
This percentage assumes no discount and a 6-month loan term.
Answer (D) is incorrect.
This percentage assumes a 6-month loan term.

Question #28: Question: Which project(s) should Maloney undertake during the upcoming year
assuming it has no budget restrictions? (please refer the fact pattern below)
[Fact Pattern #164] Maloney Company uses a 12% hurdle rate for all capital expenditures and
has done the following analysis for four projects for the upcoming year:
Project 1 Project 2 Project 3 Project 4
Initial outlay $4,960,000 $5,440,000 $4,000,000 $5,960,000
Annual net cash inflows:
Year 1 1,600,000 1,900,000 1,300,000 2,000,000
Year 2 1,900,000 2,500,000 1,400,000 2,700,000
Year 3 1,800,000 1,800,000 1,600,000 1,800,000
Year4 1,600,000 1,200,000 800,000 1,300,000
Net present value 281,280 293,240 (75,960) 85,520
Profitability 106% 105% 98% 101%
Internal rate of
return 14% 15% 11% 13%

Correct Answer: C. Projects 1, 2 and 4.


A. All of the projects.
B. Projects 1, 2 and 3.
C. Projects 1, 2 and 4.
D. Projects 1 and 2.
Explanation:
Answer (A) is incorrect.
Project 3 has a negative NPV.
Answer (B) is incorrect.
Project 3 has a negative NPV.
Answer (C) is correct.
A company using the net present value (NPV) method should undertake all projects with positive
NPVs that are not mutually exclusive. Given that Projects 1, 2, and 4 have positive NPVs, those
projects should be undertaken. Furthermore, a company using the internal rate of return (IRR) as
a decision rule ordinarily chooses projects with a return greater than the cost of capital. Given a
12% cost of capital, Projects 1, 2, and 4 should be chosen using an IRR criterion if they are not
mutually exclusive. Use of the profitability index yields a similar decision because a project with
an index greater than 100% should be undertaken.
Answer (D) is incorrect.
Project 4 has a positive NPV and should be undertaken.

Question #29: The basis for the components of an Enterprise Risk Management system as
defined by COSO in Enterprise Risk Management — Integrated Framework is
Correct Answer: B. The internal environment of the organization.
A. The organization's control activities.
B. The internal environment of the organization.
C. Assessment of risk.
D. Identification of the organization's strategic objectives.
Explanation:
A. Control activities are the policies and procedures implemented to ensure that risk
responses are effectively implemented. They are not the basis for the other components.
B. The internal environment of the organization - the attitude in the organization toward risk
and risk management - is the basis for all the other components of an Enterprise Risk
Management (ERM) system.
C. Risk assessment - the process of analyzing risks from the perspective of the likelihood of
the risk's occurring and the potential impact of the event if it does occur - is the core of an
Enterprise Risk System (ERM). However, it is not the basis for the other components.
D. Identification of the organization's strategic objectives is an important component of an
Enterprise Risk Management (ERM) system. However, it is not the basis for the other
components.

Question #30: Sands, Inc. uses a calendar year for financial reporting. The company is
authorized to issue 5,000,000 shares of $10 par common stock. At no time has Sands issued any
potentially dilutive securities. Listed below is a summary of Sands' common stock activities.
Number of common shares issued and outstanding at Dec. 31, 20X6:1,000,000
Shares issued as a result of a 10% stock dividend on Sept. 30, 20X7: 100,000
Shares issued for cash on March 31, 20X8: 1,000,000
Number of common shares issued and outstanding at Dec. 31, 20X8:2,100,000
A two-for-one stock split of Sands' common stock took place on March 31, 20X9.
The weighted-average number of common shares used in computing earnings per common share
for 20X8 on the 20X9 comparative income statement was:
Correct Answer: C. 3,700,000
A. 4,200,000
B. 2,100,000.
C. 3,700,000
D. 1,850,000.
Explanation:
A. This is the number of common shares outstanding as of December 31, 20X8 multiplied
by 2. However, the weighted-average number of common shares outstanding is different
from the year-end number of shares outstanding.
B. This is the number of common shares outstanding as of December 31, 20X8.
C. In the 20X9 financial statements, the 20X8 EPS calculations will have to include the
effect of any stock splits or stock dividends that occurred in 20X9. In 20X9 there was a 2-
for-1 stock split. In order to properly compare the EPSs of the different years, the
company must go back and multiply the 20X8 weighted-average shares outstanding by 2.
At the end of 20X7 there were 1,100,000 shares outstanding. This was the January 1,
20X7 beginning amount of 1,000,000 shares and the 100,000 shares that were issued in
September 20X7 as a stock dividend. Shares issued as a stock dividend are considered to
have been outstanding from January 1 of the first year presented.

To this; the company needs to add the effect of any 20X8 share transactions. The only
transaction was the issuance of 1,000,000 shares on March 31. Since these shares were
outstanding for only 9 months of the year, they will count as only 75% (9112) for the
weighted-average share calculation. These shares will become 750,000 shares for this
calculation, giving the company 1,850,000 shares outstanding as of December 31, 20X8,
on the 20X8 comparative income statement.

Since a two-for-one stock split took place on March 31, 20X9, the 20X8 weighted-
average number of shares outstanding needs to be multiplied by 2 when reporting the
20X8 weighted-average number of shares outstanding on the 20X9 comparative income
statements. Therefore, the weighted-average number of shares outstanding for 20X8 on
the 20X9 comparative statements is 3,700,000.
D. This is the number of shares used in the 20X8 financial statements for the 20X8 EPS
calculation. This number needs to be adjusted for the stock split that took place in 20X9.

Question #31: All other things being equal, which one of the following factors would result in an
increase in cash reported on the balance sheet from one period to the next?
Correct Answer: D. Reduction of days sales outstanding of accounts receivable.
A. Increase in the level of inventory held.
B. Increase in the speed with which accounts payable invoices are paid.
C. Decrease in the accrued vacation liability.
D. Reduction of days sales outstanding of accounts receivable.
Explanation:
A. Holding more inventory will require a greater cash investment for that inventory. This
will reduce the amount of cash we hold compared to the previous period, not increase it.
B. Paying our payables faster will mean that in the current period we paid for more total
days of purchases than we had done in the past. This will reduce the amount of cash from
the previous period.
C. Changing the amount of the accrued vacation liability will not impact the amount of cash
the company has. Decreasing the accrued vacation liability will increase net income, but
not cash.
D. A reduction of the number of days of sales outstanding in accounts receivable means that
we are collecting our receivables faster. This will decrease accounts receivable and
increase the cash balance from one period to the next because there will be fewer days of
sales outstanding in accounts receivable than there were in the previous period.

Question #32: What are the four standards of ethical conduct as expressed in the IMA Statement
of Ethical Professional Practice (2005)?
Correct Answer: D. Competence, Confidentiality, Integrity and Credibility
A. Honesty, Fairness, Objectivity and Responsibility
B. Fairness, Confidentiality, Integrity and Credibility
C. Competence, Confidentiality, Objectivity and Responsibility
D. Competence, Confidentiality, Integrity and Credibility
Explanation:
A. These are overarching ethical principles.
B. Three of these are standards of ethical conduct, but one is an overarching ethical
principle.
C. Two of these are standards of ethical conduct, but two are overarching ethical principles.
D. The four standards of ethical conduct are Competence, Confidentiality, Integrity and
Credibility.

Question #33: A company has developed a new technologically advanced paper-thin solar panel
for residential home use. The company has decided to start selling this solar panel worldwide
next month. As this is a technologically innovative product, one risk that the company faces to
sell the solar panel is possible product failure. This risk of product failure is best representative
of what type of risk to the company?
Correct Answer: B. Operational risk.
A. Inherent risk.
B. Operational risk.
C. Residual risk.
D. Business risk.
Explanation:
A. Inherent risk is the level of risk in each event before any mitigation action is taken. The
risk of product failure is not an inherent risk.
B. Operational risk is risk that results from inadequate or failed internal processes, people or
systems. The risk of product failure is an operational risk.
C. Residual risk is the level of risk that remains after management has taken action to
mitigate the risk. The risk of product failure is not a residual risk.
D. Business risk for a firm is the risk of changes in its earnings before interest or taxes.
Business risk depends on a variety of factors, including the variability of demand, sales
price, and the price of inputs as well as the amount of the company's operating leverage.
The risk of product failure is not a business risk.

Question #34: Depoole Company is a manufacturer of industrial products and employs a


calendar year for financial reporting purposes. Assume that total quick assets exceeded total
current liabilities both before and after the transaction described. Further assume that Depoole
has positive profits during the year and a credit balance throughout the year in its retained
earnings account. The early liquidation of a long-term note with cash affects the
Correct Answer: D. Quick ratio to a greater degree than the current ratio.
A. Current ratio but not the quick ratio
B. Current and quick ratio to the same degree
C. Current ratio to a greater degree than the quick ratio.
D. Quick ratio to a greater degree than the current ratio.
Explanation:
A. Both the current and the quick ratios will be affected by this transaction because this
transaction reduces the current and quick assets used in the calculation of these ratios.
B. While both of these ratios will decrease as a result of this transaction, they will not be
affected equally. This is because the numerator in the current and quick ratios are
different. The numerator in the current ratio includes inventory, but the numerator in the
quick ratio does not include inventory.
C. While both of these ratios will decrease as a result of this transaction, the current ratio
will not be affected by a greater degree than the quick ratio. This is because the
numerator in the current and quick ratios are different. The numerator in the current ratio
includes inventory, but the numerator in the quick ratio does not include inventory.
Therefore, the quick ratio will be affected by a greater degree than the current ratio.
D. The early liquidation of a long-term note with cash will reduce the level of current and
quick assets but will not affect the level of current liabilities. Therefore, both the current
and quick ratios will decrease as a result of this transaction. However, the degree of the
reduction in the quick ratio will be greater than the degree of the reduction in the current
ratio because the numerator in the quick ratio is smaller. This is because the quick ratio
includes only cash, marketable securities and accounts receivable in the numerator. It
does not include inventory or other current assets such as pre-paids in the numerator.
Degree of reduction means percentage of reduction, not an absolute amount of reduction.
Question #35: Project 1 has an expected NPV of $120,000 and a standard deviation of $200,000.
Project 2 has an expected NPV of $100,000 and a standard deviation of $150,000. The
correlation between these two projects is 0.80. What is the coefficient of variation for the
portfolio of projects?
Correct Answer: C. 1.51
A. 1.67
B. 1.59
C. 1.51
D. 0.63
Explanation:
Answer (A) is incorrect.
This figure is for Project 1 only.
Answer (B) is incorrect.
This figure assumes a correlation of 1.0.
Answer (C) is correct.
The coefficient of variation is useful when the rates of return and standard deviations of two
investments differ. It measures the risk per unit of return by dividing the standard deviation by
the expected return. Thus, for Project 1, dividing $200,000 by $120,000 produces a coefficient of
1.67. For Project 2, the calculation is to divide $150,000 by $100,000, or 1.50. If the two projects
had perfect correlation (=1.0), then you could combine the calculations ($350,000 ÷ $220,000 =
1.59). However, with a correlation of less than one, the risk will be something less than 1.59.
Answer (D) is incorrect.
This figure is the inverse of the unadjusted portfolio coefficient.

Question #36: Who is not among those to whom practitioners of management accounting and
financial management have an obligation to maintain the highest standards of ethical conduct
according to the IMA Statement of Ethical Professional Practice?
Correct Answer: A. Immediate family members
A. Immediate family members
B. The organizations they serve
C. The public
D. Their profession
Explanation:
A. Practitioners of management accounting and financial management have an obligation to
the public, their profession, the organizations they serve, and themselves to maintain the
highest standards of ethical conduct. Their immediate family members are not included.
B. Practitioners of management accounting and financial management have an obligation to
the organizations they serve to maintain the highest standards of ethical conduct.
C. Practitioners of management accounting and financial management have an obligation to
the public to maintain the highest standards of ethical conduct.
D. Practitioners of management accounting and financial management have an obligation to
their profession to maintain the highest standards of ethical conduct.

Question #37: Company A has a formal corporate code of ethics while company B does not. The
code of ethics covers such things as purchase agreements and relationships with vendors as well
as many other issues to guide individual behavior within the company. Which of the following
statements can be logically inferred?
I. Company A exhibits a higher standard of ethical behavior than does company B.
II. Company A has established objective criteria by which an employee's actions can be
evaluated.
III. The absence of a formal corporate code of ethics in company B would prevent a
successful audit of ethical behavior in that company.
Correct Answer: B. II only.
A. I and II only.
B. II only.
C. III only.
D. II and III only.
Explanation:
A. The existence of a corporate code of ethics, by itself, does not ensure higher standards of
ethical behavior. It must be complemented by follow-up policies and monitoring
activities to ensure adherence to the code.
B. A formalized corporate code of ethics presents objective criteria by which actions can be
evaluated and would thus serve as criteria against which activities could be evaluated.
The existence of a corporate code of ethics, by itself, does not ensure higher standards of
ethical behavior. It must be complemented by follow-up policies and monitoring
activities to ensure adherence to the code. Standards which would influence individual
actions can occur in other places than the corporate code of ethics. For example, there
may be defined policies regarding purchasing activities that may serve the same purpose
as a code of ethics. These policies also serve as criteria against which activities may be
evaluated.
C. The absence of a formal corporate code of ethics in company B would not prevent a
successful audit of ethical behavior in that company. Standards which would influence
individual actions can be documented in places other than in a corporate code of ethics.
For example, there may be defined policies regarding purchasing activities that may serve
the same purpose as a code of ethics. These policies also serve as criteria against which
activities may be evaluated.
D. Standards which would influence individual actions can be documented in places other
than in a corporate code of ethics. For example, there may be defined policies regarding
purchasing activities that may serve the same purpose as a code of ethics. These policies
also serve as criteria against which activities may be evaluated.

Question #38: Which of the following is not a political risk of investing in a foreign country?
Correct Answer: A foreign customer might default on its debt.
A. Rebellions could result in destruction of property.
B. Assets could be expropriated.
C. Foreign-exchange controls could limit the repatriation of profits.
D. A foreign customer might default on its debt.
Explanation:
Answer (A) is incorrect.
These are all political risks.
Answer (B) is incorrect.
These are all political risks.
Answer (C) is incorrect.
These are all political risks.
Answer (D) is correct.
Political risks include the threat of expropriation of company assets, destruction of assets in
rebellions in third-world nations, and limitations on the repatriation of profits (or even initial
investments). Default by a foreign customer is not a political risk, but a risk of doing business
either locally or internationally.

Question #39: For the year just ended, Beechwood Corporation had income from operations of
$198,000 and net income of $96,000. Additional financial information is given below.
January Decembe
1 r 31
7% bonds payable $95,00 $77,000
0
Common stock ($10 par 300,00
value) 0 300,000
Reserve for bond
retirement 12,000 28,000
155,00
Retained earnings 0 206,000

Beechwood has no other equity issues outstanding. Beechwood's return on shareholders' equity
for the year just ended is
Correct Answer: D. 19.2%.
A. 19.9%.
B. 39.5%.
C. 32.0%.
D. 19.2%.
Explanation:
A. Return on Equity is Net Income +Average Total Equity. Total equity is all of the line
items given except for the 7% bonds payable. Reserve for bond retirement is an equity
account (see correct answer for an explanation of this). This answer omits the reserve for
bond retirement from the calculation of average total equity. Please see the explanation
for the correct answer for more information about the reserve for bond retirement.
B. Return on Equity is Net Income +Average Total Equity. Total equity is all of the line
items given except for the 7% bonds payable. Reserve for bond retirement is an equity
account (see correct answer for an explanation of this). This answer correctly includes the
reserve for bond retirement as an equity account in the calculation of average total equity.
However, it uses income from operations instead of net income.
C. Return on Equity is Net Income + Average Total Equity. Total equity is all of the line
items given except for the 7% bonds payable. Reserve for bond retirement is an equity
account (see correct answer for an explanation of this). This is net income + average
common stock. Equity includes not only common stock but also the reserve for bond
retirement and retained earnings. Please see the explanation for the correct answer for
more information about the reserve for bond retirement.
D. Return on Equity is Net Income + Average Total Equity. The trick in this question is
calculating what Average Total Equity is. Total equity is all of the line items given except
for the 7% bonds payable. Reserve for bond retirement is an equity account. The reserve
for bond retirement account is an appropriation of retained earnings. All retained earnings
start out classified as Unappropriated Retained Earnings. "Unappropriated" means that
the dividends are available to be distributed to shareholders in the form of dividends.
Occasionally, however, a company does not want to distribute its retained earnings and
this can be communicated to the shareholders (and potential shareholders) through the
process of appropriating retained earnings. The appropriation of retained earnings is done
by the board of directors and there is only one result of this action. Funds are transferred
in the accounting system from retained earnings to the appropriated retained earnings
account, which here is called reserve for bond retirement. This action informs the readers
of the financial statements that some of the retained earnings are not available for
distribution. There is no legal meaning to this, no time period involved (the board can de-
nappropriate the retained earnings at any time), and there are no involved accounting
processes to this. Retained earnings is debited to reduce it, and reserve for bond
retirement is credited to increase it. So the reserve for bond retirement would be a line
item in the equity section of the balance sheet and it is treated just like retained earnings
in calculating total equity.

Total Equity at the beginning of the year is $467,000 ($300,000 + $12,000 + $155,000).
At the end of the year, it is $534,000. So Average Total Equity is ($467,000 + $534,000)
+ 2, or $500,500.
Return on Equity is $96,000 + $500,500, which equals 19.18% or 19.2% rounded.

Question #40: When the price of gasoline went from $2.50 to $3.00 per gallon, the
demand for SUVs dropped from 300,000 units to 240,000 units. What is the cross
elasticity of demand of SUVs and gasoline prices and the nature of the relationship
between the two?
Correct Answer: B. -1.22; complementary products.
A. 1.22; substitute products.
B. -1.22; complementary products.
C. -0.82; complementary products.
D. 0.82; substitute products.
Explanation:
The cross elasticity is calculated as:
% change in demand for SUVs =
change∈quantity demanded −60,000
= =−0.22
average quantity demnaded 300,000+240,000
2
change∈ price 0.50
= =0.18
% change in price of gasoline = average price 2.50+3.00
2

% change∈demand of SUV −0.22


ESD = = =−1.22 .
% change∈the price of gasoline 0.18
Since the cross elasticity is negative, SUVs and gasoline are complements.
Question #41: A change in the liability for warranty costs requires
Correct Answer: B. Reporting current and future financial statements on the new basis
A. Presenting the effect of pro forma data on income and earnings per share for all prior
periods presented
B. Reporting current and future financial statements on the new basis
C. Restating prior-period financial statements
D. Reporting an adjustment to the beginning retained earnings balance in the statement of
retained earnings
Explanation:
A. A change in the liability for warranty costs is a change in an accounting estimate. Prior
period financial statements are not restated as a result of a change in accounting
estimates; and pro forma data is never used. See the correct answer for a complete
explanation.
B. A change in the liability for warranty costs is a change in an accounting estimate. A
change in accounting estimate requires prospective treatment. No changes are made to
prior period financial statements or beginning retained earnings. The change in estimate
is accounted for in the current period and in future periods.
C. A change in the liability for warranty costs is a change in an accounting estimate. Prior
period financial statements are not restated as a result of a change in accounting
estimates. See the correct answer for a complete explanation.
D. A change in the liability for warranty costs is a change in an accounting estimate. A
change in accounting estimate does not require an adjustment to beginning retained
earnings.

Question #42: To determine the operating cycle for a retail department store, which one of the
following pairs of items is needed?
Correct Answer: B. Cash turnover and net sales.
A. Days' sales in accounts receivable and average merchandise inventory.
B. Cash turnover and net sales.
C. Accounts receivable turnover and inventory turnover.
D. Asset turnover and return on sales.
Explanation:
Answer (A) is incorrect.
Cost of sales must be known to calculate days' sales in inventory.
Answer (B) is incorrect.
These items are insufficient to permit determination of the operating cycle.
Answer (C) is correct.
The operating cycle is the time needed to turn cash into inventory, inventory into receivables,
and receivables back into cash. For a retailer, it is the time from purchase of inventory to
collection of cash. Thus, the operating cycle of a retailer is equal to the sum of the number of
days' sales in inventory and the number of days' sales in receivables. Inventory turnover equals
cost of goods sold divided by average inventory. The days' sales in inventory equals 365 (or
another period chosen by the analyst) divided by the inventory turnover. Accounts receivable
turnover equals net credit sales divided by average receivables. The days' sales in receivables
equals 365 (or other number) divided by the accounts receivable turnover.
Answer (D) is incorrect.
These items are insufficient to permit determination of the operating cycle.

Question #43: A major difference between risk in investing and other types of risk is
Correct Answer: A. risk in investing has the potential for either a positive or negative event,
whereas other types of risk have the potential only for a negative event.
A. Risk in investing has the potential for either a positive or negative event, whereas other
types of risk have the potential only for a negative event.
B. Other types of risk can be managed with insurance, whereas it is not possible to manage
risk in investing.
C. Risk in investing is operational risk; whereas other types of risk are strategic risks.
D. Risk in investing has the potential for great losses, whereas other types of risk have the
potential for either great losses or great gains.
Explanation:
A. Risk in investing is defined as the variability of actual returns from expected returns, and
this variability may be either a gain or a loss. So risk in investing is the potential for
either a positive event (a gain in excess of the expected return) or a negative event (a loss
or a gain that is lower than the expected return). Enterprise risk is defined as "any event
or action that can keep an organization from achieving its objectives." Thus, types of
enterprise risk other than investing risks have the potential only for a negative event to
occur.
B. Some but not all types of other risk can be managed with insurance; and risk in investing
can be managed and reduced, though not eliminated, through proper investment
diversification and through hedging activities.
C. Risk in investing is one type of financial risk, which in turn is a type of enterprise risk.
Strategic risk is another type of enterprise risk, but not the only type. The four types of
enterprise risk are strategic risk; financial risk; operational risk and hazard risk.
D. Risk in investing is defined as the variability of actual returns from expected returns; and
this variability may be either a gain or a loss. Therefore; risk in investing is not limited to
the potential for losses but it also includes the potential for gains above the expected
return. Enterprise risk is defined as "any event or action that can keep an organization
from achieving its objectives." Therefore; types of enterprise risk other than investing
risks do not involve the potential for gains.

Question #44: A firm's financial risk is a function of how it manages and maintains its debt.
Which one of the following sets of ratios characterizes the firm with the greatest amount of
financial risk?
Correct Answer: D. High debt-to-equity ratio, low interest-coverage ratio; volatile return on
equity.
A. High debt-to-equity ratio, high interest-coverage ratio, volatile return on equity.
B. Low debt-to-equity ratio; low interest-coverage ratio, volatile return on equity.
C. High debt-to-equity ratio, high interest-coverage ratio, stable return on equity.
D. High debt-to-equity ratio, low interest-coverage ratio; volatile return on equity.
Explanation:
A. A company with the greatest amount of financial risk would have a low interest coverage
ratio. See the correct answer for a complete explanation.
B. A company with the greatest amount of financial risk would have a high debt-to-equity
ratio. See the correct answer for a complete explanation.
C. A company with the greatest amount of financial risk would have a low interest coverage
ratio and a volatile return on equity. See the correct answer for a complete explanation.
D. A company that has the greatest amount of financial risk will have a high debt-to-equity
ratio, a low interest-coverage ratio (meaning that their profits are not much greater than
their interest payments) and a volatile return on equity (which indicates that profits
fluctuate greatly from one period to the next).

Question #45: Which of the following is not a benefit of risk management?


Correct Answer: B. Lower taxes.
A. Better contingency planning.
B. Lower taxes.
C. Improved cost control.
D. Increased shareholder value.
Explanation:
A. Contingency planning is the development of a "backup plan," or a plan that enables the
organization to respond appropriately to an unplanned event. It is used for emergency
response and post-disaster recovery. Contingency planning is particularly important in
information systems in order to insure that critical systems resources will be available
and operations can continue during an emergency situation. Better contingency planning
is a benefit of risk management because risk assessment and risk management enable
management to be aware of and prepared for more possible events that could prevent
them from achieving their objectives.
B. Lower taxes do not result from risk management, so they cannot be a benefit of risk
management.
C. Cost control is improved as a result of risk management because there are fewer
unpleasant surprises and fewer disruptions in the operations of the business.
D. Increased shareholder value is a benefit of risk management because risk management
can minimize losses and maximize opportunities.

Question #46: Associated Supply, Inc. is considering introducing a new product that will require
a $250,000 investment of capital. The necessary funds would be raised through a bank loan at an
interest rate of 8%. The fixed operating costs associated with the product would be $122,500
while the contribution margin percentage would be 42%. Assuming a selling price of $15 per
unit, determine the number of units (rounded to the nearest whole unit) Associated would have to
sell to generate earnings before interest and taxes (EBIT) of 32% of the amount of capital
invested in the new product.
Correct Answer: C. 32,143 units.
A. 23,276 units.
B. 35,318 units.
C. 32,143 units.
D. 25,575 units.
Explanation:
The required EBIT = 32% x $250,000 = $80,000. The required number of units = (Fixed costs +
EBIT) / Contribution margin per unit = ($122,500 + $80,000) / ($15 x 42%) = 32,143.

Question #47: The opportunity cost of making a component part where there is no alternative use
for the factory is:
Correct Answer: D. Zero.
A. The variable manufacturing cost of the component.
B. The total manufacturing cost of the component.
C. The total variable cost of the component.
D. Zero.
Explanation:
There is no foregone opportunity. Therefore, there is no opportunity cost. Opportunity cost is the
cash flow foregone or sacrificed by choosing one option over the next best alternative. It is the
cost of pursuing one alternative as opposed to another.

Question #48: Fact Pattern: Henderson, Inc., has purchased a new fleet of trucks to deliver its
merchandise. The trucks have a useful life of 8 years and cost a total of $500,000. Henderson
expects its net increase in after-tax cash flow to be $150,000 in Year 1, $175,000 in Year 2,
$125,000 in Year 3, and $100,000 in each of the remaining years.
Question: What is the payback reciprocal for Henderson's fleet of trucks?
Correct Answer: A. 29%
A. 29%
B. 25%
C. 24%
D. 20%
Explanation:
Answer (A) is correct.
The payback reciprocal for an investment is found by dividing 1 by the payback time. The
payback time for this investment is 3.5 years, and the payback reciprocal is 1 divided by 3.5, or
29%.
Answer (B) is incorrect.
This percentage includes the additional $50,000 of Year 4 in the payback time.
Answer (C) is incorrect.
This percentage takes the average inflow of all 8 years and divides that into the $500,000 initial
investment in the payback time.
Answer (D) is incorrect.
This percentage uses only the cash flows from the remaining 5 years in the payback time.
Question #49: Question: Foster's net present value for this project is (please refer the fact pattern
below)
[Fact Pattern #159] Foster Manufacturing is analyzing a capital investment project that is
forecasted to produce the following cash flows and net income:
After-tax
cash Net
Year flows income
$(20,000
0 ) $0
1 6,000 2,000
2 6,000 2,000
3 8,000 2,000
4 8,000 2,000

Foster's cost of capital is 12%.


Correct Answer: B. $924
A. $(1,600)
B. $924
C. $6,074
D. $6,998
Explanation:
Answer (A) is incorrect.
The amount of $(1,600) is not a meaningful amount
Answer (B) is correct.
The net present value of this project at 12% can be calculated as follows:
After-
tax cash Net Present
flows income value
Investme $(20,00 $(20,00
nt 0) $0 0)
1 6,000 2,000 5,358
2 6,000 2,000 4,782
3 8,000 2,000 5,696
4 8,000 2,000 5,088
$924

Answer (C) is incorrect.


The present value of the net incomes is $6,074, which is not a meaningful amount.
Answer (D) is incorrect.
The amount of $6,998 results from discounting the sums of the after-tax cash flows and net
income amounts.

Question #50: Lazar Industries produces two products. Crates and Trunks. Per unit selling prices,
costs, and resource utilization for these products are:

Crates Trunks
Selling price $20 $30
Direct material costs $5 $5
Direct labor costs 8 10
Variable selling
costs 1 2
Machine hours per
unit 2 4

Production of Crates and Trunks involves joint processes and use of the same facilities. The total
fixed factory overhead cost is $2,000,000 and total fixed selling and administrative costs are
$840,000. Production and sales are scheduled for 500,000 Crates and 700,000 Trunks. Lazar has
a normal capacity to produce a total of 2,000,000 units in any combination of Crates and Trunks,
and maintains no direct materials, work-in-process, or finished goods inventory.
Due to plant renovations Lazar Industries will be limited to 1,000,000 machine hours. What is
the maximum amount of contribution margin Lazar can generate during the renovation period?
Correct Answer: A. $2,000,000.
A. $2,000,000.
B. $7,000,000.
C. $3,000,000.
D. $1,500,000.
Explanation:
Contribution per machine hour is calculated as:
Contribution per machine hour = (unit contribution margin) / (machine hours per unit)
Unit contribution margin (CMU) = unit selling price — unit variable costs
Unit variable cost = direct materials + direct labor + variable overhead + variable selling
Unit variable cost, crates = $5 + $8 + $2 + $1 = $16
CMU for crates = ($20 — $16) = $4
Crate's contribution per machine hour = $4 / 2 hours = $2.00 per machine hour
Unit variable cost, trunks = $5 + $10 + $5 + $2 = $22
CMU for trunks = ($30 - $22) = $8
Trunk's contribution per machine hour = $8 / 4 hours = $2.00 per machine hour
The maximum total contribution margin that Lazar can generate in the coming year can then be
calculated as:
Maximum total contribution margin = ($2.00 per machine hour)(1,000,000 machine hours) =
$2,000,000.

Question #51: Which of the following is true about the impact of price inflation on financial ratio
analysis?
Correct Answer: C. Inflation impacts financial ratio analysis for one firm over time, as well as
comparative analysis of firms of different ages.
A. Inflation impacts financial ratio analysis for one firm over time, but not comparative
analysis of firms of different ages.
B. Inflation has no impact on financial ratio analysis.
C. Inflation impacts financial ratio analysis for one firm over time, as well as comparative
analysis of firms of different ages.
D. Inflation impacts comparative analysis of firms of different ages, but not financial ratio
analysis for one firm over time.
Explanation:
A. Inflation impacts both aspects
B. Inflation badly distorts firms' balance sheets, depreciation charges, inventory costs, and
profits.
C. Inflation impacts both aspects.
D. Inflation impacts both aspects.

Question #52: Please see the question below:


Garland Corporation's Income Statement for the year just ended is shown below.
$900,00
Net sales 0
Cost of goods sold
$125,00
Inventory-beginning 0
Purchases 540,000
Goods available for
sale 665,000
Inventory-ending 138,000
Cost of goods sold 527,000
Gross profit 373,000
Operating expenses 175,000
Income from $198,00
operations 0

Garland's average inventory turnover ratio is


Correct Answer: A. 4.01.
A. 4.01.
B. 6.84.
C. 3.82.
D. 6.52.
Explanation:
A. The inventory turnover ratio is Cost of Goods Sold + Average Inventory. Average
inventory is the average of the beginning and ending inventories, which is ($125,000 +
$138,000) ÷ 2, or $131,500. So the inventory turnover ratio is $527,000 $131,500, which
equals 4.01.
B. The inventory turnover ratio is Cost of Goods Sold ÷ Average Inventory. This is Sales ÷
Average Inventory.
C. The inventory turnover ratio is Cost of Goods Sold ÷ Average Inventory. This is Cost of
Goods Sold ÷ Ending Inventory.
D. The inventory turnover ratio is Cost of Goods Sold ÷ Average Inventory. This is Sales ÷
Ending Inventory.

Question #53: The graph below shows:


Correct Answer: A. a relatively inelastic demand.
A. a relatively inelastic demand.
B. a perfectly inelastic demand.
C. a relatively elastic demand.
D. a perfectly elastic demand.
Explanation:
A relatively inelastic demand means that a percentage change in price will result in a smaller
percentage change in quantity demanded. It is shown by a steep demand curve, which is not quite
perfectly vertical.

Question #54: Fact Pattern: Capital Invest, Inc., uses a 12% hurdle rate for all capital
expenditures following analysis for four projects for the upcoming year:
Question: Which project(s) should Capital Invest undertake during the upcoming year if it has
only $600,000 of funds available?
Project Project Project Project
1 2 3 4
Initial capital $200,00 $298,00 $248,00 $272,00
outlay 0 0 0 0
Annual net cash inflows
Year 1 65,000 100,000 80,000 95,000
Year 2 70,000 135,000 95,000 125,000
Year 3 80,000 90,000 90,000 90,000
Year 4 40,000 65,000 80,000 60,000
Net present value (3,798) 4,276 14,064 14,662
Profitability
index 98% 101% 106% 105%
Internal rate of
return 11% 13% 14% 15%
Correct Answer: D. Projects 3 and 4.
A. Projects 1 and 3.
B. Projects 2. 3. and 4.
C. Projects 2 and 3.
D. Projects 3 and 4.

Explanation:
Answer (A) is incorrect.
Project 1 has a negative NPV.
Answer (B) is incorrect.
This answer violates the $600,000 limitation.
Answer (C) is incorrect.
The combined NPV of Projects 2 and 3 is less than the combined NPV of Projects 3 and 4.
Answer (D) is correct.
Given that only $600,000 is available and that each project costs $200,000 or more, no more than
two projects can be undertaken. Because Projects 3 and 4 have the greatest NPVs, profitability
indexes, and IRRs, they are the projects in which the company should invest.

Question #55: The benefits of direct foreign investment by multinational corporations include all
of the following except
Correct Answer: D. More expropriation opportunities.
A. Easier access to scarce resources.
B. Improved earnings opportunities.
C. Improved international understanding.
D. More expropriation opportunities.
Explanation:
Answer (A) is incorrect.
They are all benefits of international investment.
Answer (B) is incorrect.
They are all benefits of international investment.
Answer (C) is incorrect.
They are all benefits of international investment.
Answer (D) is correct.
Benefits include easier access to scarce resources, improved earnings opportunities, and
improved international understanding. Expropriation is not a benefit. It is the risk that a foreign
government will nationalize a company's assets.

Question #56: Please see the Question Below:


The following financial information is given for Niska Corporation.

Year 1 Year 2
book value of assets $18,000 $26,000
market value of equity 18,000 60,000
12 months ended year 1 12 months ended year 2
sales $1,000 $1,300
cost of goods sold 500 700
operating income $500 $600
depreciation expense 200 200
interest expense 100 100
pre-tax income $200 $300
income tax expense 80 120
ney income $120 $180

Using a common-size income statement, did operating income and net income for Niska increase
or decrease?
Correct Answer: C. Operating income decreased; Net income increased.
A. Operating income increased; Net income decreased.
B. Operating income decreased; Net income decreased.
C. Operating income decreased; Net income increased.
D. Operating income increased; Net income increased.

Explanation:
A. The most common form of a vertical common-size income statement relates each item to
sales as a percentage of sales. Operating income as a percentage of sales did not increase
from Year 1 to Year 2; and net income as a percentage of sales did not decrease from
Year 1 to Year 2.
B. The most common form of a vertical common-size income statement relates each item to
sales as a percentage of sales. Net income as a percentage of sales did not decrease from
Year 1 to Year 2.
C. The most common form of a vertical common-size income statement relates each item to
sales as a percentage of sales. In Year 1, operating income was $500, which was 50% of
sales. In Year 2, operating income was $600, which was 46% of sales. Therefore;
operating income decreased as a percentage of sales. In Year 1, net income was $120,
which was 12% of sales. In Year 2, net income was $180, which was 13% of sales.
Therefore, net income increased as a percentage of sales.
D. The most common form of a vertical common-size income statement relates each item to
sales as a percentage of sales. Operating income as a percentage of sales did not increase
from Year 1 to Year 2.

Question #57: Please see the question below:


Cornwall Corporation's net accounts receivable were $68,000 and $47,000 at the beginning and
end of the year, respectively. Comwalfs condensed Income Statement is shown below. Sales
$900,000 Cost of goods sold $27,000 Operating expenses 175,000 Operating income 198,000
Income tax 79,000 Net income $119,000
Cornwall's average number of days' sales in accounts receivable (using a 360-day year) is
Correct Answer: D. 23 days.
A. 19 days.
B. 8 days.
C. 13 days.
D. 23 days.
Explanation:
A. This answer results from using year-end accounts receivable instead of the average
balance in accounts receivable during the year. The number of days' sales in accounts
receivable can be calculated in two different ways, both of which are two-step processes.
(1) Calculate the accounts receivable turnover ratio, then divide the number of days in the
year by the accounts receivable turnover ratio. The accounts receivable turnover ratio is
net credit sales divided by average accounts receivable. (2) Divide the annual credit sales
by the number of days in the year to find the amount of credit sales made per day. Then
divide average accounts receivable by the amount of credit sales made per day to find the
number of days' sales in receivables.
B. This is not the correct answer. Please see the correct answer for an explanation.
C. This is not the correct answer. Please see the correct answer for an explanation.
D. The number of days' sales in accounts receivable can be calculated in two different ways,
both of which are two-step processes. (1) Calculate the accounts receivable turnover
ratio, then divide the number of days in the year by the accounts receivable turnover
ratio. The accounts receivable turnover ratio is net credit sales divided by average
accounts receivable. Average accounts receivable is the average of the beginning and
ending receivable balances, or ($68,000 + $47,000) ÷ 2, which equals $57,500. Since we
are not told that Cornwall made any cash sales, we must assume that all sales were on
credit. The accounts receivable turnover ratio is $900,000 ÷ $57,500, which equals 15.65
times. 360 ÷ 15.65 = 23 days. (2) Divide the annual credit sales by the number of days in
the year to find the amount of credit sales made per day. Then divide average accounts
receivable by the amount of credit sales made per day to find the number of days' sales in
receivables. $900,000 ÷ 360 = $2,500. $57,500 ÷ $2,500 = 23 days. You can use either of
these formulas to find the number of days' sales in receivables. Whichever one makes the
most sense to you is the one you should use, because you will have less trouble learning
it.

Question #58: Sands, Inc. uses a calendar year for financial reporting. The company is
authorized to issue 5,000,000 shares of $10
par common stock. At no time has Sands issued any potentially dilutive securities. Listed below
is a summary of Sands' common stock activities.
Number of common shares issued and outstanding at Dec. 31, 20X6:1,000,000
Shares issued as a result of a 10% stock dividend on Sept. 30, 20X7: 100,000
Shares issued for cash on March 31, 20X8: 1,000,000
Number of common shares issued and outstanding at Dec. 31, 20X8:2,100,000
A two-for-one stock split of Sands' common stock took place on March 31, 20X9.
The weighted-average number of common shares used in computing earnings per common share
for 20X8 on the 20X8 comparative income statement was
Correct Answer: B. 1,850,000
A. 3,700,000
B. 1,850,000
C. 2,100,000
D. 1,600,000
Explanation:
A. This answer includes the effect of the 2-for-1 stock split that occurred on March 31,
20X9. However, the question asks for the weighted-average number of common shares
outstanding for 20X8 on the 20X8 comparative income statement. The stock split did not
take place until after the 20X8 comparative income statement was issued.
B. At the end of 20X7 there were 1,100,000 shares outstanding. To this, the company needs
to add the effect of any 20X8 share transactions. The only transaction was the issuance of
1,000,000 shares on March 31. Since these shares were outstanding for only 9 months of
the year, they will count as only 75% for the weighted-average share calculation. These
shares will become 750,000 shares for this calculation, giving the company 1,850,000
shares outstanding for the 20X8 EPS calculation.
C. This answer assumes that the shares that were issued in 20X8 were outstanding for the
entire period. In fact, they were outstanding for only 9 months and therefore should be
considered to be only 750,000 (1,000,000 x 9112) shares for the purposes of the
weighted-average number of common shares outstanding calculation.
D. This answer incorrectly assumes that the shares issued on March 31, 20X8 were
outstanding for only 6 months of the year, instead of 9 months.

Question #59: A company paid out one-half of last year's earnings in dividends. Earnings
increased by 20%, and the amount of its dividends increased by 15% in the current year. The
company's dividend payout ratio for the current year was
Correct Answer: • 47.9%
A. 50%
B. 57.5%
C. 47.9%
D. 78%
Explanation:
Answer (A) is incorrect.
The prior-year payout ratio is 50%.
Answer (B) is incorrect.
The figure of 57.5% is 115% of the prior-year payout ratio.
Answer (C) is correct.
The prior-year dividend payout ratio was 50%. Hence, if prior-year net income was X, the total
dividend payout would have been 50%X. If earnings increase by 20%, current-year income will
be 120%X. If dividends increase by 15%, the total dividends paid out will be 57.5%X (115% x
50%X), and the new dividend payout ratio will be 47.9% (57.5%X ÷ 120%X).
Answer (D) is incorrect.
The figure of 78% equals 65% of 120%.
Question #60: Depoole Company is a manufacturer of industrial products and employs a
calendar year for financial reporting purposes. Assume that total quick assets exceeded total
current liabilities both before and after the transaction described. Further assume that Depoole
has positive profits during the year and a credit balance throughout the year in its retained
earnings account.
Payment of a trade account payable of $64,500 would
Correct Answer: D. Increase both the current and quick ratios.
A. Increase the quick ratio but the current ratio would not be affected.
B. Increase the current ratio but the quick ratio would not be affected.
C. Decrease both the current and quick ratios.
D. Increase both the current and quick ratios.
Explanation:
A. The current ratio would be affected by this transaction as both the numerator and
denominator of the current ratio would be decreased by the amount of the liability settled.
B. The quick ratio would be affected by this transaction as both the numerator and
denominator of the quick ratio would be decreased by the amount of the liability settled.
C. Because both of these ratios were greater than 1.0 prior to this transaction, both of these
ratios will increase as a result of the settlement of the liability.
D. Because Depoole has a quick and current ratio of greater than one (because total quick
assets exceeded total current liabilities), the payment of a payable would increase both
ratios. This is because the payment of a payable will reduce both the current assets and
the current liabilities by the same amount. Let us assume that Depoole had current and
quick assets of $150 and current liabilities of $100. Given that the current and quick
assets were both $150, both ratios are 1.50. Now let assume that they paid a $20 liability.
This reduced the quick and current assets to $130 and reduced the current liabilities to
$80. Now, both ratios are 1.625, which is higher than they had been.

Question #61: A company's approach to a make-buy decision:


Correct Answer: A. involves an analysis of avoidable costs.
A. involves an analysis of avoidable costs.
B. depends on whether the company is operating at or below breakeven.
C. should utilize absorption (full) costing.
D. should utilize activity-based costing.
Explanation:
The relevant make costs in a make-versus-buy decision are the opportunity costs to manufacture
a product and the costs of purchasing. The opportunity cost to manufacture a product consists of
avoidable variable and avoidable fixed costs and the cost of any lost opportunities.

Question #62: A firm is evaluating a proposed acquisition of a new machine at a purchase price
of $380,000 and installation charges that will amount to $20,000. A $15,000 increase in working
capital will be required. The machine will have a useful life of 4 years, after which it can be sold
for $50,000. The estimated annual incremental operating revenues and cash operating expenses
are $750,000 and $500,000, respectively, for each of the 4 years. The tax rate is 40%, and the
cost of capital is 12%. Straight-line depreciation is used for both financial reporting and income
tax purposes.
If the project is accepted, the initial investment will be
Correct Answer: D. $415,000
A. $345,000
B. $365,000
C. $385,000
D. $415,000
Explanation:
Answer (A) is incorrect
The amount of $345,000 results from improperly netting the installation costs and the increase in
working capital against the purchase price of the new equipment.
Answer (B) is incorrect
The amount of $365,000 results from improperly netting the increase in working capital against
the purchase price of the new equipment.
Answer (C) is incorrect
The amount of $385,000 results from improperly netting the increase in working capital against
the full cost of the new equipment.
Answer (D) is correct
The net initial investment for a capital project consists of three components the purchase of new
equipment, the increase in working capital, and the salvage value of old equipment. The
calculation is as follows:

Cost of new equipment ($380,000 + $20,000) $400,000


Increase in working capital 15,000
Salvage value of old equipment 0
Net initial investment $415,000

Question #63: All of the following are the rates used in net present value analysis except for the
Correct Answer: D. Accounting rate of return.
A. Cost of capital.
B. Hurdle rate.
C. Discount rate.
D. Accounting rate of return
Explanation:
Answer (A) is incorrect.
Cost of capital is a synonym for the rate used in NPV analysis.
Answer (B) is incorrect.
Hurdle rate is a synonym for the rate used in NPV analysis.
Answer (C) is incorrect.
Discount rate is a synonym for the rate used in NPV analysis.
Answer (D) is correct.
The NPV is the excess of the present values of the estimated cash inflows over the net cost of the
investment. The discount rate used is sometimes the cost of capital or other hurdle rate
designated by management. This rate is also called the required rate of return. The accounting
rate of return is never used in NPV analysis because it ignores the time value of money; it is
computed by dividing the accounting net income by the investment.

Question #64: Investors Group Ltd. holds an investment portfolio that has a daily value-at-risk
(VaR) of $500,000 at a 99% confidence level. This means that
Correct Answer: B. the portfolio will probably lose more than $500,000 of its value on one day
out of every 100 trading days, or two or three days out of each year.
A. the expected loss on the portfolio is $500,000.
B. the portfolio will probably lose more than $500,000 of its value on one day out of every
100 trading days, or two or three days out of each year.
C. the expected return on the portfolio is 99%.
D. the expected return on the portfolio is 99%.
Explanation:
A. VaR does not provide information about how much could be lost in the portfolio over a
given period of time but rather provides a worst case scenario and an estimation of how
often that worst case scenario will occur (called a confidence interval).
B. The Value-at-Risk model indicates the maximum loss over a given time period (here, one
day) such that there is a low probability (here, a 1% probability, or 100% minus 99%),
that the actual loss over the given period will be larger. The 1% probability translates to
the event occurring on one out of every 100 trading days, or two to three days out of each
year. Therefore, the portfolio will probably lose more than $500,000 of its value in a one-
day period two or three times each year.
C. VaR does not provide information about expected return on a portfolio, but rather
provides a worst case scenario and an estimation of how often that worst case scenario
will occur (called a confidence interval).
D. VaR does not provide information about the market value of a portfolio or how much
could be lost in the portfolio over a given period of time but rather provides a worst case
scenario and an estimation of how often that worst case scenario will occur (called a
confidence interval).

Question #65: Given fixed costs of $10,000, variable costs per unit of $195, and a unit selling
price of $370, how many units must be sold to reach a target operating income of $60,000? (Use
the contribution margin method.)
Correct Answer: C. 400 units.
A. 343 units.
B. 358 units.
C. 400 units.
D. 189 units.
Explanation:
The formula for the contribution margin method to earn a target profit level is:
¿ costs+target operating income $ 10,000+ $ 60,000 $ 70,000
QT = = = =400 units .
unit contribution margin $ 175 $ 175
Question #66: Lisa, Inc.
Statement of Financial Position
Assuming that Lisa Inc.'s net income for 20X4 was $35,000 and there were no preferred stock
dividends in arrears, Lisa's return on common equity for 20X4 was

20X4 20X3
Assets
current assets:
cash $30 $25
trading securities 20 15
accounts receivable (net) 45 30
inventories (at lower of cost of market) 60 50
prepaid items 15 20
total current assets $170 $140
long term assets:
long term investments:
available for sale investments $25 $20
property, plant & equipment:
land (at cost ) 75 75
building (net) 80 90
equipment (net) 95 100
intangible assets:
patents (net) 35 17
goodwill (net) 20 13
total long term assets $330 $315
total assets $500 $455
liabilities and equity
current liabilities:
notes payable $23 $12
accounts payable 47 28
accrued interest $15 $15
total current liabilities $85 $55
long term liabilities:
long term notes payable 10% due 12/31/20X6 $10 $10
bonds payable 12% due 12/31/20X9 15 15
total long term debt $25 $25
total liabilities $110 $80
shareholders' equity
preferred stock-5% cumulative, $100 par, nonparticipating authorized, issued and
outstanding, 1,000 shares $100 $100
common stock-$10 par 20,000 shares authorized, 15,000 shares issued and
outstanding 150 150
additional paid in capital-common 75 75
retained earnings 65 50
total equity $390 $375
total liabilities & equity $500 $455

Correct Answer: D. 10.6%


A. 7.8%
B. 10.9%
C. 12.4%
D. 10.6%
Explanation:
A. This answer includes the preferred equity in the denominator of the calculation, instead of
common equity.
B. This answer uses only the beginning common equity in the denominator, and not the
average common equity.
C. This answer uses the entire net income in the numerator rather than the income available
for common shareholders.
D. The return on common equity is calculated as the income available for common
shareholders divided by the average common equity. Income available to common
shareholders is net income minus preferred dividends. The income available to common
shareholders is $30,000 because $5,000 (5% of $100,000) of the $35,000 of income
needs to be reserved for the cumulative preferred dividend. This $30,000 is divided by
the average common equity which includes the common shares, the additional paid in
capital and the retained earnings. In total, this is $275,000 at the beginning of the year
and $290,000 at the end of the year, for an average of $282,500. Dividing the income for
common shareholders of $30,000 by the average common equity of $282,500, we get
10.6% return on common equity.

Question #67: Bargain Press is considering publishing a new textbook. The publisher has
developed the following cost data related to a production run of 6,000, the minimum possible
production run. Bargain Press will sell the textbook for $45 per copy. How many textbooks must
Bargain Press sell in order to generate operating earnings (earnings before interest and taxes) of
20% on sales? (Round your answer up to the nearest whole textbook.)
estimate
d cost
development (reviews, class testing,
editing) $35,000
typesetting 18,500
depreciation on equipment 9,320
general and administrative 7,500
miscellaneous fixed costs 4,400
printing and binding 30,000
sales staff commissions (2% of selling
price) 5,400
bookstore commissions (25% of selling
price) 67,000
author's royalties (10% of selling price) 27,000
$204,62
total costs at production of 6,000 copies 0

Correct Answer: A. 5,412 copies.


A. 5,412 copies.
B. 6,199 copies.
C. 2,076 copies.
D. 5,207 copies.
Explanation:
The number of copies required can be found by solving the following equation; where x equals
the number of copies:
Total sales - total variable costs - fixed costs = (return %) (total sales)
Total sales = $45x Fixed costs = development + typesetting + depreciation + general and
administrative + miscellaneous fixed costs + printing and binding
Fixed costs = $35,000 + $18,500 + $9,320 + $7,500 + $4,400 + $30,000 = $104,720
Total variable cost = (variable cost per unit, or VCU) (x) = VCU(x)
VCU = (sales staff commissions + bookstore commissions + royalties) / (number of copies)
VCU = ($5,400 + $67,500 + $27,000) / (6,000 copies) = $99,900 / 6,000
VCU = $16.65 per copy
$45x - $16.65(x) - $104,720 = (0.2) ($45x)
$45x - $16.65(x) - $104,720 = $9x
$28.35(x) - $104,720 = $9x
$19.35x = $104,720
x = 5,411.89; which rounds to 5,411 copies.
Question #68: What is a common approach to monitoring the effectiveness of ethical training
and employees' understanding of the company ethics policy and their compliance with it?
Correct Answer: D. Human performance feedback loop
A. Quality management
B. Continual process improvement
C. Pareto principle
D. Human performance feedback loop
Explanation:
A. This choice is not correct because it is a process management technique that would be
used to identify and evaluate risk situations risk situations related to ethical behavior in
order to prevent mistakes from occurring. When risk situations are identified; the
management accountant develops controls to address the risks related to ethical behavior.
B. This choice is not correct because it is a process management technique that involves
constant monitoring of business processes in order to learn from experience and adapt to
new situations before they create major problems.
C. This choice is not correct because the Pareto principle is a concept in quality management
that states that 20% of a population causes 80% of a specific outcome.
D. This is the correct answer because human performance feedback loop is a performance
management tool between people, usually supervisors and their staff; regarding various
subjects including their actual behavior versus defined standards or expectations. It is a
part of the performance management process for individual employees. Employee
performance is measured against specific goals to ensure that the company's ethical
expectations of its employees are understood.

Question #69: A company is considering the purchase of a new machine to replace a 5-year old
machine and has gathered the following information:
Purchase price of new machine: $50,000
Installation cost of new machine: 4,000
Market value (selling price) of the old machine: 5,000
Book value of the old machine: 2,000
Increase in net working capital if new machine is installed: 1,000
Effective income tax rate: 40%
If the company replaces the old machine with the new machine, what is the cash flow in period
0?
Correct Answer: B. $(51,200)
A. $(49,000)
B. $(51,200)
C. $(51,800)
D. $(53,000)

Explanation:
Answer (A) is incorrect.
Improperly netting the salvage value of the old equipment against the full cost of the new
equipment results in $(49,000).
Answer (B) is correct.
The net initial investment for a capital project consists of three components: the purchase of new
equipment, the increase in working capital, and the after-tax proceeds from the disposal of old
equipment. For this company, the first of these is $54,000 ($50,000 + $4,000), and the second is
$1,000. The calculation of the after-tax proceeds from the disposal of the old equipment is as
follows: Salvage value of old equipment $ 5,000
Less: current tax value (2,000)
Tax-basis gain (loss) on disposal $ 3,000
Times: tax rate x 40%
Tax detriment (benefit) from disposal $ 1,200
Salvage value of old equipment $ 5,000
Less: tax detriment from gain on disposal (1,200)
After-tax cash inflow from disposal $ 3,800
The net cash flow for period 0 is therefore:
Full cost of new equipment ($50,000 + $4,000) $54,000
Increase in working capital 1,000
After-tax proceeds from disposal of old equipment (3,800)
Net initial investment $51,200
Answer (C) is incorrect.
The amount of $(51,800) results from using the complement of the tax rate instead of the tax
rate.
Answer (D) is incorrect.
The amount of $(53,000) results from improperly netting the increase in working capital against
the full cost of the new equipment.

Question #70: Fact Pattern: Jensen Corporation's board of directors met on June 3 and declared a
regular quarterly cash dividend of $.40 per share for a total value of $200,000. The dividend is
payable on June 24 to all stockholders of record as of June 17. Excerpts from the statement of
financial position for Jensen Corporation as of May 31 are presented as follows.

cash $400,000
accounts receivable
(net) 800,000
inventories 1,200,000
$2,400,00
total current assets 0
total current $1,000,00
liabilities 0

Assume that the only transactions to affect Jensen Corporation during June are the dividend
transactions. Question: Jensen's total stockholders' equity would be
Correct Answer: D. Decreased by the dividend declaration and unchanged by the dividend
payment.
A. Unchanged by the dividend declaration and decreased by the dividend payment.
B. Decreased by the dividend declaration and increased by the dividend payment.
C. Unchanged by either the dividend declaration or the dividend payment.
D. Decreased by the dividend declaration and unchanged by the dividend payment.

Explanation:
Answer (A) is incorrect.
A dividend declaration reduces retained earnings and total stockholders' equity. The
stockholders' equity because only cash and dividends payable are reduced.
Answer (B) is incorrect.
A dividend declaration reduces retained earnings and total stockholders' equity. The
stockholders' equity because only cash and dividends payable are reduced.
Answer (C) is incorrect.
A dividend declaration reduces retained earnings and total stockholders' equity. The
stockholders' equity because only cash and dividends payable are reduced.
Answer (D) is correct.
A dividend declaration reduces retained earnings and thus total stockholders' equity. The
subsequent payment will have no effect on stockholders’ equity since only cash and dividents
payable are reduced.

Question #71: If a firm has a beta value of 1.0, then its


Correct Answer: C. Expected return should approximate the overall market.
A. Return should equal the risk-free rate.
B. Price is relatively stable.
C. Expected return should approximate the overall market.
D. Volatility is low.
Explanation:
Answer (A) is incorrect.
The risk-free rate is the rate returned by very safe and nonvolatile securities, such as U.S.
government Treasury bills.
Answer (B) is incorrect.
A beta value of 1.0 only means the price of the stock moves in concert with that of the overall
market; if the market is not stable, the stock price will not be either.
Answer (C) is correct.
The effect of an individual security on the volatility of a portfolio is measured by its sensitivity
to movements by the overall market. This sensitivity is stated in terms of a stock's beta
coefficient. If the beta coefficient is 1.0, then the price of that stock tends to move in the same
direction and to the same degree as the overall market.
Answer (D) is incorrect.
A beta value of 1.0 only means the price of the stock moves in concert with that of the overall
market; if the market is volatile, the stock price will be also.

Question #72: ( please refer the fact pattern below) Question: Volpone Company's average
number of days to collect accounts receivable for Year 2 is
[Fact Pattern 433] The following inventory and sales data are available for the current year for
Volpone Company. Volpone uses a 365-day year when computing ratios.
novemb
november er 30,
30, year2 year 1
net credit $6,205,00
sales 0
gross
receivables 350,000 320,000
inventory 960,000 780,000
cost of goods
sold 4,380,000

Correct Answer: C. 19.71 All Possible Answers:


A. 18.82
B. 19.43
C. 19.71
D. 20.59
Explanation:
Answer (A) is incorrect.
The number of 18.82 days is based on receivables of $320,000.
Answer (B) is incorrect.
The number of 19.43 days is based on a 360-day year.
Answer (C) is correct.
The average collection period equals 365 days divided by the receivables turnover (net credit
sales ÷ average accounts receivable). Turnover is 18.52 times {$6,205,000 sales ÷ [($350,000 +
$320,000) ÷ 2]}. Hence, the average collection period is 19.71 days (365 ÷ 18.52).
Answer (D) is incorrect.
The number 20.59 days is based on receivables of $350,000.

Question #73: The Chief Investment Officer is performing an analysis to determine if her firm
should invest in new equipment to produce a product recently developed by her firm. The option
would be to abandon the product. She uses the net present value (NPV) method and discounts at
the firm's cost of capital. The Chief Investment Officer is contemplating how to handle the
following items:
I. The book value of warehouse space currently used by another division.
II. Interest payments on debt to finance the equipment.
III. Increased levels of accounts payable and inventory.
IV. R&D spent in prior years and treated as a deferred asset for book and tax purposes.
Which of the above items are relevant for the Chief Investment Officer to consider in
Correct Answer: C. III only.
A. I, II, III and IV.
B. II. III and IV only.
C. III only.
D. III and IV only.
Explanation:
Answer (A) is incorrect.
The book value of warehouse space currently in use and R&D costs already incurred are
irrelevant. The consideration of interest payments on debt to finance equipment is a separate
financing decision that is not related to the decision to produce.
Answer (B) is incorrect.
The book value of warehouse space currently in use is irrelevant because it remains the same
regardless of whether or not the new product is produced. The consideration of interest payments
on debt to finance equipment is a separate financing decision that is not related to the decision to
produce. R&D costs already incurred are sunk costs and are therefore irrelevant.
Answer (C) is correct.
The relevant costs in any decision are those costs that will vary depending on the choice made.
Increased levels of accounts payable and inventory are thus items the Chief Investment Officer
should consider.
Answer (D) is incorrect.
R&D costs already incurred are irrelevant.

Question #74: A major supplier has offered a corporation a year-end special purchase whereby it
could purchase 180,000 cases of sport drink at $10 per case. The corporation normally orders
30,000 cases per month at $12 per case. The corporation's cost of capital is 9%. In calculating the
overall opportunity cost of this offer, the cost of carrying the increased inventory would be
Correct Answer: A. $32,400
A. $32,400
B. $40,500
C. $64,800
D. $81,000
Explanation:
Answer (A) is correct.
If the corporation makes the special purchase of 6 months of inventory (180,000 cases ÷ 30,000
cases per month), the average inventory for the 6-month period will be $900,000 [(180,000 x
$10) ÷ 2]. If the special purchase is not made, the average inventory for the same period will be
the average monthly inventory of $180,000 [(30,000 x $12) ÷ 2]. Accordingly, the incremental
average inventory is $720,000 ($900,000 — $180,000), and the interest cost of the incremental
6-month investment is $32,400 [($720,000 x 9%) ÷ 2].
Answer (B) is incorrect.
The amount of $40,500 is the result of assuming an incremental average inventory of $900,000.
Answer (C) is incorrect.
The interest cost for 12 months is $64,800.
Answer (D) is incorrect.
The amount of $81,000 is the result of assuming an incremental average inventory of $900,000
and a 12-month period

Question #75: The formal code of ethical conduct for a company should do all of the following
except
Correct Answer: A. reflect only the legal standards of conduct of employees and the organization
A. reflect only the legal standards of conduct of employees and the organization.
B. provide guidance on compliance requirements for domestic and international operations.
C. provide guidance on behavior for employees when making decisions.
D. effectively communicate acceptable values to all employees.
Explanation:
A. The formal code of ethical conduct for a company should not reflect only the legal
standards of conduct of employees and the organization. Relevant laws and external
industry standards should be appropriately referenced in the formal code of ethical
conduct, but they cannot be the only component of a formal code of ethical conduct.
B. The formal code of ethical conduct for a company should provide guidance on
compliance requirements for domestic and international operations. Relevant laws and
external industry standards should be appropriately referenced in the formal code of
ethical conduct.
C. The formal code of ethical conduct for a company should provide guidance on behavior
for employees when making decisions. No code can address all ethical situations
employees may face. However, the organization's values should be set forth as the basis
for ethical decision-making to help employees do the right thing in situations that may
not have clear answers.
D. The formal code of ethical conduct for a company should effectively communicate
acceptable values to all employees. The corporate values should be used as a framework
for employees to use in making decisions.

Question #76: Question: The existing machine has been in service for 5 years and could be sold
currently for $25,000. Calvin expects to realize annual before-tax reductions in labor costs of
$30,000 if the new machine is purchased and placed in service. If the new machine is purchased,
the incremental cash flows for the first year would amount to (please refer the fact pattern below)
[Fact Pattern #150] Calvin, Inc., is considering the purchase of a new state-of-the-art machine to
replace its hand-operated machine, effective tax rate is 40%, and its cost of capital is 12%. Data
regarding the existing and new machines are presented below.
existing new
machine machine
original cost $50,000 $90,000
installation costs 0 4,000
freight and insurance 0 6,000
expected end salvage
value 0 0
straight- straight
depreciation method line line
expected useful life 10 years 5 years

Correct Answer: B. $24,000


A. $18,000
B. $24,000
C. $30,000
D. $45,000
Explanation:
Answer (A) is incorrect.
The amount of $18,000 results from failing to include the depreciation tax shield.
Answer (B) is correct.
The estimated incremental after-tax operating cash flows for each year of a capital project consist
of two components; the after-tax cash inflows from operations and the difference in depreciation
tax shields between the old and new equipment. The first of these for Calvin can be calculated as
follows:
Net annual labor cost savings $30,000
Less: income tax expense ($30,000 x 40%) (12,000)
After-tax cash inflow from operations $18,000
The depreciation tax shield on the new equipment is derived as follows:
Cost of new equipment ($90,000 + $4,000 + $6,000) $100,000
Divided by: estimated useful life ÷ 5
Annual depreciation expense $ 20,000
Times: tax rate x 40%
Annual depreciation tax shield -- new equipment $ 8,000
The depreciation tax shield on the old equipment is derived as follows:
Cost of old equipment $50,000
Less: accumulated depreciation [$50,000 x (5 ÷ 10 years)] (25,000)
Current book value $25,000
Divided by: remaining useful life ÷ 5
Annual depreciation expense $ 5,000
Times: tax rate x 40%
Annual depreciation tax shield -- old equipment $ 2,000
The difference in the two depreciation tax shields is $6,000 ($8,000 — $2,000). Calvin's total
incremental cash flow for the first year of this project is therefore estimated at $24,000 ($18,000
+ $6,000).
Answer (C) is incorrect.
The amount of $30,000 is merely the before-tax savings in labor cost.
Answer (D) is incorrect.
The amount of $45,000 results from using the difference in depreciation expense between the
two pieces of equipment without accounting for the effect of income taxes.
Question #77: What is one of the reasons why a documented code of ethics is important to a
business and its employees?
Correct Answer: B. A documented code of ethics provides a framework for decision making in
situations where an explicit company policy does not exist.
A. Existence of a documented code of ethics frees senior management from responsibility
for inappropriate behavior by employees of the business.
B. A documented code of ethics provides a framework for decision making in situations
where an explicit company policy does not exist.
C. Once the code of ethics is documented, expensive company training regarding ethical
behavior standards in the company can be discontinued to save money.
D. Once the code of ethics is documented, senior management can turn their attention to
more important issues such as revenue generation and customer satisfaction.
Explanation:
A. This choice is not correct because company management is never freed from
responsibility for inappropriate behavior by employees of the business.
B. There will be many situations in a business environment where an explicit company
policy does not exist. If the company's culture is rules based but a specific circumstance
is not addressed in the rules, employees will fall back on their personal values to resolve
ethical dilemmas. That could put the organization's internal control objectives at risk,
because the decisions being made will be based on the employees' personal values and
not the values of the company. A documented code of ethics gives the employees a
framework for decision-making when no explicit rule exists. This framework should lead
to decisions that are consistent with the company's overall objectives, ensure consistency
in decisions, and lead to a common understanding of expected behaviors, since the
personal values of each individual will not be the driving factor in their decisions.
C. This choice is not correct because the documentation of a code of conduct is not a reason
to discontinue ethics training. Ethics training should be ongoing.
D. This choice is not correct because senior management should always be involved in
ethics regarding the values and culture of their company.

Question #78: An increase in the market price of a company's common stock will immediately
affect its
Correct Answer: D. Dividend yield.
A. Debt to equity ratio.
B. Basic earnings per share.
C. Dividend payout ratio.
D. Dividend yield.
Explanation:
A. The debt-to-equity ratio is total liabilities divided by total equity on the balance sheet.
The total debt-to-equity ratio is not affected by the market price of the company's shares.
B. Basic earnings per share is income available to common shareholders divided by the
weighted average number of common shares outstanding during the period. Basic
earnings per share is not affected by the market price of the company's shares.
C. The dividend payout ratio is total common dividends paid annually divided by income
available to common shareholders. The dividend payout ratio is not affected by the
market price of the company's shares.
D. The dividend yield ratio is calculated as the cash dividend divided by the market price of
the share. As the market price of the shares increases; the dividend yield will decrease.

Question #79: An avoidable cost can most accurately be described as:


Correct Answer: A. a cost that can be eliminated in whole or in part through a business
decision or action
A. A cost that can be eliminated in whole or in part through a business decision or
action.
B. A cost involved in producing additional units of a product
C. An ongoing cost that remains in force regardless of a decision or action.
D. A cost involved in producing fewer units of a product.
Explanation:
By definition; an avoidable cost (or escapable cost) is one that can be eliminated in
whole or in part by choosing one alternative over another in the decision-making
process. An avoidable cost might be eliminated by ceasing to perform an activity or by
improving the efficiency of the activity.

Question #80: Jeffries Company sells its single product for $30 per unit. The
contribution margin ratio is 45%, and fixed costs are $10,000 per month. Sales were
3,000 units in April and 4,000 units in May. How much greater is the May operating
income than the April operating income?
Correct Answer: C. $13,500
A. $16,500
B. $30,000
C. $13,500
D. $10,000
Explanation:
The difference in operating income is calculated as follows:
Difference in operating income = (contribution margin ratio) (additional units) (unit
selling price)
Difference in operating income = (0.45) (1,000 units) ($30) = $13,500.

Question #81: The most likely adverse effect on a multinational's home country is
Correct Answer: D. Loss of tax revenues.
A. Flow of royalties and dividends out of the home country
B. Manipulation of transfer prices.
C. Loss of technology
D. Loss of tax revenues.
Explanation:
Answer (A) is incorrect
Net capital outflow is more likely to be an adverse effect on the host.
Answer (B) is incorrect
Manipulation of transfer prices is more likely to be an adverse effect on the host. The
result is establishment of economically unreasonable transfer prices among subsidiaries
so that profits will be earned where taxes are lowest or restrictions on the export of
profits are least stringent.
Answer (C) is incorrect
A benefit to the host is investment in new technology
Answer (D) is correct.
Adverse effects on the home country include (1) loss of jobs and tax revenues; (2)
instability caused by reduced flexibility of operation in a foreign political system, (3) the
risk of expropriation, and (4) the competitive advantage of multinationals over domestic
rivals.

Question #82: Which one of the following statements best reflects the relationship
between the results of financial ratios calculated in a local currency versus those where a
functional currency is used?
Correct Answer: D. Financial ratio results are different under translation and re-
measurement, and ratios under translation are also often different from those in the local
currency.
A. Financial ratio results are similar under translation and re-measurement; and ratios
under translation are also often similar to those in the local currency.
B. Financial ratio results are similar under translation and re-measurement; but ratios
under translation are often different from those in the local currency.
C. Financial ratio results are different under translation and re-measurement, but ratios
under translation are often similar from those in the local currency.
D. Financial ratio results are different under translation and remeasurement7 and ratios
under translation are also often different from those in the local currency.
Explanation:
Answer (A) is incorrect
Financial ratio results are different under translation and re-measurement
Answer (B) is incorrect
Financial ratio results are different under translation and re-measurement; and ratios
under translation are also often different from those in the local currency
Answer (C) is incorrect
Ratios under translation are also often different from those in the local currency.
Answer (D) is correct.
Financial ratio results are different under translation and re-measurement, and ratios
under translation are also often different from those in the local currency.

Question #83: A make or buy cost analysis involves all of the following considerations
except:
Correct Answer: C. comparison of incremental revenue with incremental costs
A. Evaluation of an external supplier
B. Comparison of relevant internal costs with the cost to purchase externally
C. Comparison of incremental revenue With incremental costs
D. Consideration of opportunity costs.
Explanation:
The terminology "make versus buy" refers to outsourcing. Reaching a decision about
whether to make or buy generally involves a comparison of the relevant costs to make
the item internally with the cost to purchase externally. In some situations; opportunity
costs and qualitative factors also need to be considered.

Question #84: Sarbanes Oxley Section 406 defines a code of ethics as a statement of
standards that is necessary to promote which of the following corporate objectives?
Correct Answer: A. Honest and ethical conduct, including the ethical handling of actual
or apparent conflicts of interest between personal and professional relationships.
A. Honest and ethical conduct, including the ethical handling of actual or apparent
conflicts of interest between personal and professional relationships.
B. Creation of a work environment with a good work-life balance for all employees.
C. Achievement of earnings per share targets that are communicated to shareholders.
D. Increasing dividends for shareholders.
Explanation:
A. A code of ethics defines company's standards for human behavior; this answer relates
to human behavior and can be applied to a broad range of situations.
B. This answer is close to being correct. However it is not the best answer because a
code of ethics should provide guidance for human behavior in a broad sense, not only
in a narrow area like work-life balance.
C. A code of ethics defines company's standards for human behavior As such; this
answer is not correct because these goals relate to a financial objective
D. A code of ethics defines company's standards for human behavior As such; this
answer is not correct because these goals relate to a financial objective.

Question #85: The difference between the sales price and total variable costs is: X
Correct Answer: C. The contribution margin.
A. Gross operating profit
B. Net profit
C. The contribution margin.
D. The break-even point.
Explanation: The definition of contribution margin is sales price less total variable costs.

Question #86: Given fixed costs of $25,000, variable costs per unit of $1,500, and a unit
contribution margin of $300, how many units must be sold to reach a target operating
income of $200,000? (Use the contribution margin method.)
Correct Answer: B. 750 units.
A. 167 units.
B. 750 units.
C. 650 units.
D. 583 units.
Explanation:
The formula for the contribution margin method to earn a target profit level is.
¿ costs+target operating income $ 25,000+ $ 200,000 $ 225,000
QT = = = =750 units .
unit contribution margin $ 300 $ 300

Question #87: If a product's demand is elastic and there is a decrease in price, the effect
is:
Correct Answer: D. An increase in total revenue.
A. A decrease in total revenue and the demand curve shifts to the left.
B. No change in total revenue.
C. A decrease in total revenue.
D. An increase in total revenue.
Explanation:
Price elasticity (E) is the percent change in a product's quantity demanded given a
percent change in the product's price. Elastic demand occurs when the percent change in
demand is greater than the percent change in price (E>l). Since the demand is increasing
at a higher rate than the price decrease, total revenue will increase. For example, if a
change in price from $50 to $45 increases demand from 100 units to 150 units, then total
revenue (price x demand) will go from $5,000 ($50 x 100 units) to $6,750 ($45 x 150
units).

Question #88: Please see the question below:


Maydale Inc.'s financial statements show the following information.
Accounts receivable, end of Year IS 320,000 Credit sales for Year 2 Accounts
receivable, end of Year 2 400,000
Maydale's accounts receivable turnover ratio is
Correct Answer: C. 10.00.
A. 0.10
B. 11.25
C. 10.00.
D. 9.00
Explanation:
A. This is average accounts receivable divided by net credit sales for the year. The
accounts receivable turnover ratio is net credit sales divided by average accounts
receivable.
B. This is net credit sales for the year divided by year end accounts receivable. The
accounts receivable turnover ratio is net credit sales divided by average accounts
receivable Whenever we relate an income statement amount such as net credit sales
to a balance sheet amount such as accounts receivable; we need to use the average
balance of the balance sheet amount not the beginning or ending balance.
C. The accounts receivable turnover ratio is net credit sales divided by average accounts
receivable Average accounts receivable is the average of the beginning and ending
balances; which is $360,000. Net credit sales is $3,600,000, so the accounts
receivable turnover ratio is $3,600,000 ÷ $360,000, which equals 10. Accounts
receivable 'turns over" 10 times during the year
D. This is net credit sales for the year divided by the beginning of the year accounts
receivable balance The accounts receivable turnover ratio is net credit sales divided
by average accounts receivable Whenever we relate an income statement amount
such as net credit sales to a balance sheet amount such as accounts receivable; we
need to use the average balance of the balance sheet amount not the beginning or
ending balance.

Question #89: A company had $5 million in sales, $3 million in cost of goods sold, and
$1 million in selling and administrative expenses during the last fiscal year. If the
company's income tax rate was 25%, what was the company's gross profit margin
percentage?
Correct Answer: . 40%
A. 20%.
B. 50%.
C. 30%.
D. 40%
Explanation:
A. This is net income before tax divided by sales revenue. The gross profit margin
percentage is gross profit divided by sales revenue. Gross profit is sales revenue
minus cost of goods sold.
B. This is not the correct answer. The gross profit margin percentage is gross profit
divided by sales revenue. Gross profit is sales revenue minus cost of goods sold
C. This is not the correct answer. The gross profit margin percentage is gross profit
divided by sales revenue. Gross profit is sales revenue minus cost of goods sold
D. This is the correct answer. The gross profit margin percentage is gross profit divided
by sales revenue. Gross profit is sales revenue minus cost of goods sold. Gross profit
equals $5,000,000 - $3,000,000, which is $2,000,000. The gross profit margin
percentage is $2,000,000 ÷ $5,000,000, which is 0.40 or 40%.

Question #90: According to the IMA Statement of Ethical Professional Practice, one of
the overarching ethical principles is "Fairness." "Fairness" means
Correct Answer: B. acting in an impartial manner; being open-minded; tolerant and
accepting; being free from injustice.
A. Communicating information fairly and objectively
B. Acting in an impartial manner; being open-minded; tolerant and accepting; being free
from injustice.
C. Being free from personal feelings or prejudice, basing analyses and decisions on the
facts alone and basing a judgment on an established set of criteria
D. Communicating both good and bad news; not just telling your superiors what they
want to hear
Explanation:
A. This is one of the responsibilities for an IMA member with respect to the
"Credibility" standard of ethical professional conduct
B. This is the definition of "Fairness '
C. This is the definition of "Objectivity "
D. This is an example of ways to maintain integrity.

Question #91: Freely fluctuating exchange rates perform which of the following
functions?
Correct Answer: A. They automatically correct a lack of equilibrium in the balance of
payments.
A. They automatically correct a lack of equilibrium in the balance of payments.
B. They make imports cheaper and exports more expensive
C. They impose constraints on the domestic economy.
D. They eliminate the need for foreign currency hedging.
Explanation:
Answer (A) is correct.
In a freely floating exchange rate system7 the government steps aside and allows
exchange rates to be determined entirely by the market forces of supply and demand.
The advantage of such a system is that it tends to automatically correct any
disequilibrium in the balance of payments. The disadvantage is that a freely floating
system makes a country vulnerable to economic conditions in other countries.
Answer (B) is incorrect
Forcing imports to be cheaper and exports more expensive can be accomplished with a
fixed exchange rate system.
Answer (C) is incorrect
Freely floating exchange rates impose no constraints on the domestic economy
Answer (D) is incorrect
A freely floating exchange rate system does not eliminate transaction risk (which must
be hedged).

Question #92: An increase in sales resulting from an increased cash discount for prompt
payment would be expected to
Correct Answer: C. A decrease in the cash conversion cycle.
A. An increase in the operating cycle.
B. An increase in the average collection period.
C. A decrease in the cash conversion cycle.
D. A decrease in purchase discounts taken
Explanation:
Answer (A) is incorrect
The operating cycle would decrease since the average time from cash disbursement to
cash realization would be shorter
Answer (B) is incorrect
The average collection period would decrease since the average time from cash
disbursement to cash realization would be shorter
Answer (C) is correct.
If the cause of increased sales is an increase in the cash discount; it can be inferred that
the additional customers would pay during the discount period. Thus, cash would be
collected more quickly than previously and the cash conversion cycle would be
shortened.
Answer (D) is incorrect
More customers will take discounts

Question #93: A company has a strong internal control structure in its accounting
department. It has a high degree of duty segregation, regular reconciliations, strict reviews,
and comprehensive internal audits. A disgruntled fixed assets accountant has been
contemplating the embezzlement of cash receipts processed by the accounts receivable
department. The accountant plans to use these funds to sustain his gambling problem. Using
the Fraud Triangle model, what is the best assessment of fraud risk for the company's
situation?
Correct Answer: D. Medium, because opportunity is absent.
A. High, because pressure, opportunity; and rationalization are all present.
B. Low, because both opportunity and rationalization are absent.
C. Medium, because rationalization is absent.
D. Medium, because opportunity is absent.
Explanation:
A. The three sides of the Fraud Triangle are pressure, opportunity, and rationalization. All
three conditions need to be present for an employee to commit a fraudulent act against an
employer. Pressure is present, because the employee has a gambling problem.
Rationalization is present, because the employee is disgruntled and can rationalize the
fraudulent act. However, because the company has a strong internal control structure in
its accounting department, the disgruntled employee will not have much opportunity to
commit a fraudulent act.
B. The three sides of the Fraud Triangle are pressure, opportunity, and rationalization. All
three conditions need to be present for an employee to commit a fraudulent act against an
employer. Pressure is present, because the employee has a gambling problem. Because
the company has a strong internal control structure in its accounting department, the
disgruntled employee will not have much opportunity to commit a fraudulent act, so
opportunity is absent. However, because the employee is disgruntled, the employee can
rationalize the fraudulent act, so rationalization is not absent.
C. The three sides of the Fraud Triangle are pressure, opportunity, and rationalization. All
three conditions need to be present for an employee to commit a fraudulent act against an
employer. Pressure is present, because the employee has a gambling problem. Because
the company has a strong internal control structure in its accounting department, the
disgruntled employee will not have much opportunity to commit a fraudulent act.
However, rationalization is not absent. Because the employee is disgruntled, the
employee can rationalize the fraudulent act.
D. The three sides of the Fraud Triangle are pressure, opportunity, and rationalization. All
three conditions need to be present for an employee to commit a fraudulent act against an
employer. Pressure is present, because the employee has a gambling problem.
Rationalization is present, because the employee is disgruntled and can rationalize the
fraudulent act. However, because the company has a strong internal control structure in
its accounting department; the disgruntled employee will not have much opportunity to
commit a fraudulent act. The fraud risk for the company's situation is not eliminated, but
it is lowered by the strong internal control structure.

Question #94: The risk that securities cannot be sold at a reasonable price on short notice is
called
Correct Answer: D. Liquidity risk.
A. Default risk.
B. Interest-rate risk. •
C. Purchasing-power risk. •
D. Liquidity risk.
Explanation:
Answer (A) is incorrect.
Default risk is the risk that a borrower will not pay the interest or principal on a loan.
Answer (B) is incorrect.
Interest-rate risk is the risk to which investors are exposed because of changing interest rates.
Answer (C) is incorrect.
Purchasing-power risk is the risk that inflation will reduce the purchasing power of a given
sum of money.
Answer (D) is correct.
An asset is liquid if it can be converted to cash on short notice. Liquidity (marketability) risk
is the risk that assets cannot be sold at a reasonable price on short notice. If an asset is not
liquid, investors will require a higher return than for a liquid asset. The difference is the
liquidity premium.

Question #95: Which one of the following procedures would most likely help managers
identify errors in their capital budgeting decisions?
Correct Answer: C. Post-audits.
A. Value engineering.
B. Scenario analysis.
C. Post-audits.
D. Monte Carlo simulations.
Explanation:
Answer (A) is incorrect.
Value engineering is a means of reaching targeted cost levels. It is a systematic approach to
assessing all aspects of the value chain cost buildup for a product.
Answer (B) is incorrect.
Scenario analysis is a process of analyzing possible future results based upon various
assumptions.
Answer (C) is correct.
Post-investment audits should be conducted to serve as a control mechanism and to deter
managers from proposing unprofitable investments. Actual-to-expected cash flow
comparisons should be made; and unfavorable variances should be explained. Individuals
who supplied unrealistic estimates should have to explain differences.
Answer (D) is incorrect.
Monte Carlo simulations are used to account for an element of randomness. The errors in
managers' capital budgeting decisions are under the control of the managers and thus are not
random.

Question #96: Fact Pattern: The data presented below show actual figures for selected
accounts of McKeon Company for the fiscal year ended May 31, Year 1, and selected budget
figures for the Year 2 fiscal year. McKeon's controller is in the process of reviewing the Year
2 budget and calculating some key ratios based on the budget. McKeon Company monitors
yield or return ratios using the average financial position of the company. (Round all
calculations to three decimal places if necessary.)
5/31/ 5/31/
year2 year1
current assets $210,000 $180,000
noncurrent liabilities 275,000 255,000
current liabilities 78,000 85,000
long term debt 75,000 30,000
common stock ($30 par
value) 300,000 300,000
retained earnings 32,000 20,000

Question: McKeon Company's total asset turnover ratio for Year 2 is


Correct Answer: B. 0.761
A. 0.805
B. 0.761
C. 0.722
D. 0.348
Explanation:
Answer (A) is incorrect.
The total asset turnover ratio equals sales divided by average total assets.
Answer (B) is correct.
Total asset turnover ratio is equal to $350,000 sales divided by average total assets. The
amount of average total assets is equal to the average of beginning total assets of $435,000
($180,000 current assets + $255,000 noncurrent assets) and ending total assets of $485,000
($210,000 current assets + $275,000 noncurrent assets). The total asset turnover ratio is
therefore equal to .761 ($350,000 ÷ $460,000).
Answer (C) is incorrect.
The total asset turnover ratio equals sales divided by average total assets.
Answer (D) is incorrect.
The total asset turnover ratio equals sales divided by average total assets.

Question #97: Please see the question below:


The Liabilities and Shareholders' Equity section of Mica Corporation's Statement of
Financial Position is shown below.
January Decembe
1 r 31
accounts payable $32,000 $84,000
accrued liabilities 14,000 11,000
7% bonds payable 95,000 77,000
common stock ($10 par value) 300,000 300,000
reserve for bond retirement 12,000 28,000
total liabilities and shareholders' $608,00
equity 0 $706,000

Mica’s debt/equity ratio is


Correct Answer: D. 32.2%.
A. 25.1%.
B. 33.9%.
C. 25.6%.
D. 32.2%.
Explanation:
A. This is not the correct answer
B. This is total liabilities divided by common stock + retained earnings. The reserve for
bond retirement is an appropriation of retained earnings and is an equity account; as well.
C. This is December 31 long-term debt ($77,000) divided by common stock ($300,000). The
debt/equity ratio is total liabilities divided by total equity.
D. This is total liabilities divided by common stock + retained earnings. The reserve for
bond retirement is an appropriation of retained earnings and is an equity account; as well.

This is December 31 long-term debt ($77,000) divided by common stock ($300,000). The
debt/equity ratio is total liabilities divided by total equity.
The debt/equity ratio is total liabilities divided by total equity. When calculating a ratio
that consists of all balance sheet amounts, such as the debt/equity ratio, we do not use
averages. We use the most current period end figures that are available. Here, that is the
December 31 balances.
The reserve for bond retirement account is an appropriation of retained earnings. All
retained earnings start out classified as Unappropriated Retained Earnings. The term
"unappropriated" simply means that the retained earnings are available to be distributed
to shareholders in the form of dividends. Occasionally, however; a company does not
want to distribute a portion of its retained earnings, and this can be communicated to the
shareholders (and potential shareholders) through the process of appropriating a portion
of retained earnings.
The appropriation of retained earnings is done by the board of directors; and there is only
one result of this action. This action informs the readers of the financial statements that
some of the retained earnings are not available for distribution. There is no legal meaning
to this, no time period involved (the board can de-appropriate the retained earnings at any
time), and there are no involved accounting processes to this.
There are many reasons that a company may decide to appropriate retained earnings.
Among them are: creating a reserve to build a plant, meeting the requirements of a bond;
or simply providing for the future. Remember, however; that the only effect of
appropriating retained earnings is to let shareholders know that the appropriated retained
earnings will not be distributed as a dividend.
So the reserve for bond retirement is a line item in the equity section of the balance sheet
and it is treated just like retained earnings in the calculation of the debt/equity ratio.
As of December 31, Total Liabilities is $172,000 ($84,000 + $11,000 + $77,000) and
Total Equity is $534,000 ($300,000 + $28,000 + $206,000). The debt/equity ratio is
therefore $172,000 ÷ $534,000, which equals 32.2%.

Question #98: Selected data from Sheridan Corporation's year end financial statements are
presented below. The difference between average and ending inventory is immaterial.
Current ratio2.0 Quick ratio1.5 Current liabilities$120,000 Inventory turnover (based on cost
of goods sold)8 times Gross profit margin40%
Sheridan's net sales for the year were
Correct Answer: C. $800,000
A. $1,200,000
B. $240,000
C. $800,000
D. $480,000
Explanation:
A. This is the cost of goods sold divided by the gross profit margin. To calculate sales in this
way, cost of goods sold needs to be divided by (1 - the gross profit margin).
B. This answer is the value for current assets. See the correct answer for a complete
explanation. The purpose of this question is to find out whether you know how several
ratios are calculated, what they mean and how to use them. The ratios needed to answer
this question are: (1) current ratio and quick ratio; and the fact that the primary difference
between them is inventory in the numerator; (2) inventory turnover ratio; and (3) gross
profit and gross profit margin. The current ratio is current assets , current liabilities. We
know that current liabilities are $120,000 and the current ratio is 2. Therefore; current
assets must be $240,000 ($120,000 x 2).
C. The primary difference between the numerators of the current ratio and the quick ratio is
inventory. (Note that pre-paids and some other minor current asset items are excluded
also from the numerator of the quick ratio when they are known. But since no
information that would enable us to calculate those is given in this problem, we must
ignore those for this problem.) Therefore, for this problem, the quick ratio is (current
assets - inventory), $120,000, and that is given as 1.5. So (current assets - inventory) must
be equal to $180,000 ($120,000 x 1.5). Therefore; inventory is $240,000 - $180,000, or
$60,000. The inventory turnover ratio is COGS. Inventory, and we are told that is 8. If
COGS, $60,000 = 8; then COGS = $480,000 ($60,000 x 8). The gross profit margin is
40%. Since Sales - COGS = Gross Profit; if gross profit is 40% of sales, then COGS must
be 60% of sales. Thus, COGS, 0.60 would equal sales. Since we now know that COGS is
$480,000, we can calculate sales: $480,000, 0.60 = $800,000.
D. This answer is the value for cost of goods sold. See the correct answer for a complete
explanation.

Question #99: Fact Pattern: Henderson, Inc., has purchased a new fleet of trucks to deliver its
merchandise. The trucks have a useful life of 8 years and cost a total of $500,000. Henderson
expects its net increase in after-tax cash flow to be $150,000 in Year 1, $175,000 in Year 2,
$125,000 in Year 3, and $100,000 in each of the remaining years.
Question: Based on a 6% annual interest rate, what is the discounted payback period for
Henderson's fleet of trucks?
Correct Answer: C. 4.25 years.
A. 3.5 years.
B. 3.98 years.
C. 4.25 years.
D. 5.0 years.
Explanation:
Answer (A) is incorrect.
This number of years is the undiscounted payback period.

Answer (B) is incorrect.


This number of years results from not discounting the cash flows in the fourth year.
Answer (C) is correct.
The discounted payback period for an investment, assuming a 6% discount, can be found by
accumulating each year's discounted net cash flows until the initial investment is recovered.
$150,000 x .94339 = $141,508.50
175,000 x .88999 = 155,748.25
125,000 x .83962 = 104,952.50
100,000 x .79209 = 79,209.00
$481,418.25
Thus, the answer is something greater than four years. After four years, an additional
$18,581.75 ($500,000 — $481,418.25) is needed. The calculation for the fifth year is
$74,726 ($100,000 x .74726). Consequently, the discounted payback period is approximately
4.25 years [4 + ($18,581.75 ÷ $74,726)].
Answer (D) is incorrect.
The full fifth year is not necessary.

Question #100: Fact Pattern: Kell, Inc., is analyzing an investment for a new product
expected to have annual sales of 100,000 units for the next 5 years and then be discontinued.
New equipment will be purchased for $1,200,000 and cost $300,000 to install. The
equipment will be depreciated on a straight-line basis over 5 years for financial reporting
purposes and 3 years for tax purposes. At the end of the fifth year, it will cost $100,000 to
remove the equipment, which can be sold for $300,000. Additional working capital of
$400,000 will be required immediately and needed for the life of the product. The product
will sell for $80, with direct labor and material costs of $65 per unit. Annual indirect costs
will increase by $500,000. Kell's effective tax rate is 40%.
Question: In a capital budgeting analysis, what is the expected cash flow at time = 3 (third
year of operation) that Kell should use to compute the net present value?
Correct Answer: D. $800,000
A. $300,000
B. $720,000
C. $760,000
D. $800,000
Explanation:
Answer (A) is incorrect.
The amount of $300,000 is only the annual depreciation expense for financial reporting
purposes.
Answer (B) is incorrect.
The amount of $720,000 results from using 5 years for depreciating the new equipment
rather than 3.
Answer (C) is incorrect.
The amount of $760,000 results from failing to include installation cost in the full cost of the
new equipment.
Answer (D) is correct.
The estimated incremental after-tax operating cash flows for each year of a capital project
consist of two components, the after-tax cash inflows from operations and the difference in
depreciation tax shields between the old and new equipment. The first of these for Kell can
be calculated as follows:
Projected annual contribution margin (100,000 units x $15) $1,500,000
Less: annual increase in indirect costs (500,000)
Net annual cash inflow $1,000,000
Less: income tax expense ($1,000,000 x 40%) (400,000)
After-tax cash inflow from operations $ 600,000
Since no old equipment was removed, the total depreciation tax shield is simply that derived
from the new equipment:
Full cost of new equipment ($1,200,000 + $300,000) $1,500,000
Divided by: estimated useful life for tax purposes ÷ 3
Annual depreciation expense $ 500,000
Times: tax rate x 40%
Depreciation tax shield $ 200,000
The annual after-tax cash flows for this project would thus amount to $800,000 ($600,000 ÷
$200,000).
MOCK TEST 10

Question #1: The key components of enterprise risk management (ERM) include all of the
following except:
Correct Answer: C. Improved shareholder value.
A. Assess risks.
B. Monitor risk.
C. Improved shareholder value.
D. Set strategy and objectives.
Explanation:
Improve shareholder value is not a key component of ERM although it is often a direct
benefit of effective ERM implementations. The basic components found in most ERM
frameworks include: set strategy and objectives, identify risks, assess risks, treat risks,
control risks, and communicate and monitor.

Question #2: The Stewart Co. uses the Economic Order Quantity (EOQ) model for inventory
management. A decrease in which one of the following variables would increase the EOQ?
Correct Answer: B. Carrying costs.
A. Quantity demanded.
B. Carrying costs.
C. Annual sales.
D. Cost per order.
Explanation:
The EOQ is calculated by taking the square root of the annual sales multiplied by the cost per
order divided by the carrying costs per unit. Since carrying costs is in the denominator of this
equation, it has an inverse relationship to EOQ. Therefore, a decrease in the carrying costs
per unit would increase the EOQ.
Question #3: The following forecasts have been prepared for a new investment of $20
million with an 8-year life:
pessimisti expecte optimisti
c d c
market size 60,000 90,000 140,000
Market share,
% 25 30 35
unit price $750 $800 $875
unit variable
cost $500 $400 $350
fixed cost,
millions $7 $4 $3.5

Assume that Oxford employs straight-line depreciation, and that they are taxed at 35%.
Assuming an opportunity cost of capital of 14% what is the NPV of this project, based an
expected outcomes?
Correct Answer: B. $4,563,505
A. $2,626,415
B. $4,563,505
C. $6,722,109
D. $8,055,722
Explanation:
Answer (A) is incorrect.
The amount of $2,626,415 used the wrong discount factor.
Answer (B) is correct.
The first step is to calculate the annual cash flows from the project for the base case (the
expected values). These may be calculated as shown:
Value ($
in
millions
Description How calculated )
1. Revenues 90,000 × 0.30 × $800 21.600
2. Variable
cost 90,000 × 0.30 × $400 10.800
3. Fixed cost $4,000,000 4.000
4.
Depreciation $20,000,000 ÷ 8 2.500
5. Pretax Item 1- (items 2 + 3 +
profit 4) 4.300
6. Tax Item 5 × 0.35 1.505
7. Net profit Item 5- Item 6 2.795
8. Net cash
flow Item 7 ÷ item 4 5.295

This level of cash flow occurs for each of the 8 years of the project. The present value of an 8-
year, $1 annuity is 4.639 at 14%. The NPV of the project is therefore given by:
NPV = $5,295,000 x 4.639 - $20,000,000 = $4,563,505
Answer (C) is incorrect
It failed to consider depreciation.
Answer (D) is incorrect.
It failed to consider depreciation and other fixed costs.

Question #4: Allred Company sells its single product for $30 per unit. The contribution
margin ratio is 45%, and fixed costs are $10,000 per month. Allred has an effective income
tax rate of 40%. If Allred sells 1,000 units in the current month. Allred's variable expenses
would be:
Correct Answer: A. $16.500.
A. $16,500.
B. $12,000.
C. $9,900.
D. $13,500.
Explanation:
Variable expenses are calculated as:
Variable expenses = (1- contribution margin ratio) (sales amount)
Sales amount = ($30) (1,000 units) = $30.000
Variable expenses = (1- 0.45) ($30,000)
Variable expenses = (0.55) (530.000) = $16.500.
Question #5: A bank has received loan applications from three companies in the plastics
manufacturing business and currently has the funds to grant only one of these requests.
Specific data shown below has been selected from these applications for review and
comparison with industry averages.
Springfliel Herndo Industr
d Reston n y
Total sales
(millions) $4.27 $3.91 $4.86 $4.30
Net profit margin 9.55% 9.85% 10.05% 9.65%
Current ratio 1.82 2.02 1.96 1.95
Return on assets 12.0% 12.6% 11.4% 12.4%
Debt/equity ratio 52.5% 44.6% 49.6% 48.3%
Financial
leverage 1.30 1.02 1.56 1.33

Based on the information above, select the strategy that should be the most beneficial to Grand
Savings.
Correct Answer: C. Grant the loan to R. as both the debt/equity ratio and degree of financial
leverage are below the industry average.
A. The bank should not grant any loans, as none of these companies represents a good credit
risk
B. Grant the loan to S. as all the company's data approximate the industry average.
C. Grant the loan to R. as both the debt/equity ratio and degree of financial leverage are
below the industry average.
D. Grant the loan to H. as the company has the highest net profit margin and degree of
financial leverage.
Explanation:
Answer (A) is incorrect.
R is a good credit risk
Answer (B) is incorrect.
Debt makes up more than half of S's capital structure; "approximating industry averages' is
meaningless when just a few percentage points can mean the difference between a good
credit risk and a poor one.
Answer (C) is correct.
The bank's primary concern is the customer's ability to pay a loan back Crucial in deciding
the likelihood of payback is how much of the customer's capital structure is made up of debt
currently, that is, before the loan is made. R's is well below the industry average (a few
percentage points can mean the difference between a good credit risk and a poor one) and is
the lowest of the three potential customers. Also. R is clearly the least leveraged of the three
by far, as revealed by its low degree of financial leverage.
Answer (D) is incorrect.
While a high profit margin may be indicative of the ability to pay back a loan, a high degree
of financial leverage indicates the opposite, and H's is well above the industry average.

Question #6: In discounted cash flow techniques, which one of the following alternatives best
reflects the items to be incorporated in the initial net cash investment?
Correct Answer: D. Yes (Capitalized expenditures (e.g., shipping costs)). (Changes in net
working capital) Yes. Yes (Net proceeds from sale of old asset in a replacement decision).
(Impact of spontaneous changes in current liabilities ) Yes
A. No (Capitalized expenditures (e.g., shipping costs)). (Changes in networking capital )
Yes. Yes (Net proceeds from sale of old asset in a replacement decision). ( Impact of
spontaneous changes in current liabilities ) Yes
B. Yes (Capitalized expenditures (e.g., shipping costs)). (Changes in net working capital )
No. No (Net proceeds from sale of old asset in a replacement decision). ( Impact of
spontaneous changes in current liabilities )No
C. No (Capitalized expenditures (e.g., shipping costs)). (Changes in networking capital )
Yes. No (Net proceeds from sale of old asset in a replacement decision). ( Impact of
spontaneous changes in current liabilities ) No
D. Yes (Capitalized expenditures (e.g., shipping costs)). (Changes in net working capital )
Yes. Yes (Net proceeds from sale of old asset in a replacement decision). ( Impact of
spontaneous changes in current liabilities ) Yes
Explanation:
Answer (A) is incorrect.
Capitalized expenditures must be considered.
Answer (B) is incorrect.
Changes in working capital and proceeds from the sale of existing assets must be considered.
Answer (C) is incorrect.
Changes in working capital are not the only items that must be considered.
Answer (D) is correct.
All four of the items listed must be incorporated in a DCF analysis of a capital project.
Question #7: If a firm has a portfolio of six different stocks, the expected return for the
portfolio is the weighted average of the expected returns for the:
Correct Answer: D. six stocks.
A. six stocks multiplied by an average of portfolio variance.
B. six stocks multiplied by the beta of the portfolio.
C. six stocks multiplied by the portfolio standard deviation.
D. six stocks.
Explanation:
By definition, a portfolio rate of return is the weighted average of the expected returns of all
the investments composing that portfolio. The weights represent the proportions of each item
in the portfolio and sum to 100%.

Question #8: A major impact of the Foreign Corrupt Practices Act of 1977 is that registrants
subject to the Securities Exchange Act of 1934 are now required to
Correct Answer: A. Keep records that reflect the transactions and dispositions of assets and
to maintain a system of internal accounting controls.
A. Keep records that reflect the transactions and dispositions of assets and to maintain a
system of internal accounting controls.
B. Provide access to records by authorized agencies of the federal government.
C. Prepare financial statements in accord with international accounting standards.
D. Produce full, fair, and accurate periodic reports on foreign commerce and/or foreign
political party affiliations.
Explanation:
Answer (A) is correct
The main purpose of the Foreign Corrupt Practices Act of 1977 is to prevent bribery by firms
that do business in foreign countries. A major ramification is that it requires all companies
that must register with the SEC under the Securities Exchange Act of 1934 to maintain
adequate accounting records and a system of internal accounting control.
Answer (B) is incorrect.
Authorized agents of the federal government already have access to records of SEC
registrants.
Answer (C) is incorrect
Although some international accounting standards have been promulgated, they are
incomplete and have not gained widespread acceptance.
Answer (D) is incorrect.
There are no requirements for providing periodic reports on foreign commerce or foreign
political party affiliations.

Question #9: Pazer Inc. produces portable televisions. Pazer's product manager proposes to
increase the cost structure by adding voice-activated volume/channel controls to the
television and also adding three additional repair personnel to deal with products returned
due to defects. Are these costs value-added or non-value-added?
Correct Answer: D. Cost of Voice-Activated Controls: Value-added: Cost of Additional
Repair Personnel: Non-value-added.
A. Cost of Voice-Activated Controls: Non-value-added: Cost of Additional Repair
Personnel: Non-value-added.
B. Cost of Voice-Activated Controls: Non-value-added: Cost of Additional Repair
Personnel: Value-added.
C. Cost of Voice-Activated Controls: Value-added: Cost of Additional Repair Personnel:
Value-added.
D. Cost of Voice-Activated Controls: Value-added: Cost of Additional Repair Personnel:
Non-value-added.
Explanation:
If Pazer adds voice-activated controls, the actual and perceived value of portable televisions
for consumers would increase. In contrast, the company's decision to add additional repair
personnel will not impact customer value or usefulness received from the product.

Question #10: Osgood Products has announced that it plans to finance future investments so
that the firm will achieve an optimum capital structure. Which one of the following corporate
objectives is consistent with this announcement?
Correct Answer: D. Maximize the net value of the firm.
A. Minimize the cost of debt.
B. Maximize earnings per share.
C. Minimize the cost of equity.
D. Maximize the net value of the firm.
Explanation:
The goal of the firm is to maximize shareholder wealth. Shareholder wealth is the value of
the firm. It is also the present value of the firm's future cash flows at the marginal weighted-
average cost of capital (Ka). The smaller Ka is, the higher the present value. The optimal
capital structure for the firm is the one that minimizes Ka, and, therefore, maximizes the
value of the firm.

Question #11: Given an acid test ratio of 2.0, current assets of $5,000, and inventory of
$2,000, the value of current liabilities is
Correct Answer: A. $1,500:
A. $1,500
B. $2,500
C. $3,500
D. $6,000
Explanation:
Answer (A) is correct
The acid test, or quick ratio equals the quick assets (cash. marketable securities. and accounts
receivable) divided by current liabilities. Current assets equal the quick assets plus inventory
and prepaid expenses. (This question assumes that the entity has no prepaid expenses.) Given
current assets of $5,000, inventory of $2,000, and no prepaid expenses, the quick assets must
be $3,000. Because the acid test ratio is 2.0, the quick assets are double the current liabilities.
Current liabilities therefore are equal to $1,500 ($3,000 quick assets ÷ 2.0).
Answer (B) is incorrect.
Dividing the current assets by 2.0 results in $2,500. Current assets includes inventory, which
should not be included in the calculation of the acid test ratio.
Answer (C) is incorrect
Adding inventory to current assets rather than subtracting it results in $3,500.
Answer (D) is incorrect.
Multiplying the quick assets by 2 instead of dividing by 2 results in $6,000.
Question #12: Risks that an organization may face related to short-term and long-term debt
and equity decisions are commonly called:
Correct Answer: C. Financial risks.
A. Operational risks.
B. Hazard risks.
C. Financial risks.
D. Strategic risks.
Explanation:
Hazard risks relate to natural disasters such as storms, floods, hurricanes, blizzards,
earthquakes, and volcanoes. Financial risks are caused by debt/equity decisions related to
financing the business. They include liquidity (short-term bill paying) and solvency (long-
term bill paying). Operational risk relates to the relationship of fixed and variable costs in the
organization's cost structure as well as the following: internal process failures, system
failures, personnel, and legal and compliance risks.

Question #13: A company has $80 million in current assets, comprised of $30 million in
inventory and $50 million in cash and marketable securities. The company's current liabilities
total $50 million. If the company purchases an additional $10 million in inventory with $10
million in cash, the effect of this transaction on the company would be to
Correct Answer: B. Decrease the quick ratio while the current ratio remains unchanged.
A. Decrease the current ratio and increase the quick ratio.
B. Decrease the quick ratio while the current ratio remains unchanged.
C. Leave both the current ratio and the quick ratio unchanged.
D. Decrease the current ratio and decrease the quick ratio.
Explanation:
Answer (A) is incorrect.
Since the decrease in cash is offset by an increase in inventory, the current ratio remains
unchanged. Also, the decrease in cash decreases the quick ratio.
Answer (B) is correct.
The quick ratio decreases from 1.00 ($50 million ÷ $50 million) to .80 ($40 million ÷ $50
million) due to the $10 million decrease in cash. The current ratio remains unchanged
because the $10 million decrease in cash is offset by the $10 million increase in inventory.
Answer (C) is incorrect.
The decrease in cash lowers the numerator in the quick ratio, which decreases the quick ratio.
Answer (D) is incorrect.
Since the decrease in cash is offset by an increase in inventory, the current ratio remains
unchanged.

Question #14: IMA's Statement of Ethical Professional Practice includes a competence


standard, which requires an IMA member to
Correct Answer: B. Develop his or her professional proficiency on a continual basis.
A. Report information, whether favorable or unfavorable.
B. Develop his or her professional proficiency on a continual basis.
C. Discuss ethical conflicts and possible courses of action with an unbiased counselor.
D. Discuss, with subordinates, their responsibilities regarding the disclosure or information
about the firm.
Explanation:
Answer (A) is incorrect.
The credibility standard requires an IMA member to 'communicate information fairly and
objectively'
Answer (B) is correct.
One of the responsibilities of an IMA member under the competence standard is to “maintain
an appropriate level of professional expertise by continually developing knowledge and
skills”
Answer (C) is incorrect
One of the suggestions from the "Resolution of Ethical Conflict' section is to "clarify relevant
ethical issues by initiating a confidential discussion with an IMA Ethics Counselor or other
impartial advisor to obtain a better understanding of possible courses of action'
Answer (D) is incorrect.
The confidentiality standard requires an IMA member to "inform all relevant parties
regarding appropriate use of confidential information. Monitor subordinates' activities to
ensure compliance”.
Question #15: Fact Pattern: Depoole Company is a manufacturer of industrial products that
uses a calendar year for financial reporting purposes. Assume that total quick assets exceeded
total current liabilities both before and after the transaction described. Further assume that
Depoole has positive profits during the year and a credit balance throughout the year in its
retained earnings account.
Question: Depoole's collection of a current accounts receivable of $29.000 will
Correct Answer: D. Not affect the current or quick ratios.
A. Increase the current ratio.
B. Decrease the current ratio and the quick ratio.
C. Increase the quick ratio.
D. Not affect the current or quick ratios.
Explanation:
Answer (A) is incorrect.
Collecting current accounts receivable does not create a net change in current assets, quick
assets, or current liabilities, which means the current and quick ratios are not changed.
Answer (B) is incorrect.
Collecting current accounts receivable does not create a net change in current assets, quick
assets, or current liabilities, which means the current and quick ratios are not changed.
Answer (C) is incorrect
Collecting current accounts receivable does not create a net change in current assets, quick
assets, or current liabilities, which means the current and quick ratios are not changed.
Answer (D) is correct.
Collecting current accounts receivable has no effect on either the current ratio or the quick
ratio because assets (both current and quick) are reduced for the collection of receivables and
increased by the same amount for the receipt of cash. Current liabilities are unchanged by the
transaction.

Question #16: The chief financial officer of Smith Glass Inc. follows the policy of matching
the maturity of assets with the maturity of financing. The implications of this policy include
all of the following except that:
Correct Answer: D. cash, receivables, and inventory should be financed with long-term debt
or equity.
A. the seasonal expansion of cash, receivables, and inventory should be financed by short-
term debt such as vendor payables and bank debt.
B. the minimum level of cash, receivables, and inventory required to stay in business can be
considered permanent and financed with long-term debt or equity.
C. long-term assets, like plant and equipment, should be financed with long-term debt or
equity.
D. Cash, receivables, and inventory should be financed with long-term debt or equity.
Explanation:
Using the maturity matching or hedging approach to financing, a firm offsets each asset with
a financing instrument of the same approximate maturity. Not all cash, receivables, and
inventory would be financed with long-term debt or equity—only the permanent
components.

Question #17: Breakeven quantity is defined as the volume of output at which revenues are
equal to:
Correct Answer: A. total costs.
A. total costs.
B. marginal costs.
C. variable costs.
D. fixed costs.
Explanation:
Breakeven quantity is defined as the volume of output at which revenues are equal to total
costs. Therefore, profits = zero.

Question #18: Which of the following is not a key step in the Risk Management Process
(RMP)?
Correct Answer: A. Evaluate capital reserves.
A. Evaluate capital reserves.
B. Evaluate the risk exposure.
C. Monitor the risk exposure.
D. Determine the company's tolerance for risk
Explanation:
The four common steps in the RMP are: 1. Determine the company's tolerance for risk. 2.
Evaluate the risk exposure. 3. Implement an appropriate risk management strategy, and 4.
Monitor the risk exposure and the strategy.

Question #19: Fennel Products is using cost-based pricing to determine the selling price for
its new product based on the following information.
Annual
volume 25,000 units
$700,000 per
Fixed costs year
Variable costs $200 per unit
Plant
investment $3,000,000
Working
capital $1,000,000
Effective tax
rate 40%

The target price that Fennell needs to set for the new product to achieve a 15% after-tax
return on investment (ROI) would be:
Correct Answer: C. $268.
A. $228.
B. $258.
C. $268.
D. $238.
Explanation:
The target price (p) is computed by using the following formula:
(Total sales - Total variable costs - Total fixed costs)(1 - tax rate) = (Target R0I) (investment)
Total sales = (volume) (target price) = (25,000) (p)
Total variable costs = (volume) (variable cost per unit) = (25,000) ($200) = $5,000,000
Total fixed costs = $700,000
Investment includes both plant and working capital = $3,000,000 + $1,000,000 = $4,000,000
(25,000p - $5,000,000 - $700,000) (1 - 0.4) = (0.15) ($4,000,000)
(25,000p - $5,700,000) (0.6) = $600,000
15,000p - $3,420,000 = $600,000
15,000p = $4,020,000
p = $268.

Question #20: Country As currency would tend to appreciate relative to Country B's currency
when:
Correct Answer: D. Country A has a slower rate of growth in income that causes its imports
to lag behind its exports.
A. Country B switches to a more restrictive monetary policy.
B. Country B has real interest rates that are greater than real interest rates in Country A
C. Country A has a higher rate of inflation than Country B.
D. Country A has a slower rate of growth in income that causes its imports to lag behind its
exports.
Explanation:
The increase in exports would increase the demand for Country As currency causing its value
to appreciate.

Question #21: A small equipment manufacturer sells 1,500 units per year. Average
receivables turnover is 10.5. Assuming a 365-day year, the firm's average collection period is
closest to which of the following?
Correct Answer: B. 35 days.
A. 24 days.
B. 35 days.
C. 43 days.
D. 55 days.
Explanation:
Average collection period refers to the average time it takes a firm to collect its accounts
receivable. The following ratio is used to determine the average collection period:
365 days 365 days
Average Collection Period = = =34.76=35 days .
Accounts receivable turnover 10.5
Question #22: A chief financial officer has been tracking the activities of the company's nearest
competitor for several years. Among other trends, the CFO has noticed that this competitor is
able to take advantage of new technology and bring new products to market more quickly than
the CFO's company. In order to determine the reason for this, the CFO has been reviewing the
following data regarding the two companies:
Compan Competito
y r
Accounts receivable
turnover 6.85 7.35
Return on assets 15.34 14.74
Times interrest earned 15.65 12.45
Current ratio 2.11 1.23
Debt/equity ratio 42.16 55.83
Degree of financial
leverage 1.06 1.81
Price/earnings ratio 26.56 26.15

On the basis of this information, which one of the following is the best initial strategy for the
CFO to follow in attempting to improve the flexibility of the company?
Correct Answer: C. Seek additional sources of outside financing for new product introductions.
A. Seek cost cutting measures that would increase the company's profitability.
B. Investigate ways to improve asset efficiency and turnover times to improve liquidity.
C. Seek additional sources of outside financing for new product introductions.
D. Increase the company's investment in short-term securities to increase the current ratio.
Explanation:
Answer (A) is incorrect.
Cutting costs makes it harder to take advantage of new opportunities or to innovate. Cost cutting
is a last resort and the company's return on assets is already better than its competitors.
Answer (B) is incorrect.
The receivables turnover is not much different than that of the competitor.
Answer (C) is correct. The company's times interest earned, debt/equity ratio, and degree of
financial leverage all reveal that the company is less leveraged than its competitor. The two
firms' price-earnings ratios are comparable, so the company should be able to raise new capital
fairly easily, either debt or equity. Thus, the company should seek additional sources of outside
financing for new product introductions.
Answer (D) is incorrect.
Increasing investment in short-term securities would not change the current ratio.

Question #23: Clauson Inc. grants credit terms of 1/15, net 30 and projects gross sales for the
year of $2,000,000. The credit manager estimates that 40% of customers pay on the 15th day.
40% of the 30th day and 20% on the 45th day. Assuming uniform sales and a 365-day year, what
is the projected amount of overdue receivables? (Round answer to the nearest hundred.)
Correct Answer: C. $49,300.
A. $400,000.
B. $116,700.
C. $49,300.
D. $83,300.
Explanation:
The projected amount of overdue receivables can be calculated as:
Projected amount of overdue receivables = (45 days / 365 days) (20% of sales)
Projected amount of overdue receivables = (45/365) (0.2) ($2,000,000) = $49,315
Rounding to the nearest hundred, the Projected amount of overdue receivables = $49,300.

Question #24: Maple Motors buys axles in order to produce automobiles. Maple carries an
average credit balance of $25,000,000 with its axle supplier. The axle supplier provides credit
terms of 1/10 net 25. The nominal annual cost of Maple not taking the trade discount is closest to
which one of the following? Assume a 365-day year.
Correct Answer: B. 24.6%.
A. 24.0%.
B. 24.6%.
C. 14.4%.
D. 14.5%.
Explanation:
Nominal annual cost = [% discount / (100% - %discount)] x [365 / (payment date - discount
period)]
Nominal annual cost = (1% / 99%) x (365 / 15) = 24.6%.
Question #25: All of the following are potential benefits of risk management except:
Correct Answer: D. Decreased inherent risk.
A. Reduced cost of capital.
B. More effective resource allocations.
C. Increased understanding of entity objectives.
D. Decreased inherent risk.
Explanation:
Inherent risk is the risk that exists before any controls are implemented to mitigate such risk It is
the probability of a threat occurring.
Question #26: Which one of the following statements concerning the effects of leverage on
earnings before interest and taxes (EBIT) and earnings per share (EPS) is correct?
Correct Answer: A. For a firm using debt financing, a decrease in EBIT will result in a
proportionally larger decrease in EPS.:
A. For a firm using debt financing, a decrease in EBIT will result in a proportionally larger
decrease in EPS.
B. A decrease in the financial leverage of a firm will increase the beta value of the firm.
C. If Firm A has a higher degree of operating leverage than Firm B and Firm A offsets this
by using less financial leverage, then both firms will have the same variability in EBIT.
D. Financial leverage affects both EPS and EBIT, while operating leverage only affects
EBIT.
Explanation:
Answer (A) is correct
Debt financing increases a firm's financial leverage, that is, the high fixed charges that
accompany the legal obligation to pay interest to debt holders makes the firm riskier. These high
fixed costs make profitable periods more profitable and unprofitable periods worse. Thus, a
given decrease in EBIT will result in a proportionally larger decrease in EPS.
Answer (B) is incorrect.
The beta value of a firm's stock is dependent on its relationship to the rest of the market, not the
firm's financial leverage.
Answer (C) is incorrect
Financial leverage (i.e., interest expense) impacts the income statement after EBIT.
Answer (D) is incorrect.
Any income statement item that affects EBIT necessarily affects EPS as well.
Question #27: Which of the following statements about an option is not correct?
Correct Answer: B. The buyer of an option contract receives an up-front premium from the seller
of the option contract.
A. The owner of a put option has the right to sell the underlying asset at a fixed price.
B. The buyer of an option contract receives an up-front premium from the seller of the
option contract.
C. The owner of a call option has the right to buy the underlying asset from the seller.
D. The seller (writer) of an option contract receives an up-front premium from the owner of
the option contract.
Explanation:
By definition, a premium is the initial purchase price of an option and it is usually stated on a
per unit basis. The writer (seller) of an option contract receives an up-front premium from the
buyer (owner) of the option contract. This premium obligates the writer to fulfill the contract
(sell or buy the underlying asset) if the buyer chooses to exercise the option.

Question #28: An organization would like to quantify the expected loss that may occur from two
events. A systems analyst estimates that the potential monetary loss from unauthorized alteration
of confidential information in their product development database could be $2,000,000 and has a
5% likelihood of occurrence. In addition, there is a maximum potential loss of $5,000,000 with a
1% likelihood of occurrence if the entire computer lab is destroyed by fire. The total expected
loss of these events is:
Correct Answer: D. $150,000.
A. $0.
B. $50,000.
C. $100,000.
D. $150,000.
Explanation:
An expected loss can be quantified by multiplying the potential monetary loss times the
likelihood of occurrence. The total expected loss of the two items in this organization are:
($2,000,000 x 0.05) ÷ ($5,000,000 x 0.01) = $150,000.
Question #29: IMA's Statement of Ethical Professional Practice requires an IMA member to
follow the established policies of the organization when faced with an ethical conflict. If these
policies do not resolve the conflict, the member should
Correct Answer: D. Contact the next higher managerial level if initial presentation to the
immediate superior does not resolve the conflict.
A. Consult the board of directors immediately.
B. Discuss the problem with the immediate superior if (s)he is involved in the conflict.
C. Communicate the problem to authorities outside the organization.
D. Contact the next higher managerial level if initial presentation to the immediate superior
does not resolve the conflict.
Explanation:
Answer (A) is incorrect.
This course of action would be appropriate only for the chief executive officer or for his or her
immediate subordinate when the CEO is involved in the conflict.
Answer (B) is incorrect.
The proper action would be to present the matter to the next higher managerial level.
Answer (C) is incorrect.
Such action is inappropriate unless legally prescribed.
Answer (D) is correct.
In these circumstances, the problem should be discussed with the immediate superior unless
(s)he is involved. In that case initial presentation should be to the next higher managerial level. If
the problem is not satisfactorily resolved after initial presentation, the question should be
submitted to the next higher level.

Question #30: The internal rate of return (IRR) is the


Correct Answer: C. Rate of interest for which the net present value is equal to zero.
A. Hurdle rate.
B. Rate of interest for which the net present value is greater than 1.0.
C. Rate of interest for which the net present value is equal to zero.
D. Rate of return generated from the operational cash flows.
Explanation:
Answer (A) is incorrect.
The hurdle rate is a concept used to calculate the NPV of a project: it is determined by
management prior to the analysis.
Answer (B) is incorrect.
The IRR is the rate of interest at which the NPV is zero.
Answer (C) is correct.
The IRR is the interest rate at which the present value of the expected future cash inflows is
equal to the present value of the cash outflows for a project. Thus, the IRR is the interest rate that
will produce a net present value (NPV) equal to zero. The IRR method assumes that the cash
flows will be reinvested at the internal rate of return.
Answer (D) is incorrect.
The IRR is a means of evaluating potential investment projects.

Question #31: Which of the following corporations are subject to the accounting requirements of
the Foreign Corrupt Practices Act (FCPA)?
Correct Answer: D. All corporations whose securities are registered pursuant to the Securities
Exchange Act of 1934.
A. All corporations engaged in interstate commerce.
B. All domestic corporations engaged in international trade.
C. All corporations that have made a public offering under the Securities Act of 1933.
D. All corporations whose securities are registered pursuant to the Securities Exchange Act
of 1934.
Explanation:
Answer (A) is incorrect.
The accounting requirements apply only to publicly held companies registered under the 1934
act.
Answer (B) is incorrect.
The accounting requirements apply only to publicly held companies registered under the 1934
act.
Answer (C) is incorrect
The accounting requirements apply only to publicly held companies registered under the 1934
act.
Answer (D) is correct.
The accounting requirements of the FCPA apply to all companies required to register and report
under the Securities Exchange Act of 1934. These companies must maintain books, records, and
accounts in reasonable detail that accurately and fairly reflect transactions. The FCPA also
requires these companies to maintain a system of internal accounting control that provides
certain reasonable assurances, including that corporate assets are not used for bribes.

Question #32: Grapevine Corporation produces two joint products. JP-1 and JP-2, and a single
by-product. BP-1, in Department 2 of its manufacturing plant. JP-1 is subsequently transferred to
Department 3 where it is refined into a more expensive, higher-priced product. JP-1R, and a by-
product known as BP-2. Recently. Santa Fe Company introduced a product that would compete
directly with JP-1R and. as a result. Grapevine must reevaluate its decision to process JP-1
further. The market for JP-1 will not be affected by Santa Fe's product, and Grapevine plans to
continue production of JP-1, even if further processing is terminated. Should this latter action be
necessary, Department 3 will be dismantled.
Which of the following items should Grapevine consider in its decision to continue or terminate
Department 3 operations?
1. The selling price per pound of J P-1.
2. The total hourly direct labor cost in Department 3.
3. Unit marketing and packaging costs for BP-2.
4. Supervisory salaries of Department 3 personnel who will be transferred elsewhere in the
plant, if processing is terminated.
5. Department 2 joint cost allocated to JP-1 and transferred to Department 3.
6. The cost of existing JP-1R inventory.

Correct Answer: D. 1, 2, 3.
A. 2, 3, 5, 6.
B. 1, 2, 3, 4, 5.
C. 2, 3, 4.
D. 1, 2, 3.
Explanation:
The decision to sell a product or process it further is dependent on the cost of the process and the
change in market value from the further processing. The further processing costs include the
direct labor costs in Department 3 and the marketing and packaging costs related to BP-2. The
change in market value in is the value of JP-1 less the value of JP1. These are items 1, 2 and 3.

Question #33: The precautionary motive for holding cash is best described as:
Correct Answer: B. providing a buffer for unexpected cash needs that result from the
unpredictable nature of cash inflows and outflows.
A. Using surplus liquid reserves to take advantage of short-term investments or other
temporary situations.
B. Providing a buffer for unexpected cash needs that result from the unpredictable nature of
cash inflows and outflows.
C. Centralizing inventories to minimize excessive liquidity deficits.
D. Synchronizing cash inflows and outflows so that excess cash balances can be invested in
short-term instruments.
Explanation:
Corporations need sufficient financial resources (e.g., cash balances in banks and securities
as well as backup lines of credit and other short-term borrowing arrangements) to maintain
adequate liquidity. The precautionary motive for holding cash provides a buffer for
unexpected cash needs. The unpredictable nature of cash inflows and outflows means a firm
must maintain sufficient levels of cash or near-cash balances to cover expenses.

Question #34: Verla Industries is trying to decide which one of the following two options to
pursue. Either option will take effect on January 1st of the next year.
Option One - Acquire a New Finishing Machine.
The cost of the machine is $1,000,000 and will have a useful life of five years. Net pre-tax
cash flows arising from savings in labor costs will amount to $100,000 per year for five
years. Depreciation expense will be calculated using the straight-line method for both
financial and tax reporting purposes. As an incentive to purchase, Verla will receive a trade-
in allowance of $50,000 on their current fully depreciated finishing machine.
Option Two - Outsource the Finishing Work.
Verla can outsource the work to LM Inc. at a cost of $200,000 per year for five years. If they
outsource. Verla will scrap their current fully depreciated finishing machine.
Verla's effective income tax rate is 40%. The weighted-average cost of capital is 10%.
When comparing the two options, the $50,000 trade-in allowance would be considered:
Correct Answer: B. Relevant because it is a decrease in cash outflow.
A. Irrelevant because it does not affect cash.
B. Relevant because it is a decrease in cash outflow.
C. Relevant because it is an increase in cash outflows.
D. Irrelevant because it does not affect taxes.
Explanation: The $50,000 trade-in allowance decreases the initial investment in the project
and. Therefore, is relevant.

Question #35: A corporation's return on equity can be calculated if you know its
Correct Answer: A. Sustainable equity growth rate and dividend payout ratio.
A. Sustainable equity growth rate and dividend payout ratio.
B. Debt-equity ratio and market-to-book ratio.
C. Market-to-book ratio and equity multiplier.
D. Dividend yield and earnings yield.
Explanation:
Answer (A) is correct
The sustainable equity growth rate can be found by multiplying return on equity by 1 minus
the dividend payout ratio. Thus, the return on equity can be derived given the sustainable
growth rate and the dividend payout ratio.
Answer (B) is incorrect.
The market-to-book ratio cannot be used to calculate return on equity.
Answer (C) is incorrect
The equity multiplier and return on assets could calculate return on equity.
Answer (D) is incorrect.
These numbers would not provide the information needed to find the net income or average
total equity.

Question #36: What information should a company take into consideration when making a
decision on whether to add or drop a product line?
Correct Answer: C. Revenue and all expenses.
A. Revenue, variable expenses, and unavoidable fixed expenses.
B. Revenue, variable expenses, and avoidable fixed expenses.
C. Revenue and all expenses.
D. Revenue and variable expenses only.
Explanation:
A firm should produce and sell a product if the price multiplied by the number of units sold is
at least equal to its cost. The product's cost is its opportunity cost. The opportunity cost
consists of avoidable variable and avoidable fixed costs incurred by producing and selling the
product, plus any cash flows foregone by producing and selling the product. The foregone
cash flow is normally the lost contribution margin from some alternative product that cannot
be produced and sold.

Question #37: Risk assessment is a function of management’s attitude toward risk. A risk
averse management team typically:
Correct Answer: B. Is more likely to spend money to mitigate risk.
A. Does not prioritize mitigation of risks.
B. Is more likely to spend money to mitigate risk.
C. Is not willing to spend time to mitigate risk
D. Is not willing to spend money to mitigate risk
Explanation:
Risk assessment is a function of the management attitude toward risk as well as the estimate
of potential risk For example, the more risk averse a management team is, the more they will
be willing to spend on mitigating the risk Likewise, the greater the potential risk is perceived
to be, the more time and money management will be willing to spend to minimize or mitigate
the risk.

Question #38: Careful reading of an annual report will reveal that off-balance-sheet debt
includes
Correct Answer: A. Amounts due in future years under operating leases
A. Amounts due in future years under operating leases.
B. Transfers of accounts receivable without recourse.
C. Current portion of long-term debt.
D. Amounts due in future years under capital leases.
Explanation:
Answer (A) is correct
Off-balance-sheet debt includes any type of liability for which the company is responsible
but that does not appear on the balance sheet. The most common example is the amount due
in future years on operating leases. Under U.S. GAAP, operating leases are not capitalized;
instead, only the periodic payments of rent are reported when actually paid. Capital leases
(those similar to a purchase) must be capitalized and reported as liabilities.
Answer (B) is incorrect.
Transfers of accounts receivable without recourse do not create a liability for the company.
This transaction is simply a transfer of receivables for cash.
Answer (C) is incorrect
The current portion of long-term debt is shown on the balance sheet as a current liability.
Answer (D) is incorrect.
Amounts due in future years under capital leases are required to be recognized under U.S.
GAAP.

Question #39: Stanley Company uses the Capital Asset Pricing Model (CAPM) to estimate
the rate of return demanded by the market on its common stock. If the beta coefficient is 1.75
for Stanley, the risk-free rate of return in the stock market is 4.6%, and the current rate of
return for the market as measured by an appropriate index is 7%, what is the market's
required rate of return on Stanley's common stock?
Correct Answer: C. 8.8%.
A. 8.05%.
B. 11.2%.
C. 8.8%.
D. 12.25%.
Explanation:
Using the CAPM formula, the required rate of return on a common stock is calculated as:
Ke = Rf + β(Km - Rf)
Where:
Ke = required rate of return
Rf = risk-free rate (such as the return on U.S. T-bill or T-bonds)
R= beta coefficient for the company
Km = return on a market portfolio
Therefore, required rate of return on Stanley's common stock is:
Ke = 4.6%+ 1.75(7% - 4.6%)
Ke = 4.6%4- (1.75) (2.4%)
Ke = 4.6%+ 4.2% = 8.8%.

Question #40: Which one of the following statements concerning cash flow determination for
capital budgeting purposes is not correct?
Correct Answer: B. Book depreciation is relevant because it affects net income.
A. Tax depreciation must be considered because it affects cash payments for taxes.
B. Book depreciation is relevant because it affects net income.
C. Sunk costs are not incremental flows and should not be included.
D. Net working capital changes should be included in cash flow forecasts.
Explanation:
Answer (A) is incorrect.
It is a true statement relating to capital budgeting.
Answer (B) is correct.
Tax depreciation is relevant to cash flow analysis because it affects the amount of income
taxes that must be paid. However, book depreciation is not relevant because it does not affect
the amount of cash generated by an investment.
Answer (C) is incorrect.
It is a true statement relating to capital budgeting.
Answer (D) is incorrect.
It is a true statement relating to capital budgeting.

Question #41: Which one of the following is a true statement regarding organizational
ethics?
Correct Answer: D. Paying attention to "whistleblowers" plays a significant role in
maintaining an effective ethical atmosphere.
A. As long as officer and employee behavior meet the requirements of the law, the
organization can be considered to have a functioning system of ethical behavior.
B. Strong sense of ethics on the part of employees who are in the best position to appropriate
cash and other assets is the most vital part of a functioning system of ethical behavior.
C. If an organization has a strong code of ethical conduct in place, the role of employee
training can be downplayed.
D. Paying attention to "whistleblowers" plays a significant role in maintaining an effective
ethical atmosphere.
Explanation:
Answer (A) is incorrect.
A sense of ethics requires an ability to distinguish between ethical and merely legal behavior.
"Values and Ethics: From Inception to Practice" states, in part. "Many individuals at the
center of corporate scandals (of the late 20th and early 21st Century] have professed the
belief that they were innocent of any wrongdoing, including Kenneth Lay of Enron or
Conrad Black of Hollinger. The problem is that these individuals did not define their
behavior by what most of society would see as 'reasonable but rather they followed their own
particular code-in some cases, limiting the definition of ethical behavior to require
compliance with the law and nothing more!' (II. Introduction.)
Answer (B) is incorrect.
"Values and Ethics: From Inception to Practice states, in part, "Ethical behavior is not
something that applies to someone else-every single individual is responsible for behaving
ethically. Nowhere is this more important than the demonstration of ethical behavior that
managers and supervisors exhibit in the way they execute their day-to-day work..." This
phenomenon is referred to as the "tone at the top." (VI. Leadership by Example.)
Answer (C) is incorrect.
Employee training is important to maintaining an ethical organizational culture. "Values and
Ethics: From Inception to Practice" states, in part. "Every existing member of staff should
receive ongoing training, starting at the board level and cascading down throughout the
organization ... Ethics training for employees should focus on covering ethical concepts, the
organization's code, and compliance. To achieve this, training should include: ethical
concepts and thinking: What is 'behind' the issue of ethical action? [and] the organization's
code of ethics and any supporting 'rules:" (VIII. Practical Application: Converting Intent into
Operational Reality.)
Answer (D) is correct.
"Values and Ethics: From Inception to Practice states, in part, "A whistleblowing framework
(e.g., an ethics helpline) is an important component in maintaining an ethical organizational
culture. An effective feedback system includes having a confidential framework for
employees to report possible violations of the organization's code of ethics and to receive
advice on the ethical aspects of challenging decisions. Statistics show that a large number of
occupational fraud cases are detected through an employee "hotline" or other reporting
method ..." (IX. Measuring and Improving Ethical Compliance.)
Question #42: The total cost of producing 100 units of a good is $800. If a firm's average
variable cost is $5 per unit, then the firm's:
Correct Answer: D. Average fixed cost is $3.
A. Marginal cost is $8.
B. Marginal cost is $3.
C. Total variable cost is $300.
D. Average fixed cost is $3.
Explanation:
We know that the total cost of producing 100 units is $800, therefore, we can compute the
total cost per unit as:
Total cost per unit = Total Cost / # of Units produced
Total cost per unit = $800/100 units = $8 per unit
Given in this problem is the average variable cost per unit of $5, therefore, we can rearrange
the following formula to determine the average fixed per unit:
Total cost per unit = Average Fixed cost per unit + Variable cost per unit
$8 = Average Fixed cost per unit + $5
Average Fixed cost per unit = $3
Total variable costs = $500 ($5 per unit x 100 units)
The marginal cost is neither $3 nor $8 because marginal cost is defined as the cost change in
producing one more unit. The way fixed costs react to adding more units of production is by
allocating the total fixed costs over more units, thus average fixed cost per unit would
decrease.

Question #43: MultiFrame Company has the following revenue and cost budgets for the two
products it sells.
Plastic Glass
frames frames
Sales price $10.00 $15.00
Direct materials 2.00 3.00
Direct labor 3.00 5.00
Fixed overhead 3.00 4.00
Net income per $2.00 $3.00
unit

The budgeted unit sales equal the current unit demand, and total fixed overhead for the year
is budgeted at $975,000. Assume that the company plans to maintain the same proportional
mix. In numerical calculations, MultiFrame rounds to the nearest cent and unit.
The total number of units needed to break even if the budgeted direct labor costs were $2 for
plastic frames instead of $3 is:
Correct Answer: B. 144,444 units.
A. 156,000 units.
B. 144,444 units.
C. 325,000 units.
D. 154,028 units.
Explanation: The breakeven in total unit volume is calculated by dividing the fixed costs of
$975,000 by the weighted average unit contribution margin of $6.75.
Breakeven volume = $975,000/$6.75 = 144,444.44, which rounds to 144,444 units in total.
The weighted average contribution margin is the sum of (unit contribution margin * relative
weight) for each product.
The unit contribution margin for each product is calculated by taking the unit price less unit
variable costs (direct materials & direct labor).
Contribution Margin (Plastic Frames) = ($10- $2 - $2) = $6
Contribution Margin (Glass Frames) = ($15 - $3 - $5) = $7
The relative weight for each product is calculated by taking the number of units of that product,
divided by the total number of units of all products.
Relative weight (Plastic Frames) = 100,000 plastic frames / (100,000 plastic frames + 300,000
glass frames) = 0.25 or 25%
Relative weight (Glass Frames) = 300,000 glass frames / (100,000 plastic frames + 300,000 glass
frames) = 0.75 or 75%
Weighted average contribution margin = ($6) (25%) + ($7) (75%) = $1.50 + $5.25 = $6.75.

Question #44: A drilling company is evaluating a project to produce a high-tech deep-sea oil
exploration device. The investment required is $80 million for a plant with a capacity of 15,000
units a year for 5 years. The device will be sold for a price of $12,000 per unit. Sales are
expected to be 12,000 units per year. The variable cost is $7,000 and fixed costs, excluding
depreciation, are $25 million per year. Assume the drilling company employs straight-line
depreciation on all depreciable assets, and assume that they are taxed at a rate of 36%. If the
required rate of return is 12%, what is the approximate NPV of the project?
Correct Answer: B. $21,516,800
A. $17,225,000
B. $21,516,800
C. $26,780,000
D. $56,124,800
Explanation:
Answer (A) is incorrect.
The amount of $17,225,000 results from failing to deduct depreciation in calculating taxes.
Answer (B) is correct.
The following table derives the cash flows and NPV.

Item Year 0 Year 1 to 5


Investme $(80,000,00
nt 0)
$144,000,0
Revenue 00
Variable cost 84,000,000
Fixed cost 25,000,000
Depreciation 16,000,000
Pre-tax profit 19,000,000
Tax @ 36% 6,840,000
Net profit 12,160,000
Net cash flow 28,160,000
Present value
12% (28,160,000 × 101,516,00
3.605) 0
NPV = $21,516,800

NPV = $21,516,800
Answer (C) is incorrect
The amount of $26,780,000 results from failing to consider that depreciation is a noncash
expense.
Answer (D) is incorrect.
The amount of $56,124,800 is based on annual sales of 15,000 units, rather than 12,000 units.
Question #45: Company A has divisions in country X. a high tax country, and country Y, a low
tax country. It manufactures computer chips for radios in country X and sells them at cost to its
division in country Y. The division in country Y then sells the radios at wholesale and retail
prices. Which of the following is true?
Correct Answer: A. This practice is called transfer pricing, and it is used by companies to
manage their effective worldwide tax rate
A. This practice is called transfer pricing, and it is used by companies to manage their
effective worldwide tax rate.
B. This practice is called transfer pricing, and it was outlawed by the General Agreement on
Tariffs and Trade (GATT).
C. This practice is called direct foreign investment, and it was outlawed by the General
Agreement on Tariffs and Trade (GATT).
D. This practice is called direct foreign investment, and it is used by companies to manage
their effective worldwide tax rate.
Explanation:
This is an example of the practice of transfer pricing, used by companies to manage their
effective worldwide tax rate by attempting to maximize profits in the country with the lowest tax
rates. Although not outlawed by GATT, it is part of the Organization of Economic Cooperation
and Development Treaty, which requires transfers to be priced at arms-length transactions, that
is, prices that would be set by unrelated parties.

Question #46: Question: Smithco's payback period for the project will be (please refer the fact
pattern below)
[Fact Pattern #158] Smithco is considering the acquisition of scanning equipment to mechanize
its procurement process. The equipment will require extensive testing and debugging, as well as
user training, prior to its operational use. Projected after-tax cash flows are shown below.
After-tax cash
inflow/(outflow
Year s)
0 $(550,000)
1 (500,000)
2 450,000
3 350,000
4 350,000
5 350,000

Management anticipates the equipment will be sold at the beginning of Year 6 for 550.000 when
its book value is zero. Smithco's internal hurdle and effective tor rates are 14% and 40%,
respectively.
Correct Answer: D. 3.75 years.
A. 2.3 years.
B. 3.0 years.
C. 3.5 years.
D. 3.75 years.
Explanation:
Answer (A) is incorrect.
The period of 2.3 years fails to consider the outflow in Year 1.
Answer (B) is incorrect.
The period of 3.0 years fails to consider the outflow in Year 1.
Answer (C) is incorrect.
The period of 3.5 years fails to consider the outflow in Year 1.
Answer (D) is correct.
Payback is the measure of the period it takes to recover the initial investment in a project. It will
take almost 4 years to recover the initial investment of $500,000. Because of the outflow of
$500,000 during the first year, the company will be trying to recover $1,050,000 by the end of
Year 1. After recovering $450,000 in Year 2 and $350,000 in Year 3, the company will still need
to recover an additional $250,000 (1,050,000 – 450,000 – 350,000) in Year 4. Thus, it will take
about 5/7 ($250,000/$350,000) of Year 4 to recover the initial investment.

Question #47: Last year, a corporation had a total asset turnover ratio of 1.5, a profit margin of
10%, and an equity multiplier of 2. This year, if the profit margin is 8%, but the return on equity
stays the same, then what could be true?
Correct Answer: D. The equity multiplier increases to 3.0, and the total asset turnover decreases
to 1.25.
A. The equity multiplier remains 2.0, and the total asset turnover increases to 1.7.
B. The equity multiplier remains 2.0, and the total asset turnover increases to 3.5.
C. The equity multiplier increases to 2.2, and the total asset turnover remains 1.5.
D. The equity multiplier increases to 3.0, and the total asset turnover decreases to 1.25.
Explanation:
Answer (A) is incorrect.
This would yield a return on equity of 27.2% (8% x 1.7 x 2.0).
Answer (B) is incorrect.
This would yield a return on equity of 56% (8% x 3.5 x 2.0).
Answer (C) is incorrect
This would yield a return on equity of 26.4% (8% x 1.5 x 2.2).
Answer (D) is correct.
The return on equity (ROE) for last year is 30% (10% x 1.5 x 2). Given these facts, this year's
ROE is also 30% (8% x 1.25 x 3.0).

Question #48: All of the following are affected when merchandise is purchased on credit except
Correct Answer: B. Net working capital.
A. Total current assets.
B. Net working capital.
C. Total current liabilities.
D. Current ratio.
Explanation:
Answer (A) is incorrect.
Current assets are increased by the amount of inventory purchased.
Answer (B) is correct.
Working capital is the net of current assets and current liabilities. When merchandise is
purchased on credit, inventory (a current asset) is increased, and accounts payable (a current
liability) is increased by the same amount. Thus, no net change in working capital results.
Answer (C) is incorrect.
Current liabilities are increased by the amount of the credit purchase.
Answer (D) is incorrect.
The numerator (current assets) and the denominator (current liabilities) of the current ratio are
increased by the same amount.

Question #49: Fact Pattern: Jensen Corporation's board of directors met on June 3 and declared a
regular quarterly cash dividend of $.40 per share for a total value of $200,000. The dividend is
payable on June 24 to all stockholders of record as of June 17. Excerpts from the statement of
financial position for Jensen Corporation as of May 31 are presented as follows.

Cash $400,000
Accounts
receivable 800,000
Inventories 1,200,000
$2,400,00
Total current assets 0
Total current $1,000,00
liabilities 0

Assume that the only transactions to affect Jensen Corporation during June are the dividend
transactions. Question: Jensen's quick (acid test) ratio would be
Correct Answer: D. Decreased by the dividend declaration and unchanged by the dividend
payment.
A. Unchanged by the dividend declaration and decreased by the dividend payment.
B. Decreased by the dividend declaration and increased by the dividend payment.
C. Unchanged by either the dividend declaration or the dividend payment.
D. Decreased by the dividend declaration and unchanged by the dividend payment.
Explanation:
Answer (A) is incorrect.
The dividend declaration increases current liabilities and decreases the quick ratio. The
subsequent payment of a cash dividend decreases quick assets and current liabilities by the same
amount. Accordingly, the ratio (1.0) remains unchanged by the payment.
Answer (B) is incorrect.
The dividend declaration increases current liabilities and decreases the quick ratio. The
subsequent payment of a cash dividend decreases quick assets and current liabilities by the same
amount. Accordingly, the ratio (1.0) remains unchanged by the payment.
Answer (C) is incorrect
The dividend declaration increases current liabilities and decreases the quick ratio. The
subsequent payment of a cash dividend decreases quick assets and current liabilities by the same
amount. Accordingly, the ratio (1.0) remains unchanged by the payment.
Answer (D) is correct.
The quick ratio is computed by dividing quick assets (cash. marketable securities, and
receivables) by current liabilities. The dividend declaration will increase current liabilities to
$1,200,000 and decrease the quick ratio to 1.0 ($1,200,000 quick assets $1,200,000 current
liabilities). The payment of a cash dividend will decrease quick assets (cash) and the current
liabilities by the same amount. Accordingly, the ratio (1.0) remains unchanged by the payment.

Question #50: The following financial information is given.

Year 1 Year 2
Book value of assets $18,000 $26,000
Market value of equity 18,000 60,000
12 months ended 12 months ended
Year 1 Year 2
Sales $1,000 $1,300
Cost of goods sold 500 700
Operating income 500 600
Depreciation expense 200 200
Interest expense 100 100
Pretax income 200 300
Income tax expense 80 120
Net income $120 $180

Using a common-size income statement, did operating income and net income increase or
decrease?
Correct Answer: C. Decreased (Operating income). (Net income) Increased
A. Increased (Operating income). (Net income) Increased
B. Increased (Operating income). (Net income) Decreased
C. Decreased (Operating income). (Net income) Increased
D. Decreased (Operating income). (Net income) Decreased
Explanation:
Answer (A) is incorrect.
Although total operating income increased, it decreased when restated in common size.
Answer (B) is incorrect.
Although total operating income increased, it decreased when restated in common size.
Furthermore, both actual net income and common-size net income increased.
Answer (C) is correct.
A common-size income statement restates line items as a percentage of net sales. The table
below provides these percentages.

Year 1 Year 2
100.00 100.00
Sales % %
Cost of goods sold 50.00% 53.85%
Operating income 50.00% 46.15%
Depreciation
expense 20.00% 15.38%
Interest expense 10.00% 7.69%
Pretax income 20.00% 23.08%
Income tax
expense 8.00% 9.23%
Net income 12.00% 13.85%

The common-size operating income decreased, while net income increased.


Answer (D) is incorrect.
Both actual net income and common-size net income increased.

Question #51: Which one of the following is a true statement regarding organizational ethics?
Correct Answer: A. A comprehensive framework of corporate ethical behavior is a prerequisite
for an effective system of internal control.
A. A comprehensive framework of corporate ethical behavior is a prerequisite for an
effective system of internal control.
B. An effective system of internal control is a prerequisite for corporate ethical behavior.
C. If a functioning system of ethical behavior is in place, an organization is able to devote
fewer resources to developing human capital.
D. "Organizational culture” is determined mostly by the industry(ies) in which the firm
operates.
Explanation:
Answer (A) is correct
A comprehensive framework of corporate ethical behavior is a prerequisite for an effective
system of internal control. "Values and Ethics: From Inception to Practice" states, in part. 'CEOs
and CFOs have to place their own integrity on the line by attesting to compliance with an
adequate level of internal controls (as well as all other certifications). Creating a thorough,
integrated system for developing, implementing. sustaining, and monitoring ethical performance
within the organization will allow executives to make such declarations with confidence that a
code of ethics is the foundation of the organization's culture and is fully integrated into the
thinking process of every employee and business partner!' (IX. Measuring and Improving Ethical
Compliance.)
Answer (B) is incorrect.
It is more nearly true to state the opposite.
Answer (C) is incorrect.
The concept of "human capital" is important to an organization in creating a climate where
"doing the right thing” is expected. In most organizations today, labor costs constitute the
majority of operating expenses. "Values and Ethics: From Inception to Practice" states, in part.
“an organization must, to a great degree, trust that its employees are acting in its best interests.
Human 'capital' is a critical asset ... Unmotivated employees can poison the atmosphere and
reduce the teamwork and cooperation required for knowledge transfer and innovation, and they
can have a significant negative impact on relationships with suppliers and customers (IV. Values.
Ethics, and Accounting.)
Answer (D) is incorrect.
"Values and Ethics: From Inception to Practice states, in part, "Every organization already has a
culture... Step one in establishing an ethical culture must be an assessment of the existing
organizational values and culture and the development of a set of statements that define the
principles the organization believes in and should act upon. These statements and principles can
be developed by the shareholders, the board, or a governing body within the organization.' (V.
Defining and Developing the Organization's Behavioral Values.)

Question #52: All of the following are reasons why a toy manufacturer would carry accounts
receivable except:
Correct Answer: B. To offset seasonal overhead expenses.
A. To compete with the credit terms that competitors offer.
B. To offset seasonal overhead expenses.
C. For potential profit from interest income.
D. To attract new customers.
Explanation:
By definition, accounts receivable refer to the money customers owe a company resulting from
the company's decision to sell products and services on a credit basis. Credit is often described as
a sales tool.

Question #53: All of the following statements in regard to working capital are correct except:
Correct Answer: C. financing permanent inventory buildup with long-term debt is an example of
an aggressive working capital policy.
A. Financing short-term inventory buildup with long-term debt is an example of a maturity
matching working capital policy.
B. The hedging approach to financing involves matching maturities of debt with specific
financing needs.
C. Financing permanent inventory buildup with long-term debt is an example of an
aggressive working capital policy.
D. Current liabilities are an important source of financing for many small firms.
Explanation:
Maturity matching working capital policy involves matching the maturity (lives) of assets with
financing maturities. Financing a permanent buildup in inventory with long-term debt would be
an example of maturity matching.

Question #54: If an IMA member has a problem in identifying unethical behavior or resolving an
ethical conflict, the first action (s)he should normally take is to
Correct Answer: B. Discuss the problem with his or her immediate superior.
A. Consult the board of directors.
B. Discuss the problem with his or her immediate superior.
C. Notify the appropriate law enforcement agency.
D. Resign from the company.
Explanation:
Answer (A) is incorrect.
The board would be consulted initially only if the immediate superior is the chief executive
officer and that person is involved in the ethical conflict.
Answer (B) is correct.
IMA's Statement of Ethical Professional Practice states that the member should first discuss an
ethical problem with his or her immediate superior. If the superior is involved, the problem
should be taken initially to the next higher managerial level.
Answer (C) is incorrect.
An IMA member should keep information confidential except when disclosure is authorized or
legally required.
Answer (D) is incorrect.
Resignation is a last resort.

Question #55: Fact Pattern: Jensen Corporation's board of directors met on June 3 and declared a
regular quarterly cash dividend of $.40 per share for a total value of $200,000. The dividend is
payable on June 24 to all stockholders of record as of June 17. Excerpts from the statement of
financial position for Jensen Corporation as of May 31 are presented as follows.

Cash $400,000
Accounts receivable
(net) 800,000
Inventories 1,200,000
$2,400,00
Total current assets 0
Total current
liabilities $1,00,000

Question: Jensen's working capital would be


Correct Answer: D. Decreased by the dividend declaration and unchanged by the dividend
payment.
A. Unchanged by the dividend declaration and decreased by the dividend payment.
B. Decreased by the dividend declaration and increased by the dividend payment.
C. Unchanged by either the dividend declaration or the dividend payment.
D. Decreased by the dividend declaration and unchanged by the dividend payment.
Explanation:
Answer (A) is incorrect.
The declaration of a dividend increases current liabilities and reduces working capital. The
subsequent payment has no effect on working capital.
Answer (B) is incorrect.
The declaration of a dividend increases current liabilities and reduces working capital. The
subsequent payment has no effect on working capital.
Answer (C) is incorrect
The declaration of a dividend increases current liabilities and reduces working capital. The
subsequent payment has no effect on working capital.
Answer (D) is correct.
Working capital is defined as current assets minus current liabilities. The declaration of a
dividend increases current liabilities and thus reduces working capital. The subsequent payment
has no effect on working capital since current assets and current liabilities decrease by the same
amount.

Question #56: Please see the question below:


Gardena Company currently n using its lull capacity of 25,000 machine hours to manufacture
product XR-2000. LJB Corporation placed an outer with Gardener for the manufacture of 1,000
units of KT 6500. LJB would normally manufacture this component However due to a fire at its
plant, LJB needs to purchase these units to continue manufacturing other products. This is a
onetime special order, the following reflects unit cost data, and selling prices.
KT- XR-
6500 2000
Materials $27 $24
Direct labor 12 10
Variable overhead 6 5
Fixed overhead 48 40
Variable selling &
administrative 5 4
Fixed selling & administrative 12 10
Normal selling price $125 $105
Machine hours required 3 4

What is the minimum unit price that Gardener should charge LJB to manufacture 1,000 units KI-
6500?
Correct Answer: B. $96.50.
A. $93.00.
B. $96.50.
C. $110.00.
D. $125.00.
Explanation: The price Gardener should charge for KT-6500 is calculated as follows:
Price for KT-6500 = (unit variable costs) + (lost contribution margin on the units of XR-2000
that would have to be foregone / units of KT-6500)
Unit variable costs for KT - 6500 = $27 + $12 + $6 + $5 = $50
The production of 1,000 units of KT-6500 would require 3,000 machine hours
(3 hours per unit) (1,000 units) = 3,000 machine hours
The 3,000 hours would reduce the production and sale of XR-2000 by 750 units (3,000 hours
divided by 4 hours per unit = 750 units).
The XR-2000 unit contribution margin = ($105 - $24 - $10 - $5 - $4) = $62.
The lost contribution would then be calculated as follows:
Lost contribution, in total = (number of units) (contribution margin per unit)
Lost contribution, in total = (750 units) ($62) = $46,500
Lost contribution, on a per unit basis = ($46,500) / 1,000 units = $46.50 per unit of KT-6500.
Therefore, the minimum unit price that Gardener should charge LJB to manufacture 1,000 units
of KT-6500 = $50 + $46.50 = $96.50.

Question #57: Fact Pattern: The tax impact of equipment depreciation affects capital budgeting
decisions. Currently, the Modified Accelerated Cost Recovery System (MACRS) is used as the
depreciation method for most assets for tax purposes.
Question: The MACRS method of depreciation for assets with 3, 5, 7 and 10-year recovery
periods is most similar to which one of the following depreciation methods used for financial
reporting purposes?
Correct Answer: D. 200% declining-balance.
A. Straight-line.
B. Units-of-production.
C. Sum-of-the-years'-digits.
D. 200% declining-balance.
Explanation:
Answer (A) is incorrect.
The straight-line method uses the same percentage each year during an asset's life, but MACRS
uses various percentages.
Answer (B) is incorrect.
MACRS is unrelated to the units-of-production method.
Answer (C) is incorrect
MACRS is unrelated to SYD depreciation.
Answer (D) is correct.
MACRS for assets with lives of 10 years or less is based on the 200% declining-balance method
of depreciation. Thus, an asset with a 3-year life would have a straight-line rate of 33-1/3%, or a
double declining-balance rate of 66-2/3%.

Question #58: Fact Pattern: The Keego Company is planning a 5200.000 equipment investment
that has an estimated 5-year life with no estimated salvage value. The company has projected the
following annual cash flows for the investment:
Projected Present
Cash value
Year inflows of $1
$1,200,00
1 0 .91
2 60,000 .76
3 40,000 .63
4 40,000 .53
5 40,000 .44
Totals $300,000 3.27

Correct Answer: C. 2.50 years.


A. 1.67 years.
B. 4.91 years.
C. 2.50 years.
D. 1.96 years.
Explanation:
Answer (A) is incorrect.
This number of years assumes that the inflows of the first year continue at the same rate in the
second year.
Answer (B) is incorrect.
The amount of $296,400 will be recovered after 4.91 years.
Answer (C) is correct.
The payback period is the number of years required to complete the return of the original
investment. The principal problems with the payback method are that it does not consider the
time value of money and the inflows after the payback period. The inflow for the first year is
$120,000, the second year is $60,000, and the third year is $40,000, a total of $220,000. Given
an initial investment of $200,000, the payback period must be between 2 and 3 years. If the cash
inflows occur evenly throughout the year. $20,000 ($200,000 - $120,000 - $60,000) of cash
inflows are needed in year 3, which is 50% of that year's total. Thus, the answer is 2.5 years.
Answer (D) is incorrect.
Less than $180,000 will be paid back after 1.96 years.

Question #59: IMA members are obligated to maintain the highest standards of ethical conduct.
Accordingly. IMA's Statement of Ethical Professional Practice explicitly requires that I MA
members
Correct Answer: B. Not condone violations by others.
A. Obtain sufficient competent evidence when expressing an opinion.
B. Not condone violations by others.
C. Comply with generally accepted auditing standards.
D. Adhere to generally accepted accounting principles.
Explanation:
Answer (A) is incorrect.
The expression of an opinion is a function of an external auditor.
Answer (B) is correct.
The principles section of IMA's Statement of Ethical Professional Practice. IMA's overarching
ethical principles include: Honesty, Fairness, Objectivity and Responsibility. Members shall act
in accordance with these principles and shall encourage others within their organizations to
adhere to them."
Answer (C) is incorrect
Compliance with GAAS is a requirement of external auditors.
Answer (D) is incorrect.
Adherence to GAAP is not expressly required by IMA's Statement of Ethical Professional
Practice.

Question #60: Garner Products is considering a new accounts payable and cash disbursement
process which is projected to add 3 days to the disbursement schedule without having significant
negative effects on supplier relations. Daily cash outflows average $1,500,000. Garner is in a
short-term borrowing position for 8 months of the year and in an investment position for 4
months. On an annual basis, bank lending rates are expected to average 7% and marketable
securities yields are expected to average 4%. What is the maximum annual expense that Garner
could incur for this new process and still break even?
Correct Answer: D. $270,000.
A. $315,000.
B. $180,000.
C. $90,000.
D. $270,000.
Explanation:
The maximum annual expense that Garner could incur for this project would be equal to the
project's expected benefit.
The expected benefit is calculated as: Expected benefit is 4 months' interest on the funds freed
up, adding 3 days to the disbursement schedule.
Expected benefit = [$1,500,000(3 days) (0.04) (4/12)] + [$1.500.003(3 days) (0.07) (8/12)]
Expected benefit = $60,000+ $210,000 = $270,000.

Question #61: Which of the following is not a common application for cost/volume/profit (CVP)
analysis?
Correct Answer: B. Engaging in capital rationing.
A. Deciding whether to replace a piece of equipment.
B. Engaging in capital rationing.
C. Pricing for a new product.
D. Deciding whether to outsource.
Explanation:
Cost/volume/profit analysis data can be used in a wide variety of decision-making situations
such as raising or lowering existing prices, introducing a new product or service, setting
prices for new products and services, expanding product and service markets, deciding
whether to replace an existing piece of equipment, and deciding whether to make or buy a
product or service.

Question #62: If gross profit margin has decreased substantially over the past 3 years, which one
of the following best explains this decrease?
Correct Answer: C. A physical count of merchandise inventory showed missing inventory higher
than expected.
A. The cost of merchandise inventory has decreased while sales prices have remained the
same.
B. Ending merchandise inventory is higher than expected.
C. A physical count of merchandise inventory showed missing inventory higher than
expected.
D. Cost of goods sold has remained steady while total expenses have increased.
Explanation:
Answer (A) is incorrect.
Gross profit margin is net sales minus cost of goods sold. Lower costs of merchandise will
result in a lower cost of goods sold, which increases, not decreases, gross profit margin.
Answer (B) is incorrect.
Gross profit margin is net sales minus cost of goods sold. Thus, a higher ending inventory
results in a lower cost of goods sold, which results in an increase of gross profit margin.
Answer (C) is correct.
A physical count showing missing inventory results in a higher cost of goods sold, which
results in a decrease of gross profit margin.
Answer (D) is incorrect.
Gross profit margin is net sales minus cost of goods sold. Thus, gross profit margin remains
unchanged if both sales and cost of goods sold remain steady.

Question #63: Sunshine Corporation is considering the purchase of a new machine for
$800,000. The machine is capable of producing 1.6 million units of product over its useful
life. The manufacturer's engineering specifications state that the machine-related cost of
producing each unit of product should be $0.50. Sunshine's total anticipated demand over the
asset's useful life is 1.2 million units. The average cost of materials and labor for each unit is
$0.40. In considering whether to buy the new machine, would you recommend that Sunshine
use the manufacturer's engineering specification of machine-related unit production cost?
Correct Answer: D. No. the machine-related cost of producing each unit is $0.67.
A. No, the machine-related cost of producing each unit is $0.90.
B. No, the machine-related cost of producing each unit is $2.00.
C. Yes, the machine-related cost of producing each unit is $0.50.
D. No, the machine-related cost of producing each unit is $0.67.
Explanation:
The machine-related cost per unit is calculated as:
Machine-related cost per unit = (cost of machine) / (useful life in units)
Machine-related cost per unit = ($800,000) / (1,200,000 units) = $0.67 per unit.

Question #64: Please see the question below:


Aspen Company plans to sell 12,000 units of product XT and & 8,000 units of product RP.
Aspen has a capacity of 12,000 productive machine hours. The unit cost structure and
machine hours required for each product is as follows.

Unit costs XT RP
Materials $37 $24
Direct labor 12 13
Variable overhead 6 3
Fixed overhead 37 38
Machine hours
required 1.0 1.5

Aspen can purchase12,000 units of XT at $50 and/or RP units of RP at $45. Based on the
above, which one of the following actions should be recommended to Aspen’s management?
Correct Answer: C. Produce XT internally and purchase RP.
A. Purchase both XT and RP.
B. Produce both XT and RP.
C. Produce XT internally and purchase RP.
D. Produce RP internally and purchase XT.
Explanation: Aspen's goal would be to minimize costs, so producing XT internally and
purchasing RP will result in the lowest cost.
Alternative: Produce XT internally and purchase RP.
This alternative would cost $1,020,000, which is made up of the costs of both XT and RP as
follows:
Cost of XT, produced internally = (materials + direct labor + variable overhead) (number of
units)
Cost of XT, produced internally = ($37 + $12 + $6) (12,000) = ($55) (12,000) = $660,000
Cost of RP, purchased = (purchase cost) (number of units) = ($45) (8,000) = $360,000
Total cost of XT and RP, this alternative = $660,000+ $360,000 = $1,020,000
Alternative: Produce RP internally and purchase XT.
This alternative would cost $1,040,000, which would be calculated as follows:
Cost of RP, produced internally = (materials + direct labor + variable overhead) (number of
units)
Cost of RP, produced internally = ($24 + $13 + $3) (8,000) = ($40) (8,000) = $320,000
Cost of XT, purchased = (purchase cost) (number of units) = ($60) (12,000) = $720,000
Total cost of RP and XT, this alternative = $320,000+ $720,000 = $1,040,000
Alternative: Purchase both XT and RP.
This alternative would cost $1,080,000, which would be calculated as follows:
Cost of RP, purchased = (purchase cost) (number of units) = ($45) (8,000) = $360,000
Cost of XT, purchased = (purchase cost) (number of units) = ($60) (12,000) = $720,000
Total cost of RP and XT, this alternative = $360,000+ $720,000 = $1,080,000
Alternative: Produce both XT and RP.
This alternative is not feasible. XT would use the capacity of 12,000 hours (12,000 units at 1
hour each).

Question #65: The management team attended an ethics training session at the IMA Annual
Conference and subsequently made plans to enhance their ethics program. The president plans to
chair a committee of employees to review the company's behavioral values, while the CFO
intends to review the ethical standards applicable to the Finance Department. The manager of the
Human Resources Department will investigate the feasibility of establishing a whistleblowing
framework that includes a "hotline for reporting ethics violations. These activities exemplify
Correct Answer: B. Leadership by example.
A. Organizational transparency.
B. Leadership by example.
C. Measurement of ethical compliance.
D. Alignment of internal controls with ethical standards.
Explanation:
Answer (A) is incorrect.
These activities exemplify leadership by example.
Answer (B) is correct.
"Tone at the top" plays an important role in determining an organization's ethical environment.
For a code of ethics to be effective, its application must be demonstrated by those in positions of
power and leadership. Leaders must be seen living and managing by the code of ethics.
Answer (C) is incorrect.
These activities exemplify leadership by example.
Answer (D) is incorrect.
These activities exemplify leadership by example.

Question #66: A corporation experienced the following year-over-year changes:


Net profit margin Increased 25%
Total asset turnover Increased 40%
Total assets Decreased 10%
Total equity Increased 40%
Using DuPont analysis, what is the year-over-year change in return on equity (ROE)?
Correct Answer: D. Increased 12.5%.
A. Increased 95.0%.
B. Increased 63.0%.
C. Increased 12.5%.
D. Increased 10.0%.
Explanation:
Answer (A) is incorrect.
The DuPont Model for ROE is as follows: Net profit margin x Total asset turnover x Equity
multiplier (Total assets ÷ Total equity). The year-over-year change is not calculated by simply
adding and subtracting the increases and decreases from last year to this year. The incorrect
amount of 95% results from adding and subtracting the year-over-year changes (25%+ 40% -
10% + 40%).
Answer (B) is incorrect.
The year-over-year change is not calculated by simply dividing the increase in the net profit
margin by the increase in the total asset turnover.
Answer (C) is correct.
The ROE using the DuPont analysis is calculated as follows:
Net profit margin x Total asset turnover x Equity multiplier (Total assets ÷ Total equity)
The best way to solve this problem is to use actual numbers for the return on equity comparison
of this year to last year. Assuming that last year the corporation had a net profit margin of .025,
total asset turnover of 1.05, total assets of $500,000, and total equity of $200,000, last year's
ROE is equal to 6.56% [.025 x 1.05 x ($500,000 ÷ $200,000)].
By using the information given in the problem, the current-year amounts can be calculated,
resulting in a net profit margin of .03125 (increased by 25%). total asset turnover of 1.47
(increased by 40%). total assets of $450,000 (decreased by 10%). and total equity of $280,000
(increased by 40%). Therefore, this year's ROE is equal to 7.38% [.03125 x 1.47 x (450,000 ÷
280,000)]. The increase in ROE from last year to this year can now be calculated as 12.5% [(7.38
- 6.56) ÷ 6.56].
Answer (D) is incorrect.
This answer choice incorrectly multiples the year-over-year change for the net profit margin by
the year over-year change for the total asset turnover to get an increase in ROE of 10.0%.

Question #67: All of the following are included when calculating the acid test ratio except
Correct Answer: B. Prepaid insurance.
A. Six-month treasury bills.
B. Prepaid insurance.
C. Accounts receivable.
D. 60-day certificates of deposit.
Explanation:
Answer (A) is incorrect.
Six-month treasury bills are marketable securities and thus are included in the numerator of the
acid test ratio.
Answer (B) is correct.
The acid test (quick) ratio consists of the quick assets (cash. marketable securities. and net
accounts receivable) divided by current liabilities. Prepaid insurance is an illiquid current asset
and thus not appropriate to include in the numerator.
Answer (C) is incorrect
Accounts receivable are included in the numerator of the acid test ratio.
Answer (D) is incorrect.
Sixty-day certificates of deposit are cash equivalents and thus are included in the numerator of
the acid test ratio.

Question #68: The cost-plus pricing approach is generally in what formula?


Correct Answer: B. Unit cost x (1+ markup %on unit cost) = selling price.
A. Unit cost / selling price = markup percentage.
B. Unit cost x (1+ markup % on unit cost) = selling price.
C. Variable cost + fixed cost + contribution margin - selling price.
D. Cost base + gross margin = selling price.
Explanation:
Cost-plus pricing involves setting the price of a product so as to earn a set markup percentage on
sales. The cost-plus pricing approach is expressed using the following formula:
(Unit Cost) x (1 + Markup %on unit cost) = Selling Price.

Question #69: Risks that relate to losses from inadequate or failed internal processes, people and
systems are commonly called:
Correct Answer: C. Operational risks.
A. Hazard risks.
B. Financial risks.
C. Operational risks.
D. Strategic risks.
Explanation:
Hazard risks relate to natural disasters such as storms, floods, hurricanes, blizzards,
earthquakes, and volcanoes. Financial risks are caused by debt/equity decisions related to
financing the business. They include liquidity (short-term bill paying) and solvency (long-
term bill paying). Operational risk relates to the relationship of fixed and variable costs in the
organization's cost structure as well as the following: internal process failures, system
failures, personnel, and legal and compliance risks.

Question #70: The following information has been derived from a company's financial
statements:
Current assets: $640,000
Total assets: 990,000
Long-term liabilities: 130,000
Current ratio: 3.2
The company's debt to equity ratio is
Correct Answer: A. 0.50
A. 0.50
B. 0.37
C. 0.33
D. 0.13
Explanation:
Answer (A) is correct
Current liabilities are $200,000 ($640,000 ÷ 3.2). Thus, total debt load is $330,000 ($200,000
+ $130,000) and total equity is $660,000 ($990,000 - $330,000). Therefore, the debt to equity
ratio is 0.50 to 1($330,000 ÷ $660,000).
Answer (B) is incorrect.
Total debt would be $330,000 and equity would be $660,000.
Answer (C) is incorrect.
A ratio of 0.33 results from improperly dividing total liabilities by total assets instead of total
equity.
Answer (D) is incorrect.
A ratio of 0.13 results from dividing only long-term liabilities, instead of total liabilities, by
total assets, instead of net assets.

Question #71: An avoidable cost differs from an unavoidable cost in that:


Correct Answer: A. avoidable costs are decision relevant but unavoidable costs are irrelevant
A. Avoidable costs are decision relevant but unavoidable costs are irrelevant
B. Avoidable costs remain unchanged but unavoidable costs represent the benefit given up
in choosing one alternative over another.
C. Avoidable costs are past costs but unavoidable costs are yet to be incurred.
D. Avoidable costs are fixed costs but unavoidable costs can increase or decrease profit
levels.
Explanation:
Avoidable cost is relevant and unavoidable cost is irrelevant. By definition, a relevant cost is
a cost yet to be incurred; it is a future cost Relevant revenues are expected future revenues.
Costs and revenues that have already been incurred or committed are irrelevant.

Question #72: A company is considering four independent investment proposals. The


company has $3 million available for investment during the present period. The investment
outlay for each project and its projected net present value (NPV) is presented below.
Investme
Project nt cost NPV
$40,00
I $500,000 0
120,00
II 900,000 0
180,00
III 1,200,000 0
150,00
IV 1,600,000 0

Which of the following project options should be recommended to the company'


management?
Correct Answer: A. Projects I, II and III only.
A. Projects I, II and III only.
B. Projects I, II and IV only.
C. Projects II, III and IV only.
D. Projects III and IV only.
Explanation:
Answer (A) is correct.
Capital rationing exists when a firm sets a limit on the amount of funds to be invested during a
given period. In such sanctions, a firm carrot afford to undertake all profitable projects. The
profitability index (or excess present value index) is a method for rating projects to ensure that
limited resources are placed with the investment that will return the highest net present value
(NPV).
NPV of future cash flows
Profitability index =
Net investment
The indexes for Woods' potential projects can thus be calculated as follows
Investme Profitabilit
Project nt cost NPV y index
$40,00
I $500,000 0 0.080
120,00
II 900,000 0 0.133
1,200,00 180,00
III 0 0 0.150
1,600,00 150,00
IV 0 0 0.094

Raised in order of desirability, they are III, II, IV, and I. Since only $3 million is available for
funding only III, II and will be selected.
Answer (B) is incorrect.
Project II I is more desirable than Project IV.
Answer (C) is incorrect
While Project IV is more desirable than Project I. insufficient funding is available to engage
Project IV.
Answer (D) is incorrect.
Projects I and II are also desirable and sufficient funding is available.
Question #73: The amount remaining from sales revenues after variable expenses are deducted is
the:
Correct Answer: C. Contribution margin.
A. Net income.
B. Breakeven point.
C. Contribution margin.
D. Operating income.
Explanation:
By definition, contribution margin represents the amount remaining from sales revenue after
variable expenses are deducted. It is found by taking revenues and subtracting all costs of the
output that vary with respect to the number of output units.

Question #74: A company is considering an investment to open a new banana processing


division. The project in question would entail an initial investment of $45,000, and as a result of
the project cash inflows of $20,000 can be expected in each of the next 3 years. The hurdle rate
is 10%. What is the profitability index for the project?
Correct Answer: B. 1.1053
A. 1.0784
B. 1.1053
C. 1.1379
D. 1.1771
Explanation:
Answer (A) is incorrect.
It uses the wrong present value factors.
Answer (B) is correct.
At a 10% hurdle rate, the present value of the future inflows is $49,740 (20,000 x 2.487).
Thus, the net present value is $4,740 (49,740 – 45,000).
The profitability index is calculated as follows:
49,740
45,000 = 1.1053
Answer (C) is incorrect.
It uses the wrong present value factors.
Answer (D) is incorrect.
It uses the wrong present value factors.

Question #75: Question: Broomall's working capital at year end is


[Fact Pattern #16] Broomall Corporation has decided to include certain financial ratios in its
year-end annual report to shareholders. Selected information relating to its most recent fiscal
year is provided below.

Cash $10,000
Accounts receivable:
Beginning of year 24,000
End of year 20,000
Prepaid expenses 8,000
Inventory:
Beginning of year 26,000
End of year 30,000
Available-for-sale securities:
Historical cost 9,000
Fair value at year end 12,000
Accounts payable 15,000
Notes payable (due in 90 days) 25,000
Bonds payable (due in 10 years) 35,000
Net credit sales for year 220,000
Cost of goods sold 140,000

Correct Answer: A. $40,000


A. $40,000
B. $37,000
C. $28,000
D. $10,000
Explanation:
Answer (A) is correct
Working capital consists of current assets net of current liabilities. Broomall's working capital at
year end can thus be calculated as follows:
Cash $10,000
Accounts receivable: 20,000
Prepaid expenses 8,000
Inventory: 30,000
Available-for-sale
securities: 12,000
(15,000
Accounts payable )
(25,000
Notes payable )
Working capital $40,000

Answer (B) is incorrect.


The amount of $37,000 results from improperly including available-for-sale securities at cost
rather than at fair value.
Answer (C) is incorrect.
The amount of $28,000 results from improperly excluding available-for-sale securities from
current assets.
Answer (D) is incorrect.
The amount of $10,000 improperly treats only cash as working capital.

Question #76: Bargain Press is considering publishing a new textbook. The publisher has
developed the following cost data related to a production run of 6,000, the minimum possible
production run. Bargain Press will sell the textbook for $45 per copy. How many textbooks must
Bargain Press sell in order to generate operating earnings (earnings before interest and taxes) of
20% on sales? (Round your answer up to the nearest whole textbook)
Estimated
cost
Development (reviews, class testing, editing) $35,000
Typesetting 18,500
Depreciation on equipment 9,320
General and administrative 7,500
Miscellaneous fixed costs 4,400
Printing and binding 30,000
Sales staff commissions (2% of selling price) 5,400
Bookstore commission (25% of selling price) 67,500
Author's Royalties (10% of selling price) 27,000
Total costs at production of 6,000 copies $204,620
Correct Answer: A. 5,412 copies.
A. 5,412 copies.
B. 6,199 copies.
C. 2,076 copies.
D. 5,207 copies.
Explanation:
The number of copies required can be found by solving the following equation, where x equals
the number of copies:
Total sales - total variable costs - fixed costs = (return %) (total sales)
Total sales = $45x
Fixed costs = development + typesetting + depreciation + general and administrative +
miscellaneous fixed costs + printing and binding
Fixed costs = $35,000 + $18,500+ $9,320 + $7,500+ $4,400 + $30,000 = $104,720
Total variable cost = (variable cost per unit, or VCU) (x) = VCU (x)
VCU = (sales staff commissions + bookstore commissions + royalties)/ (number of copies)
VCU = ($5,400 + $67,500 + $27,000)/ (6,000 copies) = $99,900 / 6,000
VCU = $16.65 per copy
$45x - $16.65(x) - $104,720 = (0.2) ($45x)
$45x - $16.65(x) - $104,720 = $9x
$28.35(x) - $104,720 = $9x
$19.35x = $104,720
x = 5,411.89, which rounds to 5,411 copies.

Question #77: Assume that the probability distribution of NPVs is normal. The firm considers
true risk occurring if the project results in a NPV that is zero or less. If the expected NPV is
$1,000 and the standard deviation of NPV is $500, what is the probability that the project has an
NPV of 0 or less?
Correct Answer: A. Less than 3%.
A. Less than 3%.
B. Greater than 3%, but less than 9%.
C. Greater than 9%, but less than 16%.
D. Greater than 16%.
Explanation:
Answer (A) is correct
Since three standard deviations incorporate over 99% of all observations, and two standard
deviations incorporate over 95% of observations, less than 5% will not be included within two
standard deviations, and this is divided between both ends of the normal curve. Therefore, less
than 2.5% of the observations will be in the negative portion of the curve.
Answer (B) is incorrect.
The 5% of observations not covered within two standard deviations are divided between the two
tails of the curve.
Answer (C) is incorrect.
The 5% of observations not covered within two standard deviations are divided between the two
tails of the curve.
Answer (D) is incorrect.
The 5% of observations not covered within two standard deviations are divided between the two
tails of the curve.

Question #78: If a company determines that a particular risk could result in a loss of up to
$50,000 and the company is willing to spend up to $50,000 to mitigate the risk, this loss is
commonly called all of the following except:
Correct Answer: C. The strategic loss.
A. The maximum possible loss.
B. The value at risk.
C. The strategic loss.
D. The catastrophic loss.
Explanation:
In a business context, risk is defined as the level of exposure to a chance of loss. A strategic loss
occurs due to taking particular planned actions.
Question #79: The COSO ERM framework includes eight interrelated components. Which
component in the framework implements policies and procedures to help ensure that risk
responses are effectively carried out?
Correct Answer: B. Control activities.
A. Risk assessment
B. Control activities.
C. Objective setting.
D. Event identification.
Explanation:
Control activities include policies and procedures that are established and implemented to help
ensure that the risk responses are effectively carried out. The COSO model lists six control
activities: the assignment of authority and responsibility, a system of transaction authorizations,
adequate documentation and records, security of assets, independent verifications, and adequate
separation of duties.

Question #80: The graph below shows:

Correct Answer: A. A relatively inelastic demand.


A. A relatively inelastic demand.
B. A perfectly inelastic demand.
C. A relatively elastic demand.
D. A perfectly elastic demand.
Explanation:
A relatively inelastic demand means that a percentage change in price will result in a smaller
percentage change in quantity demanded. It is shown by a steep demand curve, which is not quite
perfectly vertical.

Question #81: If a product has a price elasticity of demand of 2.0, the demand is said to be:
Correct Answer: A. Elastic
A. Elastic
B. Perfectly inelastic
C. Inelastic.
D. Perfectly elastic
Explanation:
Price elasticity (E) is the percent change in a product's quantity demanded given a percent
change in the product's price. Elastic demand occurs when the percent change in demand is
greater than the percent change in price (E>1).

Question #82: According to the DuPont formula, which one of the following will not increase a
profitable firm's return on equity?
Correct Answer: D. Lowering equity multiplier.
A. Increasing total asset turnover.
B. Increasing net profit margin.
C. Lowering corporate income taxes.
D. Lowering equity multiplier.
Explanation:
Answer (A) is incorrect.
Increasing total asset turnover would increase a profitable firm's return on equity.
Answer (B) is incorrect.
Increasing net profit margin would increase a profitable firm's return on equity.
Answer (C) is incorrect.
Lowering corporate income taxes would increase a profitable firm's return on equity.
Answer (D) is correct.
Lowering the equity multiplier would not increase a profitable firm's return on equity. The
DuPont model depicts return on assets as total asset turnover (sales divided by average total
assets) times the profit margin (net income divided by sales).

Question #83: Thomas Company's capital structure consists of 30% long-term debt. 25%
preferred stock, and 45% common equity. The cost of capital for each component is shown
below.
8% before
Long term debt tax
Preferred stock 11%
Common equity 15%

If Thomas pays taxes at the rate of 40%, what is the company's after-tax weighted average cost
of capital (WACC)?
Correct Answer: C. 10.94%.
A. 11.9%.
B. 9.84%.
C. 10.94%.
D. 7.14%.
Explanation:
The WACC is calculated as:
WACC = (weight of long-term debt) (after-tax cost of long-term debt) + (weight of preferred
stock) (cost of preferred stock) + (weight of common equity) (cost of common equity)
The after-tax cost of debt is calculated as:
After-tax cost of debt = (1- tax rate) (before-tax cost of debt) After-tax cost of debt = (1- 0.4)
(0.08) = (0.6) (0.08) = 0.048, or 4.8%
Therefore. WACC = (0.3) (0.048) + (0.25) (0.11) + (0.45) (0.15) = 0.0144 + 0.0275 + 0.0675 =
0.1094, or 10.94%.
Question #84: Question: The acquisition of the new production machine by Gunning Industries
will contribute a discounted net-of-tax contribution margin of (please refer the fact pattern
below)
[Fact Pattern #155] In order to increase production capacity, Gunning Industries is considering
replacing an existing production ma chine with a new technologically improved machine
effective January 1. The following information is being considered by Gunning Industries:
The new machine would be purchased for $160,000 in cash. Shipping, installation, and testing
would cost an additional $30,000.
The new machine is expected to increase annual sales by 20,000 units at a sales price of $40 per
unit. Incremental operating costs include $30 per unit in variable costs and total fixed costs of
$40,000 per year.
The investment in the new machine will require an immediate increase in working capital of
$35,000. This cash outflow will be recovered after 5 years.
Gunning uses straight-line depreciation for financial reporting and tax reporting purposes. The
new machine has an estimated useful life of 5 years and zero salvage value.
Gunning is subject to a 40% corporate income tax r ate.
Gunning uses the net present value method to analyze investments and win employ the following
factors and rates:
Present value
of an ordinary
Present value annuity of $1
Period of $1 at 10% at 10%

1 .909 .909

2 .826 1.736

3 .751 2.487

4 .683 3.170

5 .621 3.791

Correct Answer: • $454,920


A. $242,624
B. $303,280
C. $363,936
D. $454,920
Explanation:
Answer (A) is incorrect.
The amount of $242,624 deducts fixed costs from the pretax contribution margin and applies a
60% tax rate.
Answer (B) is incorrect.
The amount of $303,280 deducts fixed costs from the after-tax contribution margin before
discounting.
Answer (C) is incorrect.
The amount of $363,936 deducts fixed costs from the contribution margin before calculating
taxes and the present value.
Answer (D) is correct.
The new machine will increase sales by 20,000 units a year. The increase in the pretax total
contribution margin will be $200,000 per year [20,000 units x ($40 SP - $30 VC)], and the
annual increase in the after-tax contribution margin will be $120,000 [$200,000 x (1.0 -.4)]. The
present value of the after-tax increase in the contribution margin over the 5-year useful life of the
machine is $454,920 ($120,000 x 3.791 PV of an ordinary annuity for 5 years at 10%).

Question #85: An entity has total assets of $7,500,000 and a current ratio of 2.3 times before
purchasing $750,000 of merchandise on credit for resale. After this purchase, the current ratio
will
Correct Answer: C. Be lower than 2.3 times.
A. Remain at 2.3 times.
B. Be higher than 2.3 times.
C. Be lower than 2.3 times.
D. Be exactly 2.53 times.
Explanation:
Answer (A) is incorrect.
The current ratio is the ratio of current assets to current liabilities. When the ratio is greater than
one, any change of equal dollar amount on both the numerator and denominator will result in a
lowering of the overall ratio (since the denominator will increase by a proportionally greater
amount). The purchase of merchandise on credit is an example of such a change: inventory
increases in the numerator and accounts payable increase in the denominator by an equal dollar
amount
Answer (B) is incorrect.
The current ratio is the ratio of current assets to current liabilities. When the ratio is greater than
one, any change of equal dollar amount on both the numerator and denominator will result in a
lowering of the overall ratio (since the denominator will increase by a proportionally greater
amount). The purchase of merchandise on credit is an example of such a change: inventory
increases in the numerator and accounts payable increase in the denominator by an equal dollar
amount
Answer (C) is correct.
The current ratio is the ratio of current assets to current liabilities. When the ratio is greater than
one, any change of equal dollar amount on both the numerator and denominator will result in a
lowering of the overall ratio (since the denominator will increase by a proportionally greater
amount). The purchase of merchandise on credit is an example of such a change: Inventory
increases in the numerator and accounts payable increases in the denominator by an equal dollar
amount.
Answer (D) is incorrect.
The current ratio is the ratio of current assets to current liabilities. When the ratio is greater than
one, any change of equal dollar amount on both the numerator and denominator will result in a
lowering of the overall ratio (since the denominator will increase by a proportionally greater
amount). The purchase of merchandise on credit is an example of such a change: inventory
increases in the numerator and accounts payable increase in the denominator by an equal dollar
amount.
Question #86: The following information pertains to the year ended December 31:
$720,00
Sales 0
Net income 120,000
Average total
assets 480,000

Which one of the following formulas depicts the use of the DuPont model to calculate Andrew's
return on assets?
Correct Answer: C. (720,000 ÷ 480,000) x (120,000 ÷ 720,000)
A. (720,000 ÷ 480,000) x (720,000 ÷ 120,000)
B. (480,000 ÷ 720,000) x (720,000 ÷ 120,000)
C. (720,000 ÷ 480,000) x (120,000 ÷ 720,000)
D. (480,000 ÷ 720,000) x (120,000 ÷ 720,000)
Explanation:
Answer (A) is incorrect.
The DuPont model depicts return on assets as total asset turnover (sales divided by average total
assets) times the profit margin (net income divided by sales).
Answer (B) is incorrect.
The DuPont model depicts return on assets as total asset turnover (sales divided by average total
assets) times the profit margin (net income divided by sales).
Answer (C) is correct.
The DuPont model depicts return on assets as total asset turnover (sales divided by average total
assets) times the profit margin (net income divided by sales). Therefore, the ROA calculation
uses the formula (($720,000 ÷ $480,000) x ($120,000 ÷ $720,000)].
Answer (D) is incorrect.
The DuPont model depicts return on assets as total asset turnover (sales divided by average total
assets) times the profit margin (net income divided by sales).

Question #87: Country R's currency would tend to depreciate relative to Country Is currency
when:
Correct Answer: D. Country R has real interest rates that are lower than real interest rates in
Country T.
A. Country T has a rapid rate of growth in income that causes imports to lag behind exports.
B. Country R switches to a more restrictive monetary policy.
C. Country R has a rate of inflation that is lower than the rate of inflation in Country T
D. Country R has real interest rates that are lower than real interest rates in Country T.
Explanation:
The lower interest rates will cause money to flow from Country R to Country T causing Country
T’s currency to appreciate relative to Country R's. R's currency would depreciate.

Question #88: Which type of short-term financing allows a business to borrow up to a specified
limit during a particular time period?
Correct Answer: B. Line of credit.
A. Promissory note.
B. Line of credit.
C. Accrued expenses.
D. Bankers' acceptance.
Explanation:
By definition, a line of credit (or credit line) is an agreement allowing a firm to borrow up to a
specified limit during a particular time period. The borrower has access to the credit amount
(which is generally substantial) but pays interest only on actual borrowing.

Question #89: Question: Foster's internal rate of return (rounded to the nearest whole percentage)
is (please refer the fact pattern below)
(Fact Pattern n159] Foster Manufacturing is analyzing a capital investment project that is
forecasted to produce the following cash flows and net income.
After-tax
cash Net
Year flows income
$(20,000
0 ) $0
1 6,000 2,000
2 6,000 2,000
3 8,000 2,000
4 8,000 2,000

Correct Answer: C. 14%


A. 5.
B. 12%
C. 14%
D. 40%
Explanation:
Answer (A) is incorrect.
The internal rate of return for this project is closest to 14%.
Answer (B) is incorrect.
The internal rate of return for this project is closer to 14%.
Answer (C) is incorrect.
A capital project’s internal rate of return is the discount rate at which the net present value of the
project’s cash flows equals zero, i.e., the rate at which discounted cash inflows equal discounted
cash outflows. The net present value of this project at 12% can be calculated as follows:
After-
tax cash PV Present
Year flows Factor value
$(20,00 =$(20,00
0 0) ×1.000 0)
1 6,000 ×0.893 =5,358
2 6,000 ×0.797 =4,782
3 8,000 ×0.712 =5,696
4 8,000 ×0.636 =5,088
$924

Thus, the answer must be greater than 12%. This problem is solved on basically a trial-and-error
basis. At 14%, the net present value can be calculated as follows:
After-
tax cash PV Present
Year flows Factor value
$(20,00 =$(20,00
0 0) ×1.000 0)
1 6,000 ×0.877 =5,262
2 6,000 ×0.769 =4,614
3 8,000 ×0.675 =5,400
4 8,000 ×0.592 =4,736
$12
The internal rate of return is thus slightly over 14%.
Answer (D) is incorrect.
The internal rate of return for this project is closest to 14%.
Question #90: A primary benefit of portfolio diversification is to:
Correct Answer: C. Reduce unsystematic risk.
A. Eliminate systematic risk.
B. Provide a more favorable borrowing position.
C. Reduce unsystematic risk.
D. Reduce exposure to foreign exchange rates.
Explanation:
Most variability resulting from unsystematic risk is avoidable through diversification.
Holding a diversified portfolio reduces unsystematic risk (sometimes called diversifiable
risk) because different portions of the market tend to perform differently at various times.
Question #91: Please see the question below:
Richardson Motors uses ten units of Part Number T305 each month in the production of large
diesel engines. The cost to manufacture one unit of T305 is presented below.

Direct materials $2,000


Materials handling (20% of direct material
cost) 400
Direct labor 16,000
Manufacturing overhead (150% of direct
labor) 24,000
Total manufacturing cost $42,400

Material handling, which is not included in manufacturing overhead, represents the direct
variable costs of the Receiving Department that are applied to direct materials and purchased
components on the basis of their cost. Richardson's annual manufacturing overhead budgets
is one-third variable and two-thirds fixed. Simpson Castings, one of Richardson's reliable
vendors, has offered to supply T305 at a unit price of $30,000.
If Richardson Motors purchases the ten T305 units from Simpson Castings, the capacity
Richardson used to manufacture these parts would be idle. Should Richardson decide to
purchase the parts from Simpson, the out-of-pocket cost per unit of T305 would:
Correct Answer: B. increase $9,600.
A. Decrease $6,400.
B. Increase $9,600.
C. Increase $3,600.
D. Decrease $4,400.
Explanation:
The cost of purchasing the T305 is $36,000 per unit = $30,000 purchase price + $6,000 (0.2
x $30,000 for material handling)).
The cost to make the T305 per unit consists of avoidable variable costs per unit, avoidable
fixed costs per unit and the unit cost of any lost opportunities, which equals $26,400 ($2.000
direct materials + $400 material handling + $16,000 direct labor + $8,000(1/3 of $24,000) in
variable overhead costs).
The difference between the cost to buy one unit ($36,000) and the cost to make one unit
($26,400) is $9,600.

Question #92: In financial statement analysis, expressing all financial statement items as a
percentage of base-year amounts is called
Correct Answer: A. Horizontal common-size analysis.
A. Horizontal common-size analysis.
B. Vertical common-size analysis.
C. Trend analysis.
D. Ratio analysis.
Explanation:
Answer (A) is correct
Expressing financial statement items as percentages of corresponding base-year figures is a
horizontal form of common-size (percentage) analysis that is useful for evaluating trends.
The base amount is assigned the value of 100%, and the amounts for other years are
denominated in percentages compared to the base year.
Answer (B) is incorrect.
Vertical common-size (percentage) analysis presents figures for a single year expressed as
percentages of a base amount on the balance sheet (e.g.. total assets) and on the income
statement (e.g.. sales).
Answer (C) is incorrect.
The term "trend analysis' is most often applied to the quantitative techniques used in
forecasting to fit a curve to given data.
Answer (D) is incorrect.
It is a general term.

Question #93: A company uses the direct write-off method to account for uncollectible
accounts receivable. If the company subsequently collects an account receivable that was
written off in a prior accounting period, the effect of the collection of the account receivable
on the current ratio and total working capital would be
Correct Answer: B. Increase (Current Ratio). (Working Capital) Increase
A. None (Current Ratio). (Working Capital) None
B. Increase (Current Ratio). (Working Capital) Increase
C. Increase (Current Ratio). (Working Capital) None
D. None (Current Ratio). (Working Capital) Decrease
Explanation:
Answer (A) is incorrect.
The current ratio and working capital increase.
Answer (B) is correct.
Because the company uses the direct write-off method, the original entry involved a debit to
a bad debt expense account (closed to retained earnings). The subsequent collection required
a debit to cash and a credit to bad debt expense or retained earnings. Thus, only one current
asset account was involved in the collection entry, and current assets (cash) increased as a
result. If current assets increase and no change occurs in current liabilities, the current ratio
and working capital both increase.
Answer (C) is incorrect.
The current ratio and working capital increase.
Answer (D) is incorrect.
The current ratio and working capital increase.

Question #94: Fact Pattern: Kell, Inc., is analyzing an investment for a new product expected
to have annual sales of 100,000 units for the next 5 years and then be discontinued. New
equipment will be purchased for $1,200,000 and cost $300,000 to install. The equipment will
be depreciated on a straight-line basis over 5 years for financial reporting purposes and 3
years for tax purposes. At the end of the fifth year, it will cost $100,000 to remove the
equipment, which can be sold for $300,000. Additional working capital of $400,000 will be
required immediately and needed for the life of the product. The product will sell for $80,
with direct labor and material costs of $65 per unit. Annual indirect costs will increase by
$500,000. Kell's effective tax rate is 40%.
Question: In a capital budgeting analysis, what is the expected cash flow at time = 5 (fifth
year of operations) that Kell should use to compute the net present value?
Correct Answer: C. $1,120,000
A. $720,000
B. $800,000
C. $1,120,000
D. $1,240,000
Explanation:
Answer (A) is incorrect.
The amount of $720,000 results from failing to include the recovery of working capital.
Answer (B) is incorrect.
The amount of $800,000 results from failing to account for the effects of income taxes on the
disposal proceeds of the equipment and from failing to include the recovery of working
capital.
Answer (C) is correct.
The estimated incremental after-tax operating flows in the final year of a capital project
consist of three components the after-tax cash inflows from operation, the depreciation tax
shield and the net termination cash flows of the project. The first of these for Kell can be
calculated as follows
Projected annual contribution on
margin (100,000 units x $15) $1,500,000
Less: annual increase in indirect costs (500,000)
Net annual cash inflow $1,000,000
Less: income tax expense ($1,000,000 ×
40%) (400,000)
After-tax cash inflow from operations $600,000

Since the equipment was depreciated over 3 years for tax purposes, no depreciation tax shield
remains in the fifth year. The termination cash flows can be calculated as follows
Proceeds from disposal of new
equipment ($300,000 - $100,000) $200,000
Less: income tax expense
($200,000 × 40%) (80,000)
After-tax cash inflow from disposal
of equipment $120,000
Add: recovery of working capital 400,000
Total termination on cash inflows $520,000

Total expected cash flows in the fifth year of the project are thus $1,120,000 ($600,000 + $0
+ $520,000).
Answer (D) is incorrect.
The amount of $1,240,000 results from improperly depreciating the equipment over the book
life rather than the tax life.

Question #95: An organization can mitigate the risk of financial loss from natural disasters
by purchasing:
Correct Answer: A. Hazard insurance.
A. Hazard insurance.
B. Health insurance.
C. Liability insurance.
D. Workers compensation insurance.
Explanation:
Hazard risks relate to natural disasters such as storms, floods, hurricanes, blizzards,
earthquakes, and volcanoes. Hazard insurance can be purchased in various forms to mitigate
(transfer) the risk of loss should a covered hazard occur.

Question #96: Last year, import of a European product costing €255 cost $242.25. This year,
the same import, which has not changed price in euros, costs $300.00. Which of the
following is true?
Correct Answer: D. The cost of the euro in U.S. dollars has increased from $0.95 to $1.176.
A. The cost of the euro in U.S. dollars has increased from $1.05 to $0.85.
B. The cost of the euro in U.S. dollars has decreased from $1.05 to $0.85.
C. The cost of the euro in U.S. dollars has decreased from $0.95 to $1.176.
D. The cost of the euro in U.S. dollars has increased from $0.95 to $1.176.
Explanation:
Notice that last year it cost fewer U.S. dollars than euros to buy the product. Therefore, the
exchange rate must be less than $1 U.S/euro. Calculating the exchange rate by dividing the
U.S. dollar amount by the euro amount, the exchange rate in the first year is $0.95
($242.25/€255). In the second year, it takes more U.S. dollars to purchase the goods, and the
exchange rate is $1.176 ($300 U.SJ €255).
Question #97: Please see the question below
The following data pertain to a 4-years project being considered by Metro Industries
• A depreciable asset that costs $1,200,000 will be acquired on January 1. The asset, which is
expected to have a $200,000 salvage value at the end of 4 years, qualifies as 3-year property
wider the Modified Accelerated Cost Recovery System (MACRS).
• The new asset will replace at existing asset that has a tax basis of $150,000 and can be sold
on the same January 1 for $180,000.
• The project is expected to provide added annual sales 30,000 units at $20. Additional cash
operating costs are: variable, $12 per unit; fixed, $90,000 per year.
• A $50,000 working capital investment that is fully recoverable at the end of the fourth year
is required.
Metro is subject to a 40% income tax rate and rounds all computations to the nearest dollar.
Assume that any gain a loss affects the taxes paid at the end of the year in which it occurred.
The company uses the net present value method to analyze investments and will employ the
following factors and rate.
Present value of
Present value of $1 annuity at
Period $1 at 12% 12% MARCS
1 0.89 0.89 33%
2 0.80 1.69 45
3 0.71 2.40 15
4 0.64 3.04 7

Correct Answer: C. $(18.000)


A. $(2,800)
B. $(18,000)
C. $(50,000)
D. $(59,200)
Explanation:
Answer (A) is incorrect.
The firm will have its working capital tied up for 4 years, which results in a cost of $18,000
at 12% interest.
Answer (B) is correct.
The working capital investment is treated as a $50,000 outflow at the beginning of the project
and a $50,000 inflow at the end of 4 years. Accordingly, the present value of the inflow after
4 years should be subtracted from the initial $50,000 outlay. The overall discounted-cash-
flow impact of the working capital investment is $18,000 [$50,000 - ($50,000 x .64 PV of $1
at 12% for four periods)].
Answer (C) is incorrect
The working capital investment is recovered at the end of the fourth year. Hence, the
working capital cost of the project is the difference between $50,000 and the present value of
$50,000 in 4 years.
Answer (D) is incorrect.
The answer cannot exceed $50,000, which is the amount of the cash outflow.
Question #98: In the current year, a firm had $15 million in sales, while total fixed costs were
held to $6 million. The firm's total assets averaged $20 million and the debt-to-equity ratio
was calculated at 0.60. If the firm's EBIT is $3 million, the interest on all debt is 9%, and the
tax rate is 40%, what is the firm's return on equity?
Correct Answer: A. 11.16%
A. 11.16%
B. 14.4%
C. 18.6%
D. 24.0%
Explanation:
Answer (A) is correct
The first step is to determine the amount of equity. Since the debt-to-equity ratio is .6, 60D =
100E, and D = .6E. Additionally, since A = L + E. we can substitute D for L. plug in the
value of assets and solve for E as follows:
$20 million = D + E
$20 million = .6E E
$20 million = 1.6E
Thus, E (equity) equals $12.5 million. Debt is therefore $7.5 million (.6 x $12.5 million).
At 9%, interest on $7.5 million of debt is $675,000. Earnings before taxes are $2,325,000
($3,000,000 EBIT - $675,000 interest). At a 40% tax rate, taxes are $930,000, which leaves a
net income of $1,395,000. Return on equity is calculated by dividing the $1,395,000 net
income by the $12,500,000 of equity capital, giving an ROE of 11.16%.
Answer (B) is incorrect.
A failure to deduct interest expense results in 14.4%.
Answer (C) is incorrect
A failure to deduct income taxes results in 18.6%.
Answer (D) is incorrect.
Using the wrong amount of equity results in 24.0%.
Question #99: IMA's Statement on Management Accounting. "Values and Ethics: From
Inception to Practice:' recommends a defined code of conduct and ethical behavior for all
organizations. One advantage of having such a code is that it
Correct Answer: A. Provides employees with guidance for handling unfamiliar situations.
A. Provides employees with guidance for handling unfamiliar situations.
B. Ensures ethical behavior by all employees
C. Shields the organization from liability in cases of loss of stockholder value due to fraud.
D. Eases the investigative process performed by police and prosecutors in cases of suspected
fraud.
Explanation:
Answer (A) is correct
"Values and Ethics: From Inception to Practice states, in part."... what does an employee do
when unplanned events occur? What reference does an individual look to for help in making
decisions?... This is why it is important to have a defined set of organizational values and
code of ethics - they create the "touchstone against which every unanticipated decision must
be judged. Failure to have every individual in the organization know and understand these
values and ethical code leads to inconsistency and. in the worst cases, unethical or fraudulent
behavior!' (IV. Values. Ethics, and Accounting.)
Answer (B) is incorrect.
A code of conduct cannot guarantee ethical behavior by employees.
Answer (C) is incorrect.
A code of conduct cannot guarantee that an organization will be shielded from liability in
cases of fraud.
Answer (D) is incorrect.
A code of conduct does not ease law enforcement's investigative process.

Question #100: An analyst is reviewing the financial statements of a company whose


operating income has declined from the prior year. The following ratios have been
calculated.
Prior year Current year
Gross profit
margin 15% 20%
Operating profit
margin 12% 10%
Inventory turnover 10.4 9.8
Based on the above, the analyst could infer that the decrease in operating income may be due
to
Correct Answer: C. Higher interest expense.
A. Lower revenue per unit sold.
B. Accumulation of unused inventory.
C. Higher interest expense.
D. An increase in advertising expense.
Explanation:
Answer (A) is incorrect.
The decrease in operating income may be due to an increase in advertising expense. Lower
revenue per unit sold would decrease both gross profit margin and operating margin.
Answer (B) is incorrect.
The decrease in operating income may be due to an increase in advertising expense.
Inventory turnover has decreased. However, this question asks for the reason for the decrease
in operating income. There is nothing to indicate that the increase in inventory has caused the
reduction in profits.
Answer (C) is incorrect.
The decrease in operating income may be due to an increase in advertising expense.
Operating profit includes selling and administrative expenses while gross profit margin does
not. However, it does not include interest expense.
Answer (D) is correct.
Operating profit margin decreased, but gross profit margin increased. This could be because
advertising expenses increased. Operating profit includes selling and administrative expenses
while gross profit margin does not.

You might also like